GMAT® OFFICIAL GUIDE 2018 GMAT

User Manual:

Open the PDF directly: View PDF PDF.
Page Count: 855

DownloadGMAT® OFFICIAL GUIDE 2018 GMAT
Open PDF In BrowserView PDF
Online Question Bank Information
Your purchase of GMAT® Official Guide 2018 offers the original purchaser access to the online question
bank and 58 Integrated Reasoning practice questions, and the diagnostic test for a period of 12 months
from the date of activation.
To obtain an access code, go to www.wiley.com/go/officialgmatbook to verify your purchase. Once you
verify your purchase, you will be emailed an access code and instructions for setting up a personal login.
Note: GMAC and Wiley are not responsible for providing access to the online companion for customers
who purchase or borrow used copies of this book. This code will only provide you with access to the
question bank. It will not work for other Wiley or GMAC password-protected websites.
For customer support, please visit http://wileyactual.com/gmat/help

The definitive guide from the makers of the GMAT® exam

Official Guide
2018
Book +

Online
The ONLY
source of real
GMAT® questions
from past
exams

This edition includes
130 never-before-seen questions

Graduate Management Admission Council®

mba.com

GMAT® OFFICIAL GUIDE 2018
Copyright © 2017 by the Graduate Management Admission Council®. All rights reserved.
Published by John Wiley & Sons, Inc., Hoboken, New Jersey.
No part of this publication may be reproduced, stored in a retrieval system or transmitted in any form
or by any means, electronic, mechanical, photocopying, recording, scanning or otherwise, except as
permitted under Sections 107 or 108 of the 1976 United States Copyright Act, without either the prior
written permission of the Publisher, or authorization through payment of the appropriate per-copy fee
to the Copyright Clearance Center, 222 Rosewood Drive, Danvers, MA 01923, (978) 750-8400,
fax (978) 646-8600, or on the web at www.copyright.com. Requests to the Publisher for permission
should be addressed to the Permissions Department, John Wiley & Sons, Inc., 111 River Street, Hoboken,
NJ 07030, (201) 748-6011, fax (201) 748-6008, or online at http://www.wiley.com/go/permissions.
The publisher and the author make no representations or warranties with respect to the accuracy or
completeness of the contents of this work and specifically disclaim all warranties, including without
limitation warranties of fitness for a particular purpose. No warranty may be created or extended by sales or
promotional materials. The advice and strategies contained herein may not be suitable for every situation.
This work is sold with the understanding that the publisher is not engaged in rendering legal, accounting,
or other professional services. If professional assistance is required, the services of a competent professional
person should be sought. Neither the publisher nor the author shall be liable for damages arising here
from. The fact that an organization or Website is referred to in this work as a citation and/or a potential
source of further information does not mean that the author or the publisher endorses the information the
organization or Website may provide or recommendations it may make. Further, readers should be aware
that Internet Websites listed in this work may have changed or disappeared between when this work was
written and when it is read.
Trademarks: Wiley, the Wiley logo, and related trademarks are trademarks or registered trademarks of John
Wiley & Sons, Inc. and/or its affiliates. The GMAC and GMAT logos, GMAC®, GMASS®, GMAT®,
GMAT CAT®, Graduate Management Admission Council®, and Graduate Management Admission Test®
are registered trademarks of the Graduate Management Admission Council® (GMAC) in the United
States and other countries. All other trademarks are the property of their respective owners. John Wiley &
Sons, Inc., is not associated with any product or vendor mentioned in this book.
For general information on our other products and services or to obtain technical support please contact our
Customer Care Department within the U.S. at (877) 762-2974, outside the U.S. at (317) 572-3993 or
fax (317) 572-4002.
John Wiley & Sons, Inc., also publishes its books in a variety of electronic formats and by print-on-demand.
Not all content that is available in standard print versions of this book may appear or be packaged in all
book formats. If you have purchased a version of this book that did not include media that is referenced by
or accompanies a standard print version, you may request this media by visiting http://booksupport.wiley.com.
For more information about Wiley products, visit us at www.wiley.com.
ISBN 978-1-119-38747-3 (pbk); ISBN 978-1-119-40232-9 (ePub); ISBN 978-1-119-40233-6 (ePDF)
Printed in the United States of America
10

9

8

7

6

5

4

3

2

1

Letter from the President and CEO, GMAC®

v

1.0
1.0
1.1
1.2
1.3
1.4
1.5
1.6
1.7
1.8
1.9
1.10
1.11

What Is the GMAT® Exam?
What Is the GMAT® Exam?
Why Take the GMAT® Exam?
GMAT® Exam Format
What Is the Content of the Test Like?
Integrated Reasoning Section
Quantitative Section
Verbal Section
Analytical Writing Assessment
What Computer Skills Will I Need?
What Are the Test Centers Like?
How Are Scores Calculated?
Test Development Process

2
3
3
4
5
6
6
6
7
7
7
7
8

2.0
2.0
2.1
2.2
2.3
2.4
2.5

How to Prepare
How to Prepare
How Should I Prepare to Take the Test?
What About Practice Tests?
How Should I Use the Diagnostic Test?
Where Can I Get Additional Practice?
General Test-Taking Suggestions

10
11
11
11
12
12
12

3.0
3.0
3.1
3.2
3.3
3.4
3.5
3.6

Diagnostic Test
Diagnostic Test
Quantitative Questions
Verbal Questions
Quantitative and Verbal Answer Keys
Interpretive Guide
Quantitative Answer Explanations
Verbal Answer Explanations

14
15
16
23
40
40
41
60

4.0
4.0
4.1
4.2
4.3
4.4

Math Review
Math Review
Arithmetic
Algebra
Geometry
Word Problems

100
101
102
114
122
134

5.0
5.0
5.1
5.2
5.3
5.4
5.5

Problem Solving
Problem Solving
Test-Taking Strategies
The Directions
Practice Questions
Answer Key
Answer Explanations

142
143
144
144
146
181
183

6.0
6.0
6.1
6.2
6.3
6.4
6.5

Data Sufficiency
Data Sufficiency
Test-Taking Strategies
The Directions
Practice Questions
Answer Key
Answer Explanations

264
265
266
268
270
288
289

Table of Contents

7.0
7.0
7.1
7.2
7.3
7.4
7.5
7.6

Reading Comprehension
Reading Comprehension
What Is Measured
Test-Taking Strategies
The Directions
Practice Questions
Answer Key
Answer Explanations

356
357
358
360
361
362
423
424

8.0
8.0
8.1
8.2
8.3
8.4
8.5
8.6

Critical Reasoning
Critical Reasoning
What Is Measured
Test-Taking Strategies
The Directions
Practice Questions
Answer Key
Answer Explanations

504
505
506
506
507
508
546
547

9.0
9.0
9.1
9.2
9.3
9.4
9.5
9.6
9.7
9.8

Sentence Correction
Sentence Correction
Basic English Grammar Rules
Study Suggestions
What Is Measured
Test-Taking Strategies
The Directions
Practice Questions
Answer Key
Answer Explanations

672
673
673
678
678
678
679
680
709
710

10.0
10.0
10.1
10.2
10.3
10.4

Integrated Reasoning
Integrated Reasoning
What Is Measured
The Question Types
Test-Taking Strategies
The Directions

788
789
789
790
797
799

11.0
11.0
11.1
11.2
11.3
11.4
11.5
11.6

Analytical Writing Assessment
Analytical Writing Assessment
What Is Measured
Test-Taking Strategies
The Directions
GMAT® Scoring Guide: Analysis of an Argument
Sample: Analysis of an Argument
Analysis of an Argument Sample Topics

800
801
801
802
802
803
805
809

Answer Sheets
Diagnostic Answer Sheet—Quantitative
Diagnostic Answer Sheet—Verbal
Problem Solving Answer Sheet
Data Sufficiency Answer Sheet
Reading Comprehension Answer Sheet
Critical Reasoning Answer Sheet
Sentence Correction Answer Sheet

841
841
841
842
843
844
845
846

Appendix A

Dear GMAT Test-Taker,
Thank you for your interest in graduate management education. Taking the GMAT® exam lets schools
know that you’re serious about your graduate business education. By using the Official Guide to prepare
for the GMAT, you’re taking a very important step toward achieving your goals and pursuing admission
to a high-quality business school or master’s program.
This book, GMAT ® Official Guide 2018, is designed to help you prepare for and build confidence to do
your best on the GMAT exam. It’s the only guide of its kind on the market that includes real GMAT
exam questions published by the Graduate Management Admission Council (GMAC), the makers of
the exam.
GMAC was founded by the world’s leading business schools in 1953. The GMAT exam was developed
to help people who aspire to careers in management demonstrate their command of the skills needed
for success in the classroom. Schools use and trust the GMAT exam as part of their admissions process
because it’s an excellent predictor of classroom success and your ability to excel in your chosen program.
Today more than 6,500 graduate programs around the world use the GMAT exam to establish their
MBA, graduate-level management degrees and specialized programs as hallmarks of excellence. Nine out
of 10 new MBA enrollments at Top 50 US full-time MBA programs are made using a GMAT
score.
These facts make us proud and drive us to keep improving the GMAT as well as play a role in helping
you find and gain admission to the best school or program for you. We’re committed to ensuring
that no talent goes undiscovered, and that more people around the world can pursue opportunities in
graduate management education.
I applaud your commitment to educational success, and I know that this book and the other Official
GMAT preparation materials available at mba.com will give you the confidence to achieve your
personal best on the GMAT exam and launch or reinvigorate a rewarding career.
I wish you the best success on all your educational and professional endeavors in the future.
Sincerely,

Sangeet Chowfla
CEO of the Graduate Management Admission Council

GMAT® Official Guide 2018

1.0 What Is the GMAT® Exam?

2

1.0 What Is the GMAT® Exam?

1.0 What Is the GMAT® Exam?
The Graduate Management Admission Test® (GMAT®) exam is a standardized exam used in
admissions decisions by more than 6,500 graduate management programs worldwide. It helps you gauge,
and demonstrate to schools, your academic potential for success in graduate-level management studies.
The four‐part exam measures your Analytical Writing, Verbal, Quantitative, and Integrated Reasoning
skills—higher‐order reasoning skills that management faculty worldwide have identified as important
for incoming students to have. “Higher-order” reasoning skills involve complex judgments, and include
critical thinking, analysis, and problem solving. Unlike undergraduate grades and curricula, which vary
in their meaning across regions and institutions, your GMAT scores provide a standardized, statistically
valid and reliable measure of how you are likely to perform academically in the core curriculum of a
graduate management program. The GMAT exam’s validity, fairness, and value in admissions have been
well‐established through numerous academic studies.
The GMAT exam is delivered entirely in English and solely on a computer. It is not a test of business
knowledge, subject matter mastery, English vocabulary, or advanced computational skills. The GMAT
exam also does not measure other factors related to success in graduate management study, such as job
experience, leadership ability, motivation, and interpersonal skills. Your GMAT score is intended to be
used as one admissions criterion among other, more subjective, criteria, such as admissions essays and
interviews.

1.1 Why Take the GMAT® Exam?
Launched in 1954 by a group of nine business schools to
provide a uniform measure of the academic skills needed to
succeed in their programs, the GMAT exam is now used
by more than 6,500 graduate management programs at
approximately 2,100 institutions worldwide.
Taking the GMAT exam helps you stand out in the
admissions process and demonstrate your readiness and
commitment to pursuing graduate management education.
Schools use GMAT scores to help them select the most
qualified applicants—because they know that candidates
who take the GMAT exam are serious about earning a
graduate business degree, and it’s a proven predictor of a
student’s ability to succeed in his or her chosen program.
When you consider which programs to apply to, you can
look at a school’s use of the GMAT exam as one indicator
of quality. Schools that use the GMAT exam typically list
score ranges or average scores in their class profiles, so you
may also find these profiles helpful in gauging the academic
competitiveness of a program you are considering and how
well your performance on the exam compares with that of
the students enrolled in the program.

Myth -vs- FACT
M – If I don’t achieve a high score
on the GMAT, I won’t get into
my top choice schools.
F – There are great schools
available for candidates at any
GMAT score range.
Fewer than 50 of the more than 250,000
people taking the GMAT exam each year get
a perfect score of 800; and many more get
into top business school programs around
the world each year. Admissions Officers
use GMAT scores as one component in their
admissions decisions, in conjunction with
undergraduate records, application essays,
interviews, letters of recommendation, and
other information when deciding whom
to accept into their programs. Visit School
Finder on mba.com to learn about schools
that are the best fit for you.

No matter how you perform on the GMAT exam, you should contact the schools that interest you
to learn more and to ask how they use GMAT scores and other criteria (such as your undergraduate
grades, essays, and letters of recommendation) in their admissions processes. School admissions
3

GMAT® Official Guide 2018

offices, web sites, and materials published by schools are the key sources of information when you are
doing research about where you might want to go to business school.
For more information on the GMAT, test preparation materials, registration, how to use and send your
GMAT scores to schools, and applying to business school, please visit mba.com.

1.2 GMAT® Exam Format
The GMAT exam consists of four separately timed sections
(see the table on the next page). For the Analytical Writing
Assessment (AWA) essay prompt, you will have 30 minutes
to type your essay on a computer keyboard. The 30-minute
Integrated Reasoning section features 12 question prompts
in four different formats. There are two 75-minute,
multiple-choice sections: the Quantitative section, with 37
questions, and the Verbal section, with 41 questions.
The Verbal and Quantitative sections of the GMAT exam
are computer adaptive, which means that the test draws
from a large bank of questions to tailor itself to your ability
level, and you won’t get many questions that are too hard
or too easy for you. The first question will be of medium
difficulty. As you answer each question, the computer scores
your answer and uses it—as well as your responses to all
preceding questions—to select the next question.
Computer‐adaptive tests become more difficult the more
questions you answer correctly, but if you get a question
that seems easier than the last one, it does not necessarily
mean you answered the last question incorrectly. The test
has to cover a range of content, both in the type of question
asked and the subject matter presented.

Myth -vs- FACT
M – Getting an easier question
means I answered the last
one wrong.
F – You should not become
distracted by the difficulty level
of a question.
Most people are not skilled at estimating
question difficulty, so don’t worry when
taking the test or waste valuable time trying
to determine the difficulty of the question you
are answering.
To ensure that everyone receives the same
content, the test selects a specific number
of questions of each type. The test may call
for your next problem to be a relatively hard
data sufficiency question involving arithmetic
operations. But, if there are no more
relatively difficult data sufficiency questions
involving arithmetic, you might be given an
easier question.

Because the computer uses your answers to select your next questions, you may not skip questions or go
back and change your answer to a previous question. If you don’t know the answer to a question, try to
eliminate as many choices as possible, then select the answer you think is best.
Though the individual questions are different, the mix of question types is the same for every GMAT
exam. Your score is determined by the difficulty and statistical characteristics of the questions you
answer as well as the number of questions you answer correctly. By adapting to each test‐taker, the
GMAT exam is able to accurately and efficiently gauge skill levels over a full range of abilities, from
very high to very low.
The test includes the types of questions found in this book and in the online Integrated Reasoning
component, but the format and presentation of the questions are different on the computer. When you
take the test:
• Only one question or question prompt at a time is presented on the computer screen.
• The answer choices for the multiple‐choice questions will be preceded by circles, rather than by
letters.
4

1.3 What Is the GMAT® Exam? What Is the Content of the Test Like?

• Different question types appear in random order in the multiple‐choice and Integrated Reasoning
sections.
• You must select your answer using the computer.
• You must choose an answer and confirm your choice before moving on to the next question.
• You may not go back to previous screens to change answers to previous questions.

Format of the GMAT® Exam
Questions

Timing

Analytical Writing
Analysis of an Argument

1

30 min.

Integrated Reasoning
Multi‐Source Reasoning
Table Analysis
Graphics Interpretation
Two‐Part Analysis

12

30 min.

Optional break
Quantitative
Problem Solving
Data Sufficiency

up to 8 min.
37

Optional break
Verbal
Reading Comprehension
Critical Reasoning
Sentence Correction

75 min.

up to 8 min.
41

75 min.

Total Time:

210 min.

1.3 What Is the Content of the Test Like?
The GMAT exam measures higher‐order analytical skills encompassing several types of reasoning. The
Analytical Writing Assessment asks you to analyze the reasoning behind an argument and respond
in writing; the Integrated Reasoning section asks you to interpret and synthesize information from
multiple sources and in different formats to make reasoned conclusions; the Quantitative section asks
you to reason quantitatively using basic arithmetic, algebra, and geometry; and the Verbal section asks
you to read and comprehend written material and to reason and evaluate arguments.
Test questions may address a variety of subjects, but all of the information you need to answer the
questions will be included on the exam, with no outside knowledge of the subject matter necessary. The
GMAT exam is not a test of business knowledge, English vocabulary, or advanced computational skills.
You will need to read and write in English and have basic math and English skills to perform well on
the test, but its difficulty comes from the English analytical and critical thinking abilities.
The questions in this book are organized by question type and from easiest to most difficult, but keep in mind
that when you take the test, you may see different types of questions in any order within each section.
5

GMAT® Official Guide 2018

1.4 Integrated Reasoning Section
The Integrated Reasoning section highlights the relevant skills that business managers in today’s datadriven world need in order to analyze sophisticated streams of data and solve complex problems. It
measures your ability to understand and evaluate multiple sources and types of information—graphic,
numeric, and verbal—as they relate to one another; use both quantitative and verbal reasoning to solve
complex problems; and solve multiple problems in relation to one another.
Four types of questions are used in the Integrated Reasoning section:
• Multi‐Source Reasoning
• Table Analysis
• Graphics Interpretation
• Two‐Part Analysis
Integrated Reasoning questions may be quantitative, verbal, or a combination of both. You will have to
interpret graphics and sort tables to extract meaning from data, but advanced statistical knowledge and
spreadsheet manipulation skills are not necessary. You will have access to an on-screen calculator with
basic functions for the Integrated Reasoning section, but note that the calculator is not available on the
Quantitative section.
To review the Integrated Reasoning question types and test‐taking tips, see chapter 10. For practice
questions of each format, with full answer explanations, visit the Integrated Reasoning online
component using your unique access code found in the inside front cover of the book.

1.5 Quantitative Section
The GMAT Quantitative section measures your ability to reason quantitatively, solve quantitative
problems, and interpret graphic data.
Two types of multiple‐choice questions are used in the Quantitative section:
• Problem Solving
• Data Sufficiency
Both are intermingled throughout the Quantitative section, and require basic knowledge of arithmetic,
elementary algebra, and commonly known concepts of geometry.
To review the basic mathematical concepts that you will need to answer Quantitative questions, see the
math review in chapter 4. For test‐taking tips specific to the question types in the Quantitative section,
practice questions, and answer explanations, see chapters 5 and 6.

1.6 Verbal Section
The GMAT Verbal section measures your ability to read and comprehend written material and to
reason and evaluate arguments. The Verbal section includes reading sections from several different
content areas. Although you may be generally familiar with some of the material, neither the reading
passages nor the questions assume detailed knowledge of the topics discussed.
6

1.7 What Is the GMAT® Exam? Analytical Writing Assessment

Three types of multiple‐choice questions are intermingled throughout the Verbal section:
• Reading Comprehension
• Critical Reasoning
• Sentence Correction
All three require basic knowledge of the English language, but the Verbal section is not a test of
advanced vocabulary.
For test‐taking tips specific to each question type in the Verbal section, practice questions, and answer
explanations, see chapters 7 through 9.

1.7 Analytical Writing Assessment
The Analytical Writing Assessment (AWA) consists of one 30‐minute writing task: Analysis of an
Argument. The AWA measures your ability to think critically, communicate your ideas, and formulate
an appropriate and constructive critique. You will type your essay on a computer keyboard.
For test‐taking tips, sample essay responses, answer explanations, and sample Analysis of an Argument
topics, see chapter 11.

1.8 What Computer Skills Will I Need?
The GMAT exam requires only computer skills. You will type your AWA essay on the computer
keyboard using standard word‐processing keystrokes. In the Integrated Reasoning and
multiple‐choice sections, you will select your responses using either your computer mouse or the
keyboard. The Integrated Reasoning section includes basic computer navigation and functions, such as
clicking on tabs and using drop‐down menus to sort tables and select answers.

1.9 What Are the Test Centers Like?
The GMAT exam is administered under standardized conditions at test centers worldwide. Each test
center has a proctored testing room with individual computer workstations that allow you to sit for the
exam under quiet conditions and with some privacy. You will be able to take two optional 8-minute
breaks during the course of the exam. You may not take notes or scratch paper with you into the testing
room, but an erasable notepad and marker will be provided for you to use during the test.

1.10 How Are Scores Calculated?
Verbal and Quantitative sections are scored on a scale of 0 to 60, with scores below 6 or above 51
extremely rare. The Total GMAT score ranges from 200 to 800 and is based on your performance in
these two sections. Your score is determined by:
• The number of questions you answer
• The number of questions you answer correctly or incorrectly
• The level of difficulty and other statistical characteristics of each question

7

GMAT® Official Guide 2018

Your Verbal, Quantitative, and Total GMAT scores are determined by a complex mathematical
procedure that takes into account the difficulty of the questions that were presented to you and how
you answered them. When you answer the easier questions correctly, you get a chance to answer harder
questions, making it possible to earn a higher score. After you have completed all the questions on the
test, or when your time is expired, the computer will calculate your scores. Your scores on the Verbal and
Quantitative sections are combined to produce your Total score which ranges from 200 to 800 in
10 point increments.
The Analytical Writing Assessment consists of one writing task, Analysis of an Argument, and your essay
will be scored two times independently. Essays are evaluated by college and university faculty members
from a variety of disciplines, including management education, who rate the overall quality of your critical
thinking and writing. (For details on how readers are qualified, visit mba.com.) In addition, your response
may be scored by an automated scoring program designed to reflect the judgment of expert readers.
Your essay is scored on a scale of 0 to 6, in half point increments, with 6 being the highest score and
0 the lowest. A score of zero is given for responses that are off topic, are in a foreign language, merely
attempt to copy the topic, consist only of keystroke characters, or are blank. Your AWA score is typically
the average of two independent ratings. If the independent scores vary by more than a point, a third
reader adjudicates, but because of ongoing training and monitoring, discrepancies are rare.
Your Integrated Reasoning section is scored on a scale of 1 to 8, in one-point increments. You must
answer all parts of a single question correctly in order to receive credit. No partial credit is given. Like
your AWA score, your Integrated Reasoning score will not count toward your Total score.
Your Analytical Writing Assessment and Integrated Reasoning scores are computed and reported separately
from the other sections of the test and have no effect on your Verbal, Quantitative, or Total scores. The
schools that you have designated to receive your scores may receive a copy of your Analytical Writing
Assessment essay with your score report. Your own copy of your score report will not include your essay.
Your GMAT score includes a percentile ranking that compares your skill level with other test-takers
from the past three years. The percentile rank of your score shows the percentage of tests taken with
scores lower than your score. Every July, percentile ranking tables are updated. Visit mba.com to view
the most recent percentile rankings tables.

1.11 Test Development Process
The GMAT exam is developed by experts who use standardized procedures to ensure high‐quality,
widely-appropriate test material. All questions are subjected to independent reviews and are revised
or discarded as necessary. Multiple‐choice questions are tested during GMAT exam administrations.
Analytical Writing Assessment tasks are tested on mba.com registrants and then assessed for their
fairness and reliability. For more information on test development, visit mba.com.

8

1.11 What Is the GMAT® Exam? Test Development Process

To register for the GMAT exam, go to www.mba.com
9

2.0 How to Prepare

10

2.0 How to Prepare

2.0 How to Prepare
2.1 How Should I Prepare to Take the Test?
The GMAT® exam is designed specifically to measure reasoning skills needed for management
education, and the test contains several question formats unique to the GMAT exam. At a minimum,
you should be familiar with the test format and the question formats before you sit for the test. Because
the GMAT exam is a timed exam, you should practice answering test questions, not only to better
understand the question formats and the skills they require, but also to help you learn to pace yourself
so you can finish each section when you sit for the exam.
Because the exam measures reasoning rather than
subject matter knowledge, you most likely will not find
it helpful to memorize facts. You do not need to study
advanced mathematical concepts, but you should be sure
your grasp of basic arithmetic, algebra, and geometry is
sound enough that you can use these skills in quantitative
problem solving. Likewise, you do not need to study
advanced vocabulary words, but you should have a firm
understanding of basic English vocabulary and grammar
for reading, writing, and reasoning.

Myth -vs- FACT
M – You need very advanced math
skills to get a high GMAT score.
F – The GMAT measures your
reasoning and critical thinking
abilities, rather than your
advanced math skills.
The GMAT exam only requires basic quantitative

This book and other study materials released by the
skills. You should review the math skills
(algebra, geometry, basic arithmetic) presented
Graduate Management Admission Council (GMAC)
in this guide (chapter 4) and the GMAT®
are the ONLY source of questions that have been retired
Official Guide Quantitative Review 2018. The
from the GMAT exam. All questions that appear or
difficulty of GMAT Quantitative questions stems
have appeared on the GMAT exam are copyrighted and
from the logic and analysis used to solve the
problems and not the underlying math skills.
owned by GMAC, which does not license them to be
reprinted elsewhere. Accessing live Integrated Reasoning,
Quantitative, or Verbal test questions in advance or sharing test content during or after you take the test
is a serious violation, which could cause your scores to be canceled and schools to be notified. In cases of
a serious violation, you may be banned from future testing and other legal remedies may be pursued.

2.2 What About Practice Tests?
The Quantitative and Verbal sections of the GMAT exam are computer adaptive, and the Integrated
Reasoning section includes questions that require you to use the computer to sort tables and navigate to
different sources of information. Our official practice materials will help you get comfortable with the
format of the test and better prepare for exam day. Two full-length GMAT practice exams are available at no
charge for those who have created an account on mba.com. The practice exams include computer‐adaptive
Quantitative and Verbal sections, plus additional practice questions, information about the test, and tutorials
to help you become familiar with how the GMAT exam will appear on the computer screen at the test center.
To maximize your studying efforts with the free practice exams, you should leverage official practice
materials as you start to prepare for the test. Take one practice test to make yourself familiar with the
exam and to get a baseline score. After you have studied using this book and other study materials, take
the second practice test to determine whether you need to shift your focus to other areas you need to
strengthen. Note that the free practice tests may include questions that are also published in this book.
As your test day approaches, consider taking more official practice tests to help measure your progress
and give you a better idea of how you might score on exam day.
11

GMAT® Official Guide 2018

2.3 How Should I Use the Diagnostic Test?
This book contains a Diagnostic Test to help you determine the types of Quantitative and Verbal
questions that you need to practice most. You should take the Diagnostic Test around the same time
that you take the first GMAT practice test. The Diagnostic Test will give you a rating—below average,
average, above average, or excellent—of your skills in each type of GMAT test question. These ratings will
help you identify areas to focus on as you prepare for the GMAT exam.
The Diagnostic Test does not include Integrated Reasoning or Analytical Writing Assessment questions.
Use the results of the Diagnostic Test to help you select the right chapter of this book to start with.
Next, read the introductory material carefully, and answer the practice questions in that chapter.
Remember, the questions in the chapters are organized by difficulty, from easiest to most difficult. Make
sure you follow the directions for each type of question and try to work as quickly and as efficiently as
possible. Then review the explanations for the correct answers, spending as much time as necessary to
familiarize yourself with the range of questions or problems presented.

2.4 Where Can I Get Additional Practice?
If you would like additional practice, GMAT® Official Guide 2018 Quantitative Review and GMAT®
Official Guide 2018 Verbal Review each offer 300 additional questions that are not published in this
guide. You can also find more Quantitative, Verbal, and Integrated Reasoning practice questions, fulllength, computer-adaptive practice exams, Analytical Writing Assessment practice prompts, and other
helpful study materials at mba.com.

2.5 General Test-Taking Suggestions
Specific test‐taking strategies for individual question types are presented later in this book. The
following are general suggestions to help you perform your best on the test.
1.

Use your time wisely.

Although the GMAT exam stresses accuracy more than speed, it is important to use your time wisely. On
average, you will have about 1¾ minutes for each Verbal question, about 2 minutes for each Quantitative
question, and about 2½ minutes for each Integrated Reasoning question, some of which have multiple
questions. Once you start the test, an onscreen clock will show the time you have left. You can hide this
display if you want, but it is a good idea to check the clock periodically to monitor your progress. The clock
will automatically alert you when 5 minutes remain for the section you are working on.
2.

Answer practice questions ahead of time.

After you become generally familiar with all question types, use the practice questions in this book
and the online Integrated Reasoning component to prepare for the actual test. It may be useful to time
yourself as you answer the practice questions to get an idea of how long you will have for each question
when you sit for the actual test, as well as to determine whether you are answering quickly enough to
finish the test in the allotted time.

12

2.0 How to Prepare General Test-Taking Suggestions

3.

Read all test directions carefully.

The directions explain exactly what is required to
answer each question type. If you read hastily, you may
miss important instructions and impact your ability to
answer correctly. To review directions during the test,
click on the Help icon. But be aware that the time you
spend reviewing directions will count against your time
allotment for that section of the test.
4.

Read each question carefully and thoroughly.

Before you answer a question, determine exactly what is
being asked and then select the best choice. Never skim
a question or the possible answers; skimming may cause
you to miss important information or nuances.
5.

Do not spend too much time on any one question.

If you do not know the correct answer, or if the
question is too time consuming, try to eliminate choices
you know are wrong, select the best of the remaining
answer choices, and move on to the next question.
Not completing sections and randomly guessing
answers to questions at the end of each test section
can significantly lower your score. As long as you have
worked on each section, you will receive a score even if
you do not finish one or more sections in the allotted
time. You will not earn points for questions you never
get to see.
6.

Confirm your answers ONLY when you are ready to
move on.

On the Quantitative and Verbal sections, once you have
selected your answer to a multiple‐choice question,
you will be asked to confirm it. Once you confirm your
response, you cannot go back and change it. You may
not skip questions. In the Integrated Reasoning section,
there may be several questions based on information
provided in the same question prompt. When there
is more than one response on a single screen, you can
change your response to any of the questions on the
screen before moving on to the next screen. However,
you may not navigate back to a previous screen to
change any responses.
7.

Myth -vs- FACT
M – It is more important to respond
correctly to the test questions
than it is to finish the test.
F – There is a significant penalty
for not completing the GMAT
exam.
Pacing is important. If you are stumped by a
question, give it your best guess and move
on. If you guess incorrectly, the computer
program will likely give you an easier
question, which you are likely to answer
correctly, and the computer will rapidly
return to giving you questions matched to
your ability. If you don’t finish the test, your
score will be reduced. Failing to answer five
verbal questions, for example, could reduce
your score from the 91st percentile to the
77th percentile.

Myth -vs- FACT
M – The first 10 questions are
critical and you should invest
the most time on those.
F – All questions count.
The computer-adaptive testing algorithm
uses each answered question to obtain an
initial estimate. However, as you continue to
answer questions, the algorithm self-corrects
by computing an updated estimate on the
basis of all the questions you have answered,
and then administers items that are closely
matched to this new estimate of your ability.
Your final score is based on all your responses
and considers the difficulty of all the
questions you answered. Taking additional
time on the first 10 questions will not game
the system and can hurt your ability to finish
the test.

Plan your essay answer before you begin to write.

The best way to approach the Analysis of an Argument
section is to read the directions carefully, take a few minutes to think about the question, and plan a
response before you begin writing. Take time to organize your ideas and develop them fully, but leave
time to reread your response and make any revisions that you think would improve it.

13

3.0 Diagnostic Test

14

3.0 Diagnostic Test

3.0 Diagnostic Test
Like the practice sections later in the book, the Diagnostic Test uses questions from real GMAT®
exams. The purpose of the Diagnostic Test is to help you determine how skilled you are in answering
each of the five types of questions on the GMAT exam: problem solving, data sufficiency, reading
comprehension, critical reasoning, and sentence correction.
Scores on the Diagnostic Test are designed to help you answer the question, “If all the questions on the
GMAT exam were like the questions in this section, how well would I do?” Your scores are classified as
being excellent, above average, average, or below average, relative to the scores of other test‐takers. You
can use this information to focus your test‐preparation activities.

		Instructions
1. Take your time answering these questions. The Diagnostic Test is not timed.
2. If you are stumped by a question, you should guess and move on, just like you should do on the
real GMAT exam.
3. You can take one segment at a time, if you want. It is better to finish an entire section
(Quantitative or Verbal) in one sitting, but this is not a requirement.
4. You can go back and change your answers in the Diagnostic Test.
5. After you take the test, check your answers using the answer key that follows the test. The number
of correct answers is your raw score.
6. Convert your raw score, using the table provided.
Note: The Diagnostic Test is designed to give you guidance on how to prepare for the GMAT exam;
however, a strong score on one type of question does not guarantee that you will perform as well on the
real GMAT exam. The statistical reliability of scores on the Diagnostic Test ranges from 0.75 to 0.89,
and the subscale classification is about 85%–90% accurate, meaning that your scores on the Diagnostic
Test are a good, but not perfect, measure of how you are likely to perform on the real test. Use the tests
on the free online software to obtain a good estimate of your expected GMAT Verbal, Quantitative, and
Total scores.
You should not compare the number of questions you got right in each section. Instead, you should
compare how your responses are rated in each section.

15

GMAT® Official Guide 2018

3.1 Quantitative Questions
Problem Solving
Solve the problem and indicate the best of the answer choices given.
Numbers: All numbers used are real numbers.
Figures: All figures accompanying problem solving questions are intended to provide information
useful in solving the problems. Figures are drawn as accurately as possible. Exceptions will
be clearly noted. Lines shown as straight are straight, and lines that appear jagged are also
straight. The positions of points, angles, regions, etc., exist in the order shown, and angle
measures are greater than zero. All figures lie in a plane unless otherwise indicated.
1.

2.

3.

16

Last month a certain music club offered a discount
to preferred customers. After the first compact disc
purchased, preferred customers paid $3.99 for each
additional compact disc purchased. If a preferred
customer purchased a total of 6 compact discs and
paid $15.95 for the first compact disc, then the dollar
amount that the customer paid for the 6 compact
discs is equivalent to which of the following?
(A)

5(4.00) + 15.90

(B)

5(4.00) + 15.95

(C)

5(4.00) + 16.00

(D)

5(4.00 − 0.01) + 15.90

(E)

5(4.00 − 0.05) + 15.95

4.

The average (arithmetic mean) of the integers from
200 to 400, inclusive, is how much greater than the
average of the integers from 50 to 100, inclusive?
(A)

150

(B)

175

(C)

200

(D)

225

(E)

300

The sequence a1, a2 , a3 , , an ,  is such that
a +a
an = n −1 n −2 for all n ≥ 3. If a3 = 4 and
2
a5 = 20 what is the value of a6 ?

5.

Among a group of 2,500 people, 35 percent invest
in municipal bonds, 18 percent invest in oil stocks,
and 7 percent invest in both municipal bonds and oil
stocks. If 1 person is to be randomly selected from the
2,500 people, what is the probability that the person
selected will be one who invests in municipal bonds but
NOT in oil stocks?
(A)

9
50

(B)

7
25

(C)

7
20

(D)

21
50

(E)

27
50

A closed cylindrical tank contains 36π cubic feet of
water and is filled to half its capacity. When the tank
is placed upright on its circular base on level ground,
the height of the water in the tank is 4 feet. When the
tank is placed on its side on level ground, what is the
height, in feet, of the surface of the water above the
ground?
(A)

2

(B)

3

(C)

4

(A)

12

(D)

6

(B)

16

(E)

9

(C)

20

(D)

24

(E)

28

3.1 Diagnostic Test Quantitative Questions

6.

7.

A marketing firm determined that, of 200 households
surveyed, 80 used neither Brand A nor Brand B soap,
60 used only Brand A soap, and for every household
that used both brands of soap, 3 used only Brand B
soap. How many of the 200 households surveyed used
both brands of soap?
(A)

15

(B)

20

(C)

30

(D)

40

(E)

45

A researcher computed the mean, the median, and the
standard deviation for a set of performance scores.
If 5 were to be added to each score, which of these
three statistics would change?
(A)

The mean only

(B)

The median only

(C)

The standard deviation only

(D)

The mean and the median

(E)

The mean and the standard deviation

y°

A certain club has 10 members, including Harry. One
of the 10 members is to be chosen at random to be
the president, one of the remaining 9 members is to
be chosen at random to be the secretary, and one of
the remaining 8 members is to be chosen at random
to be the treasurer. What is the probability that Harry
will be either the member chosen to be the secretary
or the member chosen to be the treasurer?
(A)

1
720

(B)

1
80

(C)
(D)
(E)
8.

9.

1
10
1
9
1
5

x°

v°

10.

z°

w°

In the figure shown, what is the value of
v+x+y+z+w?
(A)	 45
(B)	 90
(C)

180

(D)

270

(E)

360

2
5
of its revenue in December and its revenue in January
1
was of its revenue in November, then the store’s
4
revenue in December was how many times the average
(arithmetic mean) of its revenues in November and
January?
If a certain toy store’s revenue in November was

(A)

1
4

(B)

1
2

(C)

2
3

(D)

2

(E)

4

17

GMAT® Official Guide 2018

11.

12.

13.

Of the three‐digit integers greater than 700, how many
have two digits that are equal to each other and the
remaining digit different from the other two?

15.

109

90

(B)

108

(B)

82

(C)

107

(C)

80

(D)

106

(D)

45

(E)

105

(E)

36

16.

Positive integer y is 50 percent of 50 percent of
positive integer x, and y percent of x equals 100. What
is the value of x ?
(A)

  50

(B)

100

(C)

200

(D)

1,000

(E)

2,000

17.

20

(E)

45

Of the 84 parents who attended a meeting at a school,
35 volunteered to supervise children during the school
picnic and 11 volunteered both to supervise children
during the picnic and to bring refreshments to the
picnic. If the number of parents who volunteered
to bring refreshments was 1.5 times the number
of parents who neither volunteered to supervise
children during the picnic nor volunteered to bring
refreshments, how many of the parents volunteered to
bring refreshments?
(A)

25

(B)

36

(C)

38

(D)

42

(E)

45

(A)

1

(B)

4

(C)

2 − 2x

(D)

4x − 2

(E)

6x – 1

If n =

(B)
(C)
(D)

(C)	 8
(D)

If 3 − 2x = 2x + 1, then 4x2 =

(A)

s
If s and t are positive integers such that = 64.12,
t
which of the following could be the remainder when s
is divided by t ?

(B)	 4

18

(A)

(A)

(A)	 2

14.

The product of all the prime numbers less than 20 is
closest to which of the following powers of 10 ?

(E)
18.

19.

16
, what is the value of n ?
81
1
9
1
4
4
9
2
3
9
2

If n is the product of the integers from 1 to 8,
inclusive, how many different prime factors greater
than 1 does n have?
(A)

Four

(B)

Five

(C)

Six

(D)

Seven

(E)

Eight

If k is an integer and 2 < k < 7, for how many different
values of k is there a triangle with sides of lengths
2, 7, and k ?
(A)

One

(B)

Two

(C)

Three

(D)

Four

(E)

Five

3.1 Diagnostic Test Quantitative Questions

20.

A right circular cone is inscribed in a hemisphere so
that the base of the cone coincides with the base of
the hemisphere. What is the ratio of the height of the
cone to the radius of the hemisphere?
(A)

  II.

12

(B)

1:1

III.

18

(C)

1
:1
2

(A)

II only

(B)

I and II only

(C)

I and III only

(D)

II and III only

(E)

I, II, and III

(E)

22.

If the positive integer x is a multiple of 4 and the
positive integer y is a multiple of 6, then xy must be a
multiple of which of the following?
I.	  8

3 :1

2 :1

(D)

21.

23.

2:1

John deposited $10,000 to open a new savings
account that earned 4 percent annual interest,
compounded quarterly. If there were no other
transactions in the account, what was the amount of
money in John’s account 6 months after the account
was opened?
(A)

$10,100

(B)

$10,101

(C)

$10,200

(D)

$10,201

(E)

$10,400

A container in the shape of a right circular cylinder is
1
full of water. If the volume of water in the container
2
is 36 cubic inches and the height of the container
is 9 inches, what is the diameter of the base of the
cylinder, in inches?
(A)

16
9π

(B)

4
π

(C)

12
π

(D)

2
π

(E)

4

24.

Aaron will jog from home at x miles per hour and then
walk back home by the same route at y miles per hour.
How many miles from home can Aaron jog so that he
spends a total of t hours jogging and walking?
(A)
(B)
(C)
(D)
(E)

xt
y
x +t
xy
xyt
x+y
x + y +t
xy
y +t t
−
x
y

2
π

19

GMAT® Official Guide 2018

Data Sufficiency
Each data sufficiency problem consists of a question and two statements, labeled (1) and (2), which
contain certain data. Using these data and your knowledge of mathematics and everyday facts (such
as the number of days in July or the meaning of the word counterclockwise), decide whether the data
given are sufficient for answering the question and then indicate one of the following answer choices:
A
B
C
D
E

Statement (1) ALONE is sufficient, but statement (2) alone is not sufficient.
Statement (2) ALONE is sufficient, but statement (1) alone is not sufficient.
BOTH statements TOGETHER are sufficient, but NEITHER statement ALONE is sufficient.
EACH statement ALONE is sufficient.
Statements (1) and (2) TOGETHER are NOT sufficient.

Note: In data sufficiency problems that ask for the value of a quantity, the data given in the statements
are sufficient only when it is possible to determine exactly one numerical value for the quantity.
Example:

Q

yº

P
xº
zº

R

In ΔPQR, what is the value of x ?
(1)

PQ = PR

(2)

y = 40

Explanation: According to statement (1) PQ = PR; therefore, ΔPQR is isosceles and y = z. Since
x + y + z = 180, it follows that x + 2y = 180. Since statement (1) does not give a value for y, you cannot
answer the question using statement (1) alone. According to statement (2), y = 40; therefore, x + z = 140.
Since statement (2) does not give a value for z, you cannot answer the question using statement (2)
alone. Using both statements together, since x + 2y = 180 and the value of y is given, you can find the
value of x. Therefore, BOTH statements (1) and (2) TOGETHER are sufficient to answer the question, but
NEITHER statement ALONE is sufficient.
Numbers: All numbers used are real numbers.
Figures:
• F igures conform to the information given in the question, but will not necessarily conform to the
additional information given in statements (1) and (2).
• Lines shown as straight are straight, and lines that appear jagged are also straight.
• The positions of points, angles, regions, etc., exist in the order shown, and angle measures are
greater than zero.
• All figures lie in a plane unless otherwise indicated.

20

3.1 Diagnostic Test Quantitative Questions

25.

If the units digit of integer n is greater than 2, what is
the units digit of n ?
(1)

The units digit of n is the same as the units digit
of n2.

(2)

The units digit of n is the same as the units digit
of n3.

31.

32.
26.

27.

28.

29.

(1)

The variance for the set of measurements is 4.

(2)

For each measurement, the difference between
the mean and that measurement is 2.

What is the value of the integer p ?

Is the range of the integers 6, 3, y, 4, 5, and x greater
than 9 ?

(1)

Each of the integers 2, 3, and 5 is a factor of p.

(1)

y > 3x

(2)

Each of the integers 2, 5, and 7 is a factor of p.

(2)

y>x>3

If the length of Wanda’s telephone call was rounded
up to the nearest whole minute by her telephone
company, then Wanda was charged for how many
minutes for her telephone call?
(1)

The total charge for Wanda’s telephone call was
$6.50.

(2)

Wanda was charged $0.50 more for the first
minute of the telephone call than for each minute
after the first.

33.

34.

What is the perimeter of isosceles triangle MNP ?
(1)

MN = 16

(2)

NP = 20

In a survey of retailers, what percent had purchased
computers for business purposes?
(1)

(2)

30.

Is the standard deviation of the set of measurements
x1, x2, x3, x4, . . ., x20 less than 3 ?

85 percent of the retailers surveyed who owned
their own store had purchased computers for
business purposes.
40 percent of the retailers surveyed owned their
own store.

The only gift certificates that a certain store sold
yesterday were worth either $100 each or $10 each. If
the store sold a total of 20 gift certificates yesterday,
how many gift certificates worth $10 each did the
store sell yesterday?
(1)

(2)

35.

The gift certificates sold by the store yesterday
were worth a total of between $1,650 and
$1,800.

36.

37.

Is

5x +2
< 1?
25

(1)

5x < 1

(2)

x<0

Of the companies surveyed about the skills they
required in prospective employees, 20 percent
required both computer skills and writing skills. What
percent of the companies surveyed required neither
computer skills nor writing skills?
(1)

Of those companies surveyed that required
computer skills, half required writing skills.

(2)

45 percent of the companies surveyed required
writing skills but not computer skills.

What is the value of w + q ?
(1)

3w = 3 – 3q

(2)

5w + 5q = 5

If X and Y are points in a plane and X lies inside the
circle C with center O and radius 2, does Y lie inside
circle C ?
(1)

The length of line segment XY is 3.

(2)

The length of line segment OY is 1.5.

Is x > y ?
(1)
(2)

x=y+2
x
= y −1
2

Yesterday the store sold more than 15 gift
certificates worth $100 each.

21

GMAT® Official Guide 2018

38.

39.

40.

41.

42.

43.

22

If Paula drove the distance from her home to her
college at an average speed that was greater than
70 kilometers per hour, did it take her less than
3 hours to drive this distance?
(1)

The distance that Paula drove from her home to
her college was greater than 200 kilometers.

(2)

The distance that Paula drove from her home to
her college was less than 205 kilometers.

44.

(1)
(2)
45.

In the xy‐plane, if line k has negative slope and passes
through the point (−5,r), is the x‐intercept of line
k positive?
(1)

The slope of line k is −5.

(2)

r>0

46.

If $5,000 invested for one year at p percent simple
annual interest yields $500, what amount must be
invested at k percent simple annual interest for one
year to yield the same number of dollars?
(1)

k = 0.8p

(2)

k=8

If

x+y
> 0 , is x < 0 ?
z

If m and n are positive integers, is

47.

( m ) is an integer.
( n ) is an integer.

( m)

n

an integer?

Of the 66 people in a certain auditorium, at most
6 people have their birthdays in any one given month.
Does at least one person in the auditorium have a
birthday in January?
(1)

More of the people in the auditorium have their
birthday in February than in March.

(2)

Five of the people in the auditorium have their
birthday in March.

Last year the average (arithmetic mean) salary of the
10 employees of Company X was $42,800. What is
the average salary of the same 10 employees this
year?
(1)

For 8 of the 10 employees, this year’s salary is
15 percent greater than last year’s salary.

(2)

For 2 of the 10 employees, this year’s salary is
the same as last year’s salary.

(1)

x 7.
For k values 3, 4, and 5, the sum of the smaller
two sides is not larger than the third side; thus,
6 is the only possible value of k that satisfies the
conditions.
The correct answer is A.

3.5 Diagnostic Test Quantitative Answer Explanations

20.

A right circular cone is inscribed in a hemisphere so
that the base of the cone coincides with the base of
the hemisphere. What is the ratio of the height of the
cone to the radius of the hemisphere?
(A)

1:1

(C)

1
:1
2

(E)

Since John’s account is compounded quarterly, he
1
receives of his annual interest, or 1%, every
4
3 months. This is added to the amount already in
the account to accrue interest for the next quarter.
After 6 months, this process will have occurred
twice, so the amount in John’s account will then be
($10,000)(1.01)(1.01) = $10,000(1.01)2 = $10,201.

3 :1

(B)

(D)

Arithmetic Operations on rational numbers

The correct answer is D.

2 :1
2:1
22.

Geometry Volume

As the diagram below shows, the height of the
cone will be the radius of the hemisphere, so the
ratio is 1:1.

r

The correct answer is B.
21.

John deposited $10,000 to open a new savings
account that earned 4 percent annual interest,
compounded quarterly. If there were no other
transactions in the account, what was the amount of
money in John’s account 6 months after the account
was opened?

A container in the shape of a right circular cylinder is
1 full of water. If the volume of water in the container
2
is 36 cubic inches and the height of the container
is 9 inches, what is the diameter of the base of the
cylinder, in inches?
(A)

16
9π

(B)

4
π

(C)

12
π

(D)

2
π

(E)

4

2
π

Geometry Volume

For a right cylinder, volume = π(radius)2(height).
Since the volume of water is 36 cubic inches and
since this represents 1 the container, the water
2
is occupying 1 the container’s height, or
2
9  1  = 4.5 inches. Let r be the radius of the
 2
cylinder.

(A)

$10,100

(B)

$10,101

(C)

$10,200

(D)

$10,201

36 = π r 2 ( 4.5)

(E)

$10,400

8 = πr 2

divide both sides by 4.5

8 = r2
π

divide both sides by π

8 =r
π

take the square root of both sides

2 2
=r
π

simplify the 8 to get the radius

49

GMAT® Official Guide 2018

Then, since the diameter is twice the length of the
radius, the diameter equals
 2 2
2
2
=4
=4 2.

 π 
π
π

23.

24.

Aaron will jog from home at x miles per hour and then
walk back home by the same route at y miles per hour.
How many miles from home can Aaron jog so that he
spends a total of t hours jogging and walking?

The correct answer is E.

(A)

xt
y

If the positive integer x is a multiple of 4 and the
positive integer y is a multiple of 6, then xy must be a
multiple of which of the following?

(B)

x +t
xy

(C)

xyt
x+y

(D)

x + y +t
xy

(E)

y +t t
−
x
y

    I.	  8
  II.

12

III.

18

(A)

II only

(B)

I and II only

(C)

I and III only

(D)

II and III only

(E)

I, II, and III

Arithmetic Properties of numbers

The product xy must be a multiple of 4(6) = 24
and any of its factors. Test each alternative.
  I.

24 = 3
	8 is a factor of 24
8
MUST be a multiple of 8

24 = 2
	12 is a factor of 24
II.
12
MUST be a multiple of 12
III.

24 = 1 1
	18 is NOT a factor of 24
18
3
NEED NOT be a multiple of 18

The correct answer is B.

Algebra Simplifying algebraic expressions

Let j be the number of hours Aaron spends
jogging; then let t – j be the total number of
hours he spends walking. It can be stated that
Aaron jogs a distance of xj miles and walks a
distance of y(t – j) miles. Because Aaron travels
the same route, the miles jogged must equal the
miles walked, and they can be set equal.
xj = y (t − j ) 	set number of miles equal to
each other
xj = yt − jy

distribute the y

xj + jy = yt 	add jy to both sides to get all
terms with j to one side
j ( x + y ) = yt
j=

yt
.
x+ y

factor out the j
divide both sides by x + y

So, the number of hours Aaron spends jogging is
yt
j=
.
x+ y
The number of miles he can jog is xj or, by
 yt 
xyt
=
substitution of this value of j, x 
.

 x+ y x+ y
The correct answer is C.

50

3.5 Diagnostic Test Quantitative Answer Explanations

Data Sufficiency
The following section on data sufficiency is intended to familiarize you with the most efficient and
effective approaches to the kinds of problems common to data sufficiency. The particular questions in
this chapter are generally representative of the kinds of data sufficiency questions you will encounter
on the GMAT exam. Remember that it is the problem solving strategy that is important, not the specific
details of a particular question.

25.

If the units digit of integer n is greater than 2, what is
the units digit of n ?

26.

What is the value of the integer p ?
(1)

Each of the integers 2, 3, and 5 is a factor of p.

(1)

The units digit of n is the same as the units digit
of n2.

(2)

Each of the integers 2, 5, and 7 is a factor of p.

(2)

The units digit of n is the same as the units digit
of n3.

Arithmetic Properties of numbers

(1) These are factors of p, but it is not clear that
they are the only factors of p; NOT sufficient.

Arithmetic Arithmetic operations

(2) These are factors of p, but it is not clear that
they are the only factors of p; NOT sufficient.

If the units digit of n is greater than 2, then it can
only be the digits 3, 4, 5, 6, 7, 8, or 9.
(1) To solve this problem, it is necessary to
find a digit that is the same as the units digit of
its square. For example, both 43 squared (1,849)
and 303 squared (91,809) have a units digit of 9,
which is different from the units digit of 43 and
303. However, 25 squared (625) and 385 squared
(148,225) both have a units digit of 5, and 16 and
226 both have a units digit of 6 and their squares
(256 and 51,076, respectively) do, too. However,
there is no further information to choose between
5 or 6; NOT sufficient.

Taken together, (1) and (2) overlap, but again
there is no clear indication that these are the only
factors of p.
The correct answer is E;
both statements together are still not sufficient.
27.

If the length of Wanda’s telephone call was rounded
up to the nearest whole minute by her telephone
company, then Wanda was charged for how many
minutes for her telephone call?

(2) Once again, 5 and 6 are the only numbers
which, when cubed, will both have a 5 or 6
respectively in their units digits. However, the
information given does not distinguish between
them; NOT sufficient.

(1)

The total charge for Wanda’s telephone call was
$6.50.

(2)

Wanda was charged $0.50 more for the first
minute of the telephone call than for each minute
after the first.

Since (1) and (2) together yield the same
information but with no direction as to which
to choose, there is not enough information to
determine the answer.

Arithmetic Arithmetic operations

The correct answer is E;
both statements together are still not sufficient.

(1) This does not give any information as to the
call’s cost per minute; NOT sufficient.
(2) From this, it can be determined only that
the call was longer than one minute and that the
charge for the first minute was $0.50 more than
the charge for each succeeding minute; NOT
sufficient.

51

GMAT® Official Guide 2018

Taking (1) and (2) together, the number of
minutes cannot be determined as long as the
cost of each minute after the first is unknown.
For example, if the cost of each minute after the
first minute were $0.40, then the cost of the first
minute would be $0.90. Then the total cost of the
other minutes would be $6.50 – $0.90 = $5.60,
and $5.60 ÷ $0.40 would yield 14. In this case,
the time of the call would be 1 + 14 = 15 minutes.
If, however, the cost of each minute after the
first minute were $0.15, then the cost of the first
minute would be $0.65. Then $6.50 – $0.65 would
be $5.85, and this in turn, when divided by $0.15,
would yield 39 minutes, for a total call length of
40 minutes. More information on the cost of each
minute after the first minute is still needed.

29.

MN = 16

(2)

NP = 20

Since it is unclear whether MN or NP is one of
the equal sides, it is not possible to determine the
length of the third side or the perimeter of the
triangle. The perimeter could be either
((2)(16)) + 20 = 52 or ((2)(20)) + 16 = 56.
The correct answer is E;
both statements together are still not sufficient.

40 percent of the retailers surveyed owned their
own store.

Using the information from both (1) and (2),
the percent of surveyed owner‐retailers who had
purchased computers can be deduced, and the
percent of nonowner‐retailers can also be deduced.
However, the information that would permit a
determination of either the percent of nonowner‐
retailers who had purchased computers or the overall
percent of all retailers (both owners and nonowners)
who had purchased computers is still not provided.

The perimeter of a triangle is the sum of all three
sides. In the case of an isosceles triangle, two of
the sides are equal. To determine the perimeter
of this triangle, it is necessary to know both the
length of an equal side and the length of the base
of the triangle.

(2) Only gives the length of one side; NOT
sufficient.

(2)

(2) While this permits the percent of owners
and nonowners in the survey to be deduced, the
overall percent of retailers who had purchased
computers cannot be determined; NOT sufficient.

Geometry Triangles

(1) Only gives the length of one side; NOT
sufficient.

85 percent of the retailers surveyed who owned
their own store had purchased computers for
business purposes.

(1) With only this, it cannot be known
what percent of the retailers not owning their
own store had purchased computers, and so it
cannot be known how many retailers purchased
computers overall; NOT sufficient.

What is the perimeter of isosceles triangle MNP ?
(1)

(1)

Arithmetic Percents

The correct answer is E;
both statements together are still not sufficient.
28.

In a survey of retailers, what percent had purchased
computers for business purposes?

The correct answer is E;
both statements together are still not sufficient.
30.

The only gift certificates that a certain store sold
yesterday were worth either $100 each or $10 each. If
the store sold a total of 20 gift certificates yesterday,
how many gift certificates worth $10 each did the
store sell yesterday?
(1)

The gift certificates sold by the store yesterday
were worth a total of between $1,650 and
$1,800.

(2)

Yesterday the store sold more than 15 gift
certificates worth $100 each.

Algebra Applied problems; Simultaneous
equations; Inequalities

Let x represent the number of $100 certificates
sold, and let y represent the number of $10
certificates sold. Then the given information can
be expressed as x + y = 20 or thus y = 20 – x. The
52

3.5 Diagnostic Test Quantitative Answer Explanations

value of the $100 certificates sold is 100x, and the
value of the $10 certificates sold is 10y.

(2) For each measurement, the difference
between the mean and that measurement is 2.
Therefore, the square of each difference is 4, and
the sum of all the squares is 4 × 20 = 80. The
standard deviation is 80 = 4 = 2, which is less
20
than 3; SUFFICIENT.

(1) From this, it is known that 100x + 10y >1,650.
Since y = 20 − x, this value can be substituted for y,
and the inequality can be solved for x:
100 x + 10 y > 1, 650
100 x + 10(20 − x ) > 1, 650
substitute for y
100x + 200 − 10x > 1,650
distribute
90x + 200 > 1,650
simplify
90x > 1,450	subtract 200 from
both sides
x > 16.1
Thus, more than 16 of the $100 certificates were
sold. If 17 $100 certificates were sold, then it
must be that 3 $10 certificates were also sold for
a total of $1,730, which satisfies the condition of
being between $1,650 and $1,800. If, however,
18 $100 certificates were sold, then it must
be that 2 $10 certificates were sold, and this
totals $1,820, which is more than $1,800 and
fails to satisfy the condition. Therefore, 3 of the
$10 certificates were sold; SUFFICIENT.
(2) From this it can be known only that the
number of $10 certificates sold was 4 or fewer;
NOT sufficient.
The correct answer is A;
statement 1 alone is sufficient.
31.

Is the standard deviation of the set of measurements
x1, x2, x3, x4, …, x20 less than 3 ?
(1)

The variance for the set of measurements is 4.

(2)

For each measurement, the difference between
the mean and that measurement is 2.

Arithmetic Statistics

In determining the standard deviation, the
difference between each measurement and
the mean is squared, and then the squared
differences are added and divided by the number
of measurements. The quotient is the variance
and the positive square root of the variance is the
standard deviation.
(1) If the variance is 4, then the standard
deviation = 4 = 2, which is less than 3;
SUFFICIENT.

The correct answer is D;
each statement alone is sufficient.
32.

Is the range of the integers 6, 3, y, 4, 5, and x greater
than 9 ?
(1)

y > 3x

(2)

y>x>3

Arithmetic Statistics

The range of a set of integers is equal to the
difference between the largest integer and the
smallest integer. The range of the set of integers
3, 4, 5, and 6 is 3, which is derived from 6 – 3.
(1) Although it is known that y > 3x, the value
of x is unknown. If, for example, x = 1, then the
value of y would be greater than 3. However, if
x = 2, then the value of y would be greater than
6, and, since 6 would no longer be the largest
integer, the range would be affected. Because the
actual values of x and y are unknown, the value of
the range is also unknown; NOT sufficient.
(2) If x > 3 and y > x, then x could be 4 and
y could be 5. Then the range of the 6 integers
would still be 6 – 3 or 3. However, if x were 4 and
y were 15, then the range of the 6 integers would
be 15 – 3, or 12. There is no means to establish
the values of x and y, beyond the fact that they
both are greater than 3; NOT sufficient.
Taking (1) and (2) together, it is known that
x > 3 and that y > 3x. Since the smallest integer
that x could be is thus 4, then y > 3(4) or y > 12.
Therefore, the integer y must be 13 or larger.
When y is equal to 13, the range of the 6 integers
is 13 – 3 = 10, which is larger than 9. As y
increases in value, the value of the range will also
increase.
The correct answer is C;
both statements together are sufficient.

53

GMAT® Official Guide 2018

33.

Is

It is given that 20 percent of the surveyed
companies fell into category 1. It is necessary
to determine what percent of the surveyed
companies fell into category 4.

5x +2
< 1?
25

(1)

5x < 1

(2)

x <0

(1) This helps identify the percentage in
1
category 2. Since the companies that required
2
computer skills also required writing skills (i.e.,
those in category 1), then the other 1 of the
2
companies that required computer skills did not
require writing skills (thus category 2 = category
1). However, this information only establishes
that 20 percent required computer skills, but not
writing skills; NOT sufficient.

Algebra Inequalities

Note that x r +s = ( x r )( x s )
5x + 2 < 1
(1) If 5x < 1, then
since
25
5x + 2 = 5x ⋅ 5 2 = 5x ; SUFFICIENT.
25
25
(2) If x < 0 , then
x+2<2

5

x+2

x+2

<5

2

5 <1
25

add 2 to both sides

because a < b implies 5a < 5b

(2) While this establishes category 3, that
is, that 45 percent required writing skills but
not computer skills, no further information is
available; NOT sufficient.

divide both sides by 52 = 25;

SUFFICIENT.

Taking (1) and (2) together, the first three
categories add up to 85 percent (20 + 20 + 45).
Therefore, category 4 would be equal to
100 – 85 = 15 percent of the surveyed companies
required neither computer skills nor writing skills.

The correct answer is D;
each statement alone is sufficient.
34.

Of the companies surveyed about the skills they
required in prospective employees, 20 percent
required both computer skills and writing skills. What
percent of the companies surveyed required neither
computer skills nor writing skills?
(1)

Of those companies surveyed that required
computer skills, half required writing skills.

(2)

45 percent of the companies surveyed required
writing skills but not computer skills.

Arithmetic Percents

The surveyed companies could be placed into one
of the following four categories:
1. Requiring computer skills and requiring
writing skills
2. Requiring computer skills but not requiring
writing skills
3. Not requiring computer skills but requiring
writing skills
4. Not requiring either computer skills or writing
skills

54

The correct answer is C;
both statements together are sufficient.
35.

What is the value of w + q ?
(1)

3w = 3 − 3q

(2)

5w + 5q = 5

Algebra First- and second-degree equations

(1) If 3q is added to both sides of this equation,
it can be rewritten as 3w + 3q = 3. When each
term is then divided by 3, it yields w + q = 1;
SUFFICIENT.
(2) When each term in this equation is divided
by 5, it becomes w + q = 1; SUFFICIENT.
The correct answer is D;
each statement alone is sufficient.

3.5 Diagnostic Test Quantitative Answer Explanations

36.

If X and Y are points in a plane and X lies inside the
circle C with center O and radius 2, does Y lie inside
circle C ?
(1)

The length of line segment XY is 3.

(2)

The length of line segment OY is 1.5.

38.

Geometry Circles

(1) The maximum distance between two points
that lie on a circle is equal to the diameter, or
2 times the radius. Since the radius of circle C
is 2, the diameter in this case is 4. It cannot be
assumed, however, that X and Y are points on the
diameter; X can lie anywhere within the circle.
When the distance between X and Y is 3, it is still
possible either that Y is within the circle or that Y
is outside the circle; NOT sufficient.

(2)

x
= y −1
2

Algebra First- and second-degree equations

(1) Since 2 has to be added to y in order to
make it equal to x, it can be reasoned that x > y;
SUFFICIENT.

(2)

The distance that Paula drove from her home to
her college was less than 205 kilometers.

(1) If D is the distance Paula drove then
D > 200 and D > 200 = 2 6 so t > 2 6 and
7
7
70 70
t may or may not be less than 3; NOT sufficient.
(2)

If D is the distance Paula drove then
D < 205 and D < 205 = 2 13 so t < 2 13 < 3;
14
14
70 70
SUFFICIENT.

Is x > y ?
x = y +2

The distance that Paula drove from her home to
her college was greater than 200 kilometers.

A distance problem uses the formula distance =
rate × time. To find the time, the formula would
be rearranged as time = distance . To solve this
rate
problem, it is necessary to know the rate (given
here as 70 kilometers per hour) and the distance.

The correct answer is B;
statement 2 alone is sufficient.

(1)

(1)

Arithmetic Distance problem

(2) If the length of the line segment OY is 1.5
and the circle has a radius of 2, then the distance
from the center O to point Y is less than the
radius, and point Y must therefore lie within the
circle; SUFFICIENT.

37.

If Paula drove the distance from her home to her
college at an average speed that was greater than
70 kilometers per hour, did it take her less than
3 hours to drive this distance?

The correct answer is B;
statement 2 alone is sufficient.
39.

In the xy‐plane, if line k has negative slope and passes
through the point (–5,r), is the x‐intercept of line k
positive?
(1)

The slope of line k is –5.

(2)

r>0

(2) Multiplying both sides of this equation by
2 results in x = 2(y – 1) or x = 2 y – 2. If y were 0,
then x would be – 2, and y would be greater than
x. If y were a negative number like –2, then
x = 2(–2) – 2 = 6, and again y would be greater
than x. However, if y were a positive number such
as 4, then x = 2(4) –2 = 6, and x > y. Since there is
no other information concerning the value of y, it
cannot be determined if x > y; NOT sufficient.
The correct answer is A;
statement 1 alone is sufficient.

55

GMAT® Official Guide 2018

(2) If k = 8, then the rate is 8 percent, and
the same formula and procedure as above are
employed again; SUFFICIENT.

Geometry Coordinate geometry

The x‐intercept is the x‐coordinate of the point
in which the line k crosses the x‐axis and would
have the coordinates (x,0).
(1) Knowing the slope of the line does not
help in determining the x‐intercept, since from
point (–5,r) the line k extends in both directions.
Without knowing the value of r, the x‐intercept
could be –5 if r were 0, or it could be other
numbers, both positive and negative, depending
on the value of r; NOT sufficient.
(2) Knowing that r > 0 suggests that the
x‐intercept is not –5; the point (–5,r), where r
is a positive number, does lie in quadrant II. It
could, however, be any point with an x‐coordinate
of –5 in that quadrant and line k could have any
negative slope, and so the line k would vary with
the value of r. Therefore, the x‐intercept of line k
cannot be determined; NOT sufficient.
Using (1) and (2) together does not help in
the determination of the x‐intercept, since the
point (–5,r) could have any positive y‐coordinate
and thus line k could cross the x‐axis at many
different places.
The correct answer is E;
both statements together are still not sufficient.
40.

If $5,000 invested for one year at p percent simple
annual interest yields $500, what amount must be
invested at k percent simple annual interest for one
year to yield the same number of dollars?
(1)

k = 0.8p

(2)

k=8

Arithmetic Interest problem

With simple annual interest, the formula to use is
interest = principal × rate × time. It is given
p
that $500 = $5, 000 ×
× 1 (year), so
100
p = 10 percent interest.
(1) If p is 10 percent, then k = 0.8 p is 0.08.
Using the same formula, the time is again 1 year;
the interest is the same amount; and the rate is
0.08, or 8 percent. Thus, $500 = principal × 0.08 × 1,
or principal $6,250; SUFFICIENT.
56

The correct answer is D;
each statement alone is sufficient.
41.

If

x+y
> 0, is x < 0 ?
z

(1)

x 0, then either one of two cases holds
z
true. Either ( x + y ) > 0 and z > 0, or ( x + y ) < 0
and z < 0. In other words, in order for the term to
be greater than zero, it must be true that either 1)
both the numerator and denominator are greater
than 0 or 2) both the numerator and denominator
are less than 0.

If

(1) Regardless of whether ( x + y ) is positive or
negative, the positive or negative value of z must
be in agreement with the sign of ( x + y ) in order
x+ y
for
> 0. However, there is no information
z
about z here; NOT sufficient.
(2) If z < 0, then ( x + y ) must be less than 0.
However, this statement gives no information
about ( x + y ); NOT sufficient.
This can be solved using (1) and (2) together.
From (2), it is known that z < 0, and, going back
to the original analysis, for the term to be greater
than zero, ( x + y ) must also be less than 0.
If x + y < 0 then x < –y. But x < y from (1) so
x+x < −y + y
2x < 0

x < 0.

The correct answer is C;
both statements together are sufficient.

3.5 Diagnostic Test Quantitative Answer Explanations

42.

(1) These data will produce an average of
2160 = 72°
for last April in City X. However,
30
there is no information regarding the median for
comparison; NOT sufficient.

Does the integer k have at least three different positive
prime factors?
(1)

k
is an integer.
15

(2)

k
is an integer.
10

(2) The median is the middle temperature of
the data. As such, 50 percent of the daily high
temperatures will be at or above the median, and
50 percent will be at or below the median. If
60 percent of the daily high temperatures were
less than the average daily high temperature, then
the average of the daily highs must be greater
than the median; SUFFICIENT.

Arithmetic Properties of numbers

(1) The prime factors of 15 are 3 and 5. So in
this case, k has at least 2 different positive prime
factors, but it is unknown if there are more
positive prime factors; NOT sufficient.
(2) The prime factors of 10 are 2 and 5, showing
that k has at least these 2 different positive prime
factors, but k might or might not have more;
NOT sufficient.
Taking (1) and (2) together, since k is divisible
by both 10 and 15, it must be divisible by their
different positive prime factors of 2, 3, and 5.
Thus k has at least 3 different positive prime
factors.
The correct answer is C;
both statements together are sufficient.
43.

In City X last April, was the average (arithmetic mean)
daily high temperature greater than the median daily
high temperature?
(1)

In City X last April, the sum of the 30 daily high
temperatures was 2,160°.

(2)

In City X last April, 60 percent of the daily high
temperatures were less than the average daily
high temperature.

Arithmetic Statistics

The formula for calculating the arithmetic mean,
or the average, is as follows:

The correct answer is B;
statement 2 alone is sufficient.
44.

( m)

n

If m and n are positive integers, is
(1)
(2)

( m ) is an integer.
( n ) is an integer.

an integer?

Arithmetic Properties of numbers

( )
( )

(1) If m is an integer and n is a positive
n
integer, then m is an integer because an
integer raised to a positive integer is an integer;
SUFFICIENT.

( )

(2) The information that n is an integer
is not helpful in answering the question. For
example, if m = 2 and n = 9, 9 = 3, which is
9
an integer, but 2 = 16 2 , which is not
an integer. But if m = 4 and n = 9, then 9 = 3,

( ) (

)

( )

which is an integer, and 4
integer; NOT sufficient.

9

= 29 = 512 is an

The correct answer is A;
statement 1 alone is sufficient.

Average = sum of v values
v

57

GMAT® Official Guide 2018

45.

Of the 66 people in a certain auditorium, at most 6
people have birthdays in any one given month. Does at
least one person in the auditorium have a birthday in
January?
(1)

More of the people in the auditorium have
birthdays in February than in March.

(2)

Five of the people in the auditorium have
birthdays in March.

Arithmetic Statistics

(1) Since all 10 employees did not receive the
same 15 percent increase, it cannot be assumed
that the mean this year is 15 percent higher than
last year. It remains unknown whether these
8 salaries were the top 8 salaries, the bottom
8 salaries, or somewhere in‐between. Without
this type of information from last year, the
mean for this year cannot be determined; NOT
sufficient.

Algebra Sets and functions

(2) If 2 salaries remained the same as last
year, then 8 salaries changed. Without further
information about the changes, the mean for this
year cannot be determined; NOT sufficient.

Because it is given that 6 is the greatest number
of individuals who can have birthdays in any
particular month, these 66 people could be evenly
distributed across 11 of the 12 months of the
year. That is to say, it could be possible for the
distribution to be 11 × 6 = 66, and thus any given
month, such as January, would not have a person
with a birthday. Assume that January has no
people with birthdays, and see if this assumption
is disproved.
(1) The information that more people have
February birthdays than March birthdays
indicates that the distribution is not even.
Therefore, March is underrepresented and must
thus have fewer than 6 birthdays. Since no month
can have more than 6 people with birthdays, and
every month but January already has as many
people with birthdays as it can have, January
has to have at least 1 person with a birthday;
SUFFICIENT.
(2) Again, March is underrepresented with only
5 birthdays, and none of the other months can
have more than 6 birthdays. Therefore, the extra
birthday (from March) must occur in January;
SUFFICIENT.
The correct answer is D;
each statement alone is sufficient.
46.

58

Last year the average (arithmetic mean) salary of the
10 employees of Company X was $42,800. What is
the average salary of the same 10 employees this
year?
(1)

For 8 of the 10 employees, this year’s salary is
15 percent greater than last year’s salary.

(2)

For 2 of the 10 employees, this year’s salary is
the same as last year’s salary.

Even taking (1) and (2) together, it remains
impossible to tell the mean salary for this year
without additional data.
The correct answer is E;
both statements together are still not sufficient.
47.

In a certain classroom, there are 80 books, of which
24 are fiction and 23 are written in Spanish. How many
of the fiction books are written in Spanish?
(1)

Of the fiction books, there are 6 more that are
not written in Spanish than are written in Spanish.

(2)

Of the books written in Spanish, there are 5
more nonfiction books than fiction books.

Algebra Sets and functions

Let x represent the fiction books that are written
in Spanish. A table could be set up like the one
below, filling in the information that is known or
able to be known:
Spanish
Fiction

Non‐Spanish

x

24

Nonfiction
Total

Total
56

23

57

80

(1) If x represents the fiction books written in
Spanish, then x + 6 can now be used to represent
the fiction books that are not written in Spanish.
From the table above, it can be seen then that
x + x + 6 = 24, or 2x = 18. Therefore, x, or the
number of fiction books written in Spanish, is 9;
SUFFICIENT.

3.5 Diagnostic Test Quantitative Answer Explanations

(2) If x represents the fiction books written in
Spanish, then x + 5 can now be used to represent
the nonfiction books written in Spanish. From
the table, it can be said that x + x + 5 = 23, or
2x = 18. Therefore, x, or the number of fiction
books written in Spanish, is 9; SUFFICIENT.
The correct answer is D;
each statement alone is sufficient.
48.

If p is the perimeter of rectangle Q, what is the value of
p?
(1)

Each diagonal of rectangle Q has length 10.

(2)

The area of rectangle Q is 48.

Geometry Rectangles; Perimeter; Area

The perimeter of a rectangle is equal to 2 times
the rectangle’s length plus 2 times the rectangle’s
width, or p = 2l + 2w. The diagonals of a rectangle
are equal. In a rectangle, because a diagonal forms
a right triangle, the length of a diagonal is equal
to the square root of the length squared plus the
width squared, or d = l 2 + w 2.

Using (1) and (2) together, it is possible to solve
this problem. Since from (2) lw = 48, then
w = 48 . Substituting this into 100 = l 2 + w 2
l
from (1) the equation can be solved as follows:
100 = l 2 +  48 
 l 

2

substitution

100l 2 = l 4 + 2, 304

multiply both sides by l2

(l

factor like a quadratic

l 4 − 100l 2 + 2, 304 = 0 	move all terms to one
side
2

2

− 64 )( l 2 − 36 ) = 0
2

l = 64, l = 36

solve for l2

Since l is a length, it must be positive, so l is
48 = 6
either 8 or 6. When l = 8,w =
, and when
8
48
l = 6,w =
= 8, both of which give the same
6
perimeter.
The correct answer is C;
both statements together are sufficient.

(1) If a diagonal = 10, then 10 = l 2 + w 2 , or,
by squaring both sides, 100 = l 2 + w 2 . Without
knowing the value or the relationship between
the other two sides of the right triangle, it is
impossible to solve for l or w, and thus for the
perimeter of the rectangle; NOT sufficient.
(2) If the area of the rectangle is 48, then it can
be stated that lw = 48. However, without further
information, the perimeter cannot be determined.
For example, l could be 6 and w could be 8,
and the perimeter would then be 12 + 16 = 28.
However, it could also be that l is 4 and w is
12, and in that case the perimeter would be
8 + 24 = 32; NOT sufficient.

59

GMAT® Official Guide 2018

3.6 Verbal Answer Explanations
Reading Comprehension
The following discussion is intended to familiarize you with the most efficient and effective approaches
to the kinds of problems common to reading comprehension. The particular questions in this chapter
are generally representative of the kinds of reading comprehension questions you will encounter on the
GMAT exam. Remember that it is the problem solving strategy that is important, not the specific details
of a particular question.

A	The sentence does not explain a failure of
the economic signaling theory.

Questions 1–5 refer to the passage on page 23.
49.

Which of the following best describes the purpose of
the sentence in lines 10–15 ?
(A)

To show that economic signaling theory fails to
explain a finding

B	Economic signaling theory is about
perceptions of quality, but this explanation is
about actual quality and its correlation with
advertising.

(B)

To introduce a distinction not accounted for by
economic signaling theory

(C)

To account for an exception to a generalization
suggested by Marquardt and McGann

C	No exception is mentioned in Marquardt
and McGann’s work.

(D)

To explain why Marquardt and McGann’s research
was conducted

(E)

To offer an explanation for an observation
reported by Marquardt and McGann

D	The sentence does not examine why or how
the research was undertaken.
E

The correct answer is E.

Logical structure

Marquardt and McGann found a correlation
between highly advertised products and high‐
quality products. The connection can be explained
by understanding that companies may invest
heavily in such advertising, anticipating that
recurring purchases of high‐quality products will
eventually recover these advertising costs. The
consumers will continue to buy these products
over time because of loyalty to their high quality.
The statement in bold provides this explanation
for the correlation noted by Marquardt and
McGann.

60

C
 orrect. This statement provides an
explanation of why highly advertised
products did indeed rank high on certain
measures of product quality.

50.

The primary purpose of the passage is to
(A)

present findings that contradict one explanation
for the effects of a particular advertising practice

(B)

argue that theoretical explanations about the
effects of a particular advertising practice are of
limited value without empirical evidence

(C)

discuss how and why particular advertising
practices may affect consumers’ perceptions

(D)

contrast the research methods used in two
different studies of a particular advertising
practice

(E)

explain why a finding about consumer responses
to a particular advertising practice was
unexpected

3.6 Diagnostic Test Verbal Answer Explanations

Main idea

The primary purpose can be determined only by
evaluating the whole passage. The first paragraph
discusses consumers’ perceptions of quality
based on frequency of advertising. The second
paragraph discusses three studies that show how
consumers base their evaluations of products on
the kinds of advertising they see. Therefore, the
purpose of the whole passage is to show how
consumers’ perceptions of products are shaped by
certain advertising practices.

51.

Kirmani’s research, as described in the passage,
suggests which of the following regarding consumers’
expectations about the quality of advertised products?
(A)

Those expectations are likely to be highest if a
manufacturer runs both black‐and‐white and color
advertisements for the same product.

(B)

Those expectations can be shaped by the
presence of color in an advertisement as well as
by the frequency with which an advertisement
appears.

(C)

Those expectations are usually high for
frequently advertised new brands but not for
frequently advertised familiar brands.

(D)

Those expectations are likely to be higher for
products whose black‐and‐white advertisements
are often repeated than for those whose color
advertisements are less often repeated.

C	Correct. The passage shows how the
frequency and the kind of advertising
influence consumers’ perceptions about the
quality of the products advertised.

(E)

Those expectations are less definitively shaped
by the manufacturer’s advertisements than by
information that consumers gather from other
sources.

D	The passage reports the findings of four
studies but does not mention research
methods.

Inference

A	The passage shows that expensive
advertising works to a certain point, but not
after it; this method examines a continuum,
not a contradiction.
B	Most of the passage is devoted to empirical
evidence.

E	The passage does not indicate that any of
the findings were unexpected.
The correct answer is C.

The question’s use of the word suggests means that
the answer depends on making an inference. This
research is discussed in the second paragraph.
Kirmani found that too much advertising
tended to make the consumers believe that
manufacturers were desperate. The use of color
was also found to affect consumers’ perceptions of
brand quality. Realizing that color advertising is
more expensive than black‐and‐white, consumers
react more quickly to what they perceive to be its
overuse than they do to a repetition of black‐and‐
white advertisements.

61

GMAT® Official Guide 2018

A	This situation is not discussed in the
research, at least as it is reported in this
passage.
B	Correct. It can be inferred that consumers’
perceptions of product quality are influenced
by the use of color in an advertisement and
by the frequency of the advertisement’s
appearance.
C	The research does not make a distinction
between new and familiar brands.
D	The research indicates only that consumers
can tolerate black‐and‐white advertisements
for a longer time than color advertisements
before dismissing them as excessive.
E	There is no discussion of what consumers
learn from other sources.
The correct answer is B.
52.

Kirmani’s third study, as described in the passage,
suggests which of the following conclusions about a
black‐and‐white advertisement?
(A)

(B)

62

It can be repeated more frequently than a
comparable color advertisement could before
consumers begin to suspect low manufacturer
confidence in the quality of the advertised
product.
It will have the greatest impact on consumers’
perceptions of the quality of the advertised
product if it appears during periods when a color
version of the same advertisement is also being
used.

(C)

It will attract more attention from readers of the
print publication in which it appears if it is used
only a few times.

(D)

It may be perceived by some consumers as
more expensive than a comparable color
advertisement.

(E)

It is likely to be perceived by consumers as
a sign of higher manufacturer confidence in
the quality of the advertised product than a
comparable color advertisement would be.

Inference

Kirmani’s third study is discussed in the final
two sentences. Consumers suspect expensive
advertising results from a manufacturer’s lack
of confidence in the quality of the product.
Consumers reach the point at which they
find advertising excessive more quickly with
color advertising than with black‐and‐white
advertising because they understand that the
addition of color increases advertising expenses. It
is reasonable to infer that the reverse is also true
and thus that consumers will tolerate lengthier
repetitions of black‐and‐white advertising
without becoming suspicious of product quality.
Correct. Consumers find color advertising
A	
excessive more quickly and thus can
be expected to find black‐and‐white
advertising excessive less quickly.
B	The study does not discuss concurrent
appearances of color and black‐and‐white
advertisements for the same product.
C	The sole conclusion about frequency is that
consumers can tolerate a greater frequency
of black‐and‐white advertisements than
color advertisements.
D	It is stated that consumers understand that
color advertisements are more expensive.
E	The research certainly does not report this
finding.
The correct answer is A.

3.6 Diagnostic Test Verbal Answer Explanations

53.

A	Kirmani’s research, as reported here, does
not support this claim.

The passage suggests that Kirmani would be most
likely to agree with which of the following statements
about consumers’ perceptions of the relationship
between the frequency with which a product is
advertised and the product’s quality?
(A)

B	Kirmani’s research examines how consumers
respond to the frequency of advertising; the
research does not indicate that consumers
do not notice frequency.

Consumers’ perceptions about the frequency
with which an advertisement appears are their
primary consideration when evaluating an
advertisement’s claims about product quality.

(B)

Because most consumers do not notice the
frequency of advertisement, it has little impact
on most consumers’ expectations regarding
product quality.

(C)

Consumers perceive frequency of advertisement
as a signal about product quality only when the
advertisement is for a product that is newly on
the market.

(D)

The frequency of advertisement is not always
perceived by consumers to indicate that
manufacturers are highly confident about their
products’ quality.

(E)

Consumers who try a new product that has been
frequently advertised are likely to perceive the
advertisement’s frequency as having been an
accurate indicator of the product’s quality.

C	The research does not distinguish between
new and familiar products.
Correct. Excessive advertising may lead
D	
consumers to believe that the manufacturer
lacks confidence in the quality of the
product.
E	Kirmani’s research does not specifically
address new products.
The correct answer is D.
Questions 6–11 refer to the passage on
page 25.
54.

The main purpose of the passage is to
(A)

propose an experiment

(B)

analyze a function

(C)

refute an argument

Inference

(D)

explain a contradiction

The first sentence of the second paragraph
provides the answer to this question: at some level
of spending the manufacturer’s advertising effort
may be perceived as unreasonably high, implying low
manufacturer confidence in product quality. Thus, it
is logical to assume that if a product is advertised
too frequently, consumers may believe that the
manufacturer is spending excessive amounts on
advertising because that manufacturer is not
confident of the product’s quality.

(E)

simulate a process

Main idea

Determining the main purpose comes from
considering the passage as a whole. The first
paragraph begins by noting that the idea of the
brain as an information processor is generally
accepted by neuroscientists. The author then
presents Searle as an enemy of this position and
explains Searle’s belief that human thought is
more than information processing. The second
paragraph questions Searle’s position, and the
third asserts that the brain is an information
processor, refuting Searle’s argument.

63

GMAT® Official Guide 2018

A	The author uses the idea of a mechanical
simulation of a stomach as a metaphor for a
computer’s simulation of thought; this is not
a proposal for an experiment.

A	
Correct. Searle believes that meaning and
content cannot be reduced to algorithms.
B	The author argues for the mechanical
simulation, but offers no evidence that
Searle would agree.

B	The author analyzes Searle’s position, but no
function is analyzed.

C	This statement reflects the author’s position,
but it is the opposite of Searle’s.

Correct. The author explains Searle’s
C	
argument in order to refute it.

D	Searle asserts that only people, not
computers, have the causal powers of the
brain.

D	The author points out a weakness in Searle’s
thinking, but not a contradiction.
E	The simulation of a process is included as
a metaphor, but it is not essential to the
passage.
The correct answer is C.
55.

The correct answer is A.

Which of the following is most consistent with Searle’s
reasoning as presented in the passage?

The author of the passage would be most likely to
agree with which of the following statements about the
simulation of organ functions?

(A)

Meaning and content cannot be reduced to
algorithms.

(A)

(B)

The process of digestion can be simulated
mechanically, but not on a computer.

An artificial device that achieves the functions of
the stomach could be considered a valid model
of the stomach.

(B)

Simulated thoughts and real thoughts are
essentially similar because they are composed
primarily of information.

Computer simulations of the brain are best
used to crack the brain’s codes of meaning and
content.

(C)

A computer can use “causal powers” similar
to those of the human brain when processing
information.

Computer simulations of the brain challenge
ideas that are fundamental to psychology and
neuroscience.

(D)

Computer simulations of the world can achieve
the complexity of the brain’s representations of
the world.

Because the brain and the stomach both act
as processors, they can best be simulated by
mechanical devices.

(E)

The computer’s limitations in simulating digestion
suggest equal limitations in computer‐simulated
thinking.

(C)

(D)

(E)

Evaluation

Searle’s position is stated in the first paragraph:
because computers merely follow algorithms, they
cannot deal with important aspects of human thought
such as meaning and content. Thus, Searle believes
that meaning and content cannot be reduced to
algorithms.

64

E	The passage does not discuss computer
simulations of the world.

56.

Application

To answer this question, think about how
the author would respond to each statement.
Anticipating the author’s response depends on
understanding the author’s point of view. In this
passage, the author is arguing against Searle’s
view of the brain and in favor of the brain as
information processor. The author believes that
the computer can be a model of the brain and
uses the example of the mechanical stomach to
support his position on simulations.

3.6 Diagnostic Test Verbal Answer Explanations

A	
Correct. The first two sentences of the third
paragraph imply that a mechanical device is
a valid model.
B	The author believes a computer can simulate
the brain but does not comment on how
these simulations should be used. There is
no way to predict the author’s reaction to
this statement.
C	The author would reject this statement since
neuroscience and psychology do in fact see
the brain as an information processor.
D	The author agrees that both the brain and
the stomach act as processors; believes that
the computer, a nonmechanical device,
can simulate the brain; and offers a way
that a mechanical device could simulate
the stomach. The author does not suggest
that mechanical devices are the best way to
simulate both their processes.
E	This statement reflects Searle’s viewpoint,
which the author rejects.
The correct answer is A.
57.

It can be inferred that the author of the passage
believes that Searle’s argument is flawed by its
failure to
(A)

distinguish between syntactic and semantic
operations

(B)

explain adequately how people, unlike
computers, are able to understand meaning

(C)

provide concrete examples illustrating its claims
about thinking

(D)

understand how computers use algorithms to
process information

(E)

decipher the code that is transmitted from
neuron to neuron in the brain

Inference

The author’s attitude toward Searle’s argument is
apparent in the first paragraph, which ends with
the author’s summary of what Searle is saying.
Computers understand structures, Searle argues,
but only people understand meaning. How do
people understand meaning? The author notes
that Searle is not able to answer this question
and is able only to assert that people have causal
powers of the brain.
A	The author makes it clear in the first
paragraph that Searle does distinguish
between the two. In Searle’s view computers
are syntactic, interpreting structure or
arrangement, rather than semantic,
understanding meaning.
B	
Correct. The first paragraph ends with the
contrast between people and computers:
People, on the other hand, understand meaning
because they have something Searle obscurely
calls the causal powers of the brain. By calling
Searle’s explanation obscure, the author
implies that Searle has not adequately
clarified how people understand meaning.
C	Nothing in the passage criticizes Searle for
not providing concrete examples. Indeed, in
the second paragraph, the author anticipates
how Searle would react to one concrete
example, the computer simulation of the
stomach.
D	In the first paragraph, the author says
that Searle argues that computers simply
follow algorithms; whether or not Searle
understands how they use algorithms is
irrelevant.
E	Since, as the author suggests in the
first paragraph, Searle does not believe
information could be a code transmitted
from neuron to neuron, he cannot be
expected to decipher that code.
The correct answer is B.

65

GMAT® Official Guide 2018

58.

From the passage, it can be inferred that the author
would agree with Searle on which of the following
points?

59.

Which of the following most accurately represents
Searle’s criticism of the brain‐as‐computer metaphor,
as that criticism is described in the passage?

(A)

Computers operate by following algorithms.

(A)

The metaphor is not experimentally verifiable.

(B)

The human brain can never fully understand its
own functions.

(B)

The metaphor does not take into account the
unique powers of the brain.

(C)

The comparison of the brain to a machine is
overly simplistic.

(C)

(D)

The most accurate models of physical processes
are computer simulations.

The metaphor suggests that a brain’s functions
can be simulated as easily as those of a
stomach.

(D)

The metaphor suggests that a computer can
simulate the workings of the mind by using the
codes of neural transmission.

(E)

The metaphor is unhelpful because both the
brain and the computer process information.

(E)

Human thought and computer‐simulated thought
involve similar processes of representation.

Inference

An inference requires going beyond the material
explicitly stated in the passage to the author’s
ideas that underlie that material. The author and
Searle take opposite points of view on the brain
as information processor. Their area of agreement
is narrow. However, they do both agree that
computers work by following algorithms.
Correct. The first paragraph explains that
A	
Searle dismisses computers because they
simply follow algorithms; while the author
disagrees with Searle on virtually every
other point, no disagreement is voiced here.

Inference

Searle’s criticism of the brain‐as‐computer
metaphor is discussed in the first paragraph.
Computers are merely machines; only people are
endowed with causal powers of the brain that allow
them to understand meaning and content.
A	Searle does not believe in the value of the
metaphor, so its verification is beside the
point.

B	The first paragraph shows this to be Searle’s
position, but not the author’s.

B	
Correct. Searle believes that people have
something computers do not, causal powers
of the brain for understanding important
aspects of human thought.

C	The first paragraph shows this to be Searle’s
position, but not the author’s.

C	Comparing the brain to a computer, the
metaphor does not make this suggestion.

D	The second paragraph explains Searle’s
rejection of this position.

D	In the second paragraph, the author says, but
even if a computer could simulate the workings
of the mind, making it clear that presently it
cannot; this statement does not reflect why
Searle rejects the metaphor.

E	The final paragraph establishes this as the
author’s position, but not Searle’s.
The correct answer is A.

E	This is not the basis of Searle’s objection
since he does not accept the premise that
the brain is an information processor.
The correct answer is B.

66

3.6 Diagnostic Test Verbal Answer Explanations

60.

Questions 12–17 refer to the passage on
page 27.

A	Lines 6–10 emphasize the victories of the
reformers.

The primary purpose of the passage is to

B	The passage discusses the effects, rather than
the origins, of child labor laws.

(A)

explain why women reformers of the Progressive
Era failed to achieve their goals

(B)

discuss the origins of child labor laws in the late
nineteenth and early twentieth centuries

(C)

compare the living conditions of working‐class
and middle‐class women in the Progressive Era

(D)

discuss an oversight on the part of women
reformers of the Progressive Era

(E)

revise a traditional view of the role played by
women reformers in enacting Progressive Era
reforms

Main idea

Understanding the author’s purpose comes only
from reflecting on the passage as a whole. The
beginning of the passage notes the success of
middle‐class women reformers in improving
working conditions for women and children.
The middle discusses the position of working‐
class mothers, who were more concerned with
the economic survival of their families than
with labor reform and consequently tried to
circumvent the laws. The close of the passage
observes that, although middle‐class reformers
were right to point out exploitation of children,
they failed to understand the economic plight of
working‐class families, who needed the income
earned by every possible member. The purpose of
this passage is to show the failure of middle‐class
reformers to understand the economic position of
working‐class families.

C	Living conditions of middle‐class and
working‐class women are not compared.
D	
Correct. As is made clear, especially in
the final sentence of the passage, women
reformers failed to understand the economic
needs of working‐class families.
E	A traditional view is not compared with a
newer, revised view of the reformers.
The correct answer is D.
61.

The view mentioned in line 17 of the passage refers to
which of the following?
(A)

Some working‐class mothers’ resistance to the
enforcement of child labor laws

(B)

Reformers’ belief that child labor and industrial
home work should be abolished

(C)

Reformers’ opinions about how working‐class
families raised their children

(D)

Certain women historians’ observation that there
was a lack of consensus between women of
different classes on the issue of child labor and
industrial home work

(E)

Working‐class families’ fears about the adverse
consequences that child labor laws would have
on their ability to earn an adequate living

Inference

To find what this appearance of view refers to, it
is necessary to look back to the beginning of the
sentence. This view, not shared by working‐class
mothers, refers to the reformers’ conviction that
child labor and industrial home work were equally
inhumane practices that should be outlawed.

67

GMAT® Official Guide 2018

A	
This view must refer back to a point already
stated; resistance to child labor laws is not
discussed until the following sentence.

A	
Correct. The author uses the recent work of
women historians to support the statement
that women of different social classes were
pitted against one another.

B	
Correct. This view refers to the position
of reformers stated earlier in the same
sentence: that child labor and industrial home
work…should be outlawed.

B	The women historians have recently observed;
the verb observed introduces a statement
rather than a question.

C	
This view must refer back to a point already
stated; the reformers’ belief that resistance to
child labor laws was due to poor parenting
is discussed later in the passage.

C	The reference to women historians has to
do with working‐class mothers; it does
not challenge women’s activism and role in
social reform.

D	A number of women historians have said
that working‐class mothers did not always
share the view of middle‐class women
reformers about child labor.

D	The passage supports what the women
historians say about working‐class mothers.
E	The author does not define or present the
traditional view of child labor reform, nor is
any inherent contradiction pointed out.

E	
This view must refer back to a point already
stated; the fears of working‐class families
are examined in the following sentence.
The correct answer is B.
62.

The author of the passage mentions the observations
of women historians (lines 15–17) most probably in
order to

63.

The passage suggests that which of the following was
a reason for the difference of opinion between working‐
class mothers and women reformers on the issue of
child labor?
(A)

(A)

provide support for an assertion made in the
preceding sentence (lines 10–12)

Reformers’ belief that industrial home work was
preferable to child labor outside the home

(B)

(B)

raise a question that is answered in the last
sentence of the passage (lines 27–32)

Reformers’ belief that child labor laws should
pertain to working conditions but not to pay

(C)

(C)

introduce an opinion that challenges a statement
made in the first sentence of the passage

Working‐class mothers’ resentment at reformers’
attempts to interfere with their parenting

(D)

(D)

offer an alternative view to the one attributed in
the passage to working‐class mothers

Working‐class mothers’ belief that child labor was
an inhumane practice

(E)

(E)

point out a contradiction inherent in the
traditional view of child labor reform as it is
presented in the passage

Working‐class families’ need for every
employable member of their families to earn
money

Evaluation

In lines 10–12, the author asserts that child labor
laws pitted women of different classes against one
another. The view of the middle‐class women
reformers is stated, and then, to show that
working‐class mothers did not hold the same
opinion, the author turns to the recent work of
women historians to support this statement.

68

The correct answer is A.

3.6 Diagnostic Test Verbal Answer Explanations

Inference

The question’s use of the word suggests means
that the answer depends on making an inference.
Lines 12–23 examine the different views of
middle‐class reformers and working‐class
mothers on child labor laws. While the reformers
saw child labor as an inhumane practice that
should be outlawed, working class mothers
understood the necessity of pooling the wages of as
many family members as possible and viewed child
labor legislation as a personal economic disaster.
A	Lines 12–14 show that reformers regarded
both kinds of work as equally inhumane
practices that should be outlawed.
B	Pay is not specifically discussed in the
passage.
C	Lines 24–27 indicate that the reformers
believed working‐class resistance to child
labor laws was a sign of poor parenting,
but nothing is said about the working‐class
response to this view.
D	Lines 12–17 say that the reformers held this
position, but working class mothers did not
always share this view.
E	
Correct. Lines 17–23 explain that working‐
class families needed the wages of as many
family members as possible.
The correct answer is E.

64.

The author of the passage asserts which of the
following about women reformers who tried to abolish
child labor?
(A)

They alienated working‐class mothers by
attempting to enlist them in agitating for
progressive causes.

(B)

They underestimated the prevalence of child
labor among the working classes.

(C)

They were correct in their conviction that child
labor was deplorable but shortsighted about the
impact of child labor legislation on working‐class
families.

(D)

They were aggressive in their attempts to
enforce child labor legislation, but were
unable to prevent working‐class families from
circumventing them.

(E)

They were prevented by their nearly total
disenfranchisement from making significant
progress in child labor reform.

Supporting ideas

This question is based on information explicitly
stated in the final sentence of the passage.
Women reformers viewed child labor as a terribly
exploitative practice but they failed to take account of
the economic needs of working‐class families.
A	The passage does not say that reformers
tried to enlist working‐class mothers in
progressive causes.
B	No evidence is offered to support such a
statement.
C	
Correct. The final sentence makes clear
that the reformers recognized child labor
as exploitative but did not understand the
economic needs of working‐class families.
D	The reformers’ activities involved promoting
legislation; there is no evidence in the
passage that the reformers themselves
attempted to enforce these laws.
E.	Lines 6–10 show that the reformers
improved working conditions for
women and children, despite their
disenfranchisement.
The correct answer is C.

69

GMAT® Official Guide 2018

65.

According to the passage, one of the most striking
achievements of white middle‐class women reformers
during the Progressive Era was
(A)

gaining the right to vote in school elections

(B)

mobilizing working‐class women in the fight
against child labor

(C)

uniting women of different classes in grassroots
activism

(D)

improving the economic conditions of working‐
class families

(E)

improving women’s and children’s working
conditions

Supporting ideas

The question’s use of the phrase according to the
passage indicates that the answer can be found
through careful reading of the passage. This
question is based on information explicitly stated
in lines 7–10, which state that white middle‐class
women reformers won a variety of victories, notably
in the improvement of working conditions, especially
for women and children.
A	Lines 6–7 show that women already had the
right to vote in school elections.
B	Lines 20–24 show that working‐class
families tried to circumvent child labor laws.
C	Lines 11–12 say that one product of
grassroots activism, child labor legislation,
pitted women of different classes against one
another.
D	Lines 31–32 say that the reformers failed to
take account of the economic needs of working‐
class families.
E	
Correct. The passage states that reformers
improved the working conditions of women
and children.
The correct answer is E.

70

3.6 Diagnostic Test Verbal Answer Explanations

Critical Reasoning
The following discussion is intended to familiarize you with the most efficient and effective approaches
to critical reasoning questions. The particular questions in this chapter are generally representative of
the kinds of critical reasoning questions you will encounter on the GMAT exam. Remember that it is the
problem solving strategy that is important, not the specific details of a particular question.

66.

Vasquez‐Morrell Assurance specializes in insuring manufacturers. Whenever a policyholder makes a claim, a claims adjuster
determines the amount that Vasquez‐Morrell is obligated to pay. Vasquez‐Morrell is cutting its staff of claims adjusters by
15 percent. To ensure that the company’s ability to handle claims promptly is affected as little as possible by the staff cuts,
consultants recommend that Vasquez‐Morrell lay off those adjusters who now take longest, on average, to complete work
on claims assigned to them.
Which of the following, if true, most seriously calls into question the consultants’ criterion for selecting the staff to be laid off?
(A)

If the time that Vasquez‐Morrell takes to settle claims increases significantly, it could lose business to other insurers.

(B)

Supervisors at Vasquez‐Morrell tend to assign the most complex claims to the most capable adjusters.

(C)

At Vasquez‐Morrell, no insurance payments are made until a claims adjuster has reached a final determination on the claim.

(D)

There are no positions at Vasquez‐Morrell to which staff currently employed as claims adjusters could be reassigned.

(E)

The premiums that Vasquez‐Morrell currently charges are no higher than those charged for similar coverage by
competitors.

Evaluation of a Plan
Situation	An insurance company must reduce its staff of claims adjusters. To ensure continuing promptness

in handling claims, consultants advise the company to lay off those adjusters who take the longest
to complete claims.

Reasoning	
What problem could there be with the criterion? The consultants’ criterion is the time an adjuster takes

to settle a claim. However, some claims are naturally more complicated and require more time. If
it is true that the company now assigns these time‐consuming cases to its most capable adjusters,
then these adjusters would be likely to be the ones who take longest to complete their cases.
Laying off the adjusters who take the longest would thus mean laying off the company’s most
capable staff, which could very well decrease its ability to handle claims promptly.

A

The consultants’ advice makes sense if increased time to handle claims causes the company to lose business.

B

Correct. This statement properly identifies the problem with the consultants’ criterion.

C	This statement merely describes the process of handling a claim; it does not provide any information about
the criterion for layoffs.
D	The consultants make no recommendations for reassigning staff, so indicating that there are no positions
available does not call their advice into question.
E	The consultants do not recommend a change in premiums; noting that they are similar to competitors’
premiums does not undermine the plan that the consultants recommend.
The correct answer is B.

71

GMAT® Official Guide 2018

67.

Prolonged spells of hot, dry weather at the end of the grape‐growing season typically reduce a vineyard’s yield, because the
grapes stay relatively small. In years with such weather, wine producers can make only a relatively small quantity of wine
from a given area of vineyards. Nonetheless, in regions where wine producers generally grow their own grapes, analysts
typically expect a long, hot, dry spell late in the growing season to result in increased revenues for local wine producers.
Which of the following, if true, does most to justify the analysts’ expectation?
(A)

The lower a vineyard’s yield, the less labor is required to harvest the grapes.

(B)

Long, hot, dry spells at the beginning of the grape‐growing season are rare, but they can have a devastating effect on
a vineyard’s yield.

(C)

Grapes grown for wine production are typically made into wine at or near the vineyard in which they were grown.

(D)

When hot, dry spells are followed by heavy rains, the rains frequently destroy grape crops.

(E)

Grapes that have matured in hot, dry weather make significantly better wine than ordinary grapes.

Argument Construction
Situation	Hot, dry weather at the end of the grape‐growing season reduces yield, so winemakers can

only produce a small quantity of wine. However, analysts expect that this weather will increase
winemakers’ revenues.

Reasoning	
What additional piece of information explains the analysts’ expectations? The same conditions that

lead to low quantity also lead to something that increases revenues. What could this be? If
these weather conditions lead to higher‐quality wine that will sell for higher prices, the analysts’
expectations for increased revenues are justified.

A	Lower labor costs mean less expenditure for the winemakers; this does not explain how revenues would
increase.
B

This statement about low yields does not explain an increase in revenues.

C	The proximity of production to the vineyard is irrelevant to the question of how hot, dry weather can be
responsible for decreased yield and increased revenues.
D	This statement gives another example of weather’s effect on grape crops, but it does not explain how
revenues are increased.
Correct. This statement properly provides the explanation that the weather conditions will lead to better
E	
wines. With better wines typically commanding higher prices, the winemakers will gain the increased
revenues that the analysts anticipate.
The correct answer is E.

72

3.6 Diagnostic Test Verbal Answer Explanations

68.

In the past, most children who went sledding in the winter snow in Verland used wooden sleds with runners and steering
bars. Ten years ago, smooth plastic sleds became popular; they go faster than wooden sleds but are harder to steer and
slow. The concern that plastic sleds are more dangerous is clearly borne out by the fact that the number of children injured
while sledding was much higher last winter than it was 10 years ago.
Which of the following, if true in Verland, most seriously undermines the force of the evidence cited?
(A)

A few children still use traditional wooden sleds.

(B)

Very few children wear any kind of protective gear, such as helmets, while sledding.

(C)

Plastic sleds can be used in a much wider variety of snow conditions than wooden sleds can.

(D)

Most sledding injuries occur when a sled collides with a tree, a rock, or another sled.

(E)

Because the traditional wooden sleds can carry more than one rider, an accident involving a wooden sled can result in
several children being injured.

Argument Evaluation
Situation	Ten years ago, wooden sleds began to be replaced by plastic sleds that go faster but are harder

to control. Plastic sleds are more dangerous than wooden sleds because more children suffered
injuries last year than they did 10 years ago.

Reasoning	
What weakens this argument? This argument depends on a comparison of two kinds of sleds. Any

evidence that would either strengthen or weaken the argument must indicate a comparison.
Evidence that applies only to one kind of sled or to both kinds of sleds equally cannot weaken this
argument. Consider the implications of the evidence presented in the answer choices. If plastic
sleds can be used in a wider variety of conditions than wooden sleds can, then plastic sleds can be
used more frequently. It is possible that more frequent use, rather than the sleds themselves, has
led to more accidents.

A

The limited use of some wooden sleds does not weaken the argument.

B

The absence of protective gear would affect accidents with both kinds of sleds.

C	
Correct. This statement weakens the argument by providing an alternate explanation for the increased
accidents.
D

This statement is true of accidents with both kinds of sleds.

E	This explains why wooden sleds may be dangerous but does not weaken the argument that plastic sleds are
even more dangerous.
The correct answer is C.

73

GMAT® Official Guide 2018

69.

Metal rings recently excavated from seventh‐century settlements in the western part of Mexico were made using the
same metallurgical techniques as those used by Ecuadorian artisans before and during that period. These techniques are
sufficiently complex to make their independent development in both areas unlikely. Since the people of these two areas
were in cultural contact, archaeologists hypothesize that the metallurgical techniques used to make the rings found in
Mexico were learned by Mexican artisans from Ecuadorian counterparts.
Which of the following would it be most useful to establish in order to evaluate the archaeologists’ hypothesis?
(A)

Whether metal objects were traded from Ecuador to western Mexico during the seventh century

(B)

Whether travel between western Mexico and Ecuador in the seventh century would have been primarily by land or by
sea

(C)

Whether artisans from western Mexico could have learned complex metallurgical techniques from their Ecuadorian
counterparts without actually leaving western Mexico

(D)

Whether metal tools were used in the seventh‐century settlements in western Mexico

(E)

Whether any of the techniques used in the manufacture of the metal rings found in western Mexico are still practiced
among artisans in Ecuador today

Argument Evaluation
Situation	Metal rings excavated from seventh‐century settlements in western Mexico were made with the

same complex techniques used in Ecuador before and during a period when the two cultures
were known to be in contact. Mexican artisans are thought to have learned the techniques from
Ecuadorian artisans.

Reasoning	
What point could best be applied in evaluating this hypothesis? Consider what specific information

would help to assess the archaeologists’ theory. It is given that the two areas had some cultural
contact. If it were determined that metal objects were traded from one culture to the other, it
could be possible that the metalworking techniques were passed along as well. Such evidence
would be relevant to the hypothesis that Mexican artisans saw the work of their Ecuadorian
counterparts and, from this exchange, learned the techniques to make the metal rings.

A	
Correct. This statement properly identifies information that would be useful in the evaluation of the
archaeologists’ hypothesis.
B

The means of travel is irrelevant to the hypothesis about the source of the techniques.

C	The hypothesis is not about where Mexican artisans learned the techniques, but whether they learned
them from the Ecuadorians.
D	The existence of metal tools provides no helpful information in establishing whether the Ecuadorians were
the source of the metallurgical techniques.
E

The comparison to the present day is irrelevant to the hypothesis.

The correct answer is A.

74

3.6 Diagnostic Test Verbal Answer Explanations

70.

Following several years of declining advertising sales, the Greenville Times reorganized its advertising sales force. Before
reorganization, the sales force was organized geographically, with some sales representatives concentrating on city‐center
businesses and others concentrating on different outlying regions. The reorganization attempted to increase the sales
representatives’ knowledge of clients’ businesses by having each sales representative deal with only one type of industry or
of retailing. After the reorganization, revenue from advertising sales increased.
In assessing whether the improvement in advertising sales can properly be attributed to the reorganization, it would be most
helpful to find out which of the following?
(A)

What proportion of the total revenue of the Greenville Times is generated by advertising sales?

(B)

Has the circulation of the Greenville Times increased substantially in the last two years?

(C)

Among all the types of industry and retailing that use the Greenville Times as an advertising vehicle, which type
accounts for the largest proportion of the newspaper’s advertising sales?

(D)

Do any clients of the sales representatives of the Greenville Times have a standing order with the Times for a fixed
amount of advertising per month?

(E)

Among the advertisers in the Greenville Times, are there more types of retail business or more types of industrial
business?

Evaluation of a Plan
Situation	In the face of declining advertising sales, a newspaper reorganizes its sales force so that sales

representatives have a better understanding of businesses. Revenue from advertising sales increased
after the reorganization.

Reasoning	
What additional evidence would help determine the source of the increased revenue? In order to

attribute the increased revenue to the reorganization of the sales force, other possible causes
must be eliminated. Newspaper advertising rates are linked to circulation; when circulation
increases, higher rates can be charged and revenues will increase. An alternate explanation might
be a significant rise in circulation, so it would be particularly helpful to know if circulation had
increased.

A	The question concerns only increased revenue from advertising sales; the proportion of advertising revenue
to total revenue is outside the scope of the question.
B	
Correct. This statement provides another possible explanation for increased revenue of advertising sales,
and so the answer to this question would help to clarify the reason for the increased revenue.
C	Knowing how the advertising sales break down by type of business might be useful for other purposes, but
it does not help to show the cause of the increase.
D	A fixed amount of advertising would not explain increased revenue, so the answer to this question would
be irrelevant.
E	Distinguishing between the types of businesses will not contribute to determining whether the
reorganization was responsible for the increased revenue.
The correct answer is B.

75

GMAT® Official Guide 2018

71.

Motorists in a certain country frequently complain that traffic congestion is much worse now than it was 20 years ago.
No real measure of how much traffic congestion there was 20 years ago exists, but the motorists’ complaints are almost
certainly unwarranted. The country’s highway capacity has tripled in the last twenty years, thanks to a vigorous highway
construction program, whereas the number of automobiles registered in the country has increased by only 75 percent.
Which of the following, if true, most seriously weakens the argument?
(A)

Most automobile travel is local, and the networks of roads and streets in the country’s settled areas have changed
little over the last twenty years.

(B)

Gasoline prices are high, and miles traveled per car per year have not changed much over the last 20 years.

(C)

The country’s urban centers have well‐developed public transit systems that carry most of the people who commute
into those centers.

(D)

The average age of automobiles registered in the country is lower now than it was 20 years ago.

(E)

Radio stations have long been broadcasting regular traffic reports that inform motorists about traffic congestion.

Argument Evaluation
Situation 	Motorists complain that traffic congestion in their country is much worse than it was twenty years

ago. But these complaints have no basis since the highway capacity in this country has tripled in
the same period, whereas the number of cars registered has risen by only 75 percent.

Reasoning 	Which point most undermines the argument that the complaints are unwarranted? Consider that the

response to the generalized complaints about congestion discusses only the topic of highway
capacity. What if the congestion that motorists are complaining about is not on highways but on
local roads? Discovering that travel tends to be local in this country and that the local roads have
not been improved in the last twenty years would seriously weaken the argument.

Correct. This statement properly identifies a weakness in the argument: the response to the broad
A	
complaint addresses a different subject, highway capacity, not the issue of traffic congestion encountered by
most motorists.
B	If high gas prices actually prevented motorists from driving, and if motorists’ driving habits were the same
as they were twenty years ago, then these points should strengthen the argument that there is no basis for
their complaints.
C	The number of commuters who use public transit does not affect the argument that the motorists’
complaints have no basis.
D

The age of registered cars is irrelevant to the argument.

E	The radio broadcasts attest to the existence of traffic, but not to its increase, so they do not affect the
argument.
The correct answer is A.

76

3.6 Diagnostic Test Verbal Answer Explanations

72.

The percentage of households with an annual income of more than $40,000 is higher in Merton County than in any other
county. However, the percentage of households with an annual income of $60,000 or more is higher in Sommer County.
If the statements above are true, which of the following must also be true?
(A)

The percentage of households with an annual income of $80,000 is higher in Sommer County than in Merton County.

(B)

Merton County has the second highest percentage of households with an annual income of $60,000 or more.

(C)

Some households in Merton County have an annual income between $40,000 and $60,000.

(D)

The number of households with an annual income of more than $40,000 is greater in Merton County than in Sommer
County.

(E)

Average annual household income is higher in Sommer County than in Merton County.

Argument Construction
Situation	The percentage of households with annual incomes of more than $40,000 is higher in Merton

County than in any other county; the percentage of households with annual incomes of $60,000
or more is higher in Sommer County.

Reasoning	
On the basis of this information, what point must be true? The given information makes clear that

Merton County has some households that exceed $40,000 in annual income. Sommer County has
a higher percentage of households with annual incomes at or above $60,000. A higher percentage
of the Merton County households must in turn have annual incomes of $60,000 or less. Thus, the
annual income of some households in Merton County is between $40,000 and $60,000.

A	Since it is possible that there are no households with an annual income of $80,000 in Sommer County,
this statement does not follow from the situation.
B	It is not possible to make this determination on the basis of the available evidence; Merton County may
have no households at all with an income of more than $60,000.
C	
Correct. This statement properly identifies a conclusion that can be drawn from the given information: in
order for the percentage of $40,000‐plus incomes to be higher in Merton county than any other county
while Sommer has the highest percentage of $60,000‐plus incomes, there must be some households in
Merton County that bring in between $40,000 and $60,000 annually.
D	On the basis of information about the percentages of households, it is not possible to arrive at this
conclusion about the number of households.
E	From the given information, it is not possible to determine where the average income is greater. It is
entirely possible that the number of $60,000‐plus incomes in Sommer County is quite small and that the
number of $40,000‐plus incomes in Merton County is substantial.
The correct answer is C.

77

GMAT® Official Guide 2018

73.

Tiger beetles are such fast runners that they can capture virtually any nonflying insect. However, when running toward
an insect, a tiger beetle will intermittently stop and then, a moment later, resume its attack. Perhaps the beetles cannot
maintain their pace and must pause for a moment’s rest; but an alternative hypothesis is that while running, tiger beetles are
unable to adequately process the resulting rapidly changing visual information and so quickly go blind and stop.
Which of the following, if discovered in experiments using artificially moved prey insects, would support one of the two
hypotheses and undermine the other?
(A)

When a prey insect is moved directly toward a beetle that has been chasing it, the beetle immediately stops and runs
away without its usual intermittent stopping.

(B)

In pursuing a swerving insect, a beetle alters its course while running and its pauses become more frequent as the
chase progresses.

(C)

In pursuing a moving insect, a beetle usually responds immediately to changes in the insect’s direction, and it pauses
equally frequently whether the chase is up or down an incline.

(D)

If, when a beetle pauses, it has not gained on the insect it is pursuing, the beetle generally ends its pursuit.

(E)

The faster a beetle pursues an insect fleeing directly away from it, the more frequently the beetle stops.

Argument Evaluation
Situation 	Two hypotheses are offered to explain the sudden stop that tiger beetles make while pursuing

their prey: (1) they cannot maintain the rapid pace and must rest, and (2) they run too quickly to
process visual information and so temporarily go blind.

Reasoning 	What point would strengthen one of the two hypotheses and weaken the other? Consider the

information provided in each answer choice, remembering that information that supports one
hypothesis must necessarily detract from the other. Any information that is not about pursuit or
that affects the two hypotheses equally may be dismissed from consideration. If the frequency
of stopping increases when the beetle follows a swerving insect and must constantly change its
course, then the second hypothesis is strengthened; the beetle’s pauses increase as the variety of
visual information that it needs to deal with increases.

A	The hypotheses concern ongoing pursuit; since this information is not about the beetle’s continuing pursuit
of prey, it neither strengthens nor weakens either hypothesis.
B	
Correct. This statement provides information that strengthens the second hypothesis: the swerving pursuit
and the resulting continual course adjustments appear to be forcing the beetle to stop with increasing
frequency to sort out the erratic visual information.
C	In this experiment, since neither vision nor tiredness appears to be problematic, the beetle could be
stopping for either reason; this information neither strengthens nor weakens either hypothesis.
D

This information is irrelevant since both the hypotheses are about mid‐pursuit behaviors.

E	The correlation of frequency of stops with speed affects both hypotheses equally; the pauses could be
equally due to an inability to maintain the pace or due to a need to process the visual information.
The correct answer is B.

78

3.6 Diagnostic Test Verbal Answer Explanations

74.

Guillemots are birds of Arctic regions. They feed on fish that gather beneath thin sheets of floating ice, and they nest on
nearby land. Guillemots need 80 consecutive snow‐free days in a year to raise their chicks, so until average temperatures
in the Arctic began to rise recently, the guillemots’ range was limited to the southernmost Arctic coast. Therefore, if the
warming continues, the guillemots’ range will probably be enlarged by being extended northward along the coast.
Which of the following, if true, most seriously weakens the argument?
(A)

Even if the warming trend continues, there will still be years in which guillemot chicks are killed by an unusually early
snow.

(B)

If the Arctic warming continues, guillemots’ current predators are likely to succeed in extending their own range
farther north.

(C)

Guillemots nest in coastal areas, where temperatures are generally higher than in inland areas.

(D)

If the Arctic warming continues, much of the thin ice in the southern Arctic will disappear.

(E)

The fish that guillemots eat are currently preyed on by a wider variety of predators in the southernmost Arctic regions
than they are farther north.

Argument Evaluation
Situation 	In the southern Arctic, guillemots find their prey beneath thin sheets of ice, nest nearby, and

require 80 snow‐free days to raise their young. A warming trend means that their range may be
enlarged by extending northward along the coast.

Reasoning 	Which point weakens the argument about the enlargement of the guillemots’ range? How could the

birds move northward and simultaneously not enlarge their range? Consider the assumption
implied by the idea of enlargement. If the guillemots lost their southern habitat, then their
northward move would be a displacement rather than an enlargement. If their source of food was
no longer available to them in the southern Arctic, then they would abandon that area as part of
their range.

A	An exceptional year is not an argument against an enlarged range because an unusually early snow could
happen in the southern Arctic as well.
B	If their current predators also migrate northward, then the guillemots’ situation has not changed, so this is
not an argument against their enlarged range.
C

The argument suggests that they will move not inland, but northward along the coast.

D	
Correct. This statement properly identifies a factor that weakens the argument: the guillemots’ move
northward would not enlarge their range if they lost their food source, fish found under thin ice, in the
southern Arctic.
E	The possibility that they may find prey more easily in the north does not mean that they would abandon
the southern Arctic, and so this point does not weaken the argument.
The correct answer is D.

79

GMAT® Official Guide 2018

75.

Some batches of polio vaccine used around 1960 were contaminated with SV40, a virus that in monkeys causes various
cancers. Some researchers now claim that this contamination caused some cases of a certain cancer in humans,
mesothelioma. This claim is not undercut by the fact that a very careful survey made in the 1960s of people who had
received the contaminated vaccine found no elevated incidence of any cancer, since __________.
(A)

most cases of mesothelioma are caused by exposure to asbestos

(B)

in some countries, there was no contamination of the vaccine

(C)

SV40 is widely used in laboratories to produce cancers in animals

(D)

mesotheliomas take several decades to develop

(E)

mesothelioma was somewhat less common in 1960 than it is now

Argument Construction
Situation 	Researchers claim that contaminated polio vaccine administered in 1960 caused some cases of

mesothelioma, a type of cancer. Their claim is not undermined by the results of a 1960s survey
showing that those who received the contaminated vaccine had no elevated incidence of cancer.

Reasoning 	Why did the survey results not challenge the researchers’ claim? The survey did not reveal a higher

incidence of mesothelioma. This question then requires completing a sentence that establishes
cause. What could be the reason that the people surveyed in the 1960s showed no signs of the
disease? If the disease takes decades to develop, then those people surveyed would not yet have
shown any signs of it; less than a decade had passed between their exposure to the vaccine and the
survey.

A	The contaminated vaccine is said to have caused some cases, not most; the question remains why the survey
results pose no obstacle to the researchers’ claim.
B

The claim is only about contaminated vaccine, not uncontaminated vaccine.

C	That the virus can cause cancers in laboratory animals had already been provided as a given; this additional
information is irrelevant to the survey of people who received contaminated vaccine.
Correct. This statement properly identifies the reason that the survey does not call into question the
D	
researchers’ claim: the people surveyed in the 1960s showed no signs of disease because the cancer takes
decades to develop.
E	The frequency of mesothelioma in the general population is not related to the claim that contaminated
vaccine caused the disease in a specific population.
The correct answer is D.

80

3.6 Diagnostic Test Verbal Answer Explanations

76.

Gortland has long been narrowly self‐sufficient in both grain and meat. However, as per capita income in Gortland has risen
toward the world average, per capita consumption of meat has also risen toward the world average, and it takes several
pounds of grain to produce one pound of meat. Therefore, since per capita income continues to rise, whereas domestic
grain production will not increase, Gortland will soon have to import either grain or meat or both.
Which of the following is an assumption on which the argument depends?
(A)

The total acreage devoted to grain production in Gortland will soon decrease.

(B)

Importing either grain or meat will not result in a significantly higher percentage of Gortlanders’ incomes being spent
on food than is currently the case.

(C)

The per capita consumption of meat in Gortland is increasing at roughly the same rate across all income levels.

(D)

The per capita income of meat producers in Gortland is rising faster than the per capita income of grain producers.

(E)

People in Gortland who increase their consumption of meat will not radically decrease their consumption of grain.

Argument Construction
Situation 	A country previously self‐sufficient in grain and meat will soon have to import one or the other or

both. Consumption of meat has risen as per capita income has risen, and it takes several pounds of
grain to produce one pound of meat.

Reasoning 	What conditions must be true for the conclusion to be true? Meat consumption is rising. What about

grain consumption? A sharp reduction in the amount of grain consumed could compensate for
increased meat consumption, making the conclusion false. If people did radically decrease their
grain consumption, it might not be necessary to import grain or meat or both. Since the argument
concludes that the imports are necessary, it assumes grain consumption will not plunge.

A	The argument makes no assumptions about the acreage devoted to grain; it assumes only that the demand
for grain will rise.
B	The argument does not discuss the percentage of their income that Gortlanders spend on food, so an
assumption about this topic is not needed.
C

The argument involves only meat consumption in general, not its distribution by income level.

D	Since the argument does not refer to the incomes of meat producers and grain producers, it cannot depend
on an assumption about them.
E	
Correct. This statement properly identifies the assumption that there will be no great decrease in grain
consumption.
The correct answer is E.

81

GMAT® Official Guide 2018

77.

The Hazelton coal‐processing plant is a major employer in the Hazelton area, but national environmental regulations will
force it to close if it continues to use old, polluting processing methods. However, to update the plant to use newer, cleaner
methods would be so expensive that the plant will close unless it receives the tax break it has requested. In order to prevent
a major increase in local unemployment, the Hazelton government is considering granting the plant’s request.
Which of the following would be most important for the Hazelton government to determine before deciding whether to grant
the plant’s request?
(A)

Whether the company that owns the plant would open a new plant in another area if the present plant were closed

(B)

Whether the plant would employ far fewer workers when updated than it does now

(C)

Whether the level of pollutants presently being emitted by the plant is high enough to constitute a health hazard for
local residents

(D)

Whether the majority of the coal processed by the plant is sold outside the Hazelton area

(E)

Whether the plant would be able to process more coal when updated than it does now

Evaluation of a Plan
Situation 	Because of the expenses of mandatory updating, a plant that is a major employer in the local area

will close unless it receives the tax break it has requested from the local government.

Reasoning 	What point is most critical to the evaluation of the request? Consider the information provided

in the answer choices. The plant is important to the local government primarily because it is a
major employer of local residents. What if updating the plant significantly reduced the number
of employees needed? It is crucial for the local government to determine whether the plant will
continue to employ the same number of people once it has updated.

A

The local government is concerned only with the local area, so a new site outside that area is irrelevant.

B	
Correct. This statement properly identifies a factor that is critical to the plant’s argument and the local
government’s decision.
C	Updating is mandatory under national environmental regulations, whether the local residents are affected
by the plant’s pollutants or not.
D

At issue is the plant’s role as a major employer; where its product is sold is irrelevant.

E	The amount of coal processed by the updated plant is irrelevant to the critical issue of the number of
people employed to process that coal.
The correct answer is B.

82

3.6 Diagnostic Test Verbal Answer Explanations

78.

A physically active lifestyle has been shown to help increase longevity. In the Wistar region of Bellaria, the average age
at death is considerably higher than in any other part of the country. Wistar is the only mountainous part of Bellaria. A
mountainous terrain makes even such basic activities as walking relatively strenuous; it essentially imposes a physically
active lifestyle on people. Clearly, this circumstance explains the long lives of people in Wistar.
Which of the following, if true, most seriously weakens the argument?
(A)

In Bellaria all medical expenses are paid by the government, so that personal income does not affect the quality of
health care a person receives.

(B)

The Wistar region is one of Bellaria’s least populated regions.

(C)

Many people who live in the Wistar region have moved there in middle age or upon retirement.

(D)

The many opportunities for hiking, skiing, and other outdoor activities that Wistar’s mountains offer make it a favorite
destination for vacationing Bellarians.

(E)

Per capita spending on recreational activities is no higher in Wistar than it is in other regions of Bellaria.

Argument Evaluation
Situation 	People in one region of a country live longer than people in other areas. The higher average

age at time of death is attributed to the healthy lifestyle of the people in this region, where the
mountainous terrain demands a physically active life.

Reasoning 	What point weakens the argument? Consider what assumption underlies the argument that the

physically active lifestyle required of living in Wistar is responsible for its residents’ relative
longevity. The mountainous environment necessitates lifelong levels of rigorous physical activity
that build a more robust population. What if a significant portion of the population has not been
conditioned since childhood to the demands of the terrain? It is assumed here that the healthy
lifestyle imposed by the terrain has shaped residents from birth and accounts for their longer life
span. If many residents only moved there later in life, the argument is weakened.

A	The argument is not about the quality of health care throughout the country, but the length of the
residents’ lives in a particular region.
B

The rate of population density does not affect the argument.

C

Correct. This statement properly identifies a point that weakens the argument.

D

The area’s popularity as a vacation destination does not affect the longevity of the local residents.

E	The argument establishes that merely living in the region is strenuous; the spending on recreational
activities is irrelevant.
The correct answer is C.

83

GMAT® Official Guide 2018

79.

Cheever College offers several online courses via remote computer connection, in addition to traditional classroom‐based
courses. A study of student performance at Cheever found that, overall, the average student grade for online courses
matched that for classroom‐based courses. In this calculation of the average grade, course withdrawals were weighted as
equivalent to a course failure, and the rate of withdrawal was much lower for students enrolled in classroom‐based courses
than for students enrolled in online courses.
If the statements above are true, which of the following must also be true of Cheever College?
(A)

Among students who did not withdraw, students enrolled in online courses got higher grades, on average, than
students enrolled in classroom‐based courses.

(B)

The number of students enrolled per course at the start of the school term is much higher, on average, for the online
courses than for the classroom‐based courses.

(C)

There are no students who take both an online and a classroom‐based course in the same school term.

(D)

Among Cheever College students with the best grades, a significant majority take online, rather than classroom‐
based, courses.

(E)

Courses offered online tend to deal with subject matter that is less challenging than that of classroom‐based courses.

Argument Construction
Situation 	A comparison of online and classroom courses showed similar average grades. In determining

average grades, a course withdrawal was weighted as a course failure. The rate of withdrawal was
higher from online than from classroom courses.

Reasoning 	What conclusion about the courses can be derived from this comparison? Consider the ramifications

of the methodology used to calculate the grade averages for the two types of courses. Because of
course withdrawals, the online courses experienced a higher rate of failure, but the average grade
for these courses still matched the average grade for classroom courses. From this it is logical to
conclude that, for the two averages to match, the students who remained in the online courses
must have had higher initial average grades than those in classroom courses.

A	
Correct. This statement properly identifies the logical conclusion that the higher percentage of
withdrawals from online classes requires higher grades, on average, to compensate for the higher rate of
failure.
B

A number of students cannot be derived from a discussion of average grades and rates of withdrawal.

C

This conclusion cannot be determined on the basis of the information provided.

D	The information is about average grades; the argument does not provide any basis for a conclusion about
best grades.
E

It is impossible to determine the difficulty of subject matter from this information.

The correct answer is A.

84

3.6 Diagnostic Test Verbal Answer Explanations

80.

For years the beautiful Renaissance buildings in Palitito have been damaged by exhaust from the many tour buses that
come to the city. There has been little parking space, so most buses have idled at the curb during each stop on their tour,
and idling produces as much exhaust as driving. The city has now provided parking that accommodates a third of the tour
buses, so damage to Palitito’s buildings from the buses’ exhaust will diminish significantly.
Which of the following, if true, most strongly supports the argument?
(A)

The exhaust from Palitito’s few automobiles is not a significant threat to Palitito’s buildings.

(B)

Palitito’s Renaissance buildings are not threatened by pollution other than engine exhaust.

(C)

Tour buses typically spend less than one‐quarter of the time they are in Palitito transporting passengers from one site
to another.

(D)

More tourists come to Palitito by tour bus than by any other single means of transportation.

(E)

Some of the tour buses that are unable to find parking drive around Palitito while their passengers are visiting a site.

Argument Evaluation
Situation 	Tour buses have damaged Renaissance buildings with their exhaust fumes because lack of

parking has kept the buses idling at curbs. Providing new parking for a third of the buses should
significantly reduce the damage caused by the exhaust.

Reasoning 	What point strengthens the argument? The argument for reduced damage relies on the reduction of

the vehicles’ exhaust fumes. Any additional evidence regarding the extent to which the vehicular
emissions are likely to be reduced also supports the argument for the benefits of the new parking
spaces. Learning that tour buses spend not just a few minutes but most of their time idling at
the curb strengthens the argument. The new parking spaces will allow a third of the tour buses to
spend 75 percent of their time with their engines off, causing no damage at all.

A

If automobile exhaust is not a threat, the argument is not affected.

B	This statement does not address the question of whether the new parking will reduce the damage caused
by engine exhaust from the buses.
C	
Correct. This statement properly cites a factor that supports the argument: since most of the buses’ time
has been spent producing damaging exhaust, the new parking should reduce the damage significantly.
D	This statement about tourists’ chosen means of transportation is irrelevant to the issue of what the buses do
while in the city.
E	It is given that the new parking will only provide space for a third of the buses, and thus some buses will
continue to idle and some to drive around, continuing to contribute equally to the building damage. This
statement does not strengthen the argument.
The correct answer is C.

85

GMAT® Official Guide 2018

81.

During the 1980s and 1990s, the annual number of people who visited the Sordellian Mountains increased continually, and
many new ski resorts were built. Over the same period, however, the number of visitors to ski resorts who were caught in
avalanches decreased, even though there was no reduction in the annual number of avalanches in the Sordellian Mountains.
Which of the following, if true in the Sordellian Mountains during the 1980s and 1990s, most helps to explain the decrease?
(A)

Avalanches were most likely to happen when a large new snowfall covered an older layer of snow.

(B)

Avalanches destroyed at least some buildings in the Sordellian Mountains in every year.

(C)

People planning new ski slopes and other resort facilities used increasingly accurate information about which
locations are likely to be in the path of avalanches.

(D)

The average length of stay for people visiting the Sordellian Mountains increased slightly.

(E)

Construction of new ski resorts often led to the clearing of wooded areas that had helped prevent avalanches.

Argument Construction
Situation 	Over a certain period, new ski resorts accommodated an increasing number of visitors at the same

time that fewer visitors were caught in avalanches. Yet there were no fewer avalanches than usual
during this period.

Reasoning 	What explains the apparent contradiction of increased visitors but fewer visitors caught in avalanches?

More resort visitors would imply more avalanche‐related accidents, but the average has shifted so
that fewer visitors are being caught in the avalanches. It must be that fewer visitors are exposed
to this danger; consider the answer choices to identify a logical reason for this improvement in
their exposure. If the likely paths of avalanches had become better understood, that information
would have been applied to identify safer locations for new ski slopes and ski resorts. The facilities
would thus have been built well out of the way of avalanches, resulting in fewer visitors trapped in
avalanches.

A

This likelihood would remain true from year to year; it does not explain the decrease.

B

This point does not explain why fewer visitors were caught in these avalanches.

C

Correct. This statement properly identifies a factor that explains the decreased number of accidents.

D

The greater length of stay would seem to expose visitors to greater danger.

E	This information points to an expected increase, rather than decrease, in visitors who might be caught by
avalanches.
The correct answer is C.

86

3.6 Diagnostic Test Verbal Answer Explanations

82.

A year ago, Dietz Foods launched a yearlong advertising campaign for its canned tuna. Last year Dietz sold 12 million cans
of tuna compared to the 10 million sold during the previous year, an increase directly attributable to new customers brought
in by the campaign. Profits from the additional sales, however, were substantially less than the cost of the advertising
campaign. Clearly, therefore, the campaign did nothing to further Dietz’s economic interests.
Which of the following, if true, most seriously weakens the argument?
(A)

Sales of canned tuna account for a relatively small percentage of Dietz Foods’ profits.

(B)

Most of the people who bought Dietz’s canned tuna for the first time as a result of the campaign were already loyal
customers of other Dietz products.

(C)

A less expensive advertising campaign would have brought in significantly fewer new customers for Dietz’s canned
tuna than did the campaign Dietz Foods launched last year.

(D)

Dietz made money on sales of canned tuna last year.

(E)

In each of the past five years, there was a steep, industry‐wide decline in sales of canned tuna.

Argument Evaluation
Situation 	An advertising campaign was responsible for increased sales of canned tuna. Since the profits from

the increased sales were less than the costs of the campaign, the campaign did not contribute to
the company’s economic interests.

Reasoning 	Which point weakens the argument? Consider the basis of the argument: if profits are lower

than costs, the campaign made no contribution to the company’s financial well‐being. In what
case might this be untrue? What if the advertising campaign reversed an industry‐wide trend
of declining sales? If Dietz experienced increasing sales, while other companies experienced
decreased sales, then the campaign did contribute to the economic interests of the company, and
the argument is considerably weakened.

A	The issue is not the percentage of profits that canned tuna contributes, but the success of the advertising
campaign.
B	If the customers bought the tuna because of the campaign, it is irrelevant to the argument that they also
bought other Dietz products.
C

This information neither strengthens nor weakens the argument.

D	The argument is not about profits only, but about whether the advertising campaign contributed to the
economic interests of the company.
E	
Correct. This statement properly identifies a factor that weakens the argument: the campaign secured the
benefits of increased sales at a time when the entire industry was experiencing a decline in sales.
The correct answer is E.

87

GMAT® Official Guide 2018

Sentence Correction
The following discussion is intended to familiarize you with the most efficient and effective approaches
to sentence correction questions. The particular questions in this chapter are generally representative of
the kinds of sentence correction questions you will encounter on the GMAT exam. Remember that it is
the problem solving strategy that is important, not the specific details of a particular question.

83.

Unlike the buildings in Mesopotamian cities, which
were arranged haphazardly, the same basic plan was
followed for all cities of the Indus Valley: with houses
laid out on a north‐south, east‐west grid, and houses
and walls were built of standard‐size bricks.
(A)

the buildings in Mesopotamian cities, which were
arranged haphazardly, the same basic plan was
followed for all cities of the Indus Valley: with
houses

B	Illogically contrasts the buildings in
Mesopotamian cities with the same basic plan;
does not clarify what which were haphazard
in arrangement modifies.

(B)

the buildings in Mesopotamian cities, which were
haphazard in arrangement, the same basic plan
was used in all cities of the Indus Valley: houses
were

(C)

the arrangement of buildings in Mesopotamian
cities, which were haphazard, the cities of the
Indus Valley all followed the same basic plan:
houses

C	Illogically contrasts the arrangement of
buildings with the cities of the Indus Valley;
not clear whether which were haphazard
modifies buildings or cities; houses not
followed by a verb.

(D)

Mesopotamian cities, in which buildings were
arranged haphazardly, the cities of the Indus Valley
all followed the same basic plan: houses were

(E)

Mesopotamian cities, which had buildings that
were arranged haphazardly, the same basic plan
was used for all cities in the Indus Valley: houses
that were

Comparison-contrast; Modifying clause

The contrast introduced by unlike must be
logical and clear. Contrasting the buildings in
Mesopotamian cities with the same basic plan does
not make sense; Mesopotamian cities should be
contrasted with the cities of the Indus Valley. Also,
it needs to be clear that it was the buildings in the
cities that were arranged haphazardly rather than
the cities. The second half of the sentence needs
houses were laid out to be parallel in structure to
and houses and walls were built.

88

A	Illogically contrasts the buildings in
Mesopotamian cities with the same basic plan;
not clear whether which were arranged
haphazardly modifies cities or buildings; with
houses lacks parallelism and is confusing.

D	
Correct. In this sentence, Mesopotamian
cities are properly contrasted with the cities
of the Indus Valley; in which buildings were
arranged haphazardly expresses the idea
clearly; and houses is followed by were as
required.
E	Illogically contrasts Mesopotamian cities with
the same basic plan; houses that were lacks
parallelism and is confusing.
The correct answer is D.

3.6 Diagnostic Test Verbal Answer Explanations

84.

A	
It has no referent; not be spent is awkward;
on having to extinguish is wordy.

New data from United States Forest Service ecologists
show that for every dollar spent on controlled small‐
scale burning, forest thinning, and the training of
fire‐management personnel, it saves seven dollars that
would not be spent on having to extinguish big fires.
(A)

(B)

(C)

B	
Correct. This sentence properly uses seven
dollars as the subject of the clause to balance
every dollar in the introductory phrase; the
phrasing is concise and parallel.

that for every dollar spent on controlled small‐
scale burning, forest thinning, and the training
of fire‐management personnel, it saves seven
dollars that would not be spent on having to
extinguish

Saves does not have a subject; construction
C	
is not a complete sentence; not having to
extinguish is wordy and awkward.
D	
That introduces a subordinate rather than
main clause, making a sentence fragment;
it has no referent; not having to extinguish is
wordy and awkward.

that for every dollar spent on controlled small‐
scale burning, forest thinning, and the training
of fire‐management personnel, seven dollars
are saved that would have been spent on
extinguishing

E	Introductory that makes a sentence
fragment; that would not have been spent on
extinguishing is awkward and illogical.

that for every dollar spent on controlled small‐
scale burning, forest thinning, and the training of
fire‐management personnel saves seven dollars
on not having to extinguish

(D)

for every dollar spent on controlled small‐scale
burning, forest thinning, and the training of
fire‐management personnel, that it saves seven
dollars on not having to extinguish

(E)

for every dollar spent on controlled small‐scale
burning, forest thinning, and the training of
fire‐management personnel, that seven dollars
are saved that would not have been spent on
extinguishing

Logical predication; Rhetorical construction

The pronoun it (it saves seven dollars) has no
referent. Making seven dollars the subject of
the clause eliminates this problem, and it also
fulfills a reader’s expectation that after the
phrase beginning for every dollar another specific
amount will be given to balance it. This change
in structure also allows the awkward and wordy
clause that would not be spent on having to
extinguish to be rewritten so that spent balances
saved: seven dollars are saved that would have been
spent on extinguishing, and the unnecessary having
to is omitted.

The correct answer is B.
85.

Like the grassy fields and old pastures that the
upland sandpiper needs for feeding and nesting
when it returns in May after wintering in the Argentine
Pampas, the sandpipers vanishing in the northeastern
United States is a result of residential and industrial
development and of changes in farming practices.
(A)

the sandpipers vanishing in the northeastern
United States is a result of residential and
industrial development and of changes in

(B)

the bird itself is vanishing in the northeastern
United States as a result of residential and
industrial development and of changes in

(C)

that the birds themselves are vanishing in the
northeastern United States is due to residential
and industrial development and changes to

(D)

in the northeastern United States, sandpipers’
vanishing due to residential and industrial
development and to changes in

(E)

in the northeastern United States, the
sandpipers’ vanishing, a result of residential and
industrial development and changing

89

GMAT® Official Guide 2018

Comparison; Sentence structure

The comparison introduced by like must be
logical and clear; the point of this comparison
is that both the habitat and the bird are
disappearing for similar reasons. The comparison
must use comparable grammatical components;
the bird itself is a noun phrase and matches the
noun phrases grassy fields and old pastures.
A	Illogically compares the sandpipers vanishing
to grassy fields and old pastures; omits
apostrophe in sandpipers’ vanishing; wordy.
B	
Correct. This sentence properly compares
the bird itself to grassy fields and old pastures;
is vanishing as the verb strengthens the
sentence by making the comparison clearer.
C	Does not finish the comparison begun with
like but instead substitutes a clause (that the
birds themselves are vanishing).
D	Illogically compares the sandpipers’ vanishing
to grassy fields and old pastures; creates a
sentence fragment.
E	Illogically compares the sandpipers’ vanishing
to grassy fields and old pastures; creates a
sentence fragment.
The correct answer is B.

86.

The results of two recent unrelated studies support the
idea that dolphins may share certain cognitive abilities
with humans and great apes; the studies indicate
dolphins as capable of recognizing themselves in
mirrors—an ability that is often considered a sign of
self‐awareness—and to grasp spontaneously the mood
or intention of humans.
(A)

dolphins as capable of recognizing themselves
in mirrors—an ability that is often considered
a sign of self‐awareness—and to grasp
spontaneously

(B)

dolphins’ ability to recognize themselves in
mirrors—an ability that is often considered as a
sign of self‐awareness—and of spontaneously
grasping

(C)

dolphins to be capable of recognizing
themselves in mirrors—an ability that is often
considered a sign of self‐awareness—and to
grasp spontaneously

(D)

that dolphins have the ability of recognizing
themselves in mirrors—an ability that is often
considered as a sign of self‐awareness—and
spontaneously grasping

(E)

that dolphins are capable of recognizing
themselves in mirrors—an ability that is often
considered a sign of self‐awareness—and of
spontaneously grasping

Grammatical construction; Parallelism

In the context of this sentence, the studies indicate
must introduce a clause; the clause must begin
with that and have a subject, dolphins, and a verb,
are (the complete verb phrase would be are capable
of ). The two capabilities should be parallel:
capable of recognizing…and of spontaneously
grasping.

90

3.6 Diagnostic Test Verbal Answer Explanations

A	Context requires a clause, but this
construction is not a clause; capable of
recognizing is not parallel to to grasp
spontaneously.

A	In this context, more likely is not a complete
idiomatic expression; was…to begin is not
parallel to was and merged.
B	
Correct. In this sentence, more than likely
is the correct comparative construction; the
simple past tense began, parallel to was and
merged, fits grammatically into the sentence.

B	Construction is not a clause, and a clause is
required; dolphins’ ability to recognize is not
parallel to of spontaneously grasping.
C	A clause is required following the studies
indicate; to be capable of recognizing is not
parallel to to grasp spontaneously.

C	Subject should be followed by three verbs;
beginning from is not a verb.
D	Use of the pronoun it makes this
construction a main clause, in which case
the comma after communication must be
omitted and began must be used to be
parallel to merged; was…begun is not the
correct tense.

D	
Have the ability of is wordy and unidiomatic;
of recognizing and spontaneously grasping are
not parallel.
E	
Correct. That introduces the subordinate
clause necessary to complete this sentence
properly; of recognizing and of spontaneously
grasping are parallel.

E	In this awkward, unclear, and wordy
construction, the first it must be followed
by is, not was, because the theory is
current; the second it acts as the subject
of the subordinate clause, and this usage
requires the omission of the comma after
communication.

The correct answer is E.
87.

According to scholars, the earliest writing was
probably not a direct rendering of speech, but was
more likely to begin as a separate and distinct
symbolic system of communication, and only later
merged with spoken language.
(A)

was more likely to begin as

(B)

more than likely began as

(C)

more than likely beginning from

(D)

it was more than likely begun from

(E)

it was more likely that it began

Idiom; Verb form

This sentence is a comparison in which probably
not x is balanced by but more than likely y. When
more is used in the comparative form of an
adjective (more difficult) or adverb (more likely),
it is followed by than. The words used to show
the comparison between x and y, but more than
likely, must also introduce the correct verb form,
allowing y to fit grammatically into the rest of the
sentence. The subject of the sentence has three
verbs, all of which should be parallel: the earliest
writing was…began…merged. Was…to begin is not
parallel and results in a construction that is not
grammatically correct.

The correct answer is B.
88.

In 1995 Richard Stallman, a well‐known critic of the
patent system, testified in Patent Office hearings that,
to test the system, a colleague of his had managed to
win a patent for one of Kirchhoff’s laws, an observation
about electric current first made in 1845 and now
included in virtually every textbook of elementary
physics.
(A)

laws, an observation about electric current first
made in 1845 and

(B)

laws, which was an observation about electric
current first made in 1845 and it is

(C)

laws, namely, it was an observation about
electric current first made in 1845 and

(D)

laws, an observation about electric current first
made in 1845, it is

(E)

laws that was an observation about electric
current, first made in 1845, and is

91

GMAT® Official Guide 2018

Logical predication; Parallelism

Agreement; Idiom; Parallelism

The function of the entire long phrase
(observation…physics) that follows one of
Kirchhoff ’s laws is to describe that law. It is a
noun phrase in apposition, which means that it
has the same syntactic relation to all the other
parts of the sentence that the noun phrase one of
Kirchhoff ’s laws does. Within the long modifying
phrase, parallelism is maintained by balancing an
observation…first made with and now included.

In long sentences such as this one, the
relationship between parts of the sentence may
be difficult to see. Here, the main clause of
the sentence is excavators…say and the logical
sequence that follows is the discovery…indicates
that. The subject of this first subordinate clause
is the singular noun discovery, which should be
followed by the singular verb indicates rather than
by the plural indicate, as is done in the original
sentence. Their, used with either development
or use, has no clear or logical referent in any of
the alternatives. The subject of the following
subordinate (that) clause, which has occurred as its
verb, is a series of three phrases, which must be
parallel, especially in a sentence of this length and
complexity: the development of…, the use of…, and
the standardization of….

Correct. In this sentence, the noun phrase
A	
in apposition properly identifies and
explains the law, using parallel structure and
concise expression.

B	
Which is ambiguous because it could refer to
one or to laws; it is violates the parallelism of
first made and now included.
C	
It is ambiguous; the introduction of it
was does not allow this construction to fit
grammatically into the sentence.

D	The referent of it is unclear; it is creates a
run‐on sentence and violates the parallelism
of first made and now included.

E	
That appears to refer to laws rather than
one, but the verb is singular; setting off the
phrase first made in 1845 in commas distorts
meaning; is violates parallelism.
The correct answer is A.
89.

92

Excavators at the Indus Valley site of Harappa in
eastern Pakistan say the discovery of inscribed
shards dating to circa 2800–2600 B.C. indicate their
development of a Harappan writing system, the use
of inscribed seals impressed into clay for marking
ownership, and the standardization of weights for trade
or taxation occurred many decades, if not centuries,
earlier than was previously believed.
(A)

indicate their development of a Harappan writing
system, the use of

(B)

indicate that the development of a Harappan
writing system, using

(C)

indicates that their development of a Harappan
writing system, using

(D)

indicates the development of a Harappan writing
system, their use of

(E)

indicates that the development of a Harappan
writing system, the use of

A	
Indicate does not agree with discovery; the
pronoun their has no logical referent, and
their development is not parallel to the use
and the standardization.
B	
Indicate does not agree with discovery; using
is not parallel to the development and the
standardization.
Their has no logical referent; the series of
C	
three elements should be parallel, but here
all are different.
D	The pronoun their has no logical referent,
and their use is not parallel to the development
and the standardization; the preferred
sentence structure would have indicates
followed by that when introducing a clause.
E	
Correct. In this sentence, indicates agrees
with discovery and is followed by that to
introduce a clause; the three parallel phrases
begin with an article (the), a noun, and the
preposition of.
The correct answer is E.

3.6 Diagnostic Test Verbal Answer Explanations

90.

The Supreme Court has ruled that public universities can
collect student activity fees even with students’ objections
to particular activities, so long as the groups they give
money to will be chosen without regard to their views.
(A)

with students’ objections to particular activities,
so long as the groups they give money to will be

(B)
(C)
(D)
(E)

91.

Despite the increasing number of women graduating
from law school and passing bar examinations, the
proportion of judges and partners at major law firms
who are women have not risen to a comparable extent.
(A)

if they have objections to particular activities and
the groups that are given the money are

the proportion of judges and partners at major
law firms who are women have not risen to a
comparable extent

(B)

if they object to particular activities, but the
groups that the money is given to have to be

the proportion of women judges and partners at
major law firms have not risen comparably

(C)

from students who object to particular activities,
so long as the groups given money are

the proportion of judges and partners at major law
firms who are women has not risen comparably

(D)

though students have an objection to particular
activities, but the groups that are given the
money be

yet the proportion of women judges and partners
at major law firms has not risen to a comparable
extent

(E)

yet the proportion of judges and partners at major
law firms who are women has not risen comparably

Logical predication; Rhetorical construction

The underlined portion of the sentence fails to
establish a clear relationship among universities,
students, and groups. To which of these three does
they refer? It would appear that the universities must
give the money, but they does not have a referent.
Furthermore, they is followed by their views, and
in this case their must refer to groups. Wordy and
awkward phrasing as well as an unnecessary shift in
verb tense (will be chosen) compound the difficulty of
understanding this sentence in its original form.

Agreement; Rhetorical construction

A	
With students’ objections…is awkward and
dense; they does not have a referent; the
future will be is incorrect since the Supreme
Court has already ruled.

A	Plural verb, have risen, does not agree with
the singular subject, proportion.

B	Referent for they is student activity fees,
which cannot possibly have objections…; the
use of and is illogical.
C	
They refers to student activity fees rather than
students; but does not have the requisite sense
of with the provision that; have to be is wordy.
D	
Correct. In this sentence, from students who
object is clear and idiomatic; so long as is used
appropriately; groups given money eliminates
the problem of a pronoun without a
referent; are is the proper tense.
E	
Have an objection is an unnecessarily wordy
way to say object; the verb be does not
complete the latter part of the sentence.
The correct answer is D.

When a number of plural nouns appear in
phrases between a singular subject and the verb,
it can be easy to overlook the true subject of the
verb. Here, judges, partners, firms, and women all
occur between the singular subject, proportion,
and the verb, which should also be singular, has
risen. Concise expression is particularly important
in a long construction; to a comparable extent may
be more concisely expressed as comparably.

B	
Have risen does not agree with proportion;
here, women applies only to judges, not to
partners at major law firms.

C	
Correct. In this sentence, has risen agrees
with proportion, and comparably is more
concise than to a comparable extent. The
modifying clause who are women follows
(1) judges and (2) partners at major law firms
as closely as is possible given the content of
the sentence; this positioning has the virtue
of being clear in its meaning.

D	The contrast has already been introduced by
despite, so the addition of yet is illogical and
ungrammatical; to a comparable extent is wordy.
E	
Despite introduces the contrast; adding yet
is illogical and results in an ungrammatical
construction.
The correct answer is C.

93

GMAT® Official Guide 2018

92.

(A)

B	
Correct. This sentence properly has the
single modifier consisting of two contrasting
parts.
C	Neither and nor but acts as a logical
connector; the use of connecting results in a
sentence fragment.

Seldom more than 40 feet wide and 12 feet
deep, but it ran 363 miles across the rugged
wilderness of upstate New York, the Erie Canal
connected

(B)

Seldom more than 40 feet wide or 12 feet
deep but running 363 miles across the rugged
wilderness of upstate New York, the Erie Canal
connected

(C)

It was seldom more than 40 feet wide and
12 feet deep, and ran 363 miles across the
rugged wilderness of upstate New York, but the
Erie Canal, connecting

(D)

The Erie Canal was seldom more than 40 feet
wide or 12 feet deep and it ran 363 miles across
the rugged wilderness of upstate New York,
which connected

(E)

The Erie Canal, seldom more than 40 feet wide
and 12 feet deep, but running 363 miles across
the rugged wilderness of upstate New York,
connecting

Logical predication; Grammatical construction

The phrase seldom…deep is the first half of a
modifier that describes the Erie Canal. However,
because a comma incorrectly follows deep, this
phrase appears to be the entire modifier, which
must agree with the noun or pronoun that
immediately follows it. This phrase cannot modify
the conjunction but, and it has no referent; but it
ran is not a logical or grammatical construction
following the modifying phrase. Substituting
running for it ran creates an adjective phrase
parallel to the first adjective phrase (seldom…
deep).To contrast the small size reported in the
first phrase with the great distance reported in the
second, the two phrases may be joined with but;
together they create a single modifier correctly
modifying the Erie Canal. The Erie Canal is then
the subject of the sentence and requires the verb
connected to provide a logical statement.

94

A	
But it ran cannot logically or grammatically
follow the modifying phrase.

Seldom more than 40 feet wide and 12 feet deep,
but it ran 363 miles across the rugged wilderness
of upstate New York, the Erie Canal connected the
Hudson River at Albany to the Great Lakes at Buffalo,
providing the port of New York City with a direct water
link to the heartland of the North American continent.

D	The paired concepts of width and depth
should be joined by and, not or; this
construction calls for two main clauses to
be separated by a comma after deep; which is
ambiguous.
E	The two halves of the modifier should not
be separated by a comma after deep; the
subject is awkwardly and confusingly placed
at a great distance from the predicate;
the use of connecting rather than connected
creates a sentence fragment.
The correct answer is B.
93.

In 1923, the Supreme Court declared a minimum wage
for women and children in the District of Columbia
as unconstitutional, and ruling that it was a form of
price‐fixing and, as such, an abridgment of the right of
contract.
(A)

the Supreme Court declared a minimum wage
for women and children in the District of
Columbia as unconstitutional, and

(B)

the Supreme Court declared as unconstitutional
a minimum wage for women and children in the
District of Columbia, and

(C)

the Supreme Court declared unconstitutional a
minimum wage for women and children in the
District of Columbia,

(D)

a minimum wage for women and children
in the District of Columbia was declared
unconstitutional by the Supreme Court,

(E)

when the Supreme Court declared a minimum
wage for women and children in the District of
Columbia as unconstitutional,

3.6 Diagnostic Test Verbal Answer Explanations

Idiom; Grammatical construction

This sentence depends on the correct use of
an idiom: the court declares x unconstitutional.
The inverted form should be used here because
of the long phrases involved: the court declares
unconstitutional x. The Supreme Court is the subject
of the sentence; declared is the verb. Ruling…
contract acts as a modifier describing the action
of the main clause; because the modifier is
subordinate to the main clause, the conjunction
and must be omitted. And is used to join two
independent clauses, not a clause and its modifier.
A	
Declared…as unconstitutional is not the
correct idiom; the use of and creates an
ungrammatical construction.
B	
Declared as unconstitutional is not the
correct idiom; the use of and creates an
ungrammatical construction.
C	
Correct. In this sentence, the correct idiom
is used, and the modifier is grammatically
and logically attached to the main clause.
D	Passive voice construction is weak and
wordy; its use causes the modifier to be
misplaced and ambiguous.
E	
Declared…as unconstitutional is not the
correct idiom; when transforms the main
clause into a subordinate clause, resulting in
a sentence fragment.
The correct answer is C.

94.

Researchers have found that individuals who have been
blind from birth, and who thus have never seen anyone
gesture, nevertheless make hand motions when
speaking just as frequently and in virtually the same
way as sighted people do, and that they will gesture
even when conversing with another blind person.
(A)

who thus have never seen anyone gesture,
nevertheless make hand motions when speaking
just as frequently and in virtually the same way
as sighted people do, and that they will gesture

(B)

who thus never saw anyone gesturing,
nevertheless make hand motions when speaking
just as frequent and in virtually the same way as
sighted people did, and that they will gesture

(C)

who thus have never seen anyone gesture,
nevertheless made hand motions when speaking
just as frequently and in virtually the same way
as sighted people do, as well as gesturing

(D)

thus never having seen anyone gesture,
nevertheless made hand motions when speaking
just as frequent and in virtually the same way as
sighted people did, as well as gesturing

(E)

thus never having seen anyone gesture,
nevertheless to make hand motions when
speaking just as frequently and in virtually the
same way as sighted people do, and to gesture

Parallelism; Verb form; Diction

The researchers have found (1) that individuals…
make hand motions…as sighted people do and
(2) that they will gesture…with another blind
person. In the original sentence, the two findings
are reported in two parallel subordinate clauses
introduced by that. The verb tenses are logical and
parallel: who have been blind and who have never
seen indicate a condition that began in the past
and continues in the present; make and do refer
to present actions. The verb make (hand motions)
is correctly modified by the adverb frequently to
show how the action of the verb is carried out.
The emphatic future will gesture is properly used
here with even to emphasize the extreme or the
unexpected.

95

GMAT® Official Guide 2018

A	
Correct. Although the original sentence is
complicated, the parallelism of its structure
and phrasing allows its meaning to be clear
and its expression effective.

Idiom; Grammatical construction

Two constructions create problems in the
original sentence. The first is the unidiomatic
construction have the ability of developing; ability
must be followed by an infinitive, to develop, not
a phrase. The second problematic construction
is to develop themselves into. In this biological
context, the verb develop means to progress from
an earlier to a later stage; it is used intransitively,
which means that it cannot take an object. The
pronoun themselves acts as an object, creating a
construction that is not grammatical or logical.
Omitting the pronoun removes the problem.

B	Verbs saw and did indicate action completed
in the past; the simple past tense is not
appropriate in either case; the adjective
frequent cannot modify the verb; awkward
and muddy.
C	
Made indicates past action, but the
present tense is logically required; as well
as gesturing violates the parallelism of the
two subordinate (that) clauses; choppy and
unclear.

Ability is incorrectly followed by of
A	
developing; a pronoun cannot follow
develop, when it is used, as it is here, in its
intransitive sense.

D	
Having seen is not parallel to have been;
made and did do not show ongoing action;
frequent incorrectly modifies the verb; as
well as gesturing destroys the parallelism of
the two subordinate (that) clauses; awkward
and unclear.

B	
Correct. Ability is properly followed by the
infinitive in this sentence, and the pronoun
themselves is omitted.
C	This awkward and wordy construction
violates the parallelism of like embryonic
germ cells…embryonic stem cells….

E	Replacing the verb make with the infinitive
to make results in an ungrammatical
construction that fails to complete the
sentence.

D	The two parts of the comparison must be
parallel; like embryonic germ cells must be
followed by embryonic stem cells, not the
ability to develop.

The correct answer is A.
95.

Like embryonic germ cells, which are cells that
develop early in the formation of the fetus and that
later generate eggs or sperm, embryonic stem cells
have the ability of developing themselves into different
kinds of body tissue.
(A)

96

embryonic stem cells have the ability of
developing themselves into different kinds of
body tissue

(B)

embryonic stem cells have the ability to develop
into different kinds of body tissue

(C)

in embryonic stem cells there is the ability to
develop into different kinds of body tissue

(D)

the ability to develop themselves into different
kinds of body tissue characterizes embryonic
stem cells

(E)

the ability of developing into different kinds of
body tissue characterizes embryonic stem cells

E	
Ability is followed by the unidiomatic of
developing rather than to develop; the main
clause must begin with embryonic stem cells to
balance and complete like embryonic germ cells.
The correct answer is B.
96.

Critics contend that the new missile is a weapon
whose importance is largely symbolic, more a tool for
manipulating people’s perceptions than to fulfill a real
military need.
(A)

for manipulating people’s perceptions than to fulfill

(B)

for manipulating people’s perceptions than for
fulfilling

(C)

to manipulate people’s perceptions rather than
that it fulfills

(D)

to manipulate people’s perceptions rather than
fulfilling

(E)

to manipulate people’s perceptions than for
fulfilling

3.6 Diagnostic Test Verbal Answer Explanations

Parallelism

Logical predication

This sentence uses the comparative construction
more x than y where x and y must be parallel.
Here, x is a tool for manipulating people’s
perceptions, and y is to fulfill a real military need. A
tool does not need to be repeated in the second
half of the comparison because it is understood,
but the wording of the two phrases does need to
match. There are two acceptable solutions: (1) for
manipulating can be followed by for fulfilling or
(2) to manipulate can be followed by to fulfill.

The original sentence contains a number of
modifiers, but not all of them are correctly
expressed. The clause who trained… describes
Stella Adler, yet a relative clause such as this
one must be placed immediately after the noun
or pronoun it modifies, and this clause follows
theater rather than Adler. Replacing who trained
with training corrects the error because the phrase
training… modifies the whole preceding clause
rather than the single preceding noun. Several
generations of actors including shows the same error
in reverse; including modifies the whole phrase,
but the two actors named are not generations
of actors. The more limiting clause whose ranks
included (referring to actors) is appropriate here.

A

For manipulating is not parallel to to fulfill.

B	
Correct. For manipulating and for fulfilling
are parallel in this sentence.
C

To manipulate is not parallel to that it fulfills.

D

To manipulate is not parallel to fulfilling.

E

To manipulate is not parallel to for fulfilling.

The correct answer is B.
97.

As an actress and, more importantly, as a teacher
of acting, Stella Adler was one of the most influential
artists in the American theater, who trained several
generations of actors including Marlon Brando and
Robert De Niro.
(A)

Stella Adler was one of the most influential
artists in the American theater, who trained
several generations of actors including

(B)

Stella Adler, one of the most influential artists
in the American theater, trained several
generations of actors who include

(C)

Stella Adler was one of the most influential
artists in the American theater, training several
generations of actors whose ranks included

(D)

one of the most influential artists in the American
theater was Stella Adler, who trained several
generations of actors including

(E)

one of the most influential artists in the American
theater, Stella Adler, trained several generations
of actors whose ranks included

A	Relative (who) clause follows theater rather
than Adler; including refers to generations of
actors, when the reference should be to actors
only.
B	This construction, in which the subject is
both preceded and followed by modifiers, is
awkward; the verbs should be consistently in
the past tense, but include is present tense.
C	
Correct. In this sentence, substituting
training for who trained and whose ranks
included for including eliminates the
modification errors.
D	Introductory modifier must be immediately
followed by Stella Adler, not one…; including
refers to generations of actors rather than to
actors only.
E	Introductory modifier must be immediately
followed by Stella Adler, not one.
The correct answer is C.

97

GMAT® Official Guide 2018

98.

By developing the Secure Digital Music Initiative, the
recording industry associations of North America,
Japan, and Europe hope to create a standardized way
of distributing songs and full‐length recordings on the
Internet that will protect copyright holders and foil the
many audio pirates who copy and distribute digital
music illegally.

A	
Correct. The verbs will protect and (will )
foil are parallel in this sentence, as are the
verbs copy and distribute.

(A)

of distributing songs and full‐length recordings
on the Internet that will protect copyright holders
and foil the many audio pirates who copy and
distribute

(B)

of distributing songs and full‐length recordings
on the Internet and to protect copyright holders
and foiling the many audio pirates copying and
distributing

C	
Way for should instead be way of; the
pronoun reference in while it protects is
ambiguous; construction suggests that
protection comes from something other
than the standardized way.

(C)

(D)

(E)

D	Pronoun they has no referent; use of while
suggests that protection comes from
something other than the standardized way
of distribution.

for distributing songs and full‐length recordings
on the Internet while it protects copyright holders
and foils the many audio pirates who copy and
distribute

And it will protect distorts meaning,
E	
suggesting that protection comes in addition
to the standardized way; will protect and
foiling are not parallel.

to distribute songs and full‐length recordings
on the Internet while they will protect copyright
holders and foil the many audio pirates copying
and distributing
to distribute songs and full‐length recordings on
the Internet and it will protect copyright holders
and foiling the many audio pirates who copy and
distribute

Parallelism

The original sentence depends on the parallelism
of its verbs to make its point clearly and
effectively. A standardized way…will protect and
(will understood) foil; pirates…copy and distribute.
In the first pair of parallel verbs, will does not
need to be repeated because it is understood.

98

B	
And to protect distorts meaning, suggesting
that protection comes in addition to the
standardized way; foiling is not parallel to to
protect.

The correct answer is A.
99.

Whereas a ramjet generally cannot achieve high
speeds without the initial assistance of a rocket, high
speeds can be attained by scramjets, or supersonic
combustion ramjets, in that they reduce airflow
compression at the entrance of the engine and letting
air pass through at supersonic speeds.
(A)

high speeds can be attained by scramjets, or
supersonic combustion ramjets, in that they
reduce

(B)

that high speeds can be attained by scramjets,
or supersonic combustion ramjets, is a result of
their reducing

(C)

the ability of scramjets, or supersonic
combustion ramjets, to achieve high speeds is
because they reduce

(D)

scramjets, or supersonic combustion ramjets,
have the ability of attaining high speeds when
reducing

(E)

scramjets, or supersonic combustion ramjets,
can attain high speeds by reducing

3.6 Diagnostic Test Verbal Answer Explanations

Rhetorical construction

Diction; Parallelism

The underlined portion of the original sentence
is wordy and ineffective. Transforming it from
passive (high speeds can be attained by scramjets)
to active voice (scramjets can attain high speeds)
eliminates much of the problem. As the subject
of the main clause, scramjets correctly parallels a
ramjet, the subject of the subordinate clause; the
contrast is thus clearly and effectively drawn. In
that they reduce is wordy and awkward; it can be
replaced by the more concise phrase by reducing.

The preposition like introduces nouns and noun
phrases; the conjunction as introduces verbs or
clauses, so as is required here. The comparative
construction used here is just as x so y; x and
y must be parallel. The y clause is written in
effective subject‐verb‐object order: melting sea
ice does not increase oceanic volume. The original
wordy, awkward x clause is not parallel. To make
it parallel, melting ice cubes should be the subject
of the clause, do not cause…to overflow the verb
phrase, and a glass of water the object.

A	Passive voice contributes to a wordy,
awkward, and ineffective construction; in
that they reduce is also wordy and awkward.
B	Passive voice and subordinate (that) clause
constructions are wordy, awkward, and
ineffective.
C	
The ability…is because is not a grammatical
construction; scramjets, not the ability, should
be parallel to a ramjet.
Have the ability of attaining is wordy; when
D	
does not indicate the cause‐and‐effect
relationship.
E	
Correct. Scramjets parallels a ramjet for an
effective contrast in this sentence; the active
voice is clear and concise; by reducing shows
how scramjets attain high speeds.

A	
Like is used in place of as; the two elements
of comparison are not parallel.
B	
Like is used in place of as; that violates
parallelism.
C	
As or just as is needed to introduce the
clause; the two clauses are not parallel.
D	
That violates the parallelism of the two
clauses and creates an ungrammatical
construction.
E	
Correct. This sentence has just as properly
introducing the first clause, and the two
clauses are parallel.
The correct answer is E.

The correct answer is E.
100. It will not be possible to implicate melting sea ice in
the coastal flooding that many global warming models
have projected: just like a glass of water that will not
overflow due to melting ice cubes, so melting sea ice
does not increase oceanic volume.
(A)

like a glass of water that will not overflow due to
melting ice cubes,

(B)

like melting ice cubes that do not cause a glass
of water to overflow,

(C)

a glass of water will not overflow because of
melting ice cubes,

(D)

as melting ice cubes that do not cause a glass
of water to overflow,

(E)

as melting ice cubes do not cause a glass of
water to overflow,

99

4.0 Math Review

100

4.0 Math Review

4.0 Math Review
Although this chapter provides a review of some of the mathematical concepts of arithmetic, algebra,
and geometry, it is not intended to be a textbook. You should use this chapter to familiarize yourself
with the kinds of topics that may be tested in the GMAT® exam. You may wish to consult an
arithmetic, algebra, or geometry book for a more detailed discussion of some of the topics.
Section 4.1, “Arithmetic,” includes the following topics:
1. Properties of Integers

7. Powers and Roots of Numbers

2. Fractions

8. Descriptive Statistics

3. Decimals

9. Sets

4. Real Numbers

10. Counting Methods

5. Ratio and Proportion

11. Discrete Probability

6. Percents
Section 4.2, “Algebra,” does not extend beyond what is usually covered in a first‐year high school algebra
course. The topics included are as follows:
1. Simplifying Algebraic Expressions

6. Solving Quadratic Equations

2. Equations

7. Exponents

3. Solving Linear Equations with One
Unknown

8. Inequalities

4. Solving Two Linear Equations with
Two Unknowns

9. Absolute Value
10. Functions

5. Solving Equations by Factoring
Section 4.3, “Geometry,” is limited primarily to measurement and intuitive geometry or spatial
visualization. Extensive knowledge of theorems and the ability to construct proofs, skills that are usually
developed in a formal geometry course, are not tested. The topics included in this section are the following:
1. Lines

6. Triangles

2. Intersecting Lines and Angles

7. Quadrilaterals

3. Perpendicular Lines

8. Circles

4. Parallel Lines

9. Rectangular Solids and Cylinders

5. Polygons (Convex)

10. Coordinate Geometry

Section 4.4, “Word Problems,” presents examples of and solutions to the following types of word problems:
1. Rate Problems

6. Profit

2. Work Problems

7. Sets

3. Mixture Problems

8. Geometry Problems

4. Interest Problems

9. Measurement Problems

5. Discount

10. Data Interpretation
101

GMAT® Official Guide 2018

4.1 Arithmetic
1. Properties of Integers
An integer is any number in the set {. . . −3, −2, −1, 0, 1, 2, 3, . . .}. If x and y are integers and x ≠ 0, then
x is a divisor (factor) of y provided that y = xn for some integer n. In this case, y is also said to be divisible
by x or to be a multiple of x. For example, 7 is a divisor or factor of 28 since 28 = (7)(4), but 8 is not a
divisor of 28 since there is no integer n such that 28 = 8n.
If x and y are positive integers, there exist unique integers q and r, called the quotient and remainder,
respectively, such that y = xq + r and 0 ≤ r < x. For example, when 28 is divided by 8, the quotient is 3
and the remainder is 4 since 28 = (8)(3) + 4. Note that y is divisible by x if and only if the remainder r
is 0; for example, 32 has a remainder of 0 when divided by 8 because 32 is divisible by 8. Also, note that
when a smaller integer is divided by a larger integer, the quotient is 0 and the remainder is the smaller
integer. For example, 5 divided by 7 has the quotient 0 and the remainder 5 since 5 = (7)(0) + 5.
Any integer that is divisible by 2 is an even integer; the set of even integers is
{. . . −4, −2, 0, 2, 4, 6, 8, . . .}. Integers that are not divisible by 2 are odd integers;
{. . . −3, −1, 1, 3, 5, . . .} is the set of odd integers.
If at least one factor of a product of integers is even, then the product is even; otherwise the product is
odd. If two integers are both even or both odd, then their sum and their difference are even. Otherwise,
their sum and their difference are odd.
A prime number is a positive integer that has exactly two different positive divisors, 1 and itself. For
example, 2, 3, 5, 7, 11, and 13 are prime numbers, but 15 is not, since 15 has four different positive
divisors, 1, 3, 5, and 15. The number 1 is not a prime number since it has only one positive divisor. Every
integer greater than 1 either is prime or can be uniquely expressed as a product of prime factors. For
example, 14 = (2)(7), 81 = (3)(3)(3)(3), and 484 = (2)(2)(11)(11).
The numbers −2, −1, 0, 1, 2, 3, 4, 5 are consecutive integers. Consecutive integers can be represented by
n, n + 1, n + 2, n + 3, . . ., where n is an integer. The numbers 0, 2, 4, 6, 8 are consecutive even integers, and 1,
3, 5, 7, 9 are consecutive odd integers. Consecutive even integers can be represented by 2n, 2n + 2, 2n + 4, . . .,
and consecutive odd integers can be represented by 2n + 1, 2n + 3, 2n + 5, . . ., where n is an integer.
Properties of the integer 1. If n is any number, then 1 ⋅ n = n, and for any number n ≠ 0, n ⋅ 1 = 1 .
n
n
The number 1 can be expressed in many ways; for example, = 1 for any number n ≠ 0.
n
Multiplying or dividing an expression by 1, in any form, does not change the value of that expression.
Properties of the integer 0. The integer 0 is neither positive nor negative. If n is any number, then n + 0 = n
and n ⋅ 0 = 0. Division by 0 is not defined.

102

4.1 Math Review Arithmetic

2. Fractions
In a fraction n , n is the numerator and d is the denominator. The denominator of a fraction can never be
d
0, because division by 0 is not defined.
14
are
Two fractions are said to be equivalent if they represent the same number. For example, 8 and
63
36
equivalent since they both represent the number 2 . In each case, the fraction is reduced to lowest terms
9
by dividing both numerator and denominator by their greatest common divisor (gcd). The gcd of 8 and 36
is 4 and the gcd of 14 and 63 is 7.

Addition and subtraction of fractions.
Two fractions with the same denominator can be added or subtracted by performing the required
3 4 3+ 4 = 7
operation with the numerators, leaving the denominators the same. For example, + =
5
5
5
5
5 2 5− 2 = 3
and − =
. If two fractions do not have the same denominator, express them as equivalent
7 7
7
7
3
4
and , multiply the numerator and
fractions with the same denominator. For example, to add
5
7
denominator of the first fraction by 7 and the numerator and denominator of the second fraction by 5,
21
21 + 20 = 41
20
obtaining
, respectively;
and
.
35
35 35 35
35
For the new denominator, choosing the least common multiple (lcm) of the denominators usually lessens
the work. For 2 + 1 , the lcm of 3 and 6 is 6 (not 3 × 6 = 18), so 2 + 1 = 2 × 2 + 1 = 4 + 1 = 5 .
3 6
3 6 3 2 6 6 6 6

Multiplication and division of fractions.
To multiply two fractions, simply multiply the two numerators and multiply the two denominators.
For example, 2 × 4 = 2 × 4 = 8 .
3 7 3 × 7 21
To divide by a fraction, invert the divisor (that is, find its reciprocal ) and multiply. For example,
2 ÷ 4 = 2 × 7 = 14 = 7 .
3 7 3 4 12 6
In the problem above, the reciprocal of 4 is 7 . In general, the reciprocal of a fraction n is d , where n
7
4
d
n
and d are not zero.

Mixed numbers.
A number that consists of a whole number and a fraction, for example, 7 2 , is a mixed number: 7 2
3
3
means 7 + 2 .
3
To change a mixed number into a fraction, multiply the whole number by the denominator of the
fraction and add this number to the numerator of the fraction; then put the result over the denominator
( 3 × 7 ) + 2 = 23 .
of the fraction. For example, 7 2 =
3
3
3

103

GMAT® Official Guide 2018

3. Decimals

4

.

Thousandths

5

Hundredths

6

Tenths

Ones or units

,

Tens

7

Hundreds

Thousands

In the decimal system, the position of the period or decimal point determines the place value of the
digits. For example, the digits in the number 7,654.321 have the following place values:

3

2

1

Some examples of decimals follow.
0.321 = 3 + 2 + 1 = 321
10 100 1, 000 1, 000
0.0321 = 0 + 3 + 2 + 1 = 321
10 100 1, 000 10, 000 10, 000
1.56 = 1 + 5 + 6 = 156
10 100 100
Sometimes decimals are expressed as the product of a number with only one digit to the left of the
decimal point and a power of 10. This is called scientific notation. For example, 231 can be written as
2.31 × 102 and 0.0231 can be written as 2.31 × 10−2. When a number is expressed in scientific notation,
the exponent of the 10 indicates the number of places that the decimal point is to be moved in the
number that is to be multiplied by a power of 10 in order to obtain the product. The decimal point is
moved to the right if the exponent is positive and to the left if the exponent is negative. For example,
2.013 × 104 is equal to 20,130 and 1.91 × 10−4 is equal to 0.000191.

Addition and subtraction of decimals.
To add or subtract two decimals, the decimal points of both numbers should be lined up. If one of the
numbers has fewer digits to the right of the decimal point than the other, zeros may be inserted to the
right of the last digit. For example, to add 17.6512 and 653.27, set up the numbers in a column and add:
17.6512
+ 653.2700
670.9212
Likewise for 653.27 minus 17.6512:
653.2700
−17.6512
635.6188

104

4.1 Math Review Arithmetic

Multiplication of decimals.
To multiply decimals, multiply the numbers as if they were whole numbers and then insert the decimal
point in the product so that the number of digits to the right of the decimal point is equal to the sum of
the numbers of digits to the right of the decimal points in the numbers being multiplied. For example:
2.09 (2 digits to the right)
× 1.3 (l digit to the right)
627
2090
2.717 (2 + 1 = 3 digits to the right)

Division of decimals.
To divide a number (the dividend) by a decimal (the divisor), move the decimal point of the divisor to
the right until the divisor is a whole number. Then move the decimal point of the dividend the same
number of places to the right, and divide as you would by a whole number. The decimal point in the
quotient will be directly above the decimal point in the new dividend. For example, to divide 698.12
by 12.4:

will be replaced by:
and the division would proceed as follows:

4. Real Numbers
All real numbers correspond to points on the number line and all points on the number line correspond
to real numbers. All real numbers except zero are either positive or negative.
−

3
2

–6 –5 –4 –3 –2 –1

2

0.2
0

1

2

3

4

5

6

105

GMAT® Official Guide 2018

On a number line, numbers corresponding to points to the left of zero are negative and numbers
corresponding to points to the right of zero are positive. For any two numbers on the number line, the
number to the left is less than the number to the right; for example, −4 < −3 < − 3 < −1, and 1 < 2 < 2.
2
To say that the number n is between 1 and 4 on the number line means that n > 1 and n < 4, that is, 1
< n < 4. If n is “between 1 and 4, inclusive,” then 1 ≤ n ≤ 4.
The distance between a number and zero on the number line is called the absolute value of the number.
Thus 3 and −3 have the same absolute value, 3, since they are both three units from zero. The absolute
value of 3 is denoted |3|. Examples of absolute values of numbers are
| −5| = | 5| = 5, − 7 = 7 , and | 0 | = 0.
2 2
Note that the absolute value of any nonzero number is positive.
Here are some properties of real numbers that are used frequently. If x, y, and z are real numbers, then
(1) x + y = y + x and xy = yx.
For example, 8 + 3 = 3 + 8 = 11, and (l7)(5) = (5)(l7) = 85.
(2) (x + y) + z = x + (y + z) and (xy)z = x(yz).
For example, (7 + 5) + 2 = 7 + (5 + 2) = 7 + (7) = 14, and 5 3

(

)( 3 ) = ( 5) (

(3) xy + xz = x(y + z).
For example, 718(36) + 718(64) = 718(36 + 64) = 718(l00) = 71,800.

)

3 3 = ( 5 ) ( 3) = 15.

(4) If x and y are both positive, then x + y and xy are positive.
(5) If x and y are both negative, then x + y is negative and xy is positive.
(6) If x is positive and y is negative, then xy is negative.
(7) If xy = 0, then x = 0 or y = 0. For example, 3y = 0 implies y = 0.
(8) |x + y| ≤ |x| + |y|. For example, if x = 10 and y = 2, then |x + y| = |12| = 12 = |x| + |y|;
and if x = 10 and y = −2, then |x + y| = |8| = 8 < 12 = |x| + |y|.

5. Ratio and Proportion
The ratio of the number a to the number b (b ≠ 0) is

a
.
b

A ratio may be expressed or represented in several ways. For example, the ratio of 2 to 3 can be written
as 2 to 3, 2:3, or 2 . The order of the terms of a ratio is important. For example, the ratio of the number
3
of months with exactly 30 days to the number with exactly 31 days is 4 , not 7 .
7
4

106

4.1 Math Review Arithmetic

A proportion is a statement that two ratios are equal; for example, 2 = 8 is a proportion. One way to
3 12
solve a proportion involving an unknown is to cross multiply, obtaining a new equality. For example, to
solve for n in the proportion 2 = n , cross multiply, obtaining 24 = 3n; then divide both sides by 3, to
3 12
get n = 8.

6. Percents
Percent means per hundred or number out of 100. A percent can be represented as a fraction with a
denominator of 100, or as a decimal. For example:
37% = 37 = 0.37.
100
To find a certain percent of a number, multiply the number by the percent expressed as a decimal or
fraction. For example:
20% of 90 = 0.2 × 90 = 18
or
20% of 90 = 20 × 90 = 1 × 90 = 18.
100
5

Percents greater than 100%.
Percents greater than 100% are represented by numbers greater than 1. For example:
300% = 300 = 3
100
250% of 80 = 2.5 × 80 = 200.

Percents less than 1%.
The percent 0.5% means 1 of 1 percent. For example, 0.5% of 12 is equal to 0.005 × 12 = 0.06.
2

Percent change.
Often a problem will ask for the percent increase or decrease from one quantity to another quantity.
For example, “If the price of an item increases from $24 to $30, what is the percent increase in price?”
To find the percent increase, first find the amount of the increase; then divide this increase by the
original amount, and express this quotient as a percent. In the example above, the percent increase
would be found in the following way: the amount of the increase is (30 − 24) = 6. Therefore, the
percent increase is 6 = 0.25 = 25% .
24
Likewise, to find the percent decrease (for example, the price of an item is reduced from $30 to $24),
first find the amount of the decrease; then divide this decrease by the original amount, and express this
quotient as a percent. In the example above, the amount of decrease is (30 − 24) = 6.
Therefore, the percent decrease is 6 = 0.20 = 20%.
30
Note that the percent increase from 24 to 30 is not the same as the percent decrease from 30 to 24.
107

GMAT® Official Guide 2018

In the following example, the increase is greater than 100 percent: If the cost of a certain house in 1983
was 300 percent of its cost in 1970, by what percent did the cost increase?
If n is the cost in 1970, then the percent increase is equal to 3n − n = 2n = 2 , or 200%.
n
n

7. Powers and Roots of Numbers
When a number k is to be used n times as a factor in a product, it can be expressed as kn, which means
the nth power of k. For example, 2 2 = 2 × 2 = 4 and 23 = 2 × 2 × 2 = 8 are powers of 2.
Squaring a number that is greater than 1, or raising it to a higher power, results in a larger number;
squaring a number between 0 and 1 results in a smaller number. For example:
32 = 9
2

⎛ 1⎞ = 1
⎝ 3⎠
9
2
( 0.1) = 0.01

(9 > 3)
⎛ 1 < 1⎞
⎝ 9 3⎠
(0.01 < 0.1)

A square root of a number n is a number that, when squared, is equal to n. The square root of a negative
number is not a real number. Every positive number n has two square roots, one positive and the other
negative, but n denotes the positive number whose square is n. For example, 9 denotes 3. The two
square roots of 9 are 9 = 3 and − 9 = −3.
Every real number r has exactly one real cube root, which is the number s such that s3 = r. The real cube
root of r is denoted by 3 r . Since 23 = 8, 3 8 = 2 . Similarly, 3 −8 = −2 , because (−2)3 = −8.

8. Descriptive Statistics
A list of numbers, or numerical data, can be described by various statistical measures. One of the most
common of these measures is the average, or (arithmetic) mean, which locates a type of “center” for the
data. The average of n numbers is defined as the sum of the n numbers divided by n. For example, the
average of 6, 4, 7, 10, and 4 is 6 + 4 + 7 + 10 + 4 = 31 = 6.2.
5
5
The median is another type of center for a list of numbers. To calculate the median of n numbers, first
order the numbers from least to greatest; if n is odd, the median is defined as the middle number,
whereas if n is even, the median is defined as the average of the two middle numbers. In the example
above, the numbers, in order, are 4, 4, 6, 7, 10, and the median is 6, the middle number.
For the numbers 4, 6, 6, 8, 9, 12, the median is 6 + 8 = 7. Note that the mean of these numbers is 7.5.
2
The median of a set of data can be less than, equal to, or greater than the mean. Note that for a large set
of data (for example, the salaries of 800 company employees), it is often true that about half of the data
is less than the median and about half of the data is greater than the median; but this is not always the
case, as the following data show.
3, 5, 7, 7, 7, 7, 7, 7, 8, 9, 9, 9, 9, 10, 10
Here the median is 7, but only

108

2
of the data is less than the median.
15

4.1 Math Review Arithmetic

The mode of a list of numbers is the number that occurs most frequently in the list. For example, the
mode of 1, 3, 6, 4, 3, 5 is 3. A list of numbers may have more than one mode. For example, the list
1, 2, 3, 3, 3, 5, 7, 10, 10, 10, 20 has two modes, 3 and 10.
The degree to which numerical data are spread out or dispersed can be measured in many ways. The
simplest measure of dispersion is the range, which is defined as the greatest value in the numerical data
minus the least value. For example, the range of 11, 10, 5, 13, 21 is 21 − 5 = 16. Note how the range
depends on only two values in the data.
One of the most common measures of dispersion is the standard deviation. Generally speaking, the
more the data are spread away from the mean, the greater the standard deviation. The standard deviation
of n numbers can be calculated as follows: (1) find the arithmetic mean, (2) find the differences between
the mean and each of the n numbers, (3) square each of the differences, (4) find the average of the
squared differences, and (5) take the nonnegative square root of this average. Shown below is this
calculation for the data 0, 7, 8, 10, 10, which have arithmetic mean 7.

x

x−7

(x − 7)2

0

−7

49

7

0

0

8

1

1

10

3

9

10

3

9

Total

Standard deviation

68 ≈ 3.7
5

68

Notice that the standard deviation depends on every data value, although it depends most on values
that are farthest from the mean. This is why a distribution with data grouped closely around the mean
will have a smaller standard deviation than will data spread far from the mean. To illustrate this,
compare the data 6, 6, 6.5, 7.5, 9, which also have mean 7. Note that the numbers in the second set of
data seem to be grouped more closely around the mean of 7 than the numbers in the first set. This is
reflected in the standard deviation, which is less for the second set (approximately 1.1) than for the first
set (approximately 3.7).
There are many ways to display numerical data that show how the data are distributed. One simple
way is with a frequency distribution, which is useful for data that have values occurring with varying
frequencies. For example, the 20 numbers
−4
−1

0
−5

0
0

−3
−2

−2
0

−1
−5

−1
−2

0
0

−1
0

−4
−1

are displayed on the next page in a frequency distribution by listing each different value x and the
frequency f with which x occurs.

109

GMAT® Official Guide 2018

Data Value
x

Frequency
f

−5

2

−4

2

−3

1

−2

3

−1

5

0

7

Total

20

From the frequency distribution, one can readily compute descriptive statistics:
Mean: =

( −5) ( 2) + ( −4 )( 2) + ( −3)(1) + ( −2)( 3) + ( −1) ( 5) + ( 0 )( 7 ) = −1.6
20

Median: −1 (the average of the 10th and 11th numbers)
Mode: 0 (the number that occurs most frequently)
Range: 0 − (−5) = 5
Standard deviation:

( −5 + 1.6)2 ( 2) + ( −4 + 1.6)2 ( 2 ) +…+ ( 0 + 1.6)2 (7 ) ≈ 1.7
20

9. Sets
In mathematics a set is a collection of numbers or other objects. The objects are called the elements of the
set. If S is a set having a finite number of elements, then the number of elements is denoted by |S|. Such
a set is often defined by listing its elements; for example, S = {−5, 0, l} is a set with |S| = 3.
The order in which the elements are listed in a set does not matter; thus {−5, 0, l} = {0, 1, − 5}.
If all the elements of a set S are also elements of a set T, then S is a subset of T; for example,
S = {−5, 0, 1} is a subset of T = {−5, 0, 1, 4, 10}.
For any two sets A and B, the union of A and B is the set of all elements that are in A or in B or in both.
The intersection of A and B is the set of all elements that are both in A and in B. The union is denoted
by A ∪ B and the intersection is denoted by A ∩ B. As an example, if A = {3, 4} and B = {4, 5, 6}, then
A ∪ B = {3, 4, 5, 6} and A ∩ B = {4}. Two sets that have no elements in common are said to be disjoint
or mutually exclusive.

110

4.1 Math Review Arithmetic

The relationship between sets is often illustrated with a Venn diagram in which sets are represented by
regions in a plane. For two sets S and T that are not disjoint and neither is a subset of the other, the
intersection S ∩ T is represented by the shaded region of the diagram below.
S

T

This diagram illustrates a fact about any two finite sets S and T: the number of elements in their
union equals the sum of their individual numbers of elements minus the number of elements in their
intersection (because the latter are counted twice in the sum); more concisely,
| S ∪ T | = | S | + | T | − | S ∩ T |.
This counting method is called the general addition rule for two sets. As a special case, if S and T are
disjoint, then
| S ∪ T | = | S | + |T |
since | S ∩ T | = 0.

10. Counting Methods
There are some useful methods for counting objects and sets of objects without actually listing the
elements to be counted. The following principle of multiplication is fundamental to these methods.
If an object is to be chosen from a set of m objects and a second object is to be chosen from a different
set of n objects, then there are mn ways of choosing both objects simultaneously.
As an example, suppose the objects are items on a menu. If a meal consists of one entree and one dessert
and there are 5 entrees and 3 desserts on the menu, then there are 5 × 3 = 15 different meals that can
be ordered from the menu. As another example, each time a coin is flipped, there are two possible
outcomes, heads and tails. If an experiment consists of 8 consecutive coin flips, then the experiment has
28 possible outcomes, where each of these outcomes is a list of heads and tails in some order.
A symbol that is often used with the multiplication principle is the factorial. If n is an integer greater
than 1, then n factorial, denoted by the symbol n!, is defined as the product of all the integers from 1 to
n. Therefore,
2! = (1)(2) = 2,
3! = (1)(2)(3) = 6,
4! = (1)(2)(3)(4) = 24, etc.
Also, by definition, 0! = 1! = 1.
The factorial is useful for counting the number of ways that a set of objects can be ordered. If a set of
n objects is to be ordered from 1st to nth, then there are n choices for the 1st object, n − 1 choices for
the 2nd object, n − 2 choices for the 3rd object, and so on, until there is only 1 choice for the nth object.
111

GMAT® Official Guide 2018

Thus, by the multiplication principle, the number of ways of ordering the n objects is
n(n − 1)(n − 2)(3)(2)(1) = n !.
For example, the number of ways of ordering the letters A, B, and C is 3!, or 6:
ABC, ACB, BAC, BCA, CAB, and CBA.
These orderings are called the permutations of the letters A, B, and C.
A permutation can be thought of as a selection process in which objects are selected one by one in a
certain order. If the order of selection is not relevant and only k objects are to be selected from a larger
set of n objects, a different counting method is employed.
Specifically, consider a set of n objects from which a complete selection of k objects is to be made
without regard to order, where 0 ≤ k ≤ n. Then the number of possible complete selections of k objects is
called the number of combinations of n objects taken k at a time and is denoted by ⎛ n ⎞ .
⎝ k⎠
n!
.
The value of ⎛ n ⎞ is given by ⎛ n ⎞ =
⎝ k ⎠ k ! (n − k ) !
⎝ k⎠
Note that ⎛ n ⎞ is the number of k‐element subsets of a set with n elements. For example,
⎝ k⎠
if S = {A, B, C, D, E}, then the number of 2‐element subsets of S, or the number of combinations of
5 letters taken 2 at a time, is ⎛ 5 ⎞ = 5! = 120 = 10 .
⎝ 2 ⎠ 2! 3! ( 2 )( 6 )
The subsets are {A, B}, {A, C}, {A, D}, {A, E}, {B, C}, {B, D}, {B, E}, {C, D}, {C, E}, and {D, E}. Note
that ⎛ 5 ⎞ = 10 = ⎛ 5 ⎞ because every 2‐element subset chosen from a set of 5 elements corresponds to a
⎝ 2⎠
⎝ 3⎠
unique 3‐element subset consisting of the elements not chosen.
In general, ⎛ n ⎞ = ⎛ n ⎞ .
⎝ k⎠ ⎝ n − k⎠

11. Discrete Probability
Many of the ideas discussed in the preceding three topics are important to the study of discrete
probability. Discrete probability is concerned with experiments that have a finite number of outcomes.
Given such an experiment, an event is a particular set of outcomes. For example, rolling a number cube
with faces numbered 1 to 6 (similar to a 6‐sided die) is an experiment with 6 possible outcomes:
1, 2, 3, 4, 5, or 6. One event in this experiment is that the outcome is 4, denoted {4}; another event is
that the outcome is an odd number: {1, 3, 5}.
The probability that an event E occurs, denoted by P (E), is a number between 0 and 1, inclusive.
If E has no outcomes, then E is impossible and P (E) = 0; if E is the set of all possible outcomes of the
experiment, then E is certain to occur and P (E) = 1. Otherwise, E is possible but uncertain, and
0 < P (E) < 1. If F is a subset of E, then P (F) ≤ P (E). In the example above, if the probability of each of
the 6 outcomes is the same, then the probability of each outcome is 1 , and the outcomes are said to be
6

112

4.2 Math Review Algebra

equally likely. For experiments in which all the individual outcomes are equally likely, the probability of
an event E is
P (E ) =

The number of outcomes in E
.
The total number of possible outcomes

In the example, the probability that the outcome is an odd number is
P ( {1, 3, 5} ) =

|{1, 3, 5}| 3 1
= = .
6
6 2

Given an experiment with events E and F, the following events are defined:
“not E ” is the set of outcomes that are not outcomes in E;
“E or F ” is the set of outcomes in E or F or both, that is, E ∪ F;
“E and F ” is the set of outcomes in both E and F, that is, E ∩ F.
The probability that E does not occur is P (not E) = l − P(E). The probability that “E or F ” occurs is
P (E or F ) = P (E) + P ( F ) − P (E and F ), using the general addition rule at the end of section 4.1.9
(“Sets”). For the number cube, if E is the event that the outcome is an odd number, {1, 3, 5}, and F is
the event that the outcome is a prime number, {2, 3, 5}, then P ( E and F ) = P ( {3, 5} ) = 2 = 1 and so
6 3
3
3
2
4
2
.
P ( E or F ) = P ( E ) + P ( F ) − P ( E and F ) = + − = =
6 6 6 6 3
|{1, 2, 3, 5}| 4 2
= = .
Note that the event “E or F ” is E ∪ F = {1, 2, 3, 5}, and hence P ( E or F ) =
6
6 3
If the event “E and F ” is impossible (that is, E ∩ F has no outcomes), then E and F are said to be
mutually exclusive events, and P(E and F ) = 0. Then the general addition rule is reduced to
P (E or F ) = P (E) + P (F ).
This is the special addition rule for the probability of two mutually exclusive events.
Two events A and B are said to be independent if the occurrence of either event does not alter the
probability that the other event occurs. For one roll of the number cube, let A = {2, 4, 6} and let
| A| 3 1
= = , while, presuming B occurs, the
B = {5, 6}. Then the probability that A occurs is P ( A ) =
6
6 2
probability that A occurs is
| A ∩ B | |{6}|
=
= 1.
|B|
|{5, 6}| 2
|B| 2 1
= = , while, presuming A occurs, the probability
Similarly, the probability that B occurs is P ( B ) =
6
6 3
that B occurs is
|B ∩ A|
|{6}|
=
= 1.
| A|
|{2, 4, 6}| 3
Thus, the occurrence of either event does not affect the probability that the other event occurs.
Therefore, A and B are independent.

113

GMAT® Official Guide 2018

The following multiplication rule holds for any independent events E and F: P (E and F ) = P (E)P (F ).
For the independent events A and B above, P ( A and B ) = P ( A )P ( B ) = ⎛ 1 ⎞ ⎛ 1 ⎞ = ⎛ 1 ⎞ .
⎝ 2⎠ ⎝ 3⎠ ⎝ 6⎠
Note that the event “A and B” is A ∩ B = {6}, and hence P ( A and B ) = P ( {6} ) = 1 . It follows from the
6
general addition rule and the multiplication rule above that if E and F are independent, then
P ( E or F ) = P ( E ) + P ( F ) − P ( E )P ( F ).
For a final example of some of these rules, consider an experiment with events A, B, and C for which
P (A) = 0.23, P (B) = 0.40, and P(C) = 0.85. Also, suppose that events A and B are mutually exclusive and
events B and C are independent. Then
P ( A or B ) = P ( A ) + P ( B ) (since A or B are mutually exclusive)
= 0.23 + 0.40
= 0.63
P ( B or C ) = P ( B ) + P (C ) − P ( B )P (C ) (by independence)
= 0.40 + 0.85 − (0.40)(0.85)
= 0.91
Note that P (A or C) and P (A and C) cannot be determined using the information given. But it can be
determined that A and C are not mutually exclusive since P (A ) + P (C) = 1.08, which is greater than 1,
and therefore cannot equal P (A or C ); from this it follows that P (A and C) ≥ 0.08. One can also deduce
that P (A and C) ≤ P (A) = 0.23, since A ∩ C is a subset of A, and that P (A or C) ≥ P (C) = 0.85 since C
is a subset of A ∪ C. Thus, one can conclude that 0.85 ≤ P (A or C) ≤ 1 and 0.08 ≤ P (A and C) ≤ 0.23.

4.2 Algebra
Algebra is based on the operations of arithmetic and on the concept of an unknown quantity, or
variable. Letters such as x or n are used to represent unknown quantities. For example, suppose Pam
has 5 more pencils than Fred. If F represents the number of pencils that Fred has, then the number of
pencils that Pam has is F + 5. As another example, if Jim’s present salary S is increased by 7%, then his
new salary is 1.07S. A combination of letters and arithmetic operations, such as
2
F + 5, 3x , and 19x2 − 6x + 3, is called an algebraic expression.
2x − 5
The expression 19x2 − 6x + 3 consists of the terms 19x2, −6x, and 3, where 19 is the coefficient of x2,
−6 is the coefficient of x1, and 3 is a constant term (or coefficient of x0 = 1). Such an expression is called
a second degree (or quadratic) polynomial in x since the highest power of x is 2. The expression F + 5 is a
2
first degree (or linear) polynomial in F since the highest power of F is 1. The expression 3x
is not a
2x − 5
polynomial because it is not a sum of terms that are each powers of x multiplied by coefficients.

114

4.2 Math Review Algebra

1. Simplifying Algebraic Expressions
Often when working with algebraic expressions, it is necessary to simplify them by factoring or
combining like terms. For example, the expression 6x + 5x is equivalent to (6 + 5)x, or 11x.
In the expression 9x − 3y, 3 is a factor common to both terms: 9x − 3y = 3(3x − y). In the expression
5x2 + 6y, there are no like terms and no common factors.
If there are common factors in the numerator and denominator of an expression, they can be divided
out, provided that they are not equal to zero.
For example, if x ≠ 3, then

x−3
is equal to 1; therefore,
x−3
3xy − 9 y 3 y ( x − 3)
=
x−3
x−3
= ( 3 y ) (1)
= 3y

To multiply two algebraic expressions, each term of one expression is multiplied by each term of the
other expression. For example:
(3x − 4)(9 y + x ) = 3x(9 y + x ) − 4(9 y + x )
= ( 3x ) (9 y ) + ( 3x )( x ) + ( −4 ) (9 y ) + ( −4) ( x )
= 27 xy + 3x 2 − 36 y − 4 x
An algebraic expression can be evaluated by substituting values of the unknowns in the expression. For
example, if x = 3 and y = −2, then 3xy − x2 + y can be evaluated as
3(3)( −2) − (3)2 + ( −2) = −18 − 9 − 2 = −29

2. Equations
A major focus of algebra is to solve equations involving algebraic expressions. Some examples of such
equations are
5x − 2 = 9 − x (a linear equation with one unknown)
3x + 1 = y − 2 (a linear equation with two unknowns)
5x 2 + 3x − 2 = 7 x
x ( x − 3) ( x 2 + 5)
=0
x−4

(a quadratic equation with one unknown)
(an equation that is factored on one side with 0 on the other)

The solutions of an equation with one or more unknowns are those values that make the equation true,
or “satisfy the equation,” when they are substituted for the unknowns of the equation. An equation
may have no solution or one or more solutions. If two or more equations are to be solved together, the
solutions must satisfy all the equations simultaneously.

115

GMAT® Official Guide 2018

Two equations having the same solution(s) are equivalent equations. For example, the equations
2+x = 3
4 + 2x = 6
each have the unique solution x = 1. Note that the second equation is the first equation multiplied by 2.
Similarly, the equations
3x − y = 6
6x − 2 y = 12
have the same solutions, although in this case each equation has infinitely many solutions. If any value is
assigned to x, then 3x − 6 is a corresponding value for y that will satisfy both equations; for example,
x = 2 and y = 0 is a solution to both equations, as is x = 5 and y = 9.

3. Solving Linear Equations with One Unknown
To solve a linear equation with one unknown (that is, to find the value of the unknown that satisfies the
equation), the unknown should be isolated on one side of the equation. This can be done by performing
the same mathematical operations on both sides of the equation. Remember that if the same number
is added to or subtracted from both sides of the equation, this does not change the equality; likewise,
multiplying or dividing both sides by the same nonzero number does not change the equality. For
example, to solve the equation 5x − 6 = 4 for x, the variable x can be isolated using the following steps:
3
5x − 6 = 12 (multiplying by 3)
5x = 18 (adding 6)
x = 18 (dividing by 5)
5
The solution, 18 , can be checked by substituting it for x in the original equation to determine whether
5
it satisfies that equation:
5 ⎛ 18 ⎞ − 6
⎝ 5⎠
= 18 − 6 = 12 = 4
3
3
3
Therefore, x = 18 is the solution.
5

4. Solving Two Linear Equations with Two Unknowns
For two linear equations with two unknowns, if the equations are equivalent, then there are infinitely
many solutions to the equations, as illustrated at the end of section 4.2.2 (“Equations”). If the equations
are not equivalent, then they have either one unique solution or no solution. The latter case is illustrated
by the two equations:
3x + 4y = 17
6x + 8 y = 35

116

4.2 Math Review Algebra

Note that 3x + 4y = 17 implies 6x + 8y = 34, which contradicts the second equation. Thus, no values of x
and y can simultaneously satisfy both equations.
There are several methods of solving two linear equations with two unknowns. With any method, if
a contradiction is reached, then the equations have no solution; if a trivial equation such as 0 = 0 is
reached, then the equations are equivalent and have infinitely many solutions. Otherwise, a unique
solution can be found.
One way to solve for the two unknowns is to express one of the unknowns in terms of the other using
one of the equations, and then substitute the expression into the remaining equation to obtain an
equation with one unknown. This equation can be solved and the value of the unknown substituted into
either of the original equations to find the value of the other unknown. For example, the following two
equations can be solved for x and y.
(1) 3x + 2 y = 11
(2)

x− y = 2

In equation (2), x = 2 + y. Substitute 2 + y in equation (1) for x:
3(2 + y ) + 2 y = 11
6 + 3 y + 2 y = 11
6 + 5 y = 11
5y = 5
y =1
If y = 1, then x − 1 = 2 and x = 2 + 1 = 3.
There is another way to solve for x and y by eliminating one of the unknowns. This can be done by making
the coefficients of one of the unknowns the same (disregarding the sign) in both equations and either
adding the equations or subtracting one equation from the other. For example, to solve the equations
(1) 6x + 5 y = 29
(2) 4 x − 3 y = −6
by this method, multiply equation (1) by 3 and equation (2) by 5 to get
18x + 15 y = 87
20 x − 15 y = −30
Adding the two equations eliminates y, yielding 38x = 57, or x = 3 . Finally, substituting 3 for x in one
2
2
of the equations gives y = 4. These answers can be checked by substituting both values into both of the
original equations.

117

GMAT® Official Guide 2018

5. Solving Equations by Factoring
Some equations can be solved by factoring. To do this, first add or subtract expressions to bring all the
expressions to one side of the equation, with 0 on the other side. Then try to factor the nonzero side into
a product of expressions. If this is possible, then using property (7) in section 4.1.4 (“Real Numbers”)
each of the factors can be set equal to 0, yielding several simpler equations that possibly can be solved.
The solutions of the simpler equations will be solutions of the factored equation. As an example,
consider the equation x3 − 2x2 + x = −5(x − l)2:
x 3 − 2x 2 + x + 5 ( x − 1) = 0
2

x ( x 2 − 2x + 1) + 5 ( x − 1) = 0
2

x ( x − 1) + 5 ( x − 1) = 0
2

2

( x + 5 ) ( x − 1)2 = 0
2
x + 5 = 0 or ( x − 1) = 0
x = −5 or x = 1.
For another example, consider

x ( x − 3) ( x 2 + 5 )
x−4

equals 0. Thus, x(x − 3)(x2 + 5) = 0:

= 0. A fraction equals 0 if and only if its numerator

x = 0 or x − 3 = 0 or x2 + 5 = 0
x = 0 or x = 3 or x2 + 5 = 0.
But x 2 + 5 = 0 has no real solution because x2 + 5 > 0 for every real number. Thus, the solutions are 0
and 3.
The solutions of an equation are also called the roots of the equation. These roots can be checked by
substituting them into the original equation to determine whether they satisfy the equation.

6. Solving Quadratic Equations
The standard form for a quadratic equation is
ax2 + bx + c = 0,
where a, b, and c are real numbers and a ≠ 0; for example:
x 2 + 6x + 5 = 0
3x 2 − 2x = 0, and
x2 + 4 = 0
Some quadratic equations can easily be solved by factoring. For example:
(1)

x 2 + 6x + 5 = 0
( x + 5)( x + 1) = 0
x + 5 = 0 or x + 1 = 0
x = −5 or x = −1

118

4.2 Math Review Algebra

3x 2 − 3 = 8x

(2)

3x 2 − 8x − 3 = 0
(3x + 1)( x − 3) = 0
3x + 1 = 0 or x − 3 = 0
x = − 1 or x = 3
3
A quadratic equation has at most two real roots and may have just one or even no real root. For
example, the equation x2 − 6x + 9 = 0 can be expressed as (x − 3)2 = 0, or (x − 3)(x − 3) = 0; thus the
only root is 3. The equation x2 + 4 = 0 has no real root; since the square of any real number is greater
than or equal to zero, x2 + 4 must be greater than zero.
An expression of the form a2 − b2 can be factored as (a − b)(a + b).
For example, the quadratic equation 9x2 − 25 = 0 can be solved as follows.

( 3x − 5)( 3x + 5) = 0
3x − 5 = 0 or 3x + 5 = 0
x = 5 or x = − 5
3
3
If a quadratic expression is not easily factored, then its roots can always be found using the quadratic
formula: If ax2 + bx + c = 0 (a ≠ 0), then the roots are
2
2
x = − b + b − 4 ac and x = − b − b − 4 ac
2a
2a

These are two distinct real numbers unless b2 − 4ac ≤ 0. If b2 − 4ac = 0, then these two expressions for
x are equal to − b , and the equation has only one root. If b2 − 4ac < 0, then b 2 − 4 ac is not a real
2a
number and the equation has no real roots.

7. Exponents
A positive integer exponent of a number or a variable indicates a product, and the positive integer is
the number of times that the number or variable is a factor in the product. For example, x5 means
(x)(x)(x)(x)(x); that is, x is a factor in the product 5 times.
Some rules about exponents follow.
Let x and y be any positive numbers, and let r and s be any positive integers.
(1) (xr)(xs) = x(r + s); for example, (22)(23) = 2(2 + 3) = 25 = 32.
x r = x (r − s ); for example, 4 5 = 4 5 − 2 = 4 3 = 64.
xs
42
(3) (xr)(yr) = (xy)r; for example, (33)(43) = 123 = 1,728.
r
3
r
3
⎛ x⎞
(4) ⎜ ⎟ = x r ; for example, ⎛ 2 ⎞ = 23 = 8 .
⎝ 3⎠
y
3
27
⎝ y⎠

(2)

119

GMAT® Official Guide 2018

(5) (xr)s = xrs = (xs)r; for example, (x3)4 = x12 = (x4)3.
(6) x−r= 1r ; for example, 3−2 = 12 = 1 .
3
9
x
0
0
(7) x = 1; for example, 6 = 1.
r
2
r
2
1
1
1
⎛ 1⎞
⎛ 1⎞
(8) x s = ⎜ x s ⎟ = ( x r ) s = s x r ; for example, 8 3 = ⎜ 8 3 ⎟ = (8 2 ) 3 = 3 8 2 = 3 64 = 4 and 9 2 = 9 = 3.
⎝ ⎠
⎝ ⎠
It can be shown that rules 1 − 6 also apply when r and s are not integers and are not positive, that is,
when r and s are any real numbers.

8. Inequalities
An inequality is a statement that uses one of the following symbols:
≠ not equal to
> greater than
≥ greater than or equal to
< less than
≤ less than or equal to
1
< 3 . Solving a linear inequality with one
2
4
unknown is similar to solving an equation; the unknown is isolated on one side of the inequality. As in
solving an equation, the same number can be added to or subtracted from both sides of the inequality,
or both sides of an inequality can be multiplied or divided by a positive number without changing the
truth of the inequality. However, multiplying or dividing an inequality by a negative number reverses the
order of the inequality. For example, 6 > 2, but (−1)(6) < (−1)(2).

Some examples of inequalities are 5x − 3 < 9, 6x ≥ y, and

To solve the inequality 3x − 2 > 5 for x, isolate x by using the following steps:
3x − 2 > 5
3x > 7 (adding 2 to both sides)
x > 7 (dividing both sides by 3)
3
To solve the inequality 5x − 1 < 3 for x, isolate x by using the following steps:
−2
5x − 1 < 3
−2
5x − 1 > −6 (multiplying both sides by − 2)
5x > −5
(adding 1 to both sides)
x > −1
(dividing both sides by 5)

120

4.3 Math Review Geometry

9. Absolute Value
The absolute value of x, denoted |x|, is defined to be x if x ≥ 0 and −x if x < 0. Note that
the nonnegative square root of x2, and so x 2 = |x|.

x 2 denotes

10. Functions
An algebraic expression in one variable can be used to define a function of that variable. A function is
denoted by a letter such as f or g along with the variable in the expression. For example, the expression
x3 − 5x2 + 2 defines a function f that can be denoted by
f ( x ) = x 3 − 5x 2 + 2.
The expression

2z + 7
defines a function g that can be denoted by
z +1
g ( z ) = 2z + 7 .
z +1

The symbols “f (x)” or “g (z)” do not represent products; each is merely the symbol for an expression, and
is read “f of x” or “g of z.”
Function notation provides a short way of writing the result of substituting a value for a variable. If x = 1 is
substituted in the first expression, the result can be written f (1) = −2, and f (1) is called the “value of f at
x = 1.” Similarly, if z = 0 is substituted in the second expression, then the value of g at z = 0 is g(0) = 7.
Once a function f (x) is defined, it is useful to think of the variable x as an input and f (x) as the
corresponding output. In any function there can be no more than one output for any given input. However,
more than one input can give the same output; for example, if h (x) = |x + 3|, then h (−4) = 1 = h (−2).
The set of all allowable inputs for a function is called the domain of the function. For f and g defined
above, the domain of f is the set of all real numbers and the domain of g is the set of all numbers greater
than −1. The domain of any function can be arbitrarily specified, as in the function defined by “h (x) = 9x
− 5 for 0 ≤ x ≤ 10.” Without such a restriction, the domain is assumed to be all values of x that result in
a real number when substituted into the function.
The domain of a function can consist of only the positive integers and possibly 0. For example,
a(n ) = n 2 + n for n = 0,1, 2, 3, . . . . .
5
Such a function is called a sequence and a(n) is denoted by an. The value of the sequence an at n = 3 is
a3 = 32 + 3 = 9.60. As another example, consider the sequence defined by bn = (−1)n (n!) for n = 1, 2, 3, .
5
. . . A sequence like this is often indicated by listing its values in the order b1, b2, b3, . . ., bn, . . . as follows:
−1, 2, −6, . . . , (−1)n(n!), . . . , and (−1)n(n!) is called the nth term of the sequence.

121

GMAT® Official Guide 2018

4.3 Geometry
1. Lines
In geometry, the word “line” refers to a straight line that extends without end in both directions.
Q

P

The line above can be referred to as line PQ or line  . The part of the line from P to Q is called a line
segment. P and Q are the endpoints of the segment. The notation PQ is used to denote line segment PQ
and PQ is used to denote the length of the segment.

2. Intersecting Lines and Angles
If two lines intersect, the opposite angles are called vertical angles and have the same measure. In the figure
Q
P

S

y°
R
x° y° x°

T

∠PRQ and ∠SRT are vertical angles and ∠QRS and ∠PRT are vertical angles. Also, x + y = 180° since
PRS is a straight line.

3. Perpendicular Lines
An angle that has a measure of 90° is a right angle. If two lines intersect at right angles, the lines are
perpendicular. For example:
1
2

1 and 2 above are perpendicular, denoted by 1 ⊥ 2. A right angle symbol in an angle of intersection
indicates that the lines are perpendicular.

4. Parallel Lines
If two lines that are in the same plane do not intersect, the two lines are parallel. In the figure
1
2

122

4.3 Math Review Geometry

lines 1 and 2 are parallel, denoted by 1 2. If two parallel lines are intersected by a third line, as
shown below, then the angle measures are related as indicated, where x + y = 180°.
y° x°
x° y°
y° x°
x° y°

1
2

5. Polygons (Convex)
A polygon is a closed plane figure formed by three or more line segments, called the sides of the polygon.
Each side intersects exactly two other sides at their endpoints. The points of intersection of the sides
are vertices. The term “polygon” will be used to mean a convex polygon, that is, a polygon in which each
interior angle has a measure of less than 180°.
The following figures are polygons:

The following figures are not polygons:

A polygon with three sides is a triangle; with four sides, a quadrilateral; with five sides, a pentagon; and
with six sides, a hexagon.
The sum of the interior angle measures of a triangle is 180°. In general, the sum of the interior angle
measures of a polygon with n sides is equal to (n − 2)180°. For example, this sum for a pentagon is
(5 − 2)180° = (3)180° = 540°.

Note that a pentagon can be partitioned into three triangles and therefore the sum of the angle
measures can be found by adding the sum of the angle measures of three triangles.
The perimeter of a polygon is the sum of the lengths of its sides.

123

GMAT® Official Guide 2018

The commonly used phrase “area of a triangle” (or any other plane figure) is used to mean the area of the
region enclosed by that figure.

6. Triangles
There are several special types of triangles with important properties. But one property that all triangles
share is that the sum of the lengths of any two of the sides is greater than the length of the third side, as
illustrated below.
y

x

z
x + y > z, x + z > y, and y + z > x

An equilateral triangle has all sides of equal length. All angles of an equilateral triangle have equal
measure. An isosceles triangle has at least two sides of the same length. If two sides of a triangle have the
same length, then the two angles opposite those sides have the same measure. Conversely, if two angles
of a triangle have the same measure, then the sides opposite those angles have the same length. In
isosceles triangle PQR below, x = y since PQ = QR.
Q

P

10

10

x°

y°

R

A triangle that has a right angle is a right triangle. In a right triangle, the side opposite the right angle is
the hypotenuse, and the other two sides are the legs. An important theorem concerning right triangles is
the Pythagorean theorem, which states: In a right triangle, the square of the length of the hypotenuse is
equal to the sum of the squares of the lengths of the legs.
S

6
R

8

T

In the figure above, ΔRST is a right triangle, so (RS)2 + (RT )2 = (ST )2. Here, RS = 6 and RT = 8, so
ST = 10, since 62 + 82 = 36 + 64 = 100 = (ST )2 and ST = 100. Any triangle in which the lengths of
the sides are in the ratio 3:4:5 is a right triangle. In general, if a, b, and c are the lengths of the sides of a
triangle and a 2 + b 2 = c 2, then the triangle is a right triangle.

124

4.3 Math Review Geometry

Y

K

30º

45º

J

45º

L

2

60º

X

3

Z

In 45°− 45°− 90° triangles, the lengths of the sides are in the ratio 1:1: 2 . For example, in ΔJKL,
if JL = 2, then JK = 2 and KL = 2 2. In 30°− 60°− 90° triangles, the lengths of the sides are in the ratio
1: 3:2. For example, in ΔXYZ, if XZ = 3, then XY = 3 3 and YZ = 6.
The altitude of a triangle is the segment drawn from a vertex perpendicular to the side opposite that
vertex. Relative to that vertex and altitude, the opposite side is called the base.
The area of a triangle is equal to:
(the length of the altitude) × (the length of the base)
2
B
E

A

D
8
BD = 5

C

In ΔABC, BD is the altitude to base AC and AE is the altitude to base BC . The area of ΔABC is
equal to
BD × AC = 5 × 8 = 20.
2
2
AE
×
BC
. If ΔABC above is isosceles and AB = BC, then altitude BD bisects
The area is also equal to
2
the base; that is, AD = DC = 4. Similarly, any altitude of an equilateral triangle bisects the side to which
it is drawn.
E
6 30º
D

60º

G

F

In equilateral triangle DEF, if DE = 6, then DG = 3 and EG = 3 3. The area of ΔDEF is equal to
3 3 × 6 = 9 3.
2
125

GMAT® Official Guide 2018

7. Quadrilaterals
A polygon with four sides is a quadrilateral. A quadrilateral in which both pairs of opposite sides are
parallel is a parallelogram. The opposite sides of a parallelogram also have equal length.
K

L
4

N

J

M

6

In parallelogram JKLM, JK  LM and JK = LM ; KL  JM and KL = JM.
The diagonals of a parallelogram bisect each other (that is, KN = NM and JN = NL).
The area of a parallelogram is equal to
(the length of the altitude) × (the length of the base).
The area of JKLM is equal to 4 × 6 = 24.
A parallelogram with right angles is a rectangle, and a rectangle with all sides of equal length is a square.
X

Y

3
W

Z

7

The perimeter of WXYZ = 2(3) + 2(7) = 20 and the area of WXYZ is equal to 3 × 7 = 21. The diagonals
of a rectangle are equal; therefore WY = XZ = 9 + 49 = 58 .
Q

12

R
8

P

16

S

A quadrilateral with two sides that are parallel, as shown above, is a trapezoid. The area of trapezoid
PQRS may be calculated as follows:
1 (the sum of the lengths of the bases)(the height) = 1 (QR + PS )(8) = 1 (28 × 8) = 112.
2
2
2

126

4.3 Math Review Geometry

8. Circles
A circle is a set of points in a plane that are all located the same distance from a fixed point (the center of
the circle).
A chord of a circle is a line segment that has its endpoints on the circle. A chord that passes through the
center of the circle is a diameter of the circle. A radius of a circle is a segment from the center of the circle to a
point on the circle. The words “diameter” and “radius” are also used to refer to the lengths of these segments.
The circumference of a circle is the distance around the circle. If r is the radius of the circle, then the
circumference is equal to 2π r, where π is approximately 22 or 3.14. The area of a circle of radius r is
7
equal to π r2.
K

J

O

P

7

R

In the circle above, O is the center of the circle and JK and PR are chords. PR is a diameter and OR
is a radius. If OR = 7, then the circumference of the circle is 2π (7) = 14π and the area of the circle is
π (7)2 = 49π .
The number of degrees of arc in a circle (or the number of degrees in a complete revolution) is 360.
R
O xº

S
T

In the circle with center O above, the length of arc RST is x of the circumference of the circle; for
360
1
of the circumference of the circle.
example, if x = 60, then arc RST has length
6
A line that has exactly one point in common with a circle is said to be tangent to the circle, and that
common point is called the point of tangency. A radius or diameter with an endpoint at the point of
tangency is perpendicular to the tangent line, and, conversely, a line that is perpendicular to a radius or
diameter at one of its endpoints is tangent to the circle at that endpoint.

O

T

127

GMAT® Official Guide 2018

The line  above is tangent to the circle and radius OT is perpendicular to .
If each vertex of a polygon lies on a circle, then the polygon is inscribed in the circle and the circle is
circumscribed about the polygon. If each side of a polygon is tangent to a circle, then the polygon is
circumscribed about the circle and the circle is inscribed in the polygon.
Q

C

R

D

B
P

S

E
A

F

In the figure above, quadrilateral PQRS is inscribed in a circle and hexagon ABCDEF is circumscribed
about a circle.
If a triangle is inscribed in a circle so that one of its sides is a diameter of the circle, then the triangle is
a right triangle.
Y

X

Z

O

In the circle above, XZ is a diameter and the measure of ∠XYZ is 90°.

9. Rectangular Solids and Cylinders
A rectangular solid is a three‐dimensional figure formed by 6 rectangular surfaces, as shown below. Each
rectangular surface is a face. Each solid or dotted line segment is an edge, and each point at which the
edges meet is a vertex. A rectangular solid has 6 faces, 12 edges, and 8 vertices. Opposite faces are parallel
rectangles that have the same dimensions. A rectangular solid in which all edges are of equal length is a cube.
The surface area of a rectangular solid is equal to the sum of the areas of all the faces. The volume is equal to
(length) × (width) × (height);
in other words, (area of base) × (height).
U
R

Q
4
P

128

V

T
8
3

W

S

In the rectangular solid above, the dimensions are 3, 4, and 8. The surface area is equal to
2(3 × 4) + 2(3 × 8) + 2(4 × 8) = 136. The volume is equal to 3 × 4 × 8 = 96.

4.3 Math Review Geometry

P

O
The figure above is a right circular cylinder. The two bases are circles of the same size with centers O and
P, respectively, and altitude (height) OP is perpendicular to the bases. The surface area of a right circular
cylinder with a base of radius r and height h is equal to 2(π r2) + 2π rh (the sum of the areas of the two
bases plus the area of the curved surface).
The volume of a cylinder is equal to π r2h, that is,
(area of base) × (height).

8
5

In the cylinder above, the surface area is equal to
2(25π ) + 2π (5)(8) = 130π ,
and the volume is equal to
25π (8) = 200π .

10. Coordinate Geometry
y
3
2

II

I

1
–4

–3

–2
III

–1 O
–1
–2

1

2

3

4

x

IV

–3

The figure above shows the (rectangular) coordinate plane. The horizontal line is called the x‐axis and the
perpendicular vertical line is called the y‐axis. The point at which these two axes intersect, designated O,
is called the origin. The axes divide the plane into four quadrants, I, II, III, and IV, as shown.
129

GMAT® Official Guide 2018

Each point in the plane has an x‐coordinate and a y‐coordinate. A point is identified by an ordered pair
(x,y) of numbers in which the x‐coordinate is the first number and the y‐coordinate is the second number.
y
5
4
P

3
2
1
–5 –4 –3 –2 –1 O
–1
–2
Q

1 2

3 4 5 x

–3
–4
–5

In the graph above, the (x,y) coordinates of point P are (2,3) since P is 2 units to the right of the y‐axis
(that is, x = 2) and 3 units above the x‐axis (that is, y = 3). Similarly, the (x,y) coordinates of point Q are
(−4,−3). The origin O has coordinates (0,0).
One way to find the distance between two points in the coordinate plane is to use the Pythagorean
theorem.
y
R

(–2,4)

x

O

Z

S
(3,–3)

To find the distance between points R and S using the Pythagorean theorem, draw the triangle as
shown. Note that Z has (x,y) coordinates (−2,−3), RZ = 7, and ZS = 5. Therefore, the distance between
R and S is equal to
7 2 + 5 2 = 74.
For a line in the coordinate plane, the coordinates of each point on the line satisfy a linear equation of
the form y = mx + b (or the form x = a if the line is vertical). For example, each point on the line on the
next page satisfies the equation y = − 1 x + 1 . One can verify this for the points (−2,2), (2,0), and (0,1)
2
by substituting the respective coordinates for x and y in the equation.
130

4.3 Math Review Geometry

y
(–2,2)

2
1

–3 –2

(0,1)

–1 O
–1

(2,0)
1

2

3

x

In the equation y = mx + b of a line, the coefficient m is the slope of the line and the constant term b
is the y‐intercept of the line. For any two points on the line, the slope is defined to be the ratio of the
difference in the y‐coordinates to the difference in the x‐coordinates. Using (−2, 2) and (2, 0) above, the
slope is
The difference in the y -coordinates
= 0 − 2 = −2 = − 1 .
The difference in the x -coordinates 2 − ( −2 ) 4
2
The y‐intercept is the y‐coordinate of the point at which the line intersects the y‐axis. For the line
above, the y‐intercept is 1, and this is the resulting value of y when x is set equal to 0 in the equation
y = − 1 x + 1 . The x‐intercept is the x‐coordinate of the point at which the line intersects the x‐axis. The
2
x‐intercept can be found by setting y = 0 and solving for x. For the line y = − 1 x + 1 , this gives
2
− 1 x +1 = 0
2
− 1 x = −1
2
x = 2.
Thus, the x‐intercept is 2.
Given any two points (x1,y1) and (x2,y2) with x1 ≠ x2, the equation of the line passing through these
y − y1
, then using a
points can be found by applying the definition of slope. Since the slope is m = 2
x 2 − x1
y − y1
point known to be on the line, say (x1,y1), any point (x,y) on the line must satisfy
= m , or
x − x1
y − y1 = m(x − x1). (Using (x2,y2) as the known point would yield an equivalent equation.) For example,
consider the points (−2,4) and (3,−3) on the line below.

131

GMAT® Official Guide 2018

y
(–2,4)

5
4
3
2
1

–4

–3 –2

–1 O
–1
–2

1

2

–3

3

4 x
(3,–3)

–4
–5

The slope of this line is −3 − 4 = −7 , so an equation of this line can be found using the point (3,−3)
3 − ( −2 ) 5
as follows:
y − ( −3) = − 7 ( x − 3)
5
7
y + 3 = − x + 21
5
5
y =−7x+6
5
5
The y‐intercept is

6
. The x‐intercept can be found as follows:
5
0=−7x+6
5
5
7x= 6
5
5
x=6
7

Both of these intercepts can be seen on the graph.
If the slope of a line is negative, the line slants downward from left to right; if the slope is positive, the
line slants upward. If the slope is 0, the line is horizontal; the equation of such a line is of the form y = b
since m = 0. For a vertical line, slope is not defined, and the equation is of the form x = a, where a is the
x‐intercept.
There is a connection between graphs of lines in the coordinate plane and solutions of two linear
equations with two unknowns. If two linear equations with unknowns x and y have a unique solution,
then the graphs of the equations are two lines that intersect in one point, which is the solution. If the
equations are equivalent, then they represent the same line with infinitely many points or solutions. If
the equations have no solution, then they represent parallel lines, which do not intersect.

132

4.4 Math Review Word Problems

There is also a connection between functions (see section 4.2.10) and the coordinate plane. If a function
is graphed in the coordinate plane, the function can be understood in different and useful ways.
Consider the function defined by
f (x ) = − 7 x + 6 .
5
5
If the value of the function, f (x), is equated with the variable y, then the graph of the function in the
xy‐coordinate plane is simply the graph of the equation
y =−7x+6
5
5
shown above. Similarly, any function f (x) can be graphed by equating y with the value of the function:
y = f (x ).
So for any x in the domain of the function f, the point with coordinates (x, f (x)) is on the graph of f,
and the graph consists entirely of these points.
As another example, consider a quadratic polynomial function defined by f (x)= x2 −1. One can plot
several points (x, f (x)) on the graph to understand the connection between a function and its graph:
x

f (x)

–2

3

–1

0

0

–1

1

0

2

3

(–2,3)

y

(2,3)

3
2

(–1,0)

1

(1,0)

–2 –1 O
1 2 x
–1 (0,–1)
–2

133

GMAT® Official Guide 2018

If all the points were graphed for −2 ≤ x ≤ 2, then the graph would appear as follows.
y
3
2
1
–2 –1 O
–1

1

2

x

–2

The graph of a quadratic function is called a parabola and always has the shape of the curve above,
although it may be upside down or have a greater or lesser width. Note that the roots of the equation
f (x) = x2 −1 = 0 are x = 1 and x = −1; these coincide with the x‐intercepts since x‐intercepts are
found by setting y = 0 and solving for x. Also, the y‐intercept is f (0) = −1 because this is the value of y
corresponding to x = 0. For any function f, the x‐intercepts are the solutions of the equation f (x) = 0
and the y‐intercept is the value f (0).

4.4 Word Problems
Many of the principles discussed in this chapter are used to solve word problems. The following
discussion of word problems illustrates some of the techniques and concepts used in solving such
problems.

1. Rate Problems
The distance that an object travels is equal to the product of the average speed at which it travels and
the amount of time it takes to travel that distance, that is,
Rate × Time = Distance.
Example 1: If a car travels at an average speed of 70 kilometers per hour for 4 hours, how many
kilometers does it travel?
Solution: Since rate × time = distance, simply multiply 70 km/hour × 4 hours. Thus, the car travels
280 kilometers in 4 hours.
To determine the average rate at which an object travels, divide the total distance traveled by the total
amount of traveling time.
Example 2: On a 400‐mile trip, Car X traveled half the distance at 40 miles per hour (mph) and the
other half at 50 mph. What was the average speed of Car X ?
Solution: First it is necessary to determine the amount of traveling time. During the first 200 miles, the
car traveled at 40 mph; therefore, it took 200 = 5 hours to travel the first 200 miles.
40

134

4.4 Math Review Word Problems

During the second 200 miles, the car traveled at 50 mph; therefore, it took 200 = 4 hours to travel the
50
400
4
= 44 mph. Note that the average speed is
second 200 miles. Thus, the average speed of Car X was
9
9
40
+
50
= 45.
not
2
Some rate problems can be solved by using ratios.
Example 3: If 5 shirts cost $44, then, at this rate, what is the cost of 8 shirts?
Solution: If c is the cost of the 8 shirts, then 5 = 8 . Cross multiplication results in the equation
44 c
5c = 8 × 44 = 352
c = 352 = 70.40
5
The 8 shirts cost $70.40.

2. Work Problems
In a work problem, the rates at which certain persons or machines work alone are usually given, and it is
necessary to compute the rate at which they work together (or vice versa).
The basic formula for solving work problems is 1 + 1 = 1 , where r and s are, for example, the number
r s h
of hours it takes Rae and Sam, respectively, to complete a job when working alone, and h is the number
of hours it takes Rae and Sam to do the job when working together. The reasoning is that in 1 hour Rae
does 1 of the job, Sam does 1 of the job, and Rae and Sam together do 1 of the job.
r
s
h
Example 1: If Machine X can produce 1,000 bolts in 4 hours and Machine Y can produce 1,000 bolts in
5 hours, in how many hours can Machines X and Y, working together at these constant rates, produce
1,000 bolts?
Solution:
1+1=1
4 5 h
5 + 4 =1
20 20 h
9 =1
20 h
9h = 20
h = 20 = 2 2
9
9
Working together, Machines X and Y can produce 1,000 bolts in 2 2 hours.
9

135

GMAT® Official Guide 2018

Example 2: If Art and Rita can do a job in 4 hours when working together at their respective constant
rates and Art can do the job alone in 6 hours, in how many hours can Rita do the job alone?
Solution:
1+ 1 =1
6 R 4
R+6 = 1
6R
4
4 R + 24 = 6 R
24 = 2 R
12 = R
Working alone, Rita can do the job in 12 hours.

3. Mixture Problems
In mixture problems, substances with different characteristics are combined, and it is necessary to
determine the characteristics of the resulting mixture.
Example 1: If 6 pounds of nuts that cost $1.20 per pound are mixed with 2 pounds of nuts that cost
$1.60 per pound, what is the cost per pound of the mixture?
Solution: The total cost of the 8 pounds of nuts is
6($1.20) + 2($1.60) = $10.40.
The cost per pound is

$10.40 = $1.30.
8

Example 2: How many liters of a solution that is 15 percent salt must be added to 5 liters of a solution
that is 8 percent salt so that the resulting solution is 10 percent salt?
Solution: Let n represent the number of liters of the 15% solution. The amount of salt in the 15%
solution [0.15n] plus the amount of salt in the 8% solution [(0.08)(5)] must be equal to the amount of
salt in the 10% mixture [0.10(n + 5)]. Therefore,
0.15n + 0.08(5) = 0.10(n + 5)
15n + 40 = 10n + 50
5n = 10
n = 2 liters
Two liters of the 15% salt solution must be added to the 8% solution to obtain the 10% solution.

4. Interest Problems
Interest can be computed in two basic ways. With simple annual interest, the interest is computed on
the principal only and is equal to (principal) × (interest rate) × (time). If interest is compounded, then
interest is computed on the principal as well as on any interest already earned.
136

4.4 Math Review Word Problems

Example 1: If $8,000 is invested at 6 percent simple annual interest, how much interest is earned after
3 months?
Solution: Since the annual interest rate is 6%, the interest for 1 year is

( 0.06)($8, 000 ) = $480.
The interest earned in 3 months is

3 $480 = $120.
(
)
12

Example 2: If $10,000 is invested at 10 percent annual interest, compounded semiannually, what is the
balance after 1 year?
Solution: The balance after the first 6 months would be
10, 000 + (10, 000)(0.05) = $10, 500.
The balance after one year would be

10, 500 + (10, 500)(0.05) = $11, 025.

Note that the interest rate for each 6‐month period is 5%, which is half of the 10% annual rate. The
balance after one year can also be expressed as
2

10, 000 ⎛ 1 + 0.10 ⎞ dollars.
⎝
2 ⎠

5. Discount
If a price is discounted by n percent, then the price becomes (100 − n) percent of the original price.
Example 1: A certain customer paid $24 for a dress. If that price represented a 25 percent discount on
the original price of the dress, what was the original price of the dress?
Solution: If p is the original price of the dress, then 0.75p is the discounted price and 0.75p = $24, or
p = $32. The original price of the dress was $32.
Example 2: The price of an item is discounted by 20 percent and then this reduced price is discounted by
an additional 30 percent. These two discounts are equal to an overall discount of what percent?
Solution: If p is the original price of the item, then 0.8p is the price after the first discount. The price
after the second discount is (0.7)(0.8) p = 0.56p. This represents an overall discount of 44 percent
(l00% − 56%).

6. Profit
Gross profit is equal to revenues minus expenses, or selling price minus cost.
Example: A certain appliance costs a merchant $30. At what price should the merchant sell the
appliance in order to make a gross profit of 50 percent of the cost of the appliance?
Solution: If s is the selling price of the appliance, then s − 30 = (0.5)(30), or s = $45. The merchant
should sell the appliance for $45.

137

GMAT® Official Guide 2018

7. Sets
If S is the set of numbers 1, 2, 3, and 4, you can write S = {l, 2, 3, 4}. Sets can also be represented by
Venn diagrams. That is, the relationship among the members of sets can be represented by circles.
Example 1: Each of 25 people is enrolled in history, mathematics, or both. If 20 are enrolled in history
and 18 are enrolled in mathematics, how many are enrolled in both history and mathematics?
Solution: The 25 people can be divided into three sets: those who study history only, those who study
mathematics only, and those who study history and mathematics. Thus a Venn diagram may be drawn
as follows, where n is the number of people enrolled in both courses, 20 − n is the number enrolled in
history only, and 18 − n is the number enrolled in mathematics only.
History

Mathematics
20 – n

n

18 – n

Since there is a total of 25 people, (20 − n) + n + (18 − n) = 25, or n = 13. Thirteen people are enrolled
in both history and mathematics. Note that 20 + 18 − 13 = 25, which is the general addition rule for
two sets (see section 4.1.9).
Example 2: In a certain production lot, 40 percent of the toys are red and the remaining toys are green.
Half of the toys are small and half are large. If 10 percent of the toys are red and small, and 40 toys are
green and large, how many of the toys are red and large?
Solution: For this kind of problem, it is helpful to organize the information in a table:
Red
Small

Green

10%

50%

Large
Total

Total

50%
40%

60%

100%

The numbers in the table are the percentages given. The following percentages can be computed on the
basis of what is given:
Red

Green

Total

Small

10%

40%

50%

Large

30%

20%

50%

Total

40%

60%

100%

Since 20% of the number of toys (n) are green and large, 0.20n = 40 (40 toys are green and large), or
n = 200. Therefore, 30% of the 200 toys, or (0.3)(200) = 60, are red and large.

138

4.4 Math Review Word Problems

8. Geometry Problems
The following is an example of a word problem involving geometry.

200 meters

Example:

200 meters
The figure above shows an aerial view of a piece of land. If all angles shown are right angles, what is the
perimeter of the piece of land?
Solution: For reference, label the figure as
Q

R
S

T
U

P

V
W

If all the angles are right angles, then QR + ST + UV = PW, and RS + TU + VW = PQ. Hence, the
perimeter of the land is 2PW + 2PQ = 2 × 200 + 2 × 200 = 800 meters.

9. Measurement Problems
Some questions on the GMAT involve metric units of measure, whereas others involve English units of
measure. However, except for units of time, if a question requires conversion from one unit of measure
to another, the relationship between those units will be given.
Example: A train travels at a constant rate of 25 meters per second. How many kilometers does it travel
in 5 minutes? (1 kilometer = 1,000 meters)
Solution: In 1 minute the train travels (25)(60) = 1,500 meters, so in 5 minutes it travels 7,500 meters.
7, 500
, or 7.5 kilometers.
Since 1 kilometer = 1,000 meters, it follows that 7,500 meters equals
1, 000

139

GMAT® Official Guide 2018

10. Data Interpretation
Occasionally a question or set of questions will be based on data provided in a table or graph. Some
examples of tables and graphs are given below.
Example 1:
Population by Age Group
(in thousands)
Age

Population

17 years and under

63,376

18−44 years

86,738

45−64 years

43,845

65 years and over

24,054

How many people are 44 years old or younger?
Solution: The figures in the table are given in thousands. The answer in thousands can be obtained by
adding 63,376 thousand and 86,738 thousand. The result is 150,114 thousand, which is 150,114,000.
Example 2:

30
25
20
15
10
5
0

Precipitation
(right-hand scale)

Temperature
(left-hand scale)
January April

July

October

12
10
8
6
4
2
0

Precipitation in
Centimeters

Temperature in
Degrees Celsius

AVERAGE TEMPERATURE AND
PRECIPITATION IN CITY X

What are the average temperature and precipitation in City X during April?
Solution: Note that the scale on the left applies to the temperature line graph and the one on the right
applies to the precipitation line graph. According to the graph, during April the average temperature is
approximately 14° Celsius and the average precipitation is approximately 8 centimeters.

140

4.4 Math Review Word Problems

Example 3:
DISTRIBUTION OF AL’S WEEKLY NET SALARY

Fuel and
Insurance
for Automobile
10%
Savings
25%

Food
15%

Rent and
Utilities
30%

Other
20%

Al’s weekly net salary is $350. To how many of the categories listed was at least $80 of Al’s weekly net
salary allocated?
Solution: In the circle graph, the relative sizes of the sectors are proportional to their corresponding
values and the sum of the percents given is 100%. Note that 80 is approximately 23%, so at least $80
350
was allocated to each of 2 categories—Rent and Utilities, and Savings—since their allocations are each
greater than 23%.

141

5.0 Problem Solving

142

5.0 Problem Solving

5.0 Problem Solving
The Quantitative section of the GMAT® exam uses problem solving and data sufficiency questions to
gauge your skill level. This chapter focuses on problem solving questions. Remember that quantitative
questions require knowledge of the following:
• Arithmetic
• Elementary algebra
• Commonly known concepts of geometry
Problem solving questions are designed to test your basic mathematical skills and understanding of
elementary mathematical concepts, as well as your ability to reason quantitatively, solve quantitative
problems, and interpret graphic data. The mathematics knowledge required to answer the questions
is no more advanced than what is generally taught in secondary school (or high school) mathematics
classes.
In these questions, you are asked to solve each problem and select the best of the five answer choices
given. Begin by reading the question thoroughly to determine exactly what information is given and
to make sure you understand what is being asked. Scan the answer choices to understand your options.
If the problem seems simple, take a few moments to see whether you can determine the answer. Then,
check your answer against the choices provided.
If you do not see your answer among the choices, or if the problem is complicated, take a closer look at
the answer choices and think again about what the problem is asking. See whether you can eliminate
some of the answer choices and narrow down your options. If you are still unable to narrow the answer
down to a single choice, reread the question. Keep in mind that the answer will be based solely on the
information provided in the question—don’t allow your own experience and assumptions to interfere
with your ability to find the correct answer to the question.
If you find yourself stuck on a question or unable to select the single correct answer, keep in mind that
you have about two minutes to answer each quantitative question. You may run out of time if you take
too long to answer any one question; you may simply need to pick the answer that seems to make the
most sense. Although guessing is generally not the best way to achieve a high GMAT score, making
an educated guess is a good strategy for answering questions you are unsure of. Even if your answer to
a particular question is incorrect, your answers to other questions will allow the test to accurately gauge
your ability level.
The following pages include test‐taking strategies, directions that will apply to questions of this type,
sample questions, an answer key, and explanations for all the problems. These explanations present
problem solving strategies that could be helpful in answering the questions.

143

GMAT® Official Guide 2018

5.1 Test-Taking Strategies
1.

Pace yourself.

Consult the on‐screen timer periodically. Work as carefully as possible, but do not spend
valuable time checking answers or pondering problems that you find difficult.
2.

Use the erasable notepad provided at the test center.

Working a problem out may help you avoid errors in solving the problem. If diagrams or figures
are not presented, it may help to draw your own.
3.

Read each question carefully to determine what is being asked.

For word problems, take one step at a time, reading each sentence carefully and translating the
information into equations or other useful mathematical representations.
4.

Scan the answer choices before attempting to answer a question.

Scanning the answers can prevent you from putting answers in a form that is not given
(e.g., finding the answer in decimal form, such as 0.25, when the choices are given in fractional
1
form, such as ). Also, if the question requires approximations, a shortcut could serve well
4
(e.g., you may be able to approximate 48 percent of a number by using half ).
5.

Don’t waste time trying to solve a problem that is too difficult for you.

Make your best guess and then move on to the next question.

5.2 The Directions
These directions are very similar to those you will see for problem solving questions when you take the
GMAT exam. If you read them carefully and understand them clearly before sitting for the GMAT
exam, you will not need to spend too much time reviewing them once the test begins.
Solve the problem and indicate the best of the answer choices given.
Numbers: All numbers used are real numbers.
Figures: A figure accompanying a problem solving question is intended to provide information useful
in solving the problem. Figures are drawn as accurately as possible. Exceptions will be clearly noted.
Lines shown as straight are straight, and lines that appear jagged are also straight. The positions of
points, angles, regions, etc., exist in the order shown, and angle measures are greater than zero. All
figures lie in a plane unless otherwise indicated.

144

5.2 Problem Solving The Directions

To register for the GMAT exam go to www.mba.com
145

GMAT® Official Guide 2018

5.3 Practice Questions
Solve the problem and indicate the best of the answer choices given.
Numbers: All numbers used are real numbers.
Figures: A figure accompanying a problem solving question is intended to provide information useful
in solving the problem. Figures are drawn as accurately as possible. Exceptions will be clearly
noted. Lines shown as straight are straight, and lines that appear jagged are also straight. The
positions of points, angles, regions, etc., exist in the order shown, and angle measures are
greater than zero. All figures lie in a plane unless otherwise indicated.

1.

2.

3.

146

The price of a coat in a certain store is $500. If the
price of the coat is to be reduced by $150, by what
percent is the price to be reduced?
(A)

10%

(B)

15%

(C)

20%

(D)

25%

(E)

30%

4.

1 1 5 1 1 9
+ − + + −
=
3 2 6 5 4 20
(A)
(B)

On a vacation, Rose exchanged $500.00 for euros
at an exchange rate of 0.80 euro per dollar and
3
spent of the euros she received. If she exchanged
4
the remaining euros for dollars at an exchange rate
of $1.20 per euro, what was the dollar amount she
received?

5.

0
2
15

(C)

2
5

(D)

9
20

(E)

5
6

Bouquets are to be made using white tulips and red
tulips, and the ratio of the number of white tulips to
the number of red tulips is to be the same in each
bouquet. If there are 15 white tulips and 85 red tulips
available for the bouquets, what is the greatest number
of bouquets that can be made using all the tulips
available?

(A)

$60.00

(B)

$80.00

(C)

$100.00

(A)

3

(D)

$120.00

(B)

5

(E)

$140.00

(C)

8

(D)

10

(E)

13

x

x

x

y

y

v

v

x

x

y

w

w

Each of the 12 squares shown is labeled x, y, v, or w.
What is the ratio of the number of these squares labeled
x or y to the number of these squares labeled v or w ?

6.

125% of 5 =
(A)

5.125

(B)

5.25

(C)

6

(A)

1:2

(D)

6.125

(B)

2:3

(E)

6.25

(C)

4:3

(D)

3:2

(E)

2:1

5.3 Problem Solving Practice Questions

7.

8.

Today Rebecca, who is 34 years old, and her daughter,
who is 8 years old, celebrate their birthdays. How
many years will pass before Rebecca’s age is twice her
daughter’s age?
(A)

10

(B)

14

(C)

18

(D)

22

(E)

26

When traveling at a constant speed of 32 miles per hour,
a certain motorboat consumes 24 gallons of fuel
per hour. What is the fuel consumption of this boat at
this speed measured in miles traveled per gallon of fuel?
(A)
(B)
(C)
(D)
(E)

9.

2
3
3
4
4
5
4
3
3
2

12.

A case contains c cartons. Each carton contains
b boxes, and each box contains 100 paper clips.
How many paper clips are contained in 2 cases?
(A)
(B)
(C)

10.

11.

100bc
100b
c
200bc

(D)

200b
c

(E)

200
bc

A technician makes a round‐trip to and from a certain
service center by the same route. If the technician
completes the drive to the center and then completes
10 percent of the drive from the center, what percent
of the round‐trip has the technician completed?
(A)

5%

(B)

10%

(C)

25%

(D)

40%

(E)

55%

13.

Raffle tickets numbered consecutively from 101
through 350 are placed in a box. What is the
probability that a ticket selected at random will have a
number with a hundreds digit of 2 ?
(A)

2
5

(B)

2
7

(C)

33
88

(D)

99
250

(E)

100
249

When Leo imported a certain item, he paid a 7 percent
import tax on the portion of the total value of the item
in excess of $1,000. If the amount of the import tax
that Leo paid was $87.50, what was the total value of
the item?
(A)

$1,600

(B)

$1,850

(C)

$2,250

(D)

$2,400

(E)

$2,750

A collection of 16 coins, each with a face value of
either 10 cents or 25 cents, has a total face value of
$2.35. How many of the coins have a face value of
25 cents?
(A)

3

(B)

5

(C)

7

(D)

9

(E)

11

147

GMAT® Official Guide 2018

14.

15.

16.

17.

148

The numbers of cars sold at a certain dealership on
six of the last seven business days were 4, 7, 2, 8, 3,
and 6, respectively. If the number of cars sold on the
seventh business day was either 2, 4, or 5, for which
of the three values does the average (arithmetic mean)
number of cars sold per business day for the seven
business days equal the median number of cars sold
per day for the seven days?
I.

2

II.

4

III.

5

(A)

II only

(B)

III only

(C)

I and II only

(D)

II and III only

(E)

I, II, and III

If it is assumed that 60 percent of those who receive
a questionnaire by mail will respond and 300 responses
are needed, what is the minimum number of
questionnaires that should be mailed?
(A)

400

(B)

420

(C)

480

(D)

500

(E)

600

18.

19.

20.

If 1 < x < y < z, which of the following has the greatest
value?

A rectangular floor that measures 8 meters by
10 meters is to be covered with carpet squares that
each measure 2 meters by 2 meters. If the carpet
squares cost $12 apiece, what is the total cost for the
number of carpet squares needed to cover the floor?
(A)

$200

(B)

$240

(C)

$480

(D)

$960

(E)

$1,920

If 893 × 78 = p, which of the following is equal to
893 × 79 ?
(A)

p+1

(B)

p + 78

(C)

p + 79

(D)

p + 893

(E)

p + 894

Thabo owns exactly 140 books, and each book is
either paperback fiction, paperback nonfiction, or
hardcover nonfiction. If he owns 20 more paperback
nonfiction books than hardcover nonfiction books, and
twice as many paperback fiction books as paperback
nonfiction books, how many hardcover nonfiction
books does Thabo own?
(A)

10

(B)

20

(C)

30

(A)

z(x + 1)

(D)

40

(B)

z(y + 1)

(E)

50

(C)

x(y + z)

(D)

y(x + z)

(E)

z(x + y)

A rectangular garden is to be twice as long as it is
wide. If 360 yards of fencing, including the gate, will
completely enclose the garden, what will be the length
of the garden, in yards?
(A)

120

(B)

140

(C)

160

(D)

180

(E)

200

21.

If the average (arithmetic mean) of the four numbers
3, 15, 32, and (N + 1) is 18, then N =
(A)

19

(B)

20

(C)

21

(D)

22

(E)

29

5.3 Problem Solving Practice Questions

22.

23.

24.

Abdul, Barb, and Carlos all live on the same straight
road, on which their school is also located. The school
is halfway between Abdul’s house and Barb’s house.
Barb’s house is halfway between the school and
Carlos’s house. If the school is 4 miles from Carlos’s
house, how many miles is Abdul’s house from Carlos’s
house?
(A)

1
1
3

(B)

2

(C)

4

(D)

6

(E)

8

25.

If y −

1 11
< , which of the following could be a value
2
2

of y ?
(A)

–11
11
−
2

(B)
(C)

11
2

(D)

11

(E)

22

B

During a certain time period, Car X traveled north
along a straight road at a constant rate of 1 mile per
minute and used fuel at a constant rate of 5 gallons
every 2 hours. During this time period, if Car X used
exactly 3.75 gallons of fuel, how many miles did Car X
travel?

120°
A

D

26.

In the figure shown, AC = 2 and BD = DC = 1. What is
the measure of angle ABD ?

(A)

36

(B)

37.5

(C)

40

(A)

15°

(D)

80

(B)

20°

(E)

90

(C)

30°

(D)

40°

(E)

45°

Cheryl purchased 5 identical hollow pine doors and
6 identical solid oak doors for the house she is
building. The regular price of each solid oak door
was twice the regular price of each hollow pine door.
However, Cheryl was given a discount of 25% off the
regular price of each solid oak door. If the regular price
of each hollow pine door was $40, what was the total
price of all 11 doors?
(A)

$320

(B)

$540

(C)

$560

(D)

$620

(E)

$680

C

Note: Not drawn to scale.

27.

28.

If k2 = m2, which of the following must be true?
(A)

k=m

(B)

k = –m

(C)

k = |m|

(D)

k = –|m|

(E)

|k| = |m|

Makoto, Nishi, and Ozuro were paid a total of $780
for waxing the floors at their school. Each was paid in
proportion to the number of hours he or she worked. If
Makoto worked 15 hours, Nishi worked 20 hours, and
Ozuro worked 30 hours, how much was Makoto paid?
(A)

$52

(B)

$117

(C)

$130

(D)

$180

(E)

$234
149

GMAT® Official Guide 2018

P

8

Q

32.

6
R
29.

The figure above shows a path around a triangular
piece of land. Mary walked the distance of 8 miles
from P to Q and then walked the distance of 6 miles
from Q to R. If Ted walked directly from P to R, by what
percent did the distance that Mary walked exceed the
distance that Ted walked?

150

20

(B)

40

(C)

60

(D)

80

(E)

100

(A)

30%

(B)

40%

(C)

50%

(A)

(D)

60%

4
27

(E)

80%

(B)

1
3

(C)

3

(D)

6

(E)

27
4

1
At a supermarket, John spent of his money on fresh
2
1
1
on
fruits and vegetables, on meat products, and
3
10
bakery products. If he spent the remaining $6 on
candy, how much did John spend at the supermarket?
34.

31.

(A)

2
The quotient when a certain number is divided by is
3
9
. What is the number?
2

33.

30.

A carpenter constructed a rectangular sandbox with
a capacity of 10 cubic feet. If the carpenter were to
make a similar sandbox twice as long, twice as wide,
and twice as high as the first sandbox, what would be
the capacity, in cubic feet, of the second sandbox?

(A)

$60

(B)

$80

(C)

$90

(D)

$120

(E)

$180

If a sphere with radius r is inscribed in a cube with
edges of length e, which of the following expresses the
relationship between r and e ?
1
e
2

(A)

r=

(B)

r=e

(C)

r = 2e

If (1 – 1.25)N = 1, then N =

(D)

r= e

(A)

–400

(E)

r=

(B)

–140

(C)

–4

(D)

4

(E)

400

35.

1 2
e
4

If 2x + y = 7 and x + 2y = 5, then
(A)

1

(B)

4
3

(C)

17
5

(D)

18
5

(E)

4

x+y
=
3

5.3 Problem Solving Practice Questions

36.

City X has a population 4 times as great as the
population of City Y, which has a population twice as
great as the population of City Z. What is the ratio of
the population of City X to the population of City Z ?
(A)

1:8

(B)

1:4

(C)

2:1

(D)

4:1

(E)

8:1

40ft
15ft

35ft
house
20ft

39.

Tides at Bay Cove on July 13

height of tide

11:30 a.m.
2.2 ft
3

4:30 a.m.
0.3 ft

2
1
0

time

–1

6 a.m. noon

midnight

40.

6 p.m.
–0.5 ft
37.

38.

The graph above shows the height of the tide, in feet,
above or below a baseline. Which of the following is
closest to the difference, in feet, between the heights
of the highest and lowest tides on July 13 at Bay Cove?
(A)

1.7

(B)

1.9

(C)

2.2

(D)

2.5

(E)

2.7

patio

A manufacturer of a certain product can expect that
between 0.3 percent and 0.5 percent of the units
manufactured will be defective. If the retail price is
$2,500 per unit and the manufacturer offers a full
refund for defective units, how much money can the
manufacturer expect to need to cover the refunds on
20,000 units?
(A)

Between $15,000 and $25,000

(B)

Between $30,000 and $50,000

(C)

Between $60,000 and $100,000

(D)

Between $150,000 and $250,000

(E)

Between $300,000 and $500,000

41.

A flat patio was built alongside a house as shown in the
figure above. If all angles are right angles, what is the
area of the patio in square feet?
(A)

800

(B)

875

(C)

1,000

(D)

1,100

(E)

1,125

The sum of the weekly salaries of 5 employees is
$3,250. If each of the 5 salaries is to increase by
10 percent, then the average (arithmetic mean) weekly
salary per employee will increase by
(A)

$52.50

(B)

$55.00

(C)

$57.50

(D)

$62.50

(E)

$65.00

A student’s average (arithmetic mean) test score on
4 tests is 78. What must be the student’s score
on a 5th test for the student’s average score on the
5 tests to be 80 ?
(A)

80

(B)

82

(C)

84

(D)

86

(E)

88

151

GMAT® Official Guide 2018

42.

Last week Chris earned x dollars per hour for the first
40 hours worked plus 22 dollars per hour for each
hour worked beyond 40 hours. If last week Chris
earned a total of 816 dollars by working 48 hours,
what is the value of x ?
(A)

13

(B)

14

(C)

15

(D)

16

(E)

17

46.

47.
B
y°

(y + 10)°
A
43.

44.

C

In the figure above, what is the ratio of the measure of
angle B to the measure of angle A ?

6

(B)

9

(C)

10

(D)

11

(E)

14

In a certain sequence of 8 numbers, each number
after the first is 1 more than the previous number. If
the first number is –5, how many of the numbers in the
sequence are positive?
(A)

None

(B)

One

(C)

Two

(D)

Three

(E)

Four

2 to 3

(B)

3 to 4

(C)

3 to 5

(D)

4 to 5

A total of s oranges are to be packaged in boxes that
will hold r oranges each, with no oranges left over.
When n of these boxes have been completely filled,
what is the number of boxes that remain to be filled?

(E)

5 to 6

(A)

s – nr

(B)

s−

(C)

rs – n

(D)

s
−r
n

(E)

s
−n
r

48.

If n is a prime number greater than 3, what is the
remainder when n2 is divided by 12 ?
(A)

0

(B)

1

(C)

2

(D)

3

(E)

5

1
1
1+
3
(A)
(B)
(C)
(D)
(E)

152

(A)

(A)

49.
45.

The positive two‐digit integers x and y have the same
digits, but in reverse order. Which of the following must
be a factor of x + y ?

−

1
1
1+
2

1
3
1
−
6
1
−
12
1
12
1
3
−

=

n
r

If 0 < a < b < c, which of the following statements
must be true?
I.

2a > b + c

II.

c–a>b–a

III.

c b
<
a a

(A)

I only

(B)

II only

(C)

III only

(D)

I and II

(E)

II and III

5.3 Problem Solving Practice Questions

50.

51.

In the xy-plane, the origin O is the midpoint of line
segment PQ. If the coordinates of P are (r,s), what are
the coordinates of Q ?
(A)

(r,s)

(B)

(s,–r)

(C)

(–s,–r)

(D)

(–r,s)

(E)

(–r,–s)

53.

Which of the following equations is NOT equivalent to
10y2 = (x + 2)(x – 2) ?

Company Q plans to make a new product next year
and sell each unit of this new product at a selling price
of $2. The variable costs per unit in each production
run are estimated to be 40% of the selling price, and
the fixed costs for each production run are estimated
to be $5,040. Based on these estimated costs, how
many units of the new product will Company Q need to
make and sell in order for their revenue to equal their
total costs for each production run?
(A)

4,200

(B)

3,150

(C)

2,520

(A)

30y2 = 3x2 – 12

(D)

2,100

(B)

20y2 = (2x – 4)(x + 2)

(E)

1,800

(C)

10y2 + 4 = x2

(D)

5y 2 = x 2 – 2

(E)

y2 =

S

x2 − 4
10

E

A

Monday

Tuesday

Wednesday

Thursday

Company A

45

55

50

50

Company B

10

30

30

10

Company C

34

28

28

30

Company D

39

42

41

38

The dial shown above is divided into equal‐sized
intervals. At which of the following letters will the
pointer stop if it is rotated clockwise from S through
1,174 intervals?

Company E

50

60

60

70

(A)

A

(B)

B

(C)

C

(D)

D

(E)

E

52.

The table shows the numbers of packages shipped
daily by each of five companies during a 4‐day period.
The standard deviation of the numbers of packages
shipped daily during the period was greatest for which
of the five companies?
(A)

A

(B)

B

(C)

C

(D)

D

(E)

E

D

B
C

54.

153

GMAT® Official Guide 2018

Estimated Number of Home-Schooled
Students by State, January 2001

55.

State

Number (in thousands)

A

181

B

125

C

103

D

79

E

72

57.

According to the table shown, the estimated number
of home-schooled students in State A is approximately
what percent greater than the number in State D ?
(A)

25%

(B)

55%

(C)

100%

(D)

125%

(E)

155%

When n liters of fuel were added to a tank that was
1
7
already full, the tank was filled to of its capacity. In
3
9
terms of n, what is the capacity of the tank, in liters?
(A)

10
n
9

(B)

4
n
3

(C)

3
n
2

(D)

9
n
4

(E)

7
n
3
w ft

100 ft
150 ft
w ft
Note: Not drawn to scale.

y
II

I
x

O
III

56.

154

58.

IV

The graph of the equation xy = k, where k < 0, lies in
which two of the quadrants shown above?
(A)

I and II

(B)

I and III

(C)

II and III

(D)

II and IV

(E)

III and IV

59.

The smaller rectangle in the figure above represents
the original size of a parking lot before its length
and width were each extended by w feet to make
the larger rectangular lot shown. If the area of the
enlarged lot is twice the area of the original lot, what is
the value of w ?
(A)

25

(B)

50

(C)

75

(D)

100

(E)

200

1
=
0.75 − 1
(A)

–4

(B)

–0.25

(C)

0.25

(D)

0.75

(E)

4

5.3 Problem Solving Practice Questions

60.

Kevin invested $8,000 for one year at a simple annual
interest rate of 6 percent and invested $10,000 for
one year at an annual interest rate of 8 percent
compounded semiannually. What is the total amount of
interest that Kevin earned on the two investments?

64.

(A)

320

(A)

$880

(B)

480

(B)

$1,088

(C)

520

(C)

$1,253

(D)

730

(D)

$1,280

(E)

920

(E)

$1,296
65.

61.

62.

63.

Car X averages 25.0 miles per gallon of gasoline
and Car Y averages 11.9 miles per gallon. If each car
is driven 12,000 miles, approximately how many more
gallons of gasoline will Car Y use than Car X ?

The harvest yield from a certain apple orchard was
350 bushels of apples. If x of the trees in the orchard
each yielded 10 bushels of apples, what fraction of the
harvest yield was from these x trees?
(A)

x
35

(B)

1−

(C)

10x

(D)

35 – x

(E)

350 – 10x

x
35
66.

If n is an integer, which of the following must be even?

If y is an integer, then the least possible value of
23 − 5y is
(A)

1

(B)

2

(C)

3

(D)

4

(E)

5

80 + 125 =
(A)

9 5

(B)

20 5

(C)

41 5
205

(A)

n+1

(D)

(B)

n+2

(E)

(C)

2n

(D)

2n + 1

(E)

n2

The sum
(A)
(B)

7 1
+ is between
8 9

1
3
and
2
4
3
and 1
4
1 and 1

(D)

1

(E)

y = kx + 3
67.

(C)

100

1
4

In the equation above, k is a constant. If y = 17 when
x = 2, what is the value of y when x = 4 ?
(A)

34

(B)

31

(C)

14

(D)

11

(E)

7

1
1
and 1
4
2
1
1 and 2
2

155

GMAT® Official Guide 2018

68.

69.

70.

156

Which of the following is greatest?
(A)

10 3

(B)

9 4

(C)

8 5

(D)

7 6

(E)

6 7

Al and Ben are drivers for SD Trucking Company. One
snowy day, Ben left SD at 8:00 a.m. heading east and
Al left SD at 11:00 a.m. heading west. At a particular
time later that day, the dispatcher retrieved data
from SD’s vehicle tracking system. The data showed
that, up to that time, Al had averaged 40 miles per
hour and Ben had averaged 20 miles per hour. It also
showed that Al and Ben had driven a combined total
of 240 miles. At what time did the dispatcher retrieve
data from the vehicle tracking system?
(A)

1:00 p.m.

(B)

2:00 p.m.

(C)

3:00 p.m.

(D)

5:00 p.m.

(E)

6:00 p.m.

71.

72.

73.

Of the land owned by a farmer, 90 percent was
cleared for planting. Of the cleared land, 40 percent
was planted with soybeans and 50 percent of the
cleared land was planted with wheat. If the remaining
720 acres of cleared land was planted with corn, how
many acres did the farmer own?
(A)

5,832

(B)

6,480

(C)

7,200

(D)

8,000

(E)

8,889

74.

At the start of an experiment, a certain population
consisted of 3 animals. At the end of each month after
the start of the experiment, the population size was
double its size at the beginning of that month. Which of
the following represents the population size at the end
of 10 months?
(A)

23

(B)

32

(C)

2(310)

(D)

3(210)

(E)

3(102)

1
1⎞
⎛ 1 1 1 1⎞
⎛1 1
If ⎜ + + + ⎟ = r ⎜ +
+
+
, then r =
⎝ 3 4 5 6⎠
⎝ 9 12 15 18 ⎟⎠
(A)

1
3

(B)

4
3

(C)

3

(D)

4

(E)

12

If x and y are positive integers such that y is a multiple
of 5 and 3x + 4y = 200, then x must be a multiple of
which of the following?
(A)

3

(B)

6

(C)

7

(D)

8

(E)

10

Which of the following expressions can be written as
an integer?
I.

⎛
⎜⎝ 82 + 82

II.

(82) (

82

)

)

III.

( 82 )( 82 )

(A)

None

(B)

I only

(C)

III only

(D)

I and II

(E)

I and III

82

2

5.3 Problem Solving Practice Questions

75.

76.

77.

78.

Four staff members at a certain company worked
on a project. The amounts of time that the four staff
members worked on the project were in the ratio 2 to
3 to 5 to 6. If one of the four staff members worked
on the project for 30 hours, which of the following
CANNOT be the total number of hours that the four
staff members worked on the project?
(A)

80

(B)

96

(C)

160

(D)

192

(E)

240

Pumping alone at their respective constant rates, one
1
inlet pipe fills an empty tank to of capacity in 3 hours
2
and a second inlet pipe fills the same empty tank to
2
of capacity in 6 hours. How many hours will it take
3
both pipes, pumping simultaneously at their respective
constant rates, to fill the empty tank to capacity?
(A)

3.25

(B)

3.6

(C)

4.2

(D)

4.4

(E)

5.5

In the xy‐coordinate plane, which of the following points
must lie on the line kx + 3y = 6 for every possible value
of k ?

79.

Book number

Pages in book

Total pages read

1

253

253

2

110

363

3

117

480

4

170

650

5

155

805

6

50

855

7

205

1,060

8

70

1,130

9

165

1,295

10

105

1,400

11

143

1,543

12

207

1,750

Shawana made a schedule for reading books during
4 weeks (28 days) of her summer vacation. She has
checked out 12 books from the library. The number of
pages in each book and the order in which she plans to
read the books are shown in the table above. She will
read exactly 50 pages each day. The only exception
will be that she will never begin the next book on the
same day that she finishes the previous one, and
therefore on some days she may read fewer than
50 pages. At the end of the 28th day, how many books
will Shawana have finished?
(A)

7

(A)

(1,1)

(B)

8

(B)

(0,2)

(C)

9

(C)

(2,0)

(D)

10

(D)

(3,6)

(E)

11

(E)

(6,3)

If x 2 – 2 < 0, which of the following specifies all the
possible values of x ?
(A)

0< x <2

(B)

0 512 ?

400,000
300,000
200,000
100,000
0

1990

6

(B)

7

(A)

250,000

(C)

8

(B)

280,000

(D)

9

(C)

310,000

(E)

10

(D)

325,000

(E)

340,000

1994

1996
year

1998

2000

165. For the positive integers a, b, and k, a k b means that
a k is a divisor of b, but a k +1 is not a divisor of b. If k is
a positive integer and 2k 72, then k is equal to

(A)

0.10

(A)

2

(B)

0.15

(B)

3

(C)

0.27

(C)

4

(D)

0.33

(D)

8

(E)

0.45

(E)

18

163. Each year for 4 years, a farmer increased the number
1
of trees in a certain orchard by of the number of
4
trees in the orchard the preceding year. If all of the
trees thrived and there were 6,250 trees in the orchard
at the end of the 4‐year period, how many trees were
in the orchard at the beginning of the 4‐year period?

1992

164. According to the chart shown, which of the following is
closest to the median annual number of shipments of
manufactured homes in the United States for the years
from 1990 to 2000, inclusive?

(A)

162. Sixty percent of the members of a study group are
women, and 45 percent of those women are lawyers.
If one member of the study group is to be selected
at random, what is the probability that the member
selected is a woman lawyer?

170

Number of Shipments of Manufactured Homes
in the United States, 1990–2000

number of shipments

160. The present ratio of students to teachers at a certain
school is 30 to 1. If the student enrollment were to
increase by 50 students and the number of teachers
were to increase by 5, the ratio of students to
teachers would then be 25 to 1. What is the present
number of teachers?

166. A certain characteristic in a large population has a
distribution that is symmetric about the mean m.
If 68 percent of the distribution lies within one
standard deviation d of the mean, what percent of the
distribution is less than m + d ?
(A)

16%

(B)

32%

(A)

1,250

(C)

48%

(B)

1,563

(D)

84%

(C)

2,250

(E)

92%

(D)

2,560

(E)

2,752

5.3 Problem Solving Practice Questions

167. Four extra‐large sandwiches of exactly the same size
were ordered for m students, where m > 4. Three
of the sandwiches were evenly divided among the
students. Since 4 students did not want any of the
fourth sandwich, it was evenly divided among the
remaining students. If Carol ate one piece from each
of the four sandwiches, the amount of sandwich that
she ate would be what fraction of a whole extra‐large
sandwich?

170. In a box of 12 pens, a total of 3 are defective. If a
customer buys 2 pens selected at random from the
box, what is the probability that neither pen will be
defective?
(A)

1
6

(B)

2
9

(A)

m+4
m (m − 4)

(C)

6
11

(B)

2m − 4
m (m − 4)

(D)

9
16

(C)

4m − 4
m (m − 4)

(E)

3
4

(D)

4m − 8
m (m − 4)

(E)

4m − 12
m (m − 4)

168. Which of the following equations has 1 + 2 as one of
its roots?

171. At a certain fruit stand, the price of each apple is
40 cents and the price of each orange is 60 cents.
Mary selects a total of 10 apples and oranges from the
fruit stand, and the average (arithmetic mean) price of
the 10 pieces of fruit is 56 cents. How many oranges
must Mary put back so that the average price of the
pieces of fruit that she keeps is 52 cents?

(A)

x 2 + 2x – 1 = 0

(A)

1

(B)

x2

– 2x + 1 = 0

(B)

2

(C)

x2

+ 2x + 1 = 0

(C)

3

(D)

x2

– 2x – 1 = 0

(D)

4

(E)

x2

–x–1=0

(E)

5

169. In Country C, the unemployment rate among
construction workers dropped from 16 percent on
September 1, 1992, to 9 percent on September 1,
1996. If the number of construction workers was
20 percent greater on September 1, 1996, than
on September 1, 1992, what was the approximate
percent change in the number of unemployed
construction workers over this period?

172. A pharmaceutical company received $3 million in
royalties on the first $20 million in sales of the generic
equivalent of one of its products and then $9 million
in royalties on the next $108 million in sales. By
approximately what percent did the ratio of royalties to
sales decrease from the first $20 million in sales to the
next $108 million in sales?
(A)

8%

(A)

50% decrease

(B)

15%

(B)

30% decrease

(C)

45%

(C)

15% decrease

(D)

52%

(D)

30% increase

(E)

56%

(E)

55% increase

171

GMAT® Official Guide 2018

Times at Which the Door
Opened from 8:00 to 10:00
8:00

8:06

8:30

9:05

8:03

8:10

8:31

9:11

8:04

8:18

8:54

9:29

8:04

8:19

8:57

9:31

173. The light in a restroom operates with a 15-minute timer
that is reset every time the door opens as a person
goes in or out of the room. Thus, after someone
enters or exits the room, the light remains on for only
15 minutes unless the door opens again and resets the
timer for another 15 minutes. If the times listed above
are the times at which the door opened from 8:00 to
10:00, approximately how many minutes during this
two-hour period was the light off?
(A)

10

(B)

25

(C)

35

(D)

40

(E)

70

60°
174. The parallelogram shown has four sides of equal
length. What is the ratio of the length of the shorter
diagonal to the length of the longer diagonal?
(A)
(B)
(C)
(D)
(E)

172

1
2
1
2
1
2 2
1
3
1
2 3

175. If p is the product of the integers from 1 to 30,
inclusive, what is the greatest integer k for which
3k is a factor of p ?
(A)

10

(B)

12

(C)

14

(D)

16

(E)

18

176. If n = 38 – 28, which of the following is NOT a factor
of n ?
(A)

97

(B)

65

(C)

35

(D)

13

(E)

5

177. In the figure shown, if the area of the shaded region is
3 times the area of the smaller circular region, then the
circumference of the larger circle is how many times
the circumference of the smaller circle?
(A)

4

(B)

3

(C)

2

(D)

3

(E)

2

5.3 Problem Solving Practice Questions

178. Club X has more than 10 but fewer than 40 members.
Sometimes the members sit at tables with 3 members
at one table and 4 members at each of the other tables,
and sometimes they sit at tables with 3 members at
one table and 5 members at each of the other tables.
If they sit at tables with 6 members at each table
except one and fewer than 6 members at that one
table, how many members will be at the table that has
fewer than 6 members?
(A)

1

(B)

2

(C)

3

(D)

4

(E)

5

182. A researcher plans to identify each participant in a
certain medical experiment with a code consisting of
either a single letter or a pair of distinct letters written
in alphabetical order. What is the least number of
letters that can be used if there are 12 participants,
and each participant is to receive a different code?
(A)

4

(B)

5

(C)

6

(D)

7

(E)

8
y

179. In order to complete a reading assignment on time,
Terry planned to read 90 pages per day. However,
she read only 75 pages per day at first, leaving
690 pages to be read during the last 6 days before the
assignment was to be completed. How many days in
all did Terry have to complete the assignment on time?
(A)

15

(B)

16

(C)

25

(D)

40

(E)

46

180. If s > 0 and
(A)

s

(C)

s s

(D)

s3

(E)

s2 − s

r
= s, what is r in terms of s ?
s

x
181. If 3 < x < 100, for how many values of x is the
3
square of a prime number?
(A)

Two

(B)

Three

(C)

Four

(D)

Five

(E)

Nine

1
O

1

x

183. The graph of which of the following equations is a
straight line that is parallel to line ℓ in the figure above?

1
s

(B)

2

(A)

3y – 2x = 0

(B)

3y + 2x = 0

(C)

3y + 2x = 6

(D)

2y – 3x = 6

(E)

2y + 3x = –6

184. An object thrown directly upward is at a height of h feet
after t seconds, where h = –16(t – 3)2 + 150. At what
height, in feet, is the object 2 seconds after it reaches
its maximum height?
(A)

6

(B)

86

(C)

134

(D)

150

(E)

166

173

GMAT® Official Guide 2018

185. Which of the following is equivalent to the pair of
inequalities x + 6 > 10 and x – 3 ≤ 5 ?
(A)

2 ≤ x < 16

(B)

2≤x<4

(C)

2 y. Two packages weighing 3 pounds and 5 pounds,
respectively, can be mailed separately or combined as
one package. Which method is cheaper, and how much
money is saved?
(A)

Combined, with a savings of x – y cents

(B)

Combined, with a savings of y – x cents

(C)

Combined, with a savings of x cents

(D)

Separately, with a savings of x – y cents

(E)

Separately, with a savings of y cents

191. If money is invested at r percent interest, compounded
annually, the amount of the investment will double
70
in approximately
years. If Pat’s parents invested
r
$5,000 in a long‐term bond that pays 8 percent
interest, compounded annually, what will be the
approximate total amount of the investment 18 years
later, when Pat is ready for college?
(A)

$20,000

(B)

$15,000

(C)

$12,000

(D)

$10,000

(E)

$9,000

5.3 Problem Solving Practice Questions

192. On a recent trip, Cindy drove her car 290 miles,
rounded to the nearest 10 miles, and used 12 gallons
of gasoline, rounded to the nearest gallon. The actual
number of miles per gallon that Cindy’s car got on this
trip must have been between

195. If y is the smallest positive integer such that
3,150 multiplied by y is the square of an integer,
then y must be
(A)

2

(B)

5

(A)

290
290
and
12.5
11.5

(C)

6

295
285
and
12
11.5

(D)

7

(B)

(E)

14

(C)

285
295
and
12
12

(D)

285
295
and
12.5
11.5

(E)

295
285
and
12.5
11.5

–5 –4 –3 –2 –1

196. If [x] is the greatest integer less than or equal to x,
what is the value of [–1.6] + [3.4] + [2.7] ?

0

1

2

3

4

5

x

193. Which of the following inequalities is an algebraic
expression for the shaded part of the number line
above?

(A)

3

(B)

4

(C)

5

(D)

6

(E)

7

197. In the first week of the year, Nancy saved $1. In each
of the next 51 weeks, she saved $1 more than she
had saved in the previous week. What was the total
amount that Nancy saved during the 52 weeks?

(A)

|x|≤3

(B)

|x|≤5

(A)

$1,326

(C)

|x–2|≤3

(B)

$1,352

(D)

|x–1|≤4

(C)

$1,378

(E)

|x+1|≤4

(D)

$2,652

(E)

$2,756

194. In a small snack shop, the average (arithmetic mean)
revenue was $400 per day over a 10‐day period.
During this period, if the average daily revenue was
$360 for the first 6 days, what was the average daily
revenue for the last 4 days?

198. In a certain sequence, the term xn is given by the
1
formula x n = 2x n − 1 − ( x n − 2 ) for all n ≥ 2. If x 0 = 3
2
and x1 = 2, what is the value of x 3 ?

(A)

$420

(A)

2.5

(B)

$440

(B)

3.125

(C)

$450

(C)

4

(D)

$460

(D)

5

(E)

$480

(E)

6.75

175

GMAT® Official Guide 2018

199. During a trip, Francine traveled x percent of the total
distance at an average speed of 40 miles per hour
and the rest of the distance at an average speed of
60 miles per hour. In terms of x, what was Francine’s
average speed for the entire trip?
(A)

180 − x
2

(B)

x + 60
4

(C)

300 − x
5

(D)

600
115 − x

(E)

12,000
x + 200

200. If n =

(33)43

+

3

(A)

One

(B)

Two

(C)

Three

(D)

Seven

(E)

Ten

204. For any positive integer n, the sum of the first
n(n + 1)
n positive integers equals
. What is the sum of
2
all the even integers between 99 and 301 ?

(43)33,

what is the units digit of n ?

(A)

10,100

(B)

20,200

(C)

22,650

(A)

0

(D)

40,200

(B)

2

(E)

45,150

(C)

4

(D)

6

(E)

8

205. How many prime numbers between 1 and 100 are
factors of 7,150 ?

201. Team A and Team B are competing against each other
in a game of tug-of-war. Team A, consisting of 3 males
and 3 females, decides to line up male, female, male,
female, male, female. The lineup that Team A chooses
will be one of how many different possible lineups?
(A)

9

(B)

12

(C)

15

(D)

36

(E)

720

202. A border of uniform width is placed around a
rectangular photograph that measures 8 inches by
10 inches. If the area of the border is 144 square
inches, what is the width of the border, in inches?

176

1
is expressed as a terminating decimal,
2 × 57
how many nonzero digits will d have?

203. If d =

(A)

3

(B)

4

(C)

6

(D)

8

(E)

9

(A)

One

(B)

Two

(C)

Three

(D)

Four

(E)

Five

206. A sequence of numbers a1, a2, a3, . . . is defined as
follows: a1 = 3, a2 = 5, and every term in the sequence
after a2 is the product of all terms in the sequence
preceding it, e.g., a3 = (a1)(a2) and a4 = (a1)(a2)(a3).
If an = t and n > 2, what is the value of an + 2 in terms
of t ?
(A)

4t

(B)

t2

(C)

t3

(D)

t4

(E)

t8

5.3 Problem Solving Practice Questions

207. Last year the price per share of Stock X increased
by k percent and the earnings per share of Stock X
increased by m percent, where k is greater than m.
By what percent did the ratio of price per share to
earnings per share increase, in terms of k and m ?

210. A photography dealer ordered 60 Model X cameras to
be sold for $250 each, which represents a 20 percent
markup over the dealer’s initial cost for each camera.
Of the cameras ordered, 6 were never sold and
were returned to the manufacturer for a refund of
50 percent of the dealer’s initial cost. What was the
dealer’s approximate profit or loss as a percent of the
dealer’s initial cost for the 60 cameras?

(A)

k
%
m

(B)

(k – m)%
100(k − m )
%
100 + k

(A)

7% loss

(C)

(B)

13% loss

(D)

100(k − m )
%
100 + m

(C)

7% profit

(D)

13% profit

(E)

100(k − m )
%
100 + k + m

(E)

15% profit

208. Of the 300 subjects who participated in an experiment
using virtual‐reality therapy to reduce their fear of
heights, 40 percent experienced sweaty palms,
30 percent experienced vomiting, and 75 percent
experienced dizziness. If all of the subjects
experienced at least one of these effects and
35 percent of the subjects experienced exactly two of
these effects, how many of the subjects experienced
only one of these effects?

211. Seven pieces of rope have an average (arithmetic
mean) length of 68 centimeters and a median length of
84 centimeters. If the length of the longest piece of rope
is 14 centimeters more than 4 times the length of the
shortest piece of rope, what is the maximum possible
length, in centimeters, of the longest piece of rope?
(A)

82

(B)

118

(C)

120

(A)

105

(D)

134

(B)

125

(E)

152

(C)

130

(D)

180

(E)

195

1
209. If m −1 = − , then m −2 is equal to
3

212. What is the difference between the sixth and the fifth
terms of the sequence 2, 4, 7, … whose nth term is
n + 2n − 1 ?
(A)

2

(B)

3

(A)

−9

(C)

6

(B)

−3

(D)

16

(C)

−

1
9

(E)

17

(D)

1
9

(E)

9

213. From the consecutive integers –10 to 10, inclusive,
20 integers are randomly chosen with repetitions
allowed. What is the least possible value of the product
of the 20 integers?
(A)

(−10)20

(B)

(−10)10

(C)

0

(D)

−(10)19

(E)

−(10)20
177

GMAT® Official Guide 2018

214. The letters D, G, I, I, and T can be used to form 5‐letter
strings such as DIGIT or DGIIT. Using these letters, how
many 5‐letter strings can be formed in which the two
occurrences of the letter I are separated by at least
one other letter?

217. For the past n days, the average (arithmetic mean)
daily production at a company was 50 units. If today’s
production of 90 units raises the average to 55 units
per day, what is the value of n ?
(A)

30

(A)

12

(B)

18

(B)

18

(C)

10

(C)

24

(D)

9

(D)

36

(E)

7

(E)

48
y

215. Last Sunday a certain store sold copies of Newspaper A
for $1.00 each and copies of Newspaper B for $1.25
each, and the store sold no other newspapers that
day. If r percent of the store’s revenue from newspaper
sales was from Newspaper A and if p percent of
the newspapers that the store sold were copies of
Newspaper A, which of the following expresses r in
terms of p ?

216.

178

(A)

100p
125 − p

(B)

150p
250 − p

(C)

300p
375 − p

(D)

400p
500 − p

(E)

500p
625 − p

0.99999999 0.99999991
−
=
1.0001
1.0003
(A)

10−8

(B)

3(10−8 )

(C)

3(10−4 )

(D)

2(10−4 )

(E)

10−4

1
O

1

x

218. In the coordinate system above, which of the following
is the equation of line  ?
(A)

2x − 3y = 6

(B)

2x + 3y = 6

(C)

3x + 2y = 6

(D)

2x − 3y = −6

(E)

3x − 2y = −6

219. If a two‐digit positive integer has its digits reversed,
the resulting integer differs from the original by 27. By
how much do the two digits differ?
(A)

3

(B)

4

(C)

5

(D)

6

(E)

7

5.3 Problem Solving Practice Questions

220. In an electric circuit, two resistors with resistances x
and y are connected in parallel. In this case, if r is the
combined resistance of these two resistors, then the
reciprocal of r is equal to the sum of the reciprocals of
x and y. What is r in terms of x and y ?

⎛ 1⎞
223. ⎜ ⎟
⎝ 2⎠

−3

⎛ 1⎞
⎜⎝ ⎟⎠
4

−2

⎛ 1⎞
⎜⎝ ⎟⎠
2

−48

(A)

⎛ 1⎞
⎜⎝ ⎟⎠
2

−11

(B)

(A)

xy

(B)

x+y

(C)

(C)

⎛ 1⎞
⎜⎝ ⎟⎠
2

−6

1
x+y

(D)

(D)

⎛ 1⎞
⎜⎝ ⎟⎠
8

−11

xy
x+y

(E)

(E)

⎛ 1⎞
⎜⎝ ⎟⎠
8

−6

x+y
xy

11
8

(B)

7
8

(C)

9
64

(D)

5
64

(E)

3
64

222. If

1
1
1
−
=
, then x could be
x x +1 x + 4

(A)

0

(B)

–1

(C)

–2

(D)

–3

(E)

–4

−1

a°

221. Xavier, Yvonne, and Zelda each try independently to
solve a problem. If their individual probabilities for
1 1
5
success are , , and , respectively, what is the
4 2
8
probability that Xavier and Yvonne, but not Zelda, will
solve the problem?
(A)

⎛ 1⎞
⎜⎝ ⎟⎠
16

224. The figure shown above consists of a shaded 9-sided
polygon and 9 unshaded isosceles triangles. For each
isosceles triangle, the longest side is a side of the
shaded polygon and the two sides of equal length are
extensions of the two adjacent sides of the shaded
polygon. What is the value of a ?
(A)

100

(B)

105

(C)

110

(D)

115

(E)

120

179

GMAT® Official Guide 2018

225. List T consists of 30 positive decimals, none of which
is an integer, and the sum of the 30 decimals is S. The
estimated sum of the 30 decimals, E, is defined as
follows. Each decimal in T whose tenths digit is even
is rounded up to the nearest integer, and each decimal
in T whose tenths digit is odd is rounded down to the
nearest integer; E is the sum of the resulting integers.
1
If of the decimals in T have a tenths digit that is
3
even, which of the following is a possible value of
E–S?

(A)

1

(B)

2

(C)

3

(D)

4

(E)

5

229. Of the 150 houses in a certain development,
60 percent have air-conditioning, 50 percent have a
sunporch, and 30 percent have a swimming pool. If
5 of the houses have all three of these amenities and
5 have none of them, how many of the houses have
exactly two of these amenities?

I.

–16

II.

6

III.

10

(A)

I only

(B)

I and II only

(C)

I and III only

(A)

10

(D)

II and III only

(B)

45

(E)

I, II, and III

(C)

50

(D)

55

(E)

65

6
226. If 5 − = x , then x has how many possible values?
x
(A)

None

(B)

One

(C)

Two

(D)

A finite number greater than two

(E)

An infinite number

227. Seed mixture X is 40 percent ryegrass and 60 percent
bluegrass by weight; seed mixture Y is 25 percent
ryegrass and 75 percent fescue. If a mixture of X and
Y contains 30 percent ryegrass, what percent of the
weight of the mixture is X ?

180

228. How many of the integers that satisfy the inequality
( x + 2)( x + 3)
≥ 0 are less than 5 ?
x −2

(A)

10%

(B)

1
33 %
3

(C)

40%

(D)

50%

(E)

2
66 %
3

2−14 + 2−15 + 2−16 + 2−17
is how many
5
times the value of 2( −17) ?

230. The value of

(A)

3
2

(B)

5
2

(C)

3

(D)

4

(E)

5

5.4 Problem Solving Answer Key

5.4 Answer Key
1.

E

31.

C

61.

A

91.

C

2.

D

32.

D

62.

C

92.

E

3.

E

33.

C

63.

B

93.

B

4.

A

34.

A

64.

C

94.

E

5.

B

35.

B

65.

B

95.

E

6.

E

36.

E

66.

A

96.

E

7.

C

37.

E

67.

B

97.

D

8.

D

38.

D

68.

B

98.

B

9.

C

39.

C

69.

B

99.

B

10.

E

40.

E

70.

D

100.

B

11.

A

41.

E

71.

D

101.

B

12.

C

42.

D

72.

C

102.

B

13.

B

43.

D

73.

E

103.

E

14.

B

44.

B

74.

E

104.

C

15.

D

45.

D

75.

D

105.

B

16.

E

46.

D

76.

B

106.

A

17.

A

47.

C

77.

B

107.

E

18.

B

48.

E

78.

C

108.

D

19.

D

49.

B

79.

B

109.

A

20.

B

50.

E

80.

D

110.

E

21.

C

51.

D

81.

A

111.

D

22.

D

52.

B

82.

D

112.

B

23.

E

53.

A

83.

D

113.

E

24.

C

54.

E

84.

D

114.

E

25.

C

55.

D

85.

E

115.

D

26.

C

56.

D

86.

D

116.

E

27.

E

57.

D

87.

B

117.

A

28.

D

58.

B

88.

D

118.

E

29.

B

59.

A

89.

D

119.

D

30.

C

60.

E

90.

D

120.

D

181

GMAT® Official Guide 2018

121.

B

151.

B

181.

B

211.

D

122.

D

152.

B

182.

B

212.

E

123.

D

153.

C

183.

A

213.

E

124.

E

154.

B

184.

B

214.

D

125.

D

155.

B

185.

D

215.

D

126.

C

156.

C

186.

C

216.

D

127.

D

157.

A

187.

C

217.

E

128.

B

158.

B

188.

B

218.

B

129.

A

159.

B

189.

E

219.

A

130.

D

160.

E

190.

A

220.

D

131.

A

161.

B

191.

A

221.

E

132.

E

162.

C

192.

D

222.

C

133.

D

163.

D

193.

E

223.

B

134.

B

164.

C

194.

D

224.

A

135.

A

165.

B

195.

E

225.

B

136.

A

166.

D

196.

A

226.

C

137.

A

167.

E

197.

C

227.

B

138.

E

168.

D

198.

C

228.

D

139.

D

169.

B

199.

E

229.

D

140.

D

170.

C

200.

A

230.

C

141.

D

171.

E

201.

D

142.

C

172.

C

202.

A

143.

C

173.

B

203.

B

144.

A

174.

D

204.

B

145.

D

175.

C

205.

D

146.

E

176.

C

206.

D

147.

A

177.

C

207.

D

148.

B

178.

E

208.

D

149.

E

179.

B

209.

D

150.

E

180.

D

210.

D

182

5.5 Problem Solving Answer Explanations

5.5 Answer Explanations
The following discussion is intended to familiarize you with the most efficient and effective approaches
to the kinds of problems common to problem solving questions. The particular questions in this chapter
are generally representative of the kinds of problem solving questions you will encounter on the GMAT
exam. Remember that it is the problem solving strategy that is important, not the specific details of a
particular question.

1.

Arithmetic Operations with rational numbers

The price of a coat in a certain store is $500. If the
price of the coat is to be reduced by $150, by what
percent is the price to be reduced?
(A)

10%

(B)

15%

(C)

20%

(D)

25%

(E)

30%

At the exchange rate of 0.80 euro per dollar,
Rose exchanged $500.00 for (0.80)(500) =
3
400 euros. She spent (400) = 300 euros, had
4
400 – 300 = 100 euros left, and exchanged them
for dollars at the exchange rate of $1.20 per euro.
Therefore, the dollar amount she received was
(1.20)(100) = $120.00.
The correct answer is D.

Arithmetic Percents

A reduction of $150 from $500 represents a
⎛ 150
⎞
× 100 ⎟ % = 30% .
percent decrease of ⎜
⎝ 500
⎠
Therefore, the price of the coat was reduced
by 30%.
The correct answer is E.
2.

On a vacation, Rose exchanged $500.00 for euros
at an exchange rate of 0.80 euro per dollar and
3
spent of the euros she received. If she exchanged
4
the remaining euros for dollars at an exchange rate
of $1.20 per euro, what was the dollar amount she
received?
(A)

$60.00

(B)

$80.00

(C)

$100.00

(D)

$120.00

(E)

$140.00

3.

x

x

x

y

y

v

v

x

x

y

w

w

Each of the 12 squares shown is labeled x, y, v, or
w. What is the ratio of the number of these squares
labeled x or y to the number of these squares labeled v
or w ?
(A)

1:2

(B)

2:3

(C)

4:3

(D)

3:2

(E)

2:1

Arithmetic Ratio and proportion

By a direct count, there are 8 squares labeled
x or y (5 labeled x, 3 labeled y) and there are
4 squares labeled v or w (2 labeled v, 2 labeled
w). Therefore, the ratio of the number of squares
labeled x or y to the number of squares labeled v
or w is 8:4, which reduces to 2:1.
The correct answer is E.

183

GMAT® Official Guide 2018

4.

tulips, or 85. Thus, the number of bouquets must
be a factor of 85, and so the number must be 1, 5,
17, or 85. Since the number of bouquets must be
1, 3, 5, or 15, and the number of bouquets must
be 1, 5, 17, or 85, it follows that the number of
bouquets must be 1 or 5, and thus the greatest
number of bouquets that can be made is 5. Note
that each of the 5 bouquets will have 3 white
tulips, because (5)(3) = 15, and each of the
5 bouquets will have 17 red tulips, because (5)
(17) = 85.

1 1 5 1 1 9
+ − + + −
=
3 2 6 5 4 20
(A)

0

(B)

2
15

(C)

2
5

(D)

9
20

(E)

5
6

Arithmetic Operations with rational numbers

A number that is divisible by each of the
denominators is 60. Therefore, 60 can be used as a
common denominator, which gives the following:

The correct answer is B.
6.

20 30 50 12 15 27
+
−
+
+
−
=
60 60 60 60 60 60
20 + 30 − 50 + 12 + 15 − 27 0
=
= 0.
60
60

Bouquets are to be made using white tulips and red
tulips, and the ratio of the number of white tulips to
the number of red tulips is to be the same in each
bouquet. If there are 15 white tulips and 85 red tulips
available for the bouquets, what is the greatest number
of bouquets that can be made using all the tulips
available?
(A)

3

(B)

5

(C)

8

(D)

10

(E)

13

Arithmetic Applied problems; Properties of
numbers

Because all the tulips are to be used and the same
number of white tulips will be in each bouquet,
the number of white tulips in each bouquet times
the number of bouquets must equal the total
number of white tulips, or 15. Thus, the number
of bouquets must be a factor of 15, and so the
number must be 1, 3, 5, or 15. Also, the number
of red tulips in each bouquet times the number
of bouquets must equal the total number of red
184

(A)

5.125

(B)

5.25

(C)

6

(D)

6.125

(E)

6.25

Arithmetic Percents

The correct answer is A.
5.

125% of 5 =

125% of 5 represents

125
× 5, or 1.25 × 5 = 6.25.
100

The correct answer is E.
7.

Today Rebecca, who is 34 years old, and her daughter,
who is 8 years old, celebrate their birthdays. How
many years will pass before Rebecca’s age is twice her
daughter’s age?
(A)

10

(B)

14

(C)

18

(D)

22

(E)

26

Algebra Applied problems

Let x be the desired number of years. In x years,
Rebecca will be 34 + x years old and her
daughter will be 8 + x years old. From the given
information, it follows that 34 + x = 2(8 + x). The
last equation is equivalent to 34 + x = 16 + 2x,
which has solution x = 18.
The correct answer is C.

5.5 Problem Solving Answer Explanations

8.

When traveling at a constant speed of 32 miles per
hour, a certain motorboat consumes 24 gallons of fuel
per hour. What is the fuel consumption of this boat at
this speed measured in miles traveled per gallon of
fuel?

10.

A technician makes a round‐trip to and from a certain
service center by the same route. If the technician
completes the drive to the center and then completes
10 percent of the drive from the center, what percent
of the round‐trip has the technician completed?

(A)

2
3

(A)

5%

(B)

10%

(B)

3
4

(C)

25%

(D)

40%

(C)

4
5

(E)

55%

(D)

4
3

(E)

3
2

Arithmetic Percents

In completing the drive to the service center, the
technician has completed 50% of the round‐trip.
The drive from the center is the other 50% of
the round‐trip. In completing 10% of the drive
from the center, the technician has completed
an additional 10% of 50%, or 5% of the
round‐trip. Thus, the technician has completed
50% + 5% = 55% of the round‐trip.

Arithmetic Operations with rational numbers

If the motorboat consumes 24 gallons of fuel
in 1 hour, then it consumes 1 gallon of fuel in
1
hour. If the motorboat travels 32 miles in
24
1
32 4
hour,
1 hour, then it travels
= miles in
24
24 3
which is the length of time it takes to consume
1 gallon of fuel. Thus, the motorboat travels
4
miles per gallon of fuel.
3

The correct answer is E.
11.

Raffle tickets numbered consecutively from 101
through 350 are placed in a box. What is the
probability that a ticket selected at random will have a
number with a hundreds digit of 2 ?

The correct answer is D.
(A)
9.

A case contains c cartons. Each carton contains
b boxes, and each box contains 100 paper clips.
How many paper clips are contained in 2 cases?

(B)
(C)

(A)

100bc

(B)

100b
c

(D)

(C)

200bc

(E)

(D)

200b
c

(E)

200
bc

Algebra Simplifying algebraic expressions

Each case has bc boxes, each of which has
100 paper clips. The total number of paper clips
in 2 cases is thus 2( bc )(100) = 200 bc .
The correct answer is C.

2
5
2
7
33
88
99
250
100
249

Arithmetic Probability

There are 250 integers from 101 to 350 inclusive,
100 of which (that is, 200 through 299) have a
hundreds digit of 2. Therefore, the probability that
a ticket selected from the box at random will have
100 2
a hundreds digit of 2 can be expressed as
= .
250 5
The correct answer is A.

185

GMAT® Official Guide 2018

12.

When Leo imported a certain item, he paid a 7 percent
import tax on the portion of the total value of the item in
excess of $1,000. If the amount of the import tax that
Leo paid was $87.50, what was the total value of the item?

14.

(A)

$1,600

(B)

$1,850

(C)

$2,250

(D)

$2,400

I.

2

(E)

$2,750

II.

4

Algebra First-degree equations

III.

5

Letting x represent the total value of the item,
convert the words to symbols and solve the equation.

(A)

II only

(B)

III only

7% of value in excess of $1, 000 = 87.50

(C)

I and II only

(D)

II and III only

(E)

I, II, and III

0.07( x − 1, 000) = 87.50
x − 1, 000 = 1, 250
x = 2, 250

Arithmetic Statistics

Listed in numerical order, the given numbers are
2, 3, 4, 6, 7, and 8. If the seventh number were 2
or 4, then the numbers in numerical order would
be 2, 2, 3, 4, 6, 7, and 8 or 2, 3, 4, 4, 6, 7, and 8.
In either case the median would be 4 and the
2 + 2 + 3 + 4 + 6 + 7 + 8 32
=
or
average would be
7
7
2 + 3 + 4 + 4 + 6 + 7 + 8 34
=
, neither of which
7
7
equals 4. So, for neither of the values in I or
II does the average equal the median. If the
seventh number were 5, then the numbers in
numerical order would be 2, 3, 4, 5, 6, 7, and 8.
The median would be 5 and the average would
2 + 3 + 4 + 5 + 6 + 7 + 8 35
=
= 5. Thus, for the
be
7
7
value in III, the average equals the median.

The correct answer is C.
13.

A collection of 16 coins, each with a face value of either
10 cents or 25 cents, has a total face value of $2.35.
How many of the coins have a face value of 25 cents?
(A)

3

(B)

5

(C)

7

(D)

9

(E)

11

Algebra First-degree equations

Let x represent the number of coins each with
a face value of 25 cents. Then, since there are
16 coins in all, 16 – x represents the number of
coins each with a face value of 10 cents. The total
face value of the coins is $2.35 or 235 cents so,
25x + 10(16 – x)
25x + 160 – 10x
15x + 160
15x
x

186

The numbers of cars sold at a certain dealership on
six of the last seven business days were 4, 7, 2, 8, 3,
and 6, respectively. If the number of cars sold on the
seventh business day was either 2, 4, or 5, for which
of the three values does the average (arithmetic mean)
number of cars sold per business day for the seven
business days equal the median number of cars sold
per day for the seven days?

=
=
=
=
=

235
235
235
75
5

given
distributive property
combine like terms
subtract 160 from both sides
divide both sides by 15

The correct answer is B.
15.

If it is assumed that 60 percent of those who receive a
questionnaire by mail will respond and 300 responses
are needed, what is the minimum number of
questionnaires that should be mailed?
(A)

400

(B)

420

Therefore, 5 of the coins have a face value of
25 cents.

(C)

480

(D)

500

The correct answer is B.

(E)

600

5.5 Problem Solving Answer Explanations

Arithmetic Percents

expression in C cannot be the greatest. Finally,
the value of the expression in E is greater than
the value of the expression in D, since z > y
implies xz > xy, which implies xz + yz > xy + yz,
which implies z(x + y) > y(x + z), and so the value
of the expression in D cannot be the greatest.
Since none of the values of the expressions in A,
B, C, or D can be the greatest, it follows that the
value of the expression in E is the greatest.

From the given information, 60% of the
minimum number of questionnaires is equal to
300. This has the form “60% of what number
equals 300,” and the number can be determined
by dividing 300 by 60%. Performing the
100
60
= 300 ×
= 500.
calculation gives 300 ÷
100
60
The correct answer is D.

The correct answer is E.
16.

If 1 < x < y < z, which of the following has the greatest
value?
(A)

z(x + 1)

(B)

z(y + 1)

(C)

x(y + z)

(D)

y(x + z)

(E)

z(x + y)

17.

A rectangular garden is to be twice as long as it is
wide. If 360 yards of fencing, including the gate, will
completely enclose the garden, what will be the length
of the garden, in yards?
(A)

120

(B)

140

(C)

160

Algebra Inequalities

(D)

180

This problem can be solved by calculating each
of the options for a fixed and appropriate choice
of values for each of the variables. For example, if
x = 2, y = 3, and z = 4, then 1 < x < y < z and the
values of the options are as follows:

(E)

200

Geometry Quadrilaterals; Perimeter

Let the width of the rectangle be x. Then the
length is 2x. Since the perimeter of a rectangle is
twice the sum of the length and width, it follows
that
360 = 2 ( x + 2x )

A

z(x + 1) = 4(2 + 1) = 12

B

z(y + 1) = 4(3 + 1) = 16

C

x(y + z) = 2(3 + 4) = 14

D

y(x + z) = 3(2 + 4) = 18

360 = 6x
120 = 2x

E

z(x + y) = 4(2 + 3) = 20

So, the length is 120.

This problem can also be solved by the use of
algebraic ordering principles in a way that does
not assume the same answer option is true
for each choice of values of x, y, and z such
that 1 < x < y < z. First, note that neither the
value of the expression in A nor the value of
the expression in B can ever have the greatest
value, since the value of the expression in E is
greater than the value of the expression in A
(because y > 1) and the value of the expression
in E is greater than the value of the expression
in B (because x > 1). Also, the value of the
expression in D is greater than the value of the
expression in C, since y > x implies yz > xz,
which implies xy + yz > xy + xz, which implies
y(x + z) > x(y + z), and so the value of the

The correct answer is A.
18.

A rectangular floor that measures 8 meters by
10 meters is to be covered with carpet squares that
each measure 2 meters by 2 meters. If the carpet
squares cost $12 apiece, what is the total cost for the
number of carpet squares needed to cover the floor?
(A)

$200

(B)

$240

(C)

$480

(D)

$960

(E)

$1,920

187

GMAT® Official Guide 2018

given that F + Np + Nh = 140, Np = Nh + 20, and
F = 2Np = 2(Nh + 20). It follows that

Geometry Area (Rectangles)

The area of the floor is (10 m)(8 m) = 80
and
the area of each carpet square is (2 m)(2 m) = 4 m2.
Therefore, the number of carpet squares needed
80
to cover the floor is
= 20, and these 20 carpet
4
squares have a total cost of (20)($12) = $240.
m2

F + Np + Nh = 140 given
2(Nh + 20) + (Nh + 20) + Nh = 140 by substitution
4Nh + 60 = 140 combine like
terms
4Nh = 80 subtract 60
from both sides
Nh = 20 divide both
sides by 4

The correct answer is B.
19.

If 893 × 78 = p, which of the following is equal to
893 × 79 ?
(A)

p+1

(B)

p + 78

(C)

p + 79

(D)

p + 893

(E)

p + 894

The correct answer is B.
21.

Arithmetic Properties of numbers

893 × 79 = 893 × (78 + 1)
since 79 = 78 + 1
= (893 × 78) + 893 distributive property
= p + 893
since p = 893 × 78

10

(B)

20

(C)

30

(D)

40

(E)

50

Algebra Simultaneous first-degree equations

Let F represent the number of paperback fiction
books that Thabo owns; Np, the number of
paperback nonfiction books; and Nh, the number
of hardcover nonfiction books. It is

188

19

(B)

20

(C)

21

(D)

22

(E)

29

From the given information and the
definition of average, it follows that
3 + 15 + 32 + ( N + 1)
51 + N
= 18, or
= 18.
4
4
Multiplying both sides of the last equation by 4
gives 51 + N = 72. Therefore, N = 72 – 51 = 21.

Thabo owns exactly 140 books, and each book is
either paperback fiction, paperback nonfiction, or
hardcover nonfiction. If he owns 20 more paperback
nonfiction books than hardcover nonfiction books, and
twice as many paperback fiction books as paperback
nonfiction books, how many hardcover nonfiction
books does Thabo own?
(A)

(A)

Arithmetic Statistics

The correct answer is D.
20.

If the average (arithmetic mean) of the four numbers 3,
15, 32, and (N + 1) is 18, then N =

The correct answer is C.
22.

Abdul, Barb, and Carlos all live on the same straight
road, on which their school is also located. The school
is halfway between Abdul’s house and Barb’s house.
Barb’s house is halfway between the school and
Carlos’s house. If the school is 4 miles from Carlos’s
house, how many miles is Abdul’s house from Carlos’s
house?
1
3

(A)

1

(B)

2

(C)

4

(D)

6

(E)

8

5.5 Problem Solving Answer Explanations

Geometry Applied problems

In the diagram, A represents the location of
Abdul’s house, S represents the location of the
school, B represents the location of Barb’s house,
and C represents the location of Carlos’s house.
Because the school is halfway between Abdul’s
house and Barb’s house, S is the midpoint of AB ,
and because Barb’s house is halfway between
the school and Carlos’s house, B is the midpoint
of SC. Therefore, AS = SB = BC. Finally, since
SC = 4, it follows that AS = SB = BC = 2 and
hence AC = 2 + 2 + 2 = 6.

A

S

B

shows that dividing these two known rates
leads to the appropriate unit cancellations:
mi 2 hr
mi
mi 5 gal
60
÷
= 60
×
= 24
.
hr 2 hr
hr 5 gal
gal
Therefore, after using 3.75 gal of fuel, the car has
mi
traveled 24
× 3.75 gal = 90 miles.
gal
The correct answer is E.
24.

C

The correct answer is D.
23.

During a certain time period, Car X traveled north
along a straight road at a constant rate of 1 mile per
minute and used fuel at a constant rate of 5 gallons
every 2 hours. During this time period, if Car X used
exactly 3.75 gallons of fuel, how many miles did Car X
travel?

Cheryl purchased 5 identical hollow pine doors and
6 identical solid oak doors for the house she is
building. The regular price of each solid oak door
was twice the regular price of each hollow pine door.
However, Cheryl was given a discount of 25% off the
regular price of each solid oak door. If the regular price
of each hollow pine door was $40, what was the total
price of all 11 doors?
(A)

$320

(B)

$540

(C)

$560

(D)

$620

(E)

$680

(A)

36

(B)

37.5

Algebra Applied problems; Percents

(C)

40

(D)

80

(E)

90

The price of each pine door is $40, so the price of
5 pine doors is 5($40) = $200. The price of each
oak door is twice that of a pine door, and thus
$80, which becomes (0.75)($80) = $60 when the
25% discount is applied. Therefore, the price of
6 oak doors at the 25% discount is 6($60) = $360,
and hence the total price of all 11 doors is
$200 + $360 = $560.

Arithmetic Applied problems

The car traveled at a rate of
mi ⎛ mi ⎞ ⎛
min ⎞
mi
1
= ⎜1
60
= 60
using fuel
⎟
⎜
min ⎝ min ⎠ ⎝
hr ⎟⎠
hr
5 gal
. If
at a rate of 5 gallons every 2 hours, or
2 hr
the usage rate in miles per gallon was known,
then that usage rate times 3.75 gallons would give
the number of miles.
One approach to finding this usage rate
is to consider whether multiplying or
mi
5 gal
and
dividing the known rates 60
hr
2 hr
will lead to unit cancellations that result in
miles per gallon. The following calculation

The correct answer is C.
25.

If y −

1 11
< , which of the following could be a value
2
2

of y ?
(A)

−11

(B)

−

11
2

(C)

11
2

(D)

11

(E)

22

189

GMAT® Official Guide 2018

Algebra Inequalities; Absolute value

Algebra Simplifying algebraic expressions

1 11
<
is equivalent to
2
2
11
1 11
10
12
− < y − < , or −
< y < , select the
2
2 2
2
2
10
12
= 6.
value that lies between − = −5 and
2
2
11
That value is .
2

One method of solving this is to first take
the nonnegative square root of both sides of
the equation k 2 = m 2 and then make use of the
fact that u 2 = u . Doing this gives |k| = |m|.
Alternatively, if (k, m) is equal to either of
the pairs (1,1) or (–1,1), then k 2 = m 2 is true.
However, each of the answer choices except
|k| = |m| is false for at least one of these two
pairs.

Since y −

The correct answer is C.

The correct answer is E.
B
28.

120°
A
26.

D

C

In the figure shown, AC = 2 and BD = DC = 1. What is
the measure of angle ABD ?

$52

(B)

$117

(C)

$130

(D)

$180

(E)

$234

15°

(B)

20°

Arithmetic Ratio and proportion

(C)

30°

(D)

40°

(E)

45°

Makoto, Nishi, and Ozuro worked a total of
15 + 20 + 30 = 65 hours and were paid a total of
$780. Each was paid in proportion to the number
of hours he or she worked. Therefore, Makoto was
15
paid ($780) = $180.
65

Since AC = AD + DC, and it is given that AC = 2
and DC = 1, it follows that AD = 1. Therefore,
AD = BD = 1 and ΔABD is an isosceles triangle
where the measure of ∠ABD is equal to the
measure of ∠BAD. Letting x° be the common
degree measure of these two angles, it follows that
x° + x° + 120° = 180°, or 2x° = 60°, or x° = 30°.
The correct answer is C.

190

(A)

(A)

Geometry Triangles

27.

Makoto, Nishi, and Ozuro were paid a total of $780
for waxing the floors at their school. Each was paid in
proportion to the number of hours he or she worked. If
Makoto worked 15 hours, Nishi worked 20 hours, and
Ozuro worked 30 hours, how much was Makoto paid?

If k 2 = m 2, which of the following must be true?
(A)

k=m

(B)

k = −m

(C)

k = |m|

(D)

k = −|m|

(E)

|k| = |m|

The correct answer is D.

5.5 Problem Solving Answer Explanations

P

8

1
of the total. It follows that
15
the total is (15)($6) = $90.

Q

so the $6 left was

6

The correct answer is C.
R
31.
29.

The figure above shows a path around a triangular
piece of land. Mary walked the distance of 8 miles
from P to Q and then walked the distance of 6 miles
from Q to R. If Ted walked directly from P to R, by what
percent did the distance that Mary walked exceed the
distance that Ted walked?
(A)

30%

(B)

40%

(C)

50%

(D)

60%

(E)

80%

The correct answer is B.
30.

1
At a supermarket, John spent of his money on fresh
2
1
1
fruits and vegetables, on meat products, and
3
10
on bakery products. If he spent the remaining $6 on
candy, how much did John spend at the supermarket?
(A)

$60

(B)

$80

(C)

$90

(D)

$120

(E)

$180

(A)

–400

(B)

–140

(C)

–4

(D)

4

(E)

400

Algebra Operations with rational numbers

1
Since (1 – 1.25)N = –0.25N = − N, the equation
4
1
becomes – N = 1, which has solution N = –4.
4
The correct answer is C.

Geometry Pythagorean theorem

Mary walked a distance of 6 + 8 = 14 miles.
The distance that Ted walked, PR, can be
found by using the Pythagorean theorem
6 2 + 8 2 = ( PR )2 , or ( PR )2 = 100. Taking square
roots, it follows that Ted walked 10 miles.
Therefore, the distance Mary walked exceeded the
distance Ted walked by 14 – 10 = 4 miles and 4 is
40% of 10.

If (1 – 1.25)N = 1, then N =

32.

A carpenter constructed a rectangular sandbox with
a capacity of 10 cubic feet. If the carpenter were to
make a similar sandbox twice as long, twice as wide,
and twice as high as the first sandbox, what would be
the capacity, in cubic feet, of the second sandbox?
(A)

20

(B)

40

(C)

60

(D)

80

(E)

100

Geometry Volume

When all the dimensions of a three‐dimensional
object are changed by a factor of 2, the
capacity, or volume, changes by a factor of
(2)(2)(2) = 23 = 8. Thus the capacity of the second
sandbox is 10(8) = 80 cubic feet.
The correct answer is D.

Arithmetic Fractions

The amount spent was
1 1 1 15 10 3 28 14
+ +
=
+
+
=
=
of the total
2 3 10 30 30 30 30 15
191

GMAT® Official Guide 2018

33.

2
The quotient when a certain number is divided by is
3
9
. What is the number?
2

35.

(A)

1

4
27

(B)

4
3

(B)

1
3

(C)

17
5

(C)

3

(D)

18
5

(D)

6

(E)

4

(E)

27
4

Algebra Simultaneous equations

Recall the definition of division: a ÷ b = c means
a = bc. For example, if a certain number divided
by 2 equals 3, then the number is 2 × 3 = 6. For
2
this problem, a certain number divided by
3
2 9
9
equals , so the number is × = 3.
3 2
2
This problem can also be solved by algebra. Let N
2 9
N 9
be the number. Then
= , or N = × = 3.
2 2
3 2
3
The correct answer is C.
If a sphere with radius r is inscribed in a cube with
edges of length e, which of the following expresses the
relationship between r and e ?
1
e
2

(A)

r=

(B)

r=e

(C)

r = 2e

(D)

r= e

(E)

r=

1 2
e
4

Adding the equations 2x + y = 7 and x + 2y = 5
gives 3x + 3y = 12, or x + y = 4. Dividing both
x+ y 4
sides of the last equation by 3 gives
= .
3
3
The correct answer is B.
36.

City X has a population 4 times as great as the
population of City Y, which has a population twice as
great as the population of City Z. What is the ratio of
the population of City X to the population of City Z ?
(A)

1:8

(B)

1:4

(C)

2:1

(D)

4:1

(E)

8:1

Arithmetic Ratio and Proportion

Let X, Y, and Z be the populations of Cities X,
Y, and Z, respectively. It is given that X = 4Y,
4Y
Y
X
and Y = 2Z or Z = . Then,
=
=
Y
2
Z
⎛ 2⎞ 8
(4Y)⎜ ⎟ = .
2
⎝Y ⎠ 1
The correct answer is E.

Geometry Volume

A sphere inscribed in a cube touches, but does
not extend beyond, each of the 6 sides of the
cube, and thus the diameter of the sphere is equal
to the distance between a pair of opposite sides of
the cube, which is equal to the edge length of the
cube. Therefore, the radius of the sphere is equal
1
to half the edge length of the cube, or r = e.
2
The correct answer is A.
192

x+y
=
3

(A)

Arithmetic Operations with rational numbers

34.

If 2x + y = 7 and x + 2y = 5, then

5.5 Problem Solving Answer Explanations

Arithmetic Applied problems

Tides at Bay Cove on July 13

The expected number of defective units is
between 0.3% and 0.5% of 20,000, or between
(0.003)(20,000) = 60 and (0.005)(20,000) = 100.
Since each unit has a retail price of $2,500, the
amount of money needed to cover the refunds
for the expected number of defective units is
between 60($2,500) and 100($2,500), or between
$150,000 and $250,000.

height of tide

11:30 a.m.
2.2 ft
3

4:30 a.m.
0.3 ft

2
1
0

time

–1

6 a.m. noon

midnight
6 p.m.
–0.5 ft

37.

The correct answer is D.
40ft

The graph above shows the height of the tide, in feet,
above or below a baseline. Which of the following is
closest to the difference, in feet, between the heights
of the highest and lowest tides on July 13 at Bay Cove?

15ft

patio
35ft

house

38.

(A)

1.7

(B)

1.9

(C)

2.2

(D)

2.5

(E)

2.7

20ft

39.

A flat patio was built alongside a house as shown in the
figure above. If all angles shown are right angles, what
is the area of the patio in square feet?

Arithmetic Interpretation of graphs and tables

(A)

800

From the graph, the highest tide is 2.2 ft above
the baseline and the lowest tide is 0.5 ft below the
baseline. Therefore, the difference between the
heights of the highest tide and the lowest tide
is [2.2 – (–0.5)] ft = (2.2 + 0.5) ft = 2.7 ft.

(B)

875

(C)

1,000

(D)

1,100

(E)

1,125

The correct answer is E.

Geometry Area

A manufacturer of a certain product can expect that
between 0.3 percent and 0.5 percent of the units
manufactured will be defective. If the retail price is
$2,500 per unit and the manufacturer offers a full
refund for defective units, how much money can the
manufacturer expect to need to cover the refunds on
20,000 units?
(A)

Between $15,000 and $25,000

(B)

Between $30,000 and $50,000

(C)

Between $60,000 and $100,000

(D)

Between $150,000 and $250,000

(E)

Between $300,000 and $500,000

40
15

35

20
20

The area of the patio can be calculated by
imagining the patio to be a rectangle of dimensions
40 ft by 35 ft that has a lower‐left square corner of
dimensions 20 ft by 20 ft covered up, as shown in
the figure above. The area of the patio will be the
area of the uncovered part of the rectangle, and
therefore the area of the patio, in square feet, is
(40)(35) – (20)(20) = 1,400 – 400 = 1,000.
193

GMAT® Official Guide 2018

40

40

15

35

20

41.

15

35

20

20

20

Alternatively, the area of the patio can be
calculated by dividing the patio into two
rectangles and adding the areas of the two
rectangles. This can be done in two ways:
by using rectangles of dimensions 20 ft by
35 ft and 20 ft by 15 ft, as shown in the figure
above on the left (for a total area of
700 ft2 + 300 ft2 = 1,000 ft2), or by using
rectangles of dimensions 40 ft by 15 ft
and 20 ft by 20 ft, as shown in the figure
above on the right (for a total area of
600 ft2 + 400 ft2 = 1,000 ft2).

194

$52.50

(B)

$55.00

(C)

$57.50

(D)

$62.50

(E)

$65.00

80

(B)

82

(C)

84

(D)

86

(E)

88

The average of the student’s first 4 test scores
is 78, so the sum of the first 4 test scores is
4 ( 78 ) = 312. If x represents the fifth test score,
then the sum of all 5 test scores is 312 + x and
312 + x
. But the
the average of all 5 test scores is
average of all 5 test scores is 80 so 5
312 + x
= 80
5
312 + x = 400
x = 88

The sum of the weekly salaries of 5 employees is
$3,250. If each of the 5 salaries is to increase by
10 percent, then the average (arithmetic mean) weekly
salary per employee will increase by
(A)

(A)

Arithmetic Statistics

The correct answer is C.
40.

A student’s average (arithmetic mean) test score on
4 tests is 78. What must be the student’s score on a
5th test for the student’s average score on the 5 tests
to be 80 ?

The correct answer is E.
42.

Last week Chris earned x dollars per hour for the first
40 hours worked plus 22 dollars per hour for each
hour worked beyond 40 hours. If last week Chris
earned a total of 816 dollars by working 48 hours,
what is the value of x ?

Arithmetic Applied problems; Percents

(A)

13

Let S1, S2, S3, S4, and S5 be the salaries,
in dollars, of the 5 employees. Since
the sum of the 5 salaries is 3,250, then
S1 + S2 + S3 + S4 + S5 = 3,250 and the average
S + S 2 + S3 + S4 + S5 3, 250
=
salary is 1
= 650.
5
5
After each salary is increased by 10%, the salaries
will be (1.1)S1, (1.1)S2, (1.1)S3, (1.1)S4, and
(1.1)S5 and the average salary, in dollars, will be
(1.1)S1 + (1.1)S 2 + (1.1)S3 + (1.1)S4 + (1.1)S5
=
5
⎛ S + S + S3 + S4 + S5 ⎞
1.1 × ⎜ 1 2
⎟⎠ = 1.1 × 650 = 715.
⎝
5
Therefore, the increase in the average salary is
$715 – $650 = $65.

(B)

14

(C)

15

(D)

16

(E)

17

The correct answer is E.

The correct answer is D.

Algebra Applied problems

Chris worked 40 hours at a rate of $x per hour,
48 – 40 = 8 hours at a rate of $22 per hour, and
earned a total of $816.
40x + 8(22)
40x + 176
40x
x

=
=
=
=

816
816
640
16

given information
multiply 8 and 22
subtract 176 from both sides
divide both sides by 40

5.5 Problem Solving Answer Explanations

For the more mathematically inclined, consider the
remainder when each prime number n greater than
3 is divided by 6. The remainder cannot be 0 because
that would imply that n is divisible by 6, which is
impossible since n is a prime number. The remainder
cannot be 2 or 4 because that would imply that n is
even, which is impossible since n is a prime number
greater than 3. The remainder cannot be 3 because
that would imply that n is divisible by 3, which is
impossible since n is a prime number greater than
3. Therefore, the only possible remainders when a
prime number n greater than 3 is divided by 6 are
1 and 5. Thus, n has the form 6q + 1 or 6q + 5,
where q is an integer, and, therefore, n2 has the
form 36q2 + 12q + 1 = 12(3q2 + q) + 1 or
36q2 + 60q + 25 = 12(3q2 + 5q + 2) + 1. In either
case, n2 has a remainder of 1 when divided by 12.

B
y°

(y + 10)°
A
43.

C

In the figure above, what is the ratio of the measure of
angle B to the measure of angle A ?
(A)

2 to 3

(B)

3 to 4

(C)

3 to 5

(D)

4 to 5

(E)

5 to 6

Geometry Angles

Because the sum of the degree measures of the
three interior angles of a triangle is 180, it follows
that y + (y + 10) + 90 = 180. Therefore, 2y = 80,
and hence y = 40. The ratio of the measure of
angle B to the measure of angle A can now be
y
40 4
=
= .
determined:
y + 10 50 5

The correct answer is B.
45.

1
1
1+
3

−

1
3

(B)

−

1
6

−

(C)
If n is a prime number greater than 3, what is the
remainder when n2 is divided by 12 ?
(A)

0

(B)

1

(C)

2

(D)

3

(E)

5

Arithmetic Properties of numbers

The simplest way to solve this problem is to
choose a prime number greater than 3 and divide
its square by 12 to see what the remainder is. For
example, if n = 5, then n2 = 25, and the remainder
is 1 when 25 is divided by 12. A second prime
number can be used to check the result. For
example, if n = 7, then n2 = 49, and the remainder
is 1 when 49 is divided by 12. Because only one of
the answer choices can be correct, the remainder
must be 1.

1
1+
2

(A)

The correct answer is D.
44.

1

−

=

1
12

(D)

1
12

(E)

1
3

Arithmetic Operations with rational numbers

Perform the arithmetic calculations as follows:
1
1+

1
3

−

1

1 1
−
1 4 3
1+
2 3 2
3 2
= −
4 3
9 8
=
−
12 12
9−8
=
12
1
=
12
=

The correct answer is D.
195

GMAT® Official Guide 2018

46.

Algebra Algebraic expressions

The positive two‐digit integers x and y have the same
digits, but in reverse order. Which of the following must
be a factor of x + y ?
(A)

6

(B)

9

(C)

10

If s oranges are packed r oranges to a box with no
oranges left over, then the number of boxes that
s
will be filled is . If n of these boxes are already
r
s
filled, then – n boxes remain to be filled.
r

(D)

11

The correct answer is E.

(E)

14
49.

Arithmetic Properties of numbers

Let m and n be digits. If x = 10m + n, then
y = 10n + m. Adding x and y gives
x + y = (10m + n) + (10n + m) = 11m + 11n =
11(m + n), and therefore 11 is a factor of x + y.

If 0 < a < b < c, which of the following statements must
be true?
I.

2a > b + c

II.

c–a>b–a

III.

c b
<
a a

(A)

I only

(B)

II only

(C)

III only

(D)

I and II

(E)

II and III

The correct answer is D.
47.

In a certain sequence of 8 numbers, each number
after the first is 1 more than the previous number. If
the first number is –5, how many of the numbers in the
sequence are positive?
(A)

None

(B)

One

Algebra Inequalities

(C)

Two

(D)

Three

(E)

Four

Given 0 < a < b < c, Statement I is not necessarily
true. If, for example, a = 1, b = 2, and c = 3, then
0 < a < b < c, but 2a = 2(1) < 2 + 3 = b + c.
Given 0 < a < b < c, then c > b, and subtracting
a from both sides gives c – a > b – a. Therefore,
Statement II is true.

Arithmetic Sequences

The sequence consists of eight consecutive
integers beginning with –5:
In this sequence exactly two of the numbers are
positive.

Given 0 < a < b < c, Statement III is not
necessarily true. If, for example, a = 1, b = 2, and
c 3 2 b
c = 3, then 0 < a < b < c, but = > = .
a 1 1 a

The correct answer is C.

The correct answer is B.

−5, − 4, − 3, − 2, − 1, 0, 1, 2

48.

A total of s oranges are to be packaged in boxes that
will hold r oranges each, with no oranges left over.
When n of these boxes have been completely filled,
what is the number of boxes that remain to be filled?
(A)
(B)
(C)
(D)
(E)

196

s – nr
n
s–
r
rs – n
s
–r
n
s
–n
r

50.

In the xy-plane, the origin O is the midpoint of line
segment PQ. If the coordinates of P are (r,s), what are
the coordinates of Q ?
(A)

(r,s)

(B)

(s,–r)

(C)

(–s,–r)

(D)

(–r,s)

(E)

(–r,–s)

5.5 Problem Solving Answer Explanations

Algebra Coordinate geometry

Monday Tuesday Wednesday Thursday

Let (x,y) be the coordinates of Q. The midpoint
⎛r + x s + y⎞
,
of (r,s) and (x,y) is ⎜
⎟ . Since the
⎝ 2
2 ⎠
r +x
= 0 and
midpoint is (0,0), it follows that
2
s+ y
= 0. Therefore, r + x = 0 and s + y = 0, or
2
x = –r and y = –s, or (x,y) = (–r,–s).
This problem can also be solved by observing that
Q is the reflection of P about the origin, and so
Q = (–r,–s) (i.e., change the sign of each of the
coordinates of P to obtain the coordinates of Q).
The correct answer is E.
51.

Which of the following equations is NOT equivalent to
10y2 = (x + 2)(x – 2) ?
(A)

30y2 = 3x2 – 12

(B)

20y2 = (2x – 4)(x + 2)

(C)

10y2 + 4 = x2

(D)

5y2 = x2 – 2

(E)

y2 =

2

x −4
10

Algebra Simplifying algebraic expressions

When x = 2 or x = –2, the equation becomes
10y2 = 0, or y = 0. Since, in the equation given in
(D), y does not become 0 when x = 2, it follows
that the equation given in (D) is not equivalent
to the given equation. Alternatively, when each of
the equations given in (A) through (E) is solved
for 10y2 in terms of x, only the resulting equation
in (D) fails to give an expression in terms of x
that is equivalent to (x + 2)(x – 2) = x2 – 4.
The correct answer is D.

52.

Company A

45

55

50

50

Company B

10

30

30

10

Company C

34

28

28

30

Company D

39

42

41

38

Company E

50

60

60

70

The table shows the numbers of packages shipped
daily by each of five companies during a 4‐day period.
The standard deviation of the numbers of packages
shipped daily during the period was greatest for which
of the five companies?
(A)

A

(B)

B

(C)

C

(D)

D

(E)

E

Arithmetic Statistics

Since the standard deviation of a data set is a
measure of how widely the data are scattered
about their mean, find the mean number of
packages shipped by each company and then
determine the company for which the data is
most widely scattered about its mean.
For Company A, the mean number of packages
45 + 55 + 2(50)
= 50. Two of the data
shipped is
4
points are each 50 and the other two each differ
from 50 by 5.
For Company B, the mean number of packages
2(10) + 2(30)
shipped is
= 20. Each of the data
4
points differs from 20 by 10. Thus, the data for
Company B is more widely scattered about its
mean of 20 than the data for Company A is
about its mean of 50.
For Company C, the mean number of packages
34 + 2(28) + 30
shipped is
= 30. One data point
4
is 30, two others each differ from 30 by only 2,
and the fourth data point differs from 30 by only 4.
Therefore, the data for Company C is not as
197

GMAT® Official Guide 2018

widely scattered about its mean of 30 as the data
for Company B is about its mean of 20.
For Company D, the mean number of packages
39 + 42 + 41 + 38
= 40. Two of
shipped is
4
the data points each differ from 40 by only 1
and the other two each differ from 40 by only
2. Therefore, the data for Company D is not as
widely scattered about its mean of 40 as the data
for Company B is about its mean of 20.
For Company E, the mean number of packages
50 + 2(60) + 70
shipped is
= 60. Two of the
4
data points are each 60 and the other two each
differ from 60 by 10. Therefore, the data for
Company E is not as widely scattered about its
mean of 60 as the data for Company B is about
its mean of 20.
Thus, the data for Company B is most widely
scattered about its mean and, therefore, the
standard deviation of the number of packages
shipped daily by the five companies is greatest for
Company B.

(50 − 60)2 + 2(60 − 60)2 + (70 − 60)2
4

For E:
=

200
= 50 .
4

The correct answer is B.
53.

Company Q plans to make a new product next year
and sell each unit of this new product at a selling price
of $2. The variable costs per unit in each production
run are estimated to be 40% of the selling price, and
the fixed costs for each production run are estimated
to be $5,040. Based on these estimated costs, how
many units of the new product will Company Q need to
make and sell in order for their revenue to equal their
total costs for each production run?
(A)

4,200

(B)

3,150

(C)

2,520

(D)

2,100

(E)

1,800

Algebra Applied problems

For those interested, the standard deviations for
the five companies can be calculated as follows:
(45 − 50)2 + (55 − 50)2 + 2(50 − 50)2
4

For A:
=

For B:

2(25)
=
4

25
= 12.5.
2

2(10 − 20)2 + 2(30 − 20)2
=
4

4(100)
4

= 100.
(34 − 30)2 + 2(28 − 30)2 + (30 − 30)2
For C:
4
24
=
= 6.
4
For D:
(39 − 40)2 + (42 − 40)2 + (41 − 40)2 + (38 − 40)2
4
10
=
= 2.5 .
4
198

Let x be the desired number of units to be sold
at a price of $2 each. Then the revenue for selling
these units is $2x, and the total cost for selling
these units is (40%)($2.00)x = $0.80x plus a fixed
cost of $5,040.
revenue = total cost
given requirement
2x = 0.8x + 5,040 given information
1.2x = 5,040
subtract 0.8x from both
sides
x = 4,200
divide both sides by 1.2
The correct answer is A.

5.5 Problem Solving Answer Explanations

Arithmetic Percents

S

The percent increase from the number in State D
to the number in State A is
E

A
D

⎛ 181, 000 − 79, 000
⎞
× 100 ⎟ % expression for
= ⎜⎝
⎠
79, 000
percent increase

B
C

54.

The dial shown above is divided into equal‐sized intervals.
At which of the following letters will the pointer stop if it
is rotated clockwise from S through 1,174 intervals?
(A)

A

(B)

B

(C)

C

(D)

D

(E)

E

⎛ 181 − 79
⎞
× 100 ⎟ %
=⎜
⎝ 79
⎠

reduce fraction

⎛ 102
⎞
=⎜
× 100 ⎟ %
⎝ 79
⎠

subtract

⎛ 100
⎞
× 100 ⎟ %
≈⎜
⎝ 80
⎠

approximate

= (1.25 × 100)%
= 125%

divide
multiply

The correct answer is D.

Arithmetic Properties of numbers

y

There are 8 intervals in each complete revolution.
Dividing 8 into 1,174 gives 146 with remainder
6. Therefore, 1,174 intervals is equivalent
to 146 complete revolutions followed by an
additional 6 intervals measured clockwise
from S, which places the pointer at E.

Estimated Number of Home-Schooled
Students by State, January 2001

I
x

O
III

56.

The correct answer is E.

55.

II

IV

The graph of the equation xy = k, where k < 0, lies in
which two of the quadrants shown above?
(A)

I and II

(B)

I and III

State

Number (in thousands)

(C)

II and III

A

181

(D)

II and IV

B

125

(E)

III and IV

C

103

Algebra Coordinate geometry

D

79

E

72

According to the table shown, the estimated number
of home-schooled students in State A is approximately
what percent greater than the number in State D ?

If a point lies on the graph of xy = k, then the
product of the point’s x‐ and y‐coordinates is k.
Since k is negative, it follows that for any such point,
the product of the point’s x‐ and y‐coordinates is
negative. Therefore, for any such point, the point’s
x‐ and y‐coordinates have opposite signs, and hence
the point must be in quadrant II or in quadrant IV.

(A)

25%

The correct answer is D.

(B)

55%

(C)

100%

(D)

125%

(E)

155%
199

GMAT® Official Guide 2018

57.

Geometry Area

When n liters of fuel were added to a tank that was
1
7
already full, the tank was filled to of its capacity. In
3
9
terms of n, what is the capacity of the tank, in liters?
(A)
(B)
(C)
(D)
(E)

From the given information it follows that
(100 + w)(150 + w) = 2(100)(150), or
(100 + w)(150 + w) = (200)(150). This is a
quadratic equation that can be solved by several
methods. One method is by inspection. The left
side is clearly equal to the right side when w = 50.
Another method is by factoring. Expanding the left
side gives (100)(150) + 250w + w2 = (200)(150), or
w2 + 250w – (100)(150) = 0. Factoring the left
side gives (w – 50)(w + 300) = 0, which has
w = 50 as its only positive solution.

10
n
9
4
n
3
3
n
2
9
n
4
7
n
3

The correct answer is B.
59.

Algebra Applied problems

Let C represent the capacity of the tank, in liters.
It follows that
1
7
C+n= C
3
9
7
1
n= C– C
9
3
n=

4
C
9

9
n=C
4

given
1
subtract C from both
3
sides

4
9

(B)

–0.25

(C)

0.25

(D)

0.75

(E)

4

1
1
=
0.75 − 1 3 − 1
4
1
=
1
−
4
= −4

w ft

100 ft
150 ft
w ft
Note: Not drawn to scale.

200

–4

Perform the arithmetic calculations as follows:

The correct answer is D.

58.

(A)

Arithmetic Operations with rational numbers

combine like terms
divide both sides by

1
=
0.75 − 1

The smaller rectangle in the figure above represents
the original size of a parking lot before its length and
width were each extended by w feet to make the larger
rectangular lot shown. If the area of the enlarged lot is
twice the area of the original lot, what is the value of w ?

The correct answer is A.
60.

Kevin invested $8,000 for one year at a simple annual
interest rate of 6 percent and invested $10,000 for
one year at an annual interest rate of 8 percent
compounded semiannually. What is the total amount of
interest that Kevin earned on the two investments?
(A)

$880

(B)

$1,088

(A)

25

(C)

$1,253

(B)

50

(D)

$1,280

(C)

75

(E)

$1,296

(D)

100

(E)

200

5.5 Problem Solving Answer Explanations

Arithmetic Applied problems

Since only one answer choice can be correct,
the other answer choices need not be checked.
However, for completeness:

The amount of interest after one year is the total
value of the investment after one year minus the
total initial value of the investment. The total
value of the investment after one year is
$8,000(1.06) + $10,000(1.04)2 = $8,480 + $10,816,
so the amount of interest is ($8,480 + $10,816) –
($8,000 + $10,000) = $480 + $816 = $1,296.

A

n + 1 is odd if n is even and even if n is odd.
Therefore, it is not true that n + 1 must be
even.

B

n + 2 is even if n is even and odd if n is odd.
Therefore, it is not true that n + 2 must be
even.

D

2n + 1 is odd whether n is even or odd.
Therefore, it is not true that 2n + 1 must be
even.

E

n2 is even if n is even and odd if n is odd.
Therefore, it is not true that n2 must be
even.

The correct answer is E.
61.

The harvest yield from a certain apple orchard was
350 bushels of apples. If x of the trees in the orchard
each yielded 10 bushels of apples, what fraction of the
harvest yield was from these x trees?
(A)

x
35

(B)

1−

(C)

10x

(D)

35 – x

(E)

350 – 10x

Algebra Algebraic expressions

Since each of the x trees yielded 10 bushels, the
total number of bushels yielded by these trees
was 10x. Since the yield of the entire orchard was
x
10 x
350 bushels,
was the fraction of the
=
350 35
total yield from these x trees.
The correct answer is A.
62.

The correct answer is C.

x
35

If n is an integer, which of the following must be even?
(A)

n+1

(B)

n+2

(C)

2n

(D)

2n + 1

(E)

n2

63.

The sum

(A)
(B)
(C)
(D)
(E)

7 1
+ is between
8 9

1
3
and
2
4
3
and 1
4
1
1 and 1
4
1
1
1 and 1
4
2
1
1 and 2
2

Arithmetic Operations with rational numbers

Since 1 < 1 , 7 + 1 < 7 + 1 = 1, and answer
9 8 8 9 8 8
choices C, D, and E can be eliminated. Since
7 > 6 = 3 , 7 + 1 > 3 , and answer choice A can
8 8 4 8 9 4
be eliminated. Thus, 3 < 7 + 1 < 1.
4 8 9
The correct answer is B.

Arithmetic Properties of integers

A quick look at the answer choices reveals the
expression 2n in answer choice C. 2n is a multiple
of 2 and hence must be even.

201

GMAT® Official Guide 2018

64.

Car X averages 25.0 miles per gallon of gasoline and
Car Y averages 11.9 miles per gallon. If each car is
driven 12,000 miles, approximately how many more
gallons of gasoline will Car Y use than Car X ?
(A)

320

(B)

480

(C)

520

(D)

730

(E)

920

below, it is clear that the least possible integer
value of |23 − 5y| is 2, which occurs when y = 5.
y |23 – 5y|
0
23
1
18
2
13
3
8
4
3
5
2
6
7
7
12

Arithmetic Applied problems

Car X uses 1 gallon of gasoline for every 25 miles
1
it is driven, so Car X uses
of a gallon for
25
every 1 mile it is driven. Therefore Car X will
⎛ 1⎞
use (12, 000 ) ⎜ ⎟ = 480 gallons of gasoline
⎝ 25 ⎠
when it is driven 12,000 miles. Car Y uses
119
miles
1 gallon of gasoline for every 11.9 or
10
10
it is driven, so Car Y uses
of a gallon for
119
every 1 mile it is driven. Therefore Car Y will use
10
10
(12, 000 ) ⎛⎜⎝ ⎞⎟⎠ ≈ (12, 000 ) ⎛⎜⎝ ⎞⎟⎠ = 1, 000 gallons
119
120
of gasoline when it is driven 12,000 miles. Thus,
Car Y will use approximately 1,000 – 480 = 520
more gallons of gasoline than Car X.
The correct answer is C.
65.

The correct answer is B.
66.

80 + 125 =
(A)

9 5

(B)

20 5

(C)

41 5

(D)

205

(E)

100

If y is an integer, then the least possible value of
|23 – 5y | is

Arithmetic Operations with radical expressions

(A)

1

(B)

2

Rewrite each radical in the form a b , where a
and b are positive integers and b is as small as
possible, and then add.

(C)

3

(D)

4

(E)

5

Arithmetic Absolute value; Operations with
integers

Since y is an integer, 23 − 5y is also an integer.
The task is to find the integer y for which |23 − 5y|
is the least. If y ≥ 0, −5y ≤ 0, and 23 − 5y ≤ 23. On
the other hand, if y ≤ 0, −5y ≥ 0, and 23 − 5y ≥ 23.
Therefore, the least possible value of |23 – 5y|
occurs at a nonnegative value of y. From the chart

202

Alternatively, since |23 – 5y| ≥ 0, the minimum
possible real value of |23 – 5y| is 0. The integer
value of y for which |23 – 5y| is least is the
integer closest to the solution of the equation
23
= 4.6 and the
23 – 5y = 0. The solution is y =
5
integer closest to 4.6 is 5.

80 + 125 = 16(5) + 25(5)
=

(

16

)( 5 ) + (

= 4 5+5 5
=9 5
The correct answer is A.

25

)( 5 )

5.5 Problem Solving Answer Explanations

y = kx + 3
67.

69.

In the equation above, k is a constant. If y = 17 when
x = 2, what is the value of y when x = 4 ?
(A)

34

(B)

31

(C)

14

(D)

11

(E)

7
(A)

1:00 p.m.

Algebra First-degree equations

(B)

2:00 p.m.

If y = kx + 3 and y = 17 when x = 2, then

(C)

3:00 p.m.

17 = 2k + 3

(D)

5:00 p.m.

14 = 2k

(E)

6:00 p.m.

7=k

Algebra Applied problems

Therefore, y = 7 x + 3. When x = 4,
y = 7(4) + 3 = 31.

Let t be the number of hours after 8:00 a.m.
that Ben drove. Then, in t hours, Ben
drove 20t miles and Al, who began driving
3 hours after Ben began driving, drove
40(t – 3) miles. Therefore, their combined
total distance at that time can be expressed as
(20t + 40t – 120) miles = (60t – 120) miles.
It follows that 60t – 120 = 240, or 60t = 360,
or t = 6, which corresponds to 6 hours after
8:00 a.m. or 2:00 p.m.

The correct answer is B.
68.

Al and Ben are drivers for SD Trucking Company. One
snowy day, Ben left SD at 8:00 a.m. heading east and
Al left SD at 11:00 a.m. heading west. At a particular
time later that day, the dispatcher retrieved data
from SD’s vehicle tracking system. The data showed
that, up to that time, Al had averaged 40 miles per
hour and Ben had averaged 20 miles per hour. It also
showed that Al and Ben had driven a combined total
of 240 miles. At what time did the dispatcher retrieve
data from the vehicle tracking system?

Which of the following is greatest?
(A)

10 3

(B)

9 4

(C)

8 5

(D)

7 6

(E)

6 7

Arithmetic Operations on radical expressions

Since all the expressions represent positive
numbers, the expression that has the greatest
squared value will be the expression that has the
greatest value.

(10 3 ) = 100 × 3 = 300 (not greatest)
(9 4 ) = 81 × 4 = 324 (greatest)
(8 5 ) = 64 × 5 = 320 (not greatest)
(7 6 ) = 49 × 6 = 294 (not greatest)
(6 7 ) = 36 × 7 = 252 (not greatest)

The correct answer is B.
70.

Of the land owned by a farmer, 90 percent was
cleared for planting. Of the cleared land, 40 percent
was planted with soybeans and 50 percent of the
cleared land was planted with wheat. If the remaining
720 acres of cleared land was planted with corn, how
many acres did the farmer own?

2

(A)

5,832

2

(B)

6,480

2

(C)

7,200

(D)

8,000

(E)

8,889

2
2

The correct answer is B.

203

GMAT® Official Guide 2018

71.

Arithmetic Applied problems; Percents

Arithmetic Operations with rational numbers

Corn was planted on 100% – (40% + 50%) = 10%
of the cleared land, and the cleared land
represents 90% of the farmer’s land. Therefore,
corn was planted on 10% of 90%, or
(0.10)(0.90) = 0.09 = 9%, of the farmer’s land. It
is given that corn was planted on 720 acres, so if
x is the number of acres the farmer owns, then
720
= 8, 000 .
0.09x = 720 and x =
0.09

1
1⎞
⎛ 1 1 1 1⎞
⎛1 1
+
+ ⎟
⎜⎝ + + + ⎟⎠ = r ⎜⎝ +
3 4 5 6
9 12 15 18 ⎠

The correct answer is D.

Alternatively,

⎛1 1 1
⎜⎝ + + +
3 4 5

23

(B)

32

(C)

2(310)

(D)

3(210)

(E)

3(102)

3=r

⎛1 1 1
⎜⎝ + + +
3 4 5

72.

204

1
1⎞
1⎞
⎛1 1
+
+ ⎟
⎟⎠ = r ⎜⎝ +
9 12 15 18 ⎠
6

12
10 ⎞
⎛ 20 15 12 10 ⎞
⎛ 20 15
+
+
+ ⎟ = r⎜
+
+
+
⎜⎝
⎝ 180 180 180 180 ⎟⎠
60 60 60 60 ⎠
⎛ 20 + 15 + 12 + 10 ⎞
⎛ 20 + 15 + 12 + 10 ⎞
⎜⎝
⎟⎠ = r ⎜⎝
⎟⎠
60
180
⎞
180
⎛ 20 + 15 + 12 + 10 ⎞ ⎛
=r
⎜⎝
⎟⎠ ⎜
60
⎝ 20 + 15 + 12 + 10 ⎟⎠
180
=r
60
3=r

Arithmetic Applied problems; Sequences

The population doubles each month, so multiply
the previous month’s population by 2 to get
the next month’s population. Thus, at the end
of the 1st month the population will be (3)(2),
at the end of the 2nd month the population
will be (3)(2)(2), at the end of the 3rd month
the population will be (3)(2)(2)(2), and so on.
Therefore, at the end of the 10th month the
population will be the product of 3 and ten
factors of 2, which equals 3(210).

The correct answer is C.
73.

If x and y are positive integers such that y is a multiple
of 5 and 3x + 4y = 200, then x must be a multiple of
which of the following?
(A)

3

(B)

6

(C)

7

(D)

8

The correct answer is D.

(E)

10

1
1⎞
⎛1 1
⎛ 1 1 1 1⎞
If ⎜ + + + ⎟ = r ⎜ +
+
+
, then r =
⎝ 3 4 5 6⎠
⎝ 9 12 15 18 ⎟⎠

Arithmetic Properties of numbers

(A)

1
3

(B)

4
3

(C)

3

(D)

4

(E)

12

1⎞
⎟
6⎠

⎛ 1⎞
1 = r⎜ ⎟
⎝ 3⎠

At the start of an experiment, a certain population
consisted of 3 animals. At the end of each month after
the start of the experiment, the population size was
double its size at the beginning of that month. Which of
the following represents the population size at the end
of 10 months?
(A)

1⎞
⎛ 1⎞ ⎛ 1 1 1
⎟⎠ = r ⎜⎝ ⎟⎠ ⎜⎝ + + +
6
3 3 4 5

Since it is given that y is a multiple of 5,
let y = 5q, where q is a positive integer. It is
given that 3x + 4y = 200, so 3x + 4(5q) = 200
or 3x + 20q = 200. It follows that
3x = 200 – 20q = 20(10 – q). Since 3 is prime and
is not a factor of 20, then 3 must be a factor of
10 – q, which means the only possible values

5.5 Problem Solving Answer Explanations

of q are 1, 4, and 7. This is summarized in the
following table.

75.

q 10 – q 20(10 – q) 3x x
1
9
180
180 60
4
6
120
120 40
7
3
60
60 20

(A)

80

(B)

96

Alternatively, since 3x = 20(10 – q), the factors
of the product of 3 and x must correspond to the
factors of the product of 20 and (10 – q). Since
the factors of 20 include 10, then x must have a
factor of 10, which means that x is a multiple of 10.

(C)

160

(D)

192

(E)

240

The correct answer is E.

For a certain value of x, the numbers of hours
worked on the project by the four staff members are
2x, 3x, 5x, and 6x, for a total of 16x. It is given that
one of these four numbers is equal to 30. If 2x = 30,
then x = 15 and 16x = 16(15) = 240, which is (E).
If 3x = 30, then x = 10 and 16x = 16(10) = 160,
which is (C). If 5x = 30, then x = 6 and
16x = 16(6) = 96, which is (B). If 6x = 30, then
x = 5 and 16x = 16(5) = 80, which is (A).

In each case, the value of x is a multiple of 10.

74.

Four staff members at a certain company worked
on a project. The amounts of time that the four staff
members worked on the project were in the ratio 2 to
3 to 5 to 6. If one of the four staff members worked
on the project for 30 hours, which of the following
CANNOT be the total number of hours that the four
staff members worked on the project?

Arithmetic Ratio and proportion

Which of the following expressions can be written as
an integer?
I.

(

II.

(82)

III.

(

(A)

None

(B)

I only

(C)

III only

(D)

I and II

(E)

I and III

82 + 82

(

82

82

)(

)

82

)

2

)

82

The correct answer is D.
76.

Arithmetic Operations with radical expressions

Expression I represents an integer because

(

82 + 82

) = (2
2

)

2

= (4)(82).

Pumping alone at their respective constant rates, one
1
inlet pipe fills an empty tank to of capacity in 3 hours
2
and a second inlet pipe fills the same empty tank to
2
of capacity in 6 hours. How many hours will it take
3
both pipes, pumping simultaneously at their respective
constant rates, to fill the empty tank to capacity?
(A)

3.25

Expression II does not represent an integer

(B)

3.6

because (82) 82 = 823 and 823 = 23 × 413 is
not a perfect square. Regarding this last assertion,
note that the square of any integer has the
property that each of its distinct prime factors is
repeated an even number of times. For example,
242 = (23 × 3)2 = 26 × 32 has the prime factor 2
repeated 6 times and the prime factor 3 repeated
twice. Expression III represents an integer,
82
82
82
because
=
= 1.
82
82

(C)

4.2

(D)

4.4

(E)

5.5

(

)(

82

)

The correct answer is E.
205

GMAT® Official Guide 2018

Arithmetic Applied problems

1
of the tank in 3 hours,
2
which is equivalent to the rate of filling
1
1
÷ 3 = of the tank per hour. The second
2
6
2
pipe can fill of the tank in 6 hours, which is
3
2
1
equivalent to the rate of filling ÷ 6 = of the
3
9
tank per hour. Together, they can fill the tank at
1 1
5
a rate of + =
of the tank per hour. Thus,
6 9 18
when both pipes are used at the same time, they
18
will fill the tank in
= 3.6 hours.
5
The correct answer is B.

78.

If x2 − 2 < 0, which of the following specifies all the
possible values of x ?

The first pipe can fill

77.

In the xy‐coordinate plane, which of the following points
must lie on the line kx + 3y = 6 for every possible value
of k ?
(A)

(1,1)

(B)

(0,2)

(C)

(2,0)

(D)

(3,6)

(E)

(6,3)

Algebra Coordinate geometry

Substituting the various answer choices for (x,y)
into kx + 3y = 6 gives the following equations:
A

k+3=6

B

0 + 3(2) = 6

C

2k + 3(0) = 6

D

3k + 3(6) = 6

E

6k + 3(3) = 6

Each of these, except for the equation in B, holds
for only one value of k. The equation in B does
not include k and therefore holds for every value
of k.
The correct answer is B.

206

(A)

0 2 . Choose x = –2, x = 0, and x = 2 from
these intervals, respectively, to test whether the
inequality holds. Then for these choices, the
inequality becomes (–2)2 – 2 < 0 (False),
(0)2 – 2 < 0 (True), and (2)2 – 2 < 0 (False).
Therefore, the solution consists of only the
interval − 2 < x < 2 . Alternatively, the graph
of y = x2 – 2 is easily seen to be a parabola that
opens upward with vertex at (0, –2) and
x‐intercepts at x = 2 and x = − 2 . The solution
to the inequality is the set of the x‐coordinates
of the portion of this parabola that lies below the
x‐axis, which is − 2 < x < 2 .
The correct answer is C.

5.5 Problem Solving Answer Explanations

Book 5: 4 days—50 pages on each of Days 17–19,
5 pages on Day 20 [3(50) + 5 = 155]

Book number

Pages in book

Total pages read

1

253

253

2

110

363

3

117

480

4

170

650

5

155

805

6

50

855

7

205

1,060

At this point, Shawana has read on a total of
28 days and has finished 8 books.

8

70

1,130

The correct answer is B.

9

165

1,295

10

105

1,400

11

143

1,543

12

207

1,750

Book 6: 1 day—50 pages on Day 21 [1(50) = 50]
Book 7: 5 days—50 pages on each of Days 22–25,
5 pages on Day 26 [4(50) + 5 = 205]
Book 8: 2 days—50 pages on Day 27, 20 pages on
Day 28 [50 + 20 = 70]

80.

79.

Shawana made a schedule for reading books during
4 weeks (28 days) of her summer vacation. She has
checked out 12 books from the library. The number of
pages in each book and the order in which she plans to
read the books are shown in the table above. She will
read exactly 50 pages each day. The only exception
will be that she will never begin the next book on the
same day that she finishes the previous one, and
therefore on some days she may read fewer than
50 pages. At the end of the 28th day, how many books
will Shawana have finished?

In Western Europe, x bicycles were sold in each of
the years 1990 and 1993. The bicycle producers of
Western Europe had a 42 percent share of this market
in 1990 and a 33 percent share in 1993. Which of the
following represents the decrease in the annual number
of bicycles produced and sold in Western Europe from
1990 to 1993?
(A)

9% of

x
100

(B)

14% of

x
100

(C)

75% of

x
100

(D)

9% of x

(E)

14% of x

Arithmetic Percents
(A)

7

(B)

8

(C)

9

(D)

10

(E)

11

Arithmetic Operations with integers

Book 1: 6 days—50 pages on each of Days 1–5, 3
pages on Day 6 [5(50) + 3 = 253]
Book 2: 3 days—50 pages on each of Days 7 and
8, 10 pages on Day 9 [2(50) + 10 = 110]
Book 3: 3 days—50 pages on each of Days 10 and
11, 17 pages on Day 12 [2(50) + 17 = 117]

Of the x bicycles sold in Western Europe in
1990, 42% of them were produced in Western
Europe. It follows that the number of bicycles
produced and sold in Western Europe in 1990
was 0.42x. Similarly, of the x bicycles sold in
Western Europe in 1993, 33% were produced in
Western Europe. It follows that the number of
bicycles produced and sold in Western Europe
in 1993 was 0.33x. Therefore, the decrease in
the annual number of bicycles produced and
sold in Western Europe from 1990 to 1993 was
0.42x – 0.33x = 0.09x, which is 9% of x.
The correct answer is D.

Book 4: 4 days—50 pages on each of Days 13–15,
20 pages on Day 16 [3(50) + 20 = 170]
207

GMAT® Official Guide 2018

81.

Arithmetic Fractions

If k is a positive integer, what is the remainder when
(k + 2)(k3 – k) is divided by 6 ?
(A)

0

(B)

1

(C)

2

(D)

3

(E)

4

1
Find the distance between each fraction and :
2
4 1
8
7
1
− =
−
=
7 2
14 14 14
5 1
10 9
1
− =
−
=
9 2
18 18 18
6 1
12 11
1
− =
−
=
11 2
22 22 22

Algebra Properties of numbers

Since k can be any positive integer, the remainder
must be the same regardless of the value of k. If
k = 2, for example, then (k + 2)(k 3 – k) =
(2 + 2)(23 – 2) = (4)(6), which is a multiple of 6,
and therefore, the remainder when divided by
6 is 0.

7 1
14 13
1
− =
−
=
13 2
26 26 26
9 1
9 8
1
− =
−
=
16 2
16 16 16
Each distance is the reciprocal of a positive
1
integer, the greatest of which is 26. Therefore,
26
7
is the least of the distances and
is the fraction
13
1
closest to .
2
Alternatively, asking which fraction is closest to
1
is equivalent to asking which of the fractions,
2
when doubled, is closest to 1. The fractions after
8 10 12 14
18 9
doubling are , , , , and
= . Each of
7 9 11 13
16 8
n+1
1
=1+
these fractions can be expressed as
n
n
for some integer n. Of these fractions, the one
1
closest to 1 is the one for which
is least, or in
n
other words, the one for which n is greatest. That
14
fraction is , and therefore the closest of the
13
1 ⎛ 1 ⎞ ⎛ 14 ⎞ 7
original fractions to is ⎜ ⎟ ⎜ ⎟ = .
2 ⎝ 2 ⎠ ⎝ 13 ⎠ 13

Alternatively, factor the given expression:
(k + 2)(k 3 – k) = (k + 2)(k)(k 2 – 1)
= (k + 2)(k)(k + 1)(k – 1)
Now, rearrange the factors in ascending order
(k – 1)(k)(k + 1)(k + 2), and observe that for any
positive integer k, the factors are 4 consecutive
integers, two of which are even and one of which
is divisible by 3. Therefore, (k + 2)(k 3 – k) is
divisible by both 2 and 3. Thus, (k + 2)(k 3 – k) is
divisible by 6 with 0 remainder.
The correct answer is A.
82.

Which of the following fractions is closest to
(A)
(B)
(C)
(D)
(E)

208

4
7
5
9
6
11
7
13
9
16

1
?
2

The correct answer is D.
83.

If p ≠ 0 and p –

1 − p2 r
= , then r =
p
p

(A)

p+1

(B)

2p – 1

(C)

p2 + 1

(D)

2p2 – 1

(E)

p2 + p – 1

5.5 Problem Solving Answer Explanations

Algebra Simplifying algebraic expressions

p–

1 − p2
r
=
p
p

p2 – (1 – p2) = r
2p2 – 1 = r

Algebra First-degree equations

Let x be the number that is 108 more than two2
thirds of itself. Then, 108 + x = x. Solve for x as
3
follows:

given
multiply both sides by p
combine like terms

2
108 + x = x
3
1
108 = x
3
324 = x

The correct answer is D.
84.

85.

If the range of the six numbers 4, 3, 14, 7, 10, and
x is 12, what is the difference between the greatest
possible value of x and the least possible value of x ?
(A)

0

(B)

2

(C)

12

(D)

13

(E)

15

The correct answer is E.
86.

A doctor prescribed 18 cubic centimeters of a certain
drug to a patient whose body weight was 120 pounds.
If the typical dosage is 2 cubic centimeters per
15 pounds of body weight, by what percent was the
prescribed dosage greater than the typical dosage?

Arithmetic Statistics

(A)

8%

The range of the six numbers 3, 4, 7, 10, 14, and
x is 12. If x were neither the greatest nor the
least of the six numbers, then the greatest and
least of the six numbers would be 14 and 3. But,
this cannot be possible because the range of the
six numbers would be 14 – 3 = 11 and not 12 as
stated. Therefore, x must be either the greatest or
the least of the six numbers. If x is the greatest
of the six numbers, then 3 is the least, and
x – 3 = 12. It follows that x = 15. On the other
hand, if x is the least of the six numbers, then 14
is the greatest, and 14 – x = 12. It follows that
x = 2. Thus, there are only two possible values of
x, namely 15 and 2, and so the difference between
the greatest and least possible values of x is
15 – 2 = 13.

(B)

9%

(C)

11%

(D)

12.5%

(E)

14.8%

The correct answer is D.

The correct answer is D.

Arithmetic Percents

If the typical dosage is 2 cubic centimeters per
15 pounds of body weight, then the typical
dosage for a person who weighs 120 pounds
⎛ 120 ⎞
is 2⎜
= 2(8) = 16 cubic centimeters. The
⎝ 15 ⎟⎠
prescribed dosage of 18 cubic centimeters is,
⎛ ⎛ 18 − 16 ⎞
⎞
therefore, ⎜ ⎜
× 100 ⎟ % or 12.5% greater
⎟
⎝ ⎝ 16 ⎠
⎠
than the typical dosage.

(A)

72

(B)

144

Company P had 15 percent more employees in
December than it had in January. If Company P had
460 employees in December, how many employees
did it have in January?

(C)

162

(A)

391

(D)

216

(B)

400

(E)

324

(C)

410

(D)

423

(E)

445

What number is 108 more than two-thirds of itself?

87.

209

GMAT® Official Guide 2018

Arithmetic Percents

evaporated by the original amount and multiply
by 100 to convert the decimal to a percent. Thus,
0.2
× 100 = 0.02 × 100 or 2%.
10
The correct answer is D.

It is given that 460 is 115% of the number of
employees in January. Therefore, the number of
employees in January was
460 460 ⎛ 100 ⎞ ⎛ 460 ⎞
=
⎜
⎟ =⎜
⎟ (100 ) = ( 4 )(100 ) = 400.
1.15 1.15 ⎝ 100 ⎠ ⎝ 115 ⎠

90.

If m and p are positive integers and m2 + p2 < 100,
what is the greatest possible value of mp ?

The correct answer is B.
88.

The function f is defined by f ( x ) = x − 10 for all
positive numbers x. If u = f(t) for some positive
numbers t and u, what is t in terms of u ?
u + 10

(A)
(B)

(

u + 10

)

2

(C)

u 2 + 10

(D)

(u + 10)2

(E)

(u2 + 10)2

Algebra Functions

The question can be answered by solving
u = t – 10 for t in terms of u. Adding 10 to both
sides of this equation gives u + 10 = t . Squaring
both sides of the last equation gives (u + 10)2 = t,
which gives t in terms of u.
The correct answer is D.
89.

A glass was filled with 10 ounces of water, and
0.01 ounce of the water evaporated each day during a
20‐day period. What percent of the original amount of
water evaporated during this period?

36

(B)

42

(C)

48

(D)

49

(E)

51

Arithmetic Operations with integers

Trying various integer values for m and
corresponding values of p that satisfy
m2 + p2 < 100 might be the quickest way to solve
this problem. First, m < 10 and p < 10; otherwise,
m2 + p2 < 100 is not true.
If m = 9, then for m2 + p2 < 100 to be true,
p < 100 − 81 = 19 , so p ≤ 4, and the greatest
possible value for mp is (9)(4) = 36.
Similarly, if m = 8, then p < 100 − 64 = 36, so
p ≤ 5, and the greatest possible value for mp is
(8)(5) = 40.
If m = 7, then p < 100 − 49 = 51, so p ≤ 7, and
the greatest possible value for mp is (7)(7) = 49.
If m = 6, then p < 100 − 36 = 64, so p ≤ 7, and
the greatest possible value for mp is (6)(7) = 42.

(A)

0.002%

(B)

0.02%

(C)

0.2%

Thus, the greatest possible value for mp is 49.

(D)

2%

The correct answer is D.

(E)

20%

Arithmetic Percents

Since 0.01 ounce of water evaporated each day for
20 days, a total of 20(0.01) = 0.2 ounce evaporated.
Then, to find the percent of the original amount of
water that evaporated, divide the amount that

210

(A)

If m ≤ 5 and p ≤ 9, it follows that mp ≤ 45.

5.5 Problem Solving Answer Explanations

91.

From this it follows that Equation III will hold
b2
only if 2 = 1, which can be false. For example,
a
if x = a = c = 1 and y = b = d = 2 (a choice of
x c
d b
values for which = and = are true), then
y d
c a
b2
2 1
≠ 1 and Equation III is = , which is false.
1 2
a2

x c
d b
= and = , which of the following must be
y d
c a
true?

If =

y b
=
x a
x y
=
a b
y x
=
a b

I.
II.
III.

The correct answer is C.

(A)

I only

(B)

II only

(C)

I and II only

If k is an integer and (0.0025)(0.025)(0.00025) × 10k
is an integer, what is the least possible value of k ?

(D)

I and III only

(A)

–12

(E)

I, II, and III

(B)

–6

(C)

0

(D)

6

(E)

12

92.

Algebra Ratio and proportion

Equation I is true:
x
y
y
x
d
c
y
x

=
=
=
=

c
d
d
c
b
a
b
a

given

Arithmetic Properties of numbers

Let N = (0.0025)(0.025)(0.00025) × 10k.
Rewriting each of the decimals as an
integer times a power of 10 gives
N = (25 × 10−4)(25 × 10−3)(25 × 10−5) × 10k =
(25)3 × 10k − 12. Since the units digit of (25)3 is
5, it follows that if k = 11, then the tenths digit
of N would be 5, and thus N would not be an
integer; and if k = 12, then N would be
(25)3 × 100 = (25)3, which is an integer. Therefore,
the least value of k such that N is an integer is 12.

take reciprocals
given
use last two equations

Equation II is true:
y b
= Equation I (shown true)
x a
y=

bx
multiply both sides by x
a

y x
= divide both sides by b
b a
Equation III is false, since otherwise it would
follow that:
bx
y=
from above
a

The correct answer is E.
93.

If a(a + 2) = 24 and b(b + 2) = 24, where a ≠ b, then
a+b=
(A)

–48

(B)

–2

(C)

2

(D)

46

(E)

48

y bx
=
divide both sides by a
a a2
x bx
=
use Equation III (assumed true)
b a2
x=

b 2x
multiply both sides by b
a2
211

GMAT® Official Guide 2018

Algebra Second-degree equations

a(a + 2) = 24

given

a2 + 2a = 24

use distributive property

a2 + 2a – 24 = 0

subtract 24 from both sides

(a + 6)(a – 4) = 0 factor

B

A
95.

So, a + 6 = 0, which means that a = –6, or
a – 4 = 0, which means a = 4. The equation with
the variable b has the same solutions, and so
b = –6 or b = 4.
Since a ≠ b, then a = –6 and b = 4, which means
a + b = −6 + 4 = −2, or a = 4 and b = −6, which
means that a + b = 4 + (−6) = −2
The correct answer is B.
94.

0.06N + 3,200

(B)

0.1N + 7,200

(C)

0.4N + 7,200

(D)

0.1N + 8,000

(E)

0.06N + 8,000

Algebra Percents

If N represents the total number of votes cast and
40% of the votes cast were cast by independent
voters, then 60% of the votes cast, or 0.6N votes,
were cast by voters registered with a political
party. Ms. Robbins received 10% of these, and so
Ms. Robbins received (0.10)(0.6N) = 0.06N votes
cast by voters registered with a political party.
Thus, Ms. Robbins received 0.06N votes cast
by voters registered with a political party and
8,000 votes cast by independent voters, so she
received 0.06N + 8,000 votes in all.
The correct answer is E.

212

In the figure shown, the triangle is inscribed in the
semicircle. If the length of line segment AB is 8 and the
length of line segment BC is 6, what is the length of
arc ABC ?
(A)

15π

(B)

12π

(C)

10π

(D)

7π

(E)

5π

Geometry Circles, triangles

Because ΔABC is inscribed in a semicircle,
∠ABC is a right angle. Applying the Pythagorean
theorem gives (AB)2 + (BC)2 = (AC)2. Then
substituting the given lengths, 82 + 62 = (AC)2,
and so (AC)2 = 100 and AC = 10. Thus, the
diameter of the circle is 10, the circumference of
the entire circle is 10π, and the length of arc ABC
is half the circumference of the circle, or 5π.

In a recent election, Ms. Robbins received 8,000 votes
cast by independent voters, that is, voters not
registered with a specific political party. She also
received 10 percent of the votes cast by those voters
registered with a political party. If N is the total number
of votes cast in the election and 40 percent of the
votes cast were cast by independent voters, which of
the following represents the number of votes that Ms.
Robbins received?
(A)

C

The correct answer is E.
96.

A manufacturer makes and sells 2 products, P and Q.
The revenue from the sale of each unit of P is $20.00
and the revenue from the sale of each unit of Q is
$17.00. Last year the manufacturer sold twice as
many units of Q as P. What was the manufacturer’s
average (arithmetic mean) revenue per unit sold of
these 2 products last year?
(A)

$28.50

(B)

$27.00

(C)

$19.00

(D)

$18.50

(E)

$18.00

Arithmetic Statistics

Let x represent the number of units of Product P
the manufacturer sold last year. Then 2x
represents the number of units of Product Q
the manufacturer sold last year, and x + 2x = 3x
represents the total number of units of Products P
and Q the manufacturer sold last year. The total

5.5 Problem Solving Answer Explanations

revenue from the sale of Products P and Q was
$(20x) + $(17(2x)) = $(54x), so the average
$(54 x )
revenue per unit sold was
= $18.
3x

98.

The correct answer is E.
97.

On a certain day, orangeade was made by mixing a
certain amount of orange juice with an equal amount
of water. On the next day, orangeade was made by
mixing the same amount of orange juice with twice
the amount of water. On both days, all the orangeade
that was made was sold. If the revenue from selling
the orangeade was the same for both days and if the
orangeade was sold at $0.60 per glass on the first
day, what was the price per glass on the second day?
(A)

$0.15

(B)

$0.20

(C)

$0.30

(D)

$0.40

(E)

$0.45

A worker carries jugs of liquid soap from a production
line to a packing area, carrying 4 jugs per trip. If the
jugs are packed into cartons that hold 7 jugs each,
how many jugs are needed to fill the last partially filled
carton after the worker has made 17 trips?
(A)

1

(B)

2

(C)

4

(D)

5

(E)

6

Arithmetic Remainders

Carrying 4 jugs per trip, the worker carries a
total of 4(17) = 68 jugs in 17 trips. At 7 jugs per
carton, these jugs will completely fill 9 cartons
with 5 jugs left over since (9)(7) + 5 = 68. To fill
the 10th carton, 7 – 5 = 2 jugs are needed.
The correct answer is B.
w

Arithmetic Applied problems

The ratio of the amount of orangeade made and
sold on the first day to amount of orangeade
made and sold on the second day is 2:3, because
the orangeade on the first day was 1 part orange
juice and 1 part water, while on the second day it
was 1 part orange juice and 2 parts water. Thus,
the ratio of the number of glasses of orangeade
made and sold on the first day to the number of
glasses of orangeade made and sold on the second
day is 2:3. Since the revenues for each day were
equal and 2 glasses were sold on the first day for
every 3 glasses that were sold on the second day,
2($0.60) = 3p, where p represents the price per
glass at which the orangeade was sold on the
⎛ 2⎞
second day. Therefore, p = ⎜ ⎟ ($0.60 ) = $0.40.
⎝ 3⎠
The correct answer is D.

w

99.

The figure shown above represents a modern painting
that consists of four differently colored rectangles,
each of which has length ℓ and width w. If the area of
the painting is 4,800 square inches, what is the width,
in inches, of each of the four rectangles?
(A)

15

(B)

20

(C)

25

(D)

30

(E)

40

Geometry Area

From the figure, ℓ = 3w, and the area of the
painting is ℓ(w + ℓ). Substituting 3w for ℓ gives
3w(w + 3w) = 3w(4w) = 12w2. It is given that
the area is 4,800 square inches, so 12w2 = 4,800,
w2 = 400, and w = 20.
The correct answer is B.
213

GMAT® Official Guide 2018

100. A certain fruit stand sold apples for $0.70 each and
bananas for $0.50 each. If a customer purchased
both apples and bananas from the stand for a total of
$6.30, what total number of apples and bananas did
the customer purchase?
(A)

10

(B)

11

(C)

12

(D)

13

(E)

14

Algebra First-degree equations; Operations
with integers

If each apple sold for $0.70, each banana sold for
$0.50, and the total purchase price was $6.30, then
0.70x + 0.50y = 6.30, where x and y are positive
integers representing the number of apples and
bananas, respectively, the customer purchased.
0.70x + 0.50y = 6.30
0.50y = 6.30 – 0.70x
0.50y = 0.70(9 – x)
y=

7
(9 − x )
5

Since y must be an integer, 9 – x must be
divisible by 5. Furthermore, both x and y must be
positive integers. For x = 1, 2, 3, 4, 5, 6, 7, 8, the
corresponding values of 9 – x are 8, 7, 6, 5, 4, 3, 2,
and 1. Only one of these, 5, is divisible by 5.
7
Therefore, x = 4 and y = ( 9 − 4 ) = 7 and the
5
total number of apples and bananas the customer
purchased is 4 + 7 = 11.
The correct answer is B.
101. In the xy‐plane, what is the slope of the line with
equation 3x + 7y = 9 ?

214

(A)

−

7
3

(B)

−

3
7

(C)

3
7

(D)

3

(E)

7

Algebra Coordinate geometry

Since the given equation of the line is equivalent
3
9
to 7y = –3x + 9, or y = − x + , the slope of the
7
7
3
line is − . Alternatively, choose 2 points lying on
7
the line and then use the slope formula for these
2 points. For example, substitute x = 0 in
9⎞
⎛
7y = –3x + 9 and solve for y to get ⎜ 0, ⎟ ,
⎝
7⎠
substitute y = 0 in 7y = –3x + 9 and solve for x
to get (3,0), then use the slope formula to get
9
9
−0
3
7
7
=
=− .
0 − 3 −3
7
The correct answer is B.
102. Working simultaneously and independently at an
identical constant rate, 4 machines of a certain type
can produce a total of x units of product P in 6 days.
How many of these machines, working simultaneously
and independently at this constant rate, can produce a
total of 3x units of product P in 4 days?
(A)

24

(B)

18

(C)

16

(D)

12

(E)

8

Algebra Applied problems

Define a machine day as 1 machine working for
1 day. Then, 4 machines each working 6 days is
equivalent to (4)(6) = 24 machine days. Thus,
x units of product P were produced in 24 machine
days, and 3x units of product P will require
(3)(24) = 72 machine days, which is equivalent
72
= 18 machines working independently and
to
4
simultaneously for 4 days.
The correct answer is B.

5.5 Problem Solving Answer Explanations

103. At a certain school, the ratio of the number of second
graders to the number of fourth graders is 8 to 5, and
the ratio of the number of first graders to the number
of second graders is 3 to 4. If the ratio of the number
of third graders to the number of fourth graders is 3
to 2, what is the ratio of the number of first graders to
the number of third graders?
(A)

16 to 15

(B)

9 to 5

(C)

5 to 16

(D)

5 to 4

(E)

4 to 5

Arithmetic Ratio and proportion

If F, S, T, and R represent the number of first,
second, third, and fourth graders, respectively,
S 8
F 3
then the given ratios are: (i) = , ( ii ) = ,
R 5
S 4
T 3
F
and ( iii ) = . The desired ratio is . From (i),
R 2
T
8
3
S = R , and from (ii), F = S . Combining these
5
4
3
3⎛8 ⎞ 6
results, F = S = ⎜ R ⎟ = R . From (iii),
4
4⎝5 ⎠ 5
6
F 5R 6 2 4
3
T = R . Then
=
= ⋅ = . So, the
T 3R 5 3 5
2
2
ratio of the number of first graders to the number
of third graders is 4 to 5.
The correct answer is E.

Arithmetic Operations with integers

If Δ represents addition, subtraction,
multiplication, or division, then 6 Δ 3 is equal
to either 6 + 3 = 9, or 6 – 3 = 3, or 6 × 3 = 18,
or 6 ÷ 3 = 2. Since it is given that 6 Δ 3 ≤ 3, Δ
represents either subtraction or division.
Statement I is true for subtraction since 2 – 2 = 0
but not true for division since 2 ÷ 2 = 1.
Statement II is not true for subtraction since
2 – 2 = 0 but is true for division since 2 ÷ 2 = 1.
Statement III is true for subtraction since
4 – 2 = 2 and is true for division since 4 ÷ 2 = 2.
Therefore, only Statement III must be true.
The correct answer is C.
105. The average distance between the Sun and a certain
planet is approximately 2.3 × 1014 inches. Which of the
following is closest to the average distance between
the Sun and the planet, in kilometers? (1 kilometer is
approximately 3.9 × 104 inches.)
(A)

7.1 × 108

(B)

5.9 × 109

(C)

1.6 × 1010

(D)

1.6 × 1011

(E)

5.9 × 1011

Arithmetic Measurement conversion

Convert to kilometers and then estimate.
⎛

⎞

= 2.3 × 10
( 2.3 × 10 in ) ⎜⎝ 3.91×km
⎟
10 in ⎠ 3.9 × 10
14

14

4

104. The symbol Δ denotes one of the four arithmetic
operations: addition, subtraction, multiplication, or
division. If 6 Δ 3 ≤ 3, which of the following must be
true?
I.

2Δ2=0

II.

2Δ2=1

III.

4Δ2=2

(A)

I only

(B)

II only

(C)

III only

(D)

I and II only

(E)

I, II, and III

4

km

= 2.3 × 1014 − 4 km
3.9
≈ 2 × 1010
4
= 0.5 × 1010
= 5 × 10 9
The correct answer is B.

215

GMAT® Official Guide 2018

1
106. If mn ≠ 0 and 25 percent of n equals 37 percent of
2
12n
?
m, what is the value of
m
(A)

18

(B)

32
3

(C)

8

(D)

3

(E)

9
8

The correct answer is E.
108. Last year Joe grew 1 inch and Sally grew 200 percent
more than Joe grew. How many inches did Sally grow
last year?

Algebra Percents; First-degree equations

It is given that (25%)n = (37.5%)m, or
12n
0.25n = 0.375m. The value of
can be
m n
found by first finding the value of and then
m
multiplying the result by 12. Doing this gives
12n ⎛ 0.375 ⎞
=⎜
(12) = 18. Alternatively, the
⎝ 0.25 ⎟⎠
m
numbers involved allow for a series of simple
equation transformations to be carried out, such
as the following:
0.25n
25n
50n
2n
12n
12n
m

=
=
=
=
=

0.375m
37.5m
75m
3m
18m

= 18

given
multiply both sides by 100
multiply both sides by 2
divide both sides by 25
multiply both sides by 6
divide both sides by m

The correct answer is A.
107. In the coordinate plane, a circle has center (2,−3) and
passes through the point (5,0). What is the area of the
circle?
(A)

3π

(B)

3 2π

(C)

3 3π

(D)

9π

(E)

18π

Geometry Coordinate geometry; Circles; Area

The area of a circle is given by πr2, where r is the
radius of the circle. The value of r2 is the square
of the distance from the center to a point
216

of the circle. Using the distance formula,
r2 = (2 − 5)2 + (−3 − 0)2 = 9 + 9 = 18. Therefore,
the area of the circle is 18π.

(A)

0

(B)

1

(C)

2

(D)

3

(E)

4

Arithmetic Percents

Joe grew 1 inch last year and Sally grew 200 percent
more than Joe grew, so Sally grew 1 inch plus
200 percent of 1 inch or 1 + 2(1) = 3 inches.
The correct answer is D.
109. The cost C, in dollars, to remove p percent of a certain
pollutant from a pond is estimated by using the formula
100,000p
C=
. According to this estimate, how much
100 − p
more would it cost to remove 90 percent of the
pollutant from the pond than it would cost to remove
80 percent of the pollutant?
(A)

$500,000

(B)

$100,000

(C)

$50,000

(D)

$10,000

(E)

$5,000

Algebra; Arithmetic Simplifying algebraic
expressions; Operations on rational numbers

Removing 90% of the pollutant from the pond
(100, 000 )( 90 ) = 9, 000, 000 =
would cost
100 − 90
10
900,000 dollars, and removing 80% of the
(100, 000 )(80 ) =
pollutant would cost
100 − 80
8, 000, 000
= 400, 000 dollars. The difference is,
20
then, $900,000 − $400,000 = $500,000.
The correct answer is A.

5.5 Problem Solving Answer Explanations

110. If xy ≠ 0 and x2y2 – xy = 6, which of the following could
be y in terms of x ?
I.
II.

Algebra Simplifying algebraic expressions

1
( 5, 280 ), s = 40 into the
2
given formula and calculate the value for N.
Substitute L = 2, d =

1
2x
−

2
x

III.

3
x

(A)

I only

(B)

II only

(C)

I and II

(D)

I and III

(E)

II and III

⎛ 1⎞
20 ( 2 ) ⎜ ⎟ ( 5, 280 )
⎝ 2⎠
N=
600 + 40 2
20 ( 5, 280 )
=
600 + 1, 600
=

2 ( 528 )
22
528
=
11
= 48
=

Algebra Second-degree equations

x2y2 – xy = 6 given
x2y2 – xy – 6 = 0 subtract 6 from both sides
(xy + 2)(xy – 3) = 0 factor
2
So, xy + 2 = 0, which means xy = –2 and y = − ,
x
or xy – 3 = 0, which means that xy = 3 and
3
y = . Thus, y in terms of x could be given by the
x
expressions in II or III.
The correct answer is E.
111. At a certain instant in time, the number of cars, N,
traveling on a portion of a certain highway can be
estimated by the formula
N=

20Ld

155

(B)

96

(C)

80

(D)

48

(E)

24

The correct answer is D.
112.

4.8 × 109 is closest in value to
(A)

2,200

(B)

70,000

(C)

220,000

(D)

7,000,000

(E)

22,000,000

Arithmetic Operations on radical expressions

4.8 × 10 9 = 48 × 108

600 + s 2

where L is the number of lanes in the same direction, d
is the length of the portion of the highway, in feet, and
s is the average speed of the cars, in miles per hour.
Based on the formula, what is the estimated number
1
of cars traveling on a ‐ mile portion of the highway
2
if the highway has 2 lanes in the same direction and
the average speed of the cars is 40 miles per hour?
(5,280 feet = 1 mile)
(A)

20 ( 5, 280 )
2, 200

substitute 48 × 108
for 4.8 × 10 9

= 48 × 108
≈ 49 × 108

ab = a × b
49 ≈ 48

and then
49 × 108 = 7 × 10 4

49 = 7,

(10 )

4 2

108 =

= 10 4

= 70, 000
The correct answer is B.

217

GMAT® Official Guide 2018

113. Three printing presses, R, S, and T, working together
at their respective constant rates, can do a certain
printing job in 4 hours. S and T, working together at
their respective constant rates, can do the same job
in 5 hours. How many hours would it take R, working
alone at its constant rate, to do the same job?
(A)

8

(B)

10

(C)

12

(D)

15

(E)

20

Algebra Applied problems

Let r be the portion of the job that printing
press R, working alone, completes in 1 hour;
and let s and t be the corresponding portions,
respectively, for printing press S and printing
press T. From the given information, it follows
1
1
that r + s + t = and s + t = . Subtracting
4
5
1 1
1
these two equations gives r = − = . It
4 5 20
follows that printing press R, working alone, will
1
complete
of the job in 1 hour, and therefore
20
printing press R, working alone, will complete the
job in 20 hours.
The correct answer is E.
114. For a party, three solid cheese balls with diameters of
2 inches, 4 inches, and 6 inches, respectively, were
combined to form a single cheese ball. What was the
approximate diameter, in inches, of the new cheese
4
ball? (The volume of a sphere is π r 3 , where r is the
3
radius.)
(A)

12

(B)

16

(C)

3

16

(D)

3 8

(E)

23 36

Since the diameters of the cheese balls are given
as 2 inches, 4 inches, and 6 inches, the radii of
the cheese balls are 1 inch, 2 inches, and 3 inches,
4
respectively. Using V = π r 3, the combined
3

)

Thus, if R represents the radius of the new cheese
ball, then the volume of the new cheese ball
4
4
is π R 3 = π ( 36 ) and R3 = 36, from which
3
3
it follows that R = 3 36 inches. Therefore,
the diameter of the new cheese ball is
2R = 2 3 36 inches.
The correct answer is E.
115. The sum of all the integers k such that −26 < k < 24 is
(A)

0

(B)

−2

(C)

−25

(D)

−49

(E)

−51

Arithmetic Operations on integers

In the sum of all integers k such that
−26 < k < 24, the positive integers from 1 through
23 can be paired with the negative integers from
−1 through −23. The sum of these pairs is 0
because a + (−a) = 0 for all integers a.
Therefore, the sum of all integers k such that
−26 < k < 24 is −25 + (−24) + (23)(0) = −49.
The correct answer is D.
R

S

T

0
116. The number line shown contains three points R, S, and
T, whose coordinates have absolute values r, s, and t,
respectively. Which of the following equals the average
(arithmetic mean) of the coordinates of the points R, S,
and T ?

3

Geometry Volume

218

(

4
volume of the 3 cheese balls is π 13 + 23 + 33
3
4
or π (36) cubic inches.
3

(A)

s

(B)

s+t−r

(C)

r −s−t
3

(D)

r +s+t
3

(E)

s+t −r
3

5.5 Problem Solving Answer Explanations

Arithmetic Absolute value; Number line

Algebra Sets

Because point R is to the left of 0 on the number
line, the coordinate of R is negative. It is given
that r is the absolute value of the coordinate of R
and so the coordinate of R is −r. Because points
S and T are to the right of 0 on the number line,
their coordinates are positive. It is given that s
and t are the absolute values of the coordinates of
S and T, and so the coordinates of S and T are s
and t. The arithmetic mean of the coordinates of
s+t −r
.
R, S, and T is
3

Since x students are taking music, y students are
taking art, and z students are taking both music and
art, the number of students taking only music is
x − z, and the number of students taking only art is
y − z, as illustrated by the following Venn diagram.

music

x–z

The correct answer is E.
117. Mark and Ann together were allocated n boxes of
cookies to sell for a club project. Mark sold 10 boxes
less than n and Ann sold 2 boxes less than n. If Mark
and Ann have each sold at least one box of cookies,
but together they have sold less than n boxes, what is
the value of n ?
(A)

11

(B)

12

(C)

13

(D)

14

(E)

15

Algebra Inequalities

Mark sold n − 10 boxes and Ann sold n − 2 boxes.
Because each person sold at least one box, it
follows that n − 10 ≥ 1 and n − 2 ≥ 1, which
implies that n ≥ 11. On the other hand, together
they sold less than n boxes, so (n − 10) + (n − 2) < n,
which implies that n < 12. Therefore, n is an
integer such that n ≥ 11 and n < 12, which
implies that n = 11.
The correct answer is A.
118. A certain high school has 5,000 students. Of these
students, x are taking music, y are taking art, and z
are taking both music and art. How many students are
taking neither music nor art?
(A)

5,000 − z

(B)

5,000 − x − y

(C)

5,000 − x + z

(D)

5,000 − x − y − z

(E)

5,000 − x − y + z

art

z

y–z

Therefore, the number of students taking
neither music nor art is
5,000 − [(x − z) + z + (y − z)] = 5,000 − x − y + z.
The correct answer is E.
119. Yesterday’s closing prices of 2,420 different stocks
listed on a certain stock exchange were all different
from today’s closing prices. The number of stocks
that closed at a higher price today than yesterday was
20 percent greater than the number that closed at a
lower price. How many of the stocks closed at a higher
price today than yesterday?
(A)

484

(B)

726

(C)

1,100

(D)

1,320

(E)

1,694

Arithmetic Percents

Let n be the number of stocks that closed at a
lower price today than yesterday. Then 1.2n is the
number of stocks that closed at a higher price
today than yesterday, and 1.2n is the value asked
for. Because the total number of stocks is 2,420, it
follows that n + 1.2n = 2,420, or 2.2n = 2,420.
2, 420
= 1,100, and hence
2.2
1.2n = (1.2)(1,100) = 1,320.

Therefore, n =

The correct answer is D.
219

GMAT® Official Guide 2018

120. Each person who attended a company meeting was
either a stockholder in the company, an employee
of the company, or both. If 62 percent of those
who attended the meeting were stockholders and
47 percent were employees, what percent were
stockholders who were not employees?
(A)

34%

(B)

38%

(C)

45%

(D)

53%

(E)

62%

Amount Budgeted

Amount Spent

$110,000

$117,000

40,000

42,000

2,500

2,340

Payroll
Taxes
Insurance

122. The table shows the amount budgeted and the
amount spent for each of three accounts in a certain
company. For which of these accounts did the amount
spent differ from the amount budgeted by more than
6 percent of the amount budgeted?

Arithmetic Sets

(A)

Payroll only

Let M represent the number of meeting
attendees. Then, since 62% of M or 0.62M were
stockholders and 47% of M or 0.47M were
employees, it follows that 0.62M + 0.47M = 1.09M
were either stockholders, employees, or both. Since
1.09M exceeds M, the excess 1.09M − M = 0.09M
must be the number of attendees who were both
stockholders and employees, leaving the rest
0.62M − 0.09M = 0.53M, or 53%, of the meeting
attendees to be stockholders but not employees.

(B)

Taxes only

(C)

Insurance only

(D)

Payroll and Insurance

(E)

Taxes and Insurance

The correct answer is D.
121. A gym class can be divided into 8 teams with an equal
number of players on each team or into 12 teams with
an equal number of players on each team. What is the
lowest possible number of students in the class?
(A)

20

(B)

24

(C)

36

(D)

48

(E)

96

Arithmetic Properties of numbers

The lowest value that can be divided evenly
by 8 and 12 is their least common multiple
(LCM). Since 8 = 23 and 12 = 22(3), the LCM is
23(3) = 24.
The correct answer is B.

220

Accounts

Arithmetic Percents

For Payroll, 6% of the budgeted amount
is (0.06)($110,000) = $6,600. Since
$117,000 – $110,000 = $7,000 > $6,600, the
amount spent differed from the amount budgeted
by more than 6%.
For Taxes, 6% of the budgeted amount
is (0.06)($40,000) = $2,400. Since
$42,000 – $40,000 = $2,000 < $2,400, the
amount spent did not differ from the amount
budgeted by more than 6%.
For Insurance, 6% of the budgeted
amount is (0.06)($2,500) = $150. Since
$2,500 – $2,340 = $160 > $150, the amount
spent differed from the amount budgeted by more
than 6%.
Thus, the amount spent differed from the amount
budgeted by more than 6% for Payroll and
Insurance.
The correct answer is D.

5.5 Problem Solving Answer Explanations

Then rotating this figure clockwise about point P
through an angle of 120° will produce the figure
shown below.

B

P

D

A

C

123. In the figure above, triangle ABC is equilateral, and point
P is equidistant from vertices A, B, and C. If triangle ABC
is rotated clockwise about point P, what is the minimum
number of degrees the triangle must be rotated so that
point B will be in the position where point A is now?
(A)

60

(B)

120

(C)

180

(D)

240

(E)

270

C

P
B

A

In this figure, point B is in the position where
point A was in the original figure. The triangle
was rotated clockwise about point P through
120° + 120° = 240°.

Geometry Angles

B

The correct answer is D.

P
A

C

D

Since ΔABC is equilateral, the measure of
∠ACB is 60°. Therefore, the measure of
∠BCD is 180° − 60° = 120°. Rotating the figure
clockwise about point P through an angle of 120°
will produce the figure shown below.

(A)

19

(B)

17

(C)

16

(D)

14

(E)

13

Arithmetic Operations on rational numbers

A

2
of the members must vote in favor of
3
1
a resolution, then no more than of the members
3
can be voting against it. On this 40‐member
1
1
committee, (40) = 13 , which means that no
3
3
more than 13 members can vote against the
resolution and still have it pass.
If at least

P
C

2
of the 40 members of a committee must
3
vote in favor of a resolution for it to pass. What is the
greatest number of members who could vote against
the resolution and still have it pass?

124. At least

B

The correct answer is E.
D
221

GMAT® Official Guide 2018

126. If n = 20! + 17, then n is divisible by which of the following?

number of computers assembled

20
18

I.

15

16

II.

17

14

III.

19

(A)

None

(B)

I only

(C)

II only

(D)

I and II

(E)

II and III

12
10
8
6
4
2
0

1

2

3

4

5

Arithmetic Properties of numbers

6

day
125. The graph shows the number of computers assembled
during each of 6 consecutive days. From what day to
the next day was the percent change in the number of
computers assembled the greatest in magnitude?
(A)

From Day 1 to Day 2

(B)

From Day 2 to Day 3

(C)

From Day 3 to Day 4

(D)

From Day 4 to Day 5

(E)

From Day 5 to Day 6

Arithmetic Percents

The following table shows the percent change
from each day to the next and the magnitude of
the percent change.
Time period

Because 20! is the product of all integers from 1
through 20, it follows that 20! is divisible by each
integer from 1 through 20. In particular, 20! is
divisible by each of the integers 15, 17, and 19.
Since 20! and 17 are both divisible by 17, their
sum is divisible by 17, and hence the correct
answer will include II. If n were divisible by 15,
then n − 20! would be divisible by 15. But,
n − 20! = 17 and 17 is not divisible by 15.
Therefore, the correct answer does not include I.
If n were divisible by 19, then n − 20! would be
divisible by 19. But, n − 20! = 17 and 17 is not
divisible by 19. Therefore, the correct answer does
not include III.
The correct answer is C.

Percent change

Magnitude of percent change

From Day 1 to Day 2

⎛ 12 − 20
⎞
⎛ 8
⎞
× 100 ⎟ % = ⎜ −
× 100 ⎟ % = −40%
⎜⎝
⎠
⎝ 20
⎠
20

40

From Day 2 to Day 3

⎛ 18 − 12
⎞
⎛ 6
⎞
× 100 ⎟ % = ⎜ × 100 ⎟ % = 50%
⎜⎝
⎠
⎝
⎠
12
12

50

From Day 3 to Day 4

⎛ 10 − 18
⎞
⎛ 8
⎞
× 100 ⎟ % = ⎜ − × 100 ⎟ % ≈ −44%
⎜⎝
⎠
⎝ 18
⎠
18

44

From Day 4 to Day 5

⎛ 16 − 10
⎞
⎛ 6
⎞
× 100 ⎟ % = ⎜ × 100 ⎟ % = 60%
⎜⎝
⎠
⎝
⎠
10
10

60

From Day 5 to Day 6

⎛ 8 − 16
⎞
⎛ 8
⎞
× 100 ⎟ % = ⎜ − × 100 ⎟ % = −50%
⎜⎝
⎠
⎝ 16
⎠
16

50

The correct answer is D.
222

5.5 Problem Solving Answer Explanations

127. The product of two negative numbers is 160. If the
lesser of the two numbers is 4 less than twice the
greater, what is the greater number?
(A)

–20

(B)

–16

(C)

–10

(D)

–8

(E)

–4

Algebra Second-degree equations

Let x and y be the two numbers, where x is the
lesser of the two numbers and y is the number
desired. From the given information it follows
that xy = 160 and x = 2y – 4, from which it
follows that (2y – 4)y = 160. Dividing both sides
of the last equation by 2 gives (y – 2)y = 80. Thus,
80 is to be written as a product of two negative
numbers, one that is 2 less than the other. Trying
simple factorizations of 80 quickly leads to the
value of y: (–40)(–2) = 80, (–20)(–4) = 80,
(–10)(–8) = 80. Therefore, y = –8. Note that
because –8 is one of the answer choices, it is not
necessary to ensure there are no other negative
solutions to the equation (y – 2)y = 80.
Alternatively, (y – 2)y = 80 can be written as
y2 – 2y – 80 = 0. Factoring the left side gives
(y + 8)(y – 10) = 0, and y = –8 is the only negative
solution.
The correct answer is D.
128. According to a certain estimate, the depth N(t), in
centimeters, of the water in a certain tank at t hours past
2:00 in the morning is given by N(t) = −20(t − 5)2 + 500
for 0 ≤ t ≤ 10. According to this estimate, at what time
in the morning does the depth of the water in the tank
reach its maximum?
(A)

5:30

(B)

7:00

(C)

7:30

(D)

8:00

(E)

9:00

and −20(t − 5)2 + 500 < 500. Therefore, the tank
reaches its maximum depth 5 hours after 2:00 in
the morning, which is 7:00 in the morning.
The correct answer is B.
129. After driving to a riverfront parking lot, Bob plans to run
south along the river, turn around, and return to the
parking lot, running north along the same path. After
running 3.25 miles south, he decides to run for only
50 minutes more. If Bob runs at a constant rate of
8 minutes per mile, how many miles farther south can
he run and still be able to return to the parking lot in
50 minutes?
(A)

1.5

(B)

2.25

(C)

3.0

(D)

3.25

(E)

4.75

Algebra Applied problems

After running 3.25 miles south, Bob has been
⎛ minutes ⎞
running for ( 3.25 miles ) ⎜ 8
⎟ = 26 minutes.
⎝
mile ⎠
Thus, if t is the number of additional minutes
that Bob can run south before turning around,
then the number of minutes that Bob will run
north, after turning around, will be t + 26. Since
Bob will be running a total of 50 minutes after
the initial 26 minutes of running, it follows that
t + (t + 26) = 50, or t = 12. Therefore, Bob can
12 minutes
= 1.5 miles
run south an additional
minutes
8
mile
before turning around.
The correct answer is A.

Algebra Functions

When t = 5, the value of −20(t − 5)2 + 500 is 500.
For all values of t between 0 and 10, inclusive,
except t = 5, the value of −20(t − 5)2 is negative
223

GMAT® Official Guide 2018

130. Alex deposited x dollars into a new account that earned
8 percent annual interest, compounded annually. One
year later Alex deposited an additional x dollars into the
account. If there were no other transactions and if the
account contained w dollars at the end of two years,
which of the following expresses x in terms of w ?
(A)

w
1 + 1.08

(B)

w
1.08 + 1.16

(C)

w
1.16 + 1.24
w

(D)

1.08 + (1.08 )

(E)

(1.08)

2

w
2

+ (1.08 )

3

Algebra Applied problems

At the end of the first year, the value of Alex’s
initial investment was x(1.08) dollars, and after he
deposited an additional x dollars into the account,
its value was [x(1.08) + x] dollars. At the end of
the second year, the value was w dollars, where
w = [x(1.08) + x](1.08) = x(1.08)2 + x(1.08) =
w
x[(1.08)2 + 1.08]. Thus, x =
.
2
1.08 + (1.08 )
The correct answer is D.

1
1
1
+
++
(the sum of 99 identical
300 300
300
1
1
1
+
++
.
values) is less than
201 202
299
1
to both sides of this last
Therefore, adding
300
1
1
1
+
++
inequality, it follows that
300 300
300
(the sum of 100 identical values) is less than
1
1
1
1
+
+  +
+
= M . Hence,
201 202
299 300
1
1
(100 ) ⎛⎜⎝ ⎞⎟⎠ < M or < M . Also, because
300
3
1
is greater than each of the 100 numbers
200
1
1
1
,
, ...,
, it follows that
201 202
300
1
1
1
+
+  +
(the sum of 100 identical
200 200
200
1
1
1
+
++
.
values) is greater than
201 202
300
1
⎛ 1 ⎞
Hence, (100 ) ⎜
> M or > M .
⎟
⎝ 200 ⎠
2
1
1
From < M and > M , it follows that
3
2
1
1
 512 ?

The correct answer is C.
163. Each year for 4 years, a farmer increased the number
1
of trees in a certain orchard by of the number of
4
trees in the orchard the preceding year. If all of the
trees thrived and there were 6,250 trees in the orchard
at the end of the 4‐year period, how many trees were
in the orchard at the beginning of the 4‐year period?
(A)

1,250

(B)

1,563

(C)

2,250

(A)

6

(D)

2,560

(B)

7

(E)

2,752

(C)

8

(D)

9

(E)

10

Arithmetic Operations with rational numbers

Because 5 2 = 25, a common base is 5. Rewrite
the left side with 5 as a base: 25n = (5 2 )n = 5 2n . It
follows that the desired integer is the least integer
n for which 5 2n > 512. This will be the least
integer n for which 2n > 12, or the least integer n
for which n > 6, which is 7.
The correct answer is B.
162. Sixty percent of the members of a study group are
women, and 45 percent of those women are lawyers.
If one member of the study group is to be selected
at random, what is the probability that the member
selected is a woman lawyer?
(A)

0.10

(B)

0.15

(C)

0.27

(D)

0.33

(E)

0.45

Arithmetic Operations on rational numbers

1
of
4
the number of trees in the orchard the preceding
year is equivalent to making the number of trees
increase 25% per year, compounded yearly. If
there were n trees at the beginning of the 4‐year
period, then there will be 1.25n trees at the end of
the first year, 1.25(1.25n) = (1.25)2n trees at the
end of the second year, 1.25[(1.25)2n] = (1.25)3n
trees at the end of the third year, and
1.25[(1.25)3n] = (1.25)4n trees at the end of the
fourth year. Hence, 6,250 = (1.25)4n and
6, 250
n=
. The arithmetic can be greatly
4
(1.25)4
54
⎛ 5⎞
simplified by rewriting (1.25)4 as ⎜ ⎟ = 4
⎝ 4⎠
4
and 6,250 as (625)(10) = (54)(10). Then
⎛ 44 ⎞
6, 250
4
4
=
(5
)(10)
⎜ 54 ⎟ = (10)(4 ) = 2, 560 .
(1.25)4
⎝ ⎠
Increasing the number of trees each year by

The correct answer is D.

235

GMAT® Official Guide 2018

number of shipments

Number of Shipments of Manufactured Homes
in the United States, 1990–2000
400,000
300,000
200,000

Since there are 11 entries in the table and 11 is
an odd number, the median of the numbers of
shipments is the 6th entry when the numbers
of shipments are arranged in order from least to
greatest. In order, from least to greatest, the first 6
entries are:
Number of shipments

100,000
0

180,000
1990

1992

1994

1996
year

1998

2000

190,000
210,000

164. According to the chart shown, which of the following is
closest to the median annual number of shipments of
manufactured homes in the United States for the years
from 1990 to 2000, inclusive?
(A)

250,000

(B)

280,000

(C)

310,000

(D)

325,000

(E)

340,000

310,000

The correct answer is C.
165. For the positive integers a, b, and k, a k b means that
a k is a divisor of b, but a k + 1 is not a divisor of b. If k is
a positive integer and 2k 72, then k is equal to

From the chart, the approximate numbers of
shipments are as follows:

236

270,000

The 6th entry is 310,000.

Arithmetic Interpretation of graphs and
tables; Statistics

Year
1990
1991
1992
1993
1994
1995
1996
1997
1998
1999
2000

270,000

Number of shipments
190,000
180,000
210,000
270,000
310,000
350,000
380,000
370,000
390,000
360,000
270,000

(A)

2

(B)

3

(C)

4

(D)

8

(E)

18

Arithmetic Property of numbers

Since 72 = (23)(32), it follows that 23 is a divisor
of 72 and 24 is not a divisor of 72. Therefore,
23 || 72, and hence k = 3.
The correct answer is B.
166. A certain characteristic in a large population has a
distribution that is symmetric about the mean m.
If 68 percent of the distribution lies within one
standard deviation d of the mean, what percent of the
distribution is less than m + d ?
(A)

16%

(B)

32%

(C)

48%

(D)

84%

(E)

92%

5.5 Problem Solving Answer Explanations

Arithmetic Statistics

Since 68% lies between m − d and m + d, a total
of (100 − 68)% = 32% lies to the left of m − d and
to the right of m + d. Because the distribution is
symmetric about m, half of the 32% lies to the
right of m + d. Therefore, 16% lies to the right of
m + d, and hence (100 − 16)% = 84% lies to the
left of m + d.
The correct answer is D.
167. Four extra‐large sandwiches of exactly the same size
were ordered for m students, where m > 4. Three
of the sandwiches were evenly divided among the
students. Since 4 students did not want any of the
fourth sandwich, it was evenly divided among the
remaining students. If Carol ate one piece from each
of the four sandwiches, the amount of sandwich that
she ate would be what fraction of a whole extra‐large
sandwich?
(A)

m+4
m (m − 4 )

(B)

2m − 4
m (m − 4 )

(C)

4m − 4
m (m − 4 )

(D)

4m − 8
m (m − 4 )

(E)

4m − 12
m (m − 4 )

Algebra Applied problems

Since each of 3 of the sandwiches was evenly
divided among m students, each piece was 1 of a
m
sandwich. Since the fourth sandwich was evenly
divided among m − 4 students, each piece was
1 of the fourth sandwich. Carol ate 1 piece
m−4
from each of the four sandwiches, so she ate a
total of

( 3) 1 +
m

168. Which of the following equations has 1 + 2 as one of
its roots?
(A)

x2 + 2x – 1 = 0

(B)

x2 – 2x + 1 = 0

(C)

x2 + 2x + 1 = 0

(D)

x2 – 2x – 1 = 0

(E)

x2 – x – 1 = 0

Algebra Second-degree equations

This problem can be solved by working backwards
to construct a quadratic equation with 1 + 2 as
a root that does not involve radicals.
x
x–1
(x – 1)2
x2 – 2x + 1
x2 – 2x – 1

=
=
=
=
=

1+ 2
2
2
2
0

set x to the desired value
subtract 1 from both sides
square both sides
expand the left side
subtract 2 from both sides

The correct answer is D.
169. In Country C, the unemployment rate among
construction workers dropped from 16 percent on
September 1, 1992, to 9 percent on September 1,
1996. If the number of construction workers was
20 percent greater on September 1, 1996, than
on September 1, 1992, what was the approximate
percent change in the number of unemployed
construction workers over this period?
(A)

50% decrease

(B)

30% decrease

(C)

15% decrease

(D)

30% increase

(E)

55% increase

1 = 3 (m − 4 ) + m = 4m − 12
m−4
m (m − 4 )
m (m − 4 )

The correct answer is E.

237

GMAT® Official Guide 2018

Arithmetic Percents

Arithmetic Probability

Let U1 and U2 be the numbers of unemployed
construction workers on September 1, 1992,
and September 1, 1996, respectively, and let
N be the number of construction workers
on September 1, 1992. Then, from the given
information, 1.2N is the number of construction
workers on September 1, 1996, U1 = 0.16N, and
U2 = 0.09(1.2N). Therefore, the percent change
from September 1, 1992, to September 1, 1996, of
unemployed construction workers is given by

Let A represent the event that the first pen
purchased is not defective and let B represent
the event that the second pen purchased is not
defective, where A and B are dependent events.
Since there are 3 defective pens in the box of
12 pens, there are 9 pens in the box that are not
defective. Using standard probability notation,
P ( A ) = 9 and P(B, given that A has occurred) =
12
8 . (Event A has occurred so there are 11 pens
11
left in the box and 8 of them are not defective.)
Then, using the multiplication rule for
dependent events, P(neither pen is defective)
= P(A and B) = P(A) × P(B, given that A has
occurred) = 9 × 8 = 6 .
12 11 11

⎛ U 2 −U1
⎞
× 100 ⎟ %
⎜⎝ U
⎠
1

⎛ 0.09 (1.2 N ) − 0.16 N
⎞
=⎜
× 100 ⎟ %
⎝
⎠
0.16 N
= ⎛ 0.108 − 0.16 × 100 ⎞ %
⎝
⎠
0.16
= ⎛ 108 − 160 × 100 ⎞ %
⎝ 160
⎠
= ⎛ − 13 × 100 ⎞ %
⎝ 40
⎠
≈ ⎛ − 13 × 100 ⎞ %
⎝ 39
⎠
≈ ⎛ − 1 × 100 ⎞ %
⎝ 3
⎠
≈ −30%

The correct answer is B.
170. In a box of 12 pens, a total of 3 are defective. If a
customer buys 2 pens selected at random from the
box, what is the probability that neither pen will be
defective?

238

(A)

1
6

(B)

2
9

(C)

6
11

(D)
(E)

Alternately, the probability of selecting 2 pens,
neither of which is defective, from a box
containing 12 pens, 3 of which are defective, and
therefore, 9 of which are non‐defective is the
number of ways to select 2 non‐defective pens
from 9 non‐defective pens over the number of
ways to select 2 pens from 12 pens
⎛ 9⎞
9×8
⎜⎝ 2⎟⎠
2
=
=
= 6.
12
11
×
12
11
⎛ ⎞
⎜⎝ 2 ⎟⎠
2
The correct answer is C.
171. At a certain fruit stand, the price of each apple is
40 cents and the price of each orange is 60 cents.
Mary selects a total of 10 apples and oranges from the
fruit stand, and the average (arithmetic mean) price of
the 10 pieces of fruit is 56 cents. How many oranges
must Mary put back so that the average price of the
pieces of fruit that she keeps is 52 cents?
(A)

1

9
16

(B)

2

(C)

3

3
4

(D)

4

(E)

5

5.5 Problem Solving Answer Explanations

Algebra Statistics

If Mary selected x apples, then she selected
(10 – x) oranges. The average price of the 10 pieces
40 x + 60 (10 − x )
of fruit is
= 56. From this,
10
40 x + 60 (10 − x )
= 56 given
10
40 x + 60 (10 − x ) = 560 multiply both sides
by 10
40 x + 600 − 60 x = 560 distribution property
600 − 20 x = 560 combine like terms
−20 x = –40 subtract 600 from
both sides
x=2

divide both sides by −20

Thus, Mary selected 2 apples and 8 oranges. Next,
let y be the number of oranges Mary needs to put
back, so that the average price of the [2 + (8 – y)]
pieces of fruit Mary keeps is 52 cents. Then,

( 40 )( 2) + ( 60 ) (8 − y )
2 + (8 − y )

= 52

80 + 480 − 60 y
= 52
10 − y
80 + 480 − 60 y = 52 (10 − y )
560 − 60 y = 520 − 52y
8y = 40

y=5

given
distributive
property
multiply both
sides by 10 − y
distributive
property
subtract
520 − 60 y
from both sides
divide both
sides by 8

Therefore, Mary must put back 5 oranges, so that
the average price of the fruit she keeps (that is,
the average price of 2 apples and 3 oranges) is
52 cents.

172. A pharmaceutical company received $3 million in
royalties on the first $20 million in sales of the generic
equivalent of one of its products and then $9 million
in royalties on the next $108 million in sales. By
approximately what percent did the ratio of royalties to
sales decrease from the first $20 million in sales to the
next $108 million in sales?
(A)

8%

(B)

15%

(C)

45%

(D)

52%

(E)

56%

Arithmetic Percents

The ratio of royalties to sales for the first
$20 million in sales is 3 , and the ratio of
20
royalties to sales for the next $108 million in sales
is 9 = 1 . The percent decrease in the royalties
108 12
to sales ratios is 100 times the quotient of the
difference in the ratios divided by the ratio of
royalties to sales for the first $20 million in sales
or
1 − 3
12 20 × 100 = ⎛ 1 − 3 ⎞ × 20 × 100
⎝ 12 20 ⎠
3
3
20
⎛ 1 20 − 1⎞ × 100
=⎝ ×
⎠
12 3
⎛5
⎞
= ⎝ − 1⎠ × 100
9
4
= − × 100
9
≈ –0.44 × 100
≈ 45% decrease
The correct answer is C.

The correct answer is E.

239

GMAT® Official Guide 2018

Times at Which the Door
Opened from 8:00 to 10:00
8:00

8:06

8:30

9:05

9:29 – 9:11 = 18 minutes, so the light is off
for 18 – 15 = 3 minutes

8:03

8:10

8:31

9:11

9:31 – 9:29 = 2 minutes

8:04

8:18

8:54

9:29

8:04

8:19

8:57

9:31

10:00 – 9:31 = 29 minutes, so the light is off for
29 – 15 = 14 minutes

173. The light in a restroom operates with a 15-minute timer
that is reset every time the door opens as a person
goes in or out of the room. Thus, after someone
enters or exits the room, the light remains on for only
15 minutes unless the door opens again and resets the
timer for another 15 minutes. If the times listed above
are the times at which the door opened from 8:00 to
10:00, approximately how many minutes during this
two-hour period was the light off?
(A)

10

(B)

25

(C)

35

(D)

40

(E)

70

Arithmetic Operations with integers

Look for two consecutive times that are more
than 15 minutes apart
8:03 – 8:00 = 3 minutes
8:04 – 8:03 = 1 minute
8:04 – 8:04 = 0 minutes
8:06 – 8:04 = 2 minutes
8:10 – 8:06 = 4 minutes
8:18 – 8:10 = 8 minutes
8:19 – 8:18 = 1 minute
8:30 – 8:19 = 11 minutes
8:31 – 8:30 = 1 minute
8:54 – 8:31 = 23 minutes, so the light is off for
23 – 15 = 8 minutes
8:57 – 8:54 = 3 minutes
9:05 – 8:57 = 8 minutes
240

9:11 – 9:05 = 6 minutes

Thus, the light is off for a total of
8 + 3 + 14 = 25 minutes during the two-hour
period.
Alternatively, the light comes on at 8:00 when
the door is opened and is scheduled to go off at
8:15. So, when the door is opened at 8:03, twice
at 8:04, at 8:06, and 8:10, the light is still on,
but by the door being opened at these times, the
timer has been reset to turn the light off at 8:18,
8:19, 8:21, and 8:25, respectively. Therefore, the
light is still on when the door is opened at 8:18,
8:19, 8:30, and 8:31, but the timer has been reset
to turn the light off at 8:33, 8:34, 8:45, and 8:46,
respectively. The light is therefore off from 8:46
until the door is opened again at 8:54, which
is an interval of 8 minutes. The light is still on
when the door is opened at 8:57, 9:05, and 9:11,
but the timer has been reset to turn the light off
at 9:12, 9:20, and 9:26, respectively. The light is
off from 9:26 until the door is opened again at
9:29, which is an interval of 3 minutes. The light
is still on when the door is opened again at 9:31,
and the timer has been reset to turn the light off
at 9:46. Since, according to the chart, the door is
not opened again before 10:00, the light remains
off from 9:46 until 10:00, which is an interval
of 14 minutes. Thus, the light is off a total of
8 + 3 + 14 = 25 minutes during the two-hour
period.
The correct answer is B.

5.5 Problem Solving Answer Explanations

60°
60°
174. The parallelogram shown has four sides of equal
length. What is the ratio of the length of the shorter
diagonal to the length of the longer diagonal?
(A)
(B)
(C)
(D)
(E)

1
2
1
2
1
2 2
1
3
1
2 3

Geometry Quadrilaterals; Triangles
120°

30°

30°

Then, because the diagonals of a parallelogram
are perpendicular and bisect each other, the two
diagonals divide the parallelogram into four
30°-60°-90° triangles, each with hypotenuse
x units long. The sides of a 30°-60°-90° triangle
are in the ratio of 1 : 3 : 2 and so, if y represents
the length of the side opposite the 60° angle, then
x = y and y = x 3 . But, y is half the length
2
3
2
of the longer diagonal, so the longer diagonal
has length x 3 units. Therefore, the ratio of the
length of the shorter diagonal to the length of the
longer diagonal is x = 1 .
x 3
3
The correct answer is D.

60°

30°

First, opposite angles of a parallelogram have
equal measure, and the sum of the measures of
adjacent angles is 180°. This means that each of
the angles adjacent to the angle labeled 60° has
measure 120°.
Since all four sides of the parallelogram have
equal length, say x units, the shorter diagonal
divides the parallelogram into two isosceles
triangles. An isosceles triangle with one angle
measuring 60° is equilateral, and so the shorter
diagonal has length x units.

175. If p is the product of the integers from 1 to 30,
inclusive, what is the greatest integer k for which 3k is
a factor of p ?
(A)

10

(B)

12

(C)

14

(D)

16

(E)

18

The longer diagonal divides the parallelogram
into two isosceles triangles with one angle
measuring 120° and each of the other angles
measuring 180 − 120 = 30°, as shown in the
2
figure above on the right.

241

GMAT® Official Guide 2018

Arithmetic Properties of numbers

The table below shows the numbers from 1 to 30,
inclusive, that have at least one factor of 3 and
how many factors of 3 each has.
Multiples of 3
between 1 and 30
3

Number of factors of 3
1

6 = 2×3

1

9 = 3× 3

2

12 = 2 × 2 × 3

1

(A)

4

15 = 3 × 5

1

(B)

3

18 = 2 × 3 × 3

2

(C)

2

21 = 3 × 7

1

24 = 2 × 2 × 2 × 3

1

27 = 3 × 3 × 3

3

30 = 2 × 3 × 5

1

The sum of the numbers in the right column is 14.
Therefore, 314 is the greatest power of 3 that is a
factor of the product of the first 30 positive integers.
The correct answer is C.
176. If n = 38 – 28, which of the following is NOT a factor of n ?
(A)

97

(B)

65

(C)

35

(D)

13

(E)

5

Arithmetic Properties of numbers

Since 38 − 28 is the difference of the perfect squares
(34)2 and (24)2, then 38 – 28 = (34 + 24)(34 – 24).
But 34 – 24 is also the difference of the perfect
squares (32)2 and (22)2 so 34 – 24 = (32 + 22)(32 – 22)
and therefore 38 – 28 = (34 + 24)(32 + 22)(32 – 22).
It follows that 38 – 28 can be factored as
(81 + 16)(9 + 4)(9 – 4) = (97)(13)(5). Therefore, 7
is not a factor of 38 – 28, and hence 35 = 5 × 7
is not a factor of 38 – 28. It is easy to see that each
of 97, 13, and 5 is a factor of 38 – 28, and so is 65,
since 65 = 5 × 13, although this additional analysis
is not needed to arrive at the correct answer.
The correct answer is C.
242

177. In the figure shown, if the area of the shaded region is
3 times the area of the smaller circular region, then the
circumference of the larger circle is how many times
the circumference of the smaller circle?

(D)

3

(E)

2

Geometry Circles

Let R represent the radius of the larger circle
and r represent the radius of the smaller circle.
Then the area of the shaded region is the area of
the larger circular region minus the area of the
smaller circular region, or πR2 – πr2. It is given
that the area of the shaded region is three times
the area of the smaller circular region, and so
πR2 – πr2 = 3πr2. Then R2 – r2 = 3r2, and so
R2 = 4r2 and R = 2r. The circumference of the
larger circle is 2πR = 2π(2r) = 2(2πr), which is
2 times the circumference of the smaller circle.
The correct answer is C.
178. Club X has more than 10 but fewer than 40 members.
Sometimes the members sit at tables with 3 members
at one table and 4 members at each of the other
tables, and sometimes they sit at tables with
3 members at one table and 5 members at each of
the other tables. If they sit at tables with 6 members at
each table except one and fewer than 6 members at
that one table, how many members will be at the table
that has fewer than 6 members?
(A)

1

(B)

2

(C)

3

(D)

4

(E)

5

5.5 Problem Solving Answer Explanations

Arithmetic Properties of numbers

Let n be the number of members that Club X
has. Since the members can be equally divided
into groups of 4 each with 3 left over, and the
members can be equally divided into groups of
5 each with 3 left over, it follows that n – 3 is
divisible by both 4 and 5. Therefore, n – 3 must be
a multiple of (4)(5) = 20. Also, because the only
multiple of 20 that is greater than 10 and less
than 40 is 20, it follows that n – 3 = 20, or n = 23.
Finally, when these 23 members are divided into
the greatest number of groups of 6 each, there
will be 5 members left over, since 23 = (3)(6) + 5.
The correct answer is E.
179. In order to complete a reading assignment on time,
Terry planned to read 90 pages per day. However,
she read only 75 pages per day at first, leaving
690 pages to be read during the last 6 days before the
assignment was to be completed. How many days in
all did Terry have to complete the assignment on time?
(A)

15

(B)

16

(C)

25

(D)

40

(E)

46

Algebra Applied problems

Let n be the number of days that Terry read at
the slower rate of 75 pages per day. Then 75n is
the number of pages Terry read at this slower rate,
and 75n + 690 is the total number of pages Terry
needs to read. Also, n + 6 is the total number
of days that Terry will spend on the reading
assignment. The requirement that Terry average
90 pages per day is equivalent to 75n + 690 = 90.
n+6
Then
75n + 690 = 90
n+6
75n + 690 = 90n + 540
150 = 15n
10 = n

Therefore, the total number of days that Terry
has to complete the assignment on time is
n + 6 = 10 + 6 = 16.
The correct answer is B.
180. If s > 0 and
(A)

r
= s, what is r in terms of s ?
s

1
s

(B)

s

(C)

s s

(D)

s3

(E)

s3 − s

Algebra Equations

Solve the equation for r as follows:
r =s
s
r = s2
s
r = s3

square both sides of the equation
multiply both sides by s

The correct answer is D.
x
181. If 3 < x < 100, for how many values of x is the
3
square of a prime number?
(A)

Two

(B)

Three

(C)

Four

(D)

Five

(E)

Nine

Arithmetic Properties of numbers

If x is the square of a prime number, then
3
possible values of x are 22, 32, 52, 72, ….
3
Therefore, possible values of x are 3 × 22 = 12,
3 × 32 = 27, 3 × 52 = 75, 3 × 72 = 147, …. Since
only three of these values, namely 12, 27, and 75,
are between 3 and 100, there are three values of x
such that x is the square of a prime number.
3
The correct answer is B.

243

GMAT® Official Guide 2018

182. A researcher plans to identify each participant in a
certain medical experiment with a code consisting of
either a single letter or a pair of distinct letters written
in alphabetical order. What is the least number of
letters that can be used if there are 12 participants,
and each participant is to receive a different code?
(A)

4

(B)

5

(C)

6

(D)

7

(E)

8

Arithmetic Elementary combinatorics

None of the essential aspects of the problem
is affected if the letters are restricted to be the
first n letters of the alphabet, for various positive
integers n. With the 3 letters a, b, and c, there are
6 codes: a, b, c, ab, ac, and bc. With the 4 letters
a, b, c, and d, there are 10 codes: a, b, c, d, ab, ac,
ad, bc, bd, and cd. Clearly, more than 12 codes
are possible with 5 or more letters, so the least
number of letters that can be used is 5.
The correct answer is B.
y

Algebra Coordinate geometry

From the graph, line  contains points (–3,0) and
(0,2), so the slope of line  is 0 − 2 = 2 . Any
−3 − 0 3
line parallel to line  has slope 2 . Rewrite each
3
of the equations given in the answer choices in
slope‐intercept form y = mx + b, where m is the
slope and b is the y‐intercept, to find the equation
whose graph is a line with slope 2 . For answer
3
choice A, 3y – 2x = 0 so 3y = 2x and y = 2 x .
3
The graph of this equation is a line with slope 2 .
3
The correct answer is A.
184. An object thrown directly upward is at a height of h feet
after t seconds, where h = –16(t – 3)2 + 150. At what
height, in feet, is the object 2 seconds after it reaches
its maximum height?
(A)

6

(B)

86

(C)

134

(D)

150

(E)

166

Algebra Applied problems
2
1
O

1

x

183. The graph of which of the following equations is a
straight line that is parallel to line  in the figure above?

244

(A)

3y − 2 x = 0

(B)

3y + 2 x = 0

(C)

3y + 2 x = 6

(D)

2y − 3 x = 6

(E)

2y + 3x = −6

Since (t – 3)2 is positive when t ≠ 3 and zero
when t = 3, it follows that the minimum value
of (t – 3)2 occurs when t = 3. Therefore, the
maximum value of –16(t – 3)2, and also the
maximum value of –16(t – 3)2 + 150, occurs
when t = 3. Hence, the height 2 seconds after the
maximum height is the value of h when t = 5, or
–16(5 – 3)2 + 150 = 86.
The correct answer is B.
185. Which of the following is equivalent to the pair of
inequalities x + 6 > 10 and x – 3 ≤ 5 ?
(A)

2 ≤ x < 16

(B)

2≤x<4

(C)

2 10

187. There are 8 teams in a certain league and each team
plays each of the other teams exactly once. If each
game is played by 2 teams, what is the total number of
games played?
(A)

15

(B)

16

(C)

28

(D)

56

Since x > 4 , then 4 < x . Combining 4 < x and
x ≤ 8 gives 4 < x ≤ 8.

(E)

64

The correct answer is D.

Since no team needs to play itself, each team needs
to play 7 other teams. In addition, each game needs
to be counted only once, rather than once for each
team that plays that game. Since two teams play
each game, 8 + 7 = 28 games are needed.
2
The correct answer is C.

x>4
x−3≤5
x ≤8

186. David has d books, which is 3 times as many as Jeff
1
and as many as Paula. How many books do the
2
three of them have altogether, in terms of d ?
(A)

5
d
6

(B)

7
d
3

(C)

10
d
3

(D)

7
d
2

(E)

9
d
2

Algebra Applied problems; Simultaneous
equations

Let J be the number of books that Jeff has, and
let P be the number of books Paula has. Then,
the given information about David’s books can
be expressed as d = 3J and d = 1 P. Solving these
2
two equations for J and P gives d = J and 2d = P.
3
d
Thus, d + J + P = d + + 2d = 3 1 d = 10 d.
3
3
3
The correct answer is C.

Arithmetic Operations on rational numbers

188. At his regular hourly rate, Don had estimated the labor
cost of a repair job as $336 and he was paid that
amount. However, the job took 4 hours longer than he
had estimated and, consequently, he earned $2 per
hour less than his regular hourly rate. What was the
time Don had estimated for the job, in hours?
(A)

28

(B)

24

(C)

16

(D)

14

(E)

12

Algebra Second-degree equations

Let r be Don’s regular hourly rate and t be the
number of hours he estimated the repair job to
take. Then rt = 336 is Don’s estimated labor cost.
Since Don was paid $336 for doing t + 4 hours of
work at an hourly rate of r − 2, it also follows that
(r − 2)(t + 4) = 336. Then

( r − 2)( t + 4 ) = 336
rt − 2t + 4r − 8 = 336
−2t + 4r − 8 = 0
−2t 2 + 4rt − 8t = 0
−2t 2 + 4 ( 336 ) − 8t = 0
t 2 + 4t − 672 = 0

( t − 24 )( t + 28 ) = 0

since rt = 336 from above
multiply both sides by t
since rt = 336
divide both sides by − 2
factor
245

GMAT® Official Guide 2018

Alternatively, from the third line above,

D

− 2t + 4r − 8 = 0
− 2t + 4 ⎛ 336 ⎞ − 8 = 0 since rt = 336 from above
⎝ t ⎠
gives r = 336
t

p 2 need not be less than q. For example,
p
2
if p = 2 and q = 100 , =
and
q 100
q 100 100
= 2 =
= 25 > 1.
p2
2
4
p 3
However, if p = 3 and q = 4 , then =
q 4
q
and 2 = 42 = 4 < 1 . NEED NOT be
p
3
9

−2t 2 + 4 ( 336 ) − 8t = 0 multiply both sides by t
t 2 + 4t − 672 = 0 divide both sides by −2

( t − 24 )( t + 28) = 0

factor

So, t − 24 = 0, which means t = 24, or t + 28 = 0,
which means t = −28. Since an estimated time
cannot be negative, t = 24.
The correct answer is B.

greater than 1.
E

p
q

(B)
(C)
(D)
(E)

p
q2
p
2q
q
p2
q
p

Arithmetic Properties of numbers

p
Since p and q are positive integers, 0 < < 1 .
q
p
A Since < 1, then q > p . Taking the square
q
root of both sides of the inequality gives
q > p . Then, p = p , so here the
q
q
denominator will still be larger than the
numerator. CANNOT be greater than 1.

246

B

Squaring the denominator increases the
denominator, which decreases the value of
the fraction. CANNOT be greater than 1.

C

Multiplying the denominator by 2 increases
the denominator, which decreases the value
of the fraction. CANNOT be greater than 1.

Again, since

p
< 1, then q > p . Thus, the
q

q
, always has a value greater
p
than 1 because the numerator will always
be a larger positive integer than the
denominator. MUST be greater than 1.

reciprocal,

p
189. If < 1, and p and q are positive integers, which of the
q
following must be greater than 1 ?
(A)

p
< 1 , then q > p . When p 2 < q ,
q
this expression will be greater than 1, but

Since

The correct answer is E.
190. To mail a package, the rate is x cents for the first
pound and y cents for each additional pound, where
x > y . Two packages weighing 3 pounds and 5 pounds,
respectively, can be mailed separately or combined as
one package. Which method is cheaper, and how much
money is saved?
(A)

Combined, with a savings of x − y cents

(B)

Combined, with a savings of y − x cents

(C)

Combined, with a savings of x cents

(D)

Separately, with a savings of x − y cents

(E)

Separately, with a savings of y cents

Algebra Applied problems

Shipping the two packages separately would cost
1x + 2 y for the 3‐pound package and 1x + 4 y for
the 5‐pound package. Shipping them together (as
a single 8‐pound package) would cost 1x + 7 y . By
calculating the sum of the costs for shipping the
two packages separately minus the cost for

5.5 Problem Solving Answer Explanations

shipping the one combined package, it is possible
to determine the difference in cost, as shown.

((1x + 2 y ) + (1x + 4 y )) − (1x + 7 y )

(cost for 3 lb.
+ cost for 5 lb.)
− cost for 8 lb.

192. On a recent trip, Cindy drove her car 290 miles,
rounded to the nearest 10 miles, and used 12 gallons
of gasoline, rounded to the nearest gallon. The actual
number of miles per gallon that Cindy’s car got on this
trip must have been between

combine like
terms

(A)

290
290
and
12.5
11.5

= 2x + 6 y − 1x − 7 y

distribute the
negative

(B)

295
285
and
12
11.5

=x− y

combine like
terms

(C)

285
295
and
12
12

(D)

285
295
and
12.5
11.5

(E)

295
285
and
12.5
11.5

= ( 2x + 6 y ) − (1x + 7 y )

Since x > y , this value is positive, which means it
costs more to ship two packages separately. Thus
it is cheaper to mail one combined package at a
cost savings of x − y cents.
The correct answer is A.
191. If money is invested at r percent interest, compounded
annually, the amount of the investment will double
70
in approximately
years. If Pat’s parents invested
r
$5,000 in a long‐term bond that pays 8 percent
interest, compounded annually, what will be the
approximate total amount of the investment 18 years
later, when Pat is ready for college?
(A)

$20,000

(B)

$15,000

(C)

$12,000

(D)

$10,000

(E)

$9,000

Algebra Applied problems

Since the investment will double in
70 = 70 = 8.75 ≈ 9 years, the value of the
r
8
investment over 18 years can be approximated by
doubling its initial value twice. Therefore, the
approximate value will be ($5, 000 ) ( 2 )( 2 ) = $20, 000.
The correct answer is A.

Arithmetic Estimation

The lowest number of miles per gallon can be
calculated using the lowest possible miles and
the highest amount of gasoline. Also, the highest
number of miles per gallon can be calculated
using the highest possible miles and the lowest
amount of gasoline.
Since the miles are rounded to the nearest
10 miles, the number of miles is between 285
and 295. Since the gallons are rounded to the
nearest gallon, the number of gallons is between
11.5 and 12.5. Therefore, the lowest number
lowest miles
= 285
of miles per gallon is
highest gallons 12.5
and the highest number of miles per gallon is
highest miles 295
=
lowest gallons 11.5
The correct answer is D.
–5 –4 –3 –2 –1

0

1

2

3

4

5

x

193. Which of the following inequalities is an algebraic
expression for the shaded part of the number line
above?
(A)

x ≤3

(B)

x ≤5

(C)

x −2 ≤ 3

(D)

x −1 ≤ 4

(E)

x +1 ≤ 4
247

GMAT® Official Guide 2018

Algebra Inequalities

The number line above shows −5 ≤ x ≤ 3 . To turn
this into absolute value notation, as all the choices
are written, the numbers need to be opposite
signs of the same value.
Since the distance between −5 and 3 is 8
(3 – (–5) = 8), that distance needs to be split in
half with −4 to one side and 4 to the other. Each
of these two values is 1 more than the values in
the inequality above, so adding 1 to all terms in
the inequality gives −4 ≤ x + 1 ≤ 4 , which is the
same as x + 1 ≤ 4 .
The correct answer is E.
194. In a small snack shop, the average (arithmetic mean)
revenue was $400 per day over a 10‐day period.
During this period, if the average daily revenue was
$360 for the first 6 days, what was the average daily
revenue for the last 4 days?
(A)

$420

(B)

$440

(C)

$450

(D)

$460

(E)

$480

(A)

2

(B)

5

(C)

6

(D)

7

(E)

14

Arithmetic Properties of numbers

To find the smallest positive integer y such that
3,150y is the square of an integer, first find the
prime factorization of 3,150 by a method similar
to the following:
3,150 = 10 × 315
= ( 2 × 5 ) × ( 3 × 105 )
= 2 × 5 × 3 × ( 5 × 21)
= 2 × 5 × 3 × 5 × (3 × 7 )
= 2 × 32 × 5 2 × 7

Arithmetic; Algebra Statistics; Applied
problems

Let x be the average daily revenue
for the last 4 days. Using the formula
sum of values
average =
, the information
number of values
regarding the average revenues for the 10‐day
and 6‐day periods can be expressed as follows and
solved for x:
6 ($360 ) + 4 x
$400 =
10
$4, 000 = $2,160 + 4 x multiply both sides by 10
$1,840 = 4 x

subtract $2,160 from
both sides

$460 = x

divide both sides by 4

The correct answer is D.

195. If y is the smallest positive integer such that 3,150
multiplied by y is the square of an integer, then y must be

To be a perfect square, 3,150y must have an
even number of each of its prime factors. At a
minimum, y must have one factor of 2 and one
factor of 7 so that 3,150y has two factors of each
of the primes 2, 3, 5, and 7. The smallest positive
integer value of y is then (2)(7) = 14.
The correct answer is E.
196. If [x] is the greatest integer less than or equal to x,
what is the value of [ −1.6] + [ 3.4 ] + [2.7] ?
(A)

3

(B)

4

(C)

5

(D)

6

(E)

7

Arithmetic Computation with integers

The greatest integer that is less than or equal
to −1.6 is −2. It cannot be −1 because −1 is
greater than −1.6. The greatest integer that is
less than or equal to 3.4 is 3. It cannot be 4
because 4 is greater than 3.4. The greatest integer
that is less than or equal to 2.7 is 2. It cannot
be 3 because 3 is greater than 2.7. Therefore,
[ −1.6] + [ 3.4 ] + [ 2.7 ] = −2 + 3 + 2 = 3.
The correct answer is A.

248

5.5 Problem Solving Answer Explanations

197. In the first week of the year, Nancy saved $1. In each
of the next 51 weeks, she saved $1 more than she
had saved in the previous week. What was the total
amount that Nancy saved during the 52 weeks?
(A)

$1,326

(B)

$1,352

(C)

$1,378

(D)

$2,652

(E)

$2,756

Arithmetic Operations on rational numbers

In dollars, the total amount saved is the sum
of 1, (1 + 1), (1 + 1 + 1), and so on, up to and
including the amount saved in the 52nd week,
which was $52. Therefore, the total amount saved
in dollars was 1 + 2 + 3 + … + 50 + 51 + 52. This
sum can be easily evaluated by grouping the terms
as (1 + 52) + (2 + 51) + (3 + 50) + … + (26 + 27),
which results in the number 53 added to itself
26 times. Therefore, the sum is (26)(53) = 1,378.
Alternatively, the formula for the sum of the
n (n + 1)
. Therefore,
first n positive integers is
2
the sum of the first 52 positive integers is
52 ( 53)
= 26 ( 53) = 1, 378.
2
The correct answer is C.
198. In a certain sequence, the term xn is given by the
1
formula x n = 2x n − 1 − ( x n − 2 ) for all n ≥ 2. If x0 = 3
2
and x1 = 2, what is the value of x3 ?
(A)

2.5

(B)

3.125

(C)

4

(D)

5

(E)

6.75

Algebra Simplifying algebraic expressions

Given the formula xn = 2xn − 1 − 1 ( xn − 2 ) with
2
x0 = 3 and x1 = 2, then
x 2 = 2x1 − 1 x0
2
1
= 2(2) − (3)
2
5
=
2
x3 = 2x 2 − 1 x1
2
⎛ 5⎞ 1
= 2 ⎜⎝ ⎟⎠ − (2)
2
2
=5–1
=4
The correct answer is C.
199. During a trip, Francine traveled x percent of the total
distance at an average speed of 40 miles per hour
and the rest of the distance at an average speed of
60 miles per hour. In terms of x, what was Francine’s
average speed for the entire trip?
(A)

180 − x
2

(B)

x + 60
4

(C)

300 − x
5

(D)

600
115 − x

(E)

12,000
x + 200

Algebra Applied problems

Assume for simplicity that the total distance of
Francine’s trip is 100 miles. Then the table below
gives all of the pertinent information.
Distance

Rate

x

40

100 – x

60

Time = Distance
Rate
x
40
100 − x
60

249

GMAT® Official Guide 2018

The total time for Francine’s trip is
x + 100 − x
3x + 2(100 − x )
=
40
60
120
120
3x + 2(100 − x )
120
3x + 200 − 2x
=
120
= x + 200
120
Francine’s average speed over the entire trip is
total distance = 100 = 12, 000 .
x + 200 x + 200
total time
120
=

The correct answer is E.
200. If n = (33)43+ (43)33, what is the units digit of n ?
(A)

0

(B)

2

(C)

4

(D)

6

(E)

8

Arithmetic Properties of numbers

If the units digit of an integer n is 3, then the
units digits of n1, n2, n3, n4, n5, n6, n7, and n8
are, respectively, 3, 9, 7, 1, 3, 9, 7, and 1. Thus,
the units digit of the powers of n form the
sequence in which the digits 3, 9, 7, and 1 repeat
indefinitely in that order. Since 43 = (10)(4) + 3, the
43rd number in the sequence is 7, and therefore
the units digit of (33)43 is 7. Since 33 = (8)(4) + 1, the
33rd number of this sequence is 3, and therefore,
the units digit of (43)33 is 3. Thus, the units digit
of (33)43 + (43)33 is the units digit of 7 + 3, which
is 0.
The correct answer is A.

201. Team A and Team B are competing against each other
in a game of tug-of-war. Team A, consisting of 3 males
and 3 females, decides to line up male, female, male,
female, male, female. The lineup that Team A chooses
will be one of how many different possible lineups?
(A)

9

(B)

12

(C)

15

(D)

36

(E)

720

Arithmetic Elementary combinatorics

Any of the 3 males can be first in the line, and
any of the 3 females can be second. Either of the
2 remaining males can be next, followed by either
of the 2 remaining females. The last 2 places in
the line are filled with the only male left followed
by the only female left. By the multiplication
principle, there are 3 × 3 × 2 × 2 × 1 × 1 = 36
different lineups possible.
The correct answer is D.
202. A border of uniform width is placed around a
rectangular photograph that measures 8 inches by
10 inches. If the area of the border is 144 square
inches, what is the width of the border, in inches?
(A)

3

(B)

4

(C)

6

(D)

8

(E)

9

Algebra Second-degree equations

8

10

x

8 + 2x

x
10 + 2x

Note: Figure not drawn to scale.
Let x be the width, in inches, of the border. The
photograph with the border has dimensions
(10 + 2x) inches and (8 + 2x) inches with an area
of (10 + 2x)(8 + 2x) = (80 + 36x + 4x2) square
250

5.5 Problem Solving Answer Explanations

inches. The photograph without the border has
dimensions 10 inches and 8 inches with an area
of (10)(8) = 80 square inches. The area of the
border is then the difference between the areas
of the photograph with and without the border
or (80 + 36x + 4x2) – 80 = 36x + 4x2 square
inches. It is given that the area of the border is
144 square inches so,
=
=
=
=

36x + 4x2
4x2 + 36x − 144
x2 + 9x − 36
(x − 3)(x + 12)

144
0
0
0

Thus, after discarding x = –12 since the width of
the border must be positive, x = 3.
The correct answer is A.
1
is expressed as a terminating decimal,
23 × 57
how many nonzero digits will d have?

203. If d =

One

(B)

Two

(C)

Three

(D)

Seven

(E)

Ten

(A)

10,100

(B)

20,200

(C)

22,650

(D)

40,200

(E)

45,150

Algebra Simplifying expressions; Arithmetic
Computation with integers

So, x – 3 = 0, which means x = 3, or x + 12 = 0,
which means x = –12.

(A)

204. For any positive integer n, the sum of the first
n ( n + 1)
. What is the sum of
n positive integers equals
2
all the even integers between 99 and 301 ?

The given formula translates into
n
n (n + 1)
1 + 2 + ... + n = ∑ k =
. The sum of the
2
k =1
even integers between 99 and 301 is the sum of
the even integers from 100 through 300, or the
sum of the 50th even integer through the 150th
even integer. To get this sum, find the sum of the
first 150 even integers and subtract the sum of
the first 49 even integers. In symbols,
150

49

150

49

k =1

k =1

k =1

k =1

∑ 2 k − ∑ 2 k = 2∑ k − 2∑ k
⎛ 150 (150 + 1) ⎞
⎛ 49 ( 49 + 1) ⎞
= 2⎜
− 2⎜
⎟⎠
⎟
⎝
2
⎝
2
⎠
= 150 (151) − 49 ( 50 )
= 50 ⎡⎣ 3 (151) − 49 ⎤⎦

Arithmetic Operations on rational numbers

= 50 ( 453 − 49 )

It will be helpful to use the fact that a factor that is
an integer power of 10 has no effect on the number
of nonzero digits a terminating decimal has.
1
= 3 1 3 × 14
7
2 ×5
2 ×5
5
3

= ⎛⎜ 1 ⎞⎟ × ⎛⎜ 1 ⎞⎟
⎝ 2 × 5⎠ ⎝ 5⎠
3

= 20, 200
The correct answer is B.

3

= ⎛⎜ 1 ⎞⎟ × ⎛⎜ 1 ⎞⎟
⎝ 10 ⎠ ⎝ 5 ⎠

= 50 ( 404 )

4

205. How many prime numbers between 1 and 100 are
factors of 7,150 ?

4

(A)

One

(B)

Two

(C)

Three

= 10 × (0.0016)

(D)

Four

= 0.0000016

(E)

Five

= 10 −3 × (0.2)4
−3

The correct answer is B.

251

GMAT® Official Guide 2018

Arithmetic Properties of numbers

Algebra Percents

To find the number of prime numbers between 1
and 100 that are factors of 7,150, find the prime
factorization of 7,150 using a method similar to
the following:

If P and E are the price and earnings per share
before the increase, then ⎛⎜ 1 + k ⎞⎟ P and
⎝
100 ⎠
⎛ 1 + m ⎞ E are the price and earnings per share
⎜⎝
⎟
100 ⎠
after the increase. Therefore, the percent increase
in the ratio of price per share to earnings per
share can be expressed as follows:

7,150 = 10 × 715
= ( 2 × 5 ) × ( 5 × 143)
= 2 × 5 × 5 × (11 × 13)
Thus, 7,150 has four prime factors: 2, 5, 11, and 13.
The correct answer is D.
206. A sequence of numbers a1, a2, a3, . . . is defined as
follows: a1 = 3, a2 = 5, and every term in the sequence
after a2 is the product of all terms in the sequence
preceding it, e.g., a3 = (a1)(a2) and a4 = (a1)(a2)(a3). If
an = t and n > 2, what is the value of an + 2 in terms of t ?
(A)

4t

(B)

t2

(C)

t3

(D)

t4

(E)

t8

Algebra Sequences

It is given that an = (a1)(a2) … (an – 1) and an = t.
Therefore, an + 1 = (a1)(a2) … (an – 1)(an) =
(an)(an) = t 2 and an + 2 = (a1)(a2) … (an)(an + 1) =
(an + 1)(an + 1) = (t 2)(t 2) = t 4.
The correct answer is D.
207. Last year the price per share of Stock X increased
by k percent and the earnings per share of Stock X
increased by m percent, where k is greater than m.
By what percent did the ratio of price per share to
earnings per share increase, in terms of k and m ?
(A)

k
%
m

(B)

(k − m)%

(C)
(D)
(E)

252

100 (k − m )
%
100 + k
100 (k − m )
%
100 + m
100 (k − m )
%
100 + k + m

⎛ ( ratio after increases ) − ( ratio before increases )
⎞
× 100 ⎟ %
⎜⎝
( ratio before increases)
⎠
⎡⎛ ( ratio after increases)
⎤
⎞
= ⎢⎜
− 1⎟ × 100 ⎥ %
⎣⎝ ( ratio before increases ) ⎠
⎦
⎡⎛ ⎛
⎤
⎞
k ⎞
⎢⎜ ⎝ 1 + 100 ⎠ P
⎥
⎟
⎢⎜
⎥
⎟
⎢⎜ ⎛ 1 + m ⎞ E
⎥
⎟
⎝
⎠
100
⎢
⎜
⎟
=
− 1 × 100 ⎥ %
⎢⎜
⎥
P
⎟
⎢⎜
⎥
⎟
E
⎢⎜
⎥
⎟
⎢⎜
⎥
⎟
⎠
⎢⎣⎝
⎥⎦
⎡⎛ ⎛
⎤
⎞
k ⎞
⎢⎜ ⎝ 1 + 100 ⎠ P
⎥
⎟
⋅
⎢⎜
⎥
⎟
⎢⎜ ⎛ 1 + m ⎞ E
⎥
⎟
⎝
⎠
100
⎢
⎜
⎟
=
− 1 × 100 ⎥ %
⎢⎜
⎥
P
⎟
⎢⎜
⎥
⎟
E
⎢⎜
⎥
⎟
⎢⎜
⎥
⎟
⎠
⎢⎣⎝
⎥⎦
⎡⎛ ⎛
⎤
⎞
k ⎞
⎢⎜ ⎝ 1 + 100 ⎠
⎥
⎟
⎢⎜
⎥
⎟
⎢⎜ ⎛ 1 + m ⎞
⎥
⎟
⎝
⎠
100
⎢
⎜
⎟
=
− 1 × 100 ⎥ %
⎢⎜
⎥
⎟
1
⎢⎜
⎥
⎟
⎢⎜
⎥
⎟
⎢⎜
⎥
⎟
⎠
⎢⎣⎝
⎥⎦
⎡⎛ 1 + k
⎤
⎞
⎢⎜
⎥
100
⎟
=⎢
− 1 × 100 ⎥ %
⎜1+ m
⎟
⎟⎠
⎢⎜⎝
⎥
100
⎣
⎦

5.5 Problem Solving Answer Explanations

⎡⎛ 1 + k
⎤
⎞
⎢⎜
⎥
100
100
⎟
=⎢
− 1 × 100 ⎥ %
⋅
⎜
m 100 ⎟
⎟⎠
⎢⎜⎝ 1 +
⎥
100
⎣
⎦
⎡
⎤
= ⎢⎛ 100 + k − 1⎞ × 100 ⎥ %
⎝
⎠
100
+
m
⎣
⎦
⎛ (100 + k ) − (100 + m )
⎞
=⎜
× 100 ⎟ %
⎝
⎠
100 + m
= ⎛ k − m × 100 ⎞ %
⎝ 100 + m
⎠
=

100 ( k − m )
%
100 + m

The correct answer is D.
208. Of the 300 subjects who participated in an experiment
using virtual‐reality therapy to reduce their fear of
heights, 40 percent experienced sweaty palms,
30 percent experienced vomiting, and 75 percent
experienced dizziness. If all of the subjects
experienced at least one of these effects and
35 percent of the subjects experienced exactly two of
these effects, how many of the subjects experienced
only one of these effects?
(A)

105

(B)

125

(C)

130

(D)

180

(E)

195

Arithmetic Applied problems

Let a be the number who experienced only one
of the effects, b be the number who experienced
exactly two of the effects, and c be the number
who experienced all three of the effects. Then
a + b + c = 300, since each of the 300 participants
experienced at least one of the effects. From the
given information, b = 105 (35% of 300), which
gives a + 105 + c = 300, or a + c = 195 (Eq. 1).
Also, if the number who experienced sweaty
palms (40% of 300, or 120) is added to the
number who experienced vomiting (30% of 300,
or 90), and this sum is added to the number who
experienced dizziness (75% of 300, or 225), then
each participant who experienced only one of the
effects is counted exactly once, each participant

who experienced exactly two of the effects is
counted exactly twice, and each participant who
experienced all three of the effects is counted
exactly 3 times. Therefore, a + 2b + 3c =
120 + 90 + 225 = 435. Using b = 105, it follows
that a + 2(105) + 3c = 435, or a + 3c = 225 (Eq. 2).
Then solving the system defined by Eq. 1 and Eq. 2,

{

a + c = 195
a + 3c = 225

multiply 1st equation by −3

⎧−3a − 3c = −585
add equations
⎨
⎩⎪ a + 3c = 225
−2a = −360, or a = 180
The correct answer is D.
209. If m −1 =

1
, then m−2 is equal to
3

(A)

−9

(B)

−3

(C)

−

1
9

(D)

1
9

(E)

9

Arithmetic Negative exponents

Using rules of exponents, m −2 = m −1 ⋅ 2 = (m −1 ) ,
2

2

and since m −1 = − 1 , m −2 = ⎛ − 1 ⎞ = 1 .
⎝ 3⎠
3
9
The correct answer is D.
210. A photography dealer ordered 60 Model X cameras
to be sold for $250 each, which represents a
20 percent markup over the dealer’s initial cost for
each camera. Of the cameras ordered, 6 were never
sold and were returned to the manufacturer for a
refund of 50 percent of the dealer’s initial cost. What
was the dealer’s approximate profit or loss as a
percent of the dealer’s initial cost for the 60 cameras?
(A)

7% loss

(B)

13% loss

(C)

7% profit

(D)

13% profit

(E)

15% profit
253

GMAT® Official Guide 2018

Arithmetic Percents

Given that $250 is 20% greater than a camera’s
initial cost, it follows that the initial cost for each
camera was ⎛⎜ $ 250 ⎞⎟ . Therefore, the initial cost
⎝ 1.2 ⎠
for the 60 cameras was 60 ⎛⎜ $ 250 ⎞⎟ . The total
⎝ 1.2 ⎠
revenue is the sum of the amount obtained from
selling 60 − 6 = 54 cameras for $250 each and
the ⎛⎜ 1 ⎞⎟ ⎛⎜ $ 250 ⎞⎟ refund for each of 6 cameras,
⎝ 2 ⎠ ⎝ 1.2 ⎠
or ( 54 )($250 ) + ( 6 ) ⎛⎜ 1 ⎞⎟ ⎛⎜ $ 250 ⎞⎟ . The total
⎝ 2 ⎠ ⎝ 1.2 ⎠
profit, as a percent of the total initial cost, is
⎞
⎛ ( total revenue ) − ( total initial cost )
× 100 ⎟ % =
⎜⎝
( total initial cost )
⎠
⎛ ⎛ ( total revenue )
⎞
⎞
⎜ ⎜ total initial cost − 1⎟ × 100 ⎟ % . Using
) ⎠
⎝⎝ (
⎠
the numerical expressions obtained above,
( total revenue ) − 1
total
initial cost )
(

=

( 54 )( 250 ) + 6 ⎛⎝ 1 ⎞⎠ ⎛⎝ 250 ⎞⎠
2 1.2
⎛
250
( 60 ) ⎝ ⎞⎠
1.2

−1

54 + 3 ⎛ 1 ⎞
⎝ 1.2 ⎠
=
−1
⎛
⎞
1
( 60 ) ⎝ ⎠
1.2
=

54 (1.2 ) + 3
−1
60

by substitution

by canceling 250s

by multiplying
top and bottom by 1.2
and then canceling 1.2

67.8 1
=
−
60
= 1.13 − 1
= 0.13
Finally, (0.13 × 100)% = 13%, which represents a
profit since it is positive.
The correct answer is D.

254

211. Seven pieces of rope have an average (arithmetic
mean) length of 68 centimeters and a median length
of 84 centimeters. If the length of the longest piece of
rope is 14 centimeters more than 4 times the length
of the shortest piece of rope, what is the maximum
possible length, in centimeters, of the longest piece of
rope?
(A)

82

(B)

118

(C)

120

(D)

134

(E)

152

Algebra Statistics

Let a, b, c, d, e, f, and g be the lengths, in
centimeters, of the pieces of rope, listed from
least to greatest. From the given information it
follows that d = 84 and g = 4a + 14. Therefore,
listed from least to greatest, the lengths are a,
b, c, 84, e, f, and 4a + 14. The maximum value
of 4a + 14 will occur when the maximum value
of a is used, and this will be the case only if
the shortest 3 pieces all have the same length.
Therefore, listed from least to greatest, the lengths
are a, a, a, 84, e, f, and 4a + 14. The maximum
value for 4a + 14 will occur when e and f are
as small as possible. Since e and f are to the
right of the median, they must be at least 84
and so 84 is the least possible value for each of
e and f. Therefore, listed from least to greatest,
the lengths are a, a, a, 84, 84, 84, and 4a + 14.
Since the average length is 68, it follows that
a + a + a + 84 + 84 + 84 + ( 4 a + 14 )
= 68, or a = 30 .
7
Hence, the maximum length of the longest piece
is (4a + 14) = [4(30) + 14] = 134 centimeters.
The correct answer is D.
212. What is the difference between the sixth and the fifth
terms of the sequence 2, 4, 7, … whose nth term is
n + 2n − 1 ?
(A)

2

(B)

3

(C)

6

(D)

16

(E)

17

5.5 Problem Solving Answer Explanations

Algebra Simplifying algebraic expressions

Arithmetic Elementary combinatorics

According to the given formula, the sixth term of
the sequence is 6 + 26 − 1 = 6 + 25 and the fifth
term is 5 + 25 – 1 = 5 + 24. Then,

There are 6 ways to select the locations of the
2 occurrences of the letter I, and this number can
be determined by listing all such ways as shown
below, where the symbol * is used in place of the
letters D, G, and T:

(6 + 2 ) − ( 5 + 2 ) = (6 − 5) + ( 2
5

4

5

− 24

)

= 1 + 2 ( 2 − 1)
4

=1 + 2

= 1 + 16
= 17
The correct answer is E.
213. From the consecutive integers –10 to 10, inclusive,
20 integers are randomly chosen with repetitions
allowed. What is the least possible value of the product
of the 20 integers?
(A)

(−10)20

(B)

(−10)10

(C)

0

(D)

−(10)19

(E)

−(10)20

Arithmetic Properties of numbers

If −10 is chosen an odd number of times and
10 is chosen the remaining number of times
(for example, choose −10 once and choose
10 nineteen times, or choose −10 three times and
choose 10 seventeen times), then the product
20
of the 20 chosen numbers will be − (10 ) . Note
20
19
that − (10 ) is less than − (10 ) , the only other
negative value among the answer choices.
The correct answer is E.
214. The letters D, G, I, I, and T can be used to form 5‐letter
strings such as DIGIT or DGIIT. Using these letters, how
many 5‐letter strings can be formed in which the two
occurrences of the letter I are separated by at least
one other letter?
(A)

12

(B)

18

(C)

24

(D)

36

(E)

48

I*I**, I**I*, I***I, *I*I*, *I**I, **I*I

4

Alternatively, the number of ways to select
the locations of the 2 occurrences of the
5
letter I can be determined by using ⎛⎜ ⎞⎟ – 4 =
⎝ 2⎠
5!
– 4 = 10 – 4 = 6, which is the number of
(2!)(3!)
ways to select 2 of the 5 locations minus the 4 ways
in which the 2 selected locations are adjacent.
For each of these 6 ways to select the locations of
the 2 occurrences of the letter I, there are 6 ways
to select the locations of the letters D, G, and T,
which can be determined by using 3! = 6 or by
listing all such ways:
DGT, DTG, GDT, GTD, TDG, TGD
It follows that the number of ways to select the
locations of the 5 letters to form 5‐letter strings is
(6)(6) = 36.
The correct answer is D.
215. Last Sunday a certain store sold copies of
Newspaper A for $1.00 each and copies of
Newspaper B for $1.25 each, and the store sold no
other newspapers that day. If r percent of the store’s
revenue from newspaper sales was from Newspaper A
and if p percent of the newspapers that the store sold
were copies of Newspaper A, which of the following
expresses r in terms of p ?
(A)

100p
125 − p

(B)

150p
250 − p

(C)

300p
375 − p

(D)

400p
500 − p

(E)

500p
625 − p
255

GMAT® Official Guide 2018

Algebra Simultaneous equations

Let N be the total number of newspapers
that the store sold. Then, the number of
copies of Newspaper A the store sold was
⎛ p ⎞
p % of N = ⎜
N and the revenue from
⎝ 100 ⎟⎠
those copies of Newspaper A, in dollars, was
p
p
(1.00 ) ⎛⎜⎝ ⎞⎟⎠ N = ⎛⎜⎝ ⎞⎟⎠ N . The number of
100
100
copies of Newspaper B the store sold was
⎛ 100 − p ⎞
N and the revenue
(100 − p)% of N = ⎜
⎝ 100 ⎟⎠
from those copies of Newspaper B, in dollars,
⎛ 100 − p ⎞
⎛ 100 − p ⎞
N = ⎛ 5⎞ ⎜
was (1.25 ) ⎜
⎟
⎟ N.
⎝
⎠
⎝ 100 ⎠
4 ⎝ 100 ⎠
The store’s total revenue from newspaper sales,
⎛ p ⎞
⎛ 100 − p ⎞
N + ⎛ 5⎞ ⎜
in dollars, was ⎜
⎟
⎟ N,
⎝
⎠
⎝ 100 ⎠
4 ⎝ 100 ⎠
and the fraction of that revenue from the sale of
Newspaper A was
p
p
N
100
100
=
p
4 p ⎛ 500 − 5 p ⎞
⎛ 100 − p ⎞
5
⎛
⎞
N+
N
+⎜
⎟
⎝ 4 ⎠ ⎜⎝ 100 ⎟⎠
100
400 ⎝ 400 ⎠
p
100
=
4 p + 500 − 5 p
400
p
= 100
500 − p
400
⎛ p ⎞ ⎛ 400 ⎞
=⎜
⎝ 100 ⎟⎠ ⎜⎝ 500 − p ⎟⎠
4p
=
500 − p
Since r percent of the store’s newspaper sales
r = 4p ,
revenue was from Newspaper A,
100 500 − p
400 p
and so r =
.
500 − p
The correct answer is D.

256

216.

0.99999999 0.99999991
−
=
1.0001
1.0003
(A)

10−8

(B)

3(10−8 )

(C)

3(10−4 )

(D)

2(10−4 )

(E)

10−4

Arithmetic Operations on rational numbers

Calculations with lengthy decimals can be
avoided by writing 0.99999999 as 1 − 10−8,
0.99999991 as 1 − 9(10 −8 ), 1.0001 as 1 + 10 −4 , and
1.0003 as 1 + 3(10 −4 ). Doing this gives
−8
1 − 10 −8 − 1 − 9 (10 )
1 + 10 −4 1 + 3 (10 −4 )

⎡⎣1 + 10 −4 ⎤⎦ ⎡⎣1 − 10 −4 ⎤⎦ 1 − 9 (10 −8 )
=
−
1 + 10 −4
1 + 3 (10 −4 )
= 1 − 10
1
=

−4

−

1 − 9(10 −8 )
1 + 3 (10 −4 )

⎡⎣1 − 10 −4 ⎤⎦ ⎡⎣1 + 3 (10 −4 ) ⎤⎦ − ⎡⎣1 − 9 (10 −8 ) ⎤⎦
1 + 3 (10 −4 )

=

1 + 3 (10 −4 ) − 10 −4 − 3 (10 −8 ) − 1 + 9 (10 −8 )

=

2 (10 −4 ) + 6 (10 −8 )

1 + 3 (10 −4 )

1 + 3 (10 −4 )

⎡⎣ 2 (10 −4 ) ⎤⎦ ⎡⎣1 + 3 (10 −4 ) ⎤⎦
=
1 + 3 (10 −4 )
= 2 (10 −4 )

The correct answer is D.
217. For the past n days, the average (arithmetic mean)
daily production at a company was 50 units. If today’s
production of 90 units raises the average to 55 units
per day, what is the value of n ?
(A)

30

(B)

18

(C)

10

(D)

9

(E)

7

5.5 Problem Solving Answer Explanations

Arithmetic; Algebra Statistics; Applied
problems; Simultaneous equations

Let x be the total production of the past n days.
sum of values
Using the formula average =
,
number of values
the information in the problem can be expressed
in the following two equations
50 = x
n

daily average of 50 units
over the past n days

55 = x + 90
n +1

increased daily average
when including today’s
90 units

Solving the first equation for x gives x = 50n.
Then substituting 50n for x in the second
equation gives the following that can be solved
for n:
55 = 50n + 90
n +1
55(n + 1) = 50n + 90
55n + 55 = 50n + 90
5n = 35
n=7

multiply both sides
by (n + 1)
distribute the 55
subtract 50n and 55
from both sides
divide both sides by 5

y

1
1

The line is shown going through the points (0,2)
and (3,0). The slope of the line can be found with
change in y y 2 − y1
=
for
the formula slope =
change in x x 2 − x1
two points (x1,y1) and (x2,y2). Thus, the slope of
this line equals 0 − 2 = − 2 . Using the formula
3− 0
3
for a line of y = mx +b, where m is the slope and
b is the y‐intercept (in this case, 2), an equation
for this line is y = − 2 x + 2 . Since this equation
3
must be compared to the available answer choices,
the following further steps should be taken:
y = −2x+ 2
3
3y = −2x + 6

multiply both sides by 3

2x + 3y = 6

add 2x to both sides

This problem can also be solved as follows. From
the graph, when x = 0, y is positive; when y = 0,
x is positive. This eliminates all but B and C. Of
these, B is the only line containing (0,2). Still
another way is to use (0,2) to eliminate A, C, and
E, and then use (3,0) to eliminate D.
The correct answer is B.
219. If a two‐digit positive integer has its digits reversed,
the resulting integer differs from the original by 27. By
how much do the two digits differ?

The correct answer is E.

O

Geometry Simple coordinate geometry

x

(A)

3

(B)

4

(C)

5

(D)

6

(E)

7

218. In the coordinate system above, which of the following
is the equation of line  ?
(A)

2x − 3y = 6

(B)

2x + 3y = 6

(C)

3x + 2y = 6

(D)

2x − 3y = −6

(E)

3x − 2y = −6

257

GMAT® Official Guide 2018

Algebra Applied problems

Let the one two‐digit integer be represented by
10t + s, where s and t are digits, and let the other
integer with the reversed digits be represented by
10s + t. The information that the difference between
the integers is 27 can be expressed in the following
equation, which can be solved for the answer.
(10s + t) − (10t + s) = 27
10s + t − 10t − s = 27
9s − 9t = 27
s−t=3

distribute the negative
combine like terms
divide both sides by 9

Thus, it is seen that the two digits s and t differ by 3.
The correct answer is A.
220. In an electric circuit, two resistors with resistances x
and y are connected in parallel. In this case, if r is the
combined resistance of these two resistors, then the
reciprocal of r is equal to the sum of the reciprocals of
x and y. What is r in terms of x and y ?

y
side by multiplying the first fraction by and the
y
second fraction by x :
x
1=1+1
r x y
1= y + x
r xy xy
1= x+ y
r
xy
r=

xy
x+ y

221. Xavier, Yvonne, and Zelda each try independently to
solve a problem. If their individual probabilities for
1 1
5
success are , , and , respectively, what is the
4 2
8
probability that Xavier and Yvonne, but not Zelda, will
solve the problem?

xy

(B)

x+y

(A)

(C)

1
x+y

11
8

(B)

(D)

xy
x+y

7
8

(C)

(E)

x+y
xy

9
64

(D)

5
64

(E)

3
64

Note that two numbers are reciprocals of each
other if and only if their product is 1. Thus
the reciprocals of r, x, and y are 1 , 1 , and
r x
1
, respectively. So, according to the problem,
y
1 = 1 + 1.
To solve this equation for r, begin
r x y
by creating a common denominator on the right

invert the fractions on both sides

The correct answer is D.

(A)

Algebra Applied problems

combine the fractions on the right side

Arithmetic Probability

Since the individuals’ probabilities are
independent, they can be multiplied to figure
out the combined probability. The probability of
Xavier’s success is given as 1 , and the probability of
4
Yvonne’s success is given as 1 . Since the probability
2
of Zelda’s success is given as 5 , then the probability
8
of her NOT solving the problem is 1 − 5 = 3 .
8 8
Thus, the combined probability is
⎛ 1 ⎞ ⎛ 1 ⎞ ⎛ 3⎞ = 3 .
⎝ 4 ⎠ ⎜⎝ 2 ⎟⎠ ⎜⎝ 8 ⎟⎠ 64
The correct answer is E.

258

5.5 Problem Solving Answer Explanations

222. If

1
1
1
−
=
, then x could be
x x +1 x + 4

(A)

⎛ 1⎞
223. ⎜ ⎟
⎝ 2⎠

−3

–1

(C)

–2

(D)

–3

(E)

–4

⎛ 1⎞
⎜⎝ ⎟⎠
2

−48

⎛ 1⎞
⎜⎝ ⎟⎠
2

−11

(B)

⎛ 1⎞
⎜⎝ ⎟⎠
2

−6

(C)

⎛ 1⎞
⎜⎝ ⎟⎠
8

−11

(D)

⎛ 1⎞
⎜⎝ ⎟⎠
8

−6

(E)

Algebra Second-degree equations

Solve the equation for x. Begin by multiplying
all the terms by x ( x + 1)( x + 4 ) to eliminate the
denominators.
1− 1 = 1
x x +1 x + 4
(x + 4)(x + 1 – x) = x ( x + 1) factor the (x + 4) out
front on the left side
( x + 4 )(1) = x ( x + 1) simplify

4 = x2
±2 = x

⎛ 1⎞
⎜⎝ ⎟⎠
16

−1

=

Arithmetic Operations on rational numbers

( x + 1)( x + 4 ) − x ( x + 4 ) = x ( x + 1)

2
(x + 4) = x + x

−2

(A)

0

(B)

⎛ 1⎞
⎜⎝ ⎟⎠
4

distribute the x on
the right side
subtract x from both
sides
take the square root
of both sides

Both − 2 and 2 are square roots of 4 since
( −22 ) = 4 and ( 22 ) = 4. Thus, x could be − 2.
This problem can also be solved as follows.
( x + 1) − x = 1 ,
Rewrite the left side as,
x ( x + 1)
x ( x + 1)
then set equal to the right side to get
1
= 1 . Next, cross multiply:
x ( x + 1) x + 4
(1)(x + 4) = x(x + 1)(1). Therefore, x + 4 = x 2 + x ,
or x 2 = 4, so x = ±2 .

It is clear from the answer choices that all three
factors need to be written with a common
denominator, and they thus become
⎛ 1⎞
⎝ 2⎠

−3

= ⎛ 1⎞
⎝ 2⎠

⎛ 1⎞
⎝ 4⎠

−2

⎛⎛ 1⎞2⎞
=⎜
⎝ ⎝ 2 ⎠ ⎟⎠

⎛ 1⎞
⎝ 16 ⎠

−1

−3

−2

4
⎛
⎞
= ⎜⎛ 1⎞ ⎟
⎝ ⎝ 2⎠ ⎠
−3

= ⎛ 1⎞
⎝ 2⎠

−1

−2

= ⎛ 1⎞
⎝ 2⎠

So, ⎛ 1 ⎞ ⎛ 1 ⎞ ⎛ 1 ⎞
⎝ 2 ⎠ ⎝ 4 ⎠ ⎝ 16 ⎠
−3

−4

⎛ 1⎞ ⎛ 1⎞ ⎛ 1⎞
⎝ 2⎠ ⎝ 2⎠ ⎝ 2⎠

−4

−4

−1

−4

=

= ⎛ 1⎞
⎝ 2⎠

−3 − 4 − 4

= ⎛ 1⎞
⎝ 2⎠

−11

.

The correct answer is B.

The correct answer is C.

259

GMAT® Official Guide 2018

the sum of the measures is (9)(180 – x)°. But
the sum of the interior angles of a polygon with
n sides is (n – 2)(180°), so the sum of the interior
angles of a 9-sided polygon is (7)(180°) = 1,260°.
Therefore, (9)(180 – x)° = 1,260° and x = 40.
Finally, a = 180 – 2x = 180 – 2(40) = 100.

a°

The correct answer is A.
224. The figure shown above consists of a shaded 9-sided
polygon and 9 unshaded isosceles triangles. For each
isosceles triangle, the longest side is a side of the
shaded polygon and the two sides of equal length are
extensions of the two adjacent sides of the shaded
polygon. What is the value of a ?
(A)

100

(B)

105

(C)

110

(D)

115

(E)

120

225. List T consists of 30 positive decimals, none of which
is an integer, and the sum of the 30 decimals is S. The
estimated sum of the 30 decimals, E, is defined as
follows. Each decimal in T whose tenths digit is even
is rounded up to the nearest integer, and each decimal
in T whose tenths digit is odd is rounded down to the
nearest integer; E is the sum of the resulting integers.
1
If of the decimals in T have a tenths digit that is even,
3
which of the following is a possible value of E – S ?
I.

Geometry Polygons

x°

x°

x°

a°

x°

x°

x°

x°

− 16

II.

6

III.

10

(A)

I only

(B)

I and II only

(C)

I and III only

(D)

II and III only

(E)

I, II, and III

Arithmetic Operations on rational numbers

x°

x° x°

x°

x°

x°

Let x° represent the measure of each base angle
of the triangle with vertex angle labeled a°. Each
base angle of this triangle and one base angle of a
triangle with which it shares a vertex are vertical
angles and have the same measure. Thus, the base
angles of these triangles also have measure x°.
This pattern continues for the base angles of each
pair of triangles that share a vertex, so each base
angle of each of the 9 triangles has measure x°, as
shown above. Also, the vertex angle of each of the
9 triangles has measure a° = 180° – 2x°.
Each interior angle of the shaded polygon has
measure (180 – x)° since each forms a straight
angle with an angle that has measure x°, and
260

Since 1 of the 30 decimals in T have an even
3
tenths digit, it follows that 1 ( 30 ) = 10 decimals
3
in T have an even tenths digit. Let TE represent
the list of these 10 decimals, let SE represent the
sum of all 10 decimals in TE, and let EE represent
the estimated sum of all 10 decimals in TE after
rounding. The remaining 20 decimals in T have
an odd tenths digit. Let TO represent the list of
these 20 remaining decimals, let SO represent
the sum of all 20 decimals in TO, and let EO
represent the estimated sum of all 20 decimals in
TO after rounding. Note that E = EE + EO
and S = SE + SO and hence E − S =
(EE + EO) − (SE + SO) = (EE − SE) + (EO − SO).
The least values of EE − SE occur at the extreme
where each decimal in TE has tenths digit 8.
Here, the difference between the rounded integer
and the original decimal is greater than 0.1. (For

5.5 Problem Solving Answer Explanations

example, the difference between the integer 15
and 14.899 that has been rounded to 15 is 0.101.)
Hence, EE − SE > 10(0.1) = 1. The greatest values
of EE − SE occur at the other extreme, where
each decimal in TE has tenths digit 0. Here, the
difference between the rounded integer and the
original decimal is less than 1. (For example, the
difference between the integer 15 and 14.001 that
has been rounded to 15 is 0.999.)
Hence, EE − SE < 10(1) = 10. Thus, 1 < EE − SE < 10.

Algebra Second-degree equations

Similarly, the least values of EO − SO occur at the
extreme where each decimal in TO has tenths
digit 9. Here, the difference between the rounded
integer and the original decimal is greater than −1.
(For example, the difference between the integer
14 and 14.999 that has been rounded to 14 is
−0.999.) Hence EO − SO > 20(−1) = −20.
The greatest values of EO − SO occur at the other
extreme where each decimal in TO has tenths
digit 1. Here, the difference between the rounded
integer and the original decimal is less than
or equal to −0.1. (For example, the difference
between the integer 14 and 14.1 that has been
rounded to 14 is −0.1.)
Hence, EO − SO ≤ 20(−0.1) = −2.
Thus, −20 < EO − SO ≤ −2.

The correct answer is C.

Adding the inequalities 1 < EE − SE < 10 and −20 <
EO − SO ≤ −2 gives −19 < (EE – SE) + (EO − SO) < 8.
Therefore, −19 < (EE + EO) − (SE + SO) < 8 and
−19 < E − S < 8. Thus, of the values −16, 6,
and 10 for E − S, only −16 and 6 are possible.
Note that if T contains 10 repetitions of the
decimal 1.8 and 20 repetitions of the decimal 1.9,
S = 10(1.8) + 20(1.9) = 18 + 38 = 56,
E = 10(2) + 20(1) = 40, and E − S = 40 − 56 = −16.
Also, if T contains 10 repetitions of the decimal
1.2 and 20 repetitions of the decimal 1.1,
S = 10(1.2) + 20(1.1) = 12 + 22 = 34,
E = 10(2) + 20(1) = 40, and E − S = 40 − 34 = 6.

Solve the equation to determine how many values
are possible for x.
5− 6 = x
x
5x − 6 = x 2
0 = x 2 − 5x + 6
0 = ( x − 3)( x − 2 )
x = 3 or 2

227. Seed mixture X is 40 percent ryegrass and 60 percent
bluegrass by weight; seed mixture Y is 25 percent
ryegrass and 75 percent fescue. If a mixture of X and
Y contains 30 percent ryegrass, what percent of the
weight of the mixture is X ?
(A)

10%

(B)

1
33 %
3

(C)

40%

(D)

50%

(E)

2
66 %
3

Algebra Applied problems

Let X be the amount of seed mixture X in
the final mixture, and let Y be the amount of
seed mixture Y in the final mixture. The final
mixture of X and Y needs to contain 30 percent
ryegrass seed, so any other kinds of grass seed
are irrelevant to the solution to this problem. The
information about the ryegrass percentages for X,
Y, and the final mixture can be expressed in the
following equation and solved for X.
0.40 X + 0.25Y = 0.30 ( X + Y )
0.40 X + 0.25Y = 0.30 X + 0.30Y

The correct answer is B.
226. If 5 −

6
= x , then x has how many possible values?
x

(A)

None

(B)

One

(C)

Two

(D)

A finite number greater than two

(E)

An infinite number

0.10 X = 0.05Y

X = 0.5Y

distribute the
0.30 on the
right side
subtract 0.30X
and 0.25Y from
both sides
divide both
sides by 0.10

261

GMAT® Official Guide 2018

Using this, the percent of the weight of the
combined mixture ( X + Y ) that is X is
X = 0.5Y = 0.5Y = 0.5 = 0.333 = 33 1 %
X + Y 0.5Y + Y 1.5Y 1.5
3
The correct answer is B.

1

(B)

2

(C)

3

(D)

4

(E)

5

Algebra Inequalities

– – – 0 + + + + + + + + + + + + + + + x+3
– – – – – 0 + + + + + + + + + + + + + x+2
– – – – – – – – – – – – – – 0 + + + + x–2
–3 –2

2

x

Pictorially, the number line above shows the
algebraic signs of the expressions (x + 3), (x + 2),
and (x − 2). For example, x + 3 is 0 when x = −3,
x + 3 is negative when x < −3, and x + 3 is positive
( x + 2)( x + 3)
when x > −3. The expression
x−2
will be positive in the intervals of the number
line where the number of minus signs is even.
( x + 2)( x + 3)
Therefore
is positive for values of
x−2
x such that −3 < x < −2 and for values of x such
that x > 2. The only integer values of x in these
intervals that are also less than 5 are 3 and 4.
( x + 2)( x + 3)
Also,
will be zero if and only if
x−2
(x + 2)(x + 3) = 0, which has two integer solutions
less than 5, namely, x = −2 and x = −3.
Therefore, there are four integers less than 5 that
( x + 2)( x + 3)
satisfy
≥ 0 and those integers are
x−2
−3, −2, 3, and 4.

262

( x + 2)( x + 3)
will be positive if
x−2
(x + 2)(x + 3) and x − 2 are both positive or both
negative, and for no other values of x. On the one
hand, (x + 2)(x + 3) will be positive when x + 2
and x + 3 are both positive, which will be the case
when x > −2 and x > −3 and thus when x > −2.
On the other hand, (x + 2)(x + 3) will be positive
when x + 2 and x + 3 are both negative, which
will be the case when x < −2 and x < −3 and thus
when x < −3. So, (x + 2)(x + 3) will be positive
when x < −3 or x > −2. (This result can also be
deduced from the fact that the graph of y =
(x + 2)(x + 3) is a parabola with x‐intercepts
(−2,0) and (−3,0) that opens upward.) Since
x − 2 will be positive when x > 2, it follows that
(x + 2)(x + 3) and x − 2 are both positive when
x > 2, which includes exactly two integer values
less than 5, namely, x = 3 and x = 4. There are
no integer values of x such that (x + 2)(x + 3)
and x − 2 are both negative, since (x + 2)(x + 3)
is negative if and only if x lies between −3 and
−2 and there are no integers between −3 and −2.
Therefore, there are exactly 4 integer values of x
( x + 2)( x + 3)
≥ 0.
less than 5 such that
x−2

Also,

228. How many of the integers that satisfy the inequality
( x + 2)( x + 3)
> 0 are less than 5 ?
x −2
(A)

( x + 2)( x + 3)
will be zero if and
x−2
only if (x + 2)(x + 3) = 0, which has two integer
solutions less than 5, namely, x = −2 and x = −3.

Alternatively,

Two of the values, x = −2 and x = −3, arise from
( x + 2)( x + 3)
= 0 , and two of the
x−2
values, x = 3 and x = 4, arise from solutions to

solutions to

( x + 2)( x + 3)
> 0.
x−2
The correct answer is D.

5.5 Problem Solving Answer Explanations

229. Of the 150 houses in a certain development,
60 percent have air-conditioning, 50 percent have a
sunporch, and 30 percent have a swimming pool. If
5 of the houses have all three of these amenities and
5 have none of them, how many of the houses have
exactly two of these amenities?

2−14 + 2−15 + 2−16 + 2−17
is how many
5
times the value of 2(−17) ?

230. The value of

(A)

3
2

(A)

10

(B)

5
2

(B)

45

(C)

3

(C)

50

(D)

4

(D)

55

(E)

5

(E)

65

Arithmetic Negative exponents

2−14 + 2−15 + 2−16 + 2−17
is x times
5
the value of 2−17 , then

Arithmetic Sets

If the value of

air-conditioning

−14
−15
−16
−17
x ( 2−17 ) = 2 + 2 + 2 + 2
5

c
b
a

sunporch

5

d

f

5
e

swimming
pool

Since 60% of the 150 houses have air-conditioning,
b + c + d + 5 = 0.6(150) = 90, so b + c + d = 85
(i). Similarly, since 50% have a sunporch,
a + b + f + 5 = 0.5(150) = 75, so a + b + f = 70
(ii). Likewise, since 30% have a swimming pool,
d + e + f + 5 = 0.3(150) = 45, so d + e + f = 40
(iii). Adding equations (i), (ii), and (iii) gives
(b + c + d) + (a + b + f) + (d + e + f) = 195,
or a + 2b + c + 2d + e + 2f = 195 (iv). But
a + b + c + d + e + f + 5 + 5 = 150, or
a + b + c + d + e + f = 140 (v). Subtracting
equation (v) from equation (iv) gives
b + d + f = 55, so 55 houses have exactly two of
the amenities.

2−14 + 2−15 + 2−16 + 2−17
5
x=
2−17
−14
−15
−16
−17
= 2 + 2 + 2 + 2 × 217
5

(2
=
=2

−14

)

+ 2−15 + 2−16 + 2−17 × 217

−14 + 17

5
+2

−15 + 17

+ 2−16 + 17 + 2−17 + 17
5

3
2
1
0
= 2 +2 +2 +2
5
= 8+ 4 + 2+1
5
=3

The correct answer is C.

The correct answer is D.

263

6.0 Data Sufficiency

264

6.0 Data Sufficiency

6.0 Data Sufficiency
Data sufficiency questions appear in the Quantitative section of the GMAT® exam. Multiple‐choice
data sufficiency questions are intermingled with problem solving questions throughout the section. You
will have 75 minutes to complete the Quantitative section of the GMAT exam, or about 2 minutes to
answer each question. These questions require knowledge of the following topics:
• Arithmetic
• Elementary algebra
• Commonly known concepts of geometry
Data sufficiency questions are designed to measure your ability to analyze a quantitative problem,
recognize which given information is relevant, and determine at what point there is sufficient
information to solve a problem. In these questions, you are to classify each problem according to the
five fixed answer choices, rather than find a solution to the problem.
Each data sufficiency question consists of a question, often accompanied by some initial information,
and two statements, labeled (1) and (2), which contain additional information. You must decide whether
the information in each statement is sufficient to answer the question or—if neither statement provides
enough information—whether the information in the two statements together is sufficient. It is also
possible that the statements, in combination, do not give enough information to answer the question.
Begin by reading the initial information and the question carefully. Next, consider the first statement.
Does the information provided by the first statement enable you to answer the question? Go on to the
second statement. Try to ignore the information given in the first statement when you consider whether
the second statement provides information that, by itself, allows you to answer the question. Now you
should be able to say, for each statement, whether it is sufficient to determine the answer.
Next, consider the two statements in tandem. Do they, together, enable you to answer the question?
Look again at your answer choices. Select the one that most accurately reflects whether the statements
provide the information required to answer the question.

265

GMAT® Official Guide 2018

6.1 Test-Taking Strategies
1. Do not waste valuable time solving a problem.
You only need to determine whether sufficient information is given to solve it.

2. Consider each statement separately.
First, decide whether each statement alone gives sufficient information to solve the problem. Be sure to
disregard the information given in statement (1) when you evaluate the information given in statement
(2). If either, or both, of the statements give(s) sufficient information to solve the problem, select the
answer corresponding to the description of which statement(s) give(s) sufficient information to solve
the problem.

3. Judge the statements in tandem if neither statement is sufficient by itself.
It is possible that the two statements together do not provide sufficient information. Once you decide,
select the answer corresponding to the description of whether the statements together give sufficient
information to solve the problem.

4. Answer the question asked.
For example, if the question asks, “What is the value of y?” for an answer statement to be sufficient,
you must be able to find one and only one value for y. Being able to determine minimum or maximum
values for an answer (e.g., y = x + 2) is not sufficient, because such answers constitute a range of values
rather than the specific value of y.

5. Be very careful not to make unwarranted assumptions based on the images
represented.
Figures are not necessarily drawn to scale; they are generalized figures showing little more than
intersecting line segments and the relationships of points, angles, and regions. For example, if a figure
described as a rectangle looks like a square, do not conclude that it is actually a square just by looking at
the figure.

266

6.1 Data Sufficiency Test-Taking Strategies

If statement 1 is sufficient, then the answer must be A or D.
If statement 2 is not sufficient, then the answer must be A.
If statement 2 is sufficient, then the answer must be D.
If statement 1 is not sufficient, then the answer must be B, C, or E.
If statement 2 is sufficient, then the answer must be B.
If statement 2 is not sufficient, then the answer must be C or E.
If both statements together are sufficient, then the answer must be C.
If both statements together are still not sufficient, then the answer must be E.
Is Statement 1 Sufficient Alone?
No

Yes

Is Statement 2 Sufficient Alone?
Yes

Correct
Answer
Is D

No

Correct
Answer
Is A

Is Statement 2 Sufficient Alone?
Yes

No

Correct
Answer
Is B

Are Statements 1 & 2
Sufficient Together?
Yes

Correct
Answer
Is C

No

Correct
Answer
Is E

267

GMAT® Official Guide 2018

6.2 The Directions
These directions are similar to those you will see for data sufficiency questions when you take the
GMAT exam. If you read the directions carefully and understand them clearly before going to sit for
the test, you will not need to spend much time reviewing them when you take the GMAT exam.
Each data sufficiency problem consists of a question and two statements, labeled (1) and (2), that
give data. You have to decide whether the data given in the statements are sufficient for answering the
question. Using the data given in the statements plus your knowledge of mathematics and everyday
facts (such as the number of days in July or the meaning of counterclockwise), you must indicate whether
the data given in the statements are sufficient for answering the questions and then indicate one of the
following answer choices:
(A)

Statement (1) ALONE is sufficient, but statement (2) alone is not sufficient to answer the
question asked;

(B)

Statement (2) ALONE is sufficient, but statement (1) alone is not sufficient to answer the
question asked;

(C)

BOTH statements (1) and (2) TOGETHER are sufficient to answer the question asked,
but NEITHER statement ALONE is sufficient;

(D)

EACH statement ALONE is sufficient to answer the question asked;

(E)

Statements (1) and (2) TOGETHER are NOT sufficient to answer the question asked,
and additional data are needed.

NOTE: In data sufficiency problems that ask for the value of a quantity, the data given in the
statements are sufficient only when it is possible to determine exactly one numerical value for
the quantity.
Numbers: All numbers used are real numbers.
Figures: A figure accompanying a data sufficiency problem will conform to the information given
in the question but will not necessarily conform to the additional information given in statements
(1) and (2).
Lines shown as straight can be assumed to be straight and lines that appear jagged can also be assumed
to be straight.
You may assume that the positions of points, angles, regions, and so forth exist in the order shown and
that angle measures are greater than zero degrees.
All figures lie in a plane unless otherwise indicated.

268

6.2 Data Sufficiency The Directions

To register for the GMAT exam go to www.mba.com
269

GMAT® Official Guide 2018

6.3 Practice Questions
Each data sufficiency problem consists of a question and two statements, labeled (1) and (2), which
contain certain data. Using these data and your knowledge of mathematics and everyday facts
(such as the number of days in July or the meaning of the word counterclockwise), decide whether
the data given are sufficient for answering the question and then indicate one of the following
answer choices:
A
B
C
D
E

Statement (1) ALONE is sufficient, but statement (2) alone is not sufficient.
Statement (2) ALONE is sufficient, but statement (1) alone is not sufficient.
BOTH statements TOGETHER are sufficient, but NEITHER statement ALONE is sufficient.
EACH statement ALONE is sufficient.
Statements (1) and (2) TOGETHER are NOT sufficient.

Note: In data sufficiency problems that ask for the value of a quantity, the data given in the statements
are sufficient only when it is possible to determine exactly one numerical value for the quantity.
Example:
P
xº
yº
Q

zº

R

In, ΔPQR what is the value of x ?
(1)

PQ = PR

(2)

y = 40

Explanation: According to statement (1) PQ = PR; therefore, ΔPQR is isosceles and y = z. Since
x + y + z = 180, it follows that x + 2y = 180. Since statement (1) does not give a value for y, you cannot
answer the question using statement (1) alone. According to statement (2), y = 40; therefore,
x + z = 140. Since statement (2) does not give a value for z, you cannot answer the question using
statement (2) alone. Using both statements together, since x + 2y = 180 and the value of y is given,
you can find the value of x. Therefore, BOTH statements (1) and (2) TOGETHER are sufficient to
answer the questions, but NEITHER statement ALONE is sufficient.
Numbers: All numbers used are real numbers.
Figures:
• Figures conform to the information given in the question, but will not necessarily conform to the
additional information given in statements (1) and (2).
• Lines shown as straight are straight, and lines that appear jagged are also straight.
• The positions of points, angles, regions, etc., exist in the order shown, and angle measures are greater
than zero.
• All figures lie in a plane unless otherwise indicated.

270

6.3 Data Sufficiency Practice Questions

B

y
Q

20°

P

R
x

O

C
D
A

231. In the figure above, point D is on AC. What is the
degree measure of ∠BAC ?
(1)

The measure of ∠BDC is 60°.

(2)

The degree measure of ∠BAC is less than the
degree measure of ∠BCD.

232. A citrus fruit grower receives $15 for each crate of
oranges shipped and $18 for each crate of grapefruit
shipped. How many crates of oranges did the grower
ship last week?
(1)

(2)

Points P and Q have the same x‐coordinate.

(2)

Points P and R have the same y‐coordinate.

236. Each of the marbles in a jar is either red or white
or blue. If one marble is to be selected at random
from the jar, what is the probability that the marble
will be blue?
(1)

There are a total of 24 marbles in the jar, 8 of
which are red.

(2)

The probability that the marble selected will be
1
white is .
2
y

Exactly 20 quarts of the mixture are needed.

(2)

Exactly 10 quarts of the yellow paint are available.

234. If Pat saved $600 of his earnings last month, how
much did Pat earn last month?
Pat spent 1 of his earnings last month for living
2
expenses and saved 1 of the remainder.
3
Of his earnings last month, Pat paid twice as
much in taxes as he saved.

w

x

Last week the grower received a total of
$38,700 from the crates of oranges and
grapefruit shipped.

(1)

(2)

(1)

Last week the number of crates of oranges that
the grower shipped was 20 more than twice the
number of crates of grapefruit shipped.

233. A certain mixture of paint requires blue, yellow, and
red paints in ratios of 2:3:1, respectively, and no other
ingredients. If there are ample quantities of the blue
and red paints available, is there enough of the yellow
paint available to make the desired amount of the
mixture?

(1)

235. In the xy‐plane above, is angle QPR a right angle?

z

237. In the figure above, what is the value of z ?
(1)

x=y=1

(2)

w=2

238. Water is pumped into a partially filled tank at a
constant rate through an inlet pipe. At the same time,
water is pumped out of the tank at a constant rate
through an outlet pipe. At what rate, in gallons per
minute, is the amount of water in the tank increasing?
(1)

The amount of water initially in the tank is
200 gallons.

(2)

Water is pumped into the tank at a rate of
10 gallons per minute and out of the tank at a
1
rate of 10 gallons every 2 minutes.
2

271

GMAT® Official Guide 2018

239. What is the value of 10 percent of y ?
(1)

5 percent of y is 60.

(1)

−6 < r < 5

(2)

y is 80 percent of 1,500.

(2)

6 < w < 10

240. What is the cube root of w ?
(1)

The 5th root of w is 64.

(2)

The 15th root of w is 4.

241. If sequence S has 120 terms, what is the 105th term
of S ?
(1)

The first term of S is –8.

(2)

Each term of S after the first term is 10 more
than the preceding term.

242. The number of seats in the first row of an auditorium
is 18 and the number of seats in each row thereafter
is 2 more than in the previous row. What is the total
number of seats in the rows of the auditorium?
(1)

The number of rows of seats in the auditorium
is 27.

(2)

The number of seats in the last row is 70.

243. If u > 0 and v > 0, which is greater, uv or vu ?
(1)

u=1

(2)

v>2

244. What was the range of the selling prices of the
30 wallets sold by a certain store yesterday?
(1)
(2)

1
of the wallets had a selling price of $24 each.
3
1
The lowest selling price of the wallets was the
3
highest selling price of the wallets.

245. Three houses are being sold through a real estate
agent. What is the asking price for the house with the
second‐largest asking price?
(1)
(2)

272

246. Is rw = 0 ?

The difference between the greatest and the
least asking price is $130,000.
The difference between the two greater asking
prices is $85,000.

247. Is x =
(1)
(2)

1
?
y
xy = 1
1
=1
xy

248. How many people in a group of 50 own neither a fax
machine nor a laser printer?
(1)

The total number of people in the group who own
a fax machine or a laser printer or both is less
than 50.

(2)

The total number of people in the group who own
both a fax machine and a laser printer is 15.

249. What is the value of w–2 ?
(1)
(2)

1
2
w3 = 8

w–1 =

250. A certain investment earned a fixed rate of 4 percent
interest per year, compounded annually, for five years.
The interest earned for the third year of the investment
was how many dollars greater than that for the first
year?
(1)

The amount of the investment at the beginning of
the second year was $4,160.00.

(2)

The amount of the investment at the beginning of
the third year was $4,326.40.

251. On a recent trip, Mary drove 50 miles. What was the
average speed at which she drove the 50 miles?
(1)

She drove 30 miles at an average speed of
60 miles per hour and then drove the remaining
20 miles at an average speed of 50 miles
per hour.

(2)

She drove a total of 54 minutes.

252. What is the value of t ?
(1)

s+t=6+s

(2)

t3 = 216

6.3 Data Sufficiency Practice Questions

253. For a certain car repair, the total charge consisted of
a charge for parts, a charge for labor, and a 6 percent
sales tax on both the charge for parts and the charge
for labor. If the charge for parts, excluding sales tax,
was $50.00, what was the total charge for the repair?
(1)

The sales tax on the charge for labor was $9.60.

(2)

The total sales tax was $12.60.

259. In each of the last five years, Company K donated
p percent of its annual profits to a certain scholarship
fund. Did Company K donate more than $10,000 to
the scholarship fund last year?
(1)

Two years ago, Company K had annual profits
of $3 million and donated $15,000 to the
scholarship fund.

(2)

Last year, Company K had annual profits of
$2.5 million.

254. George has a total of B books in his library, 25 of
which are hardcover fiction books. What is the
value of B ?
(1)

40 of the B books are fiction and the rest are
nonfiction.

(2)

60 of the B books are hardcovers and the rest
are paperbacks.

255. Is w + h4 positive?
(1)

h is positive.

(2)

w is positive.

256. In Mr. Smith’s class, what is the ratio of the number of
boys to the number of girls?
(1)
(2)

There are 3 times as many girls as boys in
Mr. Smith’s class.
1
The number of boys is of the total number of
4
boys and girls in Mr. Smith’s class.

257. If the sequence S has 300 terms, what is the 293rd
term of S ?
(1)

The 298th term of S is –616, and each
term of S after the first is 2 less than the
preceding term.

(2)

The first term of S is –22.

258. If a is a 3-digit integer and b is a 3-digit integer, is the
units digit of the product of a and b greater than 5 ?
(1)

The units digit of a is 4.

(2)

The units digit of b is 7.

x

z
y

260. Is the area of the triangular region above less than 20 ?
(1)

x2 + y2 ≠ z2

(2)

x + y < 13

261. A, B, C, and D are points on a line. If C is the midpoint
of line segment AB and if D is the midpoint of line
segment CB, is the length of line segment DB greater
than 5 ?
(1)

The length of line segment AC is greater than 8.

(2)

The length of line segment CD is greater than 6.

262. The people in a line waiting to buy tickets to a show
are standing one behind the other. Adam and Beth
are among the people in the line, and Beth is standing
behind Adam with a number of people between them. If
the number of people in front of Adam plus the number
of people behind Beth is 18, how many people in the
line are behind Beth?
(1)

There are a total of 32 people in the line.

(2)

23 people in the line are behind Adam.

263. Square ABCD is inscribed in circle O. What is the area
of square region ABCD ?
(1)

The area of circular region O is 64π.

(2)

The circumference of circle O is 16π.

264. Lines k and m are parallel to each other. Is the slope of
line k positive?
(1)

Line k passes through the point (3,2).

(2)

Line m passes through the point (–3,2).
273

GMAT® Official Guide 2018

265. In cross section, a tunnel that carries one lane of
one‐way traffic is a semicircle with radius 4.2 m. Is the
tunnel large enough to accommodate the truck that is
approaching the entrance to the tunnel?
(1)

The maximum width of the truck is 2.4 m.

(2)

The maximum height of the truck is 4 m.

266. In a certain group of 50 people, how many are doctors
who have a law degree?

271. If n is an integer, is n even?
(1)

n2 − 1 is an odd integer.

(2)

3n + 4 is an even integer.

272. Carmen currently works 30 hours per week at her
part‐time job. If her gross hourly wage were to
increase by $1.50, how many fewer hours could she
work per week and still earn the same gross weekly
pay as before the increase?

(1)

In the group, 36 people are doctors.

(1)

Her gross weekly pay is currently $225.00.

(2)

In the group, 18 people have a law degree.

(2)

An increase of $1.50 would represent an
increase of 20 percent of her current gross
hourly wage.

267. Of a group of 50 households, how many have at least
one cat or at least one dog, but not both?
(1)

The number of households that have at least one
cat and at least one dog is 4.

(2)

The number of households that have no cats and
no dogs is 14.

268. On a certain date, Hannah invested $5,000 at
x percent simple annual interest and a different amount
at y percent simple annual interest. What amount did
Hannah invest at y percent simple annual interest?
(1)
(2)

The total amount of interest earned by Hannah’s
two investments in one year was $900.
Hannah invested the $5,000 at 6 percent simple
annual interest.

269. Do at least 60 percent of the students in Pat’s class
walk to school?

(1)

The plumber charged $92 for one of the two
jobs.

(2)

The plumber charged $138 for one of the two
jobs.

274. If x and y are positive numbers, is
(1)

x>1

(2)

x ?
y +1 y

a 9
?
<
b 11

a
< 0.818
b
b
> 1.223
a

276. Every object in a box is either a sphere or a cube, and
every object in the box is either red or green. How
many objects are in the box?
(1)

There are six cubes and five green objects in
the box.

(2)

There are two red spheres in the box.

6.3 Data Sufficiency Practice Questions

277. If x and y are positive integers, is xy even?
(1)

x2 + y2 − 1 is divisible by 4.

(2)

x + y is odd.

278. If a and b are integers, is a + b + 3 an odd integer?
(1)

ab is an odd integer.

(2)

a − b is an even integer.

279. If x and y are positive integers, what is the value of
x+ y ?
(1)

x + y = 15

(2)

xy = 6

1
+ kv 2 gallons of fuel per mile
12
when its speed is v miles per hour, where k is a
constant. At what speed should the truck travel so that
5
it uses
gallon of fuel per mile?
12
1
.
(1) The value of k is
10,800

280. A certain truck uses

(2)

When the truck travels at 30 miles per hour, it
1
uses gallon of fuel per mile.
6

t
281. If x = 2t and y = , what is the value of x2 − y2 ?
3
(1)

t −3=6

(2)

t3 = –27

2

282. Last school year, each of the 200 students at a certain
high school attended the school for the entire year.
If there were 8 cultural performances at the school
during the last school year, what was the average
(arithmetic mean) number of students attending each
cultural performance?
(1)

Last school year, each student attended at least
one cultural performance.

(2)

Last school year, the average number of cultural
performances attended per student was 4.

283. A clothing manufacturer makes jackets that are
wool or cotton or a combination of wool and cotton.
The manufacturer has 3,000 pounds of wool and
2,000 pounds of cotton on hand. Is this enough wool
and cotton to make at least 1,000 jackets?
(1)

Each wool jacket requires 4 pounds of wool, and
no cotton.

(2)

Each cotton jacket requires 6 pounds of cotton,
and no wool.

284. Each month, Jim receives a base salary plus a
10 percent commission on the price of each car he
sells that month. If Jim sold 15 cars last month, what
was the total amount of base salary and commissions
that Jim received that month?
(1)

Last month, Jim’s base salary was $3,000.

(2)

Last month, Jim sold 3 cars whose prices
totaled $60,000 and 5 cars whose prices
totaled $120,000.

285. If x is a positive integer greater than 1, what is the
value of x ?
(1)

2x is a common factor of 18 and 24.

(2)

x is a factor of 6.

286. By what percentage was the price of a certain
television set discounted for a sale?
(1)

The price of the television set before it was
discounted for the sale was 25 percent greater
than the discounted price.

(2)

The price of the television set was discounted by
$60 for the sale.

287. Jack wants to use a circular rug on his rectangular
office floor to cover two small circular stains, each
π
less than
square feet in area and each more than
100
3 feet from the nearest wall. Can the rug be placed to
cover both stains?
(1)

Jack’s rug covers an area of 9π square feet.

(2)

The centers of the stains are less than 4 feet apart.

275

GMAT® Official Guide 2018

292. If r and t are three-digit positive integers, is r greater
than t ?

×

a

b

c

a

d

e

f

(1)

The tens digit of r is greater than each of the
three digits of t.

b

e

g

h

(2)

The tens digit of r is less than either of the other
two digits of r.

c

f

h

j

288. In the multiplication table above, each letter represents
an integer. What is the value of c ?
(1)

c=f

(2)

h≠0

289. If n is an integer, is
(1)

n > –10

(2)

n < 10

(0.1)n

n

greater than (10) ?

290. For a basic monthly fee of F yen (¥F), Naoko’s first
cell phone plan allowed him to use a maximum of
420 minutes on calls during the month. Then, for
each of x additional minutes he used on calls, he was
charged ¥M, making his total charge for the month ¥T,
where T = F + xM. What is the value of F ?
(1)

Naoko used 450 minutes on calls the first month
and the total charge for the month was ¥13,755.

(2)

Naoko used 400 minutes on calls the second
month and the total charge for the month was
¥13,125.

291. The weights of all dishes of type X are exactly the
same, and the weights of all dishes of type Y are
exactly the same. Is the weight of 1 dish of type X less
than the weight of 1 dish of type Y ?

276

(1)

The total weight of 3 dishes of type X and
2 dishes of type Y is less than the total weight of
2 dishes of type X and 4 dishes of type Y.

(2)

The total weight of 4 dishes of type X and
3 dishes of type Y is less than the total weight of
3 dishes of type X and 4 dishes of type Y.

293. Is the product of two positive integers x and y divisible
by the sum of x and y ?
(1)

x=y

(2)

x=2

294. A certain high school with a total enrollment of
900 students held a science fair for three days last
week. How many of the students enrolled in the high
school attended the science fair on all three days?
(1)

Of the students enrolled in the school, 30 percent
attended the science fair on two or more days.

(2)

Of the students enrolled in the school, 10 percent
of those that attended the science fair on at
least one day attended on all three days.

295. The average (arithmetic mean) price of the 3 items that
Kate purchased from a clothing store was $50. If there
was no sales tax on any item that had a price of less than
$80 and 6 percent sales tax on all other items, what was
the total sales tax on the 3 items that Kate purchased?
(1)

The price of the most expensive item that
Kate purchased from the store was $100.

(2)

The price of the least expensive item that
Kate purchased from the store was $10.

296. A company makes and sells two products, P and Q.
The costs per unit of making and selling P and Q
are $8.00 and $9.50, respectively, and the selling
prices per unit of P and Q are $10.00 and $13.00,
respectively. In one month the company sold a total
of 834 units of these products. Was the total profit on
these items more than $2,000.00 ?
(1)

During the month, more units of P than units of Q
were sold.

(2)

During the month, at least 100 units of Q were sold.

6.3 Data Sufficiency Practice Questions

297. Jill has applied for a job with each of two different
companies. What is the probability that she will get job
offers from both companies?
(1)

The probability that she will get a job offer from
neither company is 0.3.

(2)

The probability that she will get a job offer from
exactly one of the two companies is 0.5.

302. Terry holds 12 cards, each of which is red, white,
green, or blue. If a person is to select a card randomly
from the cards Terry is holding, is the probability less
1
than that the card selected will be either red or white?
2
(1)

(2)
298. A scientist recorded the number of eggs in each of
10 birds’ nests. What was the standard deviation of the
numbers of eggs in the 10 nests?
(1)

The average (arithmetic mean) number of eggs
for the 10 nests was 4.

(2)

Each of the 10 nests contained the same
number of eggs.

299. A conveyor belt moves bottles at a constant speed of
120 centimeters per second. If the conveyor belt moves
a bottle from a loading dock to an unloading dock, is
the distance that the conveyor belt moves the bottle
less than 90 meters? (1 meter = 100 centimeters)
(1)

It takes the conveyor belt less than 1.2 minutes
to move the bottle from the loading dock to the
unloading dock.

(2)

It takes the conveyor belt more than 1.1 minutes
to move the bottle from the loading dock to the
unloading dock.

300. If x, y, and z are positive numbers, what is the value of
the average (arithmetic mean) of x and z ?
(1)

x−y=y−z

(2)

x 2 − y2 = z

The probability that the person will select a blue
1
card is .
3
The probability that the person will select a red
1
card is .
6

303. If y ≠ 2xz, what is the value of
(1)

2x + y = 3

(2)

z=2
y°

2xz + yz
?
2xz − y

x°

z°
304. In the parallelogram shown, what is the value of x ?
(1)

y = 2x

(2)

x + z = 120

305. In a product test of a common cold remedy, x percent
of the patients tested experienced side effects from
the use of the drug and y percent experienced relief
of cold symptoms. What percent of the patients
tested experienced both side effects and relief of cold
symptoms?
(1)

Of the 1,000 patients tested, 15 percent
experienced neither side effects nor relief of
cold symptoms.

(2)

Of the patients tested, 30 percent experienced
relief of cold symptoms without side effects.

306. Is x < 5 ?
301. The rectangular rug shown in the figure above has an
accent border. What is the area of the portion of the
rug that excludes the border?
(1)

The perimeter of the rug is 44 feet.

(2)

The width of the border on all sides is 1 foot.

(1)

x2 > 5

(2)

x2 + x < 5

307. Is 4x + y = 810 ?
(1)

x−y=9

(2)

y 1
=
x 4

277

GMAT® Official Guide 2018

308. Is zp negative?
(1)

pz 4 < 0

(2)

p + z 4 = 14

309. In each game of a certain tournament, a contestant
either loses 3 points or gains 2 points. If Pat had
100 points at the beginning of the tournament, how
many games did Pat play in the tournament?
(1)

At the end of the tournament, Pat had
104 points.

(2)

Pat played fewer than 10 games.

310. At the beginning of the year, the Finance Committee
and the Planning Committee of a certain company
each had n members, and no one was a member of
both committees. At the end of the year, 5 members
left the Finance Committee and 3 members left the
Planning Committee. How many members did the
Finance Committee have at the beginning of the year?
(1)

(2)

The ratio of the total number of members who
left at the end of the year to the total number of
members at the beginning of the year was 1:6.
At the end of the year, 21 members remained on
the Planning Committee.

311. Can a certain rectangular sheet of glass be positioned
on a rectangular tabletop so that it covers the entire
tabletop and its edges are parallel to the edges of the
tabletop?
(1)

The tabletop is 36 inches wide by 60 inches
long.

(2)

The area of one side of the sheet of glass is
2,400 square inches.

312. In a random sample of 80 adults, how many are
college graduates?

278

(1)

In the sample, the number of adults who are not
college graduates is 3 times the number who are
college graduates.

(2)

In the sample, the number of adults who are not
college graduates is 40 more than the number
who are college graduates.

313. Max purchased a guitar for a total of $624, which
consisted of the price of the guitar and the sales tax.
Was the sales tax rate greater than 3 percent?
(1)

The price of the guitar that Max purchased was
less than $602.

(2)

The sales tax for the guitar that Max purchased
was less than $30.

314. What is the sum of a certain pair of consecutive odd
integers?
(1)

At least one of the integers is negative.

(2)

At least one of the integers is positive.

315. If a < x < b and c < y < d, is x < y ?
(1)

a 0 and s > 0, is
(1)
(2)

r
1
=
3s 4
s=r+4

356. If k is an integer such that 56 < k < 66, what is the
value of k ?
(1)

If k were divided by 2, the remainder would be 1.

(2)

If k + 1 were divided by 3, the remainder would
be 0.

357. If x is a positive integer, then is x prime?

282

361. If a, b, c, and d are positive numbers, is

3x + 1 is prime.

(2)

5x + 1 is prime.

c−a
d −b

(1)

0<

(2)

ad
⎛ ad ⎞
⎜⎝ ⎟⎠ <
bc
bc

a c
< ?
b d

2

r s
< ?
s r

(1)

x+y
<4
2
2 y ?
(1)

x+y>0

(2)

yx < 0

O
P

366. If r and s are the roots of the equation x2 + bx + c = 0,
where b and c are constants, is rs < 0 ?
(1)

b<0

(2)

c<0
k ft

15 ft
k ft
15 ft
367. The figure above represents an L‐shaped garden. What
is the value of k ?
(1)

The area of the garden is 189 square feet.

(2)

The perimeter of the garden is 60 feet.

368. The only articles of clothing in a certain closet are
shirts, dresses, and jackets. The ratio of the number
of shirts to the number of dresses to the number of
jackets in the closet is 9:4:5, respectively. If there are
more than 7 dresses in the closet, what is the total
number of articles of clothing in the closet?
(1)
(2)

R
x°

Q

T

369. The figure above represents a circle graph of
Company H’s total expenses broken down by the
expenses for each of its five divisions. If O is the center
of the circle and if Company H’s total expenses are
$5,400,000, what are the expenses for Division R ?
(1)

x = 94

(2)

The total expenses for Divisions S and T are
twice as much as the expenses for Division R.

370. If x is negative, is x < –3 ?
(1)

x2 > 9

(2)

x3 < –9

371. What is the number of cans that can be packed in a
certain carton?
(1)

The interior volume of this carton is 2,304 cubic
inches.

(2)

The exterior of each can is 6 inches high and has
a diameter of 4 inches.

The total number of shirts and jackets in the
closet is less than 30.
The total number of shirts and dresses in the
closet is 26.

S

r

s

t

u

v

w

x

y

z

372. Each of the letters in the table above represents one
of the numbers 1, 2, or 3, and each of these numbers
occurs exactly once in each row and exactly once in
each column. What is the value of r ?
(1)

v+z=6

(2)

s+t+u+x=6

283

GMAT® Official Guide 2018

373. Material A costs $3 per kilogram, and Material B
costs $5 per kilogram. If 10 kilograms of Material K
consists of x kilograms of Material A and y kilograms
of Material B, is x > y ?
(1)

y>4

(2)

The cost of the 10 kilograms of Material K is
less than $40.

374. At what speed was a train traveling on a trip when it
had completed half of the total distance of the trip?
(1)

The trip was 460 miles long and took 4 hours to
complete.

(2)

The train traveled at an average rate of
115 miles per hour on the trip.

375. Tom, Jane, and Sue each purchased a new house. The
average (arithmetic mean) price of the three houses
was $120,000. What was the median price of the
three houses?
(1)

The price of Tom’s house was $110,000.

(2)

The price of Jane’s house was $120,000.

376. What is the value of x if x3 < x2 ?
(1)

–2 < x < 2

(2)

x is an integer greater than –2.

377. For any integers x and y, min(x, y) and max(x, y)
denote the minimum and the maximum of x and y,
respectively. For example, min(5, 2) = 2 and
max(5, 2) = 5. For the integer w, what is the value
of min(10, w) ?
(1)

w = max(20, z) for some integer z

(2)

w = max(10, w)

378. A certain bookcase has 2 shelves of books. On the
upper shelf, the book with the greatest number of
pages has 400 pages. On the lower shelf, the book
with the least number of pages has 475 pages. What
is the median number of pages for all of the books on
the 2 shelves?

284

(1)

There are 25 books on the upper shelf.

(2)

There are 24 books on the lower shelf.

A

B C D

E

379. In the figure above, points A, B, C, D, and E lie on a
line. A is on both circles, B is the center of the smaller
circle, C is the center of the larger circle, D is on the
smaller circle, and E is on the larger circle. What is the
area of the region inside the larger circle and outside
the smaller circle?
(1)

AB = 3 and BC = 2

(2)

CD = 1 and DE = 4

380. In planning for a car trip, Joan estimated both the
distance of the trip, in miles, and her average speed,
in miles per hour. She accurately divided her estimated
distance by her estimated average speed to obtain an
estimate for the time, in hours, that the trip would take.
Was her estimate within 0.5 hour of the actual time
that the trip took?
(1)

Joan’s estimate for the distance was within
5 miles of the actual distance.

(2)

Joan’s estimate for her average speed was within
10 miles per hour of her actual average speed.

381. A certain list consists of 3 different numbers. Does the
median of the 3 numbers equal the average (arithmetic
mean) of the 3 numbers?
(1)

The range of the 3 numbers is equal to twice the
difference between the greatest number and the
median.

(2)

The sum of the 3 numbers is equal to 3 times
one of the numbers.

382. Line ℓ lies in the xy-plane and does not pass through
the origin. What is the slope of line ℓ ?
(1)

The x-intercept of line ℓ is twice the y-intercept
of line ℓ.

(2)

The x- and y-intercepts of line ℓ are both positive.

6.3 Data Sufficiency Practice Questions

y = ax − 5

Shipment

y = x+6
y = 3x + b
383. In the xy‐plane, the straight‐line graphs of the three
equations above each contain the point (p,r). If a and b
are constants, what is the value of b ?
(1)

a=2

(2)

r = 17

T
Q
W
U

V
S
P

R

384. In the figure above, PQR and STU are identical
equilateral triangles, and PQ = 6. What is the
perimeter of polygon PQWTUVR ?
(1)

Triangle SWV has perimeter 9.

(2)

VW has length 3.5.

Fraction of the
Total Value of the
Six Shipments

S1

S2

S3

S4

S5

S6

1
4

1
5

1
6

3
20

2
15

1
10

387. Six shipments of machine parts were shipped from a
factory on two trucks, with each shipment entirely on
one of the trucks. Each shipment was labeled either
S1, S2, S3, S4, S5, or S6. The table shows the value
of each shipment as a fraction of the total value of
the six shipments. If the shipments on the first truck
1
had a value greater than of the total value of the
2
six shipments, was S3 shipped on the first truck?
(1)

S2 and S4 were shipped on the first truck.

(2)

S1 and S6 were shipped on the second truck.

388. If x, y, and z are three‐digit positive integers and if
x = y + z, is the hundreds digit of x equal to the sum
of the hundreds digits of y and z ?
(1)

The tens digit of x is equal to the sum of the
tens digits of y and z.

(2)

The units digit of x is equal to the sum of the
units digits of y and z.
Favorable Unfavorable Not Sure

385. The range of the numbers in set S is x, and the range
of the numbers in set T is y. If all of the numbers in set
T are also in set S, is x greater than y ?
(1)

Set S consists of 7 numbers.

(2)

Set T consists of 6 numbers.

386. The hypotenuse of a right triangle is 10 cm. What is
the perimeter, in centimeters, of the triangle?
(1)

The area of the triangle is 25 square
centimeters.

(2)

The 2 legs of the triangle are of equal length.

Candidate M

40

20

40

Candidate N

30

35

35

389. The table above shows the results of a survey of
100 voters who each responded “Favorable” or
“Unfavorable” or “Not Sure” when asked about their
impressions of Candidate M and of Candidate N.
What was the number of voters who responded
“Favorable” for both candidates?
(1)

The number of voters who did not respond
“Favorable” for either candidate was 40.

(2)

The number of voters who responded
“Unfavorable” for both candidates was 10.

285

GMAT® Official Guide 2018

390. A school administrator will assign each student in a group
of n students to one of m classrooms. If 3 < m < 13 < n,
is it possible to assign each of the n students to one of
the m classrooms so that each classroom has the same
number of students assigned to it?
(1)

It is possible to assign each of 3n students to
one of m classrooms so that each classroom
has the same number of students assigned to it.

(2)

It is possible to assign each of 13n students to
one of m classrooms so that each classroom
has the same number of students assigned to it.

391. If q, s, and t are all different numbers, is q < s < t ?
(1)

t − q = |t − s| + |s − q|

(2)

t>q

392. What is the median number of employees assigned per
project for the projects at Company Z ?
(1)

25 percent of the projects at Company Z have
4 or more employees assigned to each project.

(2)

35 percent of the projects at Company Z have
2 or fewer employees assigned to each project.

(1)

Last year the closing balance of Carl’s account
for April was less than $2,625.

(2)

Last year the closing balance of Carl’s account
for June was less than $2,675.

395. Are all of the numbers in a certain list of 15 numbers
equal?
(1)

The sum of all the numbers in the list is 60.

(2)

The sum of any 3 numbers in the list is 12.

396. If the average (arithmetic mean) of six numbers is 75,
how many of the numbers are equal to 75 ?
(1)

None of the six numbers is less than 75.

(2)

None of the six numbers is greater than 75.

397. What amount did Jean earn from the commission on
her sales in the first half of 1988 ?
(1)

In 1988 Jean’s commission was 5 percent of the
total amount of her sales.

(2)

The amount of Jean’s sales in the second half of
1988 averaged $10,000 per month more than in
the first half.

393. Last year, a certain company began manufacturing
product X and sold every unit of product X that it
produced. Last year the company’s total expenses
for manufacturing product X were equal to $100,000
plus 5 percent of the company’s total revenue from all
units of product X sold. If the company made a profit
on product X last year, did the company sell more than
21,000 units of product X last year?
(1)

The company’s total revenue from the sale of
product X last year was greater than $110,000.

(2)

For each unit of product X sold last year, the
company’s revenue was $5.

394. Beginning in January of last year, Carl made deposits
of $120 into his account on the 15th of each month
for several consecutive months and then made
withdrawals of $50 from the account on the 15th
of each of the remaining months of last year. There
were no other transactions in the account last year. If
the closing balance of Carl’s account for May of last
year was $2,600, what was the range of the monthly
closing balances of Carl’s account last year?

286

A

D

B

C

398. In the figure above, if the area of triangular region D is 4,
what is the length of a side of square region A ?
(1)
(2)

The area of square region B is 9.
64
The area of square region C is
.
9

6.3 Data Sufficiency Practice Questions

399. If n is a positive integer and k = 5.1 × 10n, what is the
value of k ?
(1)

6,000 < k < 500,000

(2)

k2 = 2.601 × 109

y˚

x˚
w˚

402. In a certain business, production index p is directly
proportional to efficiency index e, which is in turn directly
proportional to investment index i. What is p if i = 70 ?
(1)

e = 0.5 whenever i = 60.

(2)

p = 2.0 whenever i = 50.

403. If n is a positive integer, what is the tens digit of n ?
(1)

The hundreds digit of 10n is 6.

(2)

The tens digit of n + 1 is 7.

z˚

404. What is the value of
400. What is the value of x + y in the figure above?
(1)

w = 95

(2)

z = 125

(1)

2t
=3
t−x

(2)

t−x=5

2t + t − x
?
t−x

401. If n and k are positive integers, is n + k > 2 n ?
(1)

k > 3n

(2)

n + k > 3n

287

GMAT® Official Guide 2018

6.4 Answer Key
231.

A

266.

E

301.

E

336.

B

371.

E

232.

C

267.

C

302.

E

337.

C

372.

D

233.

C

268.

E

303.

E

338.

A

373.

B

234.

A

269.

B

304.

D

339.

B

374.

E

235.

C

270.

B

305.

E

340.

B

375.

B

236.

C

271.

D

306.

B

341.

A

376.

B

237.

C

272.

D

307.

C

342.

E

377.

D

238.

B

273.

B

308.

E

343.

D

378.

C

239.

D

274.

B

309.

E

344.

A

379.

D

240.

D

275.

D

310.

D

345.

C

380.

E

241.

C

276.

E

311.

E

346.

A

381.

D

242.

D

277.

D

312.

D

347.

A

382.

A

243.

C

278.

D

313.

A

348.

E

383.

D

244.

E

279.

C

314.

C

349.

D

384.

A

245.

E

280.

D

315.

B

350.

B

385.

E

246.

E

281.

D

316.

C

351.

D

386.

D

247.

D

282.

B

317.

D

352.

A

387.

B

248.

E

283.

C

318.

C

353.

C

388.

A

249.

D

284.

E

319.

C

354.

A

389.

A

250.

D

285.

A

320.

D

355.

D

390.

B

251.

D

286.

A

321.

E

356.

E

391.

A

252.

D

287.

C

322.

C

357.

E

392.

C

253.

D

288.

E

323.

D

358.

C

393.

B

254.

E

289.

E

324.

E

359.

D

394.

C

255.

B

290.

B

325.

E

360.

B

395.

B

256.

D

291.

B

326.

D

361.

B

396.

D

257.

A

292.

C

327.

E

362.

C

397.

E

258.

C

293.

C

328.

A

363.

D

398.

D

259.

C

294.

E

329.

C

364.

C

399.

D

260.

E

295.

A

330.

C

365.

C

400.

C

261.

B

296.

E

331.

C

366.

B

401.

A

262.

C

297.

C

332.

C

367.

A

402.

B

263.

D

298.

B

333.

B

368.

D

403.

A

264.

E

299.

A

334.

D

369.

A

404.

A

265.

C

300.

E

335.

A

370.

A

288

3.2 Diagnostic Test Verbal Questions

6.5 Answer Explanations
The following discussion of data sufficiency is intended to familiarize you with the most efficient and
effective approaches to the kinds of problems common to data sufficiency. The particular questions in
this chapter are generally representative of the kinds of data sufficiency questions you will encounter
on the GMAT exam. Remember that it is the problem solving strategy that is important, not the specific
details of a particular question.

B

232. A citrus fruit grower receives $15 for each crate of
oranges shipped and $18 for each crate of grapefruit
shipped. How many crates of oranges did the grower
ship last week?

20°

C

(1)

Last week the number of crates of oranges that
the grower shipped was 20 more than twice the
number of crates of grapefruit shipped.

(2)

Last week the grower received a total of
$38,700 from the crates of oranges and
grapefruit shipped.

D
A

231. In the figure above, point D is on AC . What is the
degree measure of ∠BAC ?
(1)

The measure of ∠BDC is 60°.

(2)

The degree measure of ∠BAC is less than the
degree measure of ∠BCD.

Algebra Simultaneous equations

If x represents the number of crates of oranges
and y represents the number of crates of
grapefruit, find a unique value for x.
(1)

Given that the measure of ∠BDC is 60°,
it follows that the measure of ∠BDA
is 180° − 60° = 120°. Therefore, since
the measures of the angles in a triangle
add to 180°, the measure of ∠BAC is
180 − (20° + 120°) = 40°; SUFFICIENT.

Translating from words into symbols gives
x = 2y + 20, but there is no information
about y and no way to find a unique value
for x from this equation. For example, if
y = 10, then x = 40, but if y = 100, then
x = 220; NOT sufficient.

(2)

Given that the degree measure of ∠BAC is less
than the degree measure of ∠BCD, when
the line segment AC rotates a few degrees
counterclockwise about point A and point D
stays on AC, then the degree measure of
∠BAC decreases (and the degree measure of
∠BCD increases, and the degree measure of
∠ABD remains 20°). Therefore, the degree
measure of ∠BAC can vary; NOT sufficient.

Translating from words to symbols gives
15x + 18y = 38,700, but there is no way to
find a unique value for x from this equation.
For example, if y = 2,150, then x = 0 and if
y = 0, then x = 2,580; NOT sufficient.

Taking (1) and (2) together gives a system of two
equations in two unknowns. Substituting the
equation from (1) into the equation from (2) gives
a single equation in the variable y. This equation
can be solved for a unique value of y from which a
unique value of x can be determined.

Geometry Angles

(1)

(2)

The correct answer is A;
statement 1 alone is sufficient.

The correct answer is C;
both statements together are sufficient.
289

GMAT® Official Guide 2018

233. A certain mixture of paint requires blue, yellow, and
red paints in ratios of 2:3:1, respectively, and no other
ingredients. If there are ample quantities of the blue and
red paints available, is there enough of the yellow paint
available to make the desired amount of the mixture?
(1)

Exactly 20 quarts of the mixture are needed.

(2)

Exactly 10 quarts of the yellow paint are available.

Arithmetic Ratios

Given that the mixture requires blue paint, yellow
paint, and red paint in the ratios 2:3:1, it follows
3
= 1 of the mixture will be yellow
that
2 + 3+1
2
paint. Determining whether there is enough yellow
paint available depends on how much of the mixture
is needed and how much yellow paint is available.
(1)

(2)

This indicates that exactly 20 quarts of the
1
paint mixture are needed, so (20) = 10 quarts
2
of yellow paint are needed. However, there
is no information about how much yellow
paint is available; NOT sufficient.
This indicates that exactly 10 quarts of
yellow paint are available, but there is no
information about how much of the mixture
or how much yellow paint is needed; NOT
sufficient.

Taking (1) and (2) together, 10 quarts of yellow
paint are needed and 10 quarts are available.

what remained, so Pat saved 1 ⎛ 1 E ⎞ = 1 E .
3⎝ 2 ⎠ 6
But Pat saved $600, so 600 = 1 E and this
6
gives a unique value for E; SUFFICIENT.
(2)

The correct answer is A;
statement 1 alone is sufficient.
y
Q

P

(2)

1
of his earnings last month for living
2
1
expenses and saved of the remainder.
3
Of his earnings last month, Pat paid twice as
much in taxes as he saved.

Arithmetic Operations with rational numbers

Let E be Pat’s earnings last month. Find a unique
value for E.
(1)

290

Pat spent 1 E for living expenses and so
2
1
1
E − E = 1 E remained. Pat saved of
3
2
2

x

235. In the xy‐plane above, is angle QPR a right angle?
(1)

Points P and Q have the same x‐coordinate.

(2)

Points P and R have the same y‐coordinate.

Geometry Simple coordinate geometry

(1)

Because points P and Q have the same
x‐coordinate, it follows that side PQ is
vertical (i.e., parallel to the y‐axis). Therefore,
angle QPR will be a right angle if and only if
side PR is horizontal (i.e., parallel to the
x‐axis). However, depending on the location
of point R in relation to point P, side PR may
or may not be horizontal; NOT sufficient.

(2)

Because points P and R have the same
y‐coordinate, it follows that side PR is
horizontal. Therefore, angle QPR will be a
right angle if and only if side PQ is vertical.
However, depending on the location of
point Q in relation to point P, side PQ may
or may not be vertical; NOT sufficient.

234. If Pat saved $600 of his earnings last month, how
much did Pat earn last month?
Pat spent

R

O

The correct answer is C;
both statements together are sufficient.

(1)

Pat saved $600 last month and paid 2($600)
in taxes, but there is no way to determine
Pat’s earnings last month; NOT sufficient.

Given (1) and (2), it follows that PQ is vertical
and PR is horizontal. Therefore, angle QPR is a
right angle.
The correct answer is C;
both statements together are sufficient.

6.5 Data Sufficiency Answer Explanations

236. Each of the marbles in a jar is either red or white or
blue. If one marble is to be selected at random from
the jar, what is the probability that the marble will be
blue?
(1)

There are a total of 24 marbles in the jar,
8 of which are red.

(2)

The probability that the marble selected will be
1
white is .
2

Arithmetic Probability

This problem can be solved by determining the
number of marbles in the jar and the number that
are blue.
(1)

(2)

This indicates that there are 24 marbles in
the jar and 8 of them are red. The number
of marbles that are blue is not known; NOT
sufficient.
This indicates neither the number of
marbles in the jar nor the number that are
blue; NOT sufficient.

Taking (1) and (2) together, there are 24 marbles
in the jar, 8 of which are red and 1 (24) = 12 of
2
which are white. Then, the number of blue
marbles is 24 − 8 − 12 = 4, and the probability
that one marble selected at random will be blue is
4 = 1.
24 6
The correct answer is C;
both statements together are sufficient.
y
w

x

(2)

Taking (1) and (2) together, then z = y + (z − y) =
1 + (z − 1), and the value of z − 1 can be
determined by applying the Pythagorean theorem
to the triangle that has hypotenuse of length
w = 2 and sides of lengths x = 1 and z − 1.
Specifically, the Pythagorean theorem gives
12 + (z − 1)2 = 22, or 1 + (z − 1)2 = 4, and solving
this equation gives (z − 1)2 = 3, or z − 1 = 3, or
z = 1 + 3.
The correct answer is C;
both statements together are sufficient.
238. Water is pumped into a partially filled tank at a
constant rate through an inlet pipe. At the same time,
water is pumped out of the tank at a constant rate
through an outlet pipe. At what rate, in gallons per
minute, is the amount of water in the tank increasing?
(1)

The amount of water initially in the tank is
200 gallons.

(2)

Water is pumped into the tank at a rate of
10 gallons per minute and out of the tank at a
1
rate of 10 gallons every 2 minutes.
2

Arithmetic Work Problem

If both the rate of the water being pumped into
the tank and the rate of the water being pumped
out of the tank are known, then the rate at which
the total amount of water in the tank is changing
can be determined, but not if only one of these
quantities is known.
(1)

This only gives the amount of water in the
tank initially; NOT sufficient.

(2)

This information provides both the needed
rates. Since the water is being pumped out
of the tank at the rate of 10 gallons every
2 1 minutes, that is, 4 gallons every minute,
2
and since 10 gallons are pumped into the tank
every minute, the rate at which the water is
increasing in the tank is 10 − 4 = 6 gallons
per minute; SUFFICIENT.

z

237. In the figure above, what is the value of z ?
(1)

x=y=1

(2)

w=2

Geometry Triangles; Pythagorean theorem

(1)

Given that x = y = 1, the length of the base
of the triangle to the right, which equals
z − y = z − 1, can vary, and thus the value of
z can vary; NOT sufficient.

Given that w = 2, all dimensions of the
rectangle to the left can vary, and thus the
value of z can vary; NOT sufficient.

The correct answer is B;
statement 2 alone is sufficient.
291

GMAT® Official Guide 2018

239. What is the value of 10 percent of y ?
(1)

5 percent of y is 60.

(2)

y is 80 percent of 1,500.

241. If sequence S has 120 terms, what is the 105th
term of S ?
(1)

The first term of S is –8.

(2)

Each term of S after the first term is 10 more
than the preceding term.

Arithmetic Percents

This problem can be solved by determining the
value of y.
(1)

(2)

Arithmetic Sequences

This problem can be solved by determining at
least one term of the sequence and how each term
is derived from the preceding term(s).

If 5 percent of y is 60, then 0.05y = 60 and
y = 60 = 1, 200; SUFFICIENT.
0.05
If y is 80 percent of 1,500, then
y = (0.80)(1,500) = 1,200; SUFFICIENT.

The correct answer is D;
each statement alone is sufficient.

(1)

This indicates that the first term of the
sequence is –8, but does not indicate
how subsequent terms are derived;
NOT sufficient.

(2)

This indicates how each term is derived
from the preceding term, but does not
indicate at least one term of the sequence;
NOT sufficient.

240. What is the cube root of w ?
(1)

The 5th root of w is 64.

(2)

The 15th root of w is 4.

Arithmetic Operations with radical expressions

Determine the cube root of w, or determine the
1

value of w 3 .
(1)

Taking (1) and (2) together, the first term is –8
and each subsequent term is 10 more than the
preceding term from which the 105th term can
be determined.
The correct answer is C;
both statements together are sufficient.

Given the 5th root of w is 64,
5

5
⎛ 1⎞3
then w = 64 and ⎜ w 5 ⎟ = 64 3.
⎝ ⎠
5
⎛ 1⎞ ⎛ 5⎞
1
⎜⎝ ⎟⎠ ⎜⎝ ⎟⎠
⎛ 15 ⎞ 3
Since ⎜ w ⎟ = w 5 3 = w 3 ,
⎝ ⎠
1
5

242. The number of seats in the first row of an auditorium
is 18 and the number of seats in each row thereafter
is 2 more than in the previous row. What is the total
number of seats in the rows of the auditorium?

5

1
⎛ 5⎞ ⎛ 1⎞
then w 3 = ⎜ 64 3 ⎟ = ⎜ 64 3 ⎟ = 4 5;
⎠
⎝
⎠ ⎝

SUFFICIENT.
(2)

Given the 15th root of w is 4, then w
5

5

⎛
⎞
⎛
⎞
and ⎜ w ⎟ = 4 5. Since ⎜ w ⎟ = w
⎝
⎠
⎝
⎠
1
15

1
3

1
15

then w = 4 ; SUFFICIENT.
5

The correct answer is D;
each statement alone is sufficient.

1
15

5
15

=4
1
3

=w ,

(1)

The number of rows of seats in the auditorium
is 27.

(2)

The number of seats in the last row is 70.

Arithmetic Sequences and series

Determine the number of seats in the auditorium.
It is given that the first row has 18 seats, and
since each row after the first row has 2 more
seats than the previous row, the second row has
20 seats, the third row has 22 seats, and so on.
The total number of seats in the auditorium can
be determined if and only if the number of rows
in the auditorium can be determined.
(1)

292

The number of rows is given to be 27, so the
total number of seats can be determined;
SUFFICIENT.

6.5 Data Sufficiency Answer Explanations

(2)

The last row has 70 seats. Let n be the
number of rows in the auditorium. Since
the first row has 18 seats, the second row
has 18 + 2 = 20 seats, and the third row has
20 + 2 = 18 + 2(2) = 22 seats, it follows that
the nth row has 18 + (n − 1)(2) seats. Then

Arithmetic Statistics

18 + (n − 1)(2) = 70

(1)

Since the range of a data set is the greatest value
in the data set minus the least value in the data
set, this problem can be solved if the least selling
price and the greatest selling price of the wallets
sold by the store yesterday can be determined.

18 + 2n − 2 = 70
2n = 54
n = 27
Thus the number of rows can be
determined; SUFFICIENT.

(2)

The correct answer is D;
each statement alone is sufficient.
243. If u > 0 and v > 0, which is greater, uv or vu ?
(1)

u=1

(2)

v>2

Arithmetic Exponents

(1)

(2)

Given that u = 1, then uv = 1v = 1 and
vu = v1 = v. If v = 2, then uv = 1 is less than
vu = 2. However, if v = 0.5, then uv = 1 is
greater than vu = 0.5; NOT sufficient.
Given that v > 2, it is possible that uv
is less than vu (for example, if u = 1 and
v = 4, then uv = 14 = 1 and vu = 41 = 4),
and it is possible that uv is greater than
vu (for example, if u = 3 and v = 4, then
uv = 34 = 81 and vu = 43 = 64);
NOT sufficient.

Taking (1) and (2) together, it follows from (1)
that uv = 1 and vu = v, and it follows from (2)
that v > 2. Therefore, uv = 1 < 2 < v = vu, and so
vu is greater than uv.
The correct answer is C;
both statements together are sufficient.
244. What was the range of the selling prices of the
30 wallets sold by a certain store yesterday?
(1)
(2)

1
of the wallets had a selling price of $24 each.
3
1
The lowest selling price of the wallets was the
3
highest selling price of the wallets.

This indicates that 10 of the wallets had
a selling price of $24 each, but does not
indicate the least selling price or the greatest
selling price of the wallets sold by the store
yesterday; NOT sufficient.
This indicates that L = 1 G, where L
3
represents the least selling price and G
represents the greatest selling price, but does
not give enough information to determine
G − L; NOT sufficient.

Taking (1) and (2) together, the least and greatest
selling prices of the wallets sold by the store
yesterday could be $10 and $30 for a range of
$30 − $10 = $20, or the least and greatest
selling prices of the wallets sold by the store
yesterday could be $20 and $60 for a range
of $60 − $20 = $40.
The correct answer is E;
both statements together are still not sufficient.
245. Three houses are being sold through a real estate
agent. What is the asking price for the house with the
second‐largest asking price?
(1)

The difference between the greatest and the
least asking price is $130,000.

(2)

The difference between the two greater asking
prices is $85,000.

Algebra Simultaneous equations

Let x, y, and z, where x ≤ y ≤ z, be the asking
prices of the three houses. This problem can be
solved by determining the value of y.
(1)

This indicates that z − x = $130,000, but
does not give the value of y; NOT sufficient.

(2)

This indicates that z − y = $85,000, but does
not give the value of y; NOT sufficient.

Taking (1) and (2) together, x, y, and z could be
$100,000, $145,000, and $230,000, respectively,
in which case the second highest selling price is
$145,000, or they could be $200,000, $245,000,
293

GMAT® Official Guide 2018

and $330,000, respectively, in which case the
second highest selling price is $245,000.
The correct answer is E;
both statements together are still not sufficient.
246. Is rw = 0 ?
(1)

–6 < r < 5

(2)

6 < w < 10

248. How many people in a group of 50 own neither a fax
machine nor a laser printer?
(1)

The total number of people in the group who
own a fax machine or a laser printer or both is
less than 50.

(2)

The total number of people in the group who own
both a fax machine and a laser printer is 15.

Algebra Sets

(1)

This indicates that the total number who
own either a fax machine or a laser printer
or both is less than 50, but does not indicate
how much less than 50 the total number is.
Thus, the number of people in the group
who own neither a fax machine nor a laser
printer cannot be determined;
NOT sufficient.

(2)

This indicates that, of the 50 people, 15 own
both a fax machine and a laser printer, but
does not indicate how many own one or the
other. Thus, the number of people in the
group who own neither a fax machine nor a
laser printer cannot be determined;
NOT sufficient.

Algebra Inequalities

This problem can be solved if it can be
determined that either or both of r and w is zero
or that neither r nor w is zero.
(1)

(2)

This indicates that r can be, but does not
have to be, zero and gives no indication of
the value of w; NOT sufficient.
This indicates that w cannot be zero, but
gives no indication of the value of r;
NOT sufficient.

Taking (1) and (2) together, r could be zero,
in which case rw = 0 or r could be nonzero, in
which case rw ≠ 0.
The correct answer is E;
both statements together are still not sufficient.
247. Is x =

1
?
y

(1)

xy = 1

(2)

1
=1
xy

Algebra Equations

(1)

(2)

This indicates that neither x nor y can be
zero because if x = 0 or y = 0, then xy = 0.
Since xy = 1 and y ≠ 0, then dividing both
sides by y gives x = 1 ; SUFFICIENT.
y
This indicates that neither x nor y can
be zero because if x = 0 or y = 0, then
xy = 0, and division by zero is undefined.
Multiplying both sides of the equation
1 = 1 by x gives 1 = x; SUFFICIENT.
xy
y

The correct answer is D;
each statement alone is sufficient.
294

Taking (1) and (2) together, it is known that
15 people own both a fax machine and a laser
printer and that the total number who own either
a fax machine or a laser printer or both is less than
50, but the exact number who own neither still
cannot be determined. For example, if 20 people
own only a fax machine and 10 people own only
a laser printer, then both (1) and (2) are true and
the number of people who own neither a fax
machine nor a laser printer is 50 − 20 − 10 − 15 = 5.
However, if 10 people own only a fax machine
and 10 people own only a laser printer, then both
(1) and (2) are true and the number of people
who own neither a fax machine nor a laser printer
is 50 − 10 − 10 − 15 = 15.
The correct answer is E;
both statements together are still not sufficient.

6.5 Data Sufficiency Answer Explanations

249. What is the value of w–2 ?
(1)
(2)

251. On a recent trip, Mary drove 50 miles. What was the
average speed at which she drove the 50 miles?

1
2
w3 = 8

w–1 =

(1)

She drove 30 miles at an average speed of
60 miles per hour and then drove the remaining
20 miles at an average speed of 50 miles
per hour.

(2)

She drove a total of 54 minutes.

Algebra Exponents
2

(1)

1
Since w–1 = , then (w–1)2 = ⎛ 1 ⎞ = 1 or
⎝ 2⎠
2
4
1
–2
w = ; SUFFICIENT.
4

Since w3 = 8, then w = 3 8 = 2 and
1
w–2 = 2–2 = ; SUFFICIENT.
4
The correct answer is D;
each statement alone is sufficient.

(2)

250. A certain investment earned a fixed rate of 4 percent
interest per year, compounded annually, for five years.
The interest earned for the third year of the investment
was how many dollars greater than that for the first year?
(1)

The amount of the investment at the beginning of
the second year was $4,160.00.

(2)

The amount of the investment at the beginning of
the third year was $4,326.40.

Arithmetic Rate problem

Mary drove 50 miles. Determine the average
speed at which she drove the 50 miles.
If the time it took Mary to drive the 50 miles
can be determined, then her average speed can
be determined: distance = speed × time, or
distance = time.
speed
(1)

It is given that she drove the first 30 miles
at an average speed of 60 miles per hour, so
she drove the first 30 miles in 30 = 1 hour.
60 2
It is also given that she drove the remaining
20 miles at an average speed of 50 miles per
hour, so she drove the remaining 20 miles in
20 = 2 hours. Thus, she drove the 50 miles
50 5
in a total of 1 + 2 = 9 hours, and thus
2 5 10
her average speed for the 50 miles can be
determined; SUFFICIENT.

(2)

It is given that she drove a total of 54 minutes,
and thus her average speed for the 50 miles
can be determined; SUFFICIENT.

Arithmetic Applied problems; Percents

This problem can be solved by determining
the dollar amount of the interest earned for
the third year minus the dollar amount of
interest earned in the first year, or, in symbols,
[A(1.04)3 − A(1.04)2] − [A(1.04)1 − A], where A
represents the dollar amount of the investment.
This can be determined if the value of A can be
determined.
(1)

(2)

This indicates that A(1.04)1 = $4,160.00, since
the amount of the investment at the beginning
of the second year is the same as the amount
of the investment at the end of the first year.
Thus, the value of A can be determined by
dividing 4,160.00 by 1.04; SUFFICIENT.
This indicates that A(1.04) = $4,326.40,
since the amount of the investment at the
beginning of the third year is the same as
the amount of the investment at the end of
the second year. Thus, the value of A can be
determined by dividing 4,326.40 by (1.04)2;
SUFFICIENT.

The correct answer is D;
each statement alone is sufficient.
252. What is the value of t ?
(1)

s+t=6+s

(2)

t3 = 216

2

The correct answer is D;
each statement alone is sufficient.

Algebra Equations

(1)

Given that s + t = 6 + s, subtracting s from
both sides gives t = 6; SUFFICIENT.

(2)

Given that t 3 = 216, taking the cube root of
both sides gives t = 6; SUFFICIENT.

The correct answer is D;
each statement alone is sufficient.
295

GMAT® Official Guide 2018

253. For a certain car repair, the total charge consisted of
a charge for parts, a charge for labor, and a 6 percent
sales tax on both the charge for parts and the charge
for labor. If the charge for parts, excluding sales tax,
was $50.00, what was the total charge for the repair?
(1)

The sales tax on the charge for labor was $9.60.

(2)

The total sales tax was $12.60.

Arithmetic Applied problems

The diagram below incorporates the given
information that George has a total of B books,
of which 25 are both fiction and hardcover. In the
diagram, x represents the number of books that
are fiction and not hardcover, and y represents
the number of books that are hardcover and not
fiction.

Arithmetic Applied problems; Percents

To find the total charge, T, for the repair,
requires determining the value, in dollars,
of T = (1.06)(50 + L) = 53 + 1.06L, where
L represents the charge, in dollars, for labor.
This value can be determined if the value
of L can be determined.
(1)

(2)

fiction

hardcover

x

25

This indicates that 0.06L = 9.60, from
which L can be determined by dividing
9.60 by 0.06; SUFFICIENT.
This indicates that (0.06)(50 + L) = 12.60.
So, 3.00 + 0.06L = 12.60 or 0.06L = 9.60,
which is the same as the equation in (1);
SUFFICIENT.

The correct answer is D;
each statement alone is sufficient.
254. George has a total of B books in his library, 25 of
which are hardcover fiction books. What is the
value of B ?
(1)

40 of the B books are fiction and the rest are
nonfiction.

(2)

60 of the B books are hardcovers and the rest
are paperbacks.

y

B – x – 25 – y

(1)

Given that 40 of the books are fiction, it
follows that x + 25 = 40, or x = 15. Using
this additional information, the diagram
(shown below) clearly shows that the value
of B cannot be determined. For example,
y = 35 and B = 80 is possible (note that
B − 40 − y = 5), and y = 35 and B = 100 is
possible (note that B − 40 − y = 25);
NOT sufficient.

fiction

hardcover

15

25

y

B – 40 – y

296

6.5 Data Sufficiency Answer Explanations

(2)

Given that 60 of the books are hardcover,
it follows that 25 + y = 60, or y = 35. Using
this additional information, the diagram
(shown below) clearly shows that the value
of B cannot be determined. For example,
x = 15 and B = 80 is possible (note that
B − x − 60 = 5), and x = 15 and B = 100 is
possible (note that B − x − 60 = 25);
NOT sufficient.

fiction

hardcover

x

25

255. Is w + h4 positive?
(1)

h is positive.

(2)

w is positive.

Arithmetic Properties of numbers

(1)

Given that h is positive, then w + h4 can be
positive (for example, when w = 1 and h = 1)
and w + h4 can be negative (for example,
when w = –2 and h = 1);
NOT sufficient.

(2)

Given that w is positive, then because
h4 ≥ 0, it follows that w + h4 ≥ w + 0 = w
and thus w + h4 > 0 since w > 0;
SUFFICIENT.

35

The correct answer is B;
statement 2 alone is sufficient.
B – x – 60

256. In Mr. Smith’s class, what is the ratio of the number of
boys to the number of girls?
(1)

Taking (1) and (2) together, there is still not
enough information to determine the value of B,
since in the diagram below B = 80 is possible and
B = 100 is possible.

(2)

There are 3 times as many girls as boys in
Mr. Smith’s class.
1
The number of boys is of the total number of
4
boys and girls in Mr. Smith’s class.

Algebra Ratio and proportion
fiction

hardcover

15

25

35

Letting B be the number of boys and G be the
number of girls, determine the value of B .
G
B
(1) It is given that G = 3B, so 1 = ;
3 G
SUFFICIENT.
(2)

B – 75

It is given that B = 1 ( B + G ). Therefore,
4
4B = B + G, or 3B = G, or B = 1 ;
G 3
SUFFICIENT.

The correct answer is D;
each statement alone is sufficient.
The correct answer is E;
both statements together are still not sufficient.

257. If the sequence S has 300 terms, what is the 293rd
term of S ?
(1)

The 298th term of S is –616, and each
term of S after the first is 2 less than the
preceding term.

(2)

The first term of S is –22.

297

GMAT® Official Guide 2018

Arithmetic Sequences and series

Determine the 293rd term of the sequence S,
or S293.
(1)

(2)

It is given that the 298th term of S is
–616, and each term of S after the first
term is 2 less than the preceding term.
Therefore, S298 = –616 = S297 − 2, and so
S297 = –616 + 2 = –614. Similarly,
S296, S295, S294, and S293 can be determined,
each from the term preceding it;
SUFFICIENT.
It is given that S1 = –22, but there is no
information on how subsequent terms
are related to S1 and therefore no way to
determine S293; NOT sufficient.

259. In each of the last five years, Company K donated
p percent of its annual profits to a certain scholarship
fund. Did Company K donate more than $10,000 to
the scholarship fund last year?
(1)

Two years ago, Company K had annual profits
of $3 million and donated $15,000 to the
scholarship fund.

(2)

Last year, Company K had annual profits of
$2.5 million.

Arithmetic Applied problems; Percents

Let $k be Company K’s profit last year.
⎛ p ⎞
Is ⎜
k > 10,000 ?
⎝ 100 ⎟⎠
(1)

The correct answer is A;
statement 1 alone is sufficient.
258. If a is a 3-digit integer and b is a 3-digit integer, is the
units digit of the product of a and b greater than 5 ?
(1)

The units digit of a is 4.

(2)

The units digit of b is 7.

Arithmetic Operations with integers

Determine whether the units digit of the product
of the two 3-digit integers a and b is greater than 5.
(1)

It is given that the units digit of a is 4. If the
units digit of b is 0 or 1, then the units digit
of ab is not greater than 5. However, if the
units digit of b is 2, then the units digit of
ab is greater than 5; NOT sufficient.

(2)

It is given that the units digit of b is 7. If
the units digit of a is 0, then the units digit
of ab is not greater than 5. However, if the
units digit of a is 1, then the units digit of
ab is greater than 5; NOT sufficient.

Taking (1) and (2) together, the units digit of a is
4 and the units digit of b is 7. Therefore, the units
digit of ab is 8, which is greater than 5.
The correct answer is C;
both statements together are sufficient.

(2)

⎛ p ⎞
(3 million) = 15,000,
Given that ⎜
⎝ 100 ⎟⎠
(15, 000)(100) 1
it follows that p =
= .
3 million
2
However, the question cannot be answered
since no information is available about the
value of k. For example, if k = 3 million, then
⎛ p ⎞
⎜⎝
⎟ k = 15,000, and the answer to the
100 ⎠
question is yes. However, if k = 1 million,
⎛ p ⎞
then ⎜
k = 5,000, and the answer to the
⎝ 100 ⎟⎠
question is no; NOT sufficient.
Given that k = 2.5 million, it follows that
⎛ p ⎞
⎛ p ⎞
⎜⎝
⎟⎠ k = ⎜⎝
⎟ (2.5 million). However,
100
100 ⎠
the question cannot be answered since no
information is available about the value of p.
⎛ p ⎞
k = 25,000,
For example, if p = 1, then ⎜
⎝ 100 ⎟⎠
and the answer to the question is yes. However,
⎛ p ⎞
if p = 0.1, then ⎜
k = 2,500, and the
⎝ 100 ⎟⎠
answer to the question is no; NOT sufficient.

Taking (1) and (2) together, the values of both
p and k can be determined. Therefore, the value of
⎛ p ⎞
⎜⎝
⎟ k can be determined, and thus the answer
100 ⎠
to the question can be determined.
The correct answer is C;
both statements together are sufficient.

298

6.5 Data Sufficiency Answer Explanations

x

z
y

260. Is the area of the triangular region above less than 20 ?
(1)

x2 + y2 ≠ z2

(2)

x + y < 13

Geometry Triangles; Area

Determine whether the area of the given triangle
is less than 20.
(1)

This indicates the triangle is not a right
triangle but gives no information about the
base and height or how to determine them
in order to find the area; NOT sufficient.

(2)

This indicates that two sides of the triangle
have lengths x and y such that x + y < 13. If,
for example, the triangle is a right triangle
with x = 3 and y = 4, then x + y = 3 + 4 =
7 < 13 and the area of the triangle is
1 (3)(4) = 6, which is less than 20. But if the
2
triangle is a right triangle with x = y = 42,
then x + y = 42 + 42 = 2 42 = 168 <
169 = 13 and the area is 1 ( 42)( 42) =
2
21, which is greater than 20; NOT sufficient.

the acute triangle, the area of the triangular region
is 1 yh = 1 ( 42)( 41) and 1 ( 42)( 41) >
2
2
2
1 ( 40 )( 40 ) or 1 ( 42)( 41) > 20. Therefore,
2
2
it is possible that the area of the triangular region
can be less than 20 (in the case of the obtuse
triangle) and it is possible that the area of the
triangular region can be greater than 20 (in the
case of the acute triangle).
The correct answer is E;
both statements together are still not sufficient.
261. A, B, C, and D are points on a line. If C is the midpoint
of line segment AB and if D is the midpoint of line
segment CB, is the length of line segment DB greater
than 5 ?
(1)

The length of line segment AC is greater than 8.

(2)

The length of line segment CD is greater than 6.

Geometry Lines and segments

It is given that C is the midpoint of AB,
so AC = CB = 1 (AB). It is given that D is
2
the midpoint of CB, so CD = DB = 1 (CB).
2
Determine if DB > 5.
(1)

Given that AC > 8, it follows that CB > 8
1
since AC = CB, and so DB = 1 (CB) > (8)
2
2
or DB > 4. However, this means that DB
could be 6 and 6 > 5 or DB could be 4.1 and
4.1 < 5; NOT sufficient.

z

(2)

Given that CD > 6, it follows that DB > 6
since CD = DB; SUFFICIENT.

y

The correct answer is B;
statement 2 alone is sufficient.

Taking (1) and (2) together, since the triangle is
not a right triangle from (1), the diagrams show
two other possibilities: an obtuse triangle and an
acute triangle.
h

x

z
y

x

h
d

For the obtuse triangle, h < x since because h is
a leg of a right triangle with hypotenuse x. The
area A is given by A = 1 yh < 1 xy. If x = y = 6,
2
2
then x + y = 6 + 6 = 12 < 13 and A < 1 (62) = 18.
2
The area of the triangular region is less than 20
for the obtuse triangle. For the acute triangle,
if x = y = 42 , then x + y = 42 + 42 =
2 42 = 168 < 169 = 13. Let d = 1. Then, h2 =
( 42)2 − 1 = 42 − 1 = 41 and h = 41 . Thus, for

262. The people in a line waiting to buy tickets to a show
are standing one behind the other. Adam and Beth
are among the people in the line, and Beth is standing
behind Adam with a number of people between them. If
the number of people in front of Adam plus the number
of people behind Beth is 18, how many people in the
line are behind Beth?
(1)

There are a total of 32 people in the line.

(2)

23 people in the line are behind Adam.
299

GMAT® Official Guide 2018

Arithmetic Order

Beth is standing in line behind Adam with a
number of people between them. Let x be the
number of people ahead of Adam, let y be the
number of people between Adam and Beth, and
let z be the number of people behind Beth. It is
given that x + z = 18. Determine z.
(1)

(2)

This indicates that there are 32 people in
the line. Two of these people are Adam
and Beth, so there are 30 other people in
line besides Adam and Beth. Therefore,
x + y + z = 30 and x + z = 18. From this,
y = 12, but z cannot be determined uniquely.
For example, if x = 5, then z = 13, but if
x = 10, then z = 8; NOT sufficient.
This indicates that y + 1 + z = 23 because
the people behind Adam consist of the
people between Adam and Beth, Beth
herself, and the people behind Beth. If, for
example, y = 4, then z = 18, but if y = 9, then
z = 13; NOT sufficient.

Taking (1) and (2) together, y = 12 from (1) and
y + z = 22 from (2). Therefore, z = 10, and there
are 10 people in line behind Beth.

from which the side length of the square can be
determined using the Pythagorean theorem.
(1)

This indicates that the area of the circle is
64π, so letting r represent the radius of the
circle, πr2 = 64π. Thus, r2 = 64, r = 8, and
d = 2(8) = 16, where d represents the length
of the diameter of the circle and also the
diagonal of the square; SUFFICIENT.

(2)

This indicates that the circumference of
the circle is 16π, so letting d represent the
length of the diameter of the circle as well
as the diagonal of the square, πd = 16π.
Thus, d = 16; SUFFICIENT.

The correct answer is D;
each statement alone is sufficient.
264. Lines k and m are parallel to each other. Is the slope of
line k positive?
(1)

Line k passes through the point (3,2).

(2)

Line m passes through the point (–3,2).

Geometry Coordinate geometry

(1)

This indicates that line k passes through
the point (3,2). One point is not enough
to determine whether the slope of line k is
positive. If, for example, line k also passes
through the point (0,0), then the slope of
line k is 2 − 0 = 2 , which is positive. But if
3− 0 3
line k also passes through the point (5,0),
then the slope of line k is 2 − 0 = 2 , which
3 − 5 −2
is negative; NOT sufficient.

(2)

This indicates that line m passes through
the point (–3,2). One point is not enough
to determine whether the slope of line m is
positive. If, for example, line m also passes
through the point (0,0), then the slope of
line m is 2 − 0 = 2 , which is negative. In
−3 − 0 −3
this case, the slope of line k is also negative
since parallel lines k and m have the same
slope. But if line m also passes through the
point (0,5), then the slope of line
m is 2 − 5 = −3 , which is positive.
−3 − 0 −3
In this case, the slope of line k is also
positive since parallel lines k and m have the
same slope; NOT sufficient.

The correct answer is C;
both statements together are sufficient.
263. Square ABCD is inscribed in circle O. What is the area
of square region ABCD ?
(1)

The area of circular region O is 64π.

(2)

The circumference of circle O is 16π.

Geometry Circles; Area

A

B

D

C

Since the area of square ABCD is the square of its
side length, to solve this problem it is necessary
to determine the side length of ABCD. From the
figure, a diameter of the circle coincides with a
diagonal of the square. If the length of a diameter of
the circle can be determined, then the length
of a diagonal of the square can be determined,
300

6.5 Data Sufficiency Answer Explanations

Taking (1) and (2) together gives no more
information than (1) or (2) alone and so whether
the slope of line k is positive cannot be determined.
The correct answer is E;
both statements together are still not sufficient.

266. In a certain group of 50 people, how many are doctors
who have a law degree?
(1)

In the group, 36 people are doctors.

(2)

In the group, 18 people have a law degree.

265. In cross section, a tunnel that carries one lane of
one‐way traffic is a semicircle with radius 4.2 m. Is the
tunnel large enough to accommodate the truck that is
approaching the entrance to the tunnel?
(1)

The maximum width of the truck is 2.4 m.

(2)

The maximum height of the truck is 4 m.

law degree

doctors
x

y

z
w

Arithmetic Sets

Geometry Circles; Pythagorean theorem

Using the labels in the Venn diagram,
w + x + y + z = 50. Determine the value of y.

(1)

(1)

This indicates that x + y = 36 but does not
give a unique value for y. For example,
if x = 18, then y = 18; but if x = 10, then
y = 26; NOT sufficient.

(2)

This indicates that y + z = 18 but does not
give a unique value for y. For example, if
z = 8, then y = 10; but if z = 10, then y = 8;
NOT sufficient.

(2)

Given that the maximum width of the truck
is 2.4 m, it is possible that the tunnel is large
enough (for example, if the maximum height
of the truck is sufficiently small) and it is
possible that the tunnel is not large enough
(for example, if the maximum height of the
truck is sufficiently large); NOT sufficient.
Given that the maximum height of the
truck is 4 m, it is possible that the tunnel is
large enough (for example, if the maximum
width is sufficiently small) and it is possible
that the tunnel is not large enough (for
example, if the maximum width of the truck
is sufficiently large); NOT sufficient.

Taking (1) and (2) together, it is possible for the
truck to pass through the tunnel, because even
if the truck were rectangular with width 2.4 m
and height 4 m, the truck could pass through
the tunnel if it were centered in the tunnel as
shown in the diagram. In this position, it follows
from the Pythagorean theorem that every point
on the truck is located at most a distance of
(1.2)2 + 4 2 meters from the center axis of the
tunnel, and this distance is less than 4.2 meters,
the radius of the tunnel, since (1.2)2 + 42 = 17.44
is less than (4.2)2 = 17.64.

Taking (1) and (2) together, if w = 6, x = 26,
y = 10, and z = 8, then w + x + y + z = 50,
x + y = 36, and y + z = 18. In this case, y = 10.
However, if w = 2, x = 30, y = 6, and z = 12, then
w + x + y + z = 50, x + y = 36, and y + z = 18. In
this case, y = 6.
Alternatively, this problem can be solved by
means of a contingency table. Letting n represent
the number of doctors with a law degree, the
table with the given information is shown below.
law degree
no law degree
total
(1)

The correct answer is C;
both statements together are sufficient.

not a doctor

total

50

This indicates that there are 36 doctors,
so the number of people who are not
doctors is 50 − 36 = 14. The table with this
information is shown below.

4m
1.2 m

doctor
n

law degree
no law degree
total

doctor
n
36

not a doctor

14

total

50
301

GMAT® Official Guide 2018

It is obvious from the table that there is not
enough information to determine the value of n,
the number of doctors with a law degree; NOT
sufficient.
(2)

This indicates that there are 18 people
with a law degree, so the number of people
without a law degree is 50 − 18 = 32. The
table with this information is shown below.

law degree
no law degree
total

doctor
n

not a doctor

total
18
32
50

It is obvious from the table that there is not enough
information to determine the value of n, the number
of doctors with a law degree; NOT sufficient.
The table with the information from both (1) and
(2) is shown below.

law degree
no law degree
total

doctor
n

not a doctor

36

14

total
18
32
50

It is obvious from the table that there is still not
enough information to determine the value of n,
the number of doctors with a law degree.
The correct answer is E;
both statements together are still not sufficient.
267. Of a group of 50 households, how many have at least
one cat or at least one dog, but not both?
(1)

The number of households that have at least one
cat and at least one dog is 4.

(2)

The number of households that have no cats and
no dogs is 14.
dog

cat
x

y

z
w

Arithmetic Sets

Using the labels on the Venn diagram,
w + x + y + z = 50. Determine the value of x + z.
302

(1)

This indicates that y = 4, so w + x + z = 46,
but the value of x + z cannot be uniquely
determined; NOT sufficient.

(2)

This indicates that w = 14, so x + y + z = 36,
but the value of x + z cannot be uniquely
determined; NOT sufficient.

Taking (1) and (2) together, 14 + x + 4 + z = 50,
so x + z = 50 − 14 − 4 = 32.
The correct answer is C;
both statements together are sufficient.
268. On a certain date, Hannah invested $5,000 at
x percent simple annual interest and a different amount
at y percent simple annual interest. What amount did
Hannah invest at y percent simple annual interest?
(1)

The total amount of interest earned by Hannah’s
two investments in one year was $900.

(2)

Hannah invested the $5,000 at 6 percent simple
annual interest.

Algebra Applied problems

Let $A be the amount that Hannah invested at
y percent. Determine the value of A.
(1)

Given that the sum of x ($5, 000) and
100
Ay
y
= 900, it
($ A ) is $900, or 50 x +
100
100
is clear that more than one value of A is
possible; NOT sufficient.

(2)

Given that x = 6, it is clear that more than
one value of A is possible; NOT sufficient.

(1) and (2) together are not sufficient because
more than one value of A is possible such that
Ay
50(6) +
= 900, or Ay = 60,000. For example,
100
A = 10,000 and y = 6 is possible, and A = 20,000
and y = 3 is also possible.
The correct answer is E;
both statements together are still not sufficient.
269. Do at least 60 percent of the students in Pat’s class
walk to school?
(1)

At least 60 percent of the female students in
Pat’s class walk to school.

(2)

The number of students in Pat’s class who walk
to school is twice the number of students who
do not walk to school.

6.5 Data Sufficiency Answer Explanations

Arithmetic Percents

This problem can be solved by determining the
total number of students in Pat’s class and the
number who walk to school.
(1)

(2)

This indicates that at least 60% of the female
students in Pat’s class walk to school. However,
it does not give any information about the
other students in Pat’s class; NOT sufficient.
Letting x represent the number of students
in Pat’s class who do not walk to school,
this indicates that 2x students in Pat’s
class walk to school and that the total
number of students in Pat’s class is
x + 2x = 3x. From this, the percent of
students in Pat’s class who walk to school is
⎡ 2x 100 ⎤ % = ⎡ 2
(100) ⎤⎥%; SUFFICIENT.
⎥⎦
⎢⎣ 3x
⎦
⎣⎢ 3

The correct answer is B;
statement 2 alone is sufficient.
270. The profit from the sale of a certain appliance
increases, though not proportionally, with the number
of units sold. Did the profit exceed $4 million on sales
of 380,000 units?
(1)

The profit exceeded $2 million on sales of
200,000 units.

(2)

The profit exceeded $5 million on sales of
350,000 units.

Arithmetic Arithmetic operations; Proportions

(1)

(2)

If the profits did increase proportionally,
it might be reasonable to expect a profit
of $4 million on sales of 400,000 units.
However, it is given that the profits do not
increase proportionally. Without knowing
how the profits increase, it is impossible to
tell the profits on sales of 380,000 units;
NOT sufficient.
It is given that the profits do increase with the
number of units sold. Therefore, since the profit
on sales of just 350,000 units well exceeded
$4 million, then sales of 350,000 + 30,000 =
380,000 units would also have a profit
exceeding $4 million; SUFFICIENT.

271. If n is an integer, is n even?
(1)

n2 − 1 is an odd integer.

(2)

3n + 4 is an even integer.

Arithmetic Properties of numbers

Determine if the integer n is even.
(1)

Since n2 − 1 is odd, n2 is even and so n is
even; SUFFICIENT.

(2)

Since 3n + 4 is even, 3n is even and so n is
even; SUFFICIENT.

The correct answer is D;
each statement alone is sufficient.
272. Carmen currently works 30 hours per week at her
part‐time job. If her gross hourly wage were to
increase by $1.50, how many fewer hours could she
work per week and still earn the same gross weekly
pay as before the increase?
(1)

Her gross weekly pay is currently $225.00.

(2)

An increase of $1.50 would represent an
increase of 20 percent of her current gross
hourly wage.

Arithmetic Operations with rational numbers

Let w be Carmen’s gross hourly wage and let n be
the number of hours fewer Carmen will need to
work. Find a unique value for n such that
30w = (30 − n)(w + 1.50).
(1)

Since Carmen’s gross weekly pay is
currently $225.00, then 30w = 225 and
w = 7.50. Substituting 7.50 for w gives
30(7.50) = (30 − n)(7.50 + 1.50), which
can be solved for a unique value of n;
SUFFICIENT.

(2)

Since 1.50 is 20 percent of Carmen’s
current gross hourly pay, 1.50 = 0.20w and
w = 7.50. This is the same information that
was gained from statement (1) and will lead
to the same result; SUFFICIENT.

The correct answer is D;
each statement alone is sufficient.

The correct answer is B;
statement 2 alone is sufficient.
303

GMAT® Official Guide 2018

273. A certain plumber charges $92 for each job completed
in 4 hours or less and $23 per hour for each job
completed in more than 4 hours. If it took the plumber
a total of 7 hours to complete two separate jobs, what
was the total amount charged by the plumber for the
two jobs?
(1)

The plumber charged $92 for one of the two
jobs.

(2)

The plumber charged $138 for one of the two
jobs.

Arithmetic Applied problems

Find the total amount charged for two jobs
lasting a total of 7 hours if the plumber charges
$92 for a job lasting 4 hours or less and $23 per
hour for a job lasting more than 4 hours.
(1)

(2)

This indicates that one of the two jobs lasted
4 hours or less. If that job lasted 2 hours, then
the other job lasted 5 hours, and the charge
for that job would be ($23)(5) = $115, making
the total for the two jobs $92 + $115 = $207.
However, if the jobs lasted 1 hour and
6 hours, respectively, then the total charge
for the two jobs would be $92 + (6)($23) =
$92 + $138 = $230; NOT sufficient.
This indicates that one job lasted
$138 = 6 hours. Therefore, the other
$23
job lasted for 1 hour and the charge for
that job was $92, making the total charge
for the two jobs $92 + $138 = $230;
SUFFICIENT.

The correct answer is B;
statement 2 alone is sufficient.
274. If x and y are positive numbers, is
(1)

x>1

(2)

x ?
y +1 y

Algebra Inequalities

Since y is positive, multiplying both sides of
x +1 x
> by y(y + 1) gives
the inequality
y +1 y
y(x + 1) > x(y + 1) or xy + y > xy + x, which is
equivalent to y > x. So, determining whether the
304

x +1 x
> is true is equivalent to
y +1 y
determining whether the inequality y > x is true.

inequality

(1)

This indicates that x > 1. If, for example,
x = 3 and y = 4, then y > x is true. However,
if x = 4 and y = 3, then y > x is not true;
NOT sufficient.

(2)

This indicates that x < y, so y > x is true;
SUFFICIENT.

The correct answer is B;
statement 2 alone is sufficient.
275. If a and b are positive integers, is
(1)

a
< 0.818
b

(2)

b
> 1.223.
a

a 9
?
<
b 11

Arithmetic Inequalities

If 11a < 9b and b is positive, then 11a < 9 and
b
a < 9 . Therefore, this problem can be solved by
b 11
determining if 11a < 9b.
(1)

From this,

a
b <

0.818

given

a <

0.818b

multiply both sides by b,
which is positive

11a <

8.998b

multiply both sides by 11

Then, since 8.998b < 9b, it follows that 11a < 9b;
SUFFICIENT.
(2)
b
>
a
b >

From this,
1.223 given
1.223a

multiply both sides by a, which is
positive

9b > 11.007a multiply both sides by 9
Then, since 11.007a > 11a, it follows that
9b > 11a; SUFFICIENT.
The correct answer is D;
each statement alone is sufficient.

6.5 Data Sufficiency Answer Explanations

276. Every object in a box is either a sphere or a cube, and
every object in the box is either red or green. How
many objects are in the box?
(1)

There are six cubes and five green objects in the
box.

(2)

There are two red spheres in the box.

This problem can be solved using a contingency
table set up as shown below, where T represents
the number to be determined.
sphere

red
green
total
(1)

sphere

x + y is odd.

Arithmetic Properties of numbers

(1)

This indicates that x 2 + y 2 − 1 is divisible
by 4, so x 2 + y 2 − 1 = 4q for some integer q.
Then x 2 + y 2 = 4q + 1, which means x 2 + y 2
is odd. Both x 2 and y 2 cannot be even
because, in that case, their sum would be
even, and both cannot be odd because, in that
case, their sum would also be even. Therefore,
one of x 2 and y 2 is even, and the other is odd.
It follows that one of x or y is even and the
other is odd, so xy is even; SUFFICIENT.

(2)

If x + y is odd, then one of x and y is even and
the other is odd because if both were even
or both were odd, the sum would be even. It
follows that xy is even; SUFFICIENT.

total
5
T

6

It is obvious that there is not enough information
to determine a unique value for T; NOT sufficient.
The following table displays the information
that there are 2 red spheres:
cube
red
green
total

sphere
2

total

T

It is obvious that there is not enough information
to determine a unique value for T; NOT sufficient.
The following table displays the information from
(1) and (2) taken together:
cube
red
green
total

(2)

T

cube

(2)

x2 + y2 − 1 is divisible by 4.

total

The following table displays the information
that there are 6 cubes and 5 green objects:

red
green
total

(1)

Determine whether the product of two positive
integers, x and y, is even.

Arithmetic Sets

cube

277. If x and y are positive integers, is xy even?

6

sphere
2

total
5
T

It is obvious that there is still not enough
information to determine a unique value for T.

The correct answer is D;
each statement alone is sufficient.
278. If a and b are integers, is a + b + 3 an odd integer?
(1)

ab is an odd integer.

(2)

a − b is an even integer.

Arithmetic Properties of numbers

Determine whether a + b + 3 is odd for integers
a and b.
(1)

This indicates that, since ab is odd, both a
and b are odd because if one of a or b is even
or both a and b are even, then ab is even.
Therefore, a + b is even and a + b + 3 is odd;
SUFFICIENT.

(2)

This indicates that, since a − b is even, both
a and b are even or both are odd because if
one of them is even and the other is odd,
then a − b is odd. Therefore, a + b is even,
and a + b + 3 is odd; SUFFICIENT.

The correct answer is D;
each statement alone is sufficient.

The correct answer is E;
both statements together are still not sufficient.
305

GMAT® Official Guide 2018

279. If x and y are positive integers, what is the value of
x+ y ?
(1)

x + y = 15

(2)

xy = 6

(2)

(2)

From this, if x = 1 and y = 36, then xy = 6
and x + y = 1 + 36 = 1 + 6 = 7 .
However, if x = 4 and y = 9, then xy = 6
and x + y = 4 + 9 = 2 + 3 = 5; NOT
sufficient.

Taking (1) and (2) together, x + y = 15 and
xy = 6, so
x(15 − x ) =

6 substitute 15 − x for y

Taking (1) and (2) together,

15x − x2 = 36 use distributive property
x2 − 15x + 36 =
(x − 12)(x − 3) =

0 collect like terms
0 factor

Thus, x = 12 or x = 3. If x = 12, then y = 3 and
x + y = 12 + 3. If x = 3, then y = 12 and

so x +

( x)

+ 2 xy +

( y)

2

(

x+ y

(1)

From this,

(

)

2

=

= x + y + 2 xy can be

determined, then the value of x +
determined, and conversely.
x+ y

)

2

y can be

= x + y + 2 xy =

15 + 2 x(15 − x ) , but the value of x is
unknown, and the value of

(

x+ y

)

2

cannot be determined; NOT sufficient.

306

)

x+ y

)

2

=

y = 27 = 3 3. Note that in the

nonalgebraic solution x +

y = 3 + 12 and

3 + 12 = 3 + 2 3 = 3 3.
The correct answer is C;
both statements together are sufficient.
1
+ kv 2 gallons of fuel per mile
12
when its speed is v miles per hour, where k is a
constant. At what speed should the truck travel so that
5
gallon of fuel per mile?
it uses
12
1
(1) The value of k is
.
10,800

280. A certain truck uses

(2)

When the truck travels at 30 miles per hour, it
1
uses gallon of fuel per mile.
6

Algebra Applied problems

This problem can be solved by determining the
positive value of v so that 1 + kv2 = 5 .
12
12
(1) This indicates the value of k and so the
value of v can be determined by solving
1 + 1 v2 = 5 for v; SUFFICIENT.
12
12 10,800

y = 3 + 12.

Algebraically, if the value of

(

x + y + 2 xy = 15 + (2)(6) = 27 and

x(15 − x) = 36 square both sides

2

2

cannot be determined; NOT sufficient.

From this, if x = 1 and y = 14, then
x + y = 15 and x + y = 1 + 14 = 1 + 14 .
But 14 < 16 = 4, so 1 + 14 < 1 + 4 = 5.
However, if x = 9 and y = 6, then x + y = 15
and x + y = 9 + 6 = 3 + 6 . But
6 > 4 = 2 so, 3+ 6 > 3 + 2 = 5; NOT
sufficient.

x +

)

(

Algebra Operations with radicals

(1)

(

From this, x + y = x + y + 2 xy =
x + 36 + 2(6), but the value of x is
x
2
unknown, and the value of x + y

(2)

This indicates that 1 + k(30)2 = 1 . The
12
6
value of k can be determined by solving
this equation for k. Then that value of k
5
can be substituted into 1 + kv2 = ,
12
12
which can then be solved for the value of v;
SUFFICIENT.

The correct answer is D;
each statement alone is sufficient.

6.5 Data Sufficiency Answer Explanations

t
281. If x = 2t and y = , what is the value of x2 − y2 ?
3
(1)

t2 − 3 = 6

(2)

t3 = –27

Algebra Simplifying algebraic expressions

This problem can be solved by determining the
2
2
value of x 2 − y 2 = (2t )2 − ⎛ t ⎞ = 4t 2 − t , and
⎝ 3⎠
9
this can be determined if the value of t2 or the
value of t can be determined.
(1)

This indicates that t2 − 3 = 6 and so
t2 = 6 + 3 = 9; SUFFICIENT.

(2)

This indicates that t3 = –27 and so
t = –3; SUFFICIENT.

The correct answer is D;
each statement alone is sufficient.
282. Last school year, each of the 200 students at a certain
high school attended the school for the entire year.
If there were 8 cultural performances at the school
during the last school year, what was the average
(arithmetic mean) number of students attending each
cultural performance?
(1)

Last school year, each student attended at least
one cultural performance.

(2)

Last school year, the average number of
cultural performances attended per student
was 4.

(2)

This indicates that, since the average number
of performances attended per student was
total attendance at performances
4,
= 4, so
total number of students
the total attendance at the performances is
(4)(200) = 800 students. It follows that the
average number of students attending each
performance is 800 = 100; SUFFICIENT.
8
The correct answer is B;
statement 2 alone is sufficient.
283. A clothing manufacturer makes jackets that are
wool or cotton or a combination of wool and cotton.
The manufacturer has 3,000 pounds of wool and
2,000 pounds of cotton on hand. Is this enough wool
and cotton to make at least 1,000 jackets?
(1)

Each wool jacket requires 4 pounds of wool, and
no cotton.

(2)

Each cotton jacket requires 6 pounds of cotton,
and no wool.

Arithmetic Applied problems

(1)

Given that each wool jacket requires
4 pounds of wool and no cotton, then at
most a total of 750 wool jackets can be
made (because 3, 000 = 750), and possibly
4
no other jackets. Therefore, it is possible
that there is enough wool and cotton to
make at least 1,000 jackets (for example, if
there is enough cotton to make 250 cotton
jackets) and it is possible that there is not
enough wool and cotton to make at least
1,000 jackets (for example, if there is less
than the amount of cotton needed to make
250 cotton jackets); NOT sufficient.

(2)

Given that each cotton jacket requires
6 pounds of cotton and no wool, then at
most a total of 333 cotton jackets can be
made (because rounding 2, 000 down to the
6
nearest integer gives 333), and possibly no
other jackets. Therefore, it is possible that
there is enough wool and cotton to make at
least 1,000 jackets (for example, if there is
enough wool to make 667 wool jackets), and
it is possible that there is not enough wool

Arithmetic Statistics

Determine the average number of students
attending each of 8 cultural performances at a
school that has 200 students.
(1)

This indicates that each student attended
at least one cultural performance, but the
average number of students attending
each performance cannot be uniquely
determined. It could be as low as
200 = 25 students if each student attended
8
exactly one performance, or it could be as
high as 200 = 200 if each student attended
1
every performance; NOT sufficient.

307

GMAT® Official Guide 2018

and cotton to make at least 1,000 jackets
(for example, if there is less than the amount
of wool needed to make 667 wool jackets);
NOT sufficient.

Arithmetic Properties of numbers

(1)

Given that 2x, where x is an integer greater
than 1, is a common factor of 18 and 24,
then 2x is an even integer greater than or
equal to 4 that is a factor of 18 and 24. The
factors of 18 are 1, 2, 3, 6, 9, and 18. The
factors of 24 are 1, 2, 3, 4, 6, 8, 12, and 24.
Since the only even integer greater than or
equal to 4 that belongs to both lists is 6, it
follows that 2x = 6, or x = 3; SUFFICIENT.

(2)

Given that x, an integer greater than 1, is a
factor of 6, then x could be 2, 3, or 6; NOT
sufficient.

Taking (1) and (2) together, there is enough
wool and cotton to make 750 wool jackets and
333 cotton jackets for a total of 1,083 jackets.
The correct answer is C;
both statements together are sufficient.
284. Each month, Jim receives a base salary plus a
10 percent commission on the price of each car he
sells that month. If Jim sold 15 cars last month, what
was the total amount of base salary and commissions
that Jim received that month?
(1)

Last month, Jim’s base salary was $3,000.

(2)

Last month, Jim sold 3 cars whose prices
totaled $60,000 and 5 cars whose prices
totaled $120,000.

The correct answer is A;
statement 1 alone is sufficient.
286. By what percent was the price of a certain television
set discounted for a sale?
(1)

The price of the television set before it was
discounted for the sale was 25 percent greater
than the discounted price.

Determine the total of Jim’s base salary plus a
10% commission on the price of each of the
15 cars he sold last month.

(2)

The price of the television set was discounted by
$60 for the sale.

(1)

This indicates that Jim’s base salary was
$3,000 but gives no information about the
prices of the 15 cars he sold last month;
NOT sufficient.

(2)

This indicates that Jim’s commission on 8 cars
that he sold last month can be determined
but gives no information about his base salary
or about the prices of the other 7 cars he sold
last month; NOT sufficient.

Let B represent the price, in dollars, for the
television before the sale and let D represent the
discounted price, in dollars. The percent by which
B was discounted to obtain D can be determined
if the value of B − D can be determined.
B
(1) This indicates that B is 25% greater
than D or that B = 1.25D. Then,
B − D 1.25 D − D 0.25
=
=
= 0.2 or 20% ;
B
1.25 D
1.25
SUFFICIENT.

Arithmetic Applied problems

Taking (1) and (2) together gives Jim’s base salary
and information about the prices of 8 of the 15 cars
he sold last month, but does not provide information
about the selling prices of the other 7 cars he sold.
The correct answer is E;
both statements together are still not sufficient.
285. If x is a positive integer greater than 1, what is the
value of x ?

308

(1)

2x is a common factor of 18 and 24.

(2)

x is a factor of 6.

Arithmetic Percents

This indicates that D = B − 60, but B is
unknown. For example, B could be 100
and the value of B − D would be 60 , or B
B
100
B
−
D would
could be 500 and the value of
B
be 60 ; NOT sufficient.
500
The correct answer is A;
statement 1 alone is sufficient.
(2)

6.5 Data Sufficiency Answer Explanations

287. Jack wants to use a circular rug on his rectangular
office floor to cover two small circular stains, each
π
less than
square feet in area and each more than
100
3 feet from the nearest wall. Can the rug be placed to
cover both stains?
(1)

Jack’s rug covers an area of 9π square feet.

(2)

The centers of the stains are less than 4 feet apart.

Geometry Applied problems; Circles

If a circle with radius r has area less than π ,
100
π
1
. Hence, r < 1 , so
, or r2 <
then πr2 <
100
10
100
the circle’s radius is less than 1 ft.
10
(1) Given that the circular rug has area 9π ft2,
the radius of the rug, R ft, satisfies the
equation πR2 = 9π, and so R2 = 9, or R = 3.
If the centers of the circular stains are 4 ft
apart, then the midpoint of the centers will
be a distance of at most ⎛ 1 + 2 + 1 ⎞
⎝ 10
10 ⎠
ft = 2.2 ft from any point on the stains.
Thus, if the circular rug (which has radius
3 ft) were placed so that its center lies on
that midpoint, then the rug would cover
both stains. However, if the centers of the
circular stains are 7 ft apart, then the circular
rug (which has diameter 6 ft) cannot be
placed to cover both stains; NOT sufficient.
(2)

Given that the centers of the circular stains
are less than 4 ft apart, it is not possible to
determine whether the circular rug can be
placed to cover both stains since nothing
is given about the size of the circular rug;
NOT sufficient.

Taking (1) and (2) together, the centers of the
circular stains are less than 4 ft apart and it
follows from the discussion in (1) that the circular
rug can be placed to cover both stains.
The correct answer is C;
both statements together are sufficient.

×

a

b

c

a

d

e

f

b

e

g

h

c

f

h

j

288. In the multiplication table above, each letter represents
an integer. What is the value of c ?
(1)

c=f

(2)

h≠0

Arithmetic Properties of numbers

(1)

Given that c = f, the examples below, in which
a = b = 1, show that the value of c could be 1
and the value of c could be 2; NOT sufficient.
×
1
1
1

1
1
1
1

×
1
1
2
(2)

1
1
1
2

1
1
1
1

1
1
1
1

1
1
1
2

2
2
2
4

Given that h ≠ 0, the examples in (1) show
that the value of c could be 1 and the value
of c could be 2; NOT sufficient.

Taking (1) and (2) together is of no more help
than either (1) or (2) taken separately because
the same examples used to show that (1) is not
sufficient also show that (2) is not sufficient.
The correct answer is E;
both statements together are still not sufficient.
289. If n is an integer, is (0.1)n greater than (10)n ?
(1)

n > –10

(2)

n < 10

309

GMAT® Official Guide 2018

Arithmetic Exponents

(0.1)n = 1 n will be greater than 10n if 1 is greater
10
than 102n, and this will be the case if and only if
n < 0. Determine if n < 0.
(1)

(2)

This indicates that n is greater than –10.
This includes nonnegative values of n as well
as negative values of n; NOT sufficient.
This indicates that n is less than 10, but
this includes negative values of n as well as
nonnegative values of n; NOT sufficient.

Taking (1) and (2) together, n can have any
value between –10 and 10 and can therefore be
negative or nonnegative.
The correct answer is E;
both statements together are still not sufficient.
290. For a basic monthly fee of F yen (¥F), Naoko’s first
cell phone plan allowed him to use a maximum of
420 minutes on calls during the month. Then, for
each of x additional minutes he used on calls, he was
charged ¥M, making his total charge for the month ¥T,
where T = F + xM. What is the value of F ?
(1)
(2)

Naoko used 450 minutes on calls the first month
and the total charge for the month was ¥13,755.
Naoko used 400 minutes on calls the second
month and the total charge for the month was
¥13,125.

Algebra Simultaneous equations

(1)

(2)

Given that Naoko used 450 minutes
and was charged ¥13,125, it follows
that F + (450 − 420)M = 13,755, or
F + 30M = 13,755. However, the value
of F cannot be determined. For example,
F = 13,005 and M = 25 is possible, and
F = 13,035 and M = 24 is possible; NOT
sufficient.

(1)

The total weight of 3 dishes of type X and
2 dishes of type Y is less than the total weight of
2 dishes of type X and 4 dishes of type Y.

(2)

The total weight of 4 dishes of type X and
3 dishes of type Y is less than the total weight of
3 dishes of type X and 4 dishes of type Y.

Algebra Inequalities

Let x be the weight of a type X dish and let y be the
weight of a type Y dish. Determine whether x < y.
(1)

Given that 3x + 2y < 2x + 4y, or x < 2y, it
is possible that x < y is true (for example, if
x = 2 and y = 3) and it is possible that x < y is
false (for example, if x = 3 and y = 2); NOT
sufficient.

(2)

Given that 4x + 3y < 3x + 4y, it follows
that x < y; SUFFICIENT.

The correct answer is B;
statement 2 alone is sufficient.
292. If r and t are three-digit positive integers, is r greater
than t ?
(1)

The tens digit of r is greater than each of the
three digits of t.

(2)

The tens digit of r is less than either of the other
two digits of r.

Arithmetic Properties of numbers

Let r3, r2, and r1 be the hundreds, tens, and units
digits, respectively, of the 3-digit integer r, and
let t3, t2, and t1 be the hundreds, tens, and units
digits, respectively, of the 3-digit integer t.
(1)

Given that r2 is greater than each of t3, t2,
and t1, then r could be greater than t (for
example, if r = 242 and t = 222) and r could
be less than t (for example, if r = 242 and
t = 333); NOT sufficient.

(2)

Given that r2 is less than either of r1 or r3,
then r could be greater than t (for example,
if r = 212 and t = 111) and r could be less
than t (for example, if r = 212 and t = 222);
NOT sufficient.

Given that Naoko used 400 minutes
and was charged ¥13,125, it follows that
F = 13,125, since Naoko did not use more
than 420 minutes; SUFFICIENT.

The correct answer is B;
statement 2 alone is sufficient.

310

291. The weights of all dishes of type X are exactly the
same, and the weights of all dishes of type Y are
exactly the same. Is the weight of 1 dish of type X
less than the weight of 1 dish of type Y ?

6.5 Data Sufficiency Answer Explanations

Taking (1) and (2) together, r3 > r2 follows from
(2) and r2 > t3 follows from (1). Therefore, r3 > t3
and hence r is greater than t.
The correct answer is C;
both statements together are sufficient.
293. Is the product of two positive integers x and y divisible
by the sum of x and y ?
(1)

x=y

(2)

x=2

Arithmetic Properties of numbers

(1)

(2)

Given that x = y, then xy could be divisible
by x + y (for example, if x = y = 2, then
xy = 4 is divisible by x + y = 4) and xy could
fail to be divisible by x + y (for example,
if x = y = 3, then xy = 9 is not divisible by
x + y = 6); NOT sufficient.
Given that x = 2, then xy could be divisible
by x + y (for example, if x = y = 2, then
xy = 4 is divisible by x + y = 4) and xy could
fail to be divisible by x + y (for example, if
x = 2 and y = 3, then xy = 6 is not divisible
by x + y = 5); NOT sufficient.

Taking (1) and (2) together, it follows that
x = y = 2, and so xy = 4 is divisible by x + y = 4.
The correct answer is C;
both statements together are sufficient.
294. A certain high school with a total enrollment of
900 students held a science fair for three days
last week. How many of the students enrolled in the high
school attended the science fair on all three days?

(1)

Given that (30%)(900) = b + c, or b + c = 270,
more than one positive integer value for c is
possible; NOT sufficient.

(2)

Given that (10%)(a + b + c) = c, or
a + b + c = 10c, or a + b = 9c, more than
one positive integer value for c is possible;
NOT sufficient.

(1) and (2) together are not sufficient because
more than one positive integer value for c is
possible. Although there are 4 variables and only
3 linear equations involving these 4 variables,
which implies there exist infinitely many real
number solutions, the variables in the present
problem are further restricted to be positive
integers. However, specific values of c can be
obtained using the equations:
I. a + b + c + n = 900 (from the information in
the question)
II. b + c = 270 (from statement (1))
III. a + b = 9c (from statement (2))
IV. 10c + n = 900 (from combining I and III)
For example, if n = 600, then c = 30 from IV,
b = 240 from II, and a = 30 from III. On the
other hand, if n = 500, then c = 40 from IV,
b = 230 from II, and a = 130 from III.
The correct answer is E;
both statements together are still not sufficient.
295. The average (arithmetic mean) price of the 3 items that
Kate purchased from a clothing store was $50. If there
was no sales tax on any item that had a price of less
than $80 and 6 percent sales tax on all other items,
what was the total sales tax on the 3 items that Kate
purchased?

(1)

Of the students enrolled in the school, 30 percent
attended the science fair on two or more days.

(1)

The price of the most expensive item that Kate
purchased from the store was $100.

(2)

Of the students enrolled in the school, 10 percent
of those that attended the science fair on at
least one day attended on all three days.

(2)

The price of the least expensive item that Kate
purchased from the store was $10.

Arithmetic Applied problems; Percents
Algebra Simultaneous equations

Let a, b, and c be the numbers of students who
attended on exactly one day, exactly two days, and
exactly three days, respectively, and let n be the
number of students who did not attend on any
of the three days. Given that a + b + c + n = 900,
determine the value of c.

Kate purchased 3 items, the average price of which
was $50, and paid 6% sales tax on each item priced
over $80. Determine the total tax that Kate paid.
Given that the average of the 3 prices is
$50, it follows that the sum of the 3 prices is
(3)($50) = $150.
311

GMAT® Official Guide 2018

(1)

(2)

One of the prices is $100. Therefore,
the sum of the 2 remaining prices is
$150 − $100 = $50. Hence, each of the
remaining items has a price that is less
than $80 and thus is not taxed. It follows
that the total sales tax on the 3 items is the
sales tax on the $100 item, which equals
(6%)($100) = $6; SUFFICIENT.

(1)

This indicates that p > q, but does not give
a specific value for p or q; NOT sufficient.

(2)

This indicates that q ≥ 100, but does not
give a specific value for q; NOT sufficient.

One of the prices is $10. Therefore,
the sum of the 2 remaining prices is
$150 − $10 = $140. If the 2 remaining
prices are $70 and $70, then none of the
items has a price greater than $80, and
hence the total sales tax on the 3 items is $0.
If the 2 remaining prices are $40 and $100,
then the total sales tax on the 3 items is
(6%)($100) = $6; NOT sufficient.

The correct answer is E;
both statements together are still not sufficient.

This solution assumes tax is not included in the
quoted prices. If tax were included in the quoted
prices, then some minor adjustments in the
numerical examples would be needed. However,
the sufficiency of (1) and insufficiency of (2)
would not be affected.
The correct answer is A;
statement 1 alone is sufficient.
296. A company makes and sells two products, P and Q.
The costs per unit of making and selling P and Q are
$8.00 and $9.50, respectively, and the selling
prices per unit of P and Q are $10.00 and $13.00,
respectively. In one month the company sold a total
of 834 units of these products. Was the total profit on
these items more than $2,000.00 ?
(1)

During the month, more units of P than units of
Q were sold.

(2)

During the month, at least 100 units of Q were sold.

Arithmetic Applied problems

It is given that the profit on each unit of P
is $10.00 − $8.00 = $2.00, and the profit on
each unit of Q is $13.00 − $9.50 = $3.50. Also
if p and q represent the number of units of
Products P and Q made and sold, then it is given
that p + q = 834. Determining if the total profit
on these items was more than $2,000 requires
determining how many units of at least one of the
products were sold.
312

Taking (1) and (2) together, p > q and q ≥ 100,
so p > 100, but a specific value for p or q cannot
be determined.

297. Jill has applied for a job with each of two different
companies. What is the probability that she will get job
offers from both companies?
(1)

The probability that she will get a job offer from
neither company is 0.3.

(2)

The probability that she will get a job offer from
exactly one of the two companies is 0.5.

Arithmetic Probability

Let P(2) be the probability that she will get
a job offer from both companies, P(1) be the
probability that she will get a job offer from
exactly one of the companies, and P(0) be the
probability that she will get a job offer from
neither company. Then P(2) + P(1) + P(0) = 1.
What is the value of P(2) ?
(1)

Given that P(0) = 0.3, it follows that
P(2) + P(1) + 0.3 = 1, or P(2) + P(1) = 0.7.
However, the value of P(2) cannot be
determined, since nothing is known about
the value of P(1) other than 0 ≤ P(1) ≤ 0.7;
NOT sufficient.

(2)

(2) Given that P(1) = 0.5, it follows that
P(2) + 0.5 + P(0) = 1, or P(2) + P(0) = 0.5.
However, the value of P(2) cannot be
determined, since nothing is known about
the value of P(0) other than 0 ≤ P(1) ≤ 0.5;
NOT sufficient.

Taking (1) and (2) together, it follows that
P(2) + 0.5 + 0.3 = 1, or P(2) = 0.2.
The correct answer is C;
both statements together are sufficient.

6.5 Data Sufficiency Answer Explanations

298. A scientist recorded the number of eggs in each of
10 birds’ nests. What was the standard deviation of the
numbers of eggs in the 10 nests?
(1)

The average (arithmetic mean) number of eggs
for the 10 nests was 4.

(2)

Each of the 10 nests contained the same
number of eggs.

moves for less than 90 = 75 seconds.
1.2
(1) This indicates that the length of time the
conveyor belt moves is less than 1.2 minutes,
which is equivalent to (1.2)(60) = 72
seconds and 72 < 75; SUFFICIENT.
(2)

Arithmetic Statistics

Note that if all of the values of a data set are equal
to the same number, say x, the difference between
each data value and the average is x − x = 0, the
sum of the squares of these differences is 0, and so
the standard deviation is 0. On the other hand, if
the values in a data set are not all equal, then the
standard deviation will be positive.
(1)

(2)

If each of the 10 nests had 4 eggs, then
the average would be 4 and the standard
deviation would be 0. If 8 nests had 4 eggs,
1 nest had 3 eggs, and 1 nest had 5 eggs,
then the average would be 4 and the
standard deviation would be positive;
NOT sufficient.
Since all of the data values are equal to the
same number, the standard deviation is 0;
SUFFICIENT.

The correct answer is B;
statement 2 alone is sufficient.

The correct answer is A;
statement 1 alone is sufficient.
300. If x, y, and z are positive numbers, what is the value of
the average (arithmetic mean) of x and z ?
(1)

x−y=y−z

(2)

x 2 − y2 = z

Algebra Statistics; Simplifying algebraic
expressions

(1)

Given that x − y = y − z, it follows that
x+z
. Therefore, y is
x + z = 2y, or y =
2
the average of x and z. However, it is not
possible to determine the value of y. For
example, x = y = z = 3 gives three positive
numbers that satisfy x − y = y − z,
and x = y = z = 4 gives three positive
numbers that satisfy x − y = y − z;
NOT sufficient.

(2)

Given that x2 − y2 = z, it is not possible to
x+z
, the average
determine the value of
2
of the x and z. For example, if x = 3,

299. A conveyor belt moves bottles at a constant speed of
120 centimeters per second. If the conveyor belt moves
a bottle from a loading dock to an unloading dock, is
the distance that the conveyor belt moves the bottle
less than 90 meters? (1 meter = 100 centimeters)
(1)

(2)

It takes the conveyor belt less than 1.2 minutes
to move the bottle from the loading dock to the
unloading dock.
It takes the conveyor belt more than 1.1 minutes
to move the bottle from the loading dock to the
unloading dock.

Arithmetic Applied problems

Since the rate at which the conveyor belt moves
is given as 120 centimeters per second, which
is equivalent to 120 = 1.2 meters per second,
100
the conveyor will move less than 90 meters if it

This indicates that the length of time
the conveyor belt moves is more than
1.1 minutes, which is equivalent to
(1.1)(60) = 66 seconds, but does not indicate
how much more than 66 seconds the
conveyor moves. If the conveyor moves for
70 seconds (70 > 66), for example, it moves
less than 90 meters since 70 < 75. However,
if the conveyor belt moves for 80 seconds
(80 > 66), then it moves more than
90 meters since 80 > 75; NOT sufficient.

y = 2, and z = 5, then x2 − y2 = z is true
(because 32 − 22 = 9 − 4 = 5) and x + z =
2
3+ 5
= 4. However, if x = 5, y = 4, and
2
z = 9, then x2 − y2 = z is true (because
313

GMAT® Official Guide 2018

x + z
=
52 − 42 = 25 − 16 = 9) and
2
5 + 9
= 7; NOT sufficient.
2
Taking (1) and (2) together, it follows
that x + z = 2y and x2 − y2 = z, so
x + (x2 − y2) = 2y, or x + x2 = y2 + 2y. Adding
1 to both sides of this last equation gives
x + x2 + 1 = y2 + 2y + 1 = (y + 1)2, which
simplifies the task of finding positive numbers
x, y, and z that satisfy both (1) and (2) and for
x + z
= y can be different.
which the values of
2
For one example, letting x = 1 gives x + x2 + 1 = 3,
and thus (y + 1)2 = 3. Solving for a positive value
of y gives y + 1 = 3 , or y = 3 − 1, and by
using x + z = 2y, the corresponding value of z is
2y − x = 2( 3 − 1) − 1 = 2 3 − 3. Note that

Geometry Rectangles; Perimeter

(1)

Given that the perimeter of the rug is 44 ft,
then the rectangular rug could have area
121 ft2 (length = 11 ft and width = 11 ft)
and the rectangular rug could have area
96 ft2 (length = 16 ft and width = 6 ft).
Moreover, no information is given about
the width of the accent border, so even if
the area of the rectangular rug could be
determined, it would still not be possible to
determine the area of the portion of the rug
that excludes the border; NOT sufficient.

(2)

Given that the width of the border is 1 ft, the
area of the portion of the rug that excludes the
border is (L − 2)(W − 2) ft2, where L ft and
W ft are the length and width, respectively, of
the rug. If L = W = 11, then (L − 2)(W − 2) = 81.
However, if L = 16 and W = 6, then
(L − 2)(W − 2) = 56; NOT sufficient.

2 3 − 3 is positive because (2 3)2 = 4(3) = 12
is greater than 32 = 9. The values x = 1, y =
3 − 1, and z = 2 3 − 3 satisfy both (1) and
(2), and for this choice of values of x, y, and z,
x+z

the value of 2 = y is 3 − 1. For another
example, letting x = 2 gives x + x2 + 1 = 7, and
thus (y + 1)2 = 7. Solving for a positive value
of y gives y + 1 = 7 , or y = 7 − 1, and by
using x + z = 2y, the corresponding value of z is
2y − x = 2( 7 − 1) − 2 = 2 7 − 4. Note that
2 7 − 4 is positive because (2 7 )2 = 4(7) = 28
is greater than 42 = 16. The values x = 2, y =
7 − 1, and z = 2 7 − 4 satisfy both (1) and
(2), and for this choice of values of x, y, and z, the
x + z
value of
= y is 7 − 1; NOT sufficient.
2
The correct answer is E;
both statements together are still not sufficient.

Taking (1) and (2) together, the area could be
81 ft2 or 56 ft2 as shown by the examples in
(1) and (2).
The correct answer is E;
both statements together are still not sufficient.
302. Terry holds 12 cards, each of which is red, white,
green, or blue. If a person is to select a card randomly
from the cards Terry is holding, is the probability less
1
than that the card selected will be either red or
2
white?
(1)

(2)

The probability that the person will select a blue
1
card is .
3
The probability that the person will select a red
1
card is .
6

Arithmetic Probability

301. The rectangular rug shown in the figure above has an
accent border. What is the area of the portion of the
rug that excludes the border?

314

(1)

The perimeter of the rug is 44 feet.

(2)

The width of the border on all sides is 1 foot.

Determine if the probability is less than 1 that a
2
card selected at random from 12 cards, each of
which is red, white, green, or blue, will be either
red or white. That is, determine whether the
number of cards that are red or white is less than
1 (12) = 6.
2

6.5 Data Sufficiency Answer Explanations

(1)

(2)

It is given that the probability is 1 that the
3
selected card will be blue. This means that
1
(12) = 4 of the cards are blue. However,
3
there is no information about how many of
the 8 remaining cards are red, how many
are white, or how many are green. If, for
example, there are 2 red cards, 2 white cards,
and 4 green cards, then the number of cards
that are red or white is 2 + 2 = 4, which is
less than 6. On the other hand, if there are
3 red cards, 4 white cards, and 1 green card,
then the number of cards that are red or
white is 3 + 4 = 7, which is not less than 6;
NOT sufficient.
It is given that the probability is 1 that the
6
selected card will be red. This means that
1 (12) = 2 of the cards are red. However,
6
there is no information about how many
of the 10 remaining cards are white, how
many are blue, or how many are green. If,
for example, there are 3 white cards, 5 blue
cards, and 2 green cards, then the number
of cards that are red or white is 2 + 3 = 5,
which is less than 6. On the other hand,
if there are 7 white cards, 1 blue card, and
2 green cards, then the number of cards that
are red or white is 2 + 7 = 9, which is not
less than 6; NOT sufficient.

Taking (1) and (2) together, there are 4 blue cards
and 2 red cards. However, there is no information
about how many of the 6 remaining cards are
white or how many are green. If, for example,
there are 3 white cards and 3 green cards, then
the number of cards that are red or white is
2 + 3 = 5, which is less than 6. On the other
hand, if there are 5 white cards and 1 green card,
then the number of cards that are red or white is
2 + 5 = 7, which is not less than 6.
The correct answer is E;
both statements together are still not sufficient.
2xz + yz
303. If y ≠ 2xz, what is the value of
?
2xz − y
(1)

2x + y = 3

(2)

z=2

Algebra Simplifying algebraic expressions

The task is to determine the value of
2xz + yz z(2x + y )
.
=
2xz − y
2xz − y
(1)

(2)

2xz + yz
=
This indicates 2x + y = 3, and so
2xz − y
3z
. However, this is not enough
2xz − y
2xz + yz
. For
to determine the value of
2xz − y
example, if x = 1, y = 1, and z = 2, then
2xz + yz
2x + y = 3 and
= 2, but if x = 0,
2xz − y
y = 3, and z = 2, then 2x + y = 3 and
2xz + yz
= –2; NOT sufficient.
2xz − y
2xz + yz
=
This indicates that z = 2, and so
2xz − y
4x + 2 y
. However, this is not enough
4x − y
2xz + yz
to determine the value of
. For
2xz − y
example, if x = 1, y = 1, and z = 2, then

2xz + yz
= 2, but if x = 0, y = 3, and z = 2,
2xz − y
2xz + yz
then
= –2; NOT sufficient.
2xz − y
Taking (1) and (2) together is not enough to
2xz + yz
because the
determine the value of
2xz − y
same examples used to show that (1) is not
sufficient were also used to show that (2) is not
sufficient.
The correct answer is E;
both statements together are still not sufficient.
y°

x°

z°
304. In the parallelogram shown, what is the value of x ?
(1)

y = 2x

(2)

x + z = 120

315

GMAT® Official Guide 2018

Geometry Angles

(1)

(2)

(1)

Given that y = 2x and the fact that adjacent
angles of a parallelogram are supplementary,
it follows that 180 = x + y = x + 2x = 3x, or
180 = 3x. Solving this equation gives x = 60;
SUFFICIENT.
Given that x + z = 120 and the fact
that opposite angles of a parallelogram
have the same measure, it follows that
120 = x + z = x + x, or 120 = 2x. Solving
this equation gives x = 60; SUFFICIENT.

side effects

(1)

Of the 1,000 patients tested, 15 percent
experienced neither side effects nor relief of
cold symptoms.

(2)

Of the patients tested, 30 percent experienced
relief of cold symptoms without side effects.

One way to solve problems about sets is by using
a contingency table. For the information given
in this problem with T representing the total
number of people tested, the table might be as
follows:

1,000

side effects

no side effects

totals

relief of
symptoms

250

300

550

no relief of
symptoms

300

150

450

totals

550

450

1,000

250
= 25% as the percent
1, 000
of patients tested who experienced both side
effects and relief of cold symptoms.
and this gives

totals
⎛ y ⎞
⎜⎝
⎟T
100 ⎠

However, if x = 55 and y = 50, then the table
becomes
side effects

no side effects

totals

relief of
symptoms

200

300

500

no relief of
symptoms

350

150

500

totals

550

450

1,000

200
= 20% as the percent
1, 000
of patients tested who experienced both side
effects and relief of cold symptoms;
NOT sufficient.
and this gives

⎛ x ⎞T
⎝ 100 ⎠

T
(2)

316

150
⎛ x ⎞ (1, 000)
⎝ 100 ⎠

totals

no relief of
symptoms
totals

⎛ y ⎞
⎜⎝
⎟ (1, 000)
100 ⎠

no relief of
symptoms

Algebra Sets

relief of
symptoms

totals

If x = 55 and y = 55, then the table becomes

305. In a product test of a common cold remedy, x percent
of the patients tested experienced side effects from
the use of the drug and y percent experienced relief
of cold symptoms. What percent of the patients tested
experienced both side effects and relief of cold
symptoms?

no side effects

no side
effects

relief of
symptoms

The correct answer is D;
each statement alone is sufficient.

side effects

Using the information in (1) gives the
following table, where 15% of 1,000 is 150.

This indicates that 30% of the patients
tested experienced relief of cold symptoms
without side effects. The same examples that
were used for (1) can be used here, also, to
show that different values can be obtained

6.5 Data Sufficiency Answer Explanations

for the percent of patients tested who
experienced both side effects and relief of
cold symptoms; NOT sufficient.
Since the examples satisfy both (1) and (2), these
statements taken together are not sufficient to
determine the percent of patients tested who
experienced both side effects and relief of cold
symptoms.
The correct answer is E;
both statements together are still not sufficient.

x2 > 5

(2)

x2 + x < 5

y 1
= , or x = 4y, it is possible
x 4
that x + y = 15 (for example, x = 12 and
y = 3) and it is possible that x + y ≠ 15 (for
example, x = 4 and y = 1); NOT sufficient.
Given that

Given (1) and (2) together, x − y = 9 and
x = 4y, it follows that 4y − y = 9, or y = 3, and
x = 4y = 4(3) = 12. Therefore, x + y = 12 + 3 = 15.
The correct answer is C;
both statements together are sufficient.
308. Is zp negative?

306. Is x < 5 ?
(1)

(2)

(1)

pz 4 < 0

(2)

p + z 4 = 14

Arithmetic Properties of numbers
Algebra Inequalities

(1)

Given that > 5, then x < 5 can be true
(for example, if x = –3) and x < 5 can be
false (for example, if x = 6); NOT sufficient.

(2)

Given that x2 + x < 5, then x < 5 − x2. Also,
since x2 ≥ 0 is true for any real number x,
it follows that –x2 ≤ 0, or after adding 5
to both sides, 5 − x2 ≤ 5. From x < 5 − x2
and 5 − x2 ≤ 5, it follows that x < 5;
SUFFICIENT.

The correct answer is B;
statement 2 alone is sufficient.
307. Is 4x + y = 810 ?
(1)
(2)

(1)

Given that pz4 < 0, zp can be negative
(choose p = –2 and z = 2) and zp can be
nonnegative (choose p = –2 and z = –2);
NOT sufficient.

(2)

Given that p + z4 = 14, the same examples
used to show that (1) is not sufficient can be
used for (2); NOT sufficient.

x2

x−y=9
y 1
=
x 4

Taking (1) and (2) together is of no more help
than either (1) or (2) taken separately because
the same examples used to show that (1) is not
sufficient also show that (2) is not sufficient.
The correct answer is E;
both statements together are still not sufficient.
309. In each game of a certain tournament, a contestant
either loses 3 points or gains 2 points. If Pat had
100 points at the beginning of the tournament, how
many games did Pat play in the tournament?

Algebra Properties of numbers

(1)

At the end of the tournament, Pat had 104 points.

The question is equivalent to determining
whether (22)x + y = (23)10, which in turn is
equivalent to determining whether 22(x + y) = 230,
which in turn is equivalent to determining
whether 2(x + y) = 30, or x + y = 15.

(2)

Pat played fewer than 10 games.

(1)

Given that x − y = 9, it is possible that
x + y = 15 (for example, x = 12 and y = 3)
and it is possible that x + y ≠ 15 (for example,
x = 10 and y = 1); NOT sufficient.

Arithmetic Computation with integers

Pat either lost 3 points or gained 2 points in
each game she played. Therefore, since she had
100 points at the beginning of the tournament,
her score at the end of the tournament was
100 − 3x + 2y, where x and y represent,
respectively, the number of games in which she
lost 3 points and the number of games in which
she gained 2 points. Determine the value of x + y.
317

GMAT® Official Guide 2018

(1)

(2)

This indicates that 100 − 3x + 2y = 104, but
no information is given about the values of x
or y. For example, the values of x and y could
be 0 and 2, respectively, so that x + y = 2
or the values of x and y could be 2 and 5,
respectively, so that x + y = 7; NOT sufficient.
This indicates that Pat played fewer than
10 games, but in each of the examples
above, Pat played fewer than 10 games;
NOT sufficient.

Since the examples given satisfy both (1) and (2),
taking (1) and (2) together does not give enough
information to determine the value of x + y.

311. Can a certain rectangular sheet of glass be positioned
on a rectangular tabletop so that it covers the entire
tabletop and its edges are parallel to the edges of the
tabletop?
(1)

The tabletop is 36 inches wide by 60 inches long.

(2)

The area of one side of the sheet of glass is
2,400 square inches.

Geometry Area

Determine whether the length and width of the
sheet of glass are greater than or equal to the
length and width of the tabletop.
(1)

The length and width of the tabletop are
given, but nothing can be determined about
the dimensions of the sheet of glass; NOT
sufficient.

(2)

The area of the sheet of glass is given,
but nothing can be determined about the
dimensions of the tabletop; NOT sufficient.

The correct answer is E;
both statements together are still not sufficient.
310. At the beginning of the year, the Finance Committee
and the Planning Committee of a certain company
each had n members, and no one was a member of
both committees. At the end of the year, 5 members
left the Finance Committee and 3 members left the
Planning Committee. How many members did the
Finance Committee have at the beginning of the year?
(1)

The ratio of the total number of members who
left at the end of the year to the total number of
members at the beginning of the year was 1:6.

(2)

At the end of the year, 21 members remained on
the Planning Committee.

Algebra Applied problems

(1)

(2)

It is given that 8 = 1 , since a total of
2n 6
3 + 5 = 8 members left at the end of the
year and there were a total of n + n =
2n members at the beginning of the year.
It follows that (8)(6) = (2n)(1), or n = 24;
SUFFICIENT.
It is given that n − 3 = 21, since 3 members
from the original n members left the
Planning Committee at the end of the
year. It follows that n = 21 + 3 = 24;
SUFFICIENT.

The correct answer is D;
each statement alone is sufficient.

318

(1) and (2) together are not sufficient because the
information given does not specify the length and
width of the sheet of glass. For example, the length
and width of the sheet of glass could be 40 inches
and 60 inches and, since 40 ≥ 36 and 60 ≥ 60, the
answer to the question would be “Yes.” On the
other hand, the length and width of the sheet of
glass could be 100 inches and 24 inches, and in
this case the answer to the question would be “No”
because 24 is not greater than or equal to 36 or 60.
The correct answer is E;
both statements together are still not sufficient.
312. In a random sample of 80 adults, how many are
college graduates?
(1)

In the sample, the number of adults who are not
college graduates is 3 times the number who are
college graduates.

(2)

In the sample, the number of adults who are not
college graduates is 40 more than the number
who are college graduates.

Algebra First-degree equations

Let C be the number of college graduates and let
N be the number who are not college graduates.
Then, C + N = 80. Find the value of C.

6.5 Data Sufficiency Answer Explanations

(1)

Since the number who are not college
graduates is 3 times the number who are,
N = 3C. Then C + 3C = 80, 4C = 80, and
C = 20; SUFFICIENT.

(2)

Since the number who are not college
graduates is 40 more than the number who
are college graduates, N = C + 40. Then
C + (C + 40) = 80, 2C + 40 = 80, 2C = 40,
and C = 20; SUFFICIENT.

The correct answer is D;
each statement alone is sufficient.
313. Max purchased a guitar for a total of $624, which
consisted of the price of the guitar and the sales tax.
Was the sales tax rate greater than 3 percent?
(1)

The price of the guitar that Max purchased was
less than $602.

(2)

The sales tax for the guitar that Max purchased
was less than $30.

However, it is also possible that the sales
tax is less than $30 and the sales tax rate is
4% (which is greater than 3%), since a 4%
sales tax rate implies P ⎛ 1 + 4 ⎞ = 624, or
⎝ 100 ⎠
62, 400
P=
= 600, and thus the sales tax
104
would be 0.04($600) = $24 < $30; NOT
sufficient.
The correct answer is A;
statement 1 alone is sufficient.
314. What is the sum of a certain pair of consecutive odd
integers?
(1)

At least one of the integers is negative.

(2)

At least one of the integers is positive.

Arithmetic Properties of numbers

(1)

Given that at least one of the integers
is negative, the sum could be –4 (if the
integers were –3 and –1) and the sum could
be 0 (if the integers were –1 and 1); NOT
sufficient.

(2)

Given that at least one of the integers is
positive, the sum could be 0 (if the integers
were –1 and 1) and the sum could be 4
(if the integers were 1 and 3); NOT
sufficient.

Arithmetic Applied problems; Percents

Letting P be the price, in dollars, of the guitar
and r% be the sales tax rate, it is given that
P ⎛ 1 + r ⎞ = 624. Determine if r > 3.
⎝ 100 ⎠
(1)

Given that P < 602, then 624 =
P ⎛ 1 + r ⎞ < 602 ⎛ 1 + r ⎞ , and so
⎝ 100 ⎠
⎝ 100 ⎠
r
624
>
. Therefore, r >
1+
100 602
100
11
3
624
>
, because
− 1 = 22 =
301
100
602
602
(11)(100) > (3)(301), and so r >
100 ⎛ 3 ⎞ = 3; SUFFICIENT.
⎝ 100 ⎠

(2)

Given that the sales tax was less than $30,
it is not possible to determine whether the
sales tax rate was greater than 3%. Since
(0.03)($624) = $18.72, a 3% sales tax rate
on the price corresponds to a sales tax that
is less than $18.72, since the price is less
than $624. Therefore, it is possible that the
sales tax is less than $30 and the sales tax
rate is less than 3%, since any sales tax rate
less than 3% would give a sales tax that is
less than $18.72 (and hence less than $30).

Taking (1) and (2) together, the smaller of the
two numbers cannot be less than –1 (otherwise
(2) would not be true) and the larger of the two
numbers cannot be greater than 1 (otherwise (1)
would not be true). Therefore, the integers must
be –1 and 1, and the sum must be 0.
The correct answer is C;
both statements together are sufficient.
315. If a < x < b and c < y < d, is x < y ?
(1)

a y, since 10 > 6.
If, on the other hand, a = 3, x = 4, b = 12,
c = 4, y = 6, and d = 7, a < c, because 3 < 4 and
x < y, since 4 < 6; NOT sufficient.
If b < c, then a < x < b < c < y < d and so
x < y; SUFFICIENT.

(1)

This indicates that the integers are
consecutive odd integers. Letting z represent
the greatest of the four integers, it follows
that (z − 6) + (z − 4) + (z − 2) + z = 64,
from which a unique value of z can be
determined; SUFFICIENT.

(2)

Letting w, x, y, and z represent four
different odd integers, where w < x < y < z,
this indicates that z − w = 6 or w = z − 6.
This means that x = z − 4 and y = z − 2 since
w, x, y, and z must be different odd integers
and it must be true that w < x < y < z.
From w + x + y + z = 64, it follows
that (z − 6) + (z − 4) + (z − 2) + z = 64
from which a unique value of z can be
determined; SUFFICIENT.

The correct answer is B;
statement 2 alone is sufficient.
316. How many people are directors of both Company K
and Company R ?
(1)

There were 17 directors present at a joint
meeting of the directors of Company K and
Company R, and no directors were absent.

(2)

Company K has 12 directors and Company R
has 8 directors.

Algebra Sets

(1)

(2)

This clarifies that Company K and
Company R together have 17 individuals
serving as directors. However, there is no
information as to the distribution of the
Company K directors, the Company R
directors, and the joint directors; NOT
sufficient.

The correct answer is D;
each statement alone is sufficient.
318. Was the number of books sold at Bookstore X last
week greater than the number of books sold at
Bookstore Y last week?
(1)

Last week, more than 1,000 books were sold
at Bookstore X on Saturday and fewer than
1,000 books were sold at Bookstore Y on
Saturday.

(2)

Last week, less than 20 percent of the books
sold at Bookstore X were sold on Saturday
and more than 20 percent of the books sold at
Bookstore Y were sold on Saturday.

This gives the number of directors in each
company but no information as to the
number of joint directors; NOT sufficient.

Taking (1) and (2) together, it is known from (2)
that there are 20 directorships in all. If at a joint
meeting, there are only 17 people present, then
20 − 17 = 3 people must be joint directors.

Arithmetic Inequalities

Determine if Bookstore X sold more books last
week than Bookstore Y.
(1)

This indicates that Bookstore X sold more
books on Saturday than Bookstore Y, but
it gives no information about the numbers
of books sold by the two bookstores on the
other days of last week; NOT sufficient.

(2)

This gives information about the percents
of the books sold last week that were sold
on Saturday, but it gives no information
about the actual numbers of books sold at

The correct answer is C;
both statements together are sufficient.
317. The sum of 4 different odd integers is 64. What is the
value of the greatest of these integers?

320

(1)

The integers are consecutive odd numbers.

(2)

Of these integers, the greatest is 6 more than
the least.

6.5 Data Sufficiency Answer Explanations

the two bookstores last week. Therefore, a
comparison of the numbers of books sold
last week by Bookstore X and Bookstore Y
is not possible; NOT sufficient.
Taking (1) and (2) together, if x represents the
number of books that Bookstore X sold on
Saturday and TX represents the total number
of books that Bookstore X sold last week,
then x > 1,000 and x < 0.2TX. It follows that
1,000 < 0.2TX and so TX > 5,000. Similarly, if
y represents the number of books that Bookstore Y
sold on Saturday and T Y represents the total
number of books that Bookstore Y sold last week,
then y < 1,000 and y > 0.2T Y. It follows that
0.2T Y < 1,000, and so T Y < 5,000. Combining
the inequalities gives T Y < 5,000 < TX, and so
T Y < TX, which means that Bookstore X sold
more books last week than Bookstore Y.

Taking (1) and (2) together, it follows that the
amount on May 30 was ($450)(1.08) = $486.
The correct answer is C;
both statements together are sufficient.
320. If x and y are positive, is x < 10 < y ?
(1)

x < y and xy = 100

(2)

x2 < 100 < y2

Algebra Inequalities

(1)

The correct answer is C;
both statements together are sufficient.
319. From May 1 to May 30 in the same year, the balance in
a checking account increased. What was the balance in
the checking account on May 30 ?
(1)

(2)

If, during this period of time, the increase in
the balance in the checking account had been
12 percent, then the balance in the account on
May 30 would have been $504.
During this period of time, the increase in the
balance in the checking account was 8 percent.

Arithmetic Applied problems; Percents

(1)

(2)

Given that the amount on May 30 would
have been $504 if the increase had been
12%, it follows that the amount on May 1
504
= $450. However, the balance on
was $
1.12
May 30 cannot be determined because the
(actual) percent increase is not given. For
example, if the increase had been 10%, then
the balance on May 30 would have been
(1.1)($450) = $495, which is different from
$504; NOT sufficient.
Given that the increase was 8%, the amount
on May 30 could be $108 (if the amount on
May 1 was $100) and the amount on May 30
could be $216 (if the amount on May 1 was
$200); NOT sufficient.

Given that x < y, multiply both sides by
x, which is positive, to get x2 < xy. Then,
since xy = 100, it follows that x2 < 100.
Similarly, multiply both sides of x < y by
y, which is positive, to get xy < y2. Again,
since xy = 100, it follows that 100 < y2.
Combining x2 < 100 and 100 < y2 gives
x2 < 100 < y2, from which it follows that
x 2 < 100 < y 2 and, therefore,
x < 10 < y, since x and y are both positive;
SUFFICIENT.

(2)

Given that x2 < 100 < y2, it follows
that x < 10 < y as shown in (1) above;
SUFFICIENT.

The correct answer is D;
each statement alone is sufficient.
321. A merchant discounted the sale price of a coat and the
sale price of a sweater. Which of the two articles of
clothing was discounted by the greater dollar amount?
(1)

The percent discount on the coat was
2 percentage points greater than the
percent discount on the sweater.

(2)

Before the discounts, the sale price of the coat
was $10 less than the sale price of the sweater.

Arithmetic Applied problems; Percents

(1)

Given that the discount on the coat was
2 percentage points greater than the
discount on the sweater, the sweater could
have been discounted by the greater dollar
amount ($30 sweater with 10% discount is a
discount of $3; $20 coat with 12% discount
is a discount of $2.40) and the coat could
have been discounted by the greater dollar
amount ($110 sweater with 10% discount is a
321

GMAT® Official Guide 2018

discount of $11; $100 coat with 12% discount
is a discount of $12); NOT sufficient.
(2)

Given that the coat’s sale price was
$10 less than the sweater’s sale price, the
same examples used to show that (1) is
not sufficient can be used for (2); NOT
sufficient.

Taking (1) and (2) together is of no more help
than either (1) or (2) taken separately because
the same examples used to show that (1) is not
sufficient also show that (2) is not sufficient.

323. If the positive integer n is added to each of the
integers 69, 94, and 121, what is the value of n ?
(1)

69 + n and 94 + n are the squares of two
consecutive integers.

(2)

94 + n and 121 + n are the squares of two
consecutive integers.

Algebra Computation with integers

Determine the value of the positive integer n.
(1)

This indicates that 69 + n and 94 + n
are the squares of two consecutive
integers. Letting x and (x + 1) represent
the consecutive integers, it follows that
(x + 1)2 − x2 = x2 + 2x + 1 − x2 = 2x + 1.
Therefore, (94 + n) − (69 + n) = 2x + 1
or 25 = 2x + 1, from which x = 12. Then
122 = 69 + n and n = 144 − 69 = 75 or
(12 + 1)2 = 94 + n and n = 169 − 94 = 75;
SUFFICIENT.

(2)

This indicates that 94 + n and 121 + n
are the squares of two consecutive
integers. Letting x and (x + 1) represent
the consecutive integers, it follows that
(x + 1)2 − x2 = x2 + 2x + 1 − x2 = 2x + 1.
Therefore, (121 + n) − (94 + n) = 2x + 1
or 27 = 2x + 1, from which x = 13. Then
132 = 94 + n and n = 169 − 94 = 75 or
(13 + 1)2 = 121 + n and n = 196 − 121 = 75;
SUFFICIENT.

The correct answer is E;
both statements together are still not sufficient.
322. A taxi company charges f cents for the first mile of
the taxi ride and m cents for each additional mile. How
much does the company charge for a 10‐mile taxi ride?
(1)

The company charges $0.90 for a 2‐mile ride.

(2)

The company charges $1.20 for a 4‐mile ride.

Arithmetic Applied problems

If a taxi company charges f cents for the first mile
and m cents for each additional mile, determine
the charge for a 10‐mile taxi ride, which can be
expressed as f + 9m
(1)

(2)

Since the charge for a 2‐mile ride is $0.90,
f + m = 0.90, and so f + 9m = 0.90 + 8m,
but the value of m is unknown; NOT
sufficient.
Since the charge for a 4‐mile ride is $1.20,
f + 3m = 1.20, and so f + 9m =1.20 + 6m, but
the value of m is unknown; NOT sufficient.

Taking (1) and (2) together and subtracting the
equation in (1) from the equation in (2) gives
2m = 0.30 from which m = 0.15. Then from (1),
f + 0.15 = 0.90 and f = 0.75. Therefore, the charge
for a 10‐mile taxi ride is $0.75 + 9($0.15) = $2.10.
Alternatively, the graphs of f + m = 0.90 and
f + 3m = 1.20 in the (f,m) coordinate plane are
lines that intersect at exactly one point, and
therefore values of f and m can be determined,
from which f + 9m can then be determined.
The correct answer is C;
both statements together are sufficient.
322

The correct answer is D;
each statement alone is sufficient.
324. Guy’s net income equals his gross income minus his
deductions. By what percent did Guy’s net income
change on January 1, 1989, when both his gross
income and his deductions increased?
(1)

Guy’s gross income increased by 4 percent on
January 1, 1989.

(2)

Guy’s deductions increased by 15 percent on
January 1, 1989.

Arithmetic Percents

Let gb represent Guy’s gross income and db
Guy’s deductions before January 1, 1989. Let
ga represent Guy’s gross income and da Guy’s
deductions after January 1, 1989. Determine by
what percent Guy’s net income (gross income

6.5 Data Sufficiency Answer Explanations

minus deductions) changed on January 1, 1989,
when both his gross income and his deductions
( g − da ) − ( gb − db )
as a percent.
increased, or find a
gb − db
(1)

(2)

Guy’s gross income increased by 4 percent
on January 1, 1989, so ga = 1.04 gb. Then,
( g a − d a ) − ( g b − d b ) (1.04 g b − d a ) − ( g b − d b )
=
,
gb − db
gb − db
However, no information is given
about the values of gb , da , or db , and so
( ga − da ) − ( gb − db )
cannot be determined;
gb − db
NOT sufficient.
Guy’s deductions increased by 15 percent
on January 1, 1989, so d a = 1.15d b.

Algebra Ratios

TM TP
=
, where TM and SM are the
S M SP
numbers of teachers and students, respectively,
in District M, and TP and SP are the numbers of
teachers and students, respectively, in District P,
S
find the value of M .
SP
Given that

(1)

unknown; NOT sufficient.
(2)

Taking (1) and (2) together gives
(1.04 g b − 1.15d b ) − ( g b − d b ) 0.04 g b − 0.15d b
,
=
gb − db
gb − db
which cannot be determined since the values of
gb and db are unknown.
The correct answer is E;
both statements together are still not sufficient.
325. If the ratio of the number of teachers to the number of
students is the same in School District M and School
District P, what is the ratio of the number of students in
School District M to the number of students in School
District P ?
(1)

There are 10,000 more students in School
District M than there are in School District P.

(2)

The ratio of the number of teachers to the number
of students in School District M is 1 to 20.

Given that

TM
T
T
= 1 and M = P ,
S M 20
S M SP

TP
= 1 . Therefore, SP = 20TP and
SP 20
S
20TM
SM = 20TM. It follows that M =
,
SP
20TP
but the values of TM and TP are unknown;
then

( g − da ) − ( gb − db )
Then, a
=
gb − db
( g a − 1.15d b ) − ( g b − d b )
. However, no
gb − db
information is given about the values of ga ,
( g − da ) − ( gb − db )
gb , or db , and so a
gb − db
cannot be determined; NOT sufficient.

Given that S M = SP + 10, 000, then
S M SP + 10, 000
, but the value of SP is
=
SP
SP

NOT sufficient.
Taking (1) and (2) together, if SP = 1,000,
1, 000
SM = 1,000 + 10,000 = 11,000, TP =
= 50,
20
S
11, 000
11
and TM =
= 550, then M = . However,
SP
1
20
if SP = 5, 000, S M = 5, 000 + 10, 000 = 15, 000,
15, 000
5, 000
= 750, then
TP =
= 250, and TM =
20
20
SM 3
S
= . Therefore, the value of M cannot be
SP 1
SP
determined.
The correct answer is E;
both statements together are still not sufficient.
326. For a certain city’s library, the average cost of
purchasing each new book is $28. The library
receives $15,000 from the city each year; the library
also receives a bonus of $2,000 if the total number
of items checked out over the course of the year
exceeds 5,000. Did the library receive the bonus last
year?
(1)

The library purchased an average of 50 new
books each month last year and received enough
money from the city to cover this cost.
323

GMAT® Official Guide 2018

(2)

The lowest number of items checked out in one
month was 459.

(2)

Arithmetic Applied problems

(1)

(2)

Given that the library purchased an average
of 50 new books each month, for the entire
year the library purchased a total of (50)(12) =
600 books for a total cost of (600)($28) =
$16,800. Excluding any possible bonus,
the library received $15,000 from the city.
Since this amount received from the city is
not enough to cover the cost of the books,
and the information provided in (1) says
that the total amount received from the city
was enough to cover the cost of the books,
it follows that the library received a bonus;
SUFFICIENT.
Given that the least number of books
checked out in one month was 459, it follows
that the total number of books checked out
for the year was at least (12)(459) = 5,508.
Since this is greater than 5,000, it follows
that the total number of books checked out
for the year was greater than 5,000 and the
library received a bonus; SUFFICIENT.

The correct answer is D;
each statement alone is sufficient.
327. Each gift certificate sold yesterday by a certain
bookstore cost either $10 or $50. If yesterday the
bookstore sold more than 5 gift certificates that
cost $50 each, what was the total number of gift
certificates sold yesterday by the bookstore?
(1)

Yesterday the bookstore sold fewer than 10 gift
certificates that cost $10 each.

(2)

The total cost of gift certificates sold yesterday
by the bookstore was $460.

Arithmetic Computation with integers

Let x be the number of $10 gift certificates sold
yesterday and let y be the number of $50 gift
certificates sold yesterday. It is given that y > 5.
Determine the value of x + y.
(1)

324

It is given that x < 10, so x could be 0, 1, 2,
3, . . ., 9. Since y > 5, y could be 6, 7, 8, . . .,
and then x + y could be, for example,
0 + 6 = 6 or 2 + 8 = 10; NOT sufficient.

It is given that 10x + 50y = 460. Several
values of x and y with y > 5 will satisfy this
equation and give different values for x + y,
as shown below; NOT sufficient.
x

y

10x + 50y

x+y

16

6

460

22

11

7

460

18

6

8

460

14

1

9

460

10

Taking (1) and (2) together, it can be seen from
the table that all the requirements are satisfied
by x = 6 and y = 8 and also by x = 1 and y = 9,
but the value of x + y is not uniquely determined;
NOT sufficient.
The correct answer is E;
both statements together are still not sufficient.
328. What is the tens digit of positive integer x ?
(1)

x divided by 100 has a remainder of 30.

(2)

x divided by 110 has a remainder of 30.

Arithmetic Properties of numbers

(1)

Having a remainder of 30 when x is divided
by 100 can only happen if x has a tens digit
of 3 and a ones digit of 0, as in 130, 230,
630, and so forth; SUFFICIENT.

(2)

When 140 is divided by 110, the quotient is
1 R30. However, 250 divided by 110 yields
a quotient of 2 R30, and 360 divided by
110 gives a quotient of 3 R30. Since there
is no consistency in the tens digit, more
information is needed; NOT sufficient.

The correct answer is A;
statement 1 alone is sufficient.
329. Three dice, each of which has its 6 sides numbered
1 through 6, are tossed. The sum of the 3 numbers that
are facing up is 12. Is at least 1 of these numbers 5 ?
(1)

None of the 3 numbers that are facing up is
divisible by 3.

(2)

Of the numbers that are facing up, 2, but not all
3, are equal.

6.5 Data Sufficiency Answer Explanations

Arithmetic Properties of integers

When three dice, each with its 6 faces numbered
1 through 6, are tossed and the sum of the three
integers facing up is 12, the possible outcomes are
{1, 5, 6}, {2, 4, 6}, {2, 5, 5}, {3, 3, 6}, {3, 4, 5}, and
{4, 4, 4}. Determine if, in the outcome described,
at least one of the numbers is 5.
(1)

This indicates that none of the numbers
is divisible by three. The outcome could
be {4, 4, 4}. In this case, none of the three
numbers is 5. On the other hand, the
outcome could be {2, 5, 5}. In this case,
at least one of the numbers is 5;
NOT sufficient.

(2)

This indicates that two of the numbers,
but not all three, are equal. The outcome
could be {3, 3, 6}. In this case, none of the
three numbers is 5. On the other hand, the
outcome could be {2, 5, 5}. In this case,
at least one of the numbers is 5; NOT
sufficient.

Taking (1) and (2) together, of the possible
outcomes {1, 5, 6}, {2, 4, 6}, {3, 3, 6}, and {3, 4, 5}
are eliminated because they do not satisfy (1), and
{4, 4, 4} is eliminated because it does not satisfy
(2). This leaves only {2, 5, 5}, and at least one
number is 5.
The correct answer is C;
both statements together are sufficient.

(2)

Taking (1) and (2) together, if r − t ≥ 0,
then r − t = r2 and t = r − r2 = r(1 − r). Since
–1 < r < 0, it follows that 1 < 1 − r < 2, and
so t is the product of a negative number
(r) and a positive number (1 − r) and is
therefore negative. Similarly, if r − t < 0,
then |r − t| = –(r − t) = t − r = r2 and
t = r2 + r = r(r + 1). Since –1 < r < 0, it follows
that 0 < r + 1 < 1, and so t is the product of a
negative number (r) and the positive number
(r + 1) and is therefore negative.
The correct answer is C;
both statements together are sufficient.
331. S is a set of points in the plane. How many distinct
triangles can be drawn that have three of the points in
S as vertices?
(1)

The number of distinct points in S is 5.

(2)

No three of the points in S are collinear.

Arithmetic Elementary combinatorics

(1)

Given that the number of points in S is 5,
the number of triangles can be 0 (if the
points are collinear) and the number of
triangles can be greater than 0 (if the points
are not all collinear); NOT sufficient.

(2)

Given that no three points of S are collinear,
the number of triangles can be 1 (if S
consists of 3 points) and the number of
triangles can be 4 (if S consists of 4 points);
NOT sufficient.

330. On the number line, point R has coordinate r and point T
has coordinate t. Is t < 0 ?
(1)

–1 < r < 0

(2)

The distance between R and T is equal to r2.

Arithmetic Number line

Determine if t, the coordinate of point T on the
number line, is negative.
(1)

This indicates that r, the coordinate of
point R on the number line, lies between
–1 and 0, but gives no information about
the location of T in relation to either
point R or 0; NOT sufficient.

This indicates that the distance between
R and T, which can be expressed as |r − t|,
is r2. If r = 2, for example, then |2 − t| = 4
so 2 − t = 4 or 2 − t = –4. If 2 − t = 4, then
t = –2, which is negative. If 2 − t = –4, then
t = 6, which is not negative; NOT sufficient.

Taking (1) and (2) together, the number of
5!
= 10,
distinct triangles must be ⎛ 5 ⎞ =
⎝ 3 ⎠ 3!(5 − 3)!
which is the number of combinations of 5 points
taken 3 at a time.
The correct answer is C;
both statements together are sufficient.

325

GMAT® Official Guide 2018

332. Stores L and M each sell a certain product at a
different regular price. If both stores discount their
regular price of the product, is the discount price at
Store M less than the discount price at Store L ?
(1)

At Store L the discount price is 10 percent
less than the regular price; at Store M the
discount price is 15 percent less than the regular
price.

(2)

At Store L the discount price is $5 less than the
regular store price; at Store M the discount price
is $6 less than the regular price.

Arithmetic Percents

Let Lr and Ld be the regular and discounted
prices, respectively, at Store L, and let Mr and Md
be the regular and discounted prices, respectively,
at Store M. Determine if Md < Ld.
(1)

Knowing that Ld = (1 − 0.10)Lr = 0.90Lr
and that Md = (1 − 0.15)Mr = 0.85Mr gives
no information for comparing Md and Ld ;
NOT sufficient.

(2)

Knowing that Ld = Lr – 5 and that
Md = Mr − 6 gives no information for
comparing Md and Ld ; NOT sufficient.

Taking (1) and (2) together gives 0.90Lr = Lr − 5
and 0.85 Mr = Mr − 6, from which it follows that
0.10Lr = 5 or Lr = 50 and 0.15Mr = 6 or Mr = 40.
Then Ld = 50 − 5 = 45 and Md = 40 − 6 = 34.
Therefore, Md < Ld.
The correct answer is C;
both statements together are sufficient.
333. If d denotes a decimal, is d ≥ 0.5 ?
(1)

When d is rounded to the nearest tenth, the
result is 0.5.

(2)

When d is rounded to the nearest integer,
the result is 1.

Arithmetic Rounding; Estimating

(1)

326

In this case, for example, the value of d
could range from the decimal 0.45 to 0.54.
Some of these, such as 0.51 or 0.52, are
greater than or equal to 0.5, and others,
such as 0.47 or 0.48, are less than 0.5;
NOT sufficient.

(2)

When the result of rounding d to the
nearest integer is 1, d could range in value
from the decimal 0.50 to 1.49, which are
greater than or equal to 0.5; SUFFICIENT.

The correct answer is B;
statement 2 alone is sufficient.
334. In the two-digit integers 3■ and 2▲, the symbols ■ and
▲ represent different digits, and the product (3■)(2▲)
is equal to 864. What digit does ■ represent?
(1)

The sum of ■ and ▲ is 10.

(2)

The product of ■ and ▲ is 24.

Arithmetic Properties of numbers

(1)

Given that ■ + ▲ = 10, then the possible
products ■▲ are (1)(9) = (9)(1) = 9,
(2)(8) = (8)(2) = 16, (3)(7) = (7)(3) = 21,
(4)(6) = (6)(4) = 24, and (5)(5) = 25. Since
(3■)(2▲) = 864 has units digit 4, it follows
that ■▲ must also have units digit 4. Since
only two of the possible products of ■▲ has
units digit 4, it follows that ■ = 4 and ▲ = 6,
or ■ = 6 and ▲ = 4. If ■ = 4 and ▲ = 6, then
(3■)(2▲) = (34) (26) = 884 ≠ 864, and if
■ = 6 and ▲ = 4, then (3■)(2▲) = (36)(24) =
864. Therefore, ■ = 6; SUFFICIENT.

(2)

Given that ■▲ = 24, and that both ■ and ▲
are digits, there are four possibilities for the
values of ■ and ▲.
(a) ■ = 3 and ▲ = 8, which gives (3■)(2▲) =
(33)(28) = 924 ≠ 864;
(b) ■ = 4 and ▲ = 6, which gives (3■)(2▲) =
(34)(26) = 884 ≠ 864;
(c) ■ = 6 and ▲ = 4, which gives (3■)(2▲) =
(36)(24) = 864;
(d) ■ = 8 and ▲ = 3, which gives (3■)(2▲) =
(38)(23) = 874 ≠ 864.
Therefore, ■ = 6; SUFFICIENT.

The correct answer is D;
each statement alone is sufficient.

6.5 Data Sufficiency Answer Explanations

M
335. Two points, N and Q (not shown), lie to the right of
point M on line ℓ. What is the ratio of the length of QN
to the length of MQ ?
(1)

Twice the length of MN is 3 times the length of MQ.

(2)

Point Q is between points M and N.

Algebra Order

(1)

(2)

Given that twice the length of MN is
3 times the length of MQ, it follows that
the points are ordered from left to right as
M, Q, and N. Thus, letting MQ = x and
QN = y, it is given that 2(x + y) = 3x and
y
the value of is to be determined. The given
x
equation can be rewritten as 2x + 2y = 3x, or
y
2y = x, or = 1 ; SUFFICIENT.
x 2
Given that Q is between M and N, the ratio
of QN to MQ can be close to zero (if Q and
N are close together and both far from M)
and the ratio of QN to MQ can be large (if
M and Q are close together and both far
from N); NOT sufficient.

The correct answer is A;
statement 1 alone is sufficient.
336. Did the sum of the prices of three shirts exceed $60 ?
(1)

The price of the most expensive of the shirts
exceeded $30.

(2)

The price of the least expensive of the shirts
exceeded $20.

Arithmetic Applied problems

(1)

(2)

Given that the price of the most expensive
shirt exceeded $30, the sum of the prices of
the shirts can be under $60 (if the prices were
$10, $10, and $35) and the sum of the prices
of the shirts can be over $60 (if the prices
were $10, $10, and $50); NOT sufficient.
Given that the price of the least expensive
shirt exceeded $20, it follows that the
sum of the prices of the shirts exceeds
3($20) = $60; SUFFICIENT.

337. What is the total number of coins that Bert and
Claire have?
(1)

Bert has 50 percent more coins than Claire.

(2)

The total number of coins that Bert and Claire
have is between 21 and 28.

Arithmetic Computation with integers

Determine the total number of coins Bert and
Claire have. If B represents the number of coins
that Bert has and C represents the number of
coins that Claire has, determine B + C.
(1)

Bert has 50% more coins than Claire, so
B = 1.5C, and B + C = 1.5C + C = 2.5C, but
the value of C can vary; NOT sufficient.

(2)

The total number of coins Bert and
Claire have is between 21 and 28, so
21 < B + C < 28 and, therefore, B + C could
be 22, 23, 24, 25, 26, or 27; NOT sufficient.

Taking (1) and (2) together, 21 < 2.5C < 28 and
then 21 < C < 28 or 8.4 < C < 11.2.
2.5
2.5
If C = 9, then B = (1.5)(9) = 13.5; if C = 10,
then B = (1.5)(10) = 15; and if C = 11, then
B = (1.5)(11) = 16.5. Since B represents a number
of coins, B is an integer. Therefore, B = 15, C = 10,
and B + C = 25.
The correct answer is C;
both statements together are sufficient.
338. A telephone station has x processors, each of which
can process a maximum of y calls at any particular
time, where x and y are positive integers. If 500 calls
are sent to the station at a particular time, can the
station process all of the calls?
(1)

x = 600

(2)

100 < y < 200

Algebra Applied problems

At a particular time, the telephone station can
process a maximum of xy calls, where x and y are
positive integers. Determine whether xy ≥ 500.

The correct answer is B;
statement 2 alone is sufficient.
327

GMAT® Official Guide 2018

(1)

Given that x = 600, it follows that
xy ≥ 600 since y ≥ 1 (y is a positive
integer); SUFFICIENT.

(2)

Given that 100 < y < 200, xy < 500 is
possible (if x = 3 and y = 150) and
xy ≥ 500 is possible (if x = 10 and y = 150);
NOT sufficient.

The correct answer is A;
statement 1 alone is sufficient.
Price per Flower
Roses

$1.00

Daisies

$0.50

339. Kim and Sue each bought some roses and some
daisies at the prices shown above. If Kim bought the
same total number of roses and daisies as Sue, was
the price of Kim’s purchase of roses and daisies higher
than the price of Sue’s purchase of roses and daisies?
(1)

Kim bought twice as many daisies as roses.

(2)

Kim bought 4 more roses than Sue bought.

340. Jazz and blues recordings accounted for 6 percent of
the $840 million revenue from the sales of recordings
in Country Y in 2000. What was the revenue from
the sales of jazz and blues recordings in Country Y in
1998 ?
(1)

Jazz and blues recordings accounted for
5 percent of the revenue from the sales of
recordings in Country Y in 1998.

(2)

The revenue from the sales of jazz and blues
recordings in Country Y increased by 40 percent
from 1998 to 2000.

Arithmetic Percents

It is given that jazz and blues recordings
accounted for 6% of the $840 million revenue
of recordings in Country Y in 2000. Determine
the revenue from the sales of jazz and blues
recordings in 1998.
(1)

This indicates that jazz and blues recordings
accounted for 5% of the revenue from
the sales of recordings in Country Y in
1998. However, no information is given
to indicate what the revenue from the
sales of recordings was in 1998. Therefore,
the revenue from sales of jazz and blues
recordings in 1998 cannot be determined;
NOT sufficient.

(2)

This indicates that the revenue from
the sales of jazz and blues recordings
in Country Y increased by 40% from
1998 to 2000. Letting R1998 and R2000
represent the revenue from the sales of
jazz and blues recordings in 1998 and
2000, respectively, then R2000 = 1.4R1998,
and so (0.06)($840 million) = 1.4R1998,
from which R1998 can be determined;
SUFFICIENT.

Algebra Applied problems

Let RK be the number of roses that Kim
bought, let RS be the number of roses that Sue
bought, and let T be the total number of roses
and daisies each bought. Then Kim bought
(T − RK) daisies and Sue bought (T − RS) daisies.
For the roses and daisies, Kim paid a total of
$[RK + 1 (T − RK)] = $ 1 (RK + T) and Sue paid
2
2
a total of $[RS + 1 (T − RS)] = $ 1 (RS + T).
2
2
Determine whether 1 (RK + T) > 1 (RS + T), or
2
2
equivalently, determine whether RK > RS .
(1)

(2)

Given that T − RK = 2RK, or T = 3RK, it is
not possible to determine whether RK > RS
because no information is provided about
the value of RS; NOT sufficient.
Given that RK = RS + 4, it follows that
RK > RS; SUFFICIENT.

The correct answer is B;
statement 2 alone is sufficient.

328

The correct answer is B;
statement 2 alone is sufficient.
341. On a certain nonstop trip, Marta averaged x miles per
hour for 2 hours and y miles per hour for the remaining
3 hours. What was her average speed, in miles
per hour, for the entire trip?
(1)

2x + 3y = 280

(2)

y = x + 10

6.5 Data Sufficiency Answer Explanations

Algebra Rate problems

Marta traveled a total of (2x + 3y) miles in
⎛ 2x + 3 y ⎞
2 + 3 = 5 hours for an average speed of ⎜
⎟
⎝
5 ⎠
2x + 3 y
.
miles per hour. Determine the value of
5
(1) Given that 2x + 3y = 280, it follows that
2x + 3 y 280
; SUFFICIENT.
=
5
5
(2) Given that y = x + 10, it follows that
2x + 3 y 2x + 3( x + 10)
=
= x + 6.
5
5
2x + 3 y
can be
Therefore, the value of
5
56 (if x = 50 and y = 60) and the value of
2x + 3 y
can be 61 (if x = 55 and y = 65);
5
NOT sufficient.

343. A tank is filled with gasoline to a depth of exactly
2 feet. The tank is a cylinder resting horizontally on
its side, with its circular ends oriented vertically. The
inside of the tank is exactly 6 feet long. What is the
volume of the gasoline in the tank?
(1)

The inside of the tank is exactly 4 feet in diameter.

(2)

The top surface of the gasoline forms a
rectangle that has an area of 24 square feet.

Geometry Cylinders; Volume

(1)

Given that the diameter of the cylindrical
tank is 4 ft, it follows that its radius is 2 ft.
Therefore, the radius and height of the
cylindrical tank are known, and hence the
volume of the gasoline is half the volume of the
cylinder, or 1 πr2h = 1 π (2 ft)2(6 ft) = 12π ft3;
2
2
SUFFICIENT.

The correct answer is A;
statement 1 alone is sufficient.
342. If x is a positive integer, what is the value of
x + 24 − x ?
(1)

x is an integer.

(2)

x + 24 is an integer.

x'
x
A

Determine the value of x + 24 − x .

B

2

C'
r
r
C

This indicates that x is a perfect square. If
x = 1, then x + 24 − x = 1 + 24 − 1 =
5 − 1 = 4. However, if x = 25, then
x + 24 − x = 25 + 24 − 25
= 7 − 5 = 2;
NOT sufficient.

(2)

2

O

Arithmetic Operations with radicals

(1)

B'

A'

This indicates that x + 24 is an integer. In
each of the examples above, x + 24 is an
integer; NOT sufficient.

Because the same examples were used to establish
that neither (1) nor (2) is sufficient, it is not
possible to determine the value of x + 24 − x
from the given information.
The correct answer is E;
both statements together are still not sufficient.

P

(2)

Given that the rectangular surface of the
gasoline forms a rectangle with area 24 ft2
and length 6 ft, the width of the rectangular
surface is 4 ft. Let r ft be the radius of the
cylinder. The figure shows a vertical cross‐
section of the cylindrical tank—a circle
with center O—with the two possibilities
for the location of the top surface of the
gasoline: the depth of the gasoline is less
than or equal to r ft ( AC marks the location
of the top surface of the gasoline and
BP = 2 ft is the depth of the gasoline) or
the depth of the gasoline is greater than or
equal to r ft ( A ′C ′ marks the location of
the top surface of the gasoline and B ′P = 2 ft
329

GMAT® Official Guide 2018

is the depth of the gasoline). For the first
possibility, ΔOBC is a right triangle with
hypotenuse OC of length r ft, so letting
OB = x ft, it follows from the Pythagorean
theorem that r2 = x2 + 22 = x2 + 4.
However, because r = OB + BP = x + 2,
it follows that r2 = (x + 2)2 = x2 + 4x + 4.
Therefore, x2 + 4 = x2 + 4x + 4, so 4x = 0,
x = 0, and AC is a diameter of the circle.
Thus, the diameter of the cylindrical tank is
4 ft, and by the same reasoning used in (1),
the volume of the gasoline can be
determined. For the second possibility,
ΔOB ′C ′ is a right triangle with hypotenuse
OC ′ of length r ft, so letting B ′O = x ′ ft, it
follows from the Pythagorean theorem that
r2 = (x ′)2 + 22 = (x ′)2 + 4. However, because
r = B ′P − B ′O = 2 − x ′, it follows that
r2 = (2 − x ′)2 = 4 − 4x ′ + (x ′)2. Therefore,
(x ′)2 + 4 = 4 − 4x ′ + (x ′)2, so 0 = –4x ′,
x ′ = 0, and A ′C ′ is a diameter of the circle.
Thus, the diameter of the cylindrical tank
is 4 ft, and by the same reasoning used
in (1), the volume of the gasoline can be
determined; SUFFICIENT.

(1)

0
q

r

s

q = –s

(2)

–t < q

q

r

s

t

Referring to the figure above, in which it may be
assumed that q, r, s, and t are different numbers,
determine if r is closest to 0.

330

r

s

t

In each case, r is closest to 0;
SUFFICIENT.
(2)

q

t

(ii) If q < r < 0 < s < t, as shown above,
r is closer to 0 than q is because r is
between 0 and q. Also, r is closer to
0 than s is because r is closer to 0 than
q is and q and s are the same distance
from 0. Moreover, r is closer to 0 than
t is because t is farther away from
0 than s is. Therefore, r is closest to 0.

t

Algebra Order

s

0

344. Of the four numbers represented on the number line
above, is r closest to zero?
(1)

r

(i) If q < 0 < r < s < t, as shown above, r is
closer to 0 than s is because r is between
0 and s, and r is clearly closer to 0 than
t is because t is farther away from
0 than s is. Also, since q and s are the
same distance from 0 and r is closer to
0 than s is, then r is closer to 0 than q is.
Therefore, r is closest to 0.

The correct answer is D;
each statement alone is sufficient.

q

Since q = –s, one of q and s is positive and
the other is negative. Since s is to the right
of q, then s is positive and q is negative.
Also, 0 is halfway between q and s, so q and
s are the same distance from 0. If r = 0, then,
of q, r, s, and t, r is closest to 0 because it IS 0.
If r ≠ 0 then either (i) q < 0 < r < s < t or
(ii) q < r < 0 < s < t .

If − t < q, then –t is to the left of q.
If t = 5, s = 4, r = 3, and q = –2, then
–5 < –2, so (2) is satisfied. In this case,
q is closest to 0. On the other hand, if t = 5,
s = 4, r = –1, and q = − 2, then –5 < –2, so
(2) is satisfied, but r is closest to 0; NOT
sufficient.

The correct answer is A;
statement 1 alone is sufficient.

6.5 Data Sufficiency Answer Explanations

345. A group consisting of several families visited an
amusement park where the regular admission fees
were ¥5,500 for each adult and ¥4,800 for each child.
Because there were at least 10 people in the group,
each paid an admission fee that was 10% less than the
regular admission fee. How many children were in the
group?
(1)
(2)

The total of the admission fees paid for the
adults in the group was ¥29,700.
The total of the admission fees paid for the children
in the group was ¥4,860 more than the total of the
admission fees paid for the adults in the group.

B

A

x°
D

C

346. What is the area of triangular region ABC above?
(1)

The product of BD and AC is 20.

(2)

x = 45

Arithmetic Simultaneous equations

Geometry Triangles; Area

Determine the number of children in a group of
at least 10 people who visited an amusement park.

BD × AC
The area of Δ ABC =
.
2
(1) The product of BD and AC is given as
20, so the area of ΔABC is 20 or 10;
2
SUFFICIENT.

(1)

(2)

This indicates that (0.9)(5,500)A = 29,700,
where A represents the number of adults in
the group. From this, the number of adults
can be determined. However, the number of
children in the group cannot be determined
without additional information about the
exact number of people in the group; NOT
sufficient.
If C and A represent the numbers
of children and adults, respectively,
in the group, this indicates that
(0.9)(4,800)C = (0.9)(5,500)A + 4,860, or
48C = 55A + 54, or 48C − 55A = 54, which
is a single equation with two variables from
which unique values of C and A cannot be
determined, even under the assumptions
that C and A are integers such that
C + A ≥ 10. For example, the values of C
and A could be 8 and 6, respectively, since
(48)(8) − (55)(6) = 54, or the values of C
and A could be 63 and 54, respectively, since
(48)(63) – (55)(54) = 54; NOT sufficient.

Taking (1) and (2) together, it follows that
29,700
= 6 and
A=
(0.9)(5, 500)
C=

(0.9)(5, 500)(6) + 4,860
= 8.
(0.9)(4,800)

The correct answer is C;
both statements together are sufficient.

(2)

With the measurement of x being 45, it is
concluded that ΔABD is a 45–45–90 right
triangle, where the length of side BD is
equal to the length of side AD. However,
with no lengths of any side known, there
is not enough information to calculate the
area; NOT sufficient.

The correct answer is A;
statement 1 alone is sufficient.
347. In the xy‐coordinate plane, is point R equidistant from
points (–3,–3) and (1,–3) ?
(1)

The x‐coordinate of point R is –1.

(2)

Point R lies on the line y = –3.

Algebra Coordinate geometry

Determine if point R is equidistant from (–3,–3)
and (1,–3). Letting R = (x,y), then R will be
equidistant from (–3,–3) and (1,–3) if and only
if R lies on the perpendicular bisector of the line
segment with endpoints (–3,–3) and (1,–3), or
equivalently, if and only if R lies on the vertical
line that consists of all points with x‐coordinate
equal to −3 + 1 = −1. Therefore, determine if x = –1.
2

331

GMAT® Official Guide 2018

(1)

Given that x = –1, then R will be
equidistant from (–3,–3) and (1,–3);
SUFFICIENT.

(2)

Given that y = –3, then both x = –1 and
x ≠ –1 are possible; NOT sufficient.

height of the students in class X and class Y
combined is 10(120) + 10(132) = 126 cm
20
and the ratio of the averages of the
individual classes is 120 . However, if
132
class X consists of 10 students, each of
whom has height 120 cm and class Y
consists of 20 students, each of whom has
height 129 cm, then the average height
of the students in class X and class Y
10(120) + 20(129)
combined is
= 126 cm
30
and the ratio of the averages of the
120
; NOT sufficient.
individual classes is
129

Alternatively, letting R = (x,y), then R will be
equidistant from (–3,–3) and (1,–3) if and only if
the distance between (x,y) and (–3,–3) is the same
as the distance between (x,y) and (1,–3), or if and
only if ( x + 3)2 + ( y + 3)2 = ( x − 1)2 + ( y + 3)2
or if and only if (x + 3)2 + (y + 3)2 =
(x − 1)2 + (y + 3)2, or if and only if
(x + 3)2 = (x − 1)2.
(1)
(2)

Given that x = –1, then (–1 + 3)2 = 4 and
(–1 − 1)2 = 4; SUFFICIENT.
Given that y = –3, then it is impossible
to determine if (x + 3)2 = (x − 1)2; NOT
sufficient.

The correct answer is A;
statement 1 alone is sufficient.
348. What is the ratio of the average (arithmetic mean)
height of students in class X to the average height of
students in class Y ?
(1)
(2)

The average height of the students in class X is
120 centimeters.
The average height of the students in class X
and class Y combined is 126 centimeters.

Arithmetic Statistics

332

Taking (1) and (2) together is of no more help
than either (1) or (2) taken separately because
the same examples used to show that (2) is not
sufficient include the information from (1).
The correct answer is E;
both statements together are still not sufficient.
349. Is the positive two‐digit integer N less than 40 ?
(1)

The units digit of N is 6 more than the tens digit.

(2)

N is 4 less than 4 times the units digit.

Arithmetic Place value

Determine if the two‐digit integer N is less than
40. Letting the tens digit be t and the units digit
be u, then N = 10t + u. Determine if 10t + u < 40.

(1)

Given that the average height of the
students in class X is 120 cm, but given no
information about the average height of the
students in class Y, the desired ratio cannot
be determined; NOT sufficient.

(1)

Given that u = t + 6, then
N = 10t + (t + 6) = 11t + 6. Since u is a digit,
u = t + 6 ≤ 9, so t ≤ 3. Therefore,
N = 11t + 6 ≤ 11(3) + 6 = 39;
SUFFICIENT.

(2)

Given that the average height of the
students in class X and class Y combined
is 126 cm, the ratio of the averages of the
individual classes cannot be determined. For
example, if class X consists of 10 students,
each of whom has height 120 cm, and
class Y consists of 10 students, each of
whom has height 132 cm, then the average

(2)

Given that N = 4u − 4, then since u is a
digit and u ≤ 9, it follows that
N = 4u − 4 ≤ 4(9) − 4 = 32; SUFFICIENT.

The correct answer is D;
each statement alone is sufficient.

6.5 Data Sufficiency Answer Explanations

350. If 2x + y = 48, what is the value of y ?
(1)

x2 = 81

(2)

x−y=2

352. At a bakery, all donuts are priced equally and all bagels
are priced equally. What is the total price of 5 donuts
and 3 bagels at the bakery?
(1)

At the bakery, the total price of 10 donuts and
6 bagels is $12.90.

(2)

At the bakery, the price of a donut is $0.15 less
than the price of a bagel.

Algebra Exponents

Since 48 = (22)8 = 216, the equation 2x + y = 48
becomes 2x + y = 216, which is equivalent to
x + y = 16.
(1)

Given that x2 = 81, then both x = 9 and
x = –9 are possible. Therefore, y = 7 is possible
(choose x = 9) and y = 25 is possible (choose
x = –9); NOT sufficient.

(2)

Given that x − y = 2, it follows that
–x + y = –2. Adding the last equation
to x + y = 16 gives 2y = 14, or y = 7;
SUFFICIENT.

The correct answer is B;
statement 2 alone is sufficient.
351. Each week a certain salesman is paid a fixed amount
equal to $300, plus a commission equal to 5 percent
of the amount of his sales that week over $1,000.
What is the total amount the salesman was paid last
week?
(1)

(2)

Algebra Simultaneous equations

Let x be the price, in dollars, of each donut and
let y be the price, in dollars, of each bagel. Find
the value of 5x + 3y.
(1)

(2)

Given that 10x + 6y = 12.90, since
5x + 3 y = 1 (10 x + 6 y ), it follows that
2
1
5x + 3 y = (12.90); SUFFICIENT.
2
Given that x = y − 0.15, then 5x + 3y =
5(y − 0.15) + 3y = 8y − 0.75, which varies as
y varies; NOT sufficient.

The correct answer is A;
statement 1 alone is sufficient.
D

The total amount the salesman was paid last
week is equal to 10 percent of the amount of his
sales last week.

A

The salesman’s sales last week totaled $5,000.

Algebra Applied problems

Let P be the salesman’s pay for last week
and let S be the amount of his sales last week.
Then P = 300 + 0.05(S − 1,000). Determine the
value of P.
(1)

(2)

Given P = 0.10S, then 0.10S =
300 + 0.05(S − 1,000). This equation can
be solved for a unique value of S, from
which the value of P can be determined;
SUFFICIENT.
Given S = 5,000, then P =
300 + 0.05(5,000 − 1,000); SUFFICIENT.

The correct answer is D;
each statement alone is sufficient.

C

B

353. In the figure above, is the area of triangular region
ABC equal to the area of triangular region DBA ?
(1)

(AC)2 = 2(AD)2

(2)

Δ ABC is isosceles.

Geometry Triangles; Area

Determine whether 1 ( AC )(CB ) = 1 ( AD )( AB )
2
2
or, equivalently, whether (AC)(CB) = (AD)(AB).
(1)

Given that (AC)2 = 2(AD)2, then 2 and 1
could be the values of AC and AD,
respectively. If, for example,CB = 2 , then
333

GMAT® Official Guide 2018

( ) ( )
2

2

AB =
2 + 2 = 2 by the
Pythagorean theorem applied to Δ ABC.
Hence, ( AC )(CB ) = ( 2)( 2) and
(AD)(AB) = (1)(2), from which it follows
that (AC)(CB) = (AD)(AB). On the other
hand, if CB = 1, then AB =

( 2)

2

Arithmetic Properties of numbers

Determine if r , where r and s are positive
s
integers, can be expressed as a decimal with a
finite number of nonzero decimal digits.
(1)

+ 12 = 3

by the Pythagorean theorem applied to
Δ ABC. Hence, ( AC )(CB ) = ( 2)(1) and
( AD )( AB ) = (1)( 3), from which it follows
that (AC)(CB) ≠ (AD)(AB); NOT sufficient.
(2)

Given that Δ ABC is isosceles then, by
varying the value of AD, the area of
Δ ABC may or may not be equal to the area
of Δ DBA. For example, if AC = CB = 1, then
(AC)(CB) = (1)(1) = 1 and AB = 2 by the
Pythagorean theorem applied to Δ ABC. If
⎛
⎞
AD = 1 , then( AD )( AB ) = ⎜ 1 ⎟ ( 2) = 1,
⎝ 2⎠
2
and (AC)(CB) = (AD)(AB). On the other
hand, if AD = 1, ( AD )( AB ) = (1)( 2) = 2
and (AC)(CB) ≠ (AD)(AB); NOT sufficient.

Given (1) and (2) together, let AC = CB = x be
the length of the legs of the isosceles triangle
ABC. Then, from (AC)2 = 2(AD)2, it follows that
x2 = 2(AD)2, and hence AD = x . Also, by the
2
Pythagorean theorem applied to Δ ABC, it
follows that AB = x 2 + x 2 = x 2 . Therefore,
it follows that (AC)(CB) = (x)(x) = x2 and
⎛
⎞
( AD )( AB ) = ⎜ x ⎟ x 2 = x 2 , and so
⎝ 2⎠
(AC)(CB) = (AD)(AB).

(

)

(2)

It is given that s is a factor of 100 and so
s = 1, 2, 4, 5, 10, 20, 25, 50, or 100. This
r
means that must be one of the quotients
s
r , r , r , r , r , r , r , r or r .
1 2 4 5 10 20 25 50
100
r
Thus, must be one of the products r(1),
s
r(0.5), r(0.25), r(0.20), r(0.1), r(0.05) r(0.04),
r(0.02), or r(0.01). In each case, r is the
s
product of an integer and a decimal with a
finite number of nonzero digits, and
hence, r can be expressed as a decimal with
s
a finite number of nonzero digits. In fact,
it suffices to note this is true for r , since
100
each of the other possibilities is a positive
integer times r (for example, r is
100
20
r
); SUFFICIENT.
5 times
100
It is given that r is a factor of 100. If r = 4
and s = 5, then r = 0.8, which is a decimal
s
with a finite number of nonzero digits.
On the other hand, if r = 4 and s = 7, then
r = 0.571428, which is not a decimal with
s
a finite number of nonzero digits; NOT
sufficient.

The correct answer is A;
statement 1 alone is sufficient.
355. If r > 0 and s > 0, is

The correct answer is C;
both statements together are sufficient.
r
be
s
expressed as a decimal with only a finite number of
nonzero digits?

354. If r and s are positive integers, can the fraction

334

(1)

s is a factor of 100.

(2)

r is a factor of 100.

(1)
(2)

r s
< ?
s r

r
1
=
3s 4
s=r+4

Algebra Ratios

Given positive numbers r and s, determine if
r < s.
s r
r =1
r 3
, then = , s = 4 , and so
(1) If
3s 4
s 4 r 3
r < s , since 3 < 4 ; SUFFICIENT.
4 3
s r

6.5 Data Sufficiency Answer Explanations

r = r
r +4
and s =
.
s r +4
r
r
r +4
> 1,
Since r + 4 > r, r < 1 and
r
r +4
so r < s ; SUFFICIENT.
s r
The correct answer is D;
each statement alone is sufficient.
(2)

If s = r + 4, then

356. If k is an integer such that 56 < k < 66, what is the
value of k ?
(1)

If k were divided by 2, the remainder would be 1.

(2)

If k + 1 were divided by 3, the remainder would
be 0.

(2)

This indicates that 5x + 1 is prime. If x = 2,
then 5x + 1 = 11 is prime and so is x.
However, if x = 6, then 5x + 1 = 31 is prime,
but 6 is not prime; NOT sufficient.

Because the same examples were used to establish
that neither (1) nor (2) is sufficient, it is not
possible to determine whether x is prime from
the given information.
The correct answer is E;
both statements together are still not sufficient.
k, n, 12, 6, 17
358. What is the value of n in the list above?

Arithmetic Properties of integers

(1)

k 0.
5
d −b 8−3 5
a
c
For these values, < is true because 2 < 6 .
3 8
b d
On the other hand, a = 4, b = 6, c = 2, and
d = 3 are also possible values of a, b, c, and
c −a = 2−4 = 2
d because
and 2 > 0. For
d − b 3−6 3
3
a
c
these values, < is false because 4 = 2 ;
b d
6 3
NOT sufficient.

1
percent of the amount that he spent on his
3
mortgage payments.

33

(2)

a c
< ?
b d

c−a
d −b

(1)

360. Last year, if Arturo spent a total of $12,000 on his
mortgage payments, real estate taxes, and home
insurance, how much did he spend on his real estate
taxes?
(1)

1
Given that R = ( M + H ), or 5R = M + H
5
and M + R + H = 12,000, which can be
rewritten as (M + H) + R = 12,000, then
5R + R = 12,000, or R = 2,000;
SUFFICIENT.

2

(2)

Given that ⎛ ad ⎞ < ad , then
⎝ bc ⎠
bc
ad < 1
bc
ad < c
b
a< c
b d

dividing both sides by the
ad
positive number
bc
multiplying both sides by the
positive number c
dividing both sides by the positive
number d; SUFFICIENT

The correct answer is B;
statement 2 alone is sufficient.

6.5 Data Sufficiency Answer Explanations

362. Is the number of members of Club X greater than the
number of members of Club Y ?
(1)

Of the members of Club X, 20 percent are also
members of Club Y.

(2)

Of the members of Club Y, 30 percent are also
members of Club X.

Arithmetic Sets

Let a be the number of members in Club X that
do not belong to Club Y, let b be the number of
members in Club Y that do not belong to Club X,
and let c be the number of members that belong to
both Club X and to Club Y. Determine whether
a + c > b + c, or equivalently, whether a > b.
(1)

(2)

If a = 80, b =79, and c = 20, then 20 percent
of the members of Club X are also members
of Club Y (because c = 20 is 20 percent of
a + c = 100) and a > b is true. However, if
a = 80, b = 80, and c = 20, then 20 percent
of the members of Club X are also members
of Club Y (because c = 20 is 20 percent of
a + c = 100) and a > b is false. Therefore, it
cannot be determined whether a > b; NOT
sufficient.
If a = 71, b = 70, and c = 30, then 30 percent
of the members of Club Y are also members
of Club X (because c = 30 is 30 percent of
b + c = 100) and a > b is true. However, if
a = 70, b =70, and c = 30, then 30 percent of
the members of Club Y are also members
of Club X (because c = 30 is 30 percent of
b + c = 100) and a > b is false. Therefore, it
cannot be determined whether a > b; NOT
sufficient.

Now assume both (1) and (2). From (1) it follows
1
c
= 0.20 = , or 5c = a + c, and so a = 4c.
that
5
a+c
3
From (2) it follows that c = 0.30 = , or
10
b+c
7
10c = 3b + 3c, and so 7c = 3b and b = c . Since
3
7
4c > c (from the statements it can be deduced
3
that c > 0), it follows that a > b. Therefore, (1) and
(2) together are sufficient.
The correct answer is C;
both statements together are sufficient.

p

q

r

s

t

363. On the number line above, p, q, r, s, and t are five
consecutive even integers in increasing order. What is
the average (arithmetic mean) of these five integers?
(1)

q + s = 24

(2)

The average (arithmetic mean) of q and r is 11.

Arithmetic Properties of numbers

Since p, q, r, s, and t are consecutive even integers
listed in numerical order, the 5 integers can also be
given as p, p + 2, p + 4, p + 6, and p + 8. Determine
the average of these 5 integers, which is the value
p + ( p + 2) + ( p + 4) + ( p + 6) + ( p + 8)
of
=
5
5 p + 20
= p+4
5
(1) Given that q + s = 24, then
(p + 2) + (p + 6) = 24.
Therefore, 2p + 8 = 24, or p = 8, and hence
p + 4 =12; SUFFICIENT.
(2)

q +r
= 11, then q + r = (2)(11) = 22,
2
or (p + 2) + (p + 4) = 22. Therefore,
2p + 6 = 22, or p = 8, and hence p + 4 = 12;
SUFFICIENT.

Given that

The correct answer is D;
each statement alone is sufficient.
364. If ⎡⎢ x ⎤⎥ denotes the least integer greater than or equal
to x, is ⎡⎢ x ⎤⎥ = 0 ?
(1)

–1 < x < 1

(2)

x<0

Algebra Functions

Determine if ⎡⎢ x ⎤⎥ , the least integer greater than
or equal to x, is equal to 0, which is the same as
determining if x satisfies –1 < x ≤ 0.
(1)

Given that –1 < x < 1, then it is possible
that ⎡⎢ x ⎤⎥ = 0 (for example, if x = 0, then
⎡⎢ x ⎤⎥ = 0) and it is possible that ⎡⎢ x ⎤⎥ ≠ 0 (for
1
example, if x = , then ⎡⎢ x ⎤⎥ = 1 ≠ 0); NOT
2
sufficient.
337

GMAT® Official Guide 2018

(2)

Given that x < 0, then it is possible that
1
⎡⎢ x ⎤⎥ = 0 (for example, if x = − , then ⎡⎢ x ⎤⎥ = 0)
2
and it is possible that ⎡⎢ x ⎤⎥ ≠ 0 (for example,
if x = –5, then ⎡⎢ x ⎤⎥ = −5 ≠ 0); NOT
sufficient.

Taking (1) and (2) together gives –1 < x < 0,
which implies that ⎡⎢ x ⎤⎥ = 0.
The correct answer is C;
both statements together are sufficient.
365. If x and y are integers, is x > y ?
(1)

x+y>0

(2)

yx<0

366. If r and s are the roots of the equation x2 + bx + c = 0,
where b and c are constants, is rs < 0 ?
(1)

b<0

(2)

c<0

Algebra Second-degree equations

Determine whether the product of the roots to
x2 + bx + c = 0, where b and c are constants, is
negative.
If r and s are the roots of the given equation, then
(x − r)(x − s) = x2 + bx + c. This implies that
x2 − (r + s)x + rs = x2 + bx + c, and so rs = c.
Therefore, rs is negative if and only if c is negative.
(1)

Given that b < 0, then c could be negative or
positive. For example, if b = –1 and
c = –6, then the given equation would be
x2 − x − 6 = (x − 3)(x + 2) = 0, and the
product of its roots would be (3)(–2), which
is negative. On the other hand, if b = –6 and
c = 5, then the given equation would be
x2 − 6x + 5 = (x − 5)(x − 1) = 0, and the
product of its roots would be (5)(1), which
is positive; NOT sufficient.

(2)

Given that c < 0, it follows from the
explanation above that rs < 0; SUFFICIENT.

Arithmetic Properties of integers

Determine if the integer x is greater than the
integer y.
(1)

(2)

It is given that x + y > 0, and so –x < y.
If, for example, x = –3 and y = 4, then
x + y = –3 + 4 = 1 > 0 and x < y. On the
other hand, if x = 4 and y = –3, then
x + y = 4 − 3 = 1 > 0 and x > y; NOT
sufficient.
It is given that yx < 0, so y < 0. If, for
example, x = 3 and y = –2,
then (–2)3 = –8 < 0 and x > y. On the

The correct answer is B;
statement 2 alone is sufficient.

other hand, if x = –3 and y = –2, then
(–2)–3 = – 1 < 0 and x < y; NOT sufficient.
8

k ft

15 ft

Taking (1) and (2) together, from (2) y is
negative and from (1) –x is less than y. Therefore,
–x is negative, and hence x is positive. Since x is
positive and y is negative, it follows that x > y.
The correct answer is C;
both statements together are sufficient.

k ft
15 ft
367. The figure above represents an L‐shaped garden. What
is the value of k ?
(1)

The area of the garden is 189 square feet.

(2)

The perimeter of the garden is 60 feet.

Geometry Polygons

(1)

338

Given that the area of the garden is 189 ft2,
and observing that area of the garden can
be calculated by imagining the garden as
a square with dimensions 15 ft by 15 ft
from which its upper‐right square corner

6.5 Data Sufficiency Answer Explanations

with dimensions (15 − k) ft by (15 − k) ft
is removed, it follows that
189 = (15)2 − (15 − k)2. Therefore, (15 − k)2 =
225 − 189 = 36, so 15 − k = 6 or 15 − k = –6.
The figure implies 15 − k > 0, so it follows
that 15 − k = 6, or k = 9; SUFFICIENT.
(2)

Given that the perimeter of the garden is
60 ft, and because 15 + 15 + k + (15 − k) +
(15 − k) + k = 60, any value of k between 0
and 15 is possible, since for any such value
of k the sum of the lengths of all the sides
would be 60 ft; NOT sufficient.

The correct answer is A;
statement 1 alone is sufficient.
368. The only articles of clothing in a certain closet are
shirts, dresses, and jackets. The ratio of the number
of shirts to the number of dresses to the number of
jackets in the closet is 9:4:5, respectively. If there are
more than 7 dresses in the closet, what is the total
number of articles of clothing in the closet?
(1)

The total number of shirts and jackets in the
closet is less than 30.

(2)

The total number of shirts and dresses in the
closet is 26.

TOTAL EXPENSES FOR THE
FIVE DIVISIONS OF COMPANY H

R
x°

Q
O
P

S
T

369. The figure above represents a circle graph of
Company H’s total expenses broken down by the
expenses for each of its five divisions. If O is the center
of the circle and if Company H’s total expenses are
$5,400,000, what are the expenses for Division R ?
(1)

x = 94

(2)

The total expenses for Divisions S and T are
twice as much as the expenses for Division R.

Geometry Circles

In this circle graph, the expenses of Division R
x
multiplied by
are equal to the value of
360
$5,400,000, or $15,000x. Therefore, it is necessary
to know the value of x in order to determine the
expenses for Division R.

Arithmetic Ratio and proportion

(1)

Letting s, d, and j represent, respectively, the
numbers of shirts, dresses, and jackets in the
closet, then s = 9x, d = 4x, and j = 5x, where x is
a positive integer. It is given that 4x > 7, and so
4x ≥ 8 or x ≥ 2 since x is an integer. Determine
the value of 9x + 4x + 5x = 18x.

The value of x is given as 94, so the
expenses of Division R can be determined;
SUFFICIENT.

(2)

This gives a comparison among the expenses
of some of the divisions of Company H, but
no information is given about the value of x;
NOT sufficient.

(1)

This indicates that 9x + 5x < 30, and so
14x ≤ 28 or x ≤ 2 since x is an integer. It
follows from x ≥ 2 and x ≤ 2 that x = 2 and
18x = 36; SUFFICIENT.

(2)

This indicates that 9x + 4x = 26 or
13x = 26, or x = 2. It follows that 18x = 36;
SUFFICIENT.

The correct answer is D;
each statement alone is sufficient.

The correct answer is A;
statement 1 alone is sufficient.
370. If x is negative, is x < –3 ?
(1)

x2 > 9

(2)

x3 < –9

Algebra Properties of numbers

(1)

Given that x2 > 9, it follows that x < −3 or
x > 3, a result that can be obtained in a
variety of ways. For example, consider the
equivalent inequality ( x

)

2

> 9 that reduces
339

GMAT® Official Guide 2018

to x > 3, or consider when the two factors
of x2 − 9 are both positive and when the
two factors of x2 − 9 are both negative, or
consider where the graph of the parabola
y = x2 − 9 is above the x‐axis, etc. Since it is
also given that x is negative, it follows that
x < –3; SUFFICIENT.
(2)

3<

Given that x –9, if x = –4, then = –64,
and so x3 < –9 and it is true that x < –3.
However, if x = –3, then x3 = –27, and so
x3 < –9, but it is not true that x < –3;
NOT sufficient.
x3

The correct answer is A;
statement 1 alone is sufficient.
371. What is the number of cans that can be packed in a
certain carton?
(1)
(2)

The interior volume of this carton is 2,304 cubic
inches.

(2)

No information about the size of the carton
is given; NOT sufficient.

Taking (1) and (2) together, there is still not
enough information to answer the question. If
the carton is a rectangular solid that is 1 inch
by 1 inch by 2,304 inches and the cans are
cylindrical with the given dimensions, then 0 cans
can be packed into the carton. However, if the
carton is a rectangular solid that is 16 inches
by 12 inches by 12 inches and the cans are
cylindrical with the given dimensions, then
1 or more cans can be packed into the carton.
The correct answer is E;
both statements together are still not sufficient.

340

t

u

v

w

x

y

z

(1)

v+z=6

(2)

s+t+u+x=6

Arithmetic Properties of numbers

In the following discussion, “row/column
convention” means that each of the numbers 1, 2,
and 3 appears exactly once in any given row and
exactly once in any given column.
(1)

Given that v + z = 6, then both v and z
are equal to 3, since no other sum of the
possible values is equal to 6. Applying the
row/column convention to row 2, and then
to row 3, it follows that neither u nor x can
be 3. Since neither u nor x can be 3, the
row/column convention applied to column 1
forces r to be 3; SUFFICIENT.

(2)

If u = 3, then s + t + x = 3. Hence, s = t =
x = 1, since the values these variables can
have does not permit another possibility.
However, this assignment of values would
violate the row/column convention for
row 1, and thus u cannot be 3. If x = 3, then
s + t + u = 3. Hence, s = t = u = 1, since the
values these variables can have does not
permit another possibility. However, this
assignment of values would violate the
row/column convention for row 1, and thus
x cannot be 3. Since neither u nor x can be
3, the row/column convention applied to
column 1 forces r to be 3; SUFFICIENT.

Geometry Rectangular solids and cylinders

No information about the size of the cans is
given; NOT sufficient.

s

372. Each of the letters in the table above represents one
of the numbers 1, 2, or 3, and each of these numbers
occurs exactly once in each row and exactly once in
each column. What is the value of r ?

The exterior of each can is 6 inches high and has
a diameter of 4 inches.

(1)

r

The correct answer is D;
each statement alone is sufficient.

6.5 Data Sufficiency Answer Explanations

373. Material A costs $3 per kilogram, and Material B costs
$5 per kilogram. If 10 kilograms of Material K consists
of x kilograms of Material A and y kilograms of
Material B, is x > y ?
(1)

y>4

(2)

The cost of the 10 kilograms of Material K is
less than $40.

Algebra Inequalities

Since x + y = 10, the relation x > y is equivalent to
x > 10 − x, or x > 5.
(1)

(2)

The given information is consistent with
x = 5.5 and y = 4.5, and the given information
is also consistent with x = y = 5. Therefore,
it is possible for x > y to be true and it is
possible for x > y to be false; NOT sufficient.
Given that 3x + 5y < 40, or
3x +5(10 − x) < 40, then 3x − 5x < 40 − 50.
It follows that −2x < –10, or x > 5;
SUFFICIENT.

The correct answer is B;
statement 2 alone is sufficient.
374. At what speed was a train traveling on a trip when it
had completed half of the total distance of the trip?
(1)
(2)

The trip was 460 miles long and took 4 hours to
complete.

Determine the speed of the train when it had
completed half the total distance of the trip.
Given that the train traveled 460 miles
in 4 hours, the train could have traveled
at the constant rate of 115 miles per hour
for 4 hours, and thus it could have been
traveling 115 miles per hour when it had
completed half the total distance of the
trip. However, the train could have traveled
150 miles per hour for the first 2 hours
(a distance of 300 miles) and 80 miles per
hour for the last 2 hours (a distance of
160 miles), and thus it could have been
traveling 150 miles per hour when it had
completed half the total distance of the trip;
NOT sufficient.

Given that the train traveled at an average
rate of 115 miles per hour, each of the
possibilities given in the explanation for
(1) could occur, since 460 miles in 4 hours
gives an average speed of 460 = 115 miles
4
per hour; NOT sufficient.

Assuming (1) and (2), each of the possibilities
given in the explanation for (1) could occur.
Therefore, (1) and (2) together are not sufficient.
The correct answer is E;
both statements together are still not sufficient.
375. Tom, Jane, and Sue each purchased a new house. The
average (arithmetic mean) price of the three houses
was $120,000. What was the median price of the
three houses?
(1)

The price of Tom’s house was $110,000.

(2)

The price of Jane’s house was $120,000.

Arithmetic Statistics

Let T, J, and S be the purchase prices for Tom’s,
Jane’s, and Sue’s new houses. Given that
the average purchase price is 120,000, or
T + J + S = 3(120,000), determine the median
purchase price.
(1)

Given T = 110,000, the median could be
120,000 (if J = 120,000 and S = 130,000) or
125,000 (if J = 125,000 and S = 125,000);
NOT sufficient.

(2)

Given J = 120,000, the following two cases
include every possibility consistent with
T + J + S = (3)(120,000), or
T + S = (2)(120,000).

The train traveled at an average rate of
115 miles per hour on the trip.

Arithmetic Applied problems

(1)

(2)

(i) T = S = 120,000
(ii) One of T or S is less than 120,000 and
the other is greater than 120,000.
In each case, the median is clearly 120,000;
SUFFICIENT.
The correct answer is B;
statement 2 alone is sufficient.

341

GMAT® Official Guide 2018

376. What is the value of x if x3 < x2 ?
(1)

–2 < x < 2

(2)

x is an integer greater than –2.

Algebra Inequalities

The inequality x3 < x2 is equivalent to x3 − x2 < 0,
or x2(x − 1) < 0. Since this inequality is false
for x = 0, it follows that x ≠ 0, and hence x2 > 0.
Therefore, x2(x − 1) < 0 can only hold if x − 1 < 0,
or if x < 1. Thus, the problem is equivalent to
determining the value of x given that x ≠ 0 and
x < 1.
(1)

Given that –2 < x < 2, it is not possible to
determine the value of x. For example, the
value of x could be –1 (note that –1 < 1)
and the value of x could be 0.5 (note that
0.5 < 1); NOT sufficient.

(2)

Given that the value of x is an integer
greater than –2, then the value of x must
be among the integers –1, 0, 1, 2, 3, . . . .
However, from the discussion above, x ≠ 0
and x < 1, so the value of x can only be –1;
SUFFICIENT.

The correct answer is B;
statement 2 alone is sufficient.
377. For any integers x and y, min(x, y) and max(x, y) denote
the minimum and the maximum of x and y, respectively.
For example, min(5, 2) = 2 and max(5, 2) = 5. For
the integer w, what is the value of min(10, w) ?
(1)

w = max(20, z) for some integer z.

(2)

w = max(10, w)

378. A certain bookcase has 2 shelves of books. On the
upper shelf, the book with the greatest number of
pages has 400 pages. On the lower shelf, the book
with the least number of pages has 475 pages. What
is the median number of pages for all of the books on
the 2 shelves?
(1)

There are 25 books on the upper shelf.

(2)

There are 24 books on the lower shelf.

Arithmetic Statistics

(1)

The information given says nothing about
the number of books on the lower shelf. If
there are fewer than 25 books on the lower
shelf, then the median number of pages will
be the number of pages in one of the books
on the upper shelf or the average number
of pages in two books on the upper shelf.
Hence, the median will be at most 400. If
there are more than 25 books on the lower
shelf, then the median number of pages will
be the number of pages in one of the books
on the lower shelf or the average number
of pages in two books on the lower shelf.
Hence, the median will be at least 475;
NOT sufficient.

(2)

An analysis very similar to that used in (1)
shows the information given is not sufficient
to determine the median; NOT sufficient.

Given both (1) and (2), it follows that there is
a total of 49 books. Therefore, the median will
be the 25th book when the books are ordered
by number of pages. Since the 25th book in this
ordering is the book on the upper shelf with the
greatest number of pages, the median is 400.
Therefore, (1) and (2) together are sufficient.

Arithmetic Properties of numbers

If w ≥ 10, then (10, w) = 10, and if w < 10,
then (10, w) = w. Therefore, the value of
min(10, w) can be determined if the value of
w can be determined.
(1)

Given that w = max(20, z) then w ≥ 20.
Hence, w ≥ 10, and so min(10, w) = 10;
SUFFICIENT.

(2)

Given that w = max(10, w), then w ≥ 10,
and so min(10, w) = 10; SUFFICIENT.

The correct answer is D;
each statement alone is sufficient.
342

The correct answer is C;
both statements together are sufficient.

6.5 Data Sufficiency Answer Explanations

Arithmetic Applied problems; Estimating

(1)
A

B C D

E

379. In the figure above, points A, B, C, D, and E lie on a
line. A is on both circles, B is the center of the smaller
circle, C is the center of the larger circle, D is on the
smaller circle, and E is on the larger circle. What is the
area of the region inside the larger circle and outside
the smaller circle?
(1)

AB = 3 and BC = 2

(2)

CD = 1 and DE = 4

Geometry Circles

If R is the radius of the larger circle and r is the
radius of the smaller circle, then the desired area
is πR2 − πr2. Thus, if both the values of R and r
can be determined, then the desired area can be
determined.
(1)

Given that AB = r = 3 and BC = 2, then
AB + BC = R = 3 + 2 = 5; SUFFICIENT.

(2)

Given that CD = 1 and DE = 4, then
CD + DE = R = 1 + 4 = 5. Since AE is a
diameter of the larger circle, then
AD + DE = 2R. Also, since AD is a diameter
of the smaller circle, then AD = 2r. Thus,
2r + DE = 2R or 2r + 4 = 10, and so r = 3;
SUFFICIENT.

The correct answer is D;
each statement alone is sufficient.
380. In planning for a trip, Joan estimated both the distance
of the trip, in miles, and her average speed, in miles
per hour. She accurately divided her estimated
distance by her estimated average speed to obtain an
estimate for the time, in hours, that the trip would take.
Was her estimate within 0.5 hour of the actual time
that the trip took?
(1)

Joan’s estimate for the distance was within
5 miles of the actual distance.

(2)

Joan’s estimate for her average speed was within
10 miles per hour of her actual average speed.

Given that Joan’s estimate for the distance
was within 5 miles of the actual distance,
it is not possible to determine whether her
estimate for the time was within 0.5 hour
without information about her estimated
average speed. For example, if her estimated
distance was 20 miles and was within
5 miles of the actual distance, then the
actual distance would be between 15 and
25 miles. If her estimated speed was
20 miles per hour (mph) and was within,
say, 10 mph of her actual speed, then
her actual speed would be between 10 and
30 mph. Her estimated time would then
be 20 = 1.0 hour and the actual time would
20
15
= 0.5 hour (least distance
be between
30
over greatest speed) and 25 = 2.5 hours
10
(greatest distance over least speed). Since
1.0 hour is between 0.5 hour and 2.5 hours,
her estimate for the time could equal the
actual time, and thus it is possible that
her estimate of the time is within 0.5 hour
of the actual time. However, her estimate
for the time could be as much as
2.5 − 1.0 = 1.5 hours over the actual time,
and thus it is possible that her estimate
of the time is not within 0.5 hour; NOT
sufficient.

(2)

Given that Joan’s estimate for her average
speed was within 10 miles per hour of her
actual average speed, the same examples
used in (1) can be used to show that it
cannot be determined whether her estimate
for the time would be within 0.5 hour of the
actual time; NOT sufficient.

Taking (1) and (2) together is of no more help
than either (1) or (2) taken separately because
the same examples used to show that (1) is not
sufficient also show that (2) is not sufficient.
The correct answer is E;
both statements together are still not sufficient.

343

GMAT® Official Guide 2018

381. A certain list consists of 3 different numbers. Does the
median of the 3 numbers equal the average (arithmetic
mean) of the 3 numbers?
(1)

The range of the 3 numbers is equal to twice the
difference between the greatest number and the
median.

(2)

The sum of the 3 numbers is equal to 3 times
one of the numbers.

Arithmetic Statistics

Geometry Coordinate geometry

Since the line does not pass through the origin,
the line is either vertical and given by the
equation x = c for some constant c such that
c ≠ 0, or the line is not vertical and given by the
equation y = mx + b for some constants m and b
such that b ≠ 0. Determine whether the line is not
vertical, and if so, determine the slope of the line,
which is the value of m.
(1)

Let the numbers be x, y, and z so that x < y < z.
x+ y+z
Determine whether y =
, or equivalently,
3
whether 3y = x + y + z, or equivalently,
whether 2y = x + z.
(1)

Given that the range is equal to twice the
difference between the greatest number and
the median, it follows that z − x = 2(z − y),
or z − x = 2z − 2y, or 2y = x + z;
SUFFICIENT.

(2)

Given that the sum of the 3 numbers equals
3 times one of the numbers, it follows that
x + y + z = 3x or x + y + z = 3y or
x + y + z = 3z. If x + y + z = 3x, then
y + z = 2x, or (y − x) + (z − x) = 0. Also,
if x + y + z = 3z, then x + y = 2z, or
0 = (z − x) + (z − y). In each of these cases,
the sum of two positive numbers is zero,
which is impossible. Therefore, it must
be true that x + y + z = 3y, from which it
follows that x + z = 2y, and hence by the
initial comments, the median of the
3 numbers equals the average of the
3 numbers; SUFFICIENT.

(2)

Given that the x-intercept of the line is
twice the y-intercept of the line, it follows
that the line is not vertical, since a vertical
line that does not pass through the origin
will not have a y-intercept. Thus, the line
is given by the equation y = mx + b. The
x-intercept of the line is the solution to
0 = mx + b, or mx = –b, which has solution
b
x = – , and the y-intercept of the line is b.
m
Therefore, – b = 2b. Since b ≠ 0, both sides
m
of the last equation can be divided by b to
1
get – = 2, or m = –1/2; SUFFICIENT.
m
Given that the x- and y-intercepts of the
line are both positive, it is not possible to
determine the slope of the line. For example,
if the line is given by y = –x + 1, then the
x-intercept is 1 (solve –x + 1 = 0), the
y-intercept is 1 (b = 1), and the slope is –1.
However, if the line is given by y = –2x + 2,
then the x-intercept is 1 (solve –2x + 2 = 0),
the y-intercept is 2 (b = 2), and the slope is
–2; NOT sufficient.

The correct answer is A;
statement 1 alone is sufficient.

The correct answer is D;
each statement alone is sufficient.

y = ax − 5
y= x+6
y = 3x + b

382. Line ℓ lies in the xy-plane and does not pass through
the origin. What is the slope of line ℓ ?

344

(1)

The x-intercept of line ℓ is twice the y-intercept
of line ℓ.

(2)

The x- and y-intercepts of line ℓ are both positive.

383. In the xy‐plane, the straight‐line graphs of the three
equations above each contain the point (p,r). If a and
b are constants, what is the value of b ?
(1)

a=2

(2)

r = 17

6.5 Data Sufficiency Answer Explanations

which is 3(6) + 3(6) = 36, minus the
perimeter of triangle SWV; SUFFICIENT.

Algebra Coordinate geometry

Since (p,r) is on each of the lines, each of the
following three equations is true:

T

(i) r = ap − 5

Q

(ii) r = p + 6
(iii) r = 3p + b

3.5

Determine the value of b.
(1)

(2)

S

Given that a = 2, Equations (i) and
(ii) become r = 2p − 5 and r = p + 6.
Subtracting equations gives 0 = p − 11, or
p = 11. Now using r = p + 6, it follows that
r = 11 + 6 = 17. Finally, using p = 11 and
r = 17 in Equation (iii) gives
17 = 3(11) + b, or b = 17 − 33 = –16;
SUFFICIENT.

P
T
Q

(2)

W
U

S
R

384. In the figure above, PQR and STU are identical
equilateral triangles, and PQ = 6. What is the perimeter
of polygon PQWTUVR ?

(2)

VW has length 3.5.

Geometry Triangles; Perimeter

(1)

W
3.5
V

P

Q

Triangle SWV has perimeter 9.

U

S

T

(1)

R

R

Figure 2

The correct answer is D;
each statement alone is sufficient.

P

U

Figure 1

Given that r = 17, Equation (ii) becomes
17 = p + 6, and so p = 17 − 6 = 11. Using
r = 17 and p = 11, Equation (iii) becomes
17 = 3(11) + b, or b = 17 − 33 = –16;
SUFFICIENT.

V

W
3.5
V
3.5

Given that triangle SWV has perimeter 9,
the perimeter of polygon PQWTUVR
can be determined, since the perimeter of
polygon PQWTUVR equals the sum of the
perimeters of the two equilateral triangles,

Given that VW = 3.5, and using the fact
that the perimeter of polygon PQWTUVR
equals the sum of the perimeters of
the two equilateral triangles, which is
3(6) + 3(6) = 36, minus the perimeter of
triangle SWV, it is not possible to determine
the perimeter of polygon PQWTUVR.
For example, the perimeter of polygon
PQWTUVR could be 36 − 3(3.5) = 25.5
(arrange triangles PQR and STU so that
VW = WS = SV = 3.5, as shown in Figure 1),
and the perimeter of polygon PQWTUVR
could be greater than 25.5 (arrange triangles
PQR and STU so that VW = 3.5, WS is
slightly greater than 3.5, and SV is close to
0, as shown in Figure 2); NOT sufficient.

The correct answer is A;
statement 1 alone is sufficient.
385. The range of the numbers in set S is x, and the range
of the numbers in set T is y. If all of the numbers in set
T are also in set S, is x greater than y ?
(1)

Set S consists of 7 numbers.

(2)

Set T consists of 6 numbers.
345

GMAT® Official Guide 2018

Arithmetic Statistics

Set S has a range of x, set T has a range of y, and
T is a subset of S. Determine if x is greater than y.
(1)

(2)

It is given that S contains exactly 7 numbers,
but nothing additional is known about T.
Thus, if S = {1, 2, 3, 4, 5, 6, 7} and
T = {1, 2, 3, 4, 5, 6}, then x = 7 − 1 = 6,
y = 6 − 1 = 5, and x is greater than y. On the
other hand, if S = {1, 2, 3, 4, 5, 6, 7} and
T = {1, 3, 4, 5, 6, 7}, then x = 7 − 1 = 6,
y = 7 − 1 = 6, and x is not greater than y;
NOT sufficient.
It is given that T contains exactly 6 numbers,
but nothing additional is known about T.
Since the same examples given in (1) can
also be used in (2), it cannot be determined
if x is greater than y; NOT sufficient.

Taking (1) and (2) together, the examples used
in (1) can be used to show that it cannot be
determined if x is greater than y.
The correct answer is E;
both statements together are still not sufficient.
386. The hypotenuse of a right triangle is 10 cm. What is
the perimeter, in centimeters, of the triangle?
(1)

The area of the triangle is 25 square
centimeters.

(2)

The 2 legs of the triangle are of equal length.

Geometry Triangles

If x and y are the lengths of the legs of the
triangle, then it is given that x2 + y2 = 100.
Determining the value of x + y + 10, the
perimeter of the triangle, is equivalent to
determining the value of x + y.
(1)

Given that the area is 25, then 1 xy = 25, or
2
xy = 50. Since ( x + y )2 = x 2 + y 2 + 2xy , it
follows that ( x + y )2 = 100 + 2(50), or
x + y = 200; SUFFICIENT.

(2)

Given that x = y, since x2 + y2 = 100, it
follows that 2x2 = 100, or x = 50. Hence,
x + y = x + x = 2x = 2 50 ; SUFFICIENT.

The correct answer is D;
each statement alone is sufficient.
346

Shipment
Fraction of the
Total Value of the
Six Shipments

S1

S2

S3

S4

S5

S6

1
4

1
5

1
6

3
20

2
15

1
10

387. Six shipments of machine parts were shipped from a
factory on two trucks, with each shipment entirely on
one of the trucks. Each shipment was labeled either
S1, S2, S3, S4, S5, or S6. The table shows the value
of each shipment as a fraction of the total value of
the six shipments. If the shipments on the first truck
had a value greater than

1 of the total value of the six
2

shipments, was S3 shipped on the first truck?
(1)

S2 and S4 were shipped on the first truck.

(2)

S1 and S6 were shipped on the second truck.

Arithmetic Operations on rational numbers

Given that the shipments on the first truck had
a value greater than 1 of the total value of the
2
6 shipments, determine if S3 was shipped on the
first truck.
To avoid dealing with fractions, it will be
convenient to create scaled values of the
shipments by multiplying each fractional value
by 60, which is the least common denominator
of the fractions. Thus, the scaled values associated
with S1, S2, S3, S4, S5, and S6 are 15, 12, 10, 9,
8, and 6, respectively. The given information is
that the scaled value of the shipments on the first
truck is greater than ⎛ 1 ⎞ (60) = 30.
⎝ 2⎠
(1) Given that the first truck includes
shipments with scaled values 12 and 9, it
may or may not be the case that S3 (the
shipment with scaled value 10) is on the
first truck. For example, the first truck could
contain only S2, S3, and S4, for a total
scaled value 12 + 10 + 9 = 31 > 30. Or, the
first truck could contain only S1, S2, and S4,
for a total scaled value 15 + 12 + 9 = 36 > 30;
NOT sufficient.
(2)

Given that the second truck includes
shipments with scaled values 15 and 6, the
second truck cannot contain S3. Otherwise,
the second truck would contain shipments
with scaled values 15, 6, and 10, for a total

6.5 Data Sufficiency Answer Explanations

scaled value 15 + 6 + 10 = 31, leaving
at most a total scaled value 29 (which is
not greater than 30) for the first truck;
SUFFICIENT.
The correct answer is B;
statement 2 alone is sufficient.
388. If x, y, and z are three‐digit positive integers and if
x = y + z, is the hundreds digit of x equal to the sum
of the hundreds digits of y and z ?

Favorable

Unfavorable

Not Sure

Candidate M

40

20

40

Candidate N

30

35

35

389. The table above shows the results of a survey of
100 voters who each responded “Favorable” or
“Unfavorable” or “Not Sure” when asked about their
impressions of Candidate M and of Candidate N. What
was the number of voters who responded “Favorable”
for both candidates?

(1)

The tens digit of x is equal to the sum of the tens
digits of y and z.

(1)

The number of voters who did not respond
“Favorable” for either candidate was 40.

(2)

The units digit of x is equal to the sum of the
units digits of y and z.

(2)

The number of voters who responded
“Unfavorable” for both candidates was 10.

Arithmetic Place value

Letting x = 100a + 10b + c, y = 100p + 10q + r,
and z = 100t + 10u + v, where a, b, c, p, q, r, t, u,
and v are digits, determine if a = p + t.
(1)

(2)

It is given that b = q + u (which implies that
c + v ≤ 9 because if c + v > 9, then in the
addition process a ten would need to be
carried over to the tens column and b would
be q + u + 1). Since b is a digit, 0 ≤ b ≤ 9.
Hence, 0 ≤ q + u ≤ 9, and so 0 ≤ 10(q + u) ≤ 90.
Therefore, in the addition process, there are
no hundreds to carry over from the tens
column to the hundreds column, so a = p + t;
SUFFICIENT.
It is given that c = r + v. If x = 687, y = 231,
and z = 456, then, y + z = 231 + 456 =
687 = x, r + v = 1 + 6 = 7 = c, and
p + t = 2 + 4 = 6 = a. On the other hand,
if x = 637, y = 392, and z = 245, then y + z =
392 + 245 = 637 = x, r + v = 2 + 5 = 7 = c,
and p + t = 3 + 2 = 5 ≠ 6 = a; NOT
sufficient.

The correct answer is A;
statement 1 alone is sufficient.

Arithmetic Sets

If x is the number of voters who responded
“Favorable” for both candidates, then it follows
from the table that the number of voters who
responded “Favorable” to at least one candidate
is 40 + 30 – x = 70 – x. This is because 40 + 30
represents the number of voters who responded
“Favorable” for Candidate M added to the
number of voters who responded “Favorable” for
Candidate N, a calculation that counts twice each
of the x voters who responded “Favorable” for
both candidates.
(1)

Given that there were 40 voters who did not
respond “Favorable” for either candidate and
there were 100 voters surveyed, the number
of voters who responded “Favorable” to
at least one candidate is 100 − 40 = 60.
Therefore, from the comments above, it
follows that 70 − x = 60, and hence x = 10;
SUFFICIENT.

(2)

The information given affects only the
numbers of voters in the categories
“Unfavorable” for Candidate M only,
“Unfavorable” for Candidate N only, and
“Unfavorable” for both candidates. Thus,
the numbers of voters in the categories
“Favorable” for Candidate M only,
“Favorable” for Candidate N only, and
“Favorable” for both candidates are not
affected. Since these latter categories are
only constrained to have certain integer
values that have a total sum of 70 − x, more
347

GMAT® Official Guide 2018

than one possibility exists for the value of x.
For example, the numbers of voters in the
categories “Favorable” for Candidate M
only, “Favorable” for Candidate N only,
and “Favorable” for both candidates could
be 25, 15, and 15, respectively, which gives
70 – x = 25 + 15 + 15, or x = 15. However,
the numbers of voters in the categories
“Favorable” for Candidate M only,
“Favorable” for Candidate N only, and
“Favorable” for both candidates could be
30, 20, and 10, respectively, which gives
70 − x = 30 + 20 + 10, or x = 10; NOT
sufficient.
The correct answer is A;
statement 1 alone is sufficient.
390. A school administrator will assign each student in a group
of n students to one of m classrooms. If 3 < m < 13 < n,
is it possible to assign each of the n students to one
of the m classrooms so that each classroom has the
same number of students assigned to it?
(1)

It is possible to assign each of 3n students
to one of m classrooms so that each
classroom has the same number of students
assigned to it.

(2)

It is possible to assign each of 13n students
to one of m classrooms so that each
classroom has the same number of students
assigned to it.

Arithmetic Properties of numbers

Determine if n is divisible by m.
(1)

Given that 3n is divisible by m, then n is
divisible by m if m = 9 and n = 27 (note that
3 < m < 13 < n, 3n = 81, and m = 9, so 3n is
divisible by m) and n is not divisible by m if
m = 9 and n = 30 (note that 3 < m < 13 < n,
3n = 90, and m = 9, so 3n is divisible by m);
NOT sufficient.

Given that 13n is divisible by m, then
q
13n = qm, or n = , for some integer q.
m 13
Since 13 is a prime number that divides
qm (because 13n = qm) and 13 does not
divide m (because m < 13), it follows that 13
divides q.

(2)

348

q
Therefore,
is an integer, and since
13
n = q
n
, then is an integer. Thus, n is divisible
m 13
m
by m; SUFFICIENT.
The correct answer is B;
statement 2 alone is sufficient.
391. If q, s, and t are all different numbers, is q < s < t ?
(1)

t − q = |t − s| + |s − q|

(2)

t>q

Algebra Absolute value

(1)

It is given that t − q = |t − s| + |s − q|,
which can be rewritten without absolute
values in four mutually exclusive and
collectively exhaustive cases by making use
of the algebraic definition of absolute value.
Recall that |x| = x if x > 0, and |x| = –x if
x < 0. Thus, for example, if t − s < 0, then
|t − s| = –(t − s).
Case 1: t > s and s > q. In this case, t − s > 0
and s − q > 0, and so t − q = |t − s| + |s − q|
is equivalent to t − q = (t − s) + (s − q),
which is an identity. Therefore, the case
for which t > s and s > q is consistent with
the given information and the assumption
t − q = |t − s| + |s − q|.
Case 2: t > s and s < q. In this case, t − s > 0
and s − q < 0, and so t − q = |t − s| + |s − q|
is equivalent to t − q = (t − s) − (s − q),
or s = q, which is not consistent with the
assumption that q, s, and t are all different
numbers. Therefore, the case for which
t > s and s < q is not consistent with the
given information and the assumption
t − q = |t − s| + |s − q|.
Case 3: t < s and s > q. In this case, t − s < 0
and s − q > 0, and so t − q = |t − s| + |s − q|
is equivalent to t − q = –(t − s) + (s − q),
or t = s, which is not consistent with the
assumption that q, s, and t are all different
numbers. Therefore, the case for which
t < s and s > q is not consistent with the
given information and the assumption
t − q = |t − s| + |s − q|.

6.5 Data Sufficiency Answer Explanations

Case 4: t < s and s < q. In this case, t − s < 0
and s − q < 0, and so t − q = |t − s| + |s − q|
is equivalent to t − q = –(t − s) − (s − q),
or t = q, which is not consistent with the
assumption that q, s, and t are all different
numbers. Therefore, the case for which
t < s and s < q is not consistent with the
given information and the assumption
t − q = |t − s| + |s − q|. The only case that
is consistent with the given information
and the assumption t − q = |t − s| + |s − q|
is Case 1. Therefore, it follows that t > s
and s > q, and this implies q < s < t;
SUFFICIENT.
(2)

Given that t > q, it is possible that q < s < t
is true (for example, when s is between t and
q) and it is possible that q < s < t is false (for
example, when s is greater than t); NOT
sufficient.

The correct answer is A;
statement 1 alone is sufficient.
392. What is the median number of employees assigned
per project for the projects at Company Z ?

that have exactly 2 employees each and
65 projects that have exactly 3 employees
each) or the median could be 4 (35 projects
that have exactly 2 employees each and
65 projects that have exactly 4 employees
each); NOT sufficient.
Given both (1) and (2), 100 − (25 + 35) percent =
40 percent of the projects have exactly
3 employees. Therefore, when the numbers of
employees per project are listed from least to
greatest, 35 percent of the numbers are 2 or less
and (35 + 40) percent = 75 percent are 3 or less,
and hence the median is 3.
The correct answer is C;
both statements together are sufficient.
393. Last year, a certain company began manufacturing
product X and sold every unit of product X that it
produced. Last year the company’s total expenses
for manufacturing product X were equal to $100,000
plus 5 percent of the company’s total revenue from all
units of product X sold. If the company made a profit
on product X last year, did the company sell more than
21,000 units of product X last year?

(1)

25 percent of the projects at Company Z have 4
or more employees assigned to each project.

(1)

The company’s total revenue from the sale of
product X last year was greater than $110,000.

(2)

35 percent of the projects at Company Z have 2
or fewer employees assigned to each project.

(2)

For each unit of product X sold last year, the
company’s revenue was $5.

Arithmetic Statistics

Algebra Applied problems

(1)

For a company that made a profit last year
from selling product X and had total expenses
for product X of $100,000 + 0.05R, where
R is the total revenue for selling product X,
determine whether the company sold more than
21,000 units of product X last year.

(2)

Although 25 percent of the projects have 4
or more employees, there is essentially no
information about the middle values of
the numbers of employees per project. For
example, if there were a total of 100 projects,
then the median could be 2 (75 projects
that have exactly 2 employees each and
25 projects that have exactly 4 employees
each) or the median could be 3 (75 projects
that have exactly 3 employees each and
25 projects that have exactly 4 employees
each); NOT sufficient.
Although 35 percent of the projects have 2
or fewer employees, there is essentially no
information about the middle values of
the numbers of employees per project. For
example, if there were a total of 100 projects,
then the median could be 3 (35 projects

Note that since the company made a profit,
revenue − cost, which is given by
R − ($100,000 + 0.05R) = 0.95R − $100,000,
must be positive.
(1)

It is given that R > $110,000. It is possible
to vary the unit price and the number of
units sold so that R > $110,000 and more
than 21,000 units were sold, and also so that
R > $110,000 and less than 21,000 units
were sold. For example, if 25,000 units
were sold for $10 per unit, then
349

GMAT® Official Guide 2018

R = 25,000($10) = $250,000 > $110,000
and 25,000 > 21,000. On the other hand, if
20,000 units were sold for $10 per unit, then
R = 20,000($10) = $200,000 > $110,000
and 20,000 < 21,000; NOT sufficient.
(2)

It is given that the company’s revenue for
each unit of product X was $5. If the
company manufactured and sold x units
of product X, then its revenue was $5x.
Because the company made a profit,
0.95($5x) − $100,000 > 0, and so

0.95($5x) − $100,000
$4.75x − $100,000
$4.75x
x

>
>
>
>

0
0
$100,000
21,052; SUFFICIENT.

To avoid long division in the last step, note
that 4.75(21,000) = 99,750, and thus from
4.75x > 100,000, it follows that x > 21,000.
The correct answer is B;
statement 2 alone is sufficient.
394. Beginning in January of last year, Carl made deposits
of $120 into his account on the 15th of each month
for several consecutive months and then made
withdrawals of $50 from the account on the 15th of
each of the remaining months of last year. There were
no other transactions in the account last year. If the
closing balance of Carl’s account for May of last year
was $2,600, what was the range of the monthly
closing balances of Carl’s account last year?
(1)

Last year the closing balance of Carl’s account
for April was less than $2,625.

(2)

Last year the closing balance of Carl’s account
for June was less than $2,675.

Arithmetic Statistics

(1)

350

$2,000 on January 1, made $120 deposits in
each of the first 11 months of the year, and
then made a $50 withdrawal on December 15,
which gives a range of monthly closing
balances of (120)(10). Also, Carl could
have had an account balance of $2,000 on
January 1, made $120 deposits in each of
the first 10 months of the year, and then
made $50 withdrawals on November 15
and on December 15, which gives a range
of monthly closing balances of (120)(9);
NOT sufficient.

If Carl began making $50 withdrawals on
or before May 15, his account balance on
April 16 would be at least $50 greater than
it was on the last day of May. Thus, his account
balance on April 16 would be at least
$2,600 + $50 = $2,650, which is contrary to
the information given in (1). Therefore,
Carl did not begin making $50 withdrawals
until June 15 or later. These observations can
be used to give at least two possible ranges.
Carl could have had an account balance of

(2)

On June 1, Carl’s account balance was the
same as its closing balance was for May,
namely $2,600. Depending on whether Carl
made a $120 deposit or a $50 withdrawal
on June 15, Carl’s account balance on June 16
was either $2,720 or $2,550. It follows from
the information given in (2) that Carl’s
balance on June 16 was $2,550. Therefore,
Carl began making $50 withdrawals on or
before June 15. These observations can be
used to give at least two possible ranges.
Carl could have had an account balance of
$2,680 on January 1, made one $120 deposit
on January 15, and then made a $50
withdrawal in each of the remaining
11 months of the year (this gives a closing
balance of $2,600 for May), which gives
a range of monthly closing balances of
(50)(11). Also, Carl could have had an account
balance of $2,510 on January 1, made $120
deposits on January 15 and on February 15,
and then made a $50 withdrawal in each of
the remaining 10 months of the year (this
gives a closing balance of $2,600 for May),
which gives a range of monthly closing
balances of (50)(10); NOT sufficient.

Given both (1) and (2), it follows from the
remarks above that Carl began making $50
withdrawals on June 15. Therefore, the changes
to Carl’s account balance for each month of last
year are known. Since the closing balance for
May is given, it follows that the closing balances
for each month of last year are known, and
hence the range of these 12 known values can be
determined.
The correct answer is C;
both statements together are sufficient.

6.5 Data Sufficiency Answer Explanations

395. Are all of the numbers in a certain list of 15 numbers
equal?

396. If the average (arithmetic mean) of six numbers is 75,
how many of the numbers are equal to 75 ?

(1)

The sum of all the numbers in the list is 60.

(1)

None of the six numbers is less than 75.

(2)

The sum of any 3 numbers in the list is 12.

(2)

None of the six numbers is greater than 75.

Arithmetic Properties of numbers

Arithmetic Statistics

(1)

If the average of six numbers is 75, then
1
of the sum of the numbers is 75. Therefore,
6
the sum of the numbers is (6)(75).

(2)

If there are 15 occurrences of the number 4
in the list, then the sum of the numbers in
the list is 60 and all the numbers in the list
are equal. If there are 13 occurrences of the
number 4 in the list, 1 occurrence of the
number 3 in the list, and 1 occurrence of
the number 5 in the list, then the sum of the
numbers in the list is 60 and not all the
numbers in the list are equal; NOT
sufficient.

(1)

If one of the numbers is greater than 75,
then we can write that number as 75 + x
for some positive number x. Consequently,
the sum of the 6 numbers must be at least
(5)(75) + (75 + x) = (6)(75) + x, which is
greater than (6)(75), contrary to the fact
that the sum is equal to (6)(75). Hence,
none of the numbers can be greater than 75.
Since none of the numbers can be less than 75
(given information) and none of the
numbers can be greater than 75, it follows
that each of the numbers is equal to 75;
SUFFICIENT.

(2)

If one of the numbers is less than 75, then
we can write that number as 75 − x for some
positive number x. Consequently, the sum
of the 6 numbers must be at most (5)(75) +
(75 − x) = (6)(75) − x, which is less than
(6)(75), contrary to the fact that the sum
is equal to (6)(75). Hence, none of the
numbers can be less than 75. Since none of
the numbers can be less than 75 and none
of the numbers can be greater than 75
(given information), it follows that each of
the numbers is equal to 75; SUFFICIENT.

Given that the sum of any 3 numbers in the
list is 12, arrange the numbers in the list in
numerical order, from least to greatest:
a1 ≤ a2 ≤ a3 ≤ . . . ≤ a15.
If a1 < 4, then a1 + a2 + a3 < 4 + a2 + a3.
Therefore, from (2), 12 < 4 + a2 + a3, or
8 < a2 + a3, and so at least one of the values
a2 and a3 must be greater than 4. Because
a2 ≤ a3, it follows that a3 > 4. Since the
numbers are arranged from least to greatest,
it follows that a4 > 4 and a5 > 4. But then,
a3 + a4 + a5 > 4 + 4 + 4 = 12, contrary to (2),
and so a1 < 4 is not true. Therefore, a1 ≥ 4.
Since a1 is the least of the 15 numbers,
an ≥ 4 for n = 1, 2, 3, …, 15.
If a15 > 4, then a13 + a14 + a15 > a13 + a14 + 4.
Therefore, from (2), 12 > a13 + a14 + 4, or
8 > a13 + a14, and so at least one of the values
a13 and a14 must be less than 4. Because
a13 ≤ a14, it follows that a13 < 4. Since the
numbers are arranged from least to greatest,
it follows that a11 < 4 and a12 < 4. But then
a11 + a12 + a13 < 4 + 4 + 4 = 12, contrary
to (2). Therefore, a15 ≤ 4. Since a15 is the
greatest of the 15 numbers, an ≤ 4
for n = 1, 2, 3, …, 15.
It has been shown that, for n = 1, 2, 3, …, 15,
each of an ≥ 4 and an ≤ 4 is true. Therefore,
an = 4 for n = 1, 2, 3, …, 15; SUFFICIENT.

The correct answer is D;
each statement alone is sufficient.
397. What amount did Jean earn from the commission on
her sales in the first half of 1988 ?
(1)

In 1988 Jean’s commission was 5 percent of the
total amount of her sales.

(2)

The amount of Jean’s sales in the second half of
1988 averaged $10,000 per month more than in
the first half.

The correct answer is B;
statement 2 alone is sufficient.
351

GMAT® Official Guide 2018

Arithmetic Applied problems

Let A be the amount of Jean’s sales in the first
half of 1988. Determine the value of A.
(1)

(2)

If the amount of Jean’s sales in the first half
of 1988 was $10,000, then her commission
in the first half of 1988 would have been
(5%)($10,000) = $500. On the other hand,
if the amount of Jean’s sales in the first half
of 1988 was $100,000, then her commission
in the first half of 1988 would have been
(5%)($100,000) = $5,000; NOT sufficient.
No information is given that relates the
amount of Jean’s sales to the amount of
Jean’s commission; NOT sufficient.

the square region C) and h is the height of the
triangle (and is equal to the length of a side of
the square region B). The area of any square is
equal to the length of a side squared. The
Pythagorean theorem is used to find the length
of a side of a right triangle, when the length of
the other 2 sides of the triangle are known and is
represented by a2 + b2 = c2, where a and b are the
lengths of the 2 perpendicular sides of the triangle
and c is the length of the hypotenuse.
Although completed calculations are provided
in what follows, keep in mind that completed
calculations are not needed to solve this problem.
(1)

Given (1) and (2), from (1) the amount of Jean’s
commission in the first half of 1988 is (5%)A.
From (2) the amount of Jean’s sales in the second
half of 1988 is A + $60,000. Both statements
together do not give information to determine
the value of A.

2

can be used: ⎛ 8 ⎞ + 32 = c 2 or 64 + 9 = c 2
⎝ 3⎠
9
145
2
or
= c . The length of a side of A is thus
9
145
; SUFFICIENT.
9

The correct answer is E;
both statements together are still not sufficient.

(2)
A

D

If the area of B is 9, then the length of each
side is 3. Therefore, h = 3. Then, b can be
determined, since the area of the triangle is,
by substitution, 4 = 1 (3b ) or 8 = 3b or 8 = b.
2
3
Once b is known, the Pythagorean theorem

B

C

If the area of C is 64 , then the length of
9
8
each side is 8 . Therefore, b = . The area
3
3
1
of a triangle is A = bh so 4 = 1 ⎛ 8 h ⎞ ,
2
2⎝3 ⎠
8
8 = h , and 3 = h. Once h is known, the
3
Pythagorean theorem can be used as above;
SUFFICIENT.

The correct answer is D;
each statement alone is sufficient.
398. In the figure above, if the area of triangular region D
is 4, what is the length of a side of square region A ?
(1)
(2)

The area of square region B is 9.
64
.
The area of square region C is
9

Geometry Area

The area of the triangular region D can be
1
represented by bh, where b is the base of the
2
triangle (and is equal to the length of a side of
352

399. If n is a positive integer and k = 5.1 × 10n, what is the
value of k ?
(1)

6,000 < k < 500,000

(2)

k2 = 2.601 × 109

6.5 Data Sufficiency Answer Explanations

Arithmetic Properties of numbers

quadrilateral formed by the four lines and
a + b + c + d = 360. Then

Given that k = 5.1 ×
n is a positive
integer, then the value of k must follow the
pattern shown in the following table:
10n, where

n
1
2
3
4
5
6
.
.
.

Given that 6,000 < k < 500,000, then k
must have the value 51,000, and so n = 4;
SUFFICIENT.

(2)

Given that k2 = 2.601 × 109, then

k = 2.601 × 10 = 2,601 × 10 = 2,601 × 10
9

6

= 51 × 10 3 = 51, 000, and so n = 4;
SUFFICIENT.
The correct answer is D;
each statement alone is sufficient.

y˚

x˚

z˚
400. What is the value of x + y in the figure above?

(2)

z = 125

= 720 − ( a + b + c + d )
= 720 − 360
= 360.
Determine the value of x + y.

6

(1)

Given that w = 95, then 95 + x + y + z = 360
and x + y + z = 265. If z = 65, for example,
then x + y = 200. On the other hand,
if z = 100, then x + y = 165; NOT sufficient.

(2)

Given that z =125, then w + x + y + 125 =
360 and w + x + y = 235. If w = 35, for
example, then x + y = 200. On the other
hand, if w = 100, then x + y = 135; NOT
sufficient.

Taking (1) and (2) together, 95 + x + y + 125 = 360,
and so x + y = 140.
The correct answer is C;
both statements together are sufficient.
401. If n and k are positive integers, is n + k > 2 n ?
(1)

k > 3n

(2)

n + k > 3n

Algebra Inequalities

w˚

w = 95

= (180 − a ) + (180 − d ) + (180 − c ) + (180 − b )

k
51
510
5,100
51,000
510,000
5,100,000
.
.
.

(1)

(1)

w+x+ y+z

Determine if n + k > 2 n . Since each side
is positive, squaring each side preserves the
inequality, so n + k > 2 n is equivalent to
( n + k )( n + k ) > (2 n )(2 n ), which in turn is
equivalent to n + k > 4n, or to k > 3n.

Geometry Angles

x° d°
w° a°

c° y°
b°
z°

In the figure above, a, b, c, and d are the
degree measures of the interior angles of the

(1)

Given that k > 3n, then n + k > 2 n ;
SUFFICIENT.

(2)

Given that n + k > 3n, then k > 2n. However,
it is possible for k > 2n to be true and k > 3n
to be false (for example, k = 3 and n = 1) and
it is possible for k > 2n to be true and k > 3n
to be true (for example, k = 4 and n = 1);
NOT sufficient.

The correct answer is A;
statement 1 alone is sufficient.
353

GMAT® Official Guide 2018

402. In a certain business, production index p is directly
proportional to efficiency index e, which is in turn
directly proportional to investment index i. What is p if
i = 70 ?
(1)

e = 0.5 whenever i = 60.

(2)

p = 2.0 whenever i = 50.

Arithmetic Properties of numbers

(1)

Given that the hundreds digit of 10n is 6,
the tens digit of n is 6, since the hundreds
digit of 10n is always equal to the tens digit
of n; SUFFICIENT.

(2)

Given that the tens digit of n + 1 is 7, it
is possible that the tens digit of n is 7 (for
example, n = 70) and it is possible that the
tens digit of n is 6 (for example, n = 69);
NOT sufficient.

Arithmetic Proportions

(1)

(2)

This gives only values for e and i, and, while
p is directly proportional to e, the nature
of this proportion is unknown. Therefore,
p cannot be determined; NOT sufficient.
Since p is directly proportional to e, which is
directly proportional to i, then p is directly
proportional to i. Therefore, the following
p 2.0
. If i = 70,
proportion can be set up: =
50
i
p 2.0
then
. Through cross multiplying,
=
70 50
this equation yields 50p = 140, or p = 2.8;
SUFFICIENT.

The correct answer is A;
statement 1 alone is sufficient.
404. What is the value of
(1)

2t
=3
t−x

(2)

t−x=5

2t + t − x
?
t−x

Algebra Simplifying algebraic expressions

The preceding approach is one method that can
be used. Another approach is as follows: It is
given that p = Ke = K(Li) = (KL)i, where K and L
are the proportionality constants, and the value of
70KL is to be determined. Statement (1) allows
us to determine the value of L, but gives nothing
about K, and thus (1) is not sufficient. Statement
(2) allows us to determine the value of KL, and
thus (2) is sufficient.

Determine the value of 2t + t − x .
t −x
(1) Since 2t = 3 and
t −x
2t + t − x = 2t + t − x = 2t + 1,
t −x
t −x t −x t −x
it follows that 2t + t − x = 3 + 1;
t −x

The correct answer is B;
statement 2 alone is sufficient.

(2)

403. If n is a positive integer, what is the tens digit of n ?
(1)

The hundreds digit of 10n is 6.

(2)

The tens digit of n + 1 is 7.

SUFFICIENT.
Given that t − x = 5, it follows that
2t + t − x = 2t + 5 = 2 t + 1, which can
t −x
5
5
vary when the value of t varies. For example,
2 t + 1 = 3 if t = 5 (choose x = 0 to have
5
t − x = 5) and 2 t + 1 = 5 if t = 10 (choose
5
x = 5 to have t − x = 5); NOT sufficient.

The correct answer is A;
statement 1 alone is sufficient.

354

6.5 Data Sufficiency Answer Explanations

To register for the GMAT exam go to www.mba.com
355

7.0 Reading Comprehension

356

7.0 Reading Comprehension Verbal Questions

7.0 Reading Comprehension
Reading comprehension questions appear in the Verbal section of the GMAT® exam. The Verbal
section uses multiple‐choice questions to measure your ability to read and comprehend written material,
to reason and evaluate arguments, and to correct written material to conform to standard written
English. Because the Verbal section includes content from a variety of topics, you may be generally
familiar with some of the material; however, neither the passages nor the questions assume knowledge
of the topics discussed. Reading comprehension questions are intermingled with critical reasoning and
sentence correction questions throughout the Verbal section of the test.
You will have 75 minutes to complete the Verbal section, or an average of about 1¾ minutes to answer
each question. Keep in mind you will need time to read the written passages—and that time is not
factored into the 1¾ minute average. Therefore, you should plan to proceed more quickly through the
reading comprehension questions in order to give yourself enough time to read the passages thoroughly.
Reading comprehension questions begin with written passages up to 350 words long. The passages
discuss topics from the social sciences, humanities, the physical or biological sciences, and such
business‐related fields as marketing, economics, and human resource management. The passages are
accompanied by questions that will ask you to interpret the passage, apply the information you gather
from the reading, and make inferences (or informed assumptions) based on the reading. For these
questions, you will see a split computer screen. The written passage will remain visible on the left side
as each question associated with that passage appears, in turn, on the right side. You will see only one
question at a time. However, the number of questions associated with each passage may vary.
As you move through the reading comprehension sample questions, try to determine a process that
works best for you. You might begin by reading a passage carefully and thoroughly. Some test‐takers
prefer to skim the passages the first time through, or even to read the first question before reading the
passage. You may want to reread any sentences that present complicated ideas or introduce terms that
are new to you. Read each question and series of answers carefully. Make sure you understand exactly
what the question is asking and what the answer choices are.
If you need to, you may go back to the passage and read any parts that are relevant to answering the
question. Specific portions of the passages may be indicated in the related questions.
The following pages describe what reading comprehension questions are designed to measure, present
the directions that will precede questions of this type, and describe the various question types. This
chapter also provides test‐taking strategies, sample questions, and detailed explanations of all the
questions. The explanations further illustrate the ways in which reading comprehension questions
evaluate basic reading skills.

357

GMAT® Official Guide 2018

7.1 What Is Measured
Reading comprehension questions measure your ability to understand, analyze, and apply information
and concepts presented in written form. All questions are to be answered on the basis of what is stated
or implied in the reading material, and no specific prior knowledge of the material is required.
The GMAT reading comprehension questions evaluate your ability to do the following:
• Understand words and statements.
Although the questions do not test your vocabulary (they will not ask you to define terms), they
do test your ability to interpret special meanings of terms as they are used in the reading passages.
The questions will also test your understanding of the English language. These questions may ask
about the overall meaning of a passage.
• Understand logical relationships between points and concepts.
This type of question may ask you to determine the strong and weak points of an argument or
evaluate the relative importance of arguments and ideas in a passage.
• Draw inferences from facts and statements.
The inference questions will ask you to consider factual statements or information presented in a
reading passage and reach conclusions on the basis of that information.
• Understand and follow the development of quantitative concepts as they are presented in
written material.

This may involve the interpretation of numerical data or the use of simple arithmetic to reach
conclusions about material in a passage.
There are six kinds of reading comprehension questions, each of which tests a different skill. The
reading comprehension questions ask about the following areas:

Main idea
Each passage is a unified whole—that is, the individual sentences and paragraphs support and develop
one main idea or central point. Sometimes you will be told the central point in the passage itself, and
sometimes it will be necessary for you to determine the central point from the overall organization or
development of the passage. You may be asked in this kind of question to
• recognize a correct restatement, or paraphrasing, of the main idea of a passage
• identify the author’s primary purpose or objective in writing the passage
• assign a title that summarizes, briefly and pointedly, the main idea developed in the passage

Supporting ideas
These questions measure your ability to comprehend the supporting ideas in a passage and differentiate
them from the main idea. The questions also measure your ability to differentiate ideas that are explicitly
stated in a passage from ideas that are implied by the author but are not explicitly stated. You may be
asked about
• facts cited in a passage
• the specific content of arguments presented by the author in support of his or her views
• descriptive details used to support or elaborate on the main idea
358

7.1 Reading Comprehension What Is Measured

Whereas questions about the main idea ask you to determine the meaning of a passage as a whole,
questions about supporting ideas ask you to determine the meanings of individual sentences and
paragraphs that contribute to the meaning of the passage as a whole. In other words, these questions ask
for the main point of one small part of the passage.

Inferences
These questions ask about ideas that are not explicitly stated in a passage but are implied by the author.
Unlike questions about supporting details, which ask about information that is directly stated in a
passage, inference questions ask about ideas or meanings that must be inferred from information that
is directly stated. Authors can make their points in indirect ways, suggesting ideas without actually
stating them. Inference questions measure your ability to understand an author’s intended meaning in
parts of a passage where the meaning is only suggested. These questions do not ask about meanings
or implications that are remote from the passage; rather, they ask about meanings that are developed
indirectly or implications that are specifically suggested by the author.
To answer these questions, you may have to
• logically take statements made by the author one step beyond their literal meanings
• recognize an alternative interpretation of a statement made by the author
• identify the intended meaning of a word used figuratively in a passage
If a passage explicitly states an effect, for example, you may be asked to infer its cause. If the author
compares two phenomena, you may be asked to infer the basis for the comparison. You may be asked
to infer the characteristics of an old policy from an explicit description of a new one. When you read a
passage, you should concentrate not only on the explicit meaning of the author’s words, but also on the
more subtle meaning implied by those words.

Applying information to a context outside the passage itself
These questions measure your ability to discern the relationships between situations or ideas presented
by the author and other situations or ideas that might parallel those in the passage. In this kind of
question, you may be asked to
• identify a hypothetical situation that is comparable to a situation presented in the passage
• select an example that is similar to an example provided in the passage
• apply ideas given in the passage to a situation not mentioned by the author
• recognize ideas that the author would probably agree or disagree with on the basis of statements
made in the passage
Unlike inference questions, application questions use ideas or situations not taken from the passage.
Ideas and situations given in a question are like those given in the passage, and they parallel ideas and
situations in the passage; therefore, to answer the question, you must do more than recall what you read.
You must recognize the essential attributes of ideas and situations presented in the passage when they
appear in different words and in an entirely new context.

359

GMAT® Official Guide 2018

Logical structure
These questions require you to analyze and evaluate the organization and logic of a passage. They may
ask you
• how a passage is constructed—for instance, does it define, compare or contrast, present a new idea,
or refute an idea?
• how the author persuades readers to accept his or her assertions
• the reason behind the author’s use of any particular supporting detail
• to identify assumptions that the author is making
• to assess the strengths and weaknesses of the author’s arguments
• to recognize appropriate counterarguments
These questions measure your ability not only to comprehend a passage but also to evaluate it critically.
However, it is important for you to realize that logical structure questions do not rely on any kind of
formal logic, nor do they require you to be familiar with specific terms of logic or argumentation. You
can answer these questions using only the information in the passage and careful reasoning.

About the style and tone
Style and tone questions ask about the expression of a passage and about the ideas in a passage that may
be expressed through its diction—the author’s choice of words. You may be asked to deduce the author’s
attitude to an idea, a fact, or a situation from the words that he or she uses to describe it. You may
also be asked to select a word that accurately describes the tone of a passage—for instance, “critical,”
“questioning,” “objective,” or “enthusiastic.”
To answer this type of question, you will have to consider the language of the passage as a whole. It
takes more than one pointed, critical word to make the tone of an entire passage “critical.” Sometimes,
style and tone questions ask what audience the passage was probably intended for or what type of
publication it probably appeared in. Style and tone questions may apply to one small part of the passage
or to the passage as a whole. To answer them, you must ask yourself what meanings are contained in
the words of a passage beyond the literal meanings. Did the author use certain words because of their
emotional content, or because a particular audience would expect to hear them? Remember, these
questions measure your ability to discern meaning expressed by the author through his or her choice
of words.

7.2 Test-Taking Strategies
1. Do not expect to be completely familiar with any of the material presented in reading
comprehension passages.

You may find some passages easier to understand than others, but all passages are designed to
present a challenge. If you have some familiarity with the material presented in a passage, do not let
this knowledge influence your choice of answers to the questions. Answer all questions on the basis
of what is stated or implied in the passage itself.

360

7.2 Reading Comprehension Test-Taking Strategies

2. Analyze each passage carefully, because the questions require you to have a specific and detailed
understanding of the material.

You may find it easier to do the analysis first before moving to the questions. You may find that
you prefer to skim the passage the first time and read more carefully once you understand what
a question asks. You may even want to read the question before reading the passage. You should
choose the method most suitable for you.
3. Focus on key words and phrases, and make every effort to avoid losing the sense of what is
discussed in the passage.

Keep the following in mind:
• Note how each fact relates to an idea or an argument.
• Note where the passage moves from one idea to the next.
• Separate main ideas from supporting ideas.
• Determine what conclusions are reached and why.
4. Read the questions carefully, making certain that you understand what is asked.

An answer choice that accurately restates information in the passage may be incorrect if it does not
answer the question. If you need to, refer back to the passage for clarification.
5. Read all the choices carefully.

Never assume that you have selected the best answer without first reading all the choices.
6. Select the choice that answers the question best in terms of the information given in the passage.

Do not rely on outside knowledge of the material to help you answer the questions.
7. Remember that comprehension—not speed—is the critical success factor when it comes to reading
comprehension questions.

7.3 The Directions
These are the directions that you will see for reading comprehension questions when you take the
GMAT exam. If you read them carefully and understand them clearly before going to sit for the test,
you will not need to spend too much time reviewing them once you are at the test center and the test is
under way.
The questions in this group are based on the content of a passage. After reading the passage, choose the
best answer to each question. Answer all questions following the passage on the basis of what is stated or
implied in the passage.

361

GMAT® Official Guide 2018

7.4 Practice Questions
Each of the reading comprehension questions is based on the content of a passage. After reading the
passage answer all questions pertaining to it on the basis of what is stated or implied in the passage.
For each question, select the best answer of the choices given.

Line

(5)

(10)

(15)

(20)

(25)

(30)

(35)

362

Biologists have advanced two theories to explain
why schooling of fish occurs in so many fish species.
Because schooling is particularly widespread among
species of small fish, both theories assume that
schooling offers the advantage of some protection
from predators.
Proponents of theory A dispute the assumption
that a school of thousands of fish is highly visible.
Experiments have shown that any fish can be seen,
even in very clear water, only within a sphere of 200
meters in diameter. When fish are in a compact group,
the spheres of visibility overlap. Thus the chance of
a predator finding the school is only slightly greater
than the chance of the predator finding a single fish
swimming alone. Schooling is advantageous to the
individual fish because a predator’s chance of finding
any particular fish swimming in the school is much
smaller than its chance of finding at least one of
the same group of fish if the fish were dispersed
throughout an area.
However, critics of theory A point out that some
fish form schools even in areas where predators
are abundant and thus little possibility of escaping
detection exists. They argue that the school continues
to be of value to its members even after detection.
They advocate theory B, the “confusion effect,” which
can be explained in two different ways.
Sometimes, proponents argue, predators simply
cannot decide which fish to attack. This indecision
supposedly results from a predator’s preference
for striking prey that is distinct from the rest of the
school in appearance. In many schools the fish are
almost identical in appearance, making it difficult for a
predator to select one. The second explanation for
the “confusion effect” has to do with the sensory
confusion caused by a large number of prey moving
around the predator. Even if the predator

makes the decision to attack a particular fish,
the movement of other prey in the school can
(40) be distracting. The predator’s difficulty can
be compared to that of a tennis player trying
to hit a tennis ball when two are approaching
simultaneously.

7.4 Reading Comprehension Practice Questions

Questions 405–408 refer to the passage.

405. According to the passage, theory B states that which
of the following is a factor that enables a schooling fish
to escape predators?
(A)

The tendency of fish to form compact groups

(B)

The movement of other fish within the school

(C)

The inability of predators to detect schools

(D)

The ability of fish to hide behind one another in a
school

(E)

The great speed with which a school can
disperse

408. The author is primarily concerned with
(A)

discussing different theories

(B)

analyzing different techniques

(C)

defending two hypotheses

(D)

refuting established beliefs

(E)

revealing new evidence

406. According to the passage, both theory A and theory B
have been developed to explain how
(A)

fish hide from predators by forming schools

(B)

forming schools functions to protect fish from
predators

(C)

schooling among fish differs from other
protective behaviors

(D)

small fish are able to make rapid decisions

(E)

small fish are able to survive in an environment
densely populated by large predators

407. According to one explanation of the “confusion
effect,” a fish that swims in a school will have greater
advantages for survival if it
(A)

tends to be visible for no more than 200 meters

(B)

stays near either the front or the rear of a school

(C)

is part of a small school rather than a large
school

(D)

is very similar in appearance to the other fish in
the school

(E)

is medium‐sized

363

GMAT® Official Guide 2018

Line

(5)

(10)

(15)

(20)

(25)

364

Scientists long believed that two nerve clusters in
the human hypothalamus, called suprachiasmatic
nuclei (SCNs), were what controlled our circadian
rhythms. Those rhythms are the biological cycles
that recur approximately every 24 hours in
synchronization with the cycle of sunlight and
darkness caused by Earth’s rotation. Studies have
demonstrated that in some animals, the SCNs
control daily fluctuations in blood pressure, body
temperature, activity level, and alertness, as well as
the nighttime release of the sleep‐promoting agent
melatonin. Furthermore, cells in the human retina
dedicated to transmitting information about light
levels to the SCNs have recently been discovered.
Four critical genes governing circadian cycles
have been found to be active in every tissue,
however, not just the SCNs, of flies, mice, and
humans. In addition, when laboratory rats that
usually ate at will were fed only once a day, peak
activity of a clock gene in their livers shifted by
12 hours, whereas the same clock gene in the
SCNs remained synchronized with light cycles. While
scientists do not dispute the role of the SCNs in
controlling core functions such as the regulation of
body temperature and blood pressure, scientists
now believe that circadian clocks in other organs
and tissues may respond to external cues other than
light—including temperature changes—that recur
regularly every 24 hours.

Questions 409–411 refer to the passage.

409. The primary purpose of the passage is to
(A)

challenge recent findings that appear to
contradict earlier findings

(B)

present two sides of an ongoing scientific debate

(C)

report answers to several questions that have
long puzzled researchers

(D)

discuss evidence that has caused a long‐
standing belief to be revised

(E)

attempt to explain a commonly misunderstood
biological phenomenon

410. The passage mentions each of the following as a
function regulated by the SCNs in some animals
EXCEPT
(A)

activity level

(B)

blood pressure

(C)

alertness

(D)

vision

(E)

temperature

7.4 Reading Comprehension Practice Questions

411. The author of the passage would probably agree with
which of the following statements about the SCNs?
(A)

The SCNs are found in other organs and tissues
of the body besides the hypothalamus.

(B)

The SCNs play a critical but not exclusive role in
regulating circadian rhythms.

(C)

The SCNs control clock genes in a number of
tissues and organs throughout the body.

(D)

The SCNs are a less significant factor in
regulating blood pressure than scientists once
believed.

(E)

The SCNs are less strongly affected by changes
in light levels than they are by other external
cues.

365

GMAT® Official Guide 2018

Line

(5)

(10)

(15)

(20)

(25)

366

In their study of whether offering a guarantee of
service quality will encourage customers to visit a
particular restaurant, Tucci and Talaga have found
that the effect of such guarantees is mixed. For
higher-priced restaurants, there is some evidence
that offering a guarantee increases the likelihood of
customer selection, probably reflecting the greater
financial commitment involved in choosing an
expensive restaurant. For lower-priced restaurants,
where one expects less assiduous service, Tucci and
Talaga found that a guarantee could actually have a
negative effect: a potential customer might think that
a restaurant offering a guarantee is worried about
its service. Moreover, since customers understand a
restaurant’s product and know what to anticipate in
terms of service, they are empowered to question its
quality. This is not generally true in the case of skilled
activities such as electrical work, where, consequently,
a guarantee might have greater customer appeal.
For restaurants generally, the main benefit of
a service guarantee probably lies not so much in
customer appeal as in managing and motivating staff.
Staff members would know what service standards
are expected of them and also know that the success
of the business relies on their adhering to those
standards. Additionally, guarantees provide some
basis for defining the skills needed for successful
service in areas traditionally regarded as unskilled,
such as waiting tables.

Questions 412–414 refer to the passage.

412. The primary purpose of the passage is to
(A)

question the results of a study that examined
the effect of service-quality guarantees in the
restaurant industry

(B)

discuss potential advantages and disadvantages
of service-quality guarantees in the restaurant
industry

(C)

examine the conventional wisdom regarding
the effect of service-quality guarantees in the
restaurant industry

(D)

argue that only certain restaurants would benefit
from the implementation of service-quality
guarantees

(E)

consider the impact that service-quality
guarantees can have on the service provided by
a restaurant

413. It can be inferred that the author of the passage would
agree with which of the following statements about the
appeal of service guarantees to customers?
(A)

Such guarantees are likely to be somewhat more
appealing to customers of restaurants than to
customers of other businesses.

(B)

Such guarantees are likely to be more appealing
to customers who know what to anticipate in
terms of service.

(C)

Such guarantees are likely to have less appeal in
situations where customers are knowledgeable
about a business’s product or service.

(D)

In situations where a high level of financial
commitment is involved, a service guarantee is
not likely to be very appealing.

(E)

In situations where customers expect a high
level of customer service, a service guarantee is
likely to make customers think that a business is
worried about its service.

7.4 Reading Comprehension Practice Questions

414. According to the passage, Tucci and Talaga found
that service guarantees, when offered by lower-priced
restaurants, can have which of the following effects?
(A)

Customers’ developing unreasonably high
expectations regarding service

(B)

Customers’ avoiding such restaurants because
they fear that the service guarantee may not be
fully honored

(C)

Customers’ interpreting the service guarantee as
a sign that management is not confident about
the quality of its service

(D)

A restaurant’s becoming concerned that its
service will not be assiduous enough to satisfy
customers

(E)

A restaurant’s becoming concerned that
customers will be more emboldened to question
the quality of the service they receive

367

GMAT® Official Guide 2018

Line

(5)

(10)

(15)

(20)

(25)

(30)

368

The argument for “monetizing”—or putting a
monetary value on—ecosystem functions may be
stated thus: Concern about the depletion of natural
resources is widespread, but this concern, in the
absence of an economic argument for conservation,
has not translated into significant conservational
progress. Some critics blame this impasse on
environmentalists, whom they believe fail to address
the economic issues of environmental degradation.
Conservation can appear unprofitable when compared
with the economic returns derived from converting
natural assets (pristine coastlines, for example) into
explicitly commercial ones (such as resort hotels).
But according to David Pearce, that illusion stems
from the fact that “services” provided by ecological
systems are not traded on the commodities market,
and thus have no readily quantifiable value. To
remedy this, says Pearce, one has to show that all
ecosystems have economic value—indeed, that all
ecological services are economic services. Tourists
visiting wildlife preserves, for example, create
jobs and generate income for national economies;
undisturbed forests and wetlands regulate water
runoff and act as water-purifying systems, saving
millions of dollars worth of damage to property
and to marine ecosystems. In Gretchen Daily’s
view, monetization, while unpopular with many
environmentalists, reflects the dominant role that
economic considerations play in human behavior,
and the expression of economic value in a common
currency helps inform environmental decision-making
processes.

Questions 415–418 refer to the passage.

415. Information in the passage suggests that David Pearce
would most readily endorse which of the following
statements concerning monetization?
(A)

Monetization represents a strategy that is
attractive to both environmentalists and their
critics.

(B)

Monetization is an untested strategy,
but it is increasingly being embraced by
environmentalists.

(C)

Monetization should at present be restricted to
ecological services and should only gradually
be extended to such commercial endeavors as
tourism and recreation.

(D)

Monetization can serve as a means of
representing persuasively the value of
environmental conservation.

(E)

Monetization should inform environmental
decision-making processes only if it is accepted
by environmentalist groups.

416. Which of the following most clearly represents an
example of an “ecological service” as that term is used
in line 20 ?
(A)

A resort hotel located in an area noted for its
natural beauty

(B)

A water-purifying plant that supplements natural
processes with nontoxic chemicals

(C)

A wildlife preserve that draws many international
travelers

(D)

A nonprofit firm that specializes in restoring
previously damaged ecosystems

(E)

A newsletter that keeps readers informed of
ecological victories and setbacks

7.4 Reading Comprehension Practice Questions

417. According to the passage, Daily sees monetization as
an indication of which of the following?
(A)

The centrality of economic interests to people’s
actions

(B)

The reluctance of the critics of environmentalism
to acknowledge the importance of conservation

(C)

The inability of financial interests and ecological
interests to reach a common ideological ground

(D)

The inevitability of environmental degradation

(E)

The inevitability of the growth of ecological
services in the future

418. Which of the following can be inferred from the
passage concerning the environmentalists mentioned
in line 8 ?
(A)

They are organized in opposition to the
generation of income produced by the sale of
ecological services.

(B)

They are fewer in number but better organized
and better connected to the media than their
opponents.

(C)

They have sometimes been charged with failing
to use a particular strategy in their pursuit of
conservational goals.

(D)

They have been in the forefront of publicizing the
extent of worldwide environmental degradation.

(E)

They define environmental progress differently
and more conservatively than do other organized
groups of environmentalists.

369

GMAT® Official Guide 2018

Line

(5)

(10)

(15)

(20)

(25)

(30)

370

Much research has been devoted to investigating
what motivates consumers to try new products.
Previous consumer research suggests that both the
price of a new product and the way it is advertised
affect consumers’ perceptions of the product’s
performance risk (the possibility that the product will
not function as consumers expect and/or will not
provide the desired benefits). Some of this research
has concluded that a relatively high price will reduce
a consumer’s perception of the performance risk
associated with purchasing a particular product,
while other studies have reported that price has little
or no effect on perceived performance risk. These
conflicting findings may simply be due to the nature
of product advertisements: a recent study indicates
that the presentation of an advertised message has a
marked effect on the relationship between price and
perceived performance risk.
Researchers have identified consumers’ perception
of the credibility of the source of an advertised
message—i.e., the manufacturer—as another factor
affecting perceived performance risk: one study
found that the greater the source credibility, the lower
the consumer’s perception of the risk of purchasing
an advertised new product. However, past research
suggests that the relationship between source
credibility and perceived performance risk may be
more complex: source credibility may interact with
price in a subtle way to affect consumers’ judgments
of the performance risk associated with an advertised
product.

Questions 419–422 refer to the passage.

419. According to the passage, the studies referred to in
line 12 reported which of the following about the effect
of price on consumers’ perception of the performance
risk associated with a new product?
(A)

Although most consumers regard price as
an important factor, their perception of the
performance risk associated with a new product
is ultimately determined by the manufacturer’s
reputation.

(B)

Price interacts with the presentation of an
advertised message to affect perceived
performance risk.

(C)

Price does not significantly affect consumers’
perception of the performance risk associated
with a new product.

(D)

Consumers tend to regard price as more important
than the manufacturer’s credibility when they are
buying from that manufacturer for the first time.

(E)

Consumers are generally less concerned about
a new product’s performance risk when that
product is relatively expensive.

420. The “past research” mentioned in line 25 suggests
which of the following about perceived performance
risk?
(A)

The more expensive a new product is, the more
likely consumers may be to credit advertised
claims about that product.

(B)

The more familiar consumers are with a
particular manufacturer, the more willing they
may be to assume some risk in the purchase
of a new product being advertised by that
manufacturer.

(C)

Consumers’ perception of the performance risk
associated with a new product being advertised
may be influenced by an interplay between the
product’s price and the manufacturer’s credibility.

(D)

Consumers may be more likely to believe that
a product will function as it is advertised to
do when they have bought products from a
particular manufacturer before.

(E)

The price of a particular advertised product
may have less impact than the manufacturer’s
credibility on consumers’ assessment of the
performance risk associated with that product.

7.4 Reading Comprehension Practice Questions

421. The passage is primarily concerned with
(A)

challenging the implications of previous research
into why consumers try new products

(B)

suggesting new marketing strategies for
attracting consumers to new products

(C)

reconciling two different views about the effect
of price on consumers’ willingness to try new
products

(D)

describing a new approach to researching why
consumers try new products

(E)

discussing certain findings regarding why
consumers try new products

422. Which of the following, if true, would most tend to
weaken the conclusions drawn from “some of this
research” (line 8)?
(A)

In a subsequent study, consumers who were
asked to evaluate new products with relatively
low prices had the same perception of the
products’ performance risk as did consumers
who were shown the same products priced more
expensively.

(B)

In a subsequent study, the quality of the
advertising for the products that consumers
perceived as having a lower performance risk
was relatively high, while the quality of the
advertising for the products that consumers
perceived as having a higher performance risk
was relatively poor.

(C)

In a subsequent study, the products that
consumers perceived as having a lower
performance risk were priced higher than the
highest priced products in the previous research.

(D)

None of the consumers involved in this research
had ever before bought products from the
manufacturers involved in the research.

(E)

Researchers found that the higher the source
credibility for a product, the more consumers
were willing to pay for it.

371

GMAT® Official Guide 2018

Line

(5)

(10)

(15)

(20)

(25)

(30)

(35)

372

Historians remain divided over the role of
banks in facilitating economic growth in the
United States in the late eighteenth and early
nineteenth centuries. Some scholars contend
that banks played a minor role in the nation’s
growing economy. Financial institutions, they
argue, appeared only after the economy had
begun to develop, and once organized, followed
conservative lending practices, providing aid to
established commercial enterprises but
shunning those, such as manufacturing and
transportation projects, that were more
uncertain and capital‐intensive (i.e., requiring
greater expenditures in the form of capital than in
labor).
A growing number of historians argue, in
contrast, that banks were crucial in transforming
the early national economy. When state
legislatures began granting more bank charters
in the 1790s and early 1800s, the supply of
credit rose accordingly. Unlike the earliest banks,
which had primarily provided short‐term loans to
well‐connected merchants, the banks of the early
nineteenth century issued credit widely. As Paul
Gilje asserts, the expansion and democratization
of credit in the early nineteenth century became
the driving force of the American economy, as
banks began furnishing large amounts of capital
to transportation and industrial enterprises. The
exception, such historians argue, was in the
South; here, the overwhelmingly agrarian nature
of the economy generated outright opposition
to banks, which were seen as monopolistic
institutions controlled by an elite group of
planters.

Questions 423–427 refer to the passage.

423. The primary purpose of the passage is to
(A)

compare the economic role played by southern
banks with the economic role played by banks
in the rest of the United States during the late
eighteenth and early nineteenth centuries

(B)

reevaluate a conventional interpretation of the
role played by banks in the American economy
during the late eighteenth and early nineteenth
centuries

(C)

present different interpretations of the role
played by banks in the American economy during
the late eighteenth and early nineteenth centuries

(D)

analyze how the increasing number of banks in
the late eighteenth and early nineteenth centuries
affected the American economy

(E)

examine how scholarly opinion regarding the role
played by banks in the American economy during
the late eighteenth and early nineteenth centuries
has changed over time

424. The passage suggests that the scholars mentioned in
line 4 would argue that the reason banks tended not to
fund manufacturing and transportation projects in the
late eighteenth and early nineteenth centuries was that
(A)

these projects, being well established and well
capitalized, did not need substantial long‐term
financing from banks

(B)

these projects entailed a level of risk that
was too great for banks’ conservative lending
practices

(C)

banks preferred to invest in other, more
speculative projects that offered the potential for
higher returns

(D)

bank managers believed that these projects
would be unlikely to contribute significantly to
economic growth in the new country

(E)

bank managers believed funding these projects
would result in credit being extended to too
many borrowers

7.4 Reading Comprehension Practice Questions

425. The passage suggests that Paul Gilje would be most
likely to agree with which of the following claims
about the lending practices of the “earliest banks”
(see line 21)?

427. Which of the following statements best describes the
function of the last sentence of the passage?
(A)

It provides evidence tending to undermine the
viewpoint of the scholars mentioned in line 5.

(A)

These lending practices were unlikely to
generate substantial profits for banks.

(B)

It resolves a conflict over the role of banks
summarized in the first paragraph.

(B)

These lending practices only benefited a narrow
sector of the economy.

(C)

It clarifies some of the reasons state legislatures
began granting more bank charters.

(C)

The restrictive nature of these lending practices
generated significant opposition outside of the
South.

(D)

It qualifies a claim made earlier in the passage
about the impact of banks on the American
economy in the early nineteenth century.

(D)

The restrictive nature of these lending practices
forced state legislatures to begin granting more
bank charters by the early nineteenth century.

(E)

It supports a claim made earlier in the passage
about how the expansion of credit affected the
economy.

(E)

These lending practices were likely to be
criticized by economic elites as being overly
restrictive.

426. The passage suggests that the opposition to banks in
the South in the early nineteenth century stemmed in
part from the perception that banks
(A)

did not benefit more than a small minority of the
people

(B)

did not support the interests of elite planters

(C)

were too closely tied to transportation and
industrial interests

(D)

were unwilling to issue the long‐term loans
required by agrarian interests

(E)

were too willing to lend credit widely

373

GMAT® Official Guide 2018

Line

(5)

(10)

(15)

(20)

(25)

(30)

374

Io and Europa, the inner two of Jupiter’s four
largest moons, are about the size of Earth’s moon
and are composed mostly or entirely of rock and
metal. Ganymede and Callisto are larger and roughly
half ice. Thus, these four moons are somewhat
analogous to the planets of the solar system, in which
the rock‐ and metal‐rich inner planets are distinct
from the much larger gas‐ and ice‐rich outer planets.
Jupiter’s moons are, however, more “systematic”:
many of their properties vary continuously with
distance from Jupiter. For example, Io is ice‐free,
Europa has a surface shell of ice, and while
Ganymede and Callisto are both ice‐rich, outermost
Callisto has more.
This compositional gradient has geological
parallels. Io is extremely geologically active, Europa
seems to be active on a more modest scale, and
Ganymede has undergone bouts of activity in its
geological past. Only Callisto reveals no geological
activity. In similar fashion, Callisto’s surface is very
heavily cratered from the impact of comets and
asteroids; Ganymede, like Earth’s moon, is heavily
cratered in parts; Europa is very lightly cratered; and
no craters have been detected on Io, even though
Jupiter’s gravity attracts comets and asteroids
passing near it, substantially increasing the
bombardment rate of the inner moons compared
to that of the outer ones. But because of Io’s high
degree of geological activity, its surface undergoes
more‐or‐less continuous volcanic resurfacing.

Questions 428–430 refer to the passage.

428. According to the passage, the difference in the amount
of cratering on Callisto’s and Io’s respective surfaces
can probably be explained by the difference between
these two moons with respect to which of the following
factors?
(A)

Size

(B)

Ice content

(C)

The rate of bombardment by comets and
asteroids

(D)

The influence of Jupiter’s other moons

(E)

The level of geological activity

429. Which of the following best describes the purpose of
the second paragraph of the passage?
(A)

To provide further evidence of the systematic
variation in the characteristics of Jupiter’s four
largest moons

(B)

To present a comprehensive theory to explain
the systematic variation in the characteristics of
Jupiter’s four largest moons

(C)

To explain the significance of the systematic
variation in the characteristics of Jupiter’s four
largest moons

(D)

To introduce facts that contradict conventional
assumptions about Jupiter’s four largest moons

(E)

To contrast the characteristics of Jupiter’s four
largest moons with the characteristics of the
planets of the solar system

7.4 Reading Comprehension Practice Questions

430. The author’s reference to Jupiter’s gravity in line 25
serves primarily to
(A)

indicate why the absence of craters on Io’s
surface is surprising

(B)

explain the presence of craters on the surface of
Jupiter’s four largest moons

(C)

provide an explanation for the lack of geological
activity on Callisto

(D)

contrast Jupiter’s characteristics with the
characteristics of its four largest moons

(E)

illustrate the similarity between Jupiter’s four
largest moons and the planets of the solar
system

375

GMAT® Official Guide 2018

Line

(5)

(10)

(15)

(20)

(25)

(30)

376

In an effort to explain why business acquisitions
often fail, scholars have begun to focus on the role
of top executives of acquired companies. Acquired
companies that retain their top executives tend to
have more successful outcomes than those that do
not. Furthermore, existing research suggests that
retaining the highest‐level top executives, such as the
CEO (chief executive officer) and COO (chief operating
officer), is related more positively to postacquisition
success than retaining lower‐ranked top executives.
However, this explanation, while insightful, suffers from
two limitations. First, the focus on positional rank does
not recognize the variation in length of service that
may exist in top executive posts across companies,
nor does it address which particular top executives
(with respect to length of service) should be retained
to achieve a successful acquisition outcome. Second,
the relationship between retained top executives and
acquisition outcomes offered by existing research
is subject to opposing theoretical explanations
related to length of service. The resource‐based view
(RBV) suggests that keeping acquired company top
executives with longer organizational tenure would lead
to more successful outcomes, as those executives
have idiosyncratic and nontransferable knowledge
of the acquired company that would be valuable for
the effective implementation of the acquisition. The
opposing position, offered by the upper echelons
perspective (UEP), suggests that retaining top
executives having short organizational tenure would
lead to more successful outcomes, as they would have
the adaptability to manage most effectively during the
uncertainty of the acquisition process.

(35)

(40)

(45)

(50)

Responding to these limitations, Bergh conducted
a study of executive retention and acquisition
outcome that focused on the organizational tenure of
retained company top executives in 104 acquisitions,
followed over 5 years. Bergh considered the
acquisition successful if the acquired company was
retained and unsuccessful if it was divested. Bergh’s
findings support the RBV position. Apparently, the
benefits of long organizational tenure lead to more
successful outcomes than the benefits of short
organizational tenure. While longer tenured top
executives may have trouble adapting to change, it
appears that their perspectives and knowledge bases
offer unique value after the acquisition. Although
from the UEP position it seems sensible to retain
less tenured executives and allow more tenured
ones to leave, such a strategy appears to lower the
probability of acquisition success.

7.4 Reading Comprehension Practice Questions

Questions 431–434 refer to the passage.

431. According to the passage, the research mentioned
in line 6 suggests which of the following about
lower‐ranked top executives and postacquisition
success?
(A)

Given that these executives are unlikely to
contribute to postacquisition success, little effort
should be spent trying to retain them.

(B)

The shorter their length of service, the less likely
it is that these executives will play a significant
role in postacquisition success.

(C)

These executives are less important to
postacquisition success than are more highly
ranked top executives.

(D)

If they have long tenures, these executives may
prove to be as important to postacquisition
success as are more highly ranked top
executives.

(E)

Postacquisition success is unlikely if these
executives are retained.

432. The resource‐based view, as described in the passage,
is based on which of the following ideas?

433. The passage suggests that Bergh and a proponent of
the upper echelons perspective would be most likely to
disagree over which of the following?
(A)

Whether there is a positive correlation between
short organizational tenure and managerial
adaptability

(B)

Whether there is a positive correlation between
long organizational tenure and the acquisition of
idiosyncratic and nontransferable knowledge

(C)

Whether adaptability is a useful trait for an
executive who is managing an acquisition
process

(D)

Whether retaining less‐tenured top executives of
an acquired company is an optimal strategy for
achieving postacquisition success

(E)

Whether retaining highest‐level top executives
of acquired companies is more important than
retaining lower‐ranked top executives

434. According to the passage, prior to Bergh’s study,
research on the role of top executives of acquired
companies in business acquisition success was limited
in which of the following ways?
(A)

It did not address how the organizational
tenure of top executives affects postacquisition
success.

(A)

The managerial skills of top executives become
strongest after the first five years of their tenure.

(B)

(B)

Company‐specific knowledge is an important
factor in the success of an acquisition process.

It did not address why some companies have
longer‐tenured CEOs than others.

(C)

(C)

The amount of nontransferable knowledge
possessed by long‐tenured top executives tends
to be underestimated.

It did not consider strategies for retaining long‐
tenured top executives of acquired companies.

(D)

Effective implementation of an acquisition
depends primarily on the ability of executives to
adapt to change.

It failed to differentiate between the contribution
of highest‐level top executives to postacquisition
success and that of lower‐ranked top executives.

(E)

It underestimated the potential contribution
that lower‐level top executives can make to
postacquisition success.

(D)

(E)

Short‐tenured executives are likely to impede
the implementation of a successful acquisition
strategy.

377

GMAT® Official Guide 2018

Line

(5)

(10)

(15)

(20)

(25)

(30)

378

When Jamaican‐born social activist Marcus
Garvey came to the United States in 1916, he
arrived at precisely the right historical moment.
What made the moment right was the return of
African American soldiers from the First World War
in 1918, which created an ideal constituency for
someone with Garvey’s message of unity, pride,
and improved conditions for African American
communities.
Hoping to participate in the traditional American
ethos of individual success, many African American
people entered the armed forces with enthusiasm,
only to find themselves segregated from white
troops and subjected to numerous indignities. They
returned to a United States that was as segregated
as it had been before the war. Considering similar
experiences, anthropologist Anthony F. C. Wallace
has argued that when a perceptible gap arises
between a culture’s expectations and the reality of
that culture, the resulting tension can inspire a
revitalization movement: an organized, conscious
effort to construct a culture that fulfills long‐
standing expectations.
Some scholars have argued that Garvey created
the consciousness from which he built, in the 1920s,
the largest revitalization movement in
African American history. But such an argument only
tends to obscure the consciousness of
identity, strength, and sense of history that already
existed in the African American community. Garvey
did not create this consciousness; rather, he gave
this consciousness its political expression.

Questions 435–438 refer to the passage.

435. According to the passage, which of the following
contributed to Marcus Garvey’s success?
(A)

He introduced cultural and historical
consciousness to the African American
community.

(B)

He believed enthusiastically in the traditional
American success ethos.

(C)

His audience had already formed a
consciousness that made it receptive to his
message.

(D)

His message appealed to critics of African
American support for United States military
involvement in the First World War.

(E)

He supported the movement to protest
segregation that had emerged prior to his arrival
in the United States.

436. The passage suggests that many African American
people responded to their experiences in the armed
forces in which of the following ways?
(A)

They maintained as civilians their enthusiastic
allegiance to the armed forces.

(B)

They questioned United States involvement in the
First World War.

(C)

They joined political organizations to protest the
segregation of African American troops and the
indignities they suffered in the military.

(D)

They became aware of the gap between their
expectations and the realities of American
culture.

(E)

They repudiated Garvey’s message of pride and
unity.

7.4 Reading Comprehension Practice Questions

437. It can be inferred from the passage that the “scholars”
mentioned in line 24 believe which of the following to
be true?
(A)

Revitalization resulted from the political activism
of returning African American soldiers following
the First World War.

(B)

Marcus Garvey had to change a number of
prevailing attitudes in order for his mass
movement to find a foothold in the United States.

(C)

The prevailing sensibility of the African American
community provided the foundation of Marcus
Garvey’s political appeal.

(D)

Marcus Garvey hoped to revitalize consciousness
of cultural and historical identity in the African
American community.

(E)

The goal of the mass movement that Marcus
Garvey helped bring into being was to build on
the pride and unity among African Americans.

438. According to the passage, many African American
people joined the armed forces during the First World
War for which of the following reasons?
(A)

They wished to escape worsening economic
conditions in African American communities.

(B)

They expected to fulfill ideals of personal
attainment.

(C)

They sought to express their loyalty to the United
States.

(D)

They hoped that joining the military would help
advance the cause of desegregation.

(E)

They saw military service as an opportunity to
fulfill Marcus Garvey’s political vision.

379

GMAT® Official Guide 2018

Line

(5)

(10)

(15)

(20)

(25)

(30)

380

In the Sonoran Desert of northwestern Mexico and
southern Arizona, the flowers of several species of
columnar cacti—cardon, saguaro, and organ
pipe—were once exclusively pollinated at night by
nectar-feeding bats, as their close relatives in arid
tropical regions of southern Mexico still are. In these
tropical regions, diurnal (daytime) visitors to columnar
cactus flowers are ineffective pollinators because,
by sunrise, the flowers’ stigmas become unreceptive
or the flowers close. Yet the flowers of the Sonoran
Desert cacti have evolved to remain open after sunrise,
allowing pollination by such diurnal visitors as bees and
birds. Why have these cacti expanded their range of
pollinators by remaining open and receptive in daylight?
This development at the northernmost range of
columnar cacti may be due to a yearly variation in the
abundance—and hence the reliability—of migratory
nectar-feeding bats. Pollinators can be unreliable
for several reasons. They can be dietary generalists
whose fidelity to a particular species depends on
the availability of alternative food sources. Or, they
can be dietary specialists, but their abundance may
vary widely from year to year, resulting in variable
pollination of their preferred food species. Finally, they
may be dietary specialists, but their abundance may
be chronically low relative to the availability of flowers.
Recent data reveals that during spring in the
Sonoran Desert, the nectar-feeding bats are
specialists feeding on cardon, saguaro, and
organpipe flowers. However, whereas cactus-flower
abundance tends to be high during spring, bat
population densities tend to be low except near
maternity roosts. Moreover, in spring, diurnal cactuspollinating birds are significantly more abundant in

(35)

(40)

(45)

this region than are the nocturnal bats. Thus, with bats
being unreliable cactus-flower pollinators, and daytime
pollinators more abundant and therefore more reliable,
selection favors the cactus flowers with traits that
increase their range of pollinators. While data suggest
that population densities of nectar-feeding bats are
also low in tropical areas of southern Mexico, where
bats are the exclusive pollinators of many species
of columnar cacti, cactus-flower density and bat
population density appear to be much more evenly
balanced there: compared with the Sonoran Desert’s
cardon and saguaro, columnar cacti in southern Mexico
produce far fewer flowers per night. Accordingly,
despite their low population density, bats are able to
pollinate nearly 100 percent of the available flowers.

7.4 Reading Comprehension Practice Questions

Questions 439–441 refer to the passage.

441. According to the passage, present-day columnar cacti
in the Sonoran Desert differ from their close relatives
in southern Mexico in that the Sonoran cacti

439. The primary purpose of the passage is to
(A)

(B)

compare the adaptive responses of several
species of columnar cacti in the Sonoran Desert
with those in the arid tropical regions of southern
Mexico
discuss some of the possible causes of the
relatively low abundance of migratory nectarfeeding bats in the Sonoran Desert

(C)

provide a possible explanation for a particular
evolutionary change in certain species of
columnar cacti in the Sonoran Desert

(D)

present recent findings that challenge a
particular theory as to why several species
of columnar cacti in the Sonoran Desert have
expanded their range of pollinators

(E)

compare the effectiveness of nocturnal and
diurnal pollination for several different species of
columnar cacti in the Sonoran Desert

(A)

have flowers that remain open after sunset

(B)

are pollinated primarily by dietary specialists

(C)

can be pollinated by nectar-feeding bats

(D)

have stigmas that are unreceptive to pollination
at night

(E)

are sometimes pollinated by diurnal pollinators

440. According to the passage, which of the following types
of nectar-feeding pollinators is likely to be an unreliable
pollinator of a particular cactus flower?
(A)

A dietary specialist whose abundance is typically
high in relation to that of the flower

(B)

A dietary specialist whose abundance is at times
significantly lower than that of the flower

(C)

A dietary generalist for whom that flower’s
nectar is not a preferred food but is the most
consistently available food

(D)

A dietary generalist for whom that flower’s nectar
is slightly preferred to other available foods

(E)

A dietary generalist that evolved from a species
of dietary specialists

381

GMAT® Official Guide 2018

Line

(5)

(10)

(15)

(20)

(25)

(30)

382

In terrestrial environments, gravity places
special demands on the cardiovascular systems of
animals. Gravitational pressure can cause blood to
pool in the lower regions of the body, making it
difficult to circulate blood to critical organs such as
the brain. Terrestrial snakes, in particular, exhibit
adaptations that aid in circulating blood against the
force of gravity.
The problem confronting terrestrial snakes is best
illustrated by what happens to sea snakes when
removed from their supportive medium. Because the
vertical pressure gradients within the blood vessels
are counteracted by similar pressure gradients in
the surrounding water, the distribution of blood
throughout the body of sea snakes remains about
the same regardless of their orientation in space,
provided they remain in the ocean. When removed
from the water and tilted at various angles with the
head up, however, blood pressure at their midpoint
drops significantly, and at brain level falls to zero.
That many terrestrial snakes in similar spatial
orientations do not experience this kind of circulatory
failure suggests that certain adaptations enable them
to regulate blood pressure more effectively in those
orientations.
One such adaptation is the closer proximity of
the terrestrial snake’s heart to its head, which helps
to ensure circulation to the brain, regardless of the
snake’s orientation in space. The heart of sea snakes
can be located near the middle of the body, a
position that minimizes the work entailed in
circulating blood to both extremities. In arboreal
snakes, however, which dwell in trees and often
assume a vertical posture, the average distance

(35)

(40)

(45)

from the heart to the head can be as little as
15 percent of overall body length. Such a location
requires that blood circulated to the tail of the
snake travel a greater distance back to the heart,
a problem solved by another adaptation. When
climbing, arboreal snakes often pause
momentarily to wiggle their bodies, causing waves
of muscle contraction that advance from the lower
torso to the head. By compressing the veins and
forcing blood forward, these contractions
apparently improve the flow of venous blood
returning to the heart.

7.4 Reading Comprehension Practice Questions

Questions 442–449 refer to the passage.

442. The passage provides information in support of which
of the following assertions?

443. According to the passage, one reason that the
distribution of blood in the sea snake changes little
while the creature remains in the ocean is that
(A)

the heart of the sea snake tends to be located
near the center of its body

(A)

The disadvantages of an adaptation to a
particular feature of an environment often
outweigh the advantages of such an adaptation.

(B)

pressure gradients in the water surrounding
the sea snake counter the effects of vertical
pressure gradients within its blood vessels

(B)

An organism’s reaction to being placed in an
environment to which it is not well adapted can
sometimes illustrate the problems that have
been solved by the adaptations of organisms
indigenous to that environment.

(C)

the sea snake assumes a vertical posture less
frequently than do the terrestrial and the arboreal
snake

(D)

the sea snake often relies on waves of muscle
contractions to help move blood from the torso
to the head

(E)

the force of pressure gradients in the water
surrounding the sea snake exceeds that of
vertical pressure gradients within its circulatory
system

(C)

(D)

(E)

The effectiveness of an organism’s adaptation to
a particular feature of its environment can only
be evaluated by examining the effectiveness with
which organisms of other species have adapted
to a similar feature of a different environment.
Organisms of the same species that inhabit
strikingly different environments will often adapt
in remarkably similar ways to the few features of
those environments that are common.
Different species of organisms living in the same
environment will seldom adapt to features of that
environment in the same way.

444. It can be inferred from the passage that which of the
following is true of species of terrestrial snakes that
often need to assume a vertical posture?
(A)

They are more likely to be susceptible to
circulatory failure in vertical postures than are
sea snakes.

(B)

Their hearts are less likely to be located at the
midpoint of their bodies than is the case with sea
snakes.

(C)

They cannot counteract the pooling of blood in
lower regions of their bodies as effectively as
sea snakes can.

(D)

The blood pressure at their midpoint decreases
significantly when they are tilted with their
heads up.

(E)

They are unable to rely on muscle contractions
to move venous blood from the lower torso to
the head.

383

GMAT® Official Guide 2018

445. The author describes the behavior of the circulatory
system of sea snakes when they are removed from the
ocean (see lines 17–20) primarily in order to
(A)

(B)

illustrate what would occur in the circulatory
system of terrestrial snakes without adaptations
that enable them to regulate their blood pressure
in vertical orientations
explain why arboreal snakes in vertical
orientations must rely on muscle contractions to
restore blood pressure to the brain

(C)

illustrate the effects of circulatory failure on the
behavior of arboreal snakes

(D)

illustrate the superiority of the circulatory system
of the terrestrial snake to that of the sea snake

(E)

explain how changes in spatial orientation can
adversely affect the circulatory system of snakes
with hearts located in relatively close proximity
to their heads

446. It can be inferred from the passage that which of the
following is a true statement about sea snakes?
(A)

384

They frequently rely on waves of muscle
contractions from the lower torso to the head to
supplement the work of the heart.

(B)

They cannot effectively regulate their blood
pressure when placed in seawater and tilted at
an angle with the head pointed downward.

(C)

They are more likely to have a heart located in
close proximity to their heads than are arboreal
snakes.

(D)

They become acutely vulnerable to the effects
of gravitational pressure on their circulatory
system when they are placed in a terrestrial
environment.

(E)

Their cardiovascular system is not as
complicated as that of arboreal snakes.

447. The author suggests that which of the following is a
disadvantage that results from the location of a snake’s
heart in close proximity to its head?
(A)

A decrease in the efficiency with which the snake
regulates the flow of blood to the brain

(B)

A decrease in the number of orientations in
space that a snake can assume without loss of
blood flow to the brain

(C)

A decrease in blood pressure at the snake’s
midpoint when it is tilted at various angles with
its head up

(D)

An increase in the tendency of blood to pool
at the snake’s head when the snake is tilted at
various angles with its head down

(E)

An increase in the amount of effort required to
distribute blood to and from the snake’s tail

448. The primary purpose of the third paragraph is to
(A)

introduce a topic that is not discussed earlier in
the passage

(B)

describe a more efficient method of achieving an
effect discussed in the previous paragraph

(C)

draw a conclusion based on information
elaborated in the previous paragraph

(D)

discuss two specific examples of phenomena
mentioned at the end of the previous paragraph

(E)

introduce evidence that undermines a view
reported earlier in the passage

7.4 Reading Comprehension Practice Questions

449. In the passage, the author is primarily concerned with
doing which of the following?
(A)

Explaining adaptations that enable the terrestrial
snake to cope with the effects of gravitational
pressure on its circulatory system

(B)

Comparing the circulatory system of the sea
snake with that of the terrestrial snake

(C)

Explaining why the circulatory system of the
terrestrial snake is different from that of the sea
snake

(D)

Pointing out features of the terrestrial snake’s
cardiovascular system that make it superior to
that of the sea snake

(E)

Explaining how the sea snake is able to
neutralize the effects of gravitational pressure on
its circulatory system

385

GMAT® Official Guide 2018

Line

(5)

(10)

(15)

(20)

(25)

386

Current feminist theory, in validating women’s own
stories of their experience, has encouraged scholars
of women’s history to view the use of women’s oral
narratives as the methodology, next to the use of
women’s written autobiography, that brings historians
closest to the “reality” of women’s lives. Such
narratives, unlike most standard histories, represent
experience from the perspective of women, affirm
the importance of women’s contributions, and furnish
present‐day women with historical continuity that is
essential to their identity, individually and collectively.
Scholars of women’s history should, however, be
as cautious about accepting oral narratives at face
value as they already are about written memories.
Oral narratives are no more likely than are written
narratives to provide a disinterested commentary on
events or people. Moreover, the stories people tell to
explain themselves are shaped by narrative devices
and storytelling conventions, as well as by other
cultural and historical factors, in ways that the
storytellers may be unaware of. The political rhetoric
of a particular era, for example, may influence
women’s interpretations of the significance of their
experience. Thus a woman who views the Second
World War as pivotal in increasing the social
acceptance of women’s paid work outside the home
may reach that conclusion partly and unwittingly
because of wartime rhetoric encouraging a positive
view of women’s participation in such work.

Questions 450–455 refer to the passage.

450. The passage is primarily concerned with
(A)

contrasting the benefits of one methodology with
the benefits of another

(B)

describing the historical origins and inherent
drawbacks of a particular methodology

(C)

discussing the appeal of a particular
methodology and some concerns about its use

(D)

showing that some historians’ adoption of a
particular methodology has led to criticism of
recent historical scholarship

(E)

analyzing the influence of current feminist views
on women’s interpretations of their experience

451. According to the passage, which of the following
shapes the oral narratives of women storytellers?
(A)

The conventions for standard histories in the
culture in which a woman storyteller lives

(B)

The conventions of storytelling in the culture in
which a woman storyteller lives

(C)

A woman storyteller’s experience with distinctive
traditions of storytelling developed by the women
in her family of origin

(D)

The cultural expectations and experiences of
those who listen to oral narratives

(E)

A woman storyteller’s familiarity with the stories
that members of other groups in her culture tell
to explain themselves

7.4 Reading Comprehension Practice Questions

452. The author of the passage would be most likely to
make which of the following recommendations to
scholars of women’s history?
(A)

(B)

(A)

They should take into account their own life
experiences when interpreting the oral accounts
of women’s historical experiences.

Relying on traditional historical sources when
women’s oral narratives are unavailable

(B)

They should assume that the observations made
in women’s oral narratives are believed by the
intended audience of the story.

Focusing on the influence of political rhetoric on
women’s perceptions to the exclusion of other
equally important factors

(C)

Attempting to discover the cultural and historical
factors that influence the stories women tell

(D)

Assuming that the conventions of women’s
written autobiographies are similar to the
conventions of women’s oral narratives

(E)

Accepting women’s oral narratives less critically
than they accept women’s written histories

(C)

They should treat skeptically observations
reported in oral narratives unless the
observations can be confirmed in standard
histories.

(D)

They should consider the cultural and historical
context in which an oral narrative was created
before arriving at an interpretation of such a
narrative.

(E)

454. According to the passage, scholars of women’s history
should refrain from doing which of the following?

They should rely on information gathered from
oral narratives only when equivalent information
is not available in standard histories.

453. Which of the following best describes the function of
the last sentence of the passage?
(A)

It describes an event that historians view as
crucial in recent women’s history.

(B)

It provides an example of how political rhetoric
may influence the interpretations of experience
reported in women’s oral narratives.

(C)

It provides an example of an oral narrative that
inaccurately describes women’s experience
during a particular historical period.

(D)

It illustrates the point that some women are more
aware than others of the social forces that shape
their oral narratives.

(E)

It identifies the historical conditions that led to
the social acceptance of women’s paid work
outside the home.

455. According to the passage, each of the following is a
difference between women’s oral narratives and most
standard histories EXCEPT:
(A)

Women’s oral histories validate the significance
of women’s achievements.

(B)

Women’s oral histories depict experience from
the point of view of women.

(C)

Women’s oral histories acknowledge the
influence of well‐known women.

(D)

Women’s oral histories present today’s women
with a sense of their historical relationship to
women of the past.

(E)

Women’s oral histories are crucial to the
collective identity of today’s women.

387

GMAT® Official Guide 2018

Line

(5)

(10)

(15)

(20)

(25)

(30)

(35)

(40)

(45)

388

Manufacturers have to do more than build large
manufacturing plants to realize economies of scale.
It is true that as the capacity of a manufacturing
operation rises, costs per unit of output fall as plant
size approaches “minimum efficient scale,” where the
cost per unit of output reaches a minimum,
determined roughly by the state of existing technology
and size of the potential market. However, minimum
efficient scale cannot be fully realized unless a steady
“throughput” (the flow of materials through a plant) is
attained. The throughput needed to maintain the
optimal scale of production requires careful
coordination not only of the flow of goods through the
production process, but also of the flow of input from
suppliers and the flow of output to wholesalers and
final consumers. If throughput falls below a critical
point, unit costs rise sharply and profits disappear. A
manufacturer’s fixed costs and “sunk costs” (original
capital investment in the physical plant) do not
decrease when production declines due to inadequate
supplies of raw materials, problems on the factory
floor, or inefficient sales networks. Consequently,
potential economies of scale are based on the
physical and engineering characteristics of the
production facilities—that is, on tangible capital—but
realized economies of scale are operational and
organizational, and depend on knowledge, skills,
experience, and teamwork—that is, on organized
human capabilities, or intangible capital.
The importance of investing in intangible capital
becomes obvious when one looks at what happens in
new capital‐intensive manufacturing industries. Such
industries are quickly dominated, not by the first firms
to acquire technologically sophisticated plants of
theoretically optimal size, but rather by the first to
exploit the full potential of such plants. Once some
firms achieve this, a market becomes extremely hard
to enter. Challengers must construct comparable
plants and do so after the first movers have already
worked out problems with suppliers or with new
production processes. Challengers must create
distribution networks and marketing systems in
markets where first movers have all the contacts and
know‐how. And challengers must recruit management
teams to compete with those that have already
mastered these functional and strategic activities.

Questions 456–460 refer to the passage.

456. The passage suggests that in order for a manufacturer
in a capital‐intensive industry to have a decisive
advantage over competitors making similar products,
the manufacturer must
(A)

be the first in the industry to build production
facilities of theoretically optimal size

(B)

make every effort to keep fixed and sunk costs
as low as possible

(C)

be one of the first to operate its manufacturing
plants at minimum efficient scale

(D)

produce goods of higher quality than those
produced by direct competitors

(E)

stockpile raw materials at production sites in
order to ensure a steady flow of such materials

457. The passage suggests that which of the following is
true of a manufacturer’s fixed and sunk costs?
(A)

The extent to which they are determined
by market conditions for the goods being
manufactured is frequently underestimated.

(B)

If they are kept as low as possible, the
manufacturer is very likely to realize significant
profits.

(C)

They are the primary factor that determines
whether a manufacturer will realize economies of
scale.

(D)

They should be on a par with the fixed and sunk
costs of the manufacturer’s competitors.

(E)

They are not affected by fluctuations in a
manufacturing plant’s throughput.

458. In the context of the passage as a whole, the second
paragraph serves primarily to
(A)

provide an example to support the argument
presented in the first paragraph

(B)

evaluate various strategies discussed in the first
paragraph

(C)

introduce evidence that undermines the
argument presented in the first paragraph

(D)

anticipate possible objections to the argument
presented in the first paragraph

(E)

demonstrate the potential dangers of a
commonly used strategy

7.4 Reading Comprehension Practice Questions

459. The passage LEAST supports the inference that
a manufacturer’s throughput could be adversely
affected by
(A)

a mistake in judgment regarding the selection of
a wholesaler

(B)

a breakdown in the factory’s machinery

(C)

a labor dispute on the factory floor

(D)

an increase in the cost per unit of output

(E)

a drop in the efficiency of the sales network

460. The primary purpose of the passage is to
(A)

point out the importance of intangible capital for
realizing economies of scale in manufacturing

(B)

show that manufacturers frequently gain a
competitive advantage from investment in large
manufacturing facilities

(C)

argue that large manufacturing facilities often
fail because of inadequate investment in both
tangible and intangible capital

(D)

suggest that most new industries are likely
to be dominated by firms that build large
manufacturing plants early

(E)

explain why large manufacturing plants usually
do not help manufacturers achieve economies of
scale

389

GMAT® Official Guide 2018

Line

(5)

(10)

(15)

(20)

(25)

(30)

(35)

(40)

(45)

(50)

390

A small number of the forest species of
lepidoptera (moths and butterflies, which exist as
caterpillars during most of their life cycle) exhibit
regularly recurring patterns of population growth
and decline—such fluctuations in population are
known as population cycles. Although many different
variables influence population levels, a regular pattern
such as a population cycle seems to imply a
dominant, driving force. Identification of that driving
force, however, has proved surprisingly elusive
despite considerable research. The common
approach of studying causes of population cycles by
measuring the mortality caused by different agents,
such as predatory birds or parasites, has been
unproductive in the case of lepidoptera. Moreover,
population ecologists’ attempts to alter cycles by
changing the caterpillars’ habitat and by reducing
caterpillar populations have not succeeded. In short,
the evidence implies that these insect populations, if
not self‐regulating, may at least be regulated by an
agent more intimately connected with the insect than
are predatory birds or parasites.
Recent work suggests that this agent may be a
virus. For many years, viral disease had been reported
in declining populations of caterpillars, but population
ecologists had usually considered viral disease to
have contributed to the decline once it was underway
rather than to have initiated it. The recent work has
been made possible by new techniques of molecular
biology that allow viral DNA to be detected at low
concentrations in the environment. Nuclear
polyhedrosis viruses are hypothesized to be the
driving force behind population cycles in lepidoptera
in part because the viruses themselves follow an
infectious cycle in which, if protected from direct
sunlight, they may remain virulent for many years
in the environment, embedded in durable crystals of
polyhedrin protein. Once ingested by a caterpillar,
the crystals dissolve, releasing the virus to infect
the insect’s cells. Late in the course of the infection,
millions of new virus particles are formed and
enclosed in polyhedrin crystals. These crystals
reenter the environment after the insect dies and
decomposes, thus becoming available to infect
other caterpillars.
One of the attractions of this hypothesis is its broad
applicability. Remarkably, despite significant differences
in habitat and behavior, many species of lepidoptera
have population cycles of similar length, between eight
and eleven years. Nuclear polyhedrosis viral infection is
one factor these disparate species share.

Questions 461–466 refer to the passage.

461. The primary purpose of the passage is to
(A)

describe the development of new techniques that
may help to determine the driving force behind
population cycles in lepidoptera

(B)

present evidence that refutes a particular theory
about the driving force behind population cycles
in lepidoptera

(C)

present a hypothesis about the driving force
behind population cycles in lepidoptera

(D)

describe the fluctuating patterns of population
cycles in lepidoptera

(E)

question the idea that a single driving force is
behind population cycles in lepidoptera

462. It can be inferred from the passage that the mortality
caused by agents such as predatory birds or parasites
was measured in an attempt to
(A)

develop an explanation for the existence of
lepidoptera population cycles

(B)

identify behavioral factors in lepidoptera that
affect survival rates

(C)

identify possible methods for controlling
lepidoptera population growth

(D)

provide evidence that lepidoptera populations
are self‐regulating

(E)

determine the life stages of lepidoptera at which
mortality rates are highest

7.4 Reading Comprehension Practice Questions

463. Which of the following, if true, would most weaken the
author’s conclusion in lines 18–22 ?
(A)

(B)

New research reveals that the number of species
of birds and parasites that prey on lepidoptera
has dropped significantly in recent years.
New experiments in which the habitats of
lepidoptera are altered in previously untried ways
result in the shortening of lepidoptera population
cycles.

(C)

Recent experiments have revealed that the
nuclear polyhedrosis virus is present in a number
of predators and parasites of lepidoptera.

(D)

Differences among the habitats of lepidoptera
species make it difficult to assess the effects of
weather on lepidoptera population cycles.

(E)

Viral disease is typically observed in a large
proportion of the lepidoptera population.

466. It can be inferred from the passage that while inside its
polyhedrin protein crystals, the nuclear polyhedrosis
virus
(A)

is exposed to direct sunlight

(B)

is attractive to predators

(C)

cannot infect caterpillars’ cells

(D)

cannot be ingested by caterpillars

(E)

cannot be detected by new techniques of
molecular biology

464. According to the passage, before the discovery of
new techniques for detecting viral DNA, population
ecologists believed that viral diseases
(A)

were not widely prevalent among insect
populations generally

(B)

affected only the caterpillar life stage of
lepidoptera

(C)

were the driving force behind lepidoptera
population cycles

(D)

attacked already declining caterpillar populations

(E)

infected birds and parasites that prey on various
species of lepidoptera

465. According to the passage, nuclear polyhedrosis
viruses can remain virulent in the environment only
when
(A)

the polyhedrin protein crystals dissolve

(B)

caterpillar populations are in decline

(C)

they are present in large numbers

(D)

their concentration in a particular area remains
low

(E)

they are sheltered from direct sunlight

391

GMAT® Official Guide 2018

Line

(5)

(10)

(15)

(20)

(25)

(30)

392

Resin is a plant secretion that hardens when
exposed to air; fossilized resin is called amber.
Although Pliny in the first century recognized that
amber was produced from “marrow discharged by
trees,” amber has been widely misunderstood to be
a semiprecious gem and has even been described
in mineralogy textbooks. Confusion also persists
surrounding the term “resin,” which was defined
before rigorous chemical analyses were available.
Resin is often confused with gum, a substance
produced in plants in response to bacterial infections,
and with sap, an aqueous solution transported
through certain plant tissues. Resin differs from both
gum and sap in that scientists have not determined a
physiological function for resin.
In the 1950s, entomologists posited that resin
may function to repel or attract insects. Fraenkel
conjectured that plants initially produced resin in
nonspecific chemical responses to insect attack
and that, over time, plants evolved that produced
resin with specific repellent effects. But some insect
species, he noted, might overcome the repellent
effects, actually becoming attracted to the resin.
This might induce the insects to feed on those
plants or aid them in securing a breeding site.
Later researchers suggested that resin mediates
the complex interdependence, or “coevolution,” of
plants and insects over time. Such ideas led to the
development of the specialized discipline of chemical
ecology, which is concerned with the role of plant
chemicals in interactions with other organisms and
with the evolution and ecology of plant antiherbivore
chemistry (plants’ chemical defenses against attack
by herbivores such as insects).

Questions 467–470 refer to the passage.

467. According to the passage, which of the following is
true of plant antiherbivore chemistry?
(A)

Changes in a plant’s antiherbivore chemistry may
affect insect feeding behavior.

(B)

A plant’s repellent effects often involve
interactions between gum and resin.

(C)

A plant’s antiherbivore responses assist in
combating bacterial infections.

(D)

Plant antiherbivore chemistry plays only a minor
role in the coevolution of plants and insects.

(E)

Researchers first studied repellent effects in
plants beginning in the 1950s.

468. Of the following topics, which would be most likely to
be studied within the discipline of chemical ecology as
it is described in the passage?
(A)

Seeds that become attached to certain insects,
which in turn carry away the seeds and aid in
the reproductive cycle of the plant species in
question

(B)

An insect species that feeds on weeds
detrimental to crop health and yield, and how
these insects might aid in agricultural production

(C)

The effects of deforestation on the life cycles
of subtropical carnivorous plants and the insect
species on which the plants feed

(D)

The growth patterns of a particular species of
plant that has proved remarkably resistant to
herbicides

(E)

Insects that develop a tolerance for feeding on a
plant that had previously been toxic to them, and
the resultant changes within that plant species

7.4 Reading Comprehension Practice Questions

469. The author refers to “bacterial infections” (see line 11)
most likely in order to
(A)

describe the physiological function that gum
performs in plants

(B)

demonstrate that sap is not the only substance
that is transported through a plant’s tissues

(C)

explain how modern chemical analysis has been
used to clarify the function of resin

(D)

show that gum cannot serve as an effective
defense against herbivores

(E)

give an example of how confusion has arisen
with regard to the nature of resin

470. The author of the passage refers to Pliny most
probably in order to
(A)

give an example of how the nature of amber has
been misunderstood in the past

(B)

show that confusion about amber has long been
more pervasive than confusion about resin

(C)

make note of the first known reference to amber
as a semiprecious gem

(D)

point out an exception to a generalization about
the history of people’s understanding of amber

(E)

demonstrate that Pliny believed amber to be a
mineral

393

GMAT® Official Guide 2018

Line

(5)

(10)

(15)

(20)

(25)

(30)

394

During the 1980s, many economic historians
studying Latin America focused on the impact of
the Great Depression of the 1930s. Most of these
historians argued that although the Depression
began earlier in Latin America than in the United
States, it was less severe in Latin America and did
not significantly impede industrial growth there.
The historians’ argument was grounded in national
government records concerning tax revenues and
exports and in government‐sponsored industrial
censuses, from which historians have drawn
conclusions about total manufacturing output
and profit levels across Latin America. However,
economic statistics published by Latin American
governments in the early twentieth century are
neither reliable nor consistent; this is especially
true of manufacturing data, which were gathered
from factory owners for taxation purposes and
which therefore may well be distorted. Moreover,
one cannot assume a direct correlation between
the output level and the profit level of a given
industry as these variables often move in opposite
directions. Finally, national and regional economies
are composed of individual firms and industries,
and relying on general, sweeping economic
indicators may mask substantial variations among
these different enterprises. For example, recent
analyses of previously unexamined data on textile
manufacturing in Brazil and Mexico suggest that the
Great Depression had a more severe impact on this
Latin American industry than scholars
had recognized.

Questions 471–473 refer to the passage.

471. The primary purpose of the passage is to
(A)

compare the impact of the Great Depression
on Latin America with its impact on the United
States

(B)

criticize a school of economic historians for
failing to analyze the Great Depression in Latin
America within a global context

(C)

illustrate the risks inherent in comparing different
types of economic enterprises to explain
economic phenomena

(D)

call into question certain scholars’ views
concerning the severity of the Great Depression
in Latin America

(E)

demonstrate that the Great Depression had
a more severe impact on industry in Latin
America than in certain other regions

472. Which of the following conclusions about the Great
Depression is best supported by the passage?
(A)

It did not impede Latin American industrial
growth as much as historians had previously
thought.

(B)

It had a more severe impact on the Brazilian and
the Mexican textile industries than it had on Latin
America as a region.

(C)

It affected the Latin American textile industry
more severely than it did any other industry in
Latin America.

(D)

The overall impact on Latin American industrial
growth should be reevaluated by economic
historians.

(E)

Its impact on Latin America should not be
compared with its impact on the United States.

7.4 Reading Comprehension Practice Questions

473. Which of the following, if true, would most strengthen
the author’s assertion regarding economic indicators in
lines 25–27 ?
(A)

During an economic depression, European textile
manufacturers’ profits rise while their industrial
output remains steady.

(B)

During a national economic recession, United
States microchips manufacturers’ profits rise
sharply while United States steel manufacturers’
profits plunge.

(C)

During the years following a severe economic
depression, textile manufacturers’ output levels
and profit levels increase in Brazil and Mexico
but not in the rest of Latin America.

(D)

Although Japanese industry as a whole recovers
after an economic recession, it does not regain
its previously high levels of production.

(E)

While European industrial output increases in the
years following an economic depression, total
output remains below that of Japan or the United
States.

395

GMAT® Official Guide 2018

Line

(5)

(10)

(15)

(20)

(25)

(30)

396

Among the myths taken as fact by the
environmental managers of most corporations is
the belief that environmental regulations affect all
competitors in a given industry uniformly. In reality,
regulatory costs—and therefore compliance—
fall unevenly, economically disadvantaging some
companies and benefiting others. For example, a
plant situated near a number of larger
noncompliant competitors is less likely to attract
the attention of local regulators than is an isolated
plant, and less attention means lower costs.
Additionally, large plants can spread compliance
costs such as waste treatment across a larger
revenue base; on the other hand, some smaller
plants may not even be subject to certain
provisions such as permit or reporting
requirements by virtue of their size. Finally, older
production technologies often continue to generate
toxic wastes that were not regulated when the
technology was first adopted. New regulations
have imposed extensive compliance costs on
companies still using older industrial coal‐fired
burners that generate high sulfur dioxide and
nitrogen oxide outputs, for example, whereas new
facilities generally avoid processes that would
create such waste products. By realizing that they
have discretion and that not all industries are
affected equally by environmental regulation,
environmental managers can help their companies
to achieve a competitive edge by anticipating
regulatory pressure and exploring all possibilities for
addressing how changing regulations will affect their
companies specifically.

Questions 474–477 refer to the passage.

474. It can be inferred from the passage that a large plant
might have to spend more than a similar but smaller
plant on environmental compliance because the larger
plant is
(A)

more likely to attract attention from local
regulators

(B)

less likely to be exempt from permit and
reporting requirements

(C)

less likely to have regulatory costs passed on to
it by companies that supply its raw materials

(D)

more likely to employ older production
technologies

(E)

more likely to generate wastes that are more
environmentally damaging than those generated
by smaller plants

475. According to the passage, which of the following
statements about sulfur dioxide and nitrogen oxide
outputs is true?
(A)

Older production technologies cannot be
adapted so as to reduce production of these
outputs as waste products.

(B)

Under the most recent environmental regulations,
industrial plants are no longer permitted to
produce these outputs.

(C)

Although these outputs are environmentally
hazardous, some plants still generate them as
waste products despite the high compliance
costs they impose.

(D)

Many older plants have developed innovative
technological processes that reduce the
amounts of these outputs generated as waste
products.

(E)

Since the production processes that generate
these outputs are less costly than alternative
processes, these less expensive processes are
sometimes adopted despite their acknowledged
environmental hazards.

7.4 Reading Comprehension Practice Questions

476. Which of the following best describes the relationship
of the statement about large plants (lines 12–17) to
the passage as a whole?
(A)

It presents a hypothesis that is disproved later in
the passage.

(B)

It highlights an opposition between two ideas
mentioned in the passage.

(C)

It provides examples to support a claim made
earlier in the passage.

(D)

It exemplifies a misconception mentioned earlier
in the passage.

(E)

It draws an analogy between two situations
described in the passage.

477. The primary purpose of the passage is to
(A)

address a widespread environmental
management problem and suggest possible
solutions

(B)

illustrate varying levels of compliance with
environmental regulation among different
corporations

(C)

describe the various alternatives to traditional
methods of environmental management

(D)

advocate increased corporate compliance with
environmental regulation

(E)

correct a common misconception about the
impact of environmental regulations

397

GMAT® Official Guide 2018

Line

(5)

(10)

(15)

(20)

(25)

(30)

(35)

(40)

398

Milankovitch proposed in the early twentieth
century that the ice ages were caused by variations
in the Earth’s orbit around the Sun. For some time
this theory was considered untestable, largely
because there was no sufficiently precise
chronology of the ice ages with which the orbital
variations could be matched.
To establish such a chronology it is necessary to
determine the relative amounts of land ice that
existed at various times in the Earth’s past. A
recent discovery makes such a determination
possible: relative land‐ice volume for a given period
can be deduced from the ratio of two oxygen
isotopes, 16 and 18, found in ocean sediments.
Almost all the oxygen in water is oxygen 16, but a
few molecules out of every thousand incorporate the
heavier isotope 18. When an ice age begins, the
continental ice sheets grow, steadily reducing the
amount of water evaporated from the ocean that
will eventually return to it. Because heavier
isotopes tend to be left behind when water
evaporates from the ocean surfaces, the remaining
ocean water becomes progressively enriched in
oxygen 18. The degree of enrichment can be
determined by analyzing ocean sediments of the
period, because these sediments are composed of
calcium carbonate shells of marine organisms,
shells that were constructed with oxygen atoms
drawn from the surrounding ocean. The higher the
ratio of oxygen 18 to oxygen 16 in a sedimentary
specimen, the more land ice there was when the
sediment was laid down.
As an indicator of shifts in the Earth’s climate,
the isotope record has two advantages. First, it is a
global record: there is remarkably little variation in
isotope ratios in sedimentary specimens taken from
different continental locations. Second, it is a
more continuous record than that taken from rocks
on land. Because of these advantages, sedimentary
evidence can be dated with sufficient accuracy
by radiometric methods to establish a precise
chronology of the ice ages. The dated isotope
record shows that the fluctuations in global ice
volume over the past several hundred thousand

(45)

(50)

(55)

years have a pattern: an ice age occurs roughly once
every 100,000 years. These data have established
a strong connection between variations in the Earth’s
orbit and the periodicity of the ice ages.
However, it is important to note that other
factors, such as volcanic particulates or variations
in the amount of sunlight received by the Earth,
could potentially have affected the climate. The
advantage of the Milankovitch theory is that it
is testable; changes in the Earth’s orbit can be
calculated and dated by applying Newton’s laws of
gravity to progressively earlier configurations of the
bodies in the solar system. Yet the lack of
information about other possible factors affecting
global climate does not make them unimportant.

Questions 478–483 refer to the passage.

478. In the passage, the author is primarily interested in
(A)

suggesting an alternative to an outdated
research method

(B)

introducing a new research method that calls an
accepted theory into question

(C)

emphasizing the instability of data gathered from
the application of a new scientific method

(D)

presenting a theory and describing a new
method to test that theory

(E)

initiating a debate about a widely accepted
theory

479. The author of the passage would be most likely to
agree with which of the following statements about the
Milankovitch theory?
(A)

It is the only possible explanation for the ice
ages.

(B)

It is too limited to provide a plausible explanation
for the ice ages, despite recent research
findings.

(C)

It cannot be tested and confirmed until further
research on volcanic activity is done.

(D)

It is one plausible explanation, though not the
only one, for the ice ages.

(E)

It is not a plausible explanation for the ice ages,
although it has opened up promising possibilities
for future research.

7.4 Reading Comprehension Practice Questions

480. It can be inferred from the passage that the isotope
record taken from ocean sediments would be less
useful to researchers if which of the following were
true?

482. It can be inferred from the passage that precipitation
formed from evaporated ocean water has
(A)

the same isotopic ratio as ocean water

(B)

less oxygen 18 than does ocean water

(A)

It indicated that lighter isotopes of oxygen
predominated at certain times.

(C)

less oxygen 18 than has the ice contained in
continental ice sheets

(B)

It had far more gaps in its sequence than the
record taken from rocks on land.

(D)

a different isotopic composition than has
precipitation formed from water on land

(C)

It indicated that climate shifts did not occur
every 100,000 years.

(E)

more oxygen 16 than has precipitation formed
from fresh water

(D)

It indicated that the ratios of oxygen 16 and
oxygen 18 in ocean water were not consistent
with those found in fresh water.

(E)

483. It can be inferred from the passage that calcium
carbonate shells

It stretched back for only a million years.

481. According to the passage, which of the following
is true of the ratios of oxygen isotopes in ocean
sediments?
(A)

They indicate that sediments found during an
ice age contain more calcium carbonate than
sediments formed at other times.

(B)

They are less reliable than the evidence from
rocks on land in determining the volume of land
ice.

(C)

They can be used to deduce the relative volume
of land ice that was present when the sediment
was laid down.

(D)

They are more unpredictable during an ice age
than in other climatic conditions.

(E)

They can be used to determine atmospheric
conditions at various times in the past.

(A)

are not as susceptible to deterioration as rocks

(B)

are less common in sediments formed during an
ice age

(C)

are found only in areas that were once covered
by land ice

(D)

contain radioactive material that can be used to
determine a sediment’s isotopic composition

(E)

reflect the isotopic composition of the water at
the time the shells were formed

399

GMAT® Official Guide 2018

Line

(5)

(10)

(15)

(20)

(25)

(30)

400

Two works published in 1984 demonstrate
contrasting approaches to writing the history of
United States women. Buel and Buel’s biography
of Mary Fish (1736–1818) makes little effort to
place her story in the context of recent
historiography on women. Lebsock, meanwhile,
attempts not only to write the history of women in
one southern community, but also to redirect two
decades of historiographical debate as to
whether women gained or lost status in the
nineteenth century as compared with the
eighteenth century. Although both books offer the
reader the opportunity to assess this controversy
regarding women’s status, only Lebsock’s deals with
it directly. She examines several different aspects
of women’s status, helping to refine and resolve the
issues. She concludes that while
women gained autonomy in some areas,
especially in the private sphere, they lost it in
many aspects of the economic sphere. More
importantly, she shows that the debate itself
depends on frame of reference: in many respects,
women lost power in relation to men, for example,
as certain jobs (delivering babies, supervising
schools) were taken over by men. Yet women also
gained power in comparison with their previous
status, owning a higher proportion of real estate,
for example. In contrast, Buel and Buel’s
biography provides ample raw material for
questioning the myth, fostered by some
historians, of a colonial golden age in the
eighteenth century but does not give the reader
much guidance in analyzing the controversy over
women’s status.

Questions 484–489 refer to the passage.

484. The primary purpose of the passage is to
(A)

examine two sides of a historiographical debate

(B)

call into question an author’s approach to a
historiographical debate

(C)

examine one author’s approach to a
historiographical debate

(D)

discuss two authors’ works in relationship to a
historiographical debate

(E)

explain the prevalent perspective on a
historiographical debate

485. The author of the passage mentions the supervision of
schools primarily in order to
(A)

remind readers of the role education played in
the cultural changes of the nineteenth century in
the United States

(B)

suggest an area in which nineteenth‐century
American women were relatively free to exercise
power

(C)

provide an example of an occupation for which
accurate data about women’s participation are
difficult to obtain

(D)

speculate about which occupations were
considered suitable for United States women of
the nineteenth century

(E)

illustrate how the answers to questions about
women’s status depend on particular contexts

7.4 Reading Comprehension Practice Questions

486. With which of the following characterizations of
Lebsock’s contribution to the controversy concerning
women’s status in the nineteenth‐century United States
would the author of the passage be most likely to
agree?
(A)

Lebsock has studied women from a formerly
neglected region and time period.

(B)

Lebsock has demonstrated the importance of
frame of reference in answering questions about
women’s status.

(C)

Lebsock has addressed the controversy by using
women’s current status as a frame of reference.

(D)

Lebsock has analyzed statistics about
occupations and property that were previously
ignored.

488. The passage suggests that Lebsock believes that
compared to nineteenth‐century American women,
eighteenth‐century American women were
(A)

in many respects less powerful in relation to men

(B)

more likely to own real estate

(C)

generally more economically independent

(D)

more independent in conducting their private
lives

(E)

less likely to work as school superintendents

489. The passage suggests that Buel and Buel’s biography
of Mary Fish provides evidence for which of the
following views of women’s history?
(A)

Lebsock has applied recent historiographical
methods to the biography of a nineteenth‐century
woman.

Women have lost power in relation to men since
the colonial era.

(B)

Women of the colonial era were not as likely to
be concerned with their status as were women in
the nineteenth century.

487. According to the passage, Lebsock’s work differs from
Buel and Buel’s work in that Lebsock’s work

(C)

The colonial era was not as favorable for women
as some historians have believed.

(D)

Women had more economic autonomy in the
colonial era than in the nineteenth century.

(E)

Women’s occupations were generally more
respected in the colonial era than in the
nineteenth century.

(E)

(A)

uses a large number of primary sources

(B)

ignores issues of women’s legal status

(C)

refuses to take a position on women’s status in
the eighteenth century

(D)

addresses larger historiographical issues

(E)

fails to provide sufficient material to support its
claims

401

GMAT® Official Guide 2018

Line

(5)

(10)

(15)

(20)

(25)

(30)

402

Acting on the recommendation of a British
government committee investigating the high
incidence in white lead factories of illness among
employees, most of whom were women, the Home
Secretary proposed in 1895 that Parliament enact
legislation that would prohibit women from holding
most jobs in white lead factories. Although the
Women’s Industrial Defence Committee (WIDC),
formed in 1892 in response to earlier legislative
attempts to restrict women’s labor, did not discount
the white lead trade’s potential health dangers, it
opposed the proposal, viewing it as yet another
instance of limiting women’s work opportunities.
Also opposing the proposal was the Society for
Promoting the Employment of Women (SPEW),
which attempted to challenge it by investigating the
causes of illness in white lead factories. SPEW
contended, and WIDC concurred, that controllable
conditions in such factories were responsible for
the development of lead poisoning. SPEW provided
convincing evidence that lead poisoning could be
avoided if workers were careful and clean and if
already extant workplace safety regulations were
stringently enforced. However, the Women’s Trade
Union League (WTUL), which had ceased in the late
1880s to oppose restrictions on women’s labor,
supported the eventually enacted proposal, in part
because safety regulations were generally not being
enforced in white lead factories, where there were
no unions (and little prospect of any) to pressure
employers to comply with safety regulations.

Questions 490–492 refer to the passage.

490. The passage suggests that WIDC differed from WTUL
in which of the following ways?
(A)

WIDC believed that the existing safety regulations
were adequate to protect women’s health,
whereas WTUL believed that such regulations
needed to be strengthened.

(B)

WIDC believed that unions could not succeed
in pressuring employers to comply with such
regulations, whereas WTUL believed that unions
could succeed in doing so.

(C)

WIDC believed that lead poisoning in white
lead factories could be avoided by controlling
conditions there, whereas WTUL believed that
lead poisoning in such factories could not
be avoided no matter how stringently safety
regulations were enforced.

(D)

At the time that the legislation concerning white
lead factories was proposed, WIDC was primarily
concerned with addressing health conditions
in white lead factories, whereas WTUL was
concerned with improving working conditions in
all types of factories.

(E)

At the time that WIDC was opposing legislative
attempts to restrict women’s labor, WTUL had
already ceased to do so.

7.4 Reading Comprehension Practice Questions

491. Which of the following, if true, would most clearly
support the contention attributed to SPEW in lines
17–20 ?
(A)

Those white lead factories that most strongly
enforced regulations concerning worker safety
and hygiene had the lowest incidences of lead
poisoning among employees.

(B)

The incidence of lead poisoning was much
higher among women who worked in white lead
factories than among women who worked in
other types of factories.

(C)

There were many household sources of lead
that could have contributed to the incidence of
lead poisoning among women who also worked
outside the home in the late nineteenth century.

(D)

White lead factories were more stringent than
were certain other types of factories in their
enforcement of workplace safety regulations.

(E)

Even brief exposure to the conditions typically
found in white lead factories could cause lead
poisoning among factory workers.

492. The passage is primarily concerned with
(A)

presenting various groups’ views of the motives
of those proposing certain legislation

(B)

contrasting the reasoning of various groups
concerning their positions on certain proposed
legislation

(C)

tracing the process whereby certain proposed
legislation was eventually enacted

(D)

assessing the success of tactics adopted by
various groups with respect to certain proposed
legislation

(E)

evaluating the arguments of various groups
concerning certain proposed legislation

403

GMAT® Official Guide 2018

Line

(5)

(10)

(15)

(20)

(25)

(30)

(35)

(40)

(45)

(50)

404

It is an odd but indisputable fact that the
seventeenth‐century English women who are
generally regarded as among the forerunners of
modern feminism are almost all identified with the
Royalist side in the conflict between Royalists and
Parliamentarians known as the English Civil Wars.
Since Royalist ideology is often associated with the
radical patriarchalism of seventeenth‐century
political theorist Robert Filmer—a patriarchalism
that equates family and kingdom and asserts the
divinely ordained absolute power of the king and,
by analogy, of the male head of the household—
historians have been understandably puzzled by the
fact that Royalist women wrote the earliest
extended criticisms of the absolute subordination
of women in marriage and the earliest systematic
assertions of women’s rational and moral equality
with men. Some historians have questioned the
facile equation of Royalist ideology with Filmerian
patriarchalism; and indeed, there may have been
no consistent differences between Royalists and
Parliamentarians on issues of family organization
and women’s political rights, but in that case one
would expect early feminists to be equally divided
between the two sides.
Catherine Gallagher argues that Royalism
engendered feminism because the ideology of
absolute monarchy provided a transition to an
ideology of the absolute self. She cites the example
of the notoriously eccentric author Margaret
Cavendish (1626–1673), duchess of Newcastle.
Cavendish claimed to be as ambitious as any
woman could be, but knowing that as a woman she
was excluded from the pursuit of power in the real
world, she resolved to be mistress of her own
world, the “immaterial world” that any person can
create within her own mind—and, as a writer, on
paper. In proclaiming what she called her
“singularity,” Cavendish insisted that she was a
self‐sufficient being within her mental empire, the
center of her own subjective universe rather than a
satellite orbiting a dominant male planet. In
justifying this absolute singularity, Cavendish
repeatedly invoked the model of the absolute
monarch, a figure that became a metaphor for the
self‐enclosed, autonomous nature of the individual
person. Cavendish’s successors among early
feminists retained her notion of woman’s sovereign
self, but they also sought to break free from the
complete political and social isolation that her
absolute singularity entailed.

Questions 493–498 refer to the passage.

493. The author of the passage refers to Robert Filmer (see
line 9) primarily in order to
(A)

show that Royalist ideology was somewhat more
radical than most historians appear to realize

(B)

qualify the claim that patriarchalism formed the
basis of Royalist ideology

(C)

question the view that most early feminists were
associated with the Royalist faction

(D)

highlight an apparent tension between Royalist
ideology and the ideas of early feminists

(E)

argue that Royalists held conflicting opinions
on issues of family organization and women’s
political rights

7.4 Reading Comprehension Practice Questions

494. The passage suggests which of the following about
the seventeenth‐century English women mentioned
in line 2 ?

497. The primary purpose of the passage is to
(A)

trace the historical roots of a modern
sociopolitical movement

(A)

Their status as forerunners of modern feminism
is not entirely justified.

(B)

present one scholar’s explanation for a puzzling
historical phenomenon

(B)

They did not openly challenge the radical
patriarchalism of Royalist Filmerian ideology.

(C)

contrast two interpretations of the ideological
origins of a political conflict

(C)

Cavendish was the first among these women to
criticize women’s subordination in marriage and
assert women’s equality with men.

(D)

establish a link between the ideology of
an influential political theorist and that of a
notoriously eccentric writer

(D)

Their views on family organization and women’s
political rights were diametrically opposed to
those of both Royalist and Parliamentarian
ideology.

(E)

call attention to some points of agreement
between opposing sides in an ideological debate

(E)

Historians would be less puzzled if more of them
were identified with the Parliamentarian side in
the English Civil Wars.

495. The passage suggests that Margaret Cavendish’s
decision to become an author was motivated, at least
in part, by a desire to
(A)

justify her support for the Royalist cause

(B)

encourage her readers to work toward
eradicating Filmerian patriarchalism

(C)

persuade other women to break free from their
political and social isolation

(D)

analyze the causes for women’s exclusion from
the pursuit of power

(E)

create a world over which she could exercise
total control

498. Which of the following, if true, would most clearly
undermine Gallagher’s explanation of the link between
Royalism and feminism?
(A)

Because of their privileged backgrounds,
Royalist women were generally better educated
than were their Parliamentarian counterparts.

(B)

Filmer himself had read some of Cavendish’s
early writings and was highly critical of her ideas.

(C)

Cavendish’s views were highly individual and
were not shared by the other Royalist women
who wrote early feminist works.

(D)

The Royalist and Parliamentarian ideologies
were largely in agreement on issues of family
organization and women’s political rights.

(E)

The Royalist side included a sizable minority
faction that was opposed to the more radical
tendencies of Filmerian patriarchalism.

496. The phrase “a satellite orbiting a dominant male planet”
(lines 41–42) refers most directly to
(A)

Cavendish’s concept that each woman is a
sovereign self

(B)

the complete political and social isolation of
absolute singularity

(C)

the immaterial world that a writer can create on
paper

(D)

the absolute subordination of women in a
patriarchal society

(E)

the metaphorical figure of the absolute monarch

405

GMAT® Official Guide 2018

Line

(5)

(10)

(15)

(20)

(25)

(30)

406

Frazier and Mosteller assert that medical research
could be improved by a move toward larger, simpler
clinical trials of medical treatments. Currently,
researchers collect far more background information
on patients than is strictly required for their trials—
substantially more than hospitals collect—thereby
escalating costs of data collection, storage, and
analysis. Although limiting information collection
could increase the risk that researchers will overlook
facts relevant to a study, Frazier and Mosteller
contend that such risk, never entirely eliminable from
research, would still be small in most studies. Only
in research on entirely new treatments are new and
unexpected variables likely to arise.
Frazier and Mosteller propose not only that
researchers limit data collection on individual
patients but also that researchers enroll more
patients in clinical trials, thereby obtaining a more
representative sample of the total population with
the disease under study. Often researchers restrict
study participation to patients who have no ailments
besides those being studied. A treatment judged
successful under these ideal conditions can then be
evaluated under normal conditions. Broadening the
range of trial participants, Frazier and Mosteller
suggest, would enable researchers to evaluate a
treatment’s efficacy for diverse patients under various
conditions and to evaluate its effectiveness for
different patient subgroups. For example, the value
of a treatment for a progressive disease may vary
according to a patient’s stage of disease. Patients’
ages may also affect a treatment’s efficacy.

Questions 499–503 refer to the passage.

499. The passage is primarily concerned with
(A)

identifying two practices in medical research that
may affect the accuracy of clinical trials

(B)

describing aspects of medical research that tend
to drive up costs

(C)

evaluating an analysis of certain shortcomings of
current medical research practices

(D)

describing proposed changes to the ways in
which clinical trials are conducted

(E)

explaining how medical researchers have
traditionally conducted clinical trials and how
such trials are likely to change

500. Which of the following can be inferred from the
passage about a study of the category of patients
referred to in lines 20–22 ?
(A)

Its findings might have limited applicability.

(B)

It would be prohibitively expensive in its attempt
to create ideal conditions.

(C)

It would be the best way to sample the total
population of potential patients.

(D)

It would allow researchers to limit information
collection without increasing the risk that
important variables could be overlooked.

(E)

Its findings would be more accurate if it
concerned treatments for a progressive
disease than if it concerned treatments for a
nonprogressive disease.

7.4 Reading Comprehension Practice Questions

501. It can be inferred from the passage that a study limited
to patients like those mentioned in lines 20–22 would
have which of the following advantages over the kind
of study proposed by Frazier and Mosteller?

(A)

They expend resources on the storage of
information likely to be irrelevant to the study
they are conducting.

(B)

It would limit the number of variables researchers
would need to consider when evaluating the
treatment under study.

They sometimes compromise the accuracy of
their findings by collecting and analyzing more
information than is strictly required for their
trials.

(C)

It would help researchers to identify subgroups
of patients with secondary conditions that might
also be treatable.

They avoid the risk of overlooking variables that
might affect their findings, even though doing so
raises their research costs.

(D)

It would enable researchers to assess the value
of an experimental treatment for the average
patient.

Because they attempt to analyze too much
information, they overlook facts that could
emerge as relevant to their studies.

(E)

In order to approximate the conditions typical
of medical treatment, they base their methods
of information collection on those used by
hospitals.

(A)

It would yield more data and its findings would be
more accurate.

(B)

It would cost less in the long term, though it
would be more expensive in its initial stages.

(C)

(D)

(E)

503. According to the passage, which of the following
describes a result of the way in which researchers
generally conduct clinical trials?

502. The author mentions patients’ ages (line 32) primarily
in order to
(A)

identify the most critical variable differentiating
subgroups of patients

(B)

cast doubt on the advisability of implementing
Frazier and Mosteller’s proposals about medical
research

(C)

indicate why progressive diseases may require
different treatments at different stages

(D)

illustrate a point about the value of enrolling a
wide range of patients in clinical trials

(E)

substantiate an argument about the problems
inherent in enrolling large numbers of patients in
clinical trials

407

GMAT® Official Guide 2018

Line

(5)

(10)

(15)

(20)

(25)

(30)

(35)

(40)

(45)

(50)

408

There are recent reports of apparently drastic
declines in amphibian populations and of extinctions
of a number of the world’s endangered amphibian
species. These declines, if real, may be signs of a
general trend toward extinction, and many
environmentalists have claimed that immediate
environmental action is necessary to remedy
this “amphibian crisis,” which, in their view, is an
indicator of general and catastrophic environmental
degradation due to human activity.
To evaluate these claims, it is useful to make a
preliminary distinction that is far too often ignored.
A declining population should not be confused with
an endangered one. An endangered population is
always rare, almost always small, and, by definition,
under constant threat of extinction even without a
proximate cause in human activities. Its disappearance,
however unfortunate, should come as no great
surprise. Moreover, chance events—which may
indicate nothing about the direction of trends in
population size—may lead to its extinction. The
probability of extinction due to such random factors
depends on the population size and is independent of
the prevailing direction of change in that size.
For biologists, population declines are potentially
more worrisome than extinctions. Persistent
declines, especially in large populations, indicate a
changed ecological context. Even here, distinctions
must again be made among declines that are only
apparent (in the sense that they are part of habitual
cycles or of normal fluctuations), declines that take
a population to some lower but still acceptable
level, and those that threaten extinction (e.g., by
taking the number of individuals below the minimum
viable population). Anecdotal reports of population
decreases cannot distinguish among these
possibilities, and some amphibian populations have
shown strong fluctuations in the past.
It is indisputably true that there is simply not
enough long‐term scientific data on amphibian
populations to enable researchers to identify real
declines in amphibian populations. Many fairly
common amphibian species declared all but extinct
after severe declines in the 1950s and 1960s
have subsequently recovered, and so might
the apparently declining populations that have
generated the current appearance of an amphibian
crisis. Unfortunately, long‐term data will not soon
be forthcoming, and postponing environmental
action while we wait for it may doom species and
whole ecosystems to extinction.

Questions 504–509 refer to the passage.

504. The primary purpose of the passage is to
(A)

assess the validity of a certain view

(B)

distinguish between two phenomena

(C)

identify the causes of a problem

(D)

describe a disturbing trend

(E)

allay concern about a particular phenomenon

505. It can be inferred from the passage that the author
believes which of the following to be true of the
environmentalists mentioned in lines 5–6 ?
(A)

They have wrongly chosen to focus on anecdotal
reports rather than on the long‐term data that
are currently available concerning amphibians.

(B)

Their recommendations are flawed because
their research focuses too narrowly on a single
category of animal species.

(C)

Their certainty that population declines in general
are caused by environmental degradation is not
warranted.

(D)

They have drawn premature conclusions
concerning a crisis in amphibian populations
from recent reports of declines.

(E)

They have overestimated the effects of chance
events on trends in amphibian populations.

506. It can be inferred from the passage that the author
believes which of the following to be true of the
amphibian extinctions that have recently been
reported?
(A)

They have resulted primarily from human
activities causing environmental degradation.

(B)

They could probably have been prevented if
timely action had been taken to protect the
habitats of amphibian species.

(C)

They should not come as a surprise, because
amphibian populations generally have been
declining for a number of years.

(D)

They have probably been caused by a
combination of chance events.

(E)

They do not clearly constitute evidence of
general environmental degradation.

7.4 Reading Comprehension Practice Questions

507. According to the passage, each of the following is true
of endangered amphibian species EXCEPT:

509. Which of the following best describes the function of
the sentence in lines 35–38 ?

(A)

They are among the rarest kinds of amphibians.

(A)

To give an example of a particular kind of study

(B)

They generally have populations that are small in
size.

(B)

To cast doubt on an assertion made in the
previous sentence

(C)

They are in constant danger of extinction.

(C)

(D)

Those with decreasing populations are the most
likely candidates for immediate extinction.

To raise an objection to a view presented in the
first paragraph

(D)

To provide support for a view presented in the
first paragraph

(E)

To introduce an idea that will be countered in the
following paragraph

(E)

They are in danger of extinction due to events
that sometimes have nothing to do with human
activities.

508. Which of the following most accurately describes the
organization of the passage?
(A)

A question is raised, a distinction regarding it is
made, and the question is answered.

(B)

An interpretation is presented, its soundness is
examined, and a warning is given.

(C)

A situation is described, its consequences are
analyzed, and a prediction is made.

(D)

Two interpretations of a phenomenon are
described, and one of them is rejected as invalid.

(E)

Two methods for analyzing a phenomenon are
compared, and further study of the phenomenon
is recommended.

409

GMAT® Official Guide 2018

Line

(5)

(10)

(15)

(20)

(25)

(30)

410

While the most abundant and dominant species
within a particular ecosystem is often crucial in
perpetuating the ecosystem, a “keystone” species,
here defined as one whose effects are much larger
than would be predicted from its abundance, can
also play a vital role. But because complex species
interactions may be involved, identifying a keystone
species by removing the species and observing
changes in the ecosystem is problematic. It might
seem that certain traits would clearly define a species
as a keystone species; for example,
Pisaster ochraceus is often a keystone predator
because it consumes and suppresses mussel
populations, which in the absence of this starfish
can be a dominant species. But such predation on a
dominant or potentially dominant species occurs in
systems that do as well as in systems that do not
have species that play keystone roles. Moreover,
whereas P. ochraceus occupies an unambiguous
keystone role on wave‐exposed rocky headlands,
in more wave‐sheltered habitats the impact of
P. ochraceus predation is weak or nonexistent,
and at certain sites sand burial is responsible for
eliminating mussels. Keystone status appears to
depend on context, whether of particular
geography or of such factors as community
diversity (for example, a reduction in species
diversity may thrust more of the remaining species
into keystone roles) and length of species
interaction (since newly arrived species in particular
may dramatically affect ecosystems).

Questions 510–513 refer to the passage.

510. The passage mentions which of the following as
a factor that affects the role of P. ochraceus as a
keystone species within different habitats?
(A)

The degree to which the habitat is sheltered from
waves

(B)

The degree to which other animals within a
habitat prey on mussels

(C)

The fact that mussel populations are often not
dominant within some habitats occupied by
P. ochraceus

(D)

The size of the P. ochraceus population within
the habitat

(E)

The fact that there is great species diversity
within some habitats occupied by P. ochraceus

511. Which of the following hypothetical experiments most
clearly exemplifies the method of identifying species’
roles that the author considers problematic?
(A)

A population of seals in an Arctic habitat is
counted in order to determine whether it is the
dominant species in that ecosystem.

(B)

A species of fish that is a keystone species in
one marine ecosystem is introduced into another
marine ecosystem to see whether the species
will come to occupy a keystone role.

(C)

In order to determine whether a species of
monkey is a keystone species within a particular
ecosystem, the monkeys are removed from that
ecosystem and the ecosystem is then studied.

(D)

Different mountain ecosystems are compared to
determine how geography affects a particular
species’ ability to dominate its ecosystem.

(E)

In a grassland experiencing a changing climate,
patterns of species extinction are traced in order
to evaluate the effect of climate changes on
keystone species in that grassland.

7.4 Reading Comprehension Practice Questions

512. Which of the following, if true, would most clearly
support the argument about keystone status advanced
in the last sentence of the passage (lines 24–31) ?
(A)

(B)

(C)

513. The passage suggests which of the following about the
identification of a species as a keystone species?
(A)

A species of bat is primarily responsible for
keeping insect populations within an ecosystem
low, and the size of the insect population in turn
affects bird species within that ecosystem.

Such an identification depends primarily on the
species’ relationship to the dominant species.

(B)

A species of iguana occupies a keystone role
on certain tropical islands, but does not play
that role on adjacent tropical islands that are
inhabited by a greater number of animal species.

Such an identification can best be made
by removing the species from a particular
ecosystem and observing changes that occur in
the ecosystem.

(C)

Such an identification is likely to be less reliable
as an ecosystem becomes less diverse.

(D)

Close observation of a savannah ecosystem
reveals that more species occupy keystone
roles within that ecosystem than biologists had
previously believed.

Such an identification seems to depend on
various factors within the ecosystem.

(E)

Such an identification can best be made by
observing predation behavior.

(D)

As a keystone species of bee becomes more
abundant, it has a larger effect on the ecosystem
it inhabits.

(E)

A species of moth that occupies a keystone role
in a prairie habitat develops coloration patterns
that camouflage it from potential predators.

411

GMAT® Official Guide 2018

Line

(5)

(10)

(15)

(20)

(25)

412

Conodonts, the spiky phosphatic remains (bones
and teeth composed of calcium phosphate) of
tiny marine animals that probably appeared about
520 million years ago, were once among the most
controversial of fossils. Both the nature of the
organism to which the remains belonged and the
function of the remains were unknown. However,
since the 1981 discovery of fossils preserving not
just the phosphatic elements but also other remains
of the tiny soft-bodied animals (also called conodonts)
that bore them, scientists’ reconstructions of the
animals’ anatomy have had important implications
for hypotheses concerning the development of the
vertebrate skeleton.
The vertebrate skeleton had traditionally been
regarded as a defensive development, champions of
this view postulating that it was only with the much
later evolution of jaws that vertebrates became
predators. The first vertebrates, which were softbodied, would have been easy prey for numerous
invertebrate carnivores, especially if these early
vertebrates were sedentary suspension feeders.
Thus, traditionalists argued, these animals developed
coverings of bony scales or plates, and teeth were
secondary features, adapted from the protective
bony scales. Indeed, external skeletons of this
type are common among the well-known fossils of
ostracoderms, jawless vertebrates that existed from
approximately 500 to 400 million years ago.

(30)

(35)

(40)

(45)

However, other paleontologists argued that many of
the definitive characteristics of vertebrates, such as
paired eyes and muscular and skeletal adaptations
for active life, would not have evolved unless the
first vertebrates were predatory. Teeth were more
primitive than external armor according to this view,
and the earliest vertebrates were predators.
The stiffening notochord along the back of the
body, V-shaped muscle blocks along the sides,
and posterior tail fins help to identify conodonts as
among the most primitive of vertebrates. The lack of
any mineralized structures apart from the elements
in the mouth indicates that conodonts were more
primitive than the armored jawless fishes such as the
ostracoderms. It now appears that the hard parts that
first evolved in the mouth of an animal improved its
efficiency as a predator, and that aggression rather
than protection was the driving force behind the origin
of the vertebrate skeleton.

7.4 Reading Comprehension Practice Questions

Questions 514–516 refer to the passage.

514. According to the passage, the anatomical evidence
provided by the preserved soft bodies of conodonts
led scientists to conclude that

516. It can be inferred that on the basis of the 1981
discovery of conodont remains, paleontologists could
draw which of the following conclusions?
(A)

The earliest vertebrates were sedentary
suspension feeders.

(B)

Ostracoderms were not the earliest vertebrates.

(A)

conodonts had actually been invertebrate
carnivores

(C)

Defensive armor preceded jaws among
vertebrates.

(B)

conodonts’ teeth were adapted from protective
bony scales

(D)

Paired eyes and adaptations for activity are
definitive characteristics of vertebrates.

(C)

conodonts were primitive vertebrate suspension
feeders

(E)

Conodonts were unlikely to have been predators.

(D)

primitive vertebrates with teeth appeared earlier
than armored vertebrates

(E)

scientists’ original observations concerning
the phosphatic remains of conodonts were
essentially correct

515. The second paragraph in the passage serves primarily to
(A)

outline the significance of the 1981 discovery of
conodont remains to the debate concerning the
development of the vertebrate skeleton

(B)

contrast the traditional view of the development
of the vertebrate skeleton with a view derived
from the 1981 discovery of conodont remains

(C)

contrast the characteristics of the ostracoderms
with the characteristics of earlier soft-bodied
vertebrates

(D)

explain the importance of the development of
teeth among the earliest vertebrate predators

(E)

present the two sides of the debate concerning
the development of the vertebrate skeleton

413

GMAT® Official Guide 2018

Line

(5)

(10)

(15)

(20)

(25)

(30)

(35)

(40)

(45)

(50)

414

Jon Clark’s study of the effect of the modernization
of a telephone exchange on exchange maintenance
work and workers is a solid contribution to a debate
that encompasses two lively issues in the history and
sociology of technology: technological determinism
and social constructivism.
Clark makes the point that the characteristics of a
technology have a decisive influence on job skills and
work organization. Put more strongly, technology can
be a primary determinant of social and managerial
organization. Clark believes this possibility has
been obscured by the recent sociological fashion,
exemplified by Braverman’s analysis, that emphasizes
the way machinery reflects social choices. For
Braverman, the shape of a technological system is
subordinate to the manager’s desire to wrest control
of the labor process from the workers. Technological
change is construed as the outcome of negotiations
among interested parties who seek to incorporate
their own interests into the design and configuration
of the machinery. This position represents the new
mainstream called social constructivism.
The constructivists gain acceptance by
misrepresenting technological determinism:
technological determinists are supposed to believe,
for example, that machinery imposes appropriate
forms of order on society. The alternative to
constructivism, in other words, is to view technology
as existing outside society, capable of directly
influencing skills and work organization.
Clark refutes the extremes of the constructivists
by both theoretical and empirical arguments.
Theoretically he defines “technology” in terms of
relationships between social and technical variables.
Attempts to reduce the meaning of technology to
cold, hard metal are bound to fail, for machinery is
just scrap unless it is organized functionally and
supported by appropriate systems of operation and
maintenance. At the empirical level Clark shows how
a change at the telephone exchange from
maintenance‐intensive electromechanical switches
to semielectronic switching systems altered work
tasks, skills, training opportunities, administration,
and organization of workers. Some changes Clark
attributes to the particular way management and
labor unions negotiated the introduction of the
technology, whereas others are seen as arising from
the capabilities and nature of the technology itself.
Thus Clark helps answer the question: “When is
social choice decisive and when are the concrete
characteristics of technology more important?”

Questions 517–524 refer to the passage.

517. The primary purpose of the passage is to
(A)

advocate a more positive attitude toward
technological change

(B)

discuss the implications for employees of the
modernization of a telephone exchange

(C)

consider a successful challenge to the
constructivist view of technological change

(D)

challenge the position of advocates of
technological determinism

(E)

suggest that the social causes of technological
change should be studied in real situations

518. Which of the following statements about the
modernization of the telephone exchange is supported
by information in the passage?
(A)

The new technology reduced the role of
managers in labor negotiations.

(B)

The modernization was implemented without the
consent of the employees directly affected by it.

(C)

The modernization had an impact that went
significantly beyond maintenance routines.

(D)

Some of the maintenance workers felt victimized
by the new technology.

(E)

The modernization gave credence to the view of
advocates of social constructivism.

519. Which of the following most accurately describes
Clark’s opinion of Braverman’s position?
(A)

He respects its wide‐ranging popularity.

(B)

He disapproves of its misplaced emphasis on the
influence of managers.

(C)

He admires the consideration it gives to the
attitudes of the workers affected.

(D)

He is concerned about its potential to impede
the implementation of new technologies.

(E)

He is sympathetic to its concern about the
impact of modern technology on workers.

7.4 Reading Comprehension Practice Questions

520. The information in the passage suggests that which of
the following statements from hypothetical sociological
studies of change in industry most clearly exemplifies
the social constructivists’ version of technological
determinism?
(A)

It is the available technology that determines
workers’ skills, rather than workers’ skills
influencing the application of technology.

(B)

All progress in industrial technology grows out of
a continuing negotiation between technological
possibility and human need.

(C)

Some organizational change is caused by
people; some is caused by computer chips.

(D)

Most major technological advances in industry
have been generated through research and
development.

(E)

Some industrial technology eliminates jobs, but
educated workers can create whole new skills
areas by the adaptation of the technology.

521. The information in the passage suggests that Clark
believes that which of the following would be true
if social constructivism had not gained widespread
acceptance?
(A)

Businesses would be more likely to modernize
without considering the social consequences of
their actions.

(B)

There would be greater understanding of the role
played by technology in producing social change.

(C)

Businesses would be less likely to understand
the attitudes of employees affected by
modernization.

(D)

Modernization would have occurred at a slower
rate.

(E)

Technology would have played a greater part in
determining the role of business in society.

522. According to the passage, constructivists employed
which of the following to promote their argument?
(A)

Empirical studies of business situations involving
technological change

(B)

Citation of managers supportive of their position

(C)

Construction of hypothetical situations that
support their view

(D)

Contrasts of their view with a misstatement of an
opposing view

(E)

Descriptions of the breadth of impact of
technological change

523. The author of the passage uses the expression “are
supposed to” in line 25 primarily in order to
(A)

suggest that a contention made by
constructivists regarding determinists is
inaccurate

(B)

define the generally accepted position of
determinists regarding the implementation of
technology

(C)

engage in speculation about the motivation of
determinists

(D)

lend support to a comment critical of the
position of determinists

(E)

contrast the historical position of determinists
with their position regarding the exchange
modernization

524. Which of the following statements about Clark’s study
of the telephone exchange can be inferred from
information in the passage?
(A)

Clark’s reason for undertaking the study was to
undermine Braverman’s analysis of the function
of technology.

(B)

Clark’s study suggests that the implementation
of technology should be discussed in the context
of conflict between labor and management.

(C)

Clark examined the impact of changes in the
technology of switching at the exchange in terms
of overall operations and organization.

(D)

Clark concluded that the implementation of new
switching technology was equally beneficial to
management and labor.

(E)

Clark’s analysis of the change in switching
systems applies only narrowly to the situation at
the particular exchange that he studied.
415

GMAT® Official Guide 2018

Line

(5)

(10)

(15)

(20)

(25)

(30)

(35)

(40)

(45)

(50)

416

Because the framers of the United States
Constitution (written in 1787) believed that protecting
property rights relating to inventions would encourage
the new nation’s economic growth, they gave
Congress—the national legislature—a constitutional
mandate to grant patents for inventions. The resulting
patent system has served as a model for those in
other nations. Recently, however, scholars have
questioned whether the American system helped
achieve the framers’ goals. These scholars have
contended that from 1794 to roughly 1830, American
inventors were unable to enforce property rights
because judges were “antipatent” and routinely
invalidated patents for arbitrary reasons. This
argument is based partly on examination of court
decisions in cases where patent holders (“patentees”)
brought suit alleging infringement of their patent
rights. In the 1820s, for instance, 75 percent
of verdicts were decided against the patentee.
The proportion of verdicts for the patentee began to
increase in the 1830s, suggesting to these scholars
that judicial attitudes toward patent rights began
shifting then.
Not all patent disputes in the early nineteenth
century were litigated, however, and litigated
cases were not drawn randomly from the
population of disputes. Therefore the rate of
verdicts in favor of patentees cannot be used
by itself to gauge changes in judicial attitudes
or enforceability of patent rights. If early judicial
decisions were prejudiced against patentees, one
might expect that subsequent courts—allegedly
more supportive of patent rights—would reject
the former legal precedents. But pre‐1830
cases have been cited as frequently as later
decisions, and they continue to be cited today,
suggesting that the early decisions, many of
which clearly declared that patent rights were
a just recompense for inventive ingenuity,
provided a lasting foundation for patent law.
The proportion of judicial decisions in favor of
patentees began to increase during the 1830s
because of a change in the underlying population
of cases brought to trial. This change was partly
due to an 1836 revision to the patent system:
an examination procedure, still in use today, was
instituted in which each application is scrutinized
for its adherence to patent law. Previously,
patents were automatically granted upon payment
of a $30 fee.

Questions 525–529 refer to the passage.

525. The passage implies that which of the following was
a reason that the proportion of verdicts in favor of
patentees began to increase in the 1830s ?
(A)

Patent applications approved after 1836 were
more likely to adhere closely to patent law.

(B)

Patent laws enacted during the 1830s better
defined patent rights.

(C)

Judges became less prejudiced against
patentees during the 1830s.

(D)

After 1836, litigated cases became less
representative of the population of patent
disputes.

(E)

The proportion of patent disputes brought to trial
began to increase after 1836.

526. The passage implies that the scholars mentioned in
line 8 would agree with which of the following criticisms
of the American patent system before 1830 ?
(A)

Its definition of property rights relating to
inventions was too vague to be useful.

(B)

Its criteria for the granting of patents were not
clear.

(C)

It made it excessively difficult for inventors to
receive patents.

(D)

It led to excessive numbers of patent‐
infringement suits.

(E)

It failed to encourage national economic growth.

527. It can be inferred from the passage that the frequency
with which pre‐1830 cases have been cited in court
decisions is an indication that
(A)

judicial support for patent rights was strongest in
the period before 1830

(B)

judicial support for patent rights did not increase
after 1830

(C)

courts have returned to judicial standards that
prevailed before 1830

(D)

verdicts favoring patentees in patent‐infringement
suits did not increase after 1830

(E)

judicial bias against patentees persisted after
1830

7.4 Reading Comprehension Practice Questions

528. It can be inferred from the passage that the author
and the scholars referred to in line 21 disagree about
which of the following aspects of the patents defended
in patent‐infringement suits before 1830 ?
(A)

Whether the patents were granted for inventions
that were genuinely useful

(B)

Whether the patents were actually relevant to the
growth of the United States economy

(C)

Whether the patents were particularly likely to be
annulled by judges

(D)

Whether the patents were routinely invalidated
for reasons that were arbitrary

(E)

Whether the patents were vindicated at a
significantly lower rate than patents in later suits

529. The author of the passage cites which of the following
as evidence challenging the argument referred to in
lines 14–15 ?
(A)

The proportion of cases that were decided
against patentees in the 1820s

(B)

The total number of patent disputes that were
litigated from 1794 to 1830

(C)

The fact that later courts drew upon the legal
precedents set in pre‐1830 patent cases

(D)

The fact that the proportion of judicial decisions
in favor of patentees began to increase during
the 1830s

(E)

The constitutional rationale for the 1836 revision
of the patent system

417

GMAT® Official Guide 2018

Line

(5)

(10)

(15)

(20)

(25)

(30)

(35)

(40)

418

Jacob Burckhardt’s view that Renaissance
European women “stood on a footing of perfect
equality” with Renaissance men has been repeatedly
cited by feminist scholars as a prelude to their
presentation of rich historical evidence of women’s
inequality. In striking contrast to Burckhardt, Joan
Kelly in her famous 1977 essay, “Did Women Have
a Renaissance?” argued that the Renaissance was
a period of economic and social decline for women
relative both to Renaissance men and to medieval
women. Recently, however, a significant trend
among feminist scholars has entailed a rejection
of both Kelly’s dark vision of the Renaissance and
Burckhardt’s rosy one. Many recent works by these
scholars stress the ways in which differences
among Renaissance women—especially in terms
of social status and religion—work to complicate
the kinds of generalizations both Burckhardt and
Kelly made on the basis of their observations about
upper‐class Italian women.
The trend is also evident, however, in works
focusing on those middle‐ and upper‐class
European women whose ability to write gives them
disproportionate representation in the historical
record. Such women were, simply by virtue of
their literacy, members of a tiny minority of the
population, so it is risky to take their descriptions of
their experiences as typical of “female experience”
in any general sense. Tina Krontiris, for example, in
her fascinating study of six Renaissance women
writers, does tend at times to conflate “women” and
“women writers,” assuming that women’s gender,
irrespective of other social differences, including
literacy, allows us to view women as a homogeneous
social group and make that group an object of
analysis. Nonetheless, Krontiris makes a significant
contribution to the field and is representative of
those authors who offer what might be called a
cautiously optimistic assessment of Renaissance
women’s achievements, although she also stresses
the social obstacles Renaissance women faced
when they sought to raise their “oppositional
voices.” Krontiris is concerned to show women
intentionally negotiating some power for themselves

(45)

(50)

(at least in the realm of public discourse) against
potentially constraining ideologies, but in her sober
and thoughtful concluding remarks, she suggests
that such verbal opposition to cultural stereotypes
was highly circumscribed; women seldom attacked
the basic assumptions in the ideologies that
oppressed them.

Questions 530–536 refer to the passage.

530. The author of the passage discusses Krontiris primarily
to provide an example of a writer who
(A)

is highly critical of the writings of certain
Renaissance women

(B)

supports Kelly’s view of women’s status during
the Renaissance

(C)

has misinterpreted the works of certain
Renaissance women

(D)

has rejected the views of both Burckhardt and
Kelly

(E)

has studied Renaissance women in a wide variety
of social and religious contexts

531. According to the passage, Krontiris’s work differs from
that of the scholars mentioned in line 12 in which of
the following ways?
(A)

Krontiris’s work stresses the achievements of
Renaissance women rather than the obstacles to
their success.

(B)

Krontiris’s work is based on a reinterpretation of
the work of earlier scholars.

(C)

Krontiris’s views are at odds with those of both
Kelly and Burkhardt.

(D)

Krontiris’s work focuses on the place of women
in Renaissance society.

(E)

Krontiris’s views are based exclusively on the
study of a privileged group of women.

7.4 Reading Comprehension Practice Questions

532. According to the passage, feminist scholars cite
Burckhardt’s view of Renaissance women primarily for
which of the following reasons?

535. The last sentence in the passage serves primarily to
(A)

suggest that Krontiris’s work is not
representative of recent trends among feminist
scholars

(B)

undermine the argument that literate women
of the Renaissance sought to oppose social
constraints imposed on them

(A)

Burckhardt’s view forms the basis for most
arguments refuting Kelly’s point of view.

(B)

Burckhardt’s view has been discredited by Kelly.

(C)

Burckhardt’s view is one that many feminist
scholars wish to refute.

(C)

Burckhardt’s work provides rich historical
evidence of inequality between Renaissance
women and men.

show a way in which Krontiris’s work illustrates
a “cautiously optimistic” assessment of
Renaissance women’s achievements

(D)

summarize Krontiris’s view of the effect of
literacy on the lives of upper‐ and middle‐class
Renaissance women

(E)

illustrate the way in which Krontiris’s study differs
from the studies done by Burckhardt and Kelly

(D)

(E)

Burckhardt’s work includes historical research
supporting the arguments of the feminist scholars.

533. It can be inferred that both Burckhardt and Kelly have
been criticized by the scholars mentioned in line 12 for
which of the following?
(A)

(B)
(C)

(D)

(E)

Assuming that women writers of the Renaissance
are representative of Renaissance women in
general

536. The author of the passage implies that the women
studied by Krontiris are unusual in which of the
following ways?
(A)

They faced obstacles less formidable than those
faced by other Renaissance women.

(B)

Failing to describe clearly the relationship
between social status and literacy among
Renaissance women

They have been seen by historians as more
interesting than other Renaissance women.

(C)

Failing to acknowledge the role played by
Renaissance women in opposing cultural
stereotypes

They were more concerned about recording
history accurately than were other Renaissance
women.

(D)

Failing to acknowledge the ways in which
social status affected the creative activities of
Renaissance women

Their perceptions are more likely to be
accessible to historians than are those of most
other Renaissance women.

(E)

Their concerns are likely to be of greater interest
to feminist scholars than are the ideas of most
other Renaissance women.

Drawing conclusions that are based on the study
of an atypical group of women

534. The author of the passage suggests that Krontiris
incorrectly assumes that
(A)

social differences among Renaissance women are
less important than the fact that they were women

(B)

literacy among Renaissance women was more
prevalent than most scholars today acknowledge

(C)

during the Renaissance, women were able to
successfully oppose cultural stereotypes relating
to gender

(D)

Renaissance women did not face many difficult
social obstacles relating to their gender

(E)

in order to attain power, Renaissance women
attacked basic assumptions in the ideologies
that oppressed them
419

GMAT® Official Guide 2018

Line

(5)

(10)

(15)

(20)

(25)

420

When asteroids collide, some collisions cause
an asteroid to spin faster; others slow it down. If
asteroids are all monoliths—single rocks—undergoing
random collisions, a graph of their rotation rates
should show a bell-shaped distribution with statistical
“tails” of very fast and very slow rotators. If asteroids
are rubble piles, however, the tail representing the
very fast rotators would be missing, because any
loose aggregate spinning faster than once every few
hours (depending on the asteroid’s bulk density)
would fly apart. Researchers have discovered that
all but five observed asteroids obey a strict limit on
rate of rotation. The exceptions are all smaller than
200 meters in diameter, with an abrupt cutoff for
asteroids larger than that.
The evident conclusion—that asteroids larger than
200 meters across are multicomponent structures or
rubble piles—agrees with recent computer modeling
of collisions, which also finds a transition at that
diameter. A collision can blast a large asteroid to bits,
but after the collision those bits will usually move
slower than their mutual escape velocity. Over several
hours, gravity will reassemble all but the fastest
pieces into a rubble pile. Because collisions among
asteroids are relatively frequent, most large bodies
have already suffered this fate. Conversely, most
small asteroids should be monolithic, because impact
fragments easily escape their feeble gravity.

Questions 537–540 refer to the passage.

537. The passage implies which of the following about the
five asteroids mentioned in line 12 ?
(A)

Their rotation rates are approximately the same.

(B)

They have undergone approximately the same
number of collisions.

(C)

They are monoliths.

(D)

They are composed of fragments that have
escaped the gravity of larger asteroids.

(E)

They were detected only recently.

538. The discovery of which of the following would call into
question the conclusion mentioned in line 16 ?
(A)

An asteroid 100 meters in diameter rotating at a
rate of once per week

(B)

An asteroid 150 meters in diameter rotating at a
rate of 20 times per hour

(C)

An asteroid 250 meters in diameter rotating at a
rate of once per week

(D)

An asteroid 500 meters in diameter rotating at a
rate of once per hour

(E)

An asteroid 1,000 meters in diameter rotating at
a rate of once every 24 hours

7.4 Reading Comprehension Practice Questions

539. According to the passage, which of the following
is a prediction that is based on the strength of the
gravitational attraction of small asteroids?
(A)

Small asteroids will be few in number.

(B)

Small asteroids will be monoliths.

(C)

Small asteroids will collide with other asteroids
very rarely.

(D)

Most small asteroids will have very fast rotation
rates.

(E)

Almost no small asteroids will have very slow
rotation rates.

540. The author of the passage mentions “escape velocity”
(see line 22) in order to help explain which of the
following?
(A)

The tendency for asteroids to become smaller
rather than larger over time

(B)

The speed with which impact fragments
reassemble when they do not escape an
asteroid’s gravitational attraction after a collision

(C)

The frequency with which collisions among
asteroids occur

(D)

The rotation rates of asteroids smaller than
200 meters in diameter

(E)

The tendency for large asteroids to persist after
collisions

421

GMAT® Official Guide 2018

Line

(5)

(10)

(15)

(20)

(25)

(30)

Most attempts by physicists to send particles
faster than the speed of light involve a remarkable
phenomenon called quantum tunneling, in which
particles travel through solid barriers that appear
to be impenetrable. If you throw a ball at a wall,
you expect it to bounce back, not to pass straight
through it. Yet subatomic particles perform the
equivalent feat. Quantum theory says that there is a
distinct, albeit small, probability that such a particle
will tunnel its way through a barrier; the probability
declines exponentially as the thickness of the
barrier increases. Though the extreme rapidity of
quantum tunneling was noted as early as 1932,
not until 1955 was it hypothesized—by Wigner and
Eisenbud—that tunneling particles sometimes
travel faster than light. Their grounds were
calculations that suggested that the time it takes a
particle to tunnel through a barrier increases with
the thickness of the barrier until tunneling time
reaches a maximum; beyond that maximum,
tunneling time stays the same regardless of
barrier thickness. This would imply that once
maximum tunneling time is reached, tunneling
speed will increase without limit as barrier thickness
increases. Several recent experiments have
supported this hypothesis that tunneling particles
sometimes reach superluminal speed. According to
measurements performed by Raymond Chiao and
colleagues, for example, photons can pass through
an optical filter at 1.7 times the speed of light.

Questions 541–543 refer to the passage.

541. The author of the passage mentions calculations about
tunneling time and barrier thickness in order to
(A)

suggest that tunneling time is unrelated to
barrier thickness

(B)

explain the evidence by which Wigner and
Eisenbud discovered the phenomenon of
tunneling

(C)

describe data recently challenged by Raymond
Chiao and colleagues

(D)

question why particles engaged in quantum
tunneling rarely achieve extremely high speeds

(E)

explain the basis for Wigner and Eisenbud’s
hypothesis

542. The passage implies that if tunneling time reached no
maximum in increasing with barrier thickness, then
(A)

tunneling speed would increase with barrier
thickness

(B)

tunneling speed would decline with barrier
thickness

(C)

tunneling speed would vary with barrier thickness

(D)

tunneling speed would not be expected to
increase without limit

(E)

successful tunneling would occur even less
frequently than it does

543. Which of the following statements about the earliest
scientific investigators of quantum tunneling can be
inferred from the passage?

422

(A)

They found it difficult to increase barrier
thickness continually.

(B)

They anticipated the later results of Chiao and
his colleagues.

(C)

They did not suppose that tunneling particles
could travel faster than light.

(D)

They were unable to observe instances of
successful tunneling.

(E)

They made use of photons to study the
phenomenon of tunneling.

7.5 Reading Comprehension Answer Key

7.5 Answer Key
405. B

433. D

461. C

489. C

517. C

406. B

434. A

462. A

490. E

518. C

407. D

435. C

463. B

491. A

519. B

408. A

436. D

464. D

492. B

520. A

409. D

437. B

465. E

493. D

521. B

410. D

438. B

466. C

494. E

522. D

411. B

439. C

467. A

495. E

523. A

412. B

440. B

468. E

496. D

524. C

413. C

441. E

469. A

497. B

525. A

414. C

442. B

470. D

498. C

526. E

415. D

443. B

471. D

499. D

527. B

416. C

444. B

472. D

500. A

528. D

417. A

445. A

473. B

501. C

528. C

418. C

446. D

474. B

502. D

530. D

419. C

447. E

475. C

503. A

531. E

420. C

448. D

476. C

504. A

532. C

421. E

449. A

477. E

505. D

533. B

422. A

450. C

478. D

506. E

534. A

423. C

451. B

479. D

507. D

535. C

424. B

452. D

480. B

508. B

536. D

425. B

453. B

481. C

509. C

537. C

426. A

454. E

482. B

510. A

538. D

427. D

455. C

483. E

511. C

539. B

428. E

456. C

484. D

512. B

540. E

429. A

457. E

485. E

513. D

541. E

430. A

458. A

486. B

514. D

542. D

431. C

459. D

487. D

515. E

543. C

432. B

460. A

488. C

516. B

423

GMAT® Official Guide 2018

7.6 Answer Explanations
The following discussion of reading comprehension is intended to familiarize you with the most efficient
and effective approaches to the kinds of problems common to reading comprehension. The particular
questions in this chapter are generally representative of the kinds of reading comprehension questions
you will encounter on the GMAT exam. Remember that it is the problem solving strategy that is
important, not the specific details of a particular question.
Questions 405–408 refer to the passage on page 362.

405. According to the passage, theory B states that which
of the following is a factor that enables a schooling fish
to escape predators?
(A)

The tendency of fish to form compact groups

(B)

The movement of other fish within the school

(C)

The inability of predators to detect schools

(D)

The ability of fish to hide behind one another in
a school

(E)

The great speed with which a school can disperse

Supporting idea
This question depends on understanding what
the passage states about theory B, the “confusion
effect.” One element of theory B is that predators
may experience sensory confusion created by large
numbers of moving fish in a school.
A
B

C

D

E

The compactness of groups of schooling fish
is an element of theory A, not theory B.
Correct. It is the movement of schooling
fish around a predator that creates sensory
confusion in the predator; this movement
may distract the predator and help protect
individual fish in the school.
According to the passage’s description of
theory A, predators are actually slightly
more likely to detect schools than they are
to detect individual fish.
Theory B does not involve fish hiding
behind one another but rather moving
around the predator.
The passage does not discuss the speed of
dispersal of schools of fish.

The correct answer is B.

406. According to the passage, both theory A and theory B
have been developed to explain how
(A)

fish hide from predators by forming schools

(B)

forming schools functions to protect fish from
predators

(C)

schooling among fish differs from other
protective behaviors

(D)

small fish are able to make rapid decisions

(E)

small fish are able to survive in an environment
densely populated by large predators

Supporting idea
The passage states in its first paragraph that two
theories were developed to explain why schooling
occurs in so many fish species and that they both
assume that schooling helps protect fish from
predators.
A

B

C
D

E

While theory A involves an explanation of
how schooling makes an individual fish less
likely to be found by predators, theory B
explains how schooling protects fish even
when they are detected by predators.
Correct. Both theory A and theory B begin
with the assumption that schooling provides
protection from predators, and each theory
offers a different explanation for how that
protection occurs.
The passage does not discuss protective
behaviors other than schooling.
The decision‐making ability of predators, not
schooling fish, is discussed in the passage;
schooling is presented as an instinctive behavior.
The passage suggests that only theory B helps
explain schooling behavior in environments
where many predators, large or otherwise, are
found, and that theory A explains schooling
in areas where predators are not as abundant.

The correct answer is B.
424

7.6 Reading Comprehension Answer Explanations

407. According to one explanation of the “confusion
effect,” a fish that swims in a school will have greater
advantages for survival if it

408. The author is primarily concerned with
(A)

discussing different theories

(B)

analyzing different techniques

(C)

defending two hypotheses

(A)

tends to be visible for no more than 200 meters

(B)

stays near either the front or the rear of a school

(D)

refuting established beliefs

(C)

is part of a small school rather than a large
school

(E)

revealing new evidence

(D)

is very similar in appearance to the other fish in
the school

Main idea

(E)

is medium‐sized

Inference
The “confusion effect” is discussed in the third
and fourth paragraphs. The first explanation
of the “confusion effect” proposes that because
predators prefer to select distinctive prey, they
find it difficult to select one fish from among
many that look the same.
A

B

C
D

E

The 200‐meter visibility of fish is part of the
explanation for theory A, not theory B (the
“confusion effect”).
The location of an individual fish within
a school is not discussed in the passage as
being important to the “confusion effect.”
The size of a school of fish is not discussed
as an element of the “confusion effect.”
Correct. Because predators, according to
the “confusion effect,” prefer to select prey
that is distinct from the rest of the school,
a fish that is similar in appearance to the
other fish in its school would most likely
enjoy a survival advantage.
The size of a fish relative to the other fish in
its school would most likely contribute to
its ability to survive: that is, if it resembled
other fish in size, it would be safer, based on
what the passage says about the “confusion
effect.” Furthermore, the passage gives no
reason to think that merely being medium‐
sized would confer any advantage (unless
the other fish were medium‐sized as well).

Determining the author’s primary concern
depends on understanding the focus of the
passage as a whole. The author presents two
theories that purport to account for why fish,
particularly small fish, tend to school and explains
the arguments of proponents of each theory.
A

B
C

D
E

Correct. The author discusses two
theories—identified as theory A and
theory B—that account for the tendency of
fish to school.
The author is not concerned with different
techniques in the passage.
The two theories of why fish school could
be referred to as hypotheses, but the author
is not primarily concerned with defending
them; rather, the passage explains how each
attempts to account for the phenomenon in
question.
The author presents, rather than refutes,
beliefs about why fish tend to school.
The author reveals no evidence, new or
otherwise, in the passage. The passage is
a general discussion of scientific opinions
based on existing evidence.

The correct answer is A.

The correct answer is D.

425

GMAT® Official Guide 2018

Questions 409–411 refer to the passage on page 364.

E

409. The primary purpose of the passage is to
(A)

challenge recent findings that appear to
contradict earlier findings

(B)

present two sides of an ongoing scientific debate

(C)

report answers to several questions that have
long puzzled researchers

(D)

discuss evidence that has caused a long‐
standing belief to be revised

(A)

activity level

(B)

blood pressure

(E)

attempt to explain a commonly misunderstood
biological phenomenon

(C)

alertness

(D)

vision

(E)

temperature

Main idea

The correct answer is D.
410. The passage mentions each of the following as a
function regulated by the SCNs in some animals
EXCEPT

This question depends on understanding the
passage as a whole. The passage begins by
describing a long‐held belief regarding humans’
circadian rhythms: that the SCNs control them.
It then goes on to explain that new findings have
led scientists to believe that other organs and
tissues may be involved in regulating the body’s
circadian rhythms as well.

This question asks about what is NOT specifically
mentioned in the passage with regard to
functions regulated by the SCN. Those functions,
as identified in the passage, are blood pressure,
body temperature, activity level, alertness, and the
release of melatonin.

A

A

B

C

D

426

The biological phenomenon of circadian
rhythms is not, at least as far as the passage
is concerned, misunderstood. Its causes are
being investigated and refined.

The passage does not challenge the more‐
recent findings. Furthermore, the recent
findings that the passage recounts do not
contradict earlier findings; rather, when
placed alongside those earlier findings,
they have led scientists to reach additional
conclusions.
The passage does not discuss a two‐sided
debate; no findings or conclusions are
disputed by any figures in the passages.
There is only one question at issue in the
passage: whether the SCN alone control
human circadian rhythms. Furthermore,
nothing in the passage suggests that
researchers have been puzzled for a long
time about this.
Correct. The new evidence regarding
circadian rhythm–related gene activity in all
the body’s tissue has led scientists to revise
their long‐standing belief that the SCN
alone control circadian rhythms.

Supporting idea

B
C
D

E

The passage includes activity level in its list
of functions regulated by the SCN.
The passage includes blood pressure in its
list of functions regulated by the SCN.
The passage includes alertness in its list of
functions regulated by the SCN.
Correct. While the passage does say
that cells in the human retina transmit
information to the SCN, there is no
suggestion that the SCN reciprocally
control vision.
The passage includes temperature in its list
of functions regulated by the SCN.

The correct answer is D.

7.6 Reading Comprehension Answer Explanations

411. The author of the passage would probably agree with
which of the following statements about the SCNs?
(A)

The SCNs are found in other organs and tissues
of the body besides the hypothalamus.

(B)

The SCNs play a critical but not exclusive role in
regulating circadian rhythms.

(C)
(D)

(E)

The SCNs control clock genes in a number of
tissues and organs throughout the body.

question the results of a study that examined
the effect of service-quality guarantees in the
restaurant industry

(B)

The SCNs are a less significant factor in
regulating blood pressure than scientists once
believed.

discuss potential advantages and disadvantages
of service-quality guarantees in the restaurant
industry

(C)

The SCNs are less strongly affected by changes
in light levels than they are by other external
cues.

examine the conventional wisdom regarding
the effect of service-quality guarantees in the
restaurant industry

(D)

argue that only certain restaurants would benefit
from the implementation of service-quality
guarantees

(E)

consider the impact that service-quality
guarantees can have on the service provided by
a restaurant

The author of the passage discusses the SCN in
the passage in order to explain that they are most
likely not, as long believed, solely responsible for
the control of our circadian rhythms.

B

C

D

E

412. The primary purpose of the passage is to
(A)

Main idea

A

Questions 412–414 refer to the passage on page 366.

The author states that the SCN are nerve
clusters in the hypothalamus, and nothing
in the passage contradicts or undermines
the supposition that they are only in the
hypothalamus.
Correct. The author points out in the
second paragraph that the SCN control core
circadian function, but that circadian clocks
found elsewhere in the body have an effect
as well.
The evidence offered in the second
paragraph about the activity of the clock
gene in rat livers suggests that these clock
genes are not under the SCN’s control. The
passage does not suggest that the SCN
control any of the non‐SCN controllers of
circadian rhythms.
The author states in the second paragraph
that scientists do not dispute the idea that
the SCN regulate blood pressure.
The first paragraph indicates that the SCN
respond to light levels; clock genes in other
tissues are the ones that may respond to
other external cues.

The correct answer is B.

Main idea
This question depends on understanding the
passage as a whole. The first paragraph describes
Tucci and Talaga’s findings regarding the effect
of service-quality guarantees: that they have
different, more positive results for higherpriced restaurants than for lower-priced ones,
which could be affected negatively. The second
paragraph explains that a particular benefit from
service guarantees could accrue to restaurants
generally.
A

B

The passage does not question the results of
Tucci and Talaga’s study; rather, the passage
appears to accept the results of the study as
accurate.
Correct. The potential advantages involve
the management and motivation of
service staff, as well as, for higher-priced
restaurants, a greater likelihood of being
selected by customers over other restaurants.
Potential disadvantages for lower-priced
restaurants include the possibility that
potential customers may believe that such
restaurants are concerned about the quality
of their service.

427

GMAT® Official Guide 2018

C

D

E

The passage does not indicate whether
there is any conventional wisdom regarding
service-quality guarantees in the restaurant
industry.
The second paragraph of the passage
suggests that restaurants in general could
potentially enjoy some benefits from
the implementation of service-quality
guarantees. For lower-priced restaurants,
these benefits could offset the possible
negative effects of service-quality guarantees
described in the first paragraph.
The second paragraph of the passage
indicates an effect that service-quality
guarantees could have on a restaurant’s staff
and the service that the staff provides, but
this is only one of the subsidiary points
contributing to the focus of the passage as
a whole. The first is more concerned with
the question of what effect these guarantees
would have on whether customers choose to
patronize that restaurant.

Inference
This question asks for an inference from the
passage about the author’s view of why and how
service guarantees would appeal to customers.
The question does not ask specifically about
service guarantees in the context of restaurants,
but rather service guarantees in general. The
end of the first paragraph addresses this general
question: a service guarantee may appeal most to
customers in the case of activities whose quality
they are less likely to know how to question.
A

B

C

The correct answer is B.
413. It can be inferred that the author of the passage would
agree with which of the following statements about the
appeal of service guarantees to customers?
(A)

Such guarantees are likely to be somewhat more
appealing to customers of restaurants than to
customers of other businesses.

(B)

Such guarantees are likely to be more appealing
to customers who know what to anticipate in
terms of service.

(C)

Such guarantees are likely to have less appeal in
situations where customers are knowledgeable
about a business’s product or service.

(D)

In situations where a high level of financial
commitment is involved, a service guarantee is
not likely to be very appealing.

(E)

In situations where customers expect a high
level of customer service, a service guarantee is
likely to make customers think that a business is
worried about its service.

D

E

The author states that a service guarantee
might have greater appeal in the case of
skilled activities than it would for restaurant
customers.
According to the author, customers who
know what to expect in terms of service—a
group that includes restaurant customers—
would likely find service guarantees less
appealing.
Correct. The author makes clear that service
guarantees would be less appealing to
restaurant customers when they know what
to expect in terms of the quality of service.
The passage provides some evidence that
where a high level of financial commitment
is involved, a service guarantee may be
more rather than less appealing than in
other situations. In discussing higher-priced
restaurants, which require a relatively high
level of financial commitment, the author
states that Tucci and Talaga found evidence
that a service guarantee would likely appeal
to customers.
The author implies that customers of
higher-priced restaurants expect a high
level of service, certainly a level higher
than that expected by customers of lowerpriced restaurants. But it is at lower-priced
restaurants that Tucci and Talaga found that
a service guarantee makes customers think
a given restaurant is concerned about its
service.

The correct answer is C.

428

7.6 Reading Comprehension Answer Explanations

414. According to the passage, Tucci and Talaga found
that service guarantees, when offered by lower-priced
restaurants, can have which of the following effects?
(A)

Customers’ developing unreasonably high
expectations regarding service

(B)

Customers’ avoiding such restaurants because
they fear that the service guarantee may not be
fully honored

(C)

Customers’ interpreting the service guarantee as
a sign that management is not confident about
the quality of its service

(D)

A restaurant’s becoming concerned that its service
will not be assiduous enough to satisfy customers

(E)

A restaurant’s becoming concerned that
customers will be more emboldened to question
the quality of the service they receive

Supporting ideas
This question requires identifying Tucci and
Talaga’s findings regarding service guarantees
offered by lower-priced restaurants. The passage
states directly that these researchers found in
these situations that a guarantee could lead
potential customers to think that the restaurant
has concerns about its service.
A

B

C

D

The passage does not report that Tucci and
Talaga found that service guarantees create
unreasonably high expectations regarding
service.
The passage does not report that Tucci and
Talaga found that customers doubted that
service guarantees would be honored.
Correct. The passage explicitly indicates
that Tucci and Talaga found that potential
customers of lower-priced restaurants could
interpret service guarantees as indicating
worries about the quality of service.
The passage indicates that Tucci and Talaga
found that customers might think that
lower-priced restaurants are offering service
guarantees because they are concerned that
the quality of their service is too low, but
the passage does not indicate that service
guarantees lead such restaurants to have
concerns about the quality of their service,
and in fact it may be that such guarantees
could lead to improvements in service.

E

The passage indicates that service guarantees
offered at lower-priced restaurants may
empower customers to question the quality
of service, but it does not indicate that
service guarantees lead restaurants to have
concerns about this.

The correct answer is C.
Questions 415–418 refer to the passage on page 368.

415. Information in the passage suggests that David Pearce
would most readily endorse which of the following
statements concerning monetization?
(A)

Monetization represents a strategy that is
attractive to both environmentalists and their
critics.

(B)

Monetization is an untested strategy,
but it is increasingly being embraced by
environmentalists.

(C)

Monetization should at present be restricted to
ecological services and should only gradually
be extended to such commercial endeavors as
tourism and recreation.

(D)

Monetization can serve as a means of
representing persuasively the value of
environmental conservation.

(E)

Monetization should inform environmental
decision-making processes only if it is accepted
by environmentalist groups.

Inference
This question requires an understanding of David
Pearce’s view of monetization. According to the
passage, Pearce finds the idea that conservation
is unprofitable to be an illusion. He argues for
showing the economic value of ecosystems in
order to make progress in conserving those
ecosystems.
A

The passage attributes to Gretchen Daily
the view that monetization is unpopular
with environmentalists. The passage gives no
reason to believe that Pearce would endorse
the idea that environmentalists currently
find monetization attractive.

429

GMAT® Official Guide 2018

B

C

D

E

The passage gives no indication that
monetization is increasingly being embraced
by environmentalists, even if Pearce thinks it
should be.
The passage indicates Pearce’s belief that
some types of tourism are also types of
ecological services that have economic value
and that they should be monetized.
Correct. Pearce believes that monetization
quantifies the value of the services provided
by ecological systems—and if that value
is quantified, people are more likely to be
persuaded to conserve those systems.
Pearce is arguing, against some
environmentalists, that monetization should
inform the decision-making process with
regard to preserving ecosystems.

The correct answer is D.

B

C

D

E

416. Which of the following most clearly represents an
example of an “ecological service” as that term is used
in line 20 ?
(A)

A resort hotel located in an area noted for its
natural beauty

(B)

A water-purifying plant that supplements natural
processes with nontoxic chemicals

(C)

A wildlife preserve that draws many international
travelers

(A)

(D)

A nonprofit firm that specializes in restoring
previously damaged ecosystems

The centrality of economic interests to people’s
actions

(B)

(E)

A newsletter that keeps readers informed of
ecological victories and setbacks

The reluctance of the critics of environmentalism
to acknowledge the importance of conservation

(C)

The inability of financial interests and ecological
interests to reach a common ideological ground

(D)

The inevitability of environmental degradation

(E)

The inevitability of the growth of ecological
services in the future

Application
Based on the passage, ecological services are
services provided by natural assets that have not
been converted into commercial assets. Thus any
example of such an ecological service requires
that the area providing it is natural.
A

430

Water purifying is an ecological service if
it is supplied by undisturbed forests and
wetlands. The word plant here must mean
a technological installation, not a botanical
organism, because it is said to supplement—
not to be part of—the natural processes.
Thus it is not a natural asset and therefore
does not provide an ecological service as
described in the passage.
Correct. The passage states that a wildlife
preserve that creates jobs and generates
income would be providing an ecological
service.
A nonprofit firm that restores damaged
ecosystems would be performing a valuable
ecology-related service, but it would not
itself be an example of a natural asset
providing an ecological service.
Environmentalists and others would most
likely find such a newsletter informative,
but it would not be an ecological service,
because it is not a service provided by a
natural asset.

The passage mentions resort hotels as an
example of explicitly commercial assets.
Although some hotels might be situated in
ecologically valuable natural environments,
any ecological services in such cases would
be contributed by the natural environments,
not by the hotels themselves.

The correct answer is C.
417. According to the passage, Daily sees monetization as
an indication of which of the following?

Supporting ideas
This question asks about Daily’s view of
monetization, and according to the passage, she
sees monetization as a practice that reflects the
dominant role that economic decisions play in human
behavior.

7.6 Reading Comprehension Answer Explanations

A

B

C

D

E

Correct. According to the passage, Daily
believes that economic interests are central
to people’s actions, and monetization of
ecological services would take that central
role realistically into account.
Monetization, as Daily sees it, is a way of
assigning value to conservation and thus
acknowledging its importance. Many
environmentalists, rather than their critics,
are reluctant to embrace monetization,
according to Daily.
For Daily, monetization represents a way for
financial interests and ecological interests
to reach a common ground; by using this
common currency, both sides can make good
decisions about the environment.
Monetization, on Daily’s view, would help
to prevent environmental degradation; the
passage does not suggest that she regards
such degradation as at all inevitable.
Daily does not see monetization as
inevitably spurring the growth of ecological
services but as more likely preventing
their decline by leaving those services
undisturbed.

The correct answer is A.
418. Which of the following can be inferred from the
passage concerning the environmentalists mentioned
in line 8 ?
(A)

They are organized in opposition to the
generation of income produced by the sale of
ecological services.

(B)

They are fewer in number but better organized
and better connected to the media than their
opponents.

(C)

They have sometimes been charged with failing
to use a particular strategy in their pursuit of
conservational goals.

(D)

They have been in the forefront of publicizing the
extent of worldwide environmental degradation.

(E)

They define environmental progress differently
and more conservatively than do other organized
groups of environmentalists.

Inference
The sentence in question states that critics blame
environmentalists for their failure to address the
economic issues of environmental degradation.
A

The passage states that in the absence of
monetization, conservation can appear
unprofitable. But this does not mean that
the environmentalists in question are
opposed to conservation generating income.
B
The passage does not address the issue
of the number of environmentalists in
question, the number of those opposed
to them, or whether either group is better
connected to the media.
C Correct. The passage indicates that critics
of the environmentalists in question believe
environmentalists are to blame for not using
an effective economics-based strategy to
promote conservation.
D Although it may be the case that the
environmentalists in question have been
prominent in publicizing worldwide
environmental degradation, the passage does
not provide grounds for inferring that they
have been.
E The passage suggests that certain critics
consider environmentalists in general to
be at fault for failing to address economic
issues. In this respect, the passage
makes no distinctions among different
environmentalist groups, organized or
otherwise.
The correct answer is C.

431

GMAT® Official Guide 2018

Questions 419–422 refer to the passage on page 370.

D

419. According to the passage, the studies referred to in
line 12 reported which of the following about the effect
of price on consumers’ perception of the performance
risk associated with a new product?

E

(A)

(B)

(C)

Although most consumers regard price as
an important factor, their perception of the
performance risk associated with a new product
is ultimately determined by the manufacturer’s
reputation.
Price interacts with the presentation of an
advertised message to affect perceived
performance risk.
Price does not significantly affect consumers’
perception of the performance risk associated
with a new product.

(D)

Consumers tend to regard price as more
important than the manufacturer’s credibility
when they are buying from that manufacturer for
the first time.

(E)

Consumers are generally less concerned about
a new product’s performance risk when that
product is relatively expensive.

Supporting ideas
The question asks about information explicitly
provided in the passage. The first paragraph
explains that there are conflicting findings in the
research about how the price of a product affects
a consumer’s perception of the performance risk
of that product. Some studies have found that
higher priced products reduce the perception
of performance risk. The other studies referred
to in line 12, however, have found little or
no connection between price and perceived
performance risk.
A

B
C

432

The passage does not mention that these
studies consider the manufacturer’s
reputation.
The passage does not mention that these
studies consider advertising messages.
Correct. The passage indicates that these
studies have found little or no connection
between relative price and consumers’
perception of performance risk.

The passage does not mention that these
studies consider the manufacturer’s
credibility.
Although some studies have found that a
relatively high price reduces the perception
of performance risk, the passage explains
that the studies referred to in line 12 have
not confirmed that finding.

The correct answer is C.
420. The “past research” mentioned in line 25 suggests
which of the following about perceived performance
risk?
(A)

The more expensive a new product is, the more
likely consumers may be to credit advertised
claims about that product.

(B)

The more familiar consumers are with a
particular manufacturer, the more willing they
may be to assume some risk in the purchase
of a new product being advertised by that
manufacturer.

(C)

Consumers’ perception of the performance risk
associated with a new product being advertised
may be influenced by an interplay between the
product’s price and the manufacturer’s credibility.

(D)

Consumers may be more likely to believe that
a product will function as it is advertised to
do when they have bought products from a
particular manufacturer before.

(E)

The price of a particular advertised product
may have less impact than the manufacturer’s
credibility on consumers’ assessment of the
performance risk associated with that product.

Supporting ideas
The question asks about information explicitly
provided in the passage. The second paragraph
explains that, according to some research,
consumers perceive a product as having less
performance risk when they trust the source
of advertising about that product. Past research,
however, suggests that performance risk is
affected not merely by the credibility of the
source, but by an interaction between source
credibility and the price of the product.

7.6 Reading Comprehension Answer Explanations

A

B

C

D

E

The passage does not indicate that the past
research addressed the question of how
the price of a product affects consumers’
perception of advertised claims. It only
says that the research suggests that the two
factors interact.
Although the passage discusses consumers’
perception of how risky a purchase might
be, it does not address the relationship
between familiarity and willingness to
assume risk.
Correct. The past research suggests
that performance risk is affected by an
interaction between the price of the product
and the credibility of the source of the
advertising about the product—in other
words, the manufacturer.
The past research suggests that consumers’
beliefs about a product’s performance are
affected not merely by their perception
of the manufacturer, but by an interplay
between source credibility and product price.
The passage does not mention any possible
role of prior experience in this interplay.
The passage does not discuss whether
price or the manufacturer’s credibility
has more of an effect on perceived
performance risk.

The correct answer is C.
421. The passage is primarily concerned with
(A)

challenging the implications of previous research
into why consumers try new products

(B)

suggesting new marketing strategies for
attracting consumers to new products

(C)

reconciling two different views about the effect
of price on consumers’ willingness to try new
products

(D)

describing a new approach to researching why
consumers try new products

(E)

discussing certain findings regarding why
consumers try new products

Main idea
The question depends on understanding the
passage as a whole. The passage begins with
a statement explaining that much research
has investigated what motivates consumers
to try new products. It then defines one such
motivating factor—perception of performance
risk. The remainder of the passage summarizes
research into how price and a manufacturer’s
advertising affect consumers’ perception of
performance risk.
A

B
C

D
E

The passage summarizes research findings
that conflict with one another but does not
support some findings over others.
The passage does not suggest any new
marketing strategies.
The first paragraph mentions a study that
could reconcile two conflicting findings, but
this is only a supporting point in the passage’s
larger purpose of summarizing research.
The passage does not describe new research
approaches.
Correct. The passage discusses studies about
performance risk, which is a factor that
motivates consumers to try new products.

The correct answer is E.
422. Which of the following, if true, would most tend to
weaken the conclusions drawn from “some of this
research” (see line 8)?
(A)

In a subsequent study, consumers who were
asked to evaluate new products with relatively
low prices had the same perception of the
products’ performance risk as did consumers
who were shown the same products priced more
expensively.

(B)

In a subsequent study, the quality of the
advertising for the products that consumers
perceived as having a lower performance risk
was relatively high, while the quality of the
advertising for the products that consumers
perceived as having a higher performance risk
was relatively poor.

(C)

In a subsequent study, the products that
consumers perceived as having a lower
performance risk were priced higher than the
highest priced products in the previous research.
433

GMAT® Official Guide 2018

(D)

(E)

None of the consumers involved in this research
had ever before bought products from the
manufacturers involved in the research.
Researchers found that the higher the source
credibility for a product, the more consumers
were willing to pay for it.

Evaluation
The question depends on evaluating the
reasoning behind the conclusions of some
research and deciding which evidence would
weaken them. The research concludes that
higher prices reduce consumers’ perception of
performance risk associated with a particular
product. This conclusion involves a claim of
cause and effect, so evidence showing that
higher prices do not cause that effect would
weaken the argument.
A

B

C

D

E

Correct. If lowering prices has no effect on
consumers’ perception of performance risk,
the conclusions of the research are called
into question.
A correlation between quality of advertising
and perceived performance risk is not
clearly relevant to the research conclusions
about the effects of price.
This answer choice provides no basis for
comparison among prices within the
subsequent study. For all we can tell, the
prices that correlated with higher perceived
performance risk in the subsequent study
may have been lower than those that
correlated with lower perceived risk. In that
case, the subsequent study would tend to
strengthen, not weaken, the conclusions
drawn from the earlier research.
Consumers’ lack of familiarity with other
products from the manufacturers is not
clearly relevant to the studies’ conclusions
about the effects of price.
Credibility of the source of advertisements
is discussed as a separate issue in the second
paragraph and is not clearly relevant to
these studies’ conclusions about the effects
of price. To the extent that it may be
obliquely relevant, it tends to strengthen,
rather than to weaken, the conclusions.

The correct answer is A.

Questions 423–427 refer to the passage on page 372.
423. The primary purpose of the passage is to
(A)

compare the economic role played by southern
banks with the economic role played by banks
in the rest of the United States during the late
eighteenth and early nineteenth centuries

(B)

reevaluate a conventional interpretation of the role
played by banks in the American economy during
the late eighteenth and early nineteenth centuries

(C)

present different interpretations of the role
played by banks in the American economy during
the late eighteenth and early nineteenth centuries

(D)

analyze how the increasing number of banks in
the late eighteenth and early nineteenth centuries
affected the American economy

(E)

examine how scholarly opinion regarding the role
played by banks in the American economy during
the late eighteenth and early nineteenth centuries
has changed over time

Main idea
The question depends on understanding the passage
as a whole. The passage describes two contrasting
views about the role banks played in the economic
growth of the United States around the turn of the
nineteenth century. The first paragraph describes the
view that banks played only a small role. The second
paragraph describes the contrasting view that banks
played a critical role.
A

B

C

D

E

The mention of banks in the South is a
small part of a larger discussion about the
role of banks in the country as a whole.
The passage describes two major views held
by historians; it does not reevaluate either of
those views.
Correct. The passage describes two different
views about the role that banks played in
America’s growing economy.
The passage does not analyze any aspect
of the relationship between the increasing
number of banks and the economy. It alludes
to the increase in numbers only within a
broader description of two contrasting views
about how banks affected the economy.
The passage suggests that at the time when
it was written, the two views it describes
were both still held among historians.

The correct answer is C.
434

7.6 Reading Comprehension Answer Explanations

424. The passage suggests that the scholars mentioned in
line 4 would argue that the reason banks tended not to
fund manufacturing and transportation projects in the
late eighteenth and early nineteenth centuries was that
(A)

these projects, being well established and well
capitalized, did not need substantial long‐term
financing from banks

425. The passage suggests that Paul Gilje would be most
likely to agree with which of the following claims about
the lending practices of the “earliest banks” (see
line 21) ?
(A)

These lending practices were unlikely to
generate substantial profits for banks.

(B)

These lending practices only benefited a narrow
sector of the economy.

(B)

these projects entailed a level of risk that was too
great for banks’ conservative lending practices

(C)

(C)

banks preferred to invest in other, more
speculative projects that offered the potential for
higher returns

The restrictive nature of these lending practices
generated significant opposition outside of the
South.

(D)

(D)

bank managers believed that these projects
would be unlikely to contribute significantly to
economic growth in the new country

The restrictive nature of these lending
practices forced state legislatures to begin
granting more bank charters by the early
nineteenth century.

(E)

bank managers believed funding these projects
would result in credit being extended to too
many borrowers

(E)

These lending practices were likely to be
criticized by economic elites as being overly
restrictive.

Inference

Inference

The question asks about information implied by the
passage. According to the scholars’ view described
in the first paragraph, banks followed conservative
lending practices: they shunned projects that were
uncertain and that required substantial investments
in capital. It follows that, according to those
scholars, the reason banks chose not to fund certain
projects was that they entailed too great a risk.

This question asks about conclusions that can
be logically inferred from information provided
in the passage. According to the second
paragraph, Paul Gilje believes that a driving
force in American economic growth in the
early nineteenth century was banks’ lending to a
larger and more diverse group of borrowers. The
question asks what this would imply about Gilje’s
view toward earlier banks—which, the passage
explains, offered credit only to well‐connected
merchants.

A

B

C

D

E

The passage indicates that manufacturing
and transportation projects were less well
established than those the banks preferred
to fund, not more so.
Correct. Because the projects were
uncertain and required a great deal of capital,
banks considered them too risky.
The passage indicates that banks followed
conservative lending practices and avoided
investments that were uncertain.
The passage does not mention banks’ beliefs
about economic growth, and it does not
provide any basis for inferring that the
scholars in question held any particular
views regarding such beliefs.
The passage does not mention, or provide a
basis for inferences about, bank managers’
concerns about numbers of borrowers.

The correct answer is B.

A
B

C
D

The profitability of banks’ lending practices
is not at issue in the discussion.
Correct. The passage says that the earliest
banks had primarily made loans only
to a narrow sector—well‐connected
merchants—and that they began lending
more broadly in the early nineteenth
century. It then cites Gilje’s view to
corroborate and explicate this claim. This
strongly suggests that Gilje agrees with the
claim.
Opposition to the earliest banks is not
mentioned or alluded to in the discussion.
The passage provides no basis for inferring
that Gilje held any particular view as to
why legislatures began granting more bank
charters.
435

GMAT® Official Guide 2018

E

The passage does not mention or provide a
basis for inference about the views of economic
elites regarding the lending practices of the
earliest banks in the United States. It thus
provides no basis for inferring that Gilje would
have any particular opinion on this topic.

The correct answer is B.
426. The passage suggests that the opposition to banks in
the South in the early nineteenth century stemmed in
part from the perception that banks
(A)

did not benefit more than a small minority of the
people

(B)

did not support the interests of elite planters

(C)

were too closely tied to transportation and
industrial interests

(D)

were unwilling to issue the long‐term loans
required by agrarian interests

(E)

were too willing to lend credit widely

Inference
The question asks about statements that can
be inferred from information provided in the
passage. The second paragraph explains that people
who opposed banks in the South saw them as
monopolies controlled by elite planters. This would
imply that those who opposed banks believed that
most people in the South did not benefit from them.
A

B
C

D

E

Correct. Since people opposed the banks
on the grounds that they were monopolies
controlled by an elite group of planters, they
likely thought banks did not benefit most of
the population.
The passage implies that people believed the
banks did serve the interests of elite planters.
The passage indicates that people believed
banks in the South were tied to planters, not
to transportation and industrial interests.
Southern banks’ willingness to provide long‐
term loans is not discussed or alluded to in
the passage.
The passage does not imply that anyone
believed banks in the South were willing to
lend credit widely. Since people believed the
banks were controlled by the elite, they more
likely thought banks were unwilling to lend
credit widely.

The correct answer is A.
436

427. Which of the following statements best describes the
function of the last sentence of the passage?
(A)

It provides evidence tending to undermine the
viewpoint of the scholars mentioned in line 5.

(B)

It resolves a conflict over the role of banks
summarized in the first paragraph.

(C)

It clarifies some of the reasons state legislatures
began granting more bank charters.

(D)

It qualifies a claim made earlier in the passage
about the impact of banks on the American
economy in the early nineteenth century.

(E)

It supports a claim made earlier in the passage
about how the expansion of credit affected the
economy.

Evaluation
This question asks about the function of the last
sentence in relation to the rest of the passage.
The first paragraph describes the view of one set
of historians. The second paragraph describes the
contrasting view of a second set of historians.
The last sentence of the passage points out
an exception mentioned by the second set of
historians.
A

B
C

D

E

The last sentence pertains to the view
of historians described in the second
paragraph, not those described in the first
paragraph.
The conflict between the two differing views
is not resolved by the passage.
The passage does not explain why
legislatures began granting more bank
charters.
Correct. The second set of historians claim
banks spurred American economic growth
at the turn of the nineteenth century, but
the last sentence adds an exception to that
claim.
The last sentence does not support the claim
made by the second set of historians, but
rather serves as an exception to that claim.

The correct answer is D.

7.6 Reading Comprehension Answer Explanations

Questions 428–430 refer to the passage on page 374.

E

428. According to the passage, the difference in the amount
of cratering on Callisto’s and Io’s respective surfaces
can probably be explained by the difference between
these two moons with respect to which of the following
factors?
(A)

Size

(B)

Ice content

(C)

The rate of bombardment by comets and
asteroids

(D)

The influence of Jupiter’s other moons

(E)

The level of geological activity

Supporting idea
Cratering is discussed in the second paragraph.
The passage states that Callisto is heavily cratered,
while Io has no detectable craters. Io is the moon
closest to Jupiter, and Callisto is the farthest
away. Their relative positions are accompanied by
a corresponding difference in geological activity:
Io is very geologically active, while Callisto is not
active at all. Io’s geological activity means that it
is being regularly resurfaced, so it is unlikely to
retain any evidence of cratering—unlike Callisto,
which experiences no resurfacing, leaving its
craters intact.
A

B

C

D

While the passage makes clear that Callisto
is larger than Io, it does not address whether
their relative size explains the difference in
their respective amounts of cratering.
According to the passage, Callisto and
Io differ in terms of their ice content,
but nothing in the passage indicates that
that content affects the cratering on their
surfaces.
The passage states that Io experiences a
higher rate of bombardment than Callisto
does, but while that bombardment most
likely causes cratering on Io, its surface does
not retain those craters. Thus, the rate of
bombardment does not, in itself, explain the
difference in cratering on the surface of the
two moons.
The only other moons of Jupiter discussed
in the passage are Ganymede and Europa,
and the passage does not consider their
effect on the cratering of Callisto and Io.

Correct. Because Io experiences a high
degree of geological activity, its surface is
continuously resurfaced, which means that
the surface shows no craters. Callisto, on
the other hand, is not geologically active
and thus is not resurfaced regularly, which
explains why its surface is heavily cratered.

The correct answer is E.
429. Which of the following best describes the purpose of
the second paragraph of the passage?
(A)

To provide further evidence of the systematic
variation in the characteristics of Jupiter’s four
largest moons

(B)

To present a comprehensive theory to explain
the systematic variation in the characteristics of
Jupiter’s four largest moons

(C)

To explain the significance of the systematic
variation in the characteristics of Jupiter’s four
largest moons

(D)

To introduce facts that contradict conventional
assumptions about Jupiter’s four largest moons

(E)

To contrast the characteristics of Jupiter’s four
largest moons with the characteristics of the
planets of the solar system

Evaluation
This question depends on understanding how the
second paragraph functions in the context of the
passage as a whole. The first paragraph discusses
the way in which the composition of Jupiter’s four
largest moons varies with distance from Jupiter,
and the second paragraph extends the idea of
distance‐based variation to geological activity and
surface appearance.
A

B

Correct. The second paragraph presents
evidence related to the amount of geological
activity and surface cratering in order to
extend the first paragraph’s suggestion that
the characteristics of Jupiter’s four largest
moons vary systematically based on their
distance from Jupiter.
The passage does not offer a theory to
account for the systematic variation in the
characteristics of Jupiter’s largest moons; it
merely describes several instances of that
variation.
437

GMAT® Official Guide 2018

C

D

E

The second paragraph provides more
examples of the systematic variation in the
characteristics of Jupiter’s largest moons
rather than explaining that variation’s
significance.
The passage describes Jupiter’s moons
but does not identify any conventional
assumptions about those moons that are
contradicted by facts introduced in the
passage.
The first paragraph suggests that Jupiter’s
moons exhibit differences in characteristics
that are more “systematic” than are those
of the planets in the solar system, but
the characteristics are not themselves
contrasted; in fact, the first paragraph
suggests that they are similar. The second
paragraph does not discuss the planets of
the solar system at all.

A

B

C

D

E

The correct answer is A.
430. The author’s reference to Jupiter’s gravity in line 25
serves primarily to
(A)

indicate why the absence of craters on Io’s
surface is surprising

(B)

explain the presence of craters on the surface of
Jupiter’s four largest moons

Questions 431–434 refer to the passage on page 376.

(C)

provide an explanation for the lack of geological
activity on Callisto

(D)

contrast Jupiter’s characteristics with the
characteristics of its four largest moons

431. According to the passage, the research mentioned
in line 6 suggests which of the following about lower‐
ranked top executives and postacquisition success?

(E)

illustrate the similarity between Jupiter’s four
largest moons and the planets of the solar
system

The correct answer is A.

(A)

Given that these executives are unlikely to
contribute to postacquisition success, little effort
should be spent trying to retain them.

(B)

The shorter their length of service, the less likely
it is that these executives will play a significant
role in postacquisition success.

(C)

These executives are less important to
postacquisition success than are more highly
ranked top executives.

(D)

If they have long tenures, these executives may
prove to be as important to postacquisition
success as are more highly ranked top
executives.

(E)

Postacquisition success is unlikely if these
executives are retained.

Evaluation
The reference to Jupiter’s gravity is part of the
author’s discussion of cratering on Jupiter’s
moons; Jupiter’s gravity is strong enough to
attract comets and asteroids that then bombard
its inner moons. A high bombardment rate
would seem to indicate that a great deal of
cratering would occur on those inner moons,
and yet the passage reports that, unexpectedly,
on the innermost moon, Io, no craters have been
detected.

438

Correct. Jupiter’s gravity attracts comets
and asteroids, which increases the
bombardment rate of its inner moons,
including Io. This bombardment makes
it surprising that Io’s surface shows no
cratering.
The passage discusses the likely effect of
Jupiter’s gravity on its inner moons but not
its outer moons; two of the large moons the
passage discusses are outer moons.
According to the passage, Callisto, an outer
moon, lacks geological activity because of
its distance from Jupiter; Jupiter’s gravity is
not offered as a contributing factor to this
inactivity.
The passage does not contrast Jupiter with
its moons; rather, it compares the moons to
one another.
The first paragraph of the passage suggests
that Jupiter’s moons vary in a way similar
to that of the planets of the solar system,
but the author does not refer to Jupiter’s
gravity to illustrate the similarity in this
variation.

7.6 Reading Comprehension Answer Explanations

Supporting idea

Evaluation

The question asks about information provided
by the passage. According to the third sentence,
research suggests that retaining the highest‐
level top executives in an acquisition is more
strongly associated with success than retaining
lower‐ranked top executives—which suggests,
in turn, that lower‐ranked top executives are
less important than top‐level executives to
postacquisition success, though it does not
suggest that they are unimportant to such success.

This question requires analysis of the reasoning
underlying one of the two explanations described
in the passage. The resource‐based view (RBV)
holds that retaining high‐level executives with
long tenure will contribute to success because
those people have important company‐specific
knowledge. This view rests on the assumption
that company‐specific knowledge is valuable to
postacquisition success.

A

B
C

D
E

The research indicates that lower‐ranked top
executives are less strongly associated with
success than are higher‐ranked executives
but does not provide advice about retention
efforts.
The research mentioned in the third
sentence does not consider length of service.
Correct. The research indicates that lower‐
ranked top executives are less strongly
associated with postacquisition success than
are the highest‐ranked executives.
The research mentioned in the third
sentence does not consider length of service.
The research suggests that lower‐ranked top
executives are less strongly associated with
postacquisition success but does not suggest
that they decrease the likelihood of success.

A

B

C

D

The correct answer is C.
432. The resource‐based view, as described in the passage,
is based on which of the following ideas?
(A)

The managerial skills of top executives become
strongest after the first five years of their tenure.

(B)

Company‐specific knowledge is an important
factor in the success of an acquisition process.

(C)

The amount of nontransferable knowledge
possessed by long‐tenured top executives tends
to be underestimated.

(D)

Effective implementation of an acquisition
depends primarily on the ability of executives to
adapt to change.

(E)

Short‐tenured executives are likely to impede
the implementation of a successful acquisition
strategy.

E

In RBV, executives with long tenure are
valuable not specifically for their managerial
skills but for their knowledge about the
acquired company. The passage does not
restrict to five years the period in which this
knowledge is gained.
Correct. RBV values executives’ knowledge
of the acquired company and is based on the
belief that company‐specific knowledge is
valuable for postacquisition success.
The passage does not indicate that RBV
claims that executives’ company‐specific
knowledge is generally undervalued. But the
passage does indicate that RBV regards such
knowledge as valuable to postacquisition
success.
In RBV, executives with long tenure are
valuable not for their ability to adapt to
change, but for their knowledge about the
acquired company.
RBV does not suggest that short‐tenured
executives impede postacquisition success,
only that they are less important to success
than the highest‐ranked executives.

The correct answer is B.
433. The passage suggests that Bergh and a proponent of
the upper echelons perspective would be most likely to
disagree over which of the following?
(A)

Whether there is a positive correlation between
short organizational tenure and managerial
adaptability

(B)

Whether there is a positive correlation between
long organizational tenure and the acquisition of
idiosyncratic and nontransferable knowledge

(C)

Whether adaptability is a useful trait for an
executive who is managing an acquisition
process
439

GMAT® Official Guide 2018

(D)

Whether retaining less‐tenured top executives of
an acquired company is an optimal strategy for
achieving postacquisition success

(E)

Whether retaining highest‐level top executives
of acquired companies is more important than
retaining lower‐ranked top executives

Inference
The question asks about conclusions that can
reasonably be drawn from the information
provided in the passage. Bergh’s study supports
the resource‐based view (RBV), which suggests
that top executives with long tenure are more
valuable to postacquisition success than other
executives. The upper echelons perspective
(UEP), in contrast, suggests that top executives
with shorter tenure are more valuable to
postacquisition success. Thus, Bergh and a
proponent of UEP would likely disagree about
whether long or short tenure top executives
are more valuable to a positive outcome in a
postacquisition situation.
A

B

C

D

E

The passage does not provide sufficiently
specific information about statistical
relationships to determine whether Bergh
and proponents of UEP would agree or
disagree about whether there is such a
positive correlation.
There is a weak suggestion in the passage
that Bergh believes such a positive
correlation exists, but there is no indication
that a proponent of UEP would question
such a correlation.
The passage does not indicate that Bergh
would disagree with proponents of UEP
that adaptability is a valuable trait in an
executive who is managing an acquisition.
Correct. The passage suggests that
proponents of UEP believe that retaining
less‐tenured top executives during and after
an acquisition is a better strategy, while
Bergh believes that retaining longer‐tenured
top executives is better.
The passage suggests that Bergh and
proponents of UEP agree that retaining
the highest‐level top executives is more
important to postacquisition success than is
retaining lower‐ranked top executives.

The correct answer is D.
440

434. According to the passage, prior to Bergh’s study,
research on the role of top executives of acquired
companies in business acquisition success was limited
in which of the following ways?
(A)

It did not address how the organizational
tenure of top executives affects postacquisition
success.

(B)

It did not address why some companies have
longer‐tenured CEOs than others.

(C)

It did not consider strategies for retaining long‐
tenured top executives of acquired companies.

(D)

It failed to differentiate between the
contribution of highest‐level top executives
to postacquisition success and that of
lower‐ranked top executives.

(E)

It underestimated the potential contribution
that lower‐level top executives can make to
postacquisition success.

Supporting idea
This question asks about information explicitly
provided in the passage. The first paragraph
summarizes research indicating that retaining
highest‐level top executives during and after
an acquisition is more strongly associated
with successful outcomes than retaining
lower‐ranking top executives. The paragraph
then states that this research has limitations,
including failing to take into account how long
the highest‐ranking executives have worked for
the company. The second paragraph explains that
Bergh’s study responds to those limitations by
analyzing the role of tenure (length of service in
the organization).
A

B

C

D

Correct. The passage indicates that the
research about the role of highest‐level
executives in acquisitions is limited by its
failure to consider tenure.
The passage does not portray the failure of
the research to address this as a limitation of
the research in question.
The passage does not portray the failure of
the research to consider this as a limitation
of the research in question.
The passage indicates that the research does,
in fact, differentiate between the respective
contributions of these two groups of top
executives.

7.6 Reading Comprehension Answer Explanations

E

Undervaluing the contributions of lower‐
level top executives is not one of the
limitations mentioned in the passage.
The correct answer is A.
Questions 435–438 refer to the passage on page 378.
435. According to the passage, which of the following
contributed to Marcus Garvey’s success?
(A)

He introduced cultural and historical
consciousness to the African American
community.

(B)

He believed enthusiastically in the traditional
American success ethos.

(C)

His audience had already formed a
consciousness that made it receptive to his
message.

(D)

His message appealed to critics of African
American support for United States military
involvement in the First World War.

(E)

He supported the movement to protest
segregation that had emerged prior to his arrival
in the United States.

Supporting idea
To answer this question, find what the passage
states explicitly about how Marcus Garvey
achieved his success. The passage begins by
stating that Garvey arrived at the right time: that
returning African American soldiers were primed
to receive what he had to say about the African
American community. These soldiers already held
strong beliefs about their rights to opportunities
for success; the passage concludes that the divide
between the soldiers’ expectations and their
experiences led to Garvey’s success.
A

B

The passage states that African American
people were in possession of a strong
cultural and historical consciousness prior to
Garvey’s arrival in the United States.
The passage attributes belief in the
traditional American success ethos to
African American people who joined the
armed forces; it does not mention Garvey’s
beliefs on this subject.

Correct. African American soldiers who
had experienced segregation during the
First World War were ready to hear what
Garvey had to say.
D Critics of African American support for
United States involvement in the First
World War are not mentioned in the
passage.
E While Garvey most likely would have
supported a movement to protest
segregation, such a movement is not
discussed in the passage.
The correct answer is C.
C

436. The passage suggests that many African American
people responded to their experiences in the armed
forces in which of the following ways?
(A)

They maintained as civilians their enthusiastic
allegiance to the armed forces.

(B)

They questioned United States involvement in the
First World War.

(C)

They joined political organizations to protest the
segregation of African American troops and the
indignities they suffered in the military.

(D)

They became aware of the gap between their
expectations and the realities of American
culture.

(E)

They repudiated Garvey’s message of pride and
unity.

Inference
According to the passage, African Americans
enthusiastically joined the armed services but
were confronted with continued segregation, both
in the military and when they returned home. The
passage does not explicitly state their response to
these experiences, but a response can be inferred.
The second paragraph refers to anthropologist
Anthony F. C. Wallace, who argued that a
revitalization movement may be brought about
by the perception of a gap between expectations
and reality, and such a revitalization did occur in
African American communities following the
First World War; thus, many African American
people may have become aware of a gap such as
Wallace described.

441

GMAT® Official Guide 2018

A

B

C

D

E

The passage states that African American
troops experienced segregation and other
indignities while in the military; these
experiences could reasonably be inferred to
have dampened their enthusiasm for the
armed forces. Regardless, the passage does
not suggest an enthusiastic allegiance.
The passage describes African American
people’s enthusiasm about joining the
military. Although they experienced
segregation and other indignities while in
the military, the passage does not suggest
that their opinion about involvement in the
war changed.
While African American troops may have
joined political organizations, the passage
does not provide any actual evidence of this
having occurred.
Correct. The fact that, as the passage states,
a revitalization movement occurred in the
African American community following the
First World War suggests that the returning
soldiers did become aware of the gap
between their expectations of an improved
situation with regard to segregation and
the reality of continued segregation in the
United States.
The passage does not suggest that African
American troops repudiated Garvey’s
message. On the contrary, it states that
Garvey built the largest revitalization
movement in African American history. This
suggests that the members of the African
American community, including the
returning soldiers, were extremely receptive
to Garvey’s message.

(D)

Marcus Garvey hoped to revitalize consciousness
of cultural and historical identity in the African
American community.

(E)

The goal of the mass movement that Marcus
Garvey helped bring into being was to build on
the pride and unity among African Americans.

Inference
To determine what it is logical to infer regarding
the scholars discussed in the third paragraph,
look at the context in which they are mentioned.
According to the passage, these scholars argue
that Garvey was responsible for creating a
particular consciousness within the African
American community, a consciousness that the
passage identifies as identity, strength, and [a]
sense of history. Unlike the passage author, these
scholars believe strongly in Garvey’s responsibility
for this consciousness, so they would most likely
reject any suggestion that it existed prior to his
arrival and activism.
A

B

C

The correct answer is D.
437. It can be inferred from the passage that the “scholars”
mentioned in line 24 believe which of the following to
be true?
(A)

442

Revitalization resulted from the political activism
of returning African American soldiers following
the First World War.

(B)

Marcus Garvey had to change a number of
prevailing attitudes in order for his mass
movement to find a foothold in the United States.

(C)

The prevailing sensibility of the African American
community provided the foundation of Marcus
Garvey’s political appeal.

D

E

According to the passage, the scholars
believe that Garvey was responsible for
the creation of the consciousness that
led to revitalization, which suggests that
revitalization resulted from Garvey’s
activism, not soldiers’ activism.
Correct. According to the passage, the
scholars believe that Garvey created the
consciousness that led to his revitalization
movement. This suggests that he had to
change prevailing attitudes in order to foster
this new consciousness.
According to the passage, the scholars believe
that Garvey created a new consciousness in
the African American community; thus, the
prevailing sensibility could not have provided
a foundation for his appeal.
According to the passage, the scholars
believe that Garvey built his revitalization
movement on a new consciousness of
cultural and historical identity, not a
previously existing one.
According to the passage, the scholars’
position is that Garvey’s movement was
built on a new sense of pride and unity that
he provided, and that that sense did not
precede Garvey’s work.

The correct answer is B.

7.6 Reading Comprehension Answer Explanations

438. According to the passage, many African American
people joined the armed forces during the First World
War for which of the following reasons?

Questions 439–441 refer to the passage on page 380.

439. The primary purpose of the passage is to
(A)

They wished to escape worsening economic
conditions in African American communities.

(B)

They expected to fulfill ideals of personal
attainment.

(C)

They sought to express their loyalty to the United
States.

(D)

They hoped that joining the military would help
advance the cause of desegregation.

(E)

They saw military service as an opportunity to
fulfill Marcus Garvey’s political vision.

Supporting idea
This question depends on identifying what the
passage states directly about African American
people’s reasons for joining the armed forces. The
reason offered by the passage is that the African
American people who entered the armed forces
did so because they were hoping to participate in
the traditional American ethos of individual success.
A

B

C

D

E

Although this is a plausible reason for
entering the armed forces, the passage does
not discuss economic conditions.
Correct. The passage states that African
American people who joined the armed
forces during the First World War wanted
to achieve individual success.
The passage does not discuss African
American people’s loyalty to the United
States.
The passage states that African American
troops experienced segregation, but it
does not suggest that they had hoped
their joining the military would promote
desegregation.
The passage suggests that African American
troops did not become aware of Marcus
Garvey’s political vision until after they
returned from the First World War.

The correct answer is B.

(A)

compare the adaptive responses of several
species of columnar cacti in the Sonoran Desert
with those in the arid tropical regions of southern
Mexico

(B)

discuss some of the possible causes of the
relatively low abundance of migratory nectarfeeding bats in the Sonoran Desert

(C)

provide a possible explanation for a particular
evolutionary change in certain species of
columnar cacti in the Sonoran Desert

(D)

present recent findings that challenge a
particular theory as to why several species
of columnar cacti in the Sonoran Desert have
expanded their range of pollinators

(E)

compare the effectiveness of nocturnal and
diurnal pollination for several different species of
columnar cacti in the Sonoran Desert

Main idea
This question depends on understanding the
passage as a whole. The first paragraph discusses
an evolutionary change undergone by columnar
cacti in the Sonoran Desert with regard to
pollination. The second paragraph offers a
possible reason for this change—migratory
nectar-feeding bats are unreliable pollinators—
and the third paragraph goes on to provide
evidence that supports the reason given in the
second paragraph.
A

B

The passage does compare the adaptations
of cacti in the Sonoran Desert with those of
cacti in southern Mexico, but it does so in
support of a larger point about the Sonoran
Desert cacti.
The relatively low abundance of migratory
nectar-feeding bats in the Sonoran Desert is
important to the passage in that it provides
a reason why the columnar cacti in that
region have made certain adaptations. But
the passage does not explain why the bats
are not particularly abundant.

443

GMAT® Official Guide 2018

C

D

E

Correct. The flowers of the columnar cacti
in the Sonoran Desert have evolved to
remain open after sunrise, and the passage
is primarily concerned with explaining why
this change may have taken place.
The passage presents recent findings that
support, rather than challenge, a theory as
to why the columnar cacti of the Sonoran
Desert have expanded their range of
pollinators. The passage does not allude
to any competing theory that may be
challenged by the findings.
Any comparison of the effectiveness of
nocturnal and diurnal pollination for
columnar cacti in the Sonoran Desert is
made in support of the passage’s primary
concern: explaining why these cacti have
come to remain open and receptive to
pollination in daylight.

The correct answer is C.
440. According to the passage, which of the following types
of nectar-feeding pollinators is likely to be an unreliable
pollinator of a particular cactus flower?

B

C

D

E

A dietary specialist whose abundance is
high in relation to the flowers on which it
feeds would likely be a reliable pollinator.
Correct. A dietary specialist whose
abundance is at times significantly lower
than that of the flower it pollinates would
be, according to the passage, unreliable.
A dietary generalist who finds the flower
of a particular species more consistently
available than other suitable food sources
would most likely be a reliable pollinator of
that flower.
A dietary generalist who prefers the flower’s
nectar would likely be a reliable pollinator of
that flower compared to other flowers.
The passage provides no reason to believe
that the evolution of a pollinator’s dietary
preference has any bearing on its reliability
as a pollinator.

The correct answer is B.
441. According to the passage, present-day columnar cacti
in the Sonoran Desert differ from their close relatives
in southern Mexico in that the Sonoran cacti

(A)

A dietary specialist whose abundance is typically
high in relation to that of the flower

(A)

have flowers that remain open after sunset

(B)

are pollinated primarily by dietary specialists

(B)

A dietary specialist whose abundance is at times
significantly lower than that of the flower

(C)

can be pollinated by nectar-feeding bats

(D)

(C)

A dietary generalist for whom that flower’s
nectar is not a preferred food but is the most
consistently available food

have stigmas that are unreceptive to pollination
at night

(E)

are sometimes pollinated by diurnal pollinators

(D)

A dietary generalist for whom that flower’s nectar
is slightly preferred to other available foods

(E)

A dietary generalist that evolved from a species
of dietary specialists

Supporting idea
This question depends on recognizing the
qualities of an unreliable pollinator, as described
in the passage. The second paragraph addresses
this issue: it explains that the unreliability of
pollinators can arise in any of three ways: they
may be dietary generalists with alternative sources
of food; they may be dietary specialists whose
own abundance varies; or they may be dietary
specialists whose abundance is chronically low in
relation to the flowers.
444

A

Supporting idea
This question depends on identifying a difference
noted in the passage between columnar cacti
in the Sonoran Desert and their relatives in
southern Mexico. The first paragraph states that
in southern Mexico, columnar cactus flowers are
not receptive to pollination by diurnal pollinators,
whereas in the Sonoran Desert, the flowers have
evolved to allow diurnal pollination.
A

The cacti in both the Sonoran Desert and
southern Mexico have flowers that remain
open after sunset, because cacti in both
locations can be pollinated nocturnally.

7.6 Reading Comprehension Answer Explanations

B

C

D

E

Sonoran Desert cacti are pollinated, at least
partially, by nectar-feeding bats, which are
dietary specialists. But the cacti in southern
Mexico are pollinated by these specialists,
too.
Sonoran Desert cacti can be pollinated by
nectar-feeding bats—but so can cacti in
southern Mexico.
Cacti in the Sonoran Desert have stigmas
that have evolved to be receptive to
pollination both at night and during the day.
Correct. The distinction between cacti in
the Sonoran Desert and those in southern
Mexico is that Sonoran Desert cacti
have evolved to allow pollination during
the day—that is, pollination by diurnal
pollinators.

The correct answer is E.
Questions 442–449 refer to the passage on page 382.

Application
This question requires recognizing a principle
underlying the passage’s overall discussion. The
passage makes a general claim about terrestrial
animals’ need to overcome the effect of gravity
on their blood circulation systems, and it then
uses the specific example of terrestrial snakes
to illustrate this claim. To help identify the
adaptations used by terrestrial snakes, the
passage describes what happens to sea snakes,
which are aquatic and less affected by gravity’s
influence, when they are subjected to a terrestrial
environment. The specific problems faced by these
snakes strongly suggest that terrestrial snakes
have developed ways to overcome these problems.
The passage then identifies specific physiological
differences between sea snakes and terrestrial
snakes that demonstrate how terrestrial snakes
overcome gravity’s influence.
A

442. The passage provides information in support of which
of the following assertions?
(A)

(B)

(C)

(D)

(E)

The disadvantages of an adaptation to a
particular feature of an environment often
outweigh the advantages of such an adaptation.
An organism’s reaction to being placed in an
environment to which it is not well adapted can
sometimes illustrate the problems that have
been solved by the adaptations of organisms
indigenous to that environment.
The effectiveness of an organism’s adaptation to
a particular feature of its environment can only
be evaluated by examining the effectiveness with
which organisms of other species have adapted
to a similar feature of a different environment.
Organisms of the same species that inhabit
strikingly different environments will often adapt
in remarkably similar ways to the few features of
those environments that are common.
Different species of organisms living in the same
environment will seldom adapt to features of that
environment in the same way.

B

C

D

E

The passage discusses how species have
successfully adapted to their specific
environments and does not mention that
these adaptations create disadvantages in
that environment.
Correct. The passage discusses the problems
faced by sea snakes when they are subjected
to a terrestrial environment and then
examines terrestrial snakes to illustrate how
certain adaptations solved these problems.
The passage is not concerned with
evaluating the effectiveness of species’
adaptations to their environments; it takes
for granted that these adaptations are
effective.
The passage is concerned with how species
adapt differently to different environments
and not with how adaptations to different
environments are similar.
The passage discusses how different
environments affect how species have
adapted, not how different species adapt to a
similar environment.

The correct answer is B.

445

GMAT® Official Guide 2018

443. According to the passage, one reason that the
distribution of blood in the sea snake changes little
while the creature remains in the ocean is that
(A)

the heart of the sea snake tends to be located
near the center of its body

(B)

pressure gradients in the water surrounding
the sea snake counter the effects of vertical
pressure gradients within its blood vessels

(C)

the sea snake assumes a vertical posture less
frequently than do the terrestrial and the arboreal
snake

(D)

(E)

the sea snake often relies on waves of muscle
contractions to help move blood from the torso
to the head
the force of pressure gradients in the water
surrounding the sea snake exceeds that of
vertical pressure gradients within its circulatory
system

Supporting ideas
This question asks for an identification of
factual information in the passage. Given that
the contrast between sea snakes and terrestrial
snakes is being used to identify adaptations
used by terrestrial animals to overcome the
effect of gravity on their circulation systems, the
passage needs initially to illustrate why it is that
sea snakes are not confronted with the same
problems that gravity causes for terrestrial snakes.
This information therefore needs to come fairly
early in the passage.
A

B

C

446

The passage identifies the location of a sea
snake’s heart as a factor that minimizes
the effort required to pump blood to both
extremities but not as a cause of the even
distribution of blood in sea snakes.
Correct. The passage states explicitly in
lines 11–17 that while sea snakes are in
the ocean, the vertical pressure gradients in
their blood vessels are counteracted by the
pressure gradients in the water.
The passage does not discuss the frequency
with which any snakes assume certain
postures.

D
E

The passage discusses muscle contractions
only in relation to arboreal snakes.
The passage states that the vertical
pressures within sea snakes’ blood vessels
are counteracted (line 13) by the water’s
pressure, which suggests that the pressures
are equalized, not that one force exceeds the
other.

The correct answer is B.
444. It can be inferred from the passage that which of the
following is true of species of terrestrial snakes that
often need to assume a vertical posture?
(A)

They are more likely to be susceptible to
circulatory failure in vertical postures than are
sea snakes.

(B)

Their hearts are less likely to be located at the
midpoint of their bodies than is the case with sea
snakes.

(C)

They cannot counteract the pooling of blood in
lower regions of their bodies as effectively as
sea snakes can.

(D)

The blood pressure at their midpoint decreases
significantly when they are tilted with their
heads up.

(E)

They are unable to rely on muscle contractions
to move venous blood from the lower torso to
the head.

Inference
This question requires using information given
about how arboreal snakes, which are frequently
in vertical postures, have adapted to gravity’s
influence to make an assumption that other
terrestrial snakes that are frequently in these
postures are likely to have similar adaptations.
The passage implies that sea snakes have hearts at
the midpoint of their bodies because the water’s
pressure gradients help distribute blood evenly. It
then illustrates that arboreal snakes have hearts
closer to their heads to help keep blood flowing
to their brains when they are in vertical postures.

7.6 Reading Comprehension Answer Explanations

A

B

C

D

E

The passage does not suggest that any of the
snakes mentioned are ill‐adapted to their
particular environments.
Correct. The passage states that arboreal
snakes have hearts close to their heads and
not at the midpoints of their bodies, so it is
reasonable to conclude that any terrestrial
snake that frequently assumes vertical
postures would be unlikely to have hearts at
their bodies’ midpoint.
As with answer choice (A), the passage does
not suggest that any species of snake is ill‐
adapted to its environment.
The passage states that sea snakes lose
pressure at their midpoints when they
are tilted on land with heads up but that
terrestrial snakes do not have this problem.
Because arboreal snakes use muscle
contractions to circulate blood when they
are vertical, it is likely that most terrestrial
snakes that frequently assume vertical
postures also have this capability.

The correct answer is B.
445. The author describes the behavior of the circulatory
system of sea snakes when they are removed from the
ocean (see lines 17–20) primarily in order to
(A)

(B)

(C)

illustrate what would occur in the circulatory
system of terrestrial snakes without adaptations
that enable them to regulate their blood pressure
in vertical orientations
explain why arboreal snakes in vertical
orientations must rely on muscle contractions to
restore blood pressure to the brain
illustrate the effects of circulatory failure on the
behavior of arboreal snakes

(D)

illustrate the superiority of the circulatory system
of the terrestrial snake to that of the sea snake

(E)

explain how changes in spatial orientation can
adversely affect the circulatory system of snakes
with hearts located in relatively close proximity
to their heads

Evaluation
Answering this question requires understanding
why sea snakes have been brought into the
passage’s overall discussion about how terrestrial
animals have overcome the influence of gravity
on their blood circulation. The passage uses the
effects that gravity has on sea snakes when they
are taken out of water to identify problems that
terrestrial snakes must have adapted to in order to
survive.
A

B

C
D

E

Correct. The passage uses the problems
sea snakes have when taken out of water to
illustrate that without certain adaptations,
terrestrial snakes would likely have similar
problems.
The passage discusses sea snakes to illustrate
problems faced by terrestrial snakes, not to
explain how terrestrial snakes have adapted
to gravity’s influence.
The passage does not discuss the effects of
circulatory failure on arboreal snakes.
The passage does not compare or contrast
the effectiveness of the various adaptations
used by different snakes.
The passage does not imply that snakes with
hearts close to their heads are adversely
affected by spatial positions.

The correct answer is A.
446. It can be inferred from the passage that which of the
following is a true statement about sea snakes?
(A)

They frequently rely on waves of muscle
contractions from the lower torso to the head to
supplement the work of the heart.

(B)

They cannot effectively regulate their blood
pressure when placed in seawater and tilted at
an angle with the head pointed downward.

(C)

They are more likely to have a heart located in
close proximity to their heads than are arboreal
snakes.

(D)

They become acutely vulnerable to the effects
of gravitational pressure on their circulatory
system when they are placed in a terrestrial
environment.

(E)

Their cardiovascular system is not as
complicated as that of arboreal snakes.

447

GMAT® Official Guide 2018

Inference

Inference

Answering this question requires understanding
why sea snakes are discussed in the passage and
what happens to them when they are taken out
of water and subjected to the force of gravity. The
second paragraph implies strongly that sea snakes
will not survive certain terrestrial situations for
which they are not adapted.

This question asks for an inference about the
location of a snake’s heart being closer to the
brain than to the midpoint of its body. In the
third paragraph, the passage states that in
terrestrial snakes, which must fight the influence
of gravity, the closer proximity of the heart to
the head ensures blood circulation to the brain.
The passage notes, however, that this makes it
more difficult for such snakes to maintain blood
circulation to the tail.

A
B

C

D

E

The passage associates muscle contractions
to circulate blood with arboreal snakes only.
According to the passage, sea snakes’
inability to regulate blood pressure occurs
only when they are taken out of water.
The passage states clearly that arboreal
snakes have hearts closer to their heads than
do sea snakes.
Correct. The passage states that in certain
postures, sea snakes placed in a terrestrial
environment will lose all blood pressure at
their brains, which is an acute vulnerability.
The passage does not provide the
information needed to compare the
complexity of the various snakes discussed.

A

B

C

D

The correct answer is D.
447. The author suggests that which of the following is a
disadvantage that results from the location of a snake’s
heart in close proximity to its head?
(A)

A decrease in the efficiency with which the snake
regulates the flow of blood to the brain

(B)

A decrease in the number of orientations in
space that a snake can assume without loss of
blood flow to the brain

(C)

A decrease in blood pressure at the snake’s
midpoint when it is tilted at various angles with
its head up

(D)

An increase in the tendency of blood to pool
at the snake’s head when the snake is tilted at
various angles with its head down

(E)

448

An increase in the amount of effort required to
distribute blood to and from the snake’s tail

E

The passage states that snakes have hearts
closer to their heads to more efficiently
circulate blood to the brain.
The passage suggests that having the heart
close to the head increases the spatial
orientations a snake can assume without
losing blood flow to the brain, rather than
decreases the number of orientations.
The passage indicates that this is true only
of sea snakes with hearts near their body’s
midpoint.
The passage mentions blood pooling in the
lower portions of a terrestrial organism’s
body but does not imply that blood can pool
at a snake’s head.
Correct. Because, as the passage states, it is
more difficult for a snake with its heart close
to its head to circulate blood to the tail, and
therefore its body is likely to put more effort
into circulating blood to the tail.

The correct answer is E.
448. The primary purpose of the third paragraph is to
(A)

introduce a topic that is not discussed earlier in
the passage

(B)

describe a more efficient method of achieving an
effect discussed in the previous paragraph

(C)

draw a conclusion based on information
elaborated in the previous paragraph

(D)

discuss two specific examples of phenomena
mentioned at the end of the previous paragraph

(E)

introduce evidence that undermines a view
reported earlier in the passage

7.6 Reading Comprehension Answer Explanations

Evaluation

Main idea

Answering this question requires recognizing
how the passage develops its main point. The
first paragraph sets up a general claim about
gravity’s influence on terrestrial organisms. The
second paragraph then describes the ill effects that
gravity has on sea snakes to identify problems that
terrestrial snakes have had to adapt to. The third
paragraph then uses examples to illustrate how
terrestrial snakes have adapted to gravity’s influence.

Answering this question involves assessing
what the passage as a whole is attempting to
do. While the passage begins by making a
general claim about gravity’s influence on the
cardiovascular systems of terrestrial animals, it
immediately points to terrestrial snakes as a good
example supporting this claim. The rest of the
passage is then devoted to illustrating, using the
observations involving sea snakes, how gravity’s
influence has shaped the cardiovascular systems
of terrestrial snakes.

A

B

C

D

E

The topic of the third paragraph is the
adaptations developed by terrestrial snakes
to survive gravity’s influence, which is part
of the discussion in both the first and second
paragraphs.
There is no comparison in the passage of the
efficiency of the different methods used by
snakes to adapt to gravity’s influence.
The third paragraph is concerned with
illustrating certain adaptations used by snakes
and offers no conclusions about the problems
terrestrial snakes have had to adapt to, which
is the topic of the second paragraph.
Correct. The end of the second paragraph
refers to certain adaptations (line 23) that the
third paragraph then goes on to identify and
discuss.
The third paragraph supports the main idea
of the passage and is not used to counter
any claim made earlier.

A

B

C

D

E

The correct answer is D.

Correct. The entire passage is devoted to an
explanation of how terrestrial snakes have
adapted to gravity’s influence.
While the passage does compare the
systems of the two snakes, it does so for the
larger purpose of demonstrating gravity’s
influence on terrestrial snakes.
The passage is more concerned with how
the systems of the two snakes are different,
rather than why, in order to identify how
terrestrial snakes have adapted to gravity’s
influence.
There is no judgment in the passage as to
the superiority of one snake’s system over
the other.
While the passage does explain how sea
snakes do this, it does so only for the larger
purpose of identifying how terrestrial snakes
have adapted to gravity’s influence.

The correct answer is A.
449. In the passage, the author is primarily concerned with
doing which of the following?
(A)

(B)
(C)

Explaining adaptations that enable the terrestrial
snake to cope with the effects of gravitational
pressure on its circulatory system
Comparing the circulatory system of the sea
snake with that of the terrestrial snake
Explaining why the circulatory system of the
terrestrial snake is different from that of the sea
snake

Questions 450–455 refer to the passage on page 386.
450. The passage is primarily concerned with
(A)

contrasting the benefits of one methodology with
the benefits of another

(B)

describing the historical origins and inherent
drawbacks of a particular methodology

(C)

discussing the appeal of a particular
methodology and some concerns about its use

(D)

Pointing out features of the terrestrial snake’s
cardiovascular system that make it superior to
that of the sea snake

(D)

showing that some historians’ adoption of a
particular methodology has led to criticism of
recent historical scholarship

(E)

Explaining how the sea snake is able to
neutralize the effects of gravitational pressure on
its circulatory system

(E)

analyzing the influence of current feminist views
on women’s interpretations of their experience
449

GMAT® Official Guide 2018

Main idea

Supporting ideas

This question asks for an abstract view of what
the passage as a whole is primarily doing. The
passage introduces a particular methodology
that scholars of women’s history have been
encouraged to employ, explaining why the use of
the methodology is supported. The passage then
goes on to raise some concerns about the use of
the methodology and cites one example in which
caution is needed.

This question asks for an identification of specific
information provided by the passage. In the
second paragraph, the passage describes certain
concerns about using oral narratives. One of these
concerns is that the stories people tell to explain
themselves are shaped by . . . storytelling conventions
(lines 17–19) and other influences tied to the
teller’s cultural and historical context.

A
B

C

D
E

The passage is primarily concerned with
only one methodology.
The passage mentions why the methodology
had been encouraged but does not give
the history of its origins; while it cautions
historians to employ the methodology
carefully, it is not concerned with drawbacks
of its proper use.
Correct. The passage discusses why the use
of a methodology is being encouraged and
then offers some concerns about its use.
The passage does not discuss any criticism
of recent scholarship in women’s history.
There is no mention in the passage that
feminist theory is influencing how women
in general think about their experiences.

A

B

C
D

E

The passage uses standard histories (line 7) to
refer to the usual work of scholars and not
to something that influences oral narratives.
Correct. The passage raises as a concern
that oral narratives may be influenced by
storytelling conventions present in the
culture of the speaker.
The passage does not mention the family of
origin of women storytellers.
The passage does not mention the
expectations of the listeners of oral
narratives.
The passage does not discuss women
storytellers’ familiarity with the oral
narratives belonging to other groups of
women.

The correct answer is B.

The correct answer is C.
451. According to the passage, which of the following
shapes the oral narratives of women storytellers?

450

(A)

The conventions of storytelling in the culture in
which a woman storyteller lives

They should take into account their own life
experiences when interpreting the oral accounts
of women’s historical experiences.

(B)

A woman storyteller’s experience with distinctive
traditions of storytelling developed by the women
in her family of origin

They should assume that the observations made
in women’s oral narratives are believed by the
intended audience of the story.

(C)

They should treat skeptically observations
reported in oral narratives unless the
observations can be confirmed in standard
histories.

(D)

They should consider the cultural and historical
context in which an oral narrative was created
before arriving at an interpretation of such a
narrative.

(E)

They should rely on information gathered from
oral narratives only when equivalent information
is not available in standard histories.

(A)

The conventions for standard histories in the
culture in which a woman storyteller lives

(B)
(C)

452. The author of the passage would be most likely to
make which of the following recommendations to
scholars of women’s history?

(D)

The cultural expectations and experiences of
those who listen to oral narratives

(E)

A woman storyteller’s familiarity with the stories
that members of other groups in her culture tell
to explain themselves

7.6 Reading Comprehension Answer Explanations

Application
Answering this question involves recognizing
what the author believes about oral narratives
and then applying this belief to a hypothetical
situation in which the author makes
recommendations to scholars of women’s
history. While acknowledging the appeal of
oral narratives to these scholars, in the second
paragraph the author urges caution when
using these narratives as sources of disinterested
commentary (line 16). The passage then states
that people’s oral narratives are shaped by cultural
and historical factors (line 20), which presumably
relate to the cultural and historical context within
which the narratives are spoken.
A
B
C

D

E

The passage does not mention the personal
life experiences of scholars.
The passage does not mention the intended
audiences of oral narratives.
The passage mentions standard histories
(line 7) only as a reference to scholarly
works that often have shortcomings.
Correct. The passage cautions that oral
narratives may be biased due to cultural
and historical factors, and it is therefore
reasonable to suppose that the author would
recommend that scholars consider this when
using such information.
The passage does not refer to oral narratives
as being valuable only for filling a gap in the
available historical record.

The correct answer is D.
453. Which of the following best describes the function of
the last sentence of the passage?
(A)

It describes an event that historians view as
crucial in recent women’s history.

(B)

It provides an example of how political rhetoric
may influence the interpretations of experience
reported in women’s oral narratives.

(C)

It provides an example of an oral narrative that
inaccurately describes women’s experience
during a particular historical period.

(D)

It illustrates the point that some women are more
aware than others of the social forces that shape
their oral narratives.

(E)

It identifies the historical conditions that led to
the social acceptance of women’s paid work
outside the home.

Evaluation
This question requires recognizing how a
particular part of the passage is related to
the overall reasoning in the passage. The first
paragraph introduces a methodology and
describes the methodology’s appeal. The second
paragraph then raises concerns about the use
of the methodology, drawing attention to the
cultural and historical bias that may be present
in oral narratives. In line 21, the passage refers
specifically to the influence political rhetoric
may have on a woman’s understanding of her
experience. In the final sentence, the passage
provides a specific hypothetical example of a
woman at the time of the Second World War to
illustrate this concern.
A

B

C
D

E

The last sentence employs a hypothetical
example and does not describe a particular
event as being important to historians.
Correct. After contending that political
rhetoric may influence oral narratives, the
passage uses the example of the Second
World War in the final sentence to support
this claim.
The last sentence does not provide a
particular example of an oral narrative.
The passage does not claim that some
women are more aware than others of the
social forces that may bear on them.
The passage does not claim that social
conditions during the Second World
War led to acceptance of women in the
workplace.

The correct answer is B.

451

GMAT® Official Guide 2018

454. According to the passage, scholars of women’s history
should refrain from doing which of the following?
(A)
(B)

Relying on traditional historical sources when
women’s oral narratives are unavailable
Focusing on the influence of political rhetoric on
women’s perceptions to the exclusion of other
equally important factors

455. According to the passage, each of the following is a
difference between women’s oral narratives and most
standard histories EXCEPT:
(A)

Women’s oral histories validate the significance
of women’s achievements.

(B)

Women’s oral histories depict experience from
the point of view of women.

(C)

Attempting to discover the cultural and historical
factors that influence the stories women tell

(C)

Women’s oral histories acknowledge the
influence of well‐known women.

(D)

Assuming that the conventions of women’s
written autobiographies are similar to the
conventions of women’s oral narratives

(D)

Women’s oral histories present today’s women
with a sense of their historical relationship to
women of the past.

(E)

Accepting women’s oral narratives less critically
than they accept women’s written histories

(E)

Women’s oral histories are crucial to the
collective identity of today’s women.

Inference

Supporting ideas

Answering this question requires recognizing
which option is directly inferable from
information in the passage. After describing
in the first paragraph why oral narratives are
appealing to historians, the passage begins the
second paragraph by imploring scholars of
women’s history to be as cautious about accepting
oral narratives … as … written memories (lines
12–14). The passage then goes on to describe
potential bias in oral narratives, suggesting that
scholars should be as critical of them as they are
of written sources.

This question asks for information that is
stated in the passage, and it requires a process
of elimination. In line 7, oral narratives are
presented as being unlike most standard histories,
and the passage then goes on in lines 7–11 to list
characteristics of oral histories that most standard
histories do not have. The answer to this question
will therefore contain a characteristic of women’s
oral histories that is not described in lines 7–11.

A

B

C

D

E

The passage does not claim that traditional
historical sources should be avoided by
scholars.
The passage mentions the influence of
political rhetoric merely as one example of
potential bias.
The passage suggests that scholars should
attempt to be aware of cultural and
historical factors.
The passage does not discuss the
conventions of women’s written
autobiographies.
Correct. The passage implies that written
histories and oral narratives should receive
the same level of critical scrutiny by
scholars.

A

B

C

D

E

The passage states that, unlike most
standard histories, women’s oral histories
affirm the importance of women’s contributions
(lines 8–9).
The passage states that, unlike most
standard histories, women’s oral histories
represent experience from the perspective of
women (lines 7–8).
Correct. The passage does not mention the
influence of well‐known women on women’s
oral histories.
The passage states that, unlike most
standard histories, women’s oral histories
furnish present‐day women with historical
continuity (lines 9–10).
The passage states that, unlike most
standard histories, women’s oral histories
furnish a historical sense that is essential to
their identity, individually and collectively
(line 11).

The correct answer is E.
The correct answer is C.
452

7.6 Reading Comprehension Answer Explanations

Questions 456–460 refer to the passage on page 388.

D

456. The passage suggests that in order for a manufacturer
in a capital‐intensive industry to have a decisive
advantage over competitors making similar products,
the manufacturer must

E

(A)

be the first in the industry to build production
facilities of theoretically optimal size

(B)

make every effort to keep fixed and sunk costs
as low as possible

(C)

be one of the first to operate its manufacturing
plants at minimum efficient scale

(D)

produce goods of higher quality than those
produced by direct competitors

(E)

stockpile raw materials at production sites in
order to ensure a steady flow of such materials

The correct answer is C.
457. The passage suggests that which of the following is
true of a manufacturer’s fixed and sunk costs?
(A)

The extent to which they are determined
by market conditions for the goods being
manufactured is frequently underestimated.

(B)

If they are kept as low as possible, the
manufacturer is very likely to realize significant
profits.

(C)

They are the primary factor that determines
whether a manufacturer will realize economies of
scale.

(D)

They should be on a par with the fixed and sunk
costs of the manufacturer’s competitors.

(E)

They are not affected by fluctuations in a
manufacturing plant’s throughput.

Inference
This question asks for an inference about what
a manufacturer in a capital‐intensive industry
must do to have an advantage over competitors
making similar products. The passage addresses
this question by stating that advantage accrues
to those firms that are the first to exploit the
full potential of optimally sized, technologically
sophisticated plants. In this context, exploiting
the full potential of such plants means operating
them at minimum efficient scale. Based on the
definition in the first paragraph, this means that
the plant must have an output of such a size that
the cost per unit of output is at a minimum.
A

B

C

The passage says that for new capital‐
intensive firms to dominate the market, it is
not enough for them to have optimally sized
plants; the plants must also be operated in a
way that fully exploits their potential.
While keeping fixed and sunk costs low
would obviously help keep overall costs
low, the passage does not suggest that this
is decisive in enabling a firm to have an
advantage over competitors.
Correct. Being among the first
manufacturers to operate plants at minimum
efficient scale means that those plants
are being exploited to their full potential.
This strategy would most likely give such
manufacturers a decisive advantage over new
firms hoping to compete effectively.

The passage does not discuss the quality of
goods made by manufacturers.
The passage does not suggest that stockpiling
raw materials is the most efficient way to
ensure a steady flow of raw materials into the
manufacturing process, though the passage
states that such a steady flow is a factor in
achieving minimum efficient scale.

Inference
This question asks about what the passage implies
about fixed and sunk costs. The passage states that
when production declines due to certain factors,
such costs remain at the same level (which may
be high), and the cost per unit produced (unit
costs) rises sharply.
A

B

The passage discusses the impact of market
conditions on determining what the optimal
size of a manufacturing plant is (which
affects fixed and sunk costs). But it makes
no claim about the frequency with which
such an impact is “underestimated.”
The passage emphasizes that failing to
keep throughput at an efficiently high level
reduces profitability because that failure
results in increased cost per unit (to which,
of course, the plant’s fixed and sunk costs
contribute). But the passage does not claim
that keeping aggregate fixed and sunk costs
very low is necessary in order to have the
most competitive production operation.
453

GMAT® Official Guide 2018

C

D

E

The passage emphasizes that the crucial
factor in achieving economies of scale
is efficient operation of the production
facilities, not the size of the firm’s fixed
and sunk costs (even though such costs are
clearly in part determined by the size and
design of the production facilities).
While a manufacturer’s fixed and sunk
costs may be on a par with those of the
manufacturer’s competitors, the passage
provides no grounds for inferring that there
is any need for them to be (for example,
physical plants that employ different
technologies may have different price tags).
Correct. According to the passage,
“throughput” refers to the flow of materials
through a plant. This flow can vary as a
result of various factors, but fixed and
sunk costs—financial resources already
committed—remain the same regardless of
such variation.

A

B

C
D

E

The correct answer is E.
458. In the context of the passage as a whole, the second
paragraph serves primarily to
(A)

provide an example to support the argument
presented in the first paragraph

(B)

evaluate various strategies discussed in the first
paragraph

The correct answer is A.
459. The passage LEAST supports the inference that a
manufacturer’s throughput could be adversely
affected by

(C)

introduce evidence that undermines the
argument presented in the first paragraph

(D)

anticipate possible objections to the argument
presented in the first paragraph

(A)

a mistake in judgment regarding the selection of
a wholesaler

(E)

demonstrate the potential dangers of a
commonly used strategy

(B)

a breakdown in the factory’s machinery

(C)

a labor dispute on the factory floor

(D)

an increase in the cost per unit of output

(E)

a drop in the efficiency of the sales network

Evaluation
This question asks about the rhetorical
function of the second paragraph. While the
first paragraph argues that a crucial factor in
achieving economies of scale is intangible capital,
or organized human capabilities, the second
paragraph uses the example of new capital‐
intensive manufacturing industries to help show
that this is indeed the case.

454

Correct. The second paragraph provides
an example that illustrates the claims
made in the first paragraph. It discusses
the way in which intangible capital—e.g.,
distribution networks, marketing systems,
smooth production processes, and qualified
management teams—enables manufacturers
in new capital‐intensive manufacturing
industries to realize economies of scale and
achieve market dominance.
The second paragraph does, in a sense,
“evaluate” investment in intangible capital:
it suggests that such investment is necessary.
However, investment in intangible capital is
the only strategy it discusses.
The second paragraph supports rather than
undermines the first paragraph’s argument.
Nothing in the second paragraph suggests
that there are, or could be, any objections to
the first paragraph’s argument.
The second paragraph discusses the
potential positive outcomes of investing
in intangible capital. It suggests that there
might be negative consequences to not
making such investments, but it does not
indicate that avoiding such investments is a
commonly used strategy.

Application
This question may be best approached by using
an elimination strategy—first finding the four
choices that can reasonably be inferred from the
passage, and then checking to make sure that the
remaining choice cannot reasonably be inferred.
This requires understanding the information the
passage gives about throughput, then making
inferences about what can cause throughput to

7.6 Reading Comprehension Answer Explanations

drop. The passage defines throughput generally
as the flow of materials through a plant and goes
on to explain that it involves coordination of the
production process itself, as well as obtaining
materials from suppliers and marketing and
distributing the manufactured products. Anything
that damages this flow of materials and products
would be said to have an adverse effect on
throughput.
A

B

C

D

E

Making a poor judgment about a wholesaler
would most likely have an adverse effect on
throughput, in that it could affect the flow of
output to wholesalers and final consumers.
A breakdown in machinery would likely
fall into the category of problems on the
factory floor mentioned in the passage and
would likely prove damaging to throughput
because of its effect on the production
process itself.
A labor dispute would also likely fall into
the category of problems on the factory
floor mentioned in the passage and would
probably cause a decline in production and
thus adversely affect throughput.
Correct. The passage emphasizes that
changes in throughput can cause increases
or decreases in costs per unit. But the
passage is not committed to any claims
about how changes in costs per unit might
affect throughput.
The passage suggests that inefficient
sales networks could cause a decline
in production. Thus a decrease in sales
efficiency would most likely adversely affect
a manufacturer’s ability to provide goods to
consumers, and thus would create problems
with throughput.

The correct answer is D.

460. The primary purpose of the passage is to
(A)

point out the importance of intangible capital for
realizing economies of scale in manufacturing

(B)

show that manufacturers frequently gain a
competitive advantage from investment in large
manufacturing facilities

(C)

argue that large manufacturing facilities often
fail because of inadequate investment in both
tangible and intangible capital

(D)

suggest that most new industries are likely
to be dominated by firms that build large
manufacturing plants early

(E)

explain why large manufacturing plants usually
do not help manufacturers achieve economies of
scale

Main idea
This question depends on understanding the
passage as a whole. In general, it makes an
argument for investing in intangible capital as
a way for manufacturers to realize economies
of scale, and it supports its argument with an
example.
A

B

C

D

E

Correct. The passage focuses on intangible
capital as a crucial factor in realizing
economies of scale.
According to the passage, manufacturers
gain competitive advantage by building
plants of optimal size that they then fully
exploit; nothing in the passage suggests that
large plants are frequently optimal.
The passage assumes that manufacturers
invest appropriately in tangible capital and
argues that it is important for them to invest
in intangible capital as well.
The passage states that new capital‐intensive
manufacturing industries are dominated
not by firms that are the first to build large
plants, but by firms that exploit the full
potential of their plants.
The passage indicates that economies of
scale can be achieved in plants of optimal
size. The passage does not suggest that large
plants cannot be optimal.

The correct answer is A.

455

GMAT® Official Guide 2018

Questions 461–466 refer to the passage on page 390.

E

461. The primary purpose of the passage is to
(A)

describe the development of new techniques that
may help to determine the driving force behind
population cycles in lepidoptera

(B)

present evidence that refutes a particular theory
about the driving force behind population cycles
in lepidoptera

(C)

present a hypothesis about the driving force
behind population cycles in lepidoptera

(D)

describe the fluctuating patterns of population
cycles in lepidoptera

(A)

develop an explanation for the existence of
lepidoptera population cycles

(E)

question the idea that a single driving force is
behind population cycles in lepidoptera

(B)

identify behavioral factors in lepidoptera that
affect survival rates

(C)

identify possible methods for controlling
lepidoptera population growth

(D)

provide evidence that lepidoptera populations
are self‐regulating

(E)

determine the life stages of lepidoptera at which
mortality rates are highest

Main idea
This question depends on understanding the
passage as a whole in order to identify its purpose.
The first paragraph defines population cycles of
lepidoptera and discusses some ways those cycles
have been studied. It suggests that a particular
agent may regulate these cycles. The second
paragraph describes a candidate for this agent:
nuclear polyhedrosis viruses. The third paragraph
explains why this hypothesis is compelling.
A

B

C

D

456

The passage is concerned with making a
case for nuclear polyhedrosis viruses as
the driving force behind at least some
lepidoptera population cycles, not with
questioning the idea that there is a driving
force.

The passage mentions new techniques in
molecular biology, but it does so in order to
explain why a particular candidate for the
agent behind population cycles has come to
light.
The theory the passage presents is that
there is a driving force behind lepidoptera
population cycles. It does not refute this
theory; rather, it offers a convincing case for
nuclear polyhedrosis viruses as that force. It
also discusses some previous approaches to
seeking plausible hypotheses but does not
focus on refuting any particular hypothesis.
Correct. The passage is primarily concerned
with presenting the hypothesis that nuclear
polyhedrosis viruses are the driving force
behind lepidoptera population cycles.
The first paragraph describes the fluctuating
patterns of lepidoptera population cycles,
but it does so to explain what population
cycles are, so that it can then go on to
attempt to account for those cycles.

The correct answer is C.
462. It can be inferred from the passage that the mortality
caused by agents such as predatory birds or parasites
was measured in an attempt to

Inference
The passage states that mortality caused by
various agents, birds and parasites among them,
was measured because this was the common
approach to studying causes of population cycles.
This in turn suggests that those scientists engaged
in such measuring in the case of lepidoptera
were attempting to come up with a definitive
explanation for why those lepidoptera population
cycles occurred.
A

B

C

Correct. Measuring mortality caused by
various agents was part of the attempt
to determine the driving force behind
lepidoptera population cycles.
The passage does not indicate that
behavioral factors in lepidoptera are related
to their mortality as caused by agents such
as predatory birds or parasites.
The passage is concerned not with
controlling lepidoptera population growth,
but rather with determining why population
cycles occur.

7.6 Reading Comprehension Answer Explanations

D

E

According to the information in the
passage, scientists sought to measure
mortality caused by particular agents in
order to determine the driving force behind
lepidoptera population cycles. In suggesting
that mortality caused by these agents is
not that force, the measurements may have
indicated that the cycles could be self‐
regulating, but they were not undertaken in
order to provide such evidence.
The passage discusses mortality primarily
in the caterpillar stage and does not suggest
that any research was directed toward
comparing caterpillar mortality rates with
mortality rates in other life stages of the
insects.

A

B

C

D

The correct answer is A.
463. Which of the following, if true, would most weaken the
author’s conclusion in lines 18–22 ?
(A)

New research reveals that the number of species
of birds and parasites that prey on lepidoptera
has dropped significantly in recent years.

(B)

New experiments in which the habitats of
lepidoptera are altered in previously untried ways
result in the shortening of lepidoptera population
cycles.

(C)

Recent experiments have revealed that the
nuclear polyhedrosis virus is present in a number
of predators and parasites of lepidoptera.

(D)

Differences among the habitats of lepidoptera
species make it difficult to assess the effects of
weather on lepidoptera population cycles.

(E)

Viral disease is typically observed in a large
proportion of the lepidoptera population.

E

The correct answer is B.
464. According to the passage, before the discovery of
new techniques for detecting viral DNA, population
ecologists believed that viral diseases
(A)

were not widely prevalent among insect
populations generally

(B)

affected only the caterpillar life stage of
lepidoptera

(C)

were the driving force behind lepidoptera
population cycles

(D)

attacked already declining caterpillar populations

(E)

infected birds and parasites that prey on various
species of lepidoptera

Evaluation
The sentence in question presents the author’s
conclusion that lepidoptera populations may be
self‐regulating or regulated by something more
closely connected to the insects than predatory
birds or parasites are. To weaken that conclusion
requires weakening its support, namely, that
mortality caused by predators and parasites seems
not to affect population cycles, and that changing
habitats and reducing populations has not altered
population cycles either.

A drop in birds and parasites preying on
lepidoptera would not weaken the author’s
conclusion; mortality caused by these
predators has not affected population cycles.
Correct. New experiments involving
changes in habitat that did succeed in
altering population cycles would suggest
that the populations are not in fact self‐
regulating, and that the search for another
cycle‐altering agent may be unnecessary.
This finding would support the idea that
the nuclear polyhedrosis virus is responsible
for population cycles—that is, that the virus
is the closely connected agent the author
concludes is responsible.
The suggestion that the effects of weather
may not have been adequately assessed is
remotely relevant to the author’s conclusion,
but the mere difficulty of assessing the
effects provides no positive reason to
suppose that weather may be the cause of
the cycles. On the other hand, answer choice
B does offer evidence for an alternative
explanation.
Viral disease is what the author ultimately
suggests is the agent that drives the
lepidoptera population cycles in question.
The wide presence of viruses in lepidoptera
could help support the author’s conclusion.

457

GMAT® Official Guide 2018

Supporting Idea
This question addresses what the passage states
directly about population ecologists’ beliefs
regarding viral diseases prior to the discovery
of new viral DNA–detection techniques. The
second paragraph of the passage states that these
ecologists believed viral disease contributed to
population decline that was already underway
rather than initiating it.

Supporting Idea
The passage states in the second paragraph that
these viruses remain virulent for many years if
they are protected from direct sunlight. They are
embedded in crystals of polyhedrin protein.

A

B

B

C

D

E

The second paragraph states that viral
disease had been reported; thus, population
ecologists were aware of its existence in
insect populations. The passage is consistent
with ecologists having believed that it was
prevalent.
The passage focuses mainly on the
caterpillar life stage of lepidoptera, but there
is nothing to suggest that scientists held
particular beliefs regarding viral diseases’
restriction to that life stage.
It is after, not before, the discovery of new
techniques for detecting viral DNA when
populations ecologists came to believe that
such diseases were the driving force behind
the population cycles.
Correct. As stated in the passage,
population ecologists believed that viral
diseases contributed to already occurring
population decline.
The passage does not discuss whether
viral diseases may infect any lepidoptera
predators.

The correct answer is D.
465. According to the passage, nuclear polyhedrosis
viruses can remain virulent in the environment only
when

458

(A)

the polyhedrin protein crystals dissolve

(B)

caterpillar populations are in decline

(C)

they are present in large numbers

(D)

their concentration in a particular area remains
low

(E)

they are sheltered from direct sunlight

A

C

D

E

The viruses remain virulent partially because
of their being contained in polyhedrin
protein crystals. They would most likely not
remain virulent if those crystals dissolved.
The viruses remain virulent even when
caterpillar populations are not in decline;
that is how the viruses initiate new
population declines.
According to the passage, viral DNA has
been detected in the environment at low
concentrations, yet the viruses are still
virulent. Thus, they need not be present in
large numbers.
Nothing in the passage indicates that the
concentration of these viruses must be low
for them to be virulent.
Correct. The passage says that if the viruses
are protected from direct sunlight, they
remain virulent for many years. The context
strongly suggests that if they are not so
protected, they do not remain virulent.

The correct answer is E.
466. It can be inferred from the passage that while inside its
polyhedrin protein crystals, the nuclear polyhedrosis
virus
(A)

is exposed to direct sunlight

(B)

is attractive to predators

(C)

cannot infect caterpillars’ cells

(D)

cannot be ingested by caterpillars

(E)

cannot be detected by new techniques of
molecular biology

Inference
The passage indicates that the polyhedrin protein
crystals protect the nuclear polyhedrosis virus when
it is in the environment. When a caterpillar ingests
those crystals, they dissolve. That releases the virus,
whereupon it infects the caterpillar’s cells. Thus it is
reasonable to infer that the virus must be released
from the crystals before it can infect the caterpillar.

7.6 Reading Comprehension Answer Explanations

A

B

C
D
E

The passage states that nuclear polyhedrosis
viruses remain embedded in polyhedrin
protein crystals if protected from direct
sunlight, not that the virus is exposed to
light when it is in the protein crystals.
Nothing in the passage indicates that any
organism preys on the virus itself or that it
attracts predators to caterpillars that it infects.
Correct. The virus must be released from the
crystals before it can infect caterpillars’ cells.
The passage states that caterpillars ingest
the polyhedrin protein crystals.
According to the passage, new techniques
of molecular biology enable the detection of
viral DNA in the environment. The nuclear
polyhedrosis virus persists in the environment
inside protein crystals. The passage suggests
that the new techniques are able to detect the
virus inside its crystals but does not provide
any evidence about whether they detect it
directly or infer its presence indirectly.

A

B

C

The correct answer is C.
Questions 467–470 refer to the passage on page 392.

D

467. According to the passage, which of the following is
true of plant antiherbivore chemistry?
(A)

Changes in a plant’s antiherbivore chemistry may
affect insect feeding behavior.

(B)

A plant’s repellent effects often involve
interactions between gum and resin.

(C)

A plant’s antiherbivore responses assist in
combating bacterial infections.

(D)

Plant antiherbivore chemistry plays only a minor
role in the coevolution of plants and insects.

(E)

Researchers first studied repellent effects in
plants beginning in the 1950s.

Supporting ideas
This question addresses what the information in
the passage indicates about plant antiherbivore
chemistry—that is, plants’ chemical defenses
against herbivore attacks. The second paragraph
of the passage cites the views of various
scientists regarding the possible role of resin
in antiherbivore chemistry; plants could have
evolved resin specifically to repel insects.

E

Correct. According to the second
paragraph, various scientists have suggested
that a change in antiherbivore chemistry,
here specifically involving resin, could repel
insects; alternatively, some insects could
have been attracted to resin, feeding more
heavily on plants that produced it. Other
researchers have suggested that even if resin
does not directly repel or attract insects,
it may indirectly affect insect-feeding
behavior by mediating changes in plants’
antiherbivore chemistry.
The first paragraph states that plants
produce gum in response to bacterial
infections. Although this does not rule out
the hypothesis that gum also contributes to
plants’ antiherbivore chemistry, the passage
provides no evidence that it does so.
According to the passage, a plant’s
antiherbivore responses have developed to
combat predators, such as insects, that eat
plants. The passage provides no evidence
that such responses also combat bacterial
infections.
The second paragraph indicates that plant
antiherbivore chemistry plays a major
role in the discipline of chemical ecology,
and chemical ecology concerns itself with
coevolution of plants and insects.
According to the passage, it was in the
1950s that entomologists began discussing
resin’s possible role in repelling and
attracting insects. The passage does not
suggest that this marked the beginning
of their study of repellent effects more
generally.

The correct answer is A.
468. Of the following topics, which would be most likely to
be studied within the discipline of chemical ecology as
it is described in the passage?
(A)

Seeds that become attached to certain insects,
which in turn carry away the seeds and aid in
the reproductive cycle of the plant species in
question

(B)

An insect species that feeds on weeds
detrimental to crop health and yield, and how
these insects might aid in agricultural production
459

GMAT® Official Guide 2018

(C)

The effects of deforestation on the life cycles
of subtropical carnivorous plants and the insect
species on which the plants feed

(D)

The growth patterns of a particular species of
plant that has proved remarkably resistant to
herbicides

(E)

Insects that develop a tolerance for feeding on a
plant that had previously been toxic to them, and
the resultant changes within that plant species

469. The author refers to “bacterial infections” (see line 11)
most likely in order to
(A)

describe the physiological function that gum
performs in plants

(B)

demonstrate that sap is not the only substance
that is transported through a plant’s tissues

(C)

explain how modern chemical analysis has been
used to clarify the function of resin

(D)

show that gum cannot serve as an effective
defense against herbivores

(E)

give an example of how confusion has arisen
with regard to the nature of resin

Application
The discipline of chemical ecology, as it is
described in the passage, deals with how plants
use chemicals to interact with other organisms—
in particular, how they defend against attack—
and how those interactions have evolved. To be
studied within that discipline, a specific topic
would need to address some aspect of that
chemical interaction.
A

B

C

D

E

The passage provides no reason to suppose
that the topic of seeds and how they travel
would be studied within chemical ecology,
given that it does not discuss how chemicals
might be involved in the reproductive cycle.
The passage provides no indication that
chemical ecology would be concerned with
how weed-destroying insects would aid
agricultural production.
The passage provides no indication that
deforestation would involve plant chemicals
or that its effects would be studied in
chemical ecology.
The passage provides no indication that a
plant’s resistance to herbicides would be
studied in chemical ecology, but the passage
does suggest that the focus of chemical
ecology is on how plants chemically interact
with other organisms.
Correct. Chemical ecology developed to
deal with the interdependence between
plants and insects. Insects’ developing
a tolerance for feeding on a once-toxic
plant, and the plants’ resultant changes, is
a situation of just such interdependence:
plants and insects coevolving.

The correct answer is E.
460

Evaluation
The author mentions bacterial infections in the
first paragraph as the reason why plants produce
the substance known as gum.
A

B

C

D

E

Correct. The author states directly that
plants produce gum in response to bacterial
infections.
The author states directly that sap is
transported through plant tissues. The
passage does not address the question of
whether bacterial infections or anything
related to them are similarly transported.
The passage indicates that rigorous chemical
analysis is now available, but scientists still
do not know resin’s function. The reference
to bacterial infections is related to gum, not
resin.
The reference to bacterial infections
indicates the actual purpose served by gum;
it does not function to show ways in which
gum is inadequate.
Gum itself serves as an example of the
confusion surrounding the nature of
resin; bacterial infections, to which gum
production is a response, do not serve as
that example.

The correct answer is A.

7.6 Reading Comprehension Answer Explanations

470. The author of the passage refers to Pliny most
probably in order to

Questions 471–473 refer to the passage on page 394.
471. The primary purpose of the passage is to

(A)
(B)

give an example of how the nature of amber has
been misunderstood in the past

(A)

show that confusion about amber has long been
more pervasive than confusion about resin

compare the impact of the Great Depression on
Latin America with its impact on the United States

(B)

criticize a school of economic historians for
failing to analyze the Great Depression in Latin
America within a global context

(C)

illustrate the risks inherent in comparing different
types of economic enterprises to explain
economic phenomena

(D)

call into question certain scholars’ views concerning
the severity of the Great Depression in Latin America

(E)

demonstrate that the Great Depression had
a more severe impact on industry in Latin
American than in certain other regions

(C)

make note of the first known reference to amber
as a semiprecious gem

(D)

point out an exception to a generalization about
the history of people’s understanding of amber

(E)

demonstrate that Pliny believed amber to be a
mineral

Evaluation
The passage states generally that amber has been
widely misunderstood but cites Pliny as noting
correctly, in the first century, that amber resulted
from a substance discharged by trees.
A

B

C

D

E

Pliny’s observation was, according
to the author, accurate and not a
misunderstanding.
The author equates confusion about amber
with confusion about resin; the reference
to Pliny does not indicate which of the
two, amber or resin, has been more widely
misunderstood.
The author indicates that others, not Pliny,
mischaracterized amber as a semiprecious
gem—and when that mischaracterization
first occurred is not identified.
Correct. Pliny’s recognition that amber
came from a substance discharged by
trees stands, in the author’s account, as
an exception to the widespread incorrect
identifications of the substance.
Others held the belief that amber was a
mineral. The passage indicates that Pliny
recognized that amber came from trees but
provides no evidence that he also considered
it a mineral.

The correct answer is D.

Main idea
This question depends on understanding the
passage as a whole. The passage first describes the
view of many economic historians of the 1980s. It
next describes the evidence on which that view is
based. The remainder of the passage raises issues
about the rationale for that view.
A

B
C

D

E

The comparison between Latin America
and the United States is only a small part of
a larger argument analyzing studies of the
Great Depression in Latin America.
The passage does not discuss a global
context for the Great Depression.
The passage does not primarily aim to
illustrate risks that may be generally
inherent in explaining economic
phenomena.
Correct. The passage claims that certain
scholars underestimate the severity of the
Great Depression in Latin America.
The passage does not claim that the impact
of the Great Depression on Latin American
industry was generally more severe than its
impact on industry elsewhere.

The correct answer is D.

461

GMAT® Official Guide 2018

472. Which of the following conclusions about the Great
Depression is best supported by the passage?
(A)

It did not impede Latin American industrial
growth as much as historians had previously
thought.

(B)

It had a more severe impact on the Brazilian and
the Mexican textile industries than it had on Latin
America as a region.

(C)

It affected the Latin American textile industry
more severely than it did any other industry in
Latin America.

(D)

The overall impact on Latin American industrial
growth should be reevaluated by economic
historians.

(E)

Its impact on Latin America should not be
compared with its impact on the United States.

E

The correct answer is D.
473. Which of the following, if true, would most strengthen
the author’s assertion regarding economic indicators in
lines 25–27 ?
(A)

During an economic depression, European textile
manufacturers’ profits rise while their industrial
output remains steady.

(B)

During a national economic recession, United
States microchips manufacturers’ profits rise
sharply while United States steel manufacturers’
profits plunge.

(C)

During the years following a severe economic
depression, textile manufacturers’ output levels
and profit levels increase in Brazil and Mexico
but not in the rest of Latin America.

(D)

Although Japanese industry as a whole recovers
after an economic recession, it does not regain
its previously high levels of production.

(E)

While European industrial output increases in the
years following an economic depression, total
output remains below that of Japan or the United
States.

Inference
This question asks which conclusion is most
strongly supported by the passage. The passage
presents the rationale of some historians for their
conclusion that the Great Depression did not
significantly interfere with economic growth in
Latin America. It then critiques that rationale
and conclusion. By questioning the historians’
claims, the passage suggests that a reevaluation of
the Great Depression’s effect on Latin America is
needed.
A

B

C

D

462

The passage does not significantly support
this. The passage indicates that, in fact, the
Great Depression impeded Latin American
economic development more than some
historians had thought.
The passage does not significantly support
this. The passage does not compare the
impact on the Brazilian and Mexican
textile industries to the impact on the Latin
American region.
The passage does not significantly support
this. The passage does not compare the
effect of the Great Depression on the textile
industry to its effect on other industries.
Correct. As presented in the passage, the
passage author’s critique of the historians’
rationale for their claims provides significant
support for the conclusion that their claims
should be reevaluated.

The passage does not significantly support
the claim that the comparison in question
should not be made.

Application
The question involves applying information
from outside the passage to a claim made by the
author. The text in lines 25–27 asserts that broad
economic indicators pertaining to a nation or
region can obscure differences between individual
firms or industries within that nation or region.
The question asks which evidence would most
strengthen the support for that conclusion.
A

This refers only to the relationship between
a single industry’s profits and its output, not
to general economic indicators.

7.6 Reading Comprehension Answer Explanations

B

C

D

E

Correct. The phrase a national recession
refers to a general economic indicator.
Suppose that in a situation described as a
national recession, one industry (microchip
manufacturing) prospers while another
industry (steel manufacturing) does not.
This would provide some additional support,
over and above that given in the passage, for
the assertion that broad economic indicators
may mask differences between industries.
Economic differences between countries do
not strengthen the support for the author’s
assertion regarding variations among
different firms and industries in one country
or region.
This has no obvious bearing on how
sweeping economic indicators can mask
differences between industries or enterprises
in a single country or region.
A comparison of different countries does
not pertain to the assertion regarding
variation among firms and industries in the
same country.

Inference
This item depends on understanding the
implications of the passage’s discussion of
differences between large and small plants. It asks
what might be true of a larger plant that would
compel it to spend more than a smaller plant on
environmental compliance. The passage addresses
this issue by stating that smaller plants are often
not subject to the same permit or reporting
requirements that larger plants are.
A

B

The correct answer is B.
Questions 474–477 refer to the passage on page 396.

C

474. It can be inferred from the passage that a large plant
might have to spend more than a similar but smaller
plant on environmental compliance because the larger
plant is

D

E
(A)

more likely to attract attention from local
regulators

(B)

less likely to be exempt from permit and
reporting requirements

(C)

less likely to have regulatory costs passed on to
it by companies that supply its raw materials

(D)

more likely to employ older production
technologies

(E)

more likely to generate wastes that are more
environmentally damaging than those generated
by smaller plants

The likelihood of attracting regulatory
attention is discussed only in the context of
comparing plants that are isolated with small
plants that are near large noncompliant
ones. The passage does not suggest that size
is generally the crucial determining factor in
attracting regulatory attention.
Correct. According to the passage, certain
permit or reporting requirements may
not apply to smaller plants; this suggests
that larger plants are less likely than
smaller plants to be exempt from these
requirements, and thus that the larger plants
would have to spend more to comply.
The passage does not discuss the passing on
of regulatory costs from suppliers to plants.
The passage does not suggest that larger
plants are any more likely than smaller
plants to employ older production
technologies.
The passage does not distinguish between
the types of wastes emitted by larger plants
and those emitted by smaller plants.

The correct answer is B.
475. According to the passage, which of the following
statements about sulfur dioxide and nitrogen oxide
outputs is true?
(A)

Older production technologies cannot be
adapted so as to reduce production of these
outputs as waste products.

(B)

Under the most recent environmental regulations,
industrial plants are no longer permitted to
produce these outputs.

(C)

Although these outputs are environmentally
hazardous, some plants still generate them as
waste products despite the high compliance
costs they impose.
463

GMAT® Official Guide 2018

(D)

Many older plants have developed innovative
technological processes that reduce the
amounts of these outputs generated as waste
products.

(E)

Since the production processes that generate
these outputs are less costly than alternative
processes, these less expensive processes are
sometimes adopted despite their acknowledged
environmental hazards.

Supporting idea
This item depends on identifying what the
passage states explicitly about outputs of sulfur
dioxide and nitrogen oxide. The passage says that
plants that produce these outputs are those that
use older industrial coal‐fired burners, and that
such plants are subject to extensive compliance
costs imposed by new regulations.
A

B

C

D

E

The passage does not address the question
of whether older production technologies
might be adapted to reduce outputs of sulfur
dioxide and nitrogen oxide.
The passage states that new regulations have
imposed high compliance costs on companies
that produce sulfur dioxide and nitrogen oxide
outputs, not that these outputs are prohibited.
Correct. The passage states that some
companies are still using the older kinds
of burners that generate sulfur dioxide
and nitrogen oxide outputs, and that new
regulations have imposed high compliance
costs on these companies.
The passage does not address the question
of whether older plants have developed new
processes to reduce the amounts of sulfur
dioxide and nitrogen oxide they produce.
Sulfur dioxide and nitrogen oxide outputs,
the passage suggests, are produced only by
older industrial coal‐fired burners; newer
facilities (using alternative processes) do
not employ this technology, the expense of
which is not mentioned in the passage.

The correct answer is C.

464

476. Which of the following best describes the relationship
of the statement about large plants (lines 12–17) to
the passage as a whole?
(A)

It presents a hypothesis that is disproved later in
the passage.

(B)

It highlights an opposition between two ideas
mentioned in the passage.

(C)

It provides examples to support a claim made
earlier in the passage.

(D)

It exemplifies a misconception mentioned earlier
in the passage.

(E)

It draws an analogy between two situations
described in the passage.

Evaluation
This question asks about the role played in the
passage by the following statement: Additionally,
large plants can spread compliance costs such as waste
treatment across a larger revenue base; on the other
hand, some smaller plants may not even be subject
to certain provisions such as permit or reporting
requirements by virtue of their size. This statement
describes situations in which compliance costs for
plants of different sizes may differ, which serve as
evidence in support of the passage’s main claim:
that environmental regulations do not affect all
competitors in a given industry uniformly.
A

B

C

The statement in question is not a
hypothesis; rather, it reports factors that
are known to affect the varying impact of
environmental regulations.
This is too vague to be a good description
of the kind of relationship the question
asks about. The statement in question does
present a contrast—it suggests that larger
plants’ compliance costs are lower under
some circumstances, while smaller plants’
compliance costs are lower under other
circumstances. But this purports to state
two facts rather than mere ideas; they are
contrasting facts but not in any meaningful
sense opposed, since they can easily coexist.
Correct. The statement provides examples
to support the initial claim made in the
passage that regulatory costs fall unevenly on
competitors in an industry: large plants can
spread compliance costs around, and smaller
plants may not even have to pay certain costs.

7.6 Reading Comprehension Answer Explanations

D

E

This statement helps to dispel, not
exemplify, a misconception mentioned
earlier in the passage—i.e., the myth
that environmental regulations affect all
companies in an industry the same way.
The statement does not suggest that the
situation of larger and smaller plants is
similar (or analogous) to any other situation
mentioned in the passage.

C

D

E
The correct answer is C.
477. The primary purpose of the passage is to
(A)

address a widespread environmental
management problem and suggest possible
solutions

(B)

illustrate varying levels of compliance with
environmental regulation among different
corporations

(C)

describe the various alternatives to traditional
methods of environmental management

(D)

advocate increased corporate compliance with
environmental regulation

(E)

correct a common misconception about the
impact of environmental regulations

Main idea
This question depends on understanding the
passage as a whole. Its first sentence indicates
its main purpose: to dispel a myth about
environmental regulations that is often taken as
fact.
A

B

The passage is not about the management
of any environmental problem, which would
be a problem about how to prevent or undo
damage to the environment. The passage
primarily aims to dispel a belief that the
passage says is widely held by environmental
managers.
The passage refers to variations in firms’
levels of compliance with environmental
regulations, but its primary purpose is not
to illustrate those varying levels, nor does
it do so.

The passage suggests that most
environmental managers are mistaken
about a key concept; its primary purpose
is not to describe traditional methods of
environmental management or alternatives to
those traditional methods, nor does it do so.
The passage takes no position on whether
companies should increase their compliance
with environmental regulation.
Correct. The passage primarily aims
to dispel the belief that environmental
regulations affect all companies in an
industry uniformly.

The correct answer is E.
Questions 478–483 refer to the passage on page 398.
478. In the passage, the author is primarily interested in
(A)

suggesting an alternative to an outdated
research method

(B)

introducing a new research method that calls an
accepted theory into question

(C)

emphasizing the instability of data gathered from
the application of a new scientific method

(D)

presenting a theory and describing a new
method to test that theory

(E)

initiating a debate about a widely accepted
theory

Main idea
This question concerns the main point of the
passage. A careful examination of the overall
structure of the passage will reveal the main
point. In the first paragraph, the author briefly
presents Milankovitch’s theory and explains why
it could not be tested early on. In the second
and third paragraphs, the author describes how
a new method allows testing of the theory and
shows how evidence from the testing supports the
theory. While the final paragraph acknowledges
that other factors should be considered, the
author’s primary interest in this passage is in
presenting Milankovitch’s theory and the recently
discovered method for testing it.

465

GMAT® Official Guide 2018

A
B

C
D

E

A new research method is described, but no
previous method is discussed.
As described in the passage, the new
method tests and confirms the theory; there
is no mention that the theory is accepted or
that the method casts doubt on it.
Nothing in the passage suggests that
“instability of data” is an issue.
Correct. The author presents Milankovitch’s
theory and describes the oxygen isotope
method of testing it.
The theory is nowhere said to be “widely
accepted” and the author does not debate
the theory.

A

B

C

The correct answer is D.
D
479. The author of the passage would be most likely to
agree with which of the following statements about the
Milankovitch theory?
(A)

It is the only possible explanation for the ice
ages.

(B)

It is too limited to provide a plausible explanation
for the ice ages, despite recent research
findings.

(C)

It cannot be tested and confirmed until further
research on volcanic activity is done.

(D)

It is one plausible explanation, though not the
only one, for the ice ages.

(E)

It is not a plausible explanation for the ice ages,
although it has opened up promising possibilities
for future research.

E

In the last paragraph, the author suggests
that because there are still other untested
factors that may have effects on climate,
other explanations are possible.
Though in the last paragraph the author
points to other factors that may be involved,
these are not presented by the author
as indicating limitations that diminish
the plausibility of the theory—they are
acknowledged merely as possibilities that
are not now understood—and nothing else
in the passage suggests that the theory is
“too limited.”
The author shows how the theory has been
tested; volcanic activity is not part of this
theory.
Correct. The author’s presentation of the
theory and the tests of the theory show that
the author finds the theory plausible; the
mention of other factors shows the author
does not think that all other explanations
have been ruled out, even if they are as yet
untested.
The theory was a plausible explanation from
its beginning, but it was not testable until
recently; scientists would be unlikely to try
to devise means to test a theory that did not
strike them as antecedently plausible.

The correct answer is D.
480. It can be inferred from the passage that the isotope
record taken from ocean sediments would be less
useful to researchers if which of the following were
true?

Application
The author’s reaction to the statements about the
Milankovitch theory must be based on how the
author treats the theory in the passage. The first,
second, and third paragraphs describe the theory
and the use of a new research method to test the
theory. The passage states that data from these
tests have established a strong connection between
variations in the Earth’s orbit and the periodicity
of the ice ages, suggesting that the author of the
passage believes the theory is plausible. In the
final paragraph, the author points to other factors
that might be involved, suggesting that the theory
might not provide a complete explanation.

466

(A)

It indicated that lighter isotopes of oxygen
predominated at certain times.

(B)

It had far more gaps in its sequence than the
record taken from rocks on land.

(C)

It indicated that climate shifts did not occur
every 100,000 years.

(D)

It indicated that the ratios of oxygen 16 and
oxygen 18 in ocean water were not consistent
with those found in fresh water.

(E)

It stretched back for only a million years.

7.6 Reading Comprehension Answer Explanations

Inference
To make an inference about the isotope record
from ocean sediments, examine what the passage
says about that record. The third paragraph
discusses that record and lists its two advantages.
First, it is a global record with remarkably little
variation in samples from varied locations.
Second, it is more continuous than the record from
rocks. If either of these advantages were not true,
then it is logical to infer that the record would be
less useful.
A

B

C

D

E

According to lines 14–16, the lighter isotope
does predominate; this is part of the record
and does not affect its usefulness.
Correct. In lines 37–42, the author states
that an advantage of the ocean record is that
it is a more continuous record than that taken
from rocks on land. If this were not true, the
ocean record would be less useful.
If the record were to show that the
shifts did not occur every 100,000 years,
Milankovitch’s theory would be weakened.
This impact on the theory does not make
the isotope record less useful to researchers.
The record is useful precisely because it can
offer evidence to confirm or refute such
theories.
This inconsistency would not affect the
usefulness of the ocean‐water record.
Researchers would simply need to
accommodate the fresh‐water inconsistency.
The record would still be useful. Lines
42–46 attest to the establishment of a
pattern based on data from the past several
hundred thousand years.

The correct answer is B.
481. According to the passage, which of the following is true
of the ratios of oxygen isotopes in ocean sediments?
(A)

They indicate that sediments found during an
ice age contain more calcium carbonate than
sediments formed at other times.

(B)

They are less reliable than the evidence from rocks
on land in determining the volume of land ice.

(C)

They can be used to deduce the relative volume
of land ice that was present when the sediment
was laid down.

(D)

They are more unpredictable during an ice age
than in other climatic conditions.

(E)

They can be used to determine atmospheric
conditions at various times in the past.

Supporting ideas
The phrase according to the passage suggests that
the answer to the question is most likely stated
in the passage. Lines 12–14 state that the relative
volume of land ice can be deduced from the ratio
of oxygen 18 to oxygen 16 in ocean sediments.
A
B

C

D
E

There is no evidence in the passage about
this point.
The ocean record is described in lines 38–39
as more continuous, so it is unlikely to be less
reliable. In any case, reliability is not discussed.
Correct. Lines 12–14 explain that the land‐
ice volume for a given period can be deduced
from the ratio of two oxygen isotopes.
There is no evidence in the passage to
support this statement.
The passage does not discuss the use of this
record in determining past atmospheric
conditions.

The correct answer is C.
482. It can be inferred from the passage that precipitation
formed from evaporated ocean water has
(A)

the same isotopic ratio as ocean water

(B)

less oxygen 18 than does ocean water

(C)

less oxygen 18 than has the ice contained in
continental ice sheets

(D)

a different isotopic composition than has
precipitation formed from water on land

(E)

more oxygen 16 than has precipitation formed
from fresh water

Inference
Any inference about precipitation from evaporated
ocean water needs to be based on what the passage
says. Lines 20–22 show that heavier isotopes tend to
be left behind when water evaporates from the ocean
surfaces. Therefore, the evaporated water would
contain less oxygen 18 and the remaining ocean
water would contain more. It is logical to infer
that precipitation formed from this evaporated
water would also contain less oxygen 18.
467

GMAT® Official Guide 2018

A

B

C

D
E

Lines 20–24 explain that the water
remaining in the ocean after evaporation has
more oxygen 18.
Correct. Since the heavier isotopes tend
to be left behind, there will be less oxygen
18 in the evaporated water and in the
precipitation that forms from it.
The passage suggests that the ocean
water evaporates and through subsequent
precipitation helps form the ice sheets, so
the amount of oxygen 18 in the ice sheets
should be similar to the amount in the
precipitation formed from the evaporated
water.
The passage does not discuss precipitation
formed from water on land.
The passage does not discuss precipitation
formed from fresh water.

The correct answer is B.

A

B

C

D

483. It can be inferred from the passage that calcium
carbonate shells
(A)

are not as susceptible to deterioration as rocks

(B)

are less common in sediments formed during an
ice age

(C)

are found only in areas that were once covered
by land ice

(D)

contain radioactive material that can be used to
determine a sediment’s isotopic composition

(E)

reflect the isotopic composition of the water at
the time the shells were formed

Inference
Any inference about calcium carbonate shells
needs to be based on what the passage says about
these shells. Lines 24–32 explain the role of these
shells in forming sediments and establishing a
chronology for ice ages. The shells were constructed
with oxygen atoms drawn from the surrounding
ocean. Lines 29–32 make it clear that if the
sediments reveal a higher ratio of oxygen 18, it
is because more oxygen 18 had been left behind
when the ocean water evaporated and contributed
to the growth of continental ice sheets. It can
thus be inferred that the shells that make up
those sediments must reflect the proportion of
oxygen 18 found in the ocean water at the time
they were formed.
468

E

The only mention of rocks in the passage
is a comparison of “gappiness” of the rock
and sedimentary specimen records in lines
38–39; this information does not allow
any firm inference to be made with respect
to relative susceptibility to deterioration,
though a more continuous record might
be the result of less susceptibility to
deterioration.
The passage does not make any reference to
the relative abundance of these shells during
ice ages; no such inference can be drawn.
The only information in the passage that
might support this statement is found in
lines 29–32, but that information, about
the correlation between oxygen ratios in
sediment specimens and land ice, describes
a relation that implies nothing about
distributions of such specimens.
Though the passage does indirectly indicate
that the shells contained radioactive
material, nothing in the passage suggests
that radioactive material is used to
determine isotopic composition.
Correct. The passage explains that oxygen
atoms in the surrounding water are one of
the building blocks of calcium carbonate
shells. The isotopic composition of the
surrounding water changes during the ice
age cycles, so it is logical that the isotopic
composition of the shells will change
depending on when they were formed.

The correct answer is E.
Questions 484–489 refer to the passage on page 400.
484. The primary purpose of the passage is to
(A)

examine two sides of a historiographical debate

(B)

call into question an author’s approach to a
historiographical debate

(C)

examine one author’s approach to a
historiographical debate

(D)

discuss two authors’ works in relationship to a
historiographical debate

(E)

explain the prevalent perspective on a
historiographical debate

7.6 Reading Comprehension Answer Explanations

Main idea

Evaluation

This question requires understanding what the
passage as a whole is attempting to do. The passage
opens by introducing two books published in
1984 that both concern the history of women in
the United States. The passage then makes it clear
that one book deals directly (line 15) with the issue
of women’s status, while the other does not. The
passage then goes on to discuss the perspective
that each book takes and what each book has to
offer for an assessment of women’s status in the
eighteenth and nineteenth centuries.

Answering this question depends on
understanding what role a particular piece of
information plays in the passage as a whole. The
author implicitly supports Lebsock’s contention
(beginning at line 20) that different frames of
reference can produce different perspectives
on the debate about women’s status in the
eighteenth and nineteenth centuries. The author
then summarizes different contexts cited by
Lebsock to support the contention about frames
of reference. As part of this summary, the author
refers to supervising schools (lines 24–25) as an
example of a job that apparently showed women
losing power.

A

B

C
D

E

The two books discussed in the passage
do not take different sides on a particular
debate but rather are described as being
more or less useful to the debate itself.
The passage focuses on how two different
books contain information useful to a particular
historiographical debate but does not call into
question the approach of either book.
The passage focuses on two authors’ works,
not one.
Correct. The passage discusses what two
different books have to offer in relation to a
particular historiographical debate.
The passage does not describe any
perspective on a particular historiographical
debate as being more prevalent than any
other.

A
B

C
D

E

The correct answer is D.
485. The author of the passage mentions the supervision of
schools primarily in order to
(A)

remind readers of the role education played in
the cultural changes of the nineteenth century in
the United States

(B)

suggest an area in which nineteenth‐century
American women were relatively free to exercise
power

(C)

provide an example of an occupation for which
accurate data about women’s participation are
difficult to obtain

(D)

speculate about which occupations were
considered suitable for United States women of
the nineteenth century

(E)

illustrate how the answers to questions about
women’s status depend on particular contexts

The passage does not discuss the role of
education in the nineteenth century.
The passage does mention some ways in
which, according to Lebsock, women …
gained power (lines 25–26) in the nineteenth
century, but supervising schools is not among
them.
The passage does not discuss the difficulty of
obtaining data about particular occupations.
The passage makes no judgments about
the suitability for women of any jobs in the
nineteenth century.
Correct. The passage mentions supervising
schools as part of an illustration of Lebsock’s
claim that the debate about women’s status
depends on the context being examined.

The correct answer is E.
486. With which of the following characterizations of
Lebsock’s contribution to the controversy concerning
women’s status in the nineteenth‐century United States
would the author of the passage be most likely to agree?
(A)

Lebsock has studied women from a formerly
neglected region and time period.

(B)

Lebsock has demonstrated the importance of
frame of reference in answering questions about
women’s status.

(C)

Lebsock has addressed the controversy by using
women’s current status as a frame of reference.

(D)

Lebsock has analyzed statistics about
occupations and property that were previously
ignored.
469

GMAT® Official Guide 2018

(E)

Lebsock has applied recent historiographical
methods to the biography of a nineteenth‐century
woman.

addresses larger historiographical issues

(E)

fails to provide sufficient material to support its
claims

Supporting ideas

Supporting ideas

Answering this question requires recognizing
information explicitly given in the passage.
The passage introduces the work of Lebsock
in line 6 and then goes on to describe several
characteristics of Lebsock’s book. In lines 20–22,
the author introduces Lebsock’s claim that the
historiographical debate about women’s status
is dependent on frame of reference and calls
that claim important; the passage then gives an
example showing how frame of reference affects
views of women’s status. In so doing, the author
displays an implicit agreement with Lebsock’s
discussion on this point.

This question asks for recognition of information
contained in the passage. In the first sentence,
the passage states that Buel and Buel’s work
and Lebsock’s work have contrasting approaches.
The passage then proceeds, using descriptions of
each work’s approach, to illustrate how the works
differ. The passage notes that Buel and Buel’s
work makes little effort to place its biographical
subject in the context of recent historiography on
women (lines 5–6), whereas Lebsock’s work
attempts to redirect two decades of historiographical
debate about women’s status.

A

B

C

D

E

The author of the passage portrays neither
the place nor time period that Lebsock
focuses on as having been neglected by
historians.
Correct. The author describes as important
Lebsock’s idea that frame of reference
informs the debate about women’s status.
According to the passage, Lebsock’s book
deals with women’s status in the eighteenth
and nineteenth centuries, not the present
status of women.
The passage does not mention or imply that
Lebsock analyzed statistics in writing her
book.
Although the passage does describe
Lebsock’s book as pertaining to an ongoing
historiographical debate, it identifies the
book’s topic as women in one southern
community (lines 7–8), not the life of a
single woman.

The correct answer is B.
487. According to the passage, Lebsock’s work differs from
Buel and Buel’s work in that Lebsock’s work

470

(D)

(A)

uses a large number of primary sources

(B)

ignores issues of women’s legal status

(C)

refuses to take a position on women’s status in
the eighteenth century

A
B
C

D

E

Primary sources are not mentioned in the
passage in relation to either work discussed.
The legal status of women is not mentioned
in the passage.
Lebsock’s work is described in the passage
as attempting to redirect the debate about
women’s status in the eighteenth and
nineteenth centuries.
Correct. The passage suggests that by
not placing its subject’s story in the
context of historiography, Buel and Buel’s
work does not therefore address larger
historiographical issues, as Lebsock’s does.
The passage tends to support Lebsock’s
views and does not refer to any lack of
support for the claims made in Lebsock’s
work.

The correct answer is D.
488. The passage suggests that Lebsock believes that
compared to nineteenth‐century American women,
eighteenth‐century American women were
(A)

in many respects less powerful in relation to men

(B)

more likely to own real estate

(C)

generally more economically independent

(D)

more independent in conducting their private
lives

(E)

less likely to work as school superintendents

7.6 Reading Comprehension Answer Explanations

Inference
This question requires making an inference
based on information given in the passage. As
part of the passage’s description of Lebsock’s
contribution to the historiographical debate
about women’s status in the eighteenth and
nineteenth centuries, Lebsock’s conclusions
about women’s autonomy are described. As
part of this description, the passage cites
Lebsock’s conclusion that nineteenth‐century
women lost economic autonomy when
compared to eighteenth‐century women
(lines 17–20).
A

B
C

D

E

The passage states that in many ways
women in the nineteenth century lost power
in relation to men (line 23), which would
imply that in those respects eighteenthcentury women had more power in
relation to men, not less. The only increase
mentioned in nineteenth‐century women’s
power is associated with owning more real
estate.
The passage states that more nineteenthcentury women owned real estate.
Correct. As the passage states, Lebsock
concluded that nineteenth‐century women
lost economic autonomy compared to
eighteenth‐century women.
The passage states that nineteenth‐century
women gained more independence in their
private lives.
The passage cites school superintendents
as an example of an occupation more
likely to be held by eighteenth‐century
women.

The correct answer is C.

(D)

Women had more economic autonomy in the
colonial era than in the nineteenth century.

(E)

Women’s occupations were generally more
respected in the colonial era than in the
nineteenth century.

Inference
This question requires understanding what the
passage implies. The approach that Buel and
Buel’s work takes is specifically described in lines
3–6 and again in lines 28–34. In lines 29–32, the
passage states that Buel and Buel’s work provides
ample raw material for questioning the myth … of a
colonial golden age in the eighteenth century, referring
to a myth about women’s status. In describing this
golden age as a myth fostered by some historians,
the passage suggests that this era was not as
favorable to women as these historians suggest.
A

B
C

D

E

The passage describes Lebsock’s work as
providing such evidence, not Buel and Buel’s
work.
The passage does not pertain to the level of
concern women had for their status.
Correct. The final paragraph of the passage
describes Buel and Buel’s work as providing
material that calls into question claims
that the eighteenth century was especially
favorable to women.
The passage refers to the economic
autonomy of women in relation to Lebsock’s
work, not Buel and Buel’s work.
The passage does not refer to whether any
particular occupations held by women were
more respected at one time or another.

The correct answer is C.
Questions 490–492 refer to the passage on page 402.

489. The passage suggests that Buel and Buel’s biography
of Mary Fish provides evidence for which of the
following views of women’s history?
(A)

Women have lost power in relation to men since
the colonial era.

(B)

Women of the colonial era were not as likely to
be concerned with their status as were women in
the nineteenth century.

(C)

The colonial era was not as favorable for women
as some historians have believed.

490. The passage suggests that WIDC differed from WTUL
in which of the following ways?
(A)

WIDC believed that the existing safety regulations
were adequate to protect women’s health,
whereas WTUL believed that such regulations
needed to be strengthened.

(B)

WIDC believed that unions could not succeed
in pressuring employers to comply with such
regulations, whereas WTUL believed that unions
could succeed in doing so.
471

GMAT® Official Guide 2018

(C)

(D)

(E)

WIDC believed that lead poisoning in white
lead factories could be avoided by controlling
conditions there, whereas WTUL believed that
lead poisoning in such factories could not
be avoided no matter how stringently safety
regulations were enforced.
At the time that the legislation concerning white
lead factories was proposed, WIDC was primarily
concerned with addressing health conditions
in white lead factories, whereas WTUL was
concerned with improving working conditions in
all types of factories.

D

E

At the time that WIDC was opposing legislative
attempts to restrict women’s labor, WTUL had
already ceased to do so.

Inference
To answer this question you need to understand
the differences between WIDC and WTUL
as they are described in the passage. The only
information about WTUL in the passage is that
it had stopped opposing restrictions on women’s
labor in the late 1880s, and that, because existing
safety regulations were not being enforced, it
supported the proposal to prohibit women from
working in white lead factories. WIDC, on the
other hand, was formed in 1892 specifically
to oppose restrictions on women’s labor, and it
opposed the proposal.
A

B

C

472

According to the passage, WIDC did
believe that existing safety regulations, if
enforced, could prevent lead poisoning.
WTUL may or may not have believed
that the safety regulations needed to be
strengthened; all the passage states is that
WTUL did not believe that the safety
regulations were likely to be enforced.
The passage states that WTUL believed that
because there were no unions to pressure
employers, the employers would not comply
with safety regulations. The passage does not
present any information on which to base a
conclusion about WIDC’s beliefs regarding
union pressure on employers.
Based on information in the passage, both
WIDC and SPEW believed that enforcing
safety regulations could protect women
against lead poisoning. WIDC supported
SPEW’s position on the matter. WTUL

believed that safety regulations were
unlikely to be enforced because of the lack
of unions.
The passage states that WIDC viewed the
proposal to restrict women’s employment
in white lead factories as an instance of
legislation designed to limit women’s work
opportunities—precisely the legislation
that WIDC was formed to oppose. Thus,
WIDC was not primarily concerned with
the factories’ health conditions.
Correct. WIDC began opposing legislative
attempts to restrict women’s labor in 1892
and continued to do so through at least
1895, when the Home Secretary proposed
prohibiting women from working in white
lead factories. WTUL stopped opposing
restrictions on women’s labor in the late
1880s, before WIDC was even founded.
Thus, the passage suggests that WTUL had
stopped opposing restrictions on women’s
labor well before WIDC worked to oppose
such legislation.

The correct answer is E.
491. Which of the following, if true, would most clearly
support the contention attributed to SPEW in lines
17–20 ?
(A)

Those white lead factories that most strongly
enforced regulations concerning worker safety
and hygiene had the lowest incidences of lead
poisoning among employees.

(B)

The incidence of lead poisoning was much
higher among women who worked in white lead
factories than among women who worked in
other types of factories.

(C)

There were many household sources of lead
that could have contributed to the incidence of
lead poisoning among women who also worked
outside the home in the late nineteenth century.

(D)

White lead factories were more stringent than
were certain other types of factories in their
enforcement of workplace safety regulations.

(E)

Even brief exposure to the conditions typically
found in white lead factories could cause lead
poisoning among factory workers.

7.6 Reading Comprehension Answer Explanations

Evaluation
This question requires the reader to find a
statement that would provide additional support
for the contention made in the following
statement: SPEW contended, and WIDC concurred,
that controllable conditions in such factories were
responsible for the development of lead poisoning.
Information suggesting that when conditions were
controlled, lead poisoning was less likely to develop
would provide support for SPEW’s contention.
A

B

C

D

E

Correct. If incidences of lead poisoning were
low in those factories that enforced hygiene
and safety regulations, that would suggest
that lead poisoning was not an inevitable
result of working in a white lead factory—but
rather that lead poisoning was the result of
poor hygiene and safety practices.
It would not be particularly surprising
for the incidence of lead poisoning to be
higher among women working in white
lead factories than among women working
in other kinds of factories—but such a
finding would say nothing about whether
controllable conditions had any effect on the
development of lead poisoning.
The existence of household sources
of lead that might contribute to lead
poisoning would weaken, not support,
SPEW’s contention that controllable
factory conditions were responsible for the
development of lead poisoning.
If white lead factories enforced workplace
safety regulations more stringently than did
some other types of factories, it might be the
case that SPEW’s contention was incorrect:
that even controlled conditions could not
prevent a high incidence of lead poisoning.
If the conditions typically found in white
lead factories were particularly bad with
regard to safety and hygiene, it could
conceivably be the case that SPEW’s
contention was true—that is, that the
conditions that caused lead poisoning
were controllable. But it might also be the
case that an uncontrollable aspect of those
conditions caused lead poisoning. Thus, this
neither supports nor undermines SPEW’s
contention clearly.

492. The passage is primarily concerned with
(A)

presenting various groups’ views of the motives
of those proposing certain legislation

(B)

contrasting the reasoning of various groups
concerning their positions on certain proposed
legislation

(C)

tracing the process whereby certain proposed
legislation was eventually enacted

(D)

assessing the success of tactics adopted by
various groups with respect to certain proposed
legislation

(E)

evaluating the arguments of various groups
concerning certain proposed legislation

Main idea
Answering this question depends on identifying
the overall point of the passage. The passage is
mainly concerned with explaining the reasons
behind the positions taken by WIDC and SPEW,
which opposed the proposal to enact legislation
prohibiting women from holding most white lead
factory jobs, and the reasoning of WTUL, which
supported the proposal.
A

B

C

D

The passage explains how WIDC viewed
the proposal, but it does not indicate
what any of the groups believed about the
motivations of the Home Secretary, who
made the proposal.
Correct. The passage contrasts the
reasoning of the WIDC and SPEW, both
of which believed that enforcing safety
regulations would make the proposed
legislation unnecessary, with the reasoning
of WTUL, which thought that safety
regulations were unlikely to be enforced and
thus supported the proposal.
The passage simply states that the proposal
was eventually enacted; it does not trace the
process by which this occurred.
The passage implies that WIDC and SPEW
were unsuccessful in their opposition to the
proposed legislation, but it identifies only
one tactic used in opposition to it: SPEW’s
attempt to challenge it by investigating the
causes of lead poisoning.

The correct answer is A.
473

GMAT® Official Guide 2018

E

The passage does not evaluate the groups’
arguments concerning the proposed
legislation; rather, it presents those
arguments without comment on their
quality or value.

The correct answer is B.

D

E

Questions 493–498 refer to the passage on page 404.

493. The author of the passage refers to Robert Filmer (see
line 9) primarily in order to

The correct answer is D.
494. The passage suggests which of the following about
the seventeenth‐century English women mentioned
in line 2?

(A)

show that Royalist ideology was somewhat more
radical than most historians appear to realize

(B)

qualify the claim that patriarchalism formed the
basis of Royalist ideology

(A)

(C)

question the view that most early feminists were
associated with the Royalist faction

Their status as forerunners of modern feminism
is not entirely justified.

(B)

(D)

highlight an apparent tension between Royalist
ideology and the ideas of early feminists

They did not openly challenge the radical
patriarchalism of Royalist Filmerian ideology.

(C)

argue that Royalists held conflicting opinions
on issues of family organization and women’s
political rights

Cavendish was the first among these women to
criticize women’s subordination in marriage and
assert women’s equality with men.

(D)

Their views on family organization and women’s
political rights were diametrically opposed to those
of both Royalist and Parliamentarian ideology.

(E)

Historians would be less puzzled if more of them
were identified with the Parliamentarian side in
the English Civil Wars.

(E)

Evaluation
This question asks about the role of Filmer in the
passage. The author states that Filmer’s radical
patriarchalism is associated with Royalist ideology
and then goes on to define radical patriarchalism
as an ideology that asserts the power of the
king and the male head of the household. Early
feminists, however, questioned the subordination
of women in marriage. Thus, there seems to be a
conflict between these two sets of ideas.
A

B

C

474

Correct. There is apparent tension between
Filmer’s radical patriarchalism, if that is
indeed essential to Royalist ideology, and
the ideas of early feminists, who questioned
such patriarchalism.
The author refers to Filmer in order to
suggest, initially, a uniformity among
Royalists regarding family and women; it
is only later in the passage that this view
becomes more complicated.

Although the passage refers to Filmer’s
view as radical patriarchalism, it provides no
evidence regarding any differences in the
degrees to which historians consider that
view, or Royalism in general, to be radical.
Filmer’s work supports the claim that
patriarchalism was the basis of Royalist
ideology; it does not qualify such a claim.
That Filmer’s approach was one of radical
patriarchalism makes it surprising that early
feminists were associated with the Royalist
faction, but it does not provide any grounds
for questioning whether they were so
associated.

Inference
The first sentence of the passage refers to women
who are both regarded as forerunners of modern
feminism and identified as Royalists. The passage
goes on to suggest that, given Royalist ideology’s
association with Filmer’s radical patriarchalism
(equating absolute power of the king with
absolute power of the male head of household),
it is surprising that feminism would find any
footing within such an ideology.
A

B

Nothing in the passage disputes the idea
that the seventeenth‐century English
women in question should be considered
the forerunners of modern feminism.
Gallagher provides the example of Margaret
Cavendish as a writer who did openly
challenge radical patriarchalism—albeit only
in her writings.

7.6 Reading Comprehension Answer Explanations

C

D

E

The passage states that Cavendish had
successors among early feminists, but it does
not indicate whether she herself was the
first seventeenth‐century English woman to
assert women’s equality.
The passage does not indicate what the
Parliamentarian view of family organization
and women’s political rights was, so there is
no way to determine whether the Royalist
forerunners of modern feminism were
opposed to that view.
Correct. The basic puzzle the passage
sets out to solve is why the forerunners
of modern feminism would have been
associated with the Royalist side, which
seems to have been based on radical
patriarchalism. Historians would most likely
have been less surprised if these women had
been identified with the Parliamentarian
side, which presumably did not embrace
radical patriarchalism.

A

B

C

D

E

The correct answer is E.
495. The passage suggests that Margaret Cavendish’s
decision to become an author was motivated, at least
in part, by a desire to
(A)

justify her support for the Royalist cause

(B)

encourage her readers to work toward
eradicating Filmerian patriarchalism

(C)

persuade other women to break free from their
political and social isolation

(D)

analyze the causes for women’s exclusion from
the pursuit of power

(E)

create a world over which she could exercise
total control

Inference
This question asks about Margaret Cavendish’s
reasons for becoming an author. The second
paragraph describes her as someone who insisted
that she was a self‐sufficient being; she understood
that, given the real‐world strictures in place, she
could achieve this self‐sufficiency in her own
mind and on paper as a writer. So her decision to
become a writer can be inferred to be motivated
by her desire to exercise power and control.

The passage states that Cavendish justified
her being the center of her own universe by
invoking the Royalist figure of the absolute
monarch; there is no suggestion in the
passage that Cavendish felt the need to
justify any support for the actual Royalist
cause.
The passage gives no direct indication that
Cavendish was even aware of Filmerian
patriarchalism.
The second paragraph states that
Cavendish’s idea of absolute singularity
carried with it the idea of social and political
isolation; Cavendish was most likely not
motivated by a desire to persuade other
women to break free from such isolation.
Cavendish took the exclusion of women
from the pursuit of power for granted;
the passage does not suggest that she was
concerned with its causes.
Correct. According to the passage,
Cavendish considered herself a self‐
sufficient being who was at the center of her
own universe; in her writing, she wanted to
create a world in which this was also true.

The correct answer is E.
496. The phrase “a satellite orbiting a dominant male planet”
(lines 41–42) refers most directly to
(A)

Cavendish’s concept that each woman is a
sovereign self

(B)

the complete political and social isolation of
absolute singularity

(C)

the immaterial world that a writer can create on
paper

(D)

the absolute subordination of women in a
patriarchal society

(E)

the metaphorical figure of the absolute monarch

Evaluation
The phrase in question is a satellite orbiting a
dominant male planet. The passage states that this
was the idea that Cavendish was reacting against;
she preferred instead the idea that she was the
center of her own universe, her own sovereign,
subject to no one.

475

GMAT® Official Guide 2018

A

B
C

D

E

The idea of a satellite orbiting a dominant
male planet refers not to Cavendish’s idea
that each woman is a sovereign self, but
rather to the idea directly opposed to that:
each woman must submit to a dominant
male.
A satellite orbiting a dominant male planet is
by definition not isolated, nor is it singular.
According to the passage, Cavendish wished
to create her own world as a writer so that
she did not have to be a satellite.
Correct. The phrase refers to the idea
that in a patriarchal society, women are as
satellites to men, who are the dominant
planets.
While radical patriarchy does equate
the monarch with the male head of the
household, the in question phrase is most
directly about the relationship, under
patriarchy, between women and men.

A
B

C

D

The correct answer is D.
497. The primary purpose of the passage is to
(A)

trace the historical roots of a modern
sociopolitical movement

(B)

present one scholar’s explanation for a puzzling
historical phenomenon

(C)

contrast two interpretations of the ideological
origins of a political conflict

(D)

establish a link between the ideology of
an influential political theorist and that of a
notoriously eccentric writer

(E)

call attention to some points of agreement
between opposing sides in an ideological debate

E

The correct answer is B.
498. Which of the following, if true, would most clearly
undermine Gallagher’s explanation of the link between
Royalism and feminism?
(A)

Because of their privileged backgrounds,
Royalist women were generally better educated
than were their Parliamentarian counterparts.

(B)

Filmer himself had read some of Cavendish’s
early writings and was highly critical of her ideas.

(C)

Cavendish’s views were highly individual and
were not shared by the other Royalist women
who wrote early feminist works.

(D)

The Royalist and Parliamentarian ideologies
were largely in agreement on issues of family
organization and women’s political rights.

(E)

The Royalist side included a sizable minority
faction that was opposed to the more radical
tendencies of Filmerian patriarchalism.

Main idea
This question asks about the passage as a
whole. The passage is mainly concerned with
outlining Catherine Gallagher’s attempt to
explain why, given Royalist ideology’s apparent
association with radical patriarchalism, Royalist
women offered feminist critiques of women’s
subordination in marriage and asserted their
equality with men.

476

The passage makes no connection between
early feminism and its modern form.
Correct. The passage presents a puzzling
historical phenomenon, that Royalist
women critiqued patriarchalism, in
the first paragraph, and then presents
Catherine Gallagher’s explanation for that
phenomenon in the second paragraph.
While the passage discusses the political
conflict between the Royalists and
Parliamentarians in the English Civil Wars
in the first paragraph, neither this conflict,
nor its ideological origins are the focus of
the passage. Furthermore, the passage does
not offer any interpretations of the origins
of the conflict.
The passage attempts to unlink the ideology
of political theorist Robert Filmer and
the eccentric author Margaret Cavendish
by suggesting that Filmer’s radical
patriarchalism was not the only way of
understanding Royalist ideology. Cavendish
provided a different understanding entirely.
While both sides of the ideological debate
did agree on the absolute monarchy, the
passage as a whole does not focus on this
agreement, but rather on the disagreement
about where, theoretically, the idea of
absolute monarchy leads.

7.6 Reading Comprehension Answer Explanations

Inference
This question asks about how to undermine the
way in which Gallagher connects Royalism and
feminism. According to Gallagher, Cavendish’s
work exemplifies the connection between these
ideas, because Cavendish took the idea of
absolute monarchy and extended that to the idea
of absolute self, an idea that should, Cavendish
believed, apply to women as well as men.
A

B

C

D

E

Gallagher’s explanation of the link between
Royalism and feminism does not depend
on the education level of Royalist women
relative to Parliamentarian women.
Filmer most likely would have been critical
of Cavendish’s ideas, had he encountered
them, but the passage does not indicate
that Gallagher’s argument had anything to
do with whether Filmer read Cavendish’s
writings.
Correct. Gallagher uses Cavendish’s
work to explain how Royalism gave rise
to feminism, but if Cavendish’s views
were completely atypical of other Royalist
women, then those views cannot explain the
link as Gallagher suggests they do.
The passage states in the first paragraph that
if the Royalists and Parliamentarians were
in agreement on issues of family organization
and women’s political rights, then feminists
should have been divided between the two
sides—but they were not. So this idea, if
true, would undermine that statement, but
not Gallagher’s argument about the link
between Royalists and feminists.
If more Royalists were opposed to Filmer’s
radical patriarchalism, then Cavendish’s
writings would seem to be more
representative of tendencies in Royalist
ideology, thus making Gallagher’s case
stronger, not weaker.

The correct answer is C.

Questions 499–503 refer to the passage on page 406.

499. The passage is primarily concerned with
(A)

identifying two practices in medical research that
may affect the accuracy of clinical trials

(B)

describing aspects of medical research that tend
to drive up costs

(C)

evaluating an analysis of certain shortcomings of
current medical research practices

(D)

describing proposed changes to the ways in
which clinical trials are conducted

(E)

explaining how medical researchers have
traditionally conducted clinical trials and how
such trials are likely to change

Main idea
This question requires an understanding of what
the passage as a whole is doing. The passage
introduces Frazier and Mosteller as proposing
changes to the ways clinical trials in medical
research are currently conducted. The rest of the
passage then describes these proposed changes
together with the support Frazier and Mosteller
provide for adopting these changes.
A

B

C

D

E

The passage identifies practices in medical
research to help illustrate the basis for
Frazier and Mosteller’s proposed changes.
The passage mentions medical research costs
as one example within the larger description
of Frazier and Mosteller’s proposed changes.
The passage is not concerned with
evaluating Frazier and Mosteller’s proposed
changes.
Correct. The passage describes the changes
proposed by Frazier and Mosteller to the
way clinical trials are conducted.
The passage is not concerned with
establishing the likelihood of any changes to
the way medical research is conducted.

The correct answer is D.

477

GMAT® Official Guide 2018

500. Which of the following can be inferred from the
passage about a study of the category of patients
referred to in lines 20–22 ?

501. It can be inferred from the passage that a study limited
to patients like those mentioned in lines 20–22 would
have which of the following advantages over the kind
of study proposed by Frazier and Mosteller?

(A)

Its findings might have limited applicability.

(B)

It would be prohibitively expensive in its attempt
to create ideal conditions.

(A)

It would yield more data and its findings would be
more accurate.

(C)

It would be the best way to sample the total
population of potential patients.

(B)

It would cost less in the long term, though it
would be more expensive in its initial stages.

(D)

It would allow researchers to limit information
collection without increasing the risk that
important variables could be overlooked.

(C)

It would limit the number of variables researchers
would need to consider when evaluating the
treatment under study.

(E)

Its findings would be more accurate if it
concerned treatments for a progressive
disease than if it concerned treatments for a
nonprogressive disease.

(D)

It would help researchers to identify subgroups
of patients with secondary conditions that might
also be treatable.

(E)

It would enable researchers to assess the value of
an experimental treatment for the average patient.

Inference
This question requires drawing an inference from
information given in the passage. In describing the
proposals put forth by Frazier and Mosteller, the
passage states in lines 15–20 that they propose using
more patients in clinical trials than are currently
being used, and that the trials would thereby obtain
a more representative sample of the total population
with the disease under study. The passage then states
that researchers often restrict (lines 20–22) their
trials to certain types of patients, therefore limiting
the applicability of their findings.
A

B

C

D

E

Correct. The passage states that the
researchers preferred to restrict the types of
patients used in their studies, thereby using
a less representative sample than if they
used a more inclusive group of patients.
The passage mentions the added expense of
clinical trials only in relation to data storage,
collection, and analysis.
The passage describes the category of
patients referred to as restricted and therefore
unrepresentative of the total population.
While the passage does mention the amount
of data collected about an individual patient,
that topic is not connected to the category of
patients referred to in lines 20–22.
The passage does not suggest that a study
using the category of patients referred to
would be more effective in investigating
progressive diseases.

The correct answer is A.
478

Inference
This question requires understanding what the
information in the passage implies. The passage
explains that Frazier and Mosteller’s proposal involves
enrolling more patients in clinical trials (lines 18–19)
than is the case with the category of patients referred
to. The passage then explains that broadening the
range of trial participants would allow an evaluation
of particular treatments under various conditions and
for different patient subgroups (line 29). This strongly
suggests that limiting the patients used to those
described in the referred text would limit the number
of variables researchers would need to consider.
A

B

C

D

E

The passage suggests that not limiting the
patients used in clinical trials will yield more
data than restricting them will.
The passage refers to the costs of clinical trials
only as they concern the collection, storage,
and analysis of data collected from participants.
Correct. By limiting the patients used to
those having the ailment under study, the
passage suggests that researchers need to
consider fewer variables in their assessment
of a treatment.
The passage suggests that not limiting the
types of patients used in clinical trials will
better allow researchers to evaluate subgroups.
The passage suggests that limiting the types of
patients available for clinical trials results in data
for specific, rather than average, populations.

The correct answer is C.

7.6 Reading Comprehension Answer Explanations

502. The author mentions patients’ ages (line 32) primarily
in order to
(A)
(B)

identify the most critical variable differentiating
subgroups of patients
cast doubt on the advisability of implementing
Frazier and Mosteller’s proposals about medical
research

(C)

indicate why progressive diseases may require
different treatments at different stages

(D)

illustrate a point about the value of enrolling a
wide range of patients in clinical trials

(E)

substantiate an argument about the problems
inherent in enrolling large numbers of patients in
clinical trials

Evaluation
Answering this question requires understanding
how a particular piece of information functions in
the passage as a whole. The passage is concerned
with describing the proposals of Frazier and
Mosteller. One of these proposals, described in the
second paragraph, involves broadening the range
of participants used in clinical trials. The passage
states that in following this proposal, Frazier
and Mosteller suggest that the effectiveness of
treatments can be assessed for different patient
subgroups. To affirm the value of broadening the
range of participants, the passage then cites two
examples of criteria by which relevant subgroups
might be identified: disease stages and patients’ ages.
A

B

C

D

E

The passage makes no judgment as to the
value of the subgroups it refers to in relation
to broadened participation in clinical trials.
The passage does not call into question
the potential effectiveness of Frazier and
Mosteller’s proposals.
The passage’s example of patients’ ages is
not intended to be causally connected to
its previous example regarding progressive
diseases.
Correct. Patients’ ages are referred to in the
passage to identify subgroups that could
be evaluated if the range of participants in
clinical trials were broadened.
The passage refers to patients’ ages in
support of Frazier and Mosteller’s proposal
that more patients be used in clinical trials.

The correct answer is D.

503. According to the passage, which of the following
describes a result of the way in which researchers
generally conduct clinical trials?
(A)

They expend resources on the storage of
information likely to be irrelevant to the study
they are conducting.

(B)

They sometimes compromise the accuracy of
their findings by collecting and analyzing more
information than is strictly required for their trials.

(C)

They avoid the risk of overlooking variables that
might affect their findings, even though doing so
raises their research costs.

(D)

Because they attempt to analyze too much
information, they overlook facts that could
emerge as relevant to their studies.

(E)

In order to approximate the conditions typical of
medical treatment, they base their methods of
information collection on those used by hospitals.

Supporting ideas
This question asks for an identification of specific
information given in the passage. The passage
describes the proposals of Frazier and Mosteller
as attempting to improve the way clinical trials
have generally been conducted. In describing how
current trials are generally conducted, the passage
states that researchers collect far more background
information on patients than is strictly required for
their trials (lines 4–6) and that they therefore
escalate the costs of the trials.
A

B

C

D

E

Correct. The passage states that researchers
generally collect more information than
they need to perform their clinical trials,
which drives up the costs of the trials.
The passage makes no judgment about the
accuracy of the information collected by
researchers who currently hold clinical trials.
The passage states that the risk of
overlooking relevant information in clinical
trials is never entirely eliminable (line 11).
The passage states that researchers generally
collect more information than is relevant,
not that they overlook relevant information.
The passage states that, in general,
researchers currently collect more
information than hospitals do (line 6).

The correct answer is A.
479

GMAT® Official Guide 2018

Questions 504–509 refer to the passage on page 408.

E

504. The primary purpose of the passage is to
(A)

assess the validity of a certain view

(B)

distinguish between two phenomena

(C)

identify the causes of a problem

(D)

describe a disturbing trend

(E)

allay concern about a particular phenomenon

Main idea
This question requires understanding, in
general terms, the purpose of the passage as a
whole. The first paragraph identifies an area of
concern: declines in amphibian populations may
constitute a crisis, one that indicates humans’
catastrophic effects on the environment. The
rest of the passage then goes on to evaluate, as
the second paragraph states, whether claims
of crisis‐level extinctions as a result of human
activity are valid. In making this evaluation,
the passage discusses the possible causes
of extinctions, biologists’ prioritization of
population declines over extinctions, and the
fact that we lack extensive long‐term data on
amphibian populations.
A

B

C

D

Correct. The passage’s main purpose is to
assess whether the view that humans are
causing crisis‐level declines in amphibian
populations is valid.
The passage takes care, particularly in the
third paragraph, to distinguish between
population declines and extinctions, but this
is not its primary purpose.
The passage makes clear that it is difficult
to identify the real extent of the problem
facing amphibian populations, much less
identify its causes.
The first paragraph notes what may seem
to be a disturbing trend—the decline
in amphibian populations—but the rest
of the passage is concerned not with
describing that trend in greater detail, but
rather with determining whether it is in
fact occurring.

The correct answer is A.
505. It can be inferred from the passage that the author
believes which of the following to be true of the
environmentalists mentioned in lines 5–6 ?
(A)

They have wrongly chosen to focus on anecdotal
reports rather than on the long‐term data that
are currently available concerning amphibians.

(B)

Their recommendations are flawed because
their research focuses too narrowly on a single
category of animal species.

(C)

Their certainty that population declines in general
are caused by environmental degradation is not
warranted.

(D)

They have drawn premature conclusions
concerning a crisis in amphibian populations
from recent reports of declines.

(E)

They have overestimated the effects of chance
events on trends in amphibian populations.

Inference
This question asks about the author’s view of
the environmentalists mentioned in the first
paragraph. These environmentalists have claimed,
based on amphibian population declines, that
the situation is a crisis and that immediate
action must be taken. The author, however,
states that the declines are only apparently
drastic and questions whether they are real,
thus suggesting that the environmentalists are
drawing conclusions in the absence of a complete
consideration of the situation.
A

B

480

While the passage provides possible
grounds for concluding that concern
about declining amphibian populations is
overblown, it concludes by suggesting that
we might, because we lack data, doom
species and ecosystems to extinction. Thus,
the overall purpose is not to allay concern.

The passage indicates that anecdotal
reports are insufficient, but so too are other
resources. The fourth paragraph of the
passage makes clear that there is not enough
long‐term data available on which to base
conclusions about amphibian populations.
The passage does not indicate that the
environmentalists under discussion have
conducted research on any animal species.

7.6 Reading Comprehension Answer Explanations

C

D

E

The passage does not indicate that the
environmentalists in question hold, with
certainty, any particular view regarding
population declines in general.
Correct. The author argues that the recent
declines may have several different causes,
and that environmentalists have jumped to a
conclusion about the cause of the declines as
well as their significance.
The environmentalists, in attributing
population declines to intentional human
activity, have more likely underestimated
than overestimated the effects of chance
events on amphibian populations.

The correct answer is D.
506. It can be inferred from the passage that the author
believes which of the following to be true of the
amphibian extinctions that have recently been
reported?
(A)

They have resulted primarily from human
activities causing environmental degradation.

(B)

They could probably have been prevented if
timely action had been taken to protect the
habitats of amphibian species.

(C)

They should not come as a surprise, because
amphibian populations generally have been
declining for a number of years.

(D)

They have probably been caused by a
combination of chance events.

(E)

They do not clearly constitute evidence of
general environmental degradation.

Inference
The author suggests throughout the passage that
recently reported amphibian extinctions may
have several different causes: they may be due
to any number of chance events, for example, or
may simply be the result of a small population
that finds itself unable to continue under difficult
conditions, whatever causes those conditions.
A

The author states in the second paragraph
that extinctions may occur without a
proximate cause in human activities and
does not make a commitment to any
particular explanation of the amphibian
extinctions.

B

C

D

E

That chance events can cause extinctions
suggests that even if habitats had been
protected, extinctions still might have
occurred.
In the second paragraph, the author says
that extinctions should come as no great
surprise, but this option is imprecise. The
amphibian populations have not generally
been declining for a number of years. The
author says in the third paragraph that
amphibian populations show strong
fluctuations; further, in the fourth paragraph,
the author says that there is insufficient
long‐term data to conclude that amphibian
populations have been, or are, in decline.
The author suggests that the extinctions
may have been caused by chance events, but
there is not enough data to know whether
or not this is probable.
Correct. The reported extinctions could
have resulted from several different causes;
thus, they are not clear evidence of general
environmental degradation.

The correct answer is E.
507. According to the passage, each of the following is true
of endangered amphibian species EXCEPT:
(A)

They are among the rarest kinds of amphibians.

(B)

They generally have populations that are small in
size.

(C)

They are in constant danger of extinction.

(D)

Those with decreasing populations are the most
likely candidates for immediate extinction.

(E)

They are in danger of extinction due to events
that sometimes have nothing to do with human
activities.

Application
This question asks what the passage does not
say is true of endangered amphibian species. The
second paragraph discusses endangered species,
stating that they are always rare, almost always
small, and, by definition, under constant threat of
extinction, which may be caused by chance events.
The possibility of their extinction, the passage
states, depends only on the population size, and
not whether that population is increasing or
decreasing.
481

GMAT® Official Guide 2018

A

B

C

D

E

The second paragraph mentions rarity as
a characteristic of endangered amphibian
species.
According to the second paragraph,
endangered amphibian species are generally
those of small populations.
The second paragraph states that an
endangered population is under constant
threat of extinction.
Correct. The last sentence of the second
paragraph states that the probability
of extinction due to chance events is
independent of how a population changes
in size. Immediate extinction would more
likely come from such events, whereas
population decline is gradual, even if fairly
rapid.
Endangered species, according to the
second paragraph, may become extinct due
to chance events—that is, events that have
nothing to do with human activities.

The correct answer is D.
508. Which of the following most accurately describes the
organization of the passage?
(A)

A question is raised, a distinction regarding it is
made, and the question is answered.

(B)

An interpretation is presented, its soundness is
examined, and a warning is given.

(C)

A situation is described, its consequences are
analyzed, and a prediction is made.

(D)
(E)

Two interpretations of a phenomenon are
described, and one of them is rejected as invalid.
Two methods for analyzing a phenomenon are
compared, and further study of the phenomenon
is recommended.

Evaluation
This question asks about the organization of
the passage as a whole. In the first paragraph,
the author tells about a situation that has
been interpreted in a particular way by
environmentalists. The passage then proceeds
to consider whether that interpretation is valid,
and while it does not come to a definitive

482

conclusion on that point, the final paragraph
warns about the possible consequences of
not taking the action recommended by the
environmentalists.
A

B

C

D

E

The passage does initially raise a question
regarding whether the environmentalists’
interpretation of events is valid, but it does
not answer that question, for the appropriate
long‐term data are not available.
Correct. The passage presents
environmentalists’ interpretation of recent
news regarding amphibians, then examines the
soundness of that interpretation. Finally, the
author warns that postponing environmental
action may have disastrous consequences.
The first paragraph describes a situation
of possibly drastic declines in amphibian
populations but does not follow this
description with an analysis of its
consequences.
The passage suggests that apparent declines
in amphibian populations may or may not
constitute a crisis, but it does not reject
either idea.
While the passage does imply, in its
final paragraph, that long‐term data on
amphibian populations should be collected,
the passage does not compare two methods
for analyzing amphibian populations or
population declines in those populations.

The correct answer is B.
509. Which of the following best describes the function of
the sentence in lines 35–38 ?
(A)

To give an example of a particular kind of study

(B)

To cast doubt on an assertion made in the
previous sentence

(C)

To raise an objection to a view presented in the
first paragraph

(D)

To provide support for a view presented in the
first paragraph

(E)

To introduce an idea that will be countered in the
following paragraph

7.6 Reading Comprehension Answer Explanations

Evaluation
The sentence in question discusses the way in
which anecdotal reports of population decreases
cannot help biologists determine whether
those decreases are normal fluctuations, take
populations to lower levels that are not actually
worrisome, or actually threaten extinctions.
This indicates that the view mentioned in the
first paragraph—reports of declines indicate a
catastrophic crisis—may be mistaken.
A

B

C

D

E

The sentence does not address a particular
kind of study; it objects to the use of
anecdotal reports in place of actual study.
The previous sentence describes the
possibilities referred to in the sentence in
question. The sentence does not cast doubt
on any of those possibilities.
Correct. The view that reports of amphibian
population declines indicate a crisis, as
presented in the first paragraph, is countered
by the objection here that there are several
possible causes for population declines, and
anecdotal reports cannot distinguish among
those possibilities.
The first paragraph is concerned with
articulating the view that amphibian
population declines constitute a crisis.
This sentence does not support that view;
instead, it offers reason to question it.
The sentence introduces the idea that
amphibian populations have fluctuated
in the past, and the following paragraph
supports this idea by stating that several
amphibian species that appeared almost
extinct in the 1950s and 1960s have
recovered. Thus, the paragraph does not
counter the sentence.

(B)

The degree to which other animals within a
habitat prey on mussels

(C)

The fact that mussel populations are often not
dominant within some habitats occupied by
P. ochraceus

(D)

The size of the P. ochraceus population within
the habitat

(E)

The fact that there is great species diversity
within some habitats occupied by P. ochraceus

Supporting idea
This question depends on recognizing what
the passage states about the factors affecting P.
ochraceus’s role as a keystone species, which is
different in different habitats. According to the
passage, P. ochraceus consumes and suppresses
mussel populations in some habitats—specifically,
those that are wave‐exposed—making it a
keystone predator in those habitats. But in wave‐
sheltered habitats, P. ochraceus does not play the
same role in suppressing mussel populations.
A

B

C

D

The correct answer is C.
Questions 510–513 refer to the passage on page 410.

E
510. The passage mentions which of the following as
a factor that affects the role of P. ochraceus as a
keystone species within different habitats?
(A)

The degree to which the habitat is sheltered from
waves

Correct. The passage clearly states that P.
ochraceus’s role in wave‐exposed habitats
differs from its role in wave‐sheltered
habitats.
The passage says that the impact of P.
ochraceus predation on mussels is not strong
in wave‐sheltered habitats, but this is
not—at least not at all sites—because other
animals are preying on the mussels; rather,
at least at some sites, it is because mussels
are controlled by sand burial.
The passage does not suggest that mussel
populations are dominant in any habitats
occupied by P. ochraceus.
The size of the P. ochraceus population affects
the size of the mussel population within
wave‐exposed habitats, but the passage
does not suggest that P. ochraceus’s role as a
keystone species depends on the size of its
population within those habitats.
The only other species the passage mentions
in conjunction with P. ochraceus habitats is
the mussel; the passage does not address
species diversity in these habitats.

The correct answer is A.

483

GMAT® Official Guide 2018

511. Which of the following hypothetical experiments most
clearly exemplifies the method of identifying species’
roles that the author considers problematic?
(A)

A population of seals in an Arctic habitat is
counted in order to determine whether it is the
dominant species in that ecosystem.

(B)

A species of fish that is a keystone species in
one marine ecosystem is introduced into another
marine ecosystem to see whether the species
will come to occupy a keystone role.

(C)

In order to determine whether a species of
monkey is a keystone species within a particular
ecosystem, the monkeys are removed from that
ecosystem and the ecosystem is then studied.

(D)

(E)

Different mountain ecosystems are compared to
determine how geography affects a particular
species’ ability to dominate its ecosystem.
In a grassland experiencing a changing climate,
patterns of species extinction are traced in order
to evaluate the effect of climate changes on
keystone species in that grassland.

C

D

E

The correct answer is C.
512. Which of the following, if true, would most clearly
support the argument about keystone status advanced
in the last sentence of the passage (lines 24–31)?
(A)

A species of bat is primarily responsible for
keeping insect populations within an ecosystem
low, and the size of the insect population in turn
affects bird species within that ecosystem.

(B)

A species of iguana occupies a keystone role
on certain tropical islands, but does not play
that role on adjacent tropical islands that are
inhabited by a greater number of animal species.

(C)

Close observation of a savannah ecosystem
reveals that more species occupy keystone
roles within that ecosystem than biologists had
previously believed.

(D)

As a keystone species of bee becomes more
abundant, it has a larger effect on the ecosystem
it inhabits.

(E)

A species of moth that occupies a keystone role
in a prairie habitat develops coloration patterns
that camouflage it from potential predators.

Application
Answering this question depends on recognizing
what the author says about identifying species’
roles in habitats and then extending that to
another situation. The author considers a
particular method of studying keystone species
problematic: removing a suspected keystone
species from its habitat and observing what
happens to the ecosystem. The author finds this
problematic because interactions among species
are complex.
A

B

484

The author does not discuss counting the
members of a population as a problematic
way of determining whether that population
is a dominant species.
The method that the author finds
problematic has to do with observing
what happens to an ecosystem when a
keystone species is removed from it, not
with observing what happens to a different
ecosystem when the species is introduced
into it.

Correct. The author states explicitly that
removing a species from a habitat in
order to determine its keystone status is
problematic. Removing the monkeys from
their habitat is a clear example of this
problematic practice.
Comparison of habitats in order to
determine geography’s effect on a
particular species’ dominance would most
likely find favor with the author, for this is
the approach the author seems to advocate
in investigating P. ochraceus’s keystone
status.
The author does not discuss tracing patterns
of extinction or changing climates in the
passage.

7.6 Reading Comprehension Answer Explanations

Evaluation
To answer this question, focus on the argument
advanced in the last sentence of the passage and
identify what information would support that
argument. In the last sentence of the passage, the
author claims that keystone status depends on
context. The author then offers three contextual
factors that may affect a species’ keystone status:
geography, community diversity (i.e., the number
of species in a given habitat), and length of
species interaction. Evidence supporting this
argument would show that context is important
to a species’ keystone status.
A

B

C

D

E

This scenario does not indicate anything
about keystone status; this is simply a
description of how species populations in a
single ecosystem affect one another.
Correct. That the iguana is a keystone
species in a location that has limited species
diversity but not a keystone species in a
location that has greater species diversity
suggests that keystone status does indeed
depend on context. Thus, this example
supports the author’s argument in the last
sentence of the passage.
That biologists were mistaken about
keystone species in a particular ecosystem
does not have a bearing on whether
keystone status is context dependent.
It is not surprising that an increase
in a species’ population would lead to
that species having a larger effect on its
ecosystem—but this does not speak directly
to the question of whether keystone status
itself depends on context.
A keystone species enhancing its ability to
survive in a single ecosystem does not lend
any support to the idea that keystone status
depends on context. The moth’s keystone
status would have to undergo some change
for this to have a bearing on the question of
context.

The correct answer is B.

513. The passage suggests which of the following about the
identification of a species as a keystone species?
(A)

Such an identification depends primarily on the
species’ relationship to the dominant species.

(B)

Such an identification can best be made
by removing the species from a particular
ecosystem and observing changes that occur in
the ecosystem.

(C)

Such an identification is likely to be less reliable
as an ecosystem becomes less diverse.

(D)

Such an identification seems to depend on
various factors within the ecosystem.

(E)

Such an identification can best be made by
observing predation behavior.

Inference
Answering this question requires identifying how
the passage suggests that keystone species should
be identified. The passage identifies a particular
way in which keystone status should not be
determined: removing a species and observing
what happens to the ecosystem. The passage also
argues that keystone status depends strongly on
context: that is, an ecosystem’s characteristics,
including its geography and inhabitants,
determine its keystone species.
A

B

C

While the passage uses an example of a
keystone species, P. ochraceus, which preys on
a species that would, in the keystone species’
absence, be dominant, there is nothing to
suggest that a keystone species must have a
particular relationship with the dominant,
or potentially dominant, species in an
ecosystem.
The passage explicitly states that this
method of identification would be
problematic.
A reduction in an ecosystem’s diversity
might alter which species occupy keystone
roles in that ecosystem, the passage suggests,
but there is no indication that identifying
such species would become more difficult.

485

GMAT® Official Guide 2018

D

E

Correct. If, as the passage suggests,
keystone status for any given species
depends on the context of the ecosystem
in which it lives, then it is likely that
identifying keystone species depends
strongly on understanding what factors
of the ecosystem contribute to creating
keystone status. The passage lists such
factors as geography, community diversity,
and species interaction.
While the passage uses a predator,
P. ochraceus, as its example of a keystone
species, there is no indication that predation
is an essential component of the actual
definition of keystone species (one whose
effects are much larger than would be predicted
from its abundance).

The correct answer is D.

Supporting ideas
This question depends on understanding how
a particular type of evidence—the preserved
soft bodies of conodonts—supports a particular
conclusion stated in the passage. The third
paragraph makes this relationship explicit,
explaining that certain features of conodonts
show them to be more primitive than other
vertebrates. Further, those features indicate
that they came before ostracoderms and other
armored jawless fishes. These remains support
the conclusion stated in the second paragraph
regarding teeth being more primitive than
external armor.
A
B

Questions 514–516 refer to the passage on page 412.

514. According to the passage, the anatomical evidence
provided by the preserved soft bodies of conodonts
led scientists to conclude that
(A)

conodonts had actually been invertebrate
carnivores

(B)

conodonts’ teeth were adapted from protective
bony scales

(C)

conodonts were primitive vertebrate suspension
feeders

(D)

primitive vertebrates with teeth appeared earlier
than armored vertebrates

(E)

scientists’ original observations concerning
the phosphatic remains of conodonts were
essentially correct

C

D

E

The passage states explicitly that conodonts
were not invertebrates but rather vertebrates.
This view is attributed to certain
traditionalists but is contradicted by other
paleontological evidence presented in the
second and third paragraphs. According to
the third paragraph, the evidence provided
by the preserved soft bodies of conodonts
undermines this traditional view.
The final sentence of the passage indicates
that the evidence in question supports the
conclusion that conodonts were predators
rather than suspension feeders.
Correct. The third paragraph explains how
conodonts’ remains support the conclusion
that teeth were more primitive than external
armor.
The second paragraph explains that
originally, scientists thought that early
vertebrates were not predators—but the
remainder of the passage indicates that
this idea is inconsistent with more recent
evidence described in the passage.

The correct answer is D.

486

7.6 Reading Comprehension Answer Explanations

515. The second paragraph in the passage serves primarily
to
(A)

outline the significance of the 1981 discovery of
conodont remains to the debate concerning the
development of the vertebrate skeleton

(B)

contrast the traditional view of the development
of the vertebrate skeleton with a view derived
from the 1981 discovery of conodont remains

(C)

contrast the characteristics of the ostracoderms
with the characteristics of earlier soft-bodied
vertebrates

(D)

explain the importance of the development of
teeth among the earliest vertebrate predators

(E)

present the two sides of the debate concerning
the development of the vertebrate skeleton

Evaluation
This question depends on understanding the
second paragraph in the context of the passage as
a whole. The second paragraph begins by noting
the traditional view of the vertebrate skeleton—
that it was a defense against predators—and then
goes on to explain that other paleontologists
argued against this idea, claiming instead that
vertebrates began as predators and that teeth were
a more primary feature than external armor.
A

B

C

The second paragraph focuses on describing
the debate rather than on the distinctive
contribution of the 1981 discovery to that
debate.
The second paragraph does not explicitly
indicate whether the opposition to the
traditional view originally rested on the
1981 discovery of conodont remains. In fact,
the surrounding discussion, in the first and
third paragraphs, suggests that the discovery
in 1981 turned out to support the opposing
view, which some paleontologists already
held at that time.
The mention of ostracoderms in the second
paragraph merely serves to indicate how
the traditionalists’ arguments might have
seemed plausible. The paragraph as a
whole is not devoted to contrasting the
ostracoderms with earlier soft-bodied
vertebrates.

D

E

The development of teeth figures in the
second paragraph, but this development
is mentioned first as a feature that some
believed to have been adapted from
protective scales; only the final sentence
of the paragraph connects teeth to early
vertebrate predators.
Correct. According to the passage, the
debate concerning the development of
the vertebrate skeleton hinges on whether
vertebrates began as predators, with teeth,
or whether skeletal defenses such as external
armor evolved first. The primary purpose of
the second paragraph is to distinguish these
two sides.

The correct answer is E.
516. It can be inferred that on the basis of the 1981
discovery of conodont remains, paleontologists could
draw which of the following conclusions?
(A)

The earliest vertebrates were sedentary
suspension feeders.

(B)

Ostracoderms were not the earliest vertebrates.

(C)

Defensive armor preceded jaws among
vertebrates.

(D)

Paired eyes and adaptations for activity are
definitive characteristics of vertebrates.

(E)

Conodonts were unlikely to have been predators.

Inference
What could paleontologists conclude, based on
the 1981 discovery of conodont remains? That
discovery, according to the passage, supported the
view of certain paleontologists that the earliest
vertebrates were predators with teeth—unlike the
ostracoderms, which had no jaws.
A

According to the second paragraph,
traditionalists believed that early vertebrates
were sedentary suspension feeders. But
the 1981 discovery supported instead the
hypothesis that early vertebrates were
predators instead.

487

GMAT® Official Guide 2018

B

C

D

E

Correct. According to the third paragraph,
the conodonts’ body structures indicated
that they were more primitive than the
ostracoderms, so the ostracoderms must not
have been the earliest vertebrates.
Traditionalists argued that teeth were
adapted from bony scales that provided
defensive armor, but the 1981 discovery
suggested that teeth preceded such scales.
Paleontologists knew prior to the
1981 discovery that paired eyes and other
adaptations are characteristics of vertebrates.
They used this knowledge to help them
interpret the 1981 discovery.
The third paragraph indicates that
conodonts, given their teeth, were most
likely predators.

A

B

C

D

E

The passage takes no position on the merits
of technological change but is concerned
only with the role of such change in society.
The passage mentions telephone exchange
workers as an example that helps
illustrate the more central debate between
determinists and constructivists.
Correct. The passage is mainly concerned
with portraying Clark’s view as a successful
challenge to constructivism.
The passage describes Clark’s view as a
successful challenge to social constructivism,
not technological determinism.
The passage is concerned with describing
a challenge to social constructivism and
not with suggesting the context in which
technological change ought to be studied.

The correct answer is C.
The correct answer is B.
Questions 517–528 refer to the passage on page 414.

517. The primary purpose of the passage is to

(A)

The new technology reduced the role of
managers in labor negotiations.

(A)

advocate a more positive attitude toward
technological change

(B)

(B)

discuss the implications for employees of the
modernization of a telephone exchange

The modernization was implemented without the
consent of the employees directly affected by it.

(C)

(C)

consider a successful challenge to the
constructivist view of technological change

The modernization had an impact that went
significantly beyond maintenance routines.

(D)

(D)

challenge the position of advocates of
technological determinism

Some of the maintenance workers felt victimized
by the new technology.

(E)

(E)

suggest that the social causes of technological
change should be studied in real situations

The modernization gave credence to the view of
advocates of social constructivism.

Main idea
This question asks for an assessment of what
the passage as a whole is doing. The passage
introduces Clark’s study as a solid contribution
(line 3) to the debate between technological
determinists and social constructivists. In the
second paragraph, Braverman is introduced as
holding a position of social constructivism, a
position that Clark takes issue with. In the final
paragraph, the passage holds that Clark refutes the
extremes of the constructivists (line 31), and Clark’s
arguments challenging social constructivism are
then described.
488

518. Which of the following statements about the
modernization of the telephone exchange is supported
by information in the passage?

Supporting ideas
This question requires recognizing information
contained in the passage. The passage states in
the first paragraph that Clark’s study focused
on the modernization of a telephone exchange
and the effect this had on maintenance work
and workers. After describing Braverman’s
analysis in the second paragraph as being at
odds with Clark’s views, the passage discusses
Clark’s views in more detail in the final
paragraph. As part of this discussion, the
passage notes that Clark shows how a change
from maintenance‐intensive electromechanical
switches to semielectronic switching systems at
the telephone exchange altered work tasks,

7.6 Reading Comprehension Answer Explanations

skills, training opportunities, administration, and
organization of workers (lines 42–44). Thus, the
passage shows that the modernization of the
telephone exchange affected much more than
maintenance routines.
A

B

C

D

E

The passage does not discuss whether new
technology reduces the role of managers in
labor negotiations.
The passage does not discuss the role of
employee consent in the modernization of
the telephone exchange.
Correct. The passage states that the
modernization of the telephone exchange
affected tasks, skills, training, administration,
and the organization of workers.
The passage does not suggest that
maintenance workers felt victimized
by the modernization of the telephone
exchange.
The passage describes modernization
as a fact viewable from a perspective of
social constructivism or technological
determinism, but that does not in itself
support either view.

The correct answer is C.
519. Which of the following most accurately describes
Clark’s opinion of Braverman’s position?
(A)

He respects its wide‐ranging popularity.

(B)

He disapproves of its misplaced emphasis on the
influence of managers.

(C)

He admires the consideration it gives to the
attitudes of the workers affected.

(D)

He is concerned about its potential to impede
the implementation of new technologies.

(E)

He is sympathetic to its concern about the
impact of modern technology on workers.

In the second paragraph, the passage describes
Clark as holding that technology can be a primary
determinant of social and managerial organization
(lines 9–11), which suggests that managers are
sometimes subordinate to technological change.
In lines 15–17, however, Braverman is described
as holding that the shape of a technological system is
subordinate to the manager’s desire to wrest control of
the labor process from the workers, which shows that
Clark and Braverman are at odds on this point.
A

B

C
D
E

The correct answer is B.
520. The information in the passage suggests that which of
the following statements from hypothetical sociological
studies of change in industry most clearly exemplifies
the social constructivists’ version of technological
determinism?
(A)

It is the available technology that determines
workers’ skills, rather than workers’ skills
influencing the application of technology.

(B)

All progress in industrial technology grows out of
a continuing negotiation between technological
possibility and human need.

(C)

Some organizational change is caused by
people; some is caused by computer chips.

(D)

Most major technological advances in industry
have been generated through research and
development.

(E)

Some industrial technology eliminates jobs, but
educated workers can create whole new skills
areas by the adaptation of the technology.

Inference
Answering this question requires inferring
what the passage’s author likely believes. The
passage describes Braverman’s position as one
of mainstream social constructivism (lines
23–24), a position that Clark takes issue with.
Although it describes Braverman’s position, the
rest of the passage is devoted to showing how
Clark’s position takes issue with Braverman’s.

Since the passage says that Clark believes
an important insight has been obscured by the
recent sociological fashion that Braverman’s
views exemplify (lines 11–14), one cannot
infer that Clark respects the popularity of
Braverman’s views.
Correct. The passage shows that Clark
believes managers to have less influence over
how technology affects an organization than
Braverman claims that they have.
The passage does not indicate that Clark
admires any aspect of Braverman’s position.
The passage does not indicate that Clark
considers impediments to modernization.
The passage does not indicate that Clark is
sympathetic to any concerns attributed to
Braverman.

489

GMAT® Official Guide 2018

Application
This question requires understanding different
points of view discussed in the passage. In
the first paragraph, the passage mentions the
debate involving technological determinism
and social constructivism. In the second and
third paragraphs, the passage uses Braverman’s
analysis to illustrate the social constructivists’
position and in the third paragraph suggests
that the constructivists are misrepresenting
technological determinism (line 24). In lines 29–30,
the constructivists are reported to hold that
technological determinism views technology as
existing outside society, capable of directly influencing
skills and work organization.
A

B

C

D

E

Correct. This statement is consistent with
the constructivists’ view that technological
determinism sees technology as outside of
society, influencing workers’ skills.
The passage states that the constructivists
hold that technological determinists are
supposed to believe . . . that machinery imposes
appropriate forms of order on society (lines
25–27), suggesting that no negotiation is
present.
According to the description of them in the
passage, constructivists portray technological
determinists as believing that technology,
not people, drives organizational change.
The passage does not portray either
constructivists or determinists as being
concerned with technological research and
development.
The passage does not portray either
constructivists or determinists as being
concerned with technology‐driven job
elimination or creation.

The correct answer is A.
521. The information in the passage suggests that Clark
believes that which of the following would be true
if social constructivism had not gained widespread
acceptance?

490

(A)

Businesses would be more likely to modernize
without considering the social consequences of
their actions.

(B)

There would be greater understanding of the role
played by technology in producing social change.

(C)

Businesses would be less likely to understand
the attitudes of employees affected by
modernization.

(D)

Modernization would have occurred at a slower
rate.

(E)

Technology would have played a greater part in
determining the role of business in society.

Inference
Answering this question involves
understanding a point of view as it is described
in the passage. The passage aligns Clark’s study
closely with the technological determinists,
summarizing his view in lines 9–11: technology
can be a primary determinant of social and
managerial organization. In the following
sentence, the passage states that Clark believes
that this possibility is obscured by the recent
sociological fashion, exemplif ied by Braverman’s
analysis (lines 11–13). After illustrating
Braverman’s analysis, the passage then states
that it represents social constructivism.
A

B

C

D

E

According to the passage, Clark holds that
constructivists obscure how modernization
might have social consequences.
Correct. According to the passage, Clark
sees constructivism as obscuring the
possibility that technology plays a primary
role in social change.
The passage does not discuss how the
attitudes of employees are perceived by their
employers.
The passage describes a debate about the
history and sociology of technology; it does
not suggest that sociological analyses affect
the pace of modernization.
The passage describes a debate about the
history and sociology of technology; it
does not suggest that sociological analyses
affect the role that technology plays in
business.

The correct answer is B.

7.6 Reading Comprehension Answer Explanations

522. According to the passage, constructivists employed
which of the following to promote their argument?
(A)

Empirical studies of business situations involving
technological change

(B)

Citation of managers supportive of their position

(C)

Construction of hypothetical situations that
support their view

(D)
(E)

(A)

suggest that a contention made by
constructivists regarding determinists is
inaccurate

(B)

Contrasts of their view with a misstatement of an
opposing view

define the generally accepted position of
determinists regarding the implementation of
technology

(C)

Descriptions of the breadth of impact of
technological change

engage in speculation about the motivation of
determinists

(D)

lend support to a comment critical of the
position of determinists

(E)

contrast the historical position of determinists
with their position regarding the exchange
modernization

Supporting ideas
Answering this question involves recognizing
information given in the passage. The passage
indicates that a debate exists between
technological determinists and social
constructivists, suggesting that these views are
in opposition. The passage goes on to state that
constructivists gain acceptance by misrepresenting
technological determinism (lines 23–24). This
misrepresentation is presented as the alternative
to constructivism (lines 27–28), suggesting
that constructivists promoted their own view
by contrasting it with a misrepresentation of
determinists’ views.
A

B

C

D

E

523. The author of the passage uses the expression “are
supposed to” in line 25 primarily in order to

The passage mentions empirical studies in
relation to Clark’s study but not Braverman’s
analysis.
The passage does not mention that
managers were supportive of any particular
point of view within the sociology of
technology.
The passage does not mention any
hypothetical situations as being used by the
constructivists in support of their view.
Correct. The passage indicates that the
constructivists have come into fashion
by contrasting their own views with
a misrepresentation of the views of
technological determinists.
The passage does not describe the
constructivists as making determinations
regarding the degree of impact that
technological change has on social or
managerial organization.

Evaluation
This question requires understanding how a
particular phrase functions in the passage as
a whole. In the third paragraph the passage
states that constructivists gain acceptance by
misrepresenting technological determinism
(lines 23–24) and follows this claim with an
example of this misrepresentation, stating that
technological determinists are supposed to believe,
for example (lines 25–26). This line implies that
the constructivist view of the determinists is
inaccurate.
A

B

C

D

E

Correct. The passage uses the expression
in part to provide an example of the
constructivists’ misrepresentation of the
determinists.
The passage indicates that the view
attributed to the determinists is a
misrepresentation, not one that is generally
accepted by determinists.
The expression in the passage is part
of a discussion about the motivation of
constructivists, not determinists.
The expression in the passage is part
of a discussion that is critical of the
constructivists, not the determinists.
The passage does not describe either the
historical position of determinists or their
position on the exchange modernization.

The correct answer is A.

The correct answer is D.
491

GMAT® Official Guide 2018

524. Which of the following statements about Clark’s study
of the telephone exchange can be inferred from
information in the passage?
(A)

(B)

Clark’s reason for undertaking the study was to
undermine Braverman’s analysis of the function
of technology.
Clark’s study suggests that the implementation
of technology should be discussed in the context
of conflict between labor and management.

(C)

Clark examined the impact of changes in the
technology of switching at the exchange in terms
of overall operations and organization.

(D)

Clark concluded that the implementation of new
switching technology was equally beneficial to
management and labor.

(E)

Clark’s analysis of the change in switching
systems applies only narrowly to the situation at
the particular exchange that he studied.

Inference
This question requires understanding what
the passage implies in its discussion of a
point of view. The details of Clark’s views are
discussed primarily in the final paragraph. The
passage states that on an empirical level, Clark
demonstrates that technological change regarding
switches at the telephone exchange altered work
tasks, skills, training opportunities, administration,
and organization of workers (lines 42–44). The
passage goes on to state Clark’s contention that
these changes even influenced negotiations
between management and labor unions.

C

D

E

Correct. According to the passage, Clark
concludes that changes to the technology of
switches had an influence on several aspects
of the overall operations and organization of
the telephone exchange.
The passage does not indicate that Clark
assesses the benefits of technological change
to either labor or management.
The passage indicates that Clark believes the
change in switching technology influenced
many aspects of the overall operations of the
telephone exchange.

The correct answer is C.
Questions 525–529 refer to the passage on page 416.

525. The passage implies that which of the following was
a reason that the proportion of verdicts in favor of
patentees began to increase in the 1830s ?
(A)

Patent applications approved after 1836 were
more likely to adhere closely to patent law.

(B)

Patent laws enacted during the 1830s better
defined patent rights.

(C)

Judges became less prejudiced against
patentees during the 1830s.

(D)

After 1836, litigated cases became less
representative of the population of patent
disputes.

(E)

The proportion of patent disputes brought to trial
began to increase after 1836.

Inference
A

B

492

The passage indicates that Clark’s
study addressed the extremes of both
technological determinism and social
constructivism. It cites Braverman as a
proponent of social constructivism but
provides no evidence that Clark’s motivation
in beginning his study was specifically to
target an analysis offered by Braverman.
The passage indicates that Clark attributed
some organizational change to the way
labor and management negotiated the
introduction of technology but does not
mention conflict between them.

The question asks which statement can be
reasonably inferred, from information provided
in the passage, to be a reason for the increase in
proportion of verdicts favoring patentees, starting
in the 1830s. The second paragraph argues that
what changed in that decade was not judges’
attitudes toward patent law, but the types of
patent cases that were litigated. It explains that
a law passed in 1836 required that, for the first
time in U.S. history, applications for patents had
to be examined for their adherence to patent law
before a patent would be issued. This information
implies that patents granted after 1836 were more
likely to adhere to patent law and were thus more
likely to be upheld in court.

7.6 Reading Comprehension Answer Explanations

A

B

C

D

E

Correct. The passage implies that patents
granted after the 1836 law went into effect
were more likely to adhere to patent law.
The passage does not indicate that any law
mentioned made changes to the definition
of patent rights; rather, the passage indicates
that the patent system was revised to require
that patent applications be reviewed for
adherence to existing law.
The passage rejects the explanation that
judges’ attitudes toward patent rights
became more favorable.
The passage indicates that the population
of disputes that were litigated changed
after 1836, but it does not suggest that the
population of litigated disputes differed
from that of patent disputes as a whole.
The passage does not indicate any change in
the proportion of patent disputes brought to
trial.

The correct answer is A.
526. The passage implies that the scholars mentioned in
line 8 would agree with which of the following criticisms
of the American patent system before 1830?

B

C

D

E

The passage does not indicate that the
scholars were critical of the criteria for
granting patents.
The scholars are concerned with inventors’
attempts to protect their patents, not the
difficulty of acquiring a patent in the first
place.
The passage does not imply that the scholars
in question believed that too many patent‐
infringement suits were brought to court,
but rather that too few succeeded.
Correct. The scholars doubt that patent
law helped to achieve its goal, which was to
encourage economic growth.

The correct answer is E.
527. It can be inferred from the passage that the frequency
with which pre‐1830 cases have been cited in court
decisions is an indication that
(A)

judicial support for patent rights was strongest in
the period before 1830

(B)

judicial support for patent rights did not increase
after 1830

(C)

courts have returned to judicial standards that
prevailed before 1830

(A)

Its definition of property rights relating to
inventions was too vague to be useful.

(D)

(B)

Its criteria for the granting of patents were not
clear.

verdicts favoring patentees in patent‐infringement
suits did not increase after 1830

(E)

(C)

It made it excessively difficult for inventors to
receive patents.

judicial bias against patentees persisted after
1830

(D)

It led to excessive numbers of patent‐
infringement suits.

(E)

It failed to encourage national economic growth.

Inference
This question asks about a statement implied
by the passage. The scholars mentioned in line 8
question whether U.S. patent law achieved its
goal. That goal is described in the first sentence
of the passage: to encourage America’s economic
growth. Thus, it is reasonable to conclude that
the scholars would criticize the pre‐1830 patent
system for failing to encourage economic growth.
A

Inference
The question asks what is indicated by the
frequency with which pre‐1830 cases have been
cited in court decisions. The second paragraph
rejects some scholars’ claims that judges prior to
the 1830s were antipatent, while judges after that
time were more accepting of patent rights. The
passage supports its critique by pointing out that
decisions made by judges before the 1830s have
been cited as precedents by later judges just as
frequently as post‐1830s decisions have been. This
implies that later judges’ attitudes toward patent
rights were similar to those of pre‐1830s judges.
Thus, there is no reason to believe judges’ attitudes
toward patent rights changed at that time.

The scholars contend that judges rejected
patents for arbitrary reasons, not because the
definition of property rights was vague.
493

GMAT® Official Guide 2018

A
B

C

D

E

The passage argues that judicial support for
patents did not change in the 1830s.
Correct. Pre‐1830s court decisions have
been cited as frequently as later decisions,
suggesting no change in judges’ attitudes.
The passage does not indicate that judicial
standards changed from, and then returned
to, those that prevailed before 1830.
Although actual numbers of favorable
verdicts are not mentioned, the passage
indicates that the proportion of verdicts
decided in favor of patentees did, in fact,
increase beginning in the 1830s.
The passage rejects the notion that judges
were biased against patentees either before
or after 1830.

A

B

C

D

The correct answer is B.
528. It can be inferred from the passage that the author
and the scholars referred to in line 21 disagree about
which of the following aspects of the patents defended
in patent‐infringement suits before 1830?

The correct answer is D.

(A)

Whether the patents were granted for inventions
that were genuinely useful

(B)

Whether the patents were actually relevant to the
growth of the United States economy

(C)

Whether the patents were particularly likely to be
annulled by judges

(A)

The proportion of cases that were decided
against patentees in the 1820s

(D)

Whether the patents were routinely invalidated
for reasons that were arbitrary

(B)

The total number of patent disputes that were
litigated from 1794 to 1830

(E)

Whether the patents were vindicated at a
significantly lower rate than patents in later suits

(C)

The fact that later courts drew upon the legal
precedents set in pre‐1830 patent cases

(D)

The fact that the proportion of judicial decisions
in favor of patentees began to increase during
the 1830s

(E)

The constitutional rationale for the 1836 revision
of the patent system

Inference
The question depends on recognizing differences
between two explanations—one favored by the
scholars mentioned in line 21, the other favored by
the author—for the frequency with which patents
were invalidated in U.S. courts prior to 1830. The
first paragraph describes the scholars’ view that
judges before 1830 were antipatent and rejected
patentees’ claims for arbitrary reasons. The author
of the passage rejects that view. As an alternate
explanation, the author in the second paragraph
implies that earlier patents often violated copyright
law; this view is supported with reference to an
1836 revision to the patent system which instituted
a procedure by which patent applications were
inspected to ensure adherence to patent law.
494

E

The author and the scholars are both
focused on protecting inventors’ property
rights, not with their inventions’ utility.
Although the passage suggests that the
scholars thought America’s patent system did
not help encourage economic growth, there
is no suggestion that either the scholars or
the author believes actual patents defended in
court were irrelevant to economic growth.
Both the scholars and the author believe
that patents defended in court prior to 1830
were more likely to be invalidated than were
patents in later legal disputes.
Correct. The scholars claim that judges
before 1830 decided against patentees for
arbitrary reasons, but the passage suggests
that the patents may have been invalidated
because they failed to adhere to patent law.
Both the scholars and the author accept
that patents were upheld in court less often
before 1830 than after.

529. The author of the passage cites which of the following
as evidence challenging the argument referred to in
lines 14–15 ?

Supporting idea
The question asks what evidence the author
brings to bear against the argument referred to
in lines 14–15. In the first paragraph, the author
summarizes scholars’ arguments to the conclusion
that judges’ attitudes toward patent rights shifted
in the 1830s, based on the fact that judges
earlier had routinely ruled against patentees in
lawsuits whereas judges after that time provided
more protection for patent rights. In the second
paragraph the author challenges the claim that

7.6 Reading Comprehension Answer Explanations

judges’ attitudes shifted. The author provides
evidence that judges after the 1830s cited legal
precedents set in pre‐1830s cases, suggesting that
their views had not changed.

A

B
A

B
C

D

E

The proportion of cases decided against
patentees in the 1920s is cited as evidence
that supports the scholars’ argument in the
first paragraph, not as evidence challenging
their views.
The total number of disputes litigated is not
mentioned in the passage.
Correct. The fact that judges after 1830
cited earlier cases as precedents is used as
evidence to challenge scholars’ claims that
judges’ attitudes shifted around 1830.
The change in the proportion of decisions
in favor of patentees is a fact that both
the scholars and the author of the passage
attempt to explain.
No constitutional rationale for the 1836 law
is mentioned in the passage.

C

D

E

The passage indicates that Krontiris uses
the Renaissance women writers’ works as
historical evidence, not that she offered any
criticism of the works themselves.
Krontiris’s work, according to the author,
is cautiously optimistic about women’s
achievements during the Renaissance. This
contradicts Kelly’s view that the status of
women declined during this time.
The author suggests that Krontiris may
have erred in taking her six subjects as
representative of all women during the
Renaissance, not that she made any
misinterpretations of their actual writing.
Correct. The author uses Krontiris as an
example of those feminist scholars who have
rejected the overgeneralized approaches of
both Kelly and Burckhardt.
The author makes clear that Krontiris’s
study focuses on literate Renaissance
women, who constituted a small minority.

The correct answer is D.

The correct answer is C.
Questions 530–536 refer to the passage on page 418.

530. The author of the passage discusses Krontiris primarily
to provide an example of a writer who

531. According to the passage, Krontiris’s work differs from
that of the scholars mentioned in line 12 in which of
the following ways?
(A)

Krontiris’s work stresses the achievements of
Renaissance women rather than the obstacles to
their success.

(A)

is highly critical of the writings of certain
Renaissance women

(B)

(B)

supports Kelly’s view of women’s status during
the Renaissance

Krontiris’s work is based on a reinterpretation of
the work of earlier scholars.

(C)

(C)

has misinterpreted the works of certain
Renaissance women

Krontiris’s views are at odds with those of both
Kelly and Burkhardt.

(D)

(D)

has rejected the views of both Burckhardt and Kelly

Krontiris’s work focuses on the place of women
in Renaissance society.

(E)

has studied Renaissance women in a wide variety
of social and religious contexts

(E)

Krontiris’s views are based exclusively on the
study of a privileged group of women.

Evaluation
This question focuses on the author’s reason
for mentioning Krontiris’s work. The passage
states that Krontiris, in her discussion of six
Renaissance women writers, is an example of
scholars who are optimistic about women’s
achievements but also suggest that these women
faced significant obstacles. She is a writer who, in
other words, agrees with neither Kelly’s negative
views nor Burckhardt’s positive approach.

Supporting idea
This question asks what the passage directly states
about the difference between Krontiris’s work
and the feminist scholars mentioned in the first
paragraph. The feminist scholars mentioned in
the first paragraph explore differences among
Renaissance women, particularly their social status
and religion, and thus complicate Burckhardt’s and
Kelly’s generalizations. Krontiris’s work, on the other
hand, focuses on Renaissance women writers, who
are a distinctly privileged and small social group.
495

GMAT® Official Guide 2018

A

B

C

D

E

The second paragraph makes clear that
Krontiris addresses the obstacles faced by
Renaissance women.
The passage does not suggest that
Krontiris is reinterpreting or drawing on
reinterpretations of the work of earlier
scholars.
The second paragraph shows that Krontiris’s
work does complicate both Burckhardt’s and
Kelly’s views, but in this, she is in agreement
with the feminist scholars mentioned in the
first paragraph.
Both Krontiris and the feminist scholars
mentioned in the first paragraph are
concerned with the place of women in
Renaissance society.
Correct. The feminist scholars mentioned
in the first paragraph are concerned with
women of different social classes and
religions, whereas Krontiris’s work focuses
on a limited social group.

The correct answer is E.
532. According to the passage, feminist scholars cite
Burckhardt’s view of Renaissance women primarily for
which of the following reasons?
(A)

Burckhardt’s view forms the basis for most
arguments refuting Kelly’s point of view.

(B)

Burckhardt’s view has been discredited by Kelly.

(C)

Burckhardt’s view is one that many feminist
scholars wish to refute.

(D)

Burckhardt’s work provides rich historical
evidence of inequality between Renaissance
women and men.

(E)

Burckhardt’s work includes historical research
supporting the arguments of the feminist
scholars.

Supporting idea
This question asks what the passage says
explicitly about why feminist scholars reference
Burckhardt’s view of Renaissance women. The
first paragraph states that Burckhardt’s view is
that Renaissance women enjoyed perfect equality
with men, and then follows that by noting how
feminist scholars have repeatedly cited this view
to contrast it with extensive evidence of women’s
inequality during the Renaissance.
496

A

B

C

D

E

The passage does not indicate that any
feminist scholars cite Burckhardt to refute
Kelly’s view. It uses Krontiris as an example
of scholars who refute Kelly’s point of view
to a certain degree, but Krontiris does not
use Burckhardt’s view as her basis for doing
so; Krontiris argues against Burckhardt as
well.
According to the first paragraph, Kelly’s
work was in certain ways inconsistent
with Burckhardt’s view, but that is not a
reason why Burckhardt’s view is cited by
feminist scholars. Rather, according to
the passage, they cite it in order to argue
against it.
Correct. Many feminist scholars wish to
refute Burckhardt’s view that Renaissance
women and men were equal.
As the first paragraph makes clear,
Burckhardt’s work emphasizes equality, not
inequality, between Renaissance women and
men.
The passage does not discuss the historical
research on which Burckhardt based his
work.

The correct answer is C.
533. It can be inferred that both Burckhardt and Kelly have
been criticized by the scholars mentioned in line 12 for
which of the following?
(A)

Assuming that women writers of the Renaissance
are representative of Renaissance women in
general

(B)

Drawing conclusions that are based on the study
of an atypical group of women

(C)

Failing to describe clearly the relationship
between social status and literacy among
Renaissance women

(D)

Failing to acknowledge the role played by
Renaissance women in opposing cultural
stereotypes

(E)

Failing to acknowledge the ways in which
social status affected the creative activities of
Renaissance women

7.6 Reading Comprehension Answer Explanations

Inference
Line 12 refers to feminist scholars who have
rejected both Kelly’s and Burckhardt’s views
of the status of Renaissance women. The next
sentence states that the feminist scholars use
class and religious differences among Renaissance
women to argue against Kelly’s and Burckhardt’s
generalizations, which were based on upper‐class
Italian women.

(C)

during the Renaissance, women were able to
successfully oppose cultural stereotypes relating
to gender

(D)

Renaissance women did not face many difficult
social obstacles relating to their gender

(E)

in order to attain power, Renaissance women
attacked basic assumptions in the ideologies
that oppressed them

Inference
A

B

C

D

E

The second paragraph suggests that
Krontiris at times conflates Renaissance
women writers and women in general,
but the passage does not indicate that the
feminist scholars believe this of Kelly or
Burckhardt.
Correct. The feminist scholars mentioned
study different types of Renaissance women
and so reject Kelly’s and Burckhardt’s
conclusions that were based on a group that
was not in fact typical.
Krontiris, not Kelly and Burckhardt, is the
scholar who, according to the passage, fails
to address the relationship between literacy
and social status.
The passage provides no grounds for
determining whether Kelly, Burckhardt,
or the feminist scholars mentioned in the
first paragraph dealt with Renaissance
women’s opposition to cultural stereotypes;
Krontiris’s work is concerned with this
question.
The first paragraph suggests that feminist
scholars criticized Kelly and Burckhardt for
failing to acknowledge the ways in which
social status complicates any generalizations
that can be made about Renaissance
women’s lives, not their creative activities
specifically.

The correct answer is B.
534. The author of the passage suggests that Krontiris
incorrectly assumes that
(A)

(B)

social differences among Renaissance women
are less important than the fact that they were
women

The first statement the author makes about
Krontiris, in the second paragraph, concerns
what the author characterizes as a problem with
Krontiris’s work. Krontiris takes the Renaissance
women writers she studies as representative of
all Renaissance women; the author says that
designating women as the most important
grouping fails to consider whether other social
differences might make for differences in
experience.
A

B

C

D

E

Correct. The author indicates that
Krontiris’s error lies in assuming that
women’s identity as women trumps social
and other differences.
The author does not suggest that Krontiris
assumes inappropriate literacy levels
among Renaissance women, but rather
that Krontiris does not give sufficient
consideration to the idea that women who
could read and write most likely led lives
very different from those of women who
could not read and write.
The author says that Krontiris suggests that
there were many cultural stereotypes that
women were not able to oppose effectively.
Krontiris, according to the author,
acknowledges the many social obstacles
faced by women on the basis of their gender.
According to the author, Krontiris’s
concluding remarks suggest that
Renaissance women seldom attacked the basic
assumptions in the ideologies that oppressed
them.

The correct answer is A.

literacy among Renaissance women was more
prevalent than most scholars today acknowledge

497

GMAT® Official Guide 2018

535. The last sentence in the passage serves primarily to
(A)

suggest that Krontiris’s work is not
representative of recent trends among feminist
scholars

(B)

undermine the argument that literate women
of the Renaissance sought to oppose social
constraints imposed on them

(C)

show a way in which Krontiris’s work illustrates
a “cautiously optimistic” assessment of
Renaissance women’s achievements

(D)

summarize Krontiris’s view of the effect of
literacy on the lives of upper‐ and middle‐class
Renaissance women

(E)

illustrate the way in which Krontiris’s study differs
from the studies done by Burckhardt and Kelly

Evaluation
The function of the final sentence of the passage
is to indicate how Krontiris’s work takes neither a
completely positive nor completely negative view
of Renaissance women’s experiences—i.e., how
her work is representative of those authors who
are cautiously optimistic about the achievements
of Renaissance women.
A

B

C

498

The passage discusses Krontiris’s work as an
example of the trend described in the latter
part of the first paragraph and mentioned
in the first line of the second paragraph.
The last sentence in the passage shows that
Krontiris’s work is in fact representative of
recent trends among feminist scholars.
The last sentence in the passage states that
Renaissance women’s opposition to cultural
stereotypes was circumscribed, but it also
suggests that these women did gain some
power for themselves. Thus, the sentence
does not serve primarily to undermine the
argument that the women sought to oppose
social constraints.
Correct. Krontiris’s work illustrates the
“cautiously optimistic” view by embracing
both the idea that Renaissance women
could gain a certain amount of power and
the idea that the extent of their opposition
was limited.

D

E

The last sentence in the passage summarizes
Krontiris’s view, but that view does not,
according to the passage, take into account
the effect of literacy on the members of a
particular social class.
The main function of the final sentence of
the passage is to take up the idea of the
cautiously optimistic assessment offered
in the penultimate sentence. This does
mark a significant departure from both
Burckhardt and Kelly, but the distinction
between their work and that of other
feminist scholars is marked more clearly
earlier in the passage.

The correct answer is C.
536. The author of the passage implies that the women
studied by Krontiris are unusual in which of the
following ways?
(A)

They faced obstacles less formidable than those
faced by other Renaissance women.

(B)

They have been seen by historians as more
interesting than other Renaissance women.

(C)

They were more concerned about recording
history accurately than were other Renaissance
women.

(D)

Their perceptions are more likely to be
accessible to historians than are those of most
other Renaissance women.

(E)

Their concerns are likely to be of greater interest
to feminist scholars than are the ideas of most
other Renaissance women.

Inference
The women Krontiris studied are unusual, the
author suggests, because they were literate, thus
putting them among the minority of Renaissance
women. That they could write, however, means
that their written reflections are part of the
historical record, whereas the direct impressions
of experiences had by Renaissance women who
could not write about their lives are lost to
history.

7.6 Reading Comprehension Answer Explanations

A

B

C

D

E

The author implies that the obstacles faced
by Krontiris’s subjects may have been
different from those faced by other women,
not that they were less formidable.
The author does not imply that the women
studied by Krontiris are seen as more
interesting; rather, the author indicates that
their work is that which is available for study.
The women Krontiris studies were able to
record their own history because they, unlike
most other Renaissance women, were literate.
This does not imply that they were more
concerned with recording history accurately.
Correct. Because Krontiris’s subjects
were literate, they were able to write
down, and thus preserve for historians,
their perceptions in a way that most other
Renaissance women were not.
The author does not suggest that feminist
scholars in general are more interested in
the concerns of middle‐ and upper‐class
literate women than they are with women of
other classes.

The correct answer is D.

rotation rates would form a bell curve when
graphed, but if asteroids were piles of rubble, the
tail of the bell curve indicating very fast rotation
rates would be missing. Among asteroids larger
than 200 meters, this tail is missing, and only the
five asteroids described as exceptions have rotation
rates falling at the very high end of the bell curve.
A

B

C

D

Questions 537–540 refer to the passage on page 420.

E
537. The passage implies which of the following about the
five asteroids mentioned in line 12 ?
(A)

Their rotation rates are approximately the same.

(B)

They have undergone approximately the same
number of collisions.

(C)

They are monoliths.

(D)

They are composed of fragments that have
escaped the gravity of larger asteroids.

(E)

The correct answer is C.
538. The discovery of which of the following would call into
question the conclusion mentioned in line 16 ?
(A)

An asteroid 100 meters in diameter rotating at a
rate of once per week

(B)

An asteroid 150 meters in diameter rotating at a
rate of 20 times per hour

(C)

An asteroid 250 meters in diameter rotating at a
rate of once per week

(D)

An asteroid 500 meters in diameter rotating at a
rate of once per hour

(E)

An asteroid 1,000 meters in diameter rotating at
a rate of once every 24 hours

They were detected only recently.

Inference
In line 12, five observed asteroids, refers to the five
asteroids whose rotation rates are exceptions to
the strict limit on the rate of rotation found in
all other observed asteroids. These five asteroids
all have diameters smaller than 200 meters.
The passage indicates that if asteroids were all
monoliths—that is, single rocks—then their

All that the passage states about the rotation
rates of these five asteroids is that they do
not obey a strict limit. The passage does
not rule out that their rates of rotation are
significantly different from one another.
According to the passage, frequent collisions
occur among asteroids. But the passage does
not suggest that asteroids that are of similar
sizes, or that have particularly high rotation
rates, will be similar in terms of the number
of collisions that they have undergone to
reach those distinctive states.
Correct. The second paragraph states that
most small asteroids should be monolithic,
and the five observed asteroids are all
smaller than 200 meters in diameter.
The five asteroids are most likely not
composed of fragments because, as the
passage states, small asteroids should be
monoliths.
The passage notes that researchers have
observed these five asteroids, along with
others, but it does not indicate when these
asteroids were originally detected.

499

GMAT® Official Guide 2018

Application
The conclusion that the text in line 16 points
to is that asteroids with diameters greater than
200 meters are multicomponent structures or rubble
piles. To call that conclusion into question, an
observation would have to suggest that asteroids
larger than 200 meters across are not such
multicomponent structures. According to the first
paragraph, rubble piles cannot be fast rotators:
spinning faster than once every few hours would
make them fly apart.
A

B

C

D

500

Nothing in the passage suggests that the
behavior of an asteroid 100 meters in
diameter is relevant to a conclusion about
the behavior of asteroids greater than
200 meters in diameter.
Nothing in the passage suggests that
the behavior of an asteroid 150 meters
in diameter would have any effect on
a conclusion about the constitution of
asteroids with diameters greater than
200 meters.
An asteroid 250 meters in diameter rotating
at a rate of once per week would be rotating
at a slow enough rate to hold together a
pile of rubble. Thus, this observation would
be entirely consistent with the conclusion
about asteroids larger than 200 meters in
diameter.
Correct. Assuming that an asteroid
composed of a pile of rubble is of a great
enough density, a rotation rate greater than
one revolution every few hours would make
it fly apart. So a 500-meter asteroid rotating
at a rate of once per hour—that is, faster
than the crucial speed—would fly apart if it
were not a monolith. The conclusion states
that all asteroids larger than 200 meters are
multicomponent structures (that is, are not
monoliths), so the discovery of a 500-meter
asteroid rotating at a rate of once an hour
would call into question that conclusion.

E

An asteroid rotating at a rate of once
every 24 hours would, regardless of size,
be rotating much more slowly than the
once every few hours that the passage claims
would make a pile of rubble of a sufficient
density fly apart. So an asteroid with a
diameter of 1,000 meters that rotated once
per day could be a pile of rubble and not
conflict with the conclusion.

The correct answer is D.
539. According to the passage, which of the following
is a prediction that is based on the strength of the
gravitational attraction of small asteroids?
(A)

Small asteroids will be few in number.

(B)

Small asteroids will be monoliths.

(C)

Small asteroids will collide with other asteroids
very rarely.

(D)

Most small asteroids will have very fast rotation
rates.

(E)

Almost no small asteroids will have very slow
rotation rates.

Supporting ideas
Regarding small asteroids, the second paragraph
states that they have feeble gravity. Any fragments
from impacts would escape that gravity, and thus,
the passage states, the small asteroids should be
monolithic.
A
B

C

Small asteroids could be few in number, but
the passage does not offer such a prediction.
Correct. This prediction is offered in
the second paragraph, based on the fact
that small asteroids do not have strong
gravitational attraction. Any impact
fragments will easily escape the weak
gravitational attraction of the small
asteroids.
The passage discusses large asteroids
collisions in more detail than smallasteroid collisions, but it provides no basis
for predicting how often large and small
asteroids will, comparatively, be involved in
such collisions.

7.6 Reading Comprehension Answer Explanations

D

E

The first paragraph indicates that the
rotation rates of small asteroids can exceed
the upper limit on the rotation rates of
large asteroids, but it does not indicate that
most small asteroids have rotation rates that
exceed this upper limit.
The passage only indicates that there are few
observed exceptions to the upper limit on
rotation rates of large asteroids, and these
exceptions are all smaller than 200 meters in
diameter; the passage does not indicate that
there are few small asteroids that have very
slow rotation rates.

The correct answer is B.
540. The author of the passage mentions “escape velocity”
(see line 22) in order to help explain which of the
following?

B

That the bits of asteroid move slower than
their escape velocity helps explain why
the fragments reassemble, but it does not
help explain the speed with which they
reassemble.
C According to the author, asteroid collisions
occur frequently, but the escape velocity of
the resulting fragments does not help to
explain that frequency.
D The concept of escape velocity may help
explain why small asteroids are monoliths,
but it has no relevance, at least as far as the
passage indicates, to those asteroids’ rotation
rates.
E Correct. After a collision, it is the asteroid
fragments’ failure to reach escape velocity
that allows the fragments’ gravitational pull
to reassemble them into a rubble pile.
The correct answer is E.

(A)

The tendency for asteroids to become smaller
rather than larger over time

Questions 541–543 refer to the passage on page 422.

(B)

The speed with which impact fragments
reassemble when they do not escape an
asteroid’s gravitational attraction after a collision

541. The author of the passage mentions calculations about
tunneling time and barrier thickness in order to

(C)

The frequency with which collisions among
asteroids occur

(A)

suggest that tunneling time is unrelated to
barrier thickness

(D)

The rotation rates of asteroids smaller than 200
meters in diameter

(B)

(E)

The tendency for large asteroids to persist after
collisions

explain the evidence by which Wigner and
Eisenbud discovered the phenomenon of
tunneling

(C)

describe data recently challenged by Raymond
Chiao and colleagues

(D)

question why particles engaged in quantum
tunneling rarely achieve extremely high speeds

(E)

explain the basis for Wigner and Eisenbud’s
hypothesis

Evaluation
This question asks about the purpose of the
author’s use of the phrase escape velocity in the
second paragraph. The author is discussing what
occurs after an asteroid collision, in which a
large asteroid might be blasted to bits. The bits,
according to the author, will move slower than
their mutual escape velocity—that is, the speed
at which they would have to move to get away
from each other and not reassemble, under the
influence of gravity, into a rubble pile.
A

The author is emphasizing the asteroid bits
that do not escape rather than those that do.
Asteroids may become smaller over time,
but the fact that most bits move slower
than their escape velocity would not help to
explain this shrinkage.

Evaluation
This question asks why the author discusses
calculations about tunneling time and barrier
thickness. According to the passage, these
calculations provided the grounds for Wigner and
Eisenbud’s hypothesis that tunneling particles
may travel faster than light.
A

The passage states that tunneling time
is related to barrier thickness, up to the
point at which tunneling time reaches a
maximum.
501

GMAT® Official Guide 2018

B

C

D

E

The passage indicates that the phenomenon
of tunneling was noted at least as early as
1932. It provides no evidence that Wigner
and Eisenbud discovered it.
The passage uses Chiao’s work to support
the idea that tunneling particles may move
faster than light, not challenge it.
The author describes calculations about
tunneling time and barrier thickness in
order to explain that particles engaged in
quantum tunneling may in fact achieve
extremely high speeds, not to explain the
rarity of the phenomenon.
Correct. The calculations about tunneling
time and barrier thickness supported
Wigner and Eisenbud’s hypothesis that
quantum tunneling could occur at speeds
faster than that of light.

A

B

C

D

E

The correct answer is E.
542. The passage implies that if tunneling time reached no
maximum in increasing with barrier thickness, then
(A)
(B)

tunneling speed would increase with barrier
thickness
tunneling speed would decline with barrier
thickness

(C)

tunneling speed would vary with barrier thickness

(D)

tunneling speed would not be expected to
increase without limit

(E)

successful tunneling would occur even less
frequently than it does

Inference
The passage states that because tunneling time
reaches a maximum, then tunneling speed must
increase as barrier thickness increases. But if
tunneling time did not reach such a maximum,
then speed need not increase without limit; the
particle could have as low a speed in thicker
barriers as in thinner ones and take longer to
tunnel through a barrier.

502

If tunneling time could not reach a
maximum, then speed might increase,
decrease, or remain the same as barrier
thickness increases.
If tunneling time could not reach a
maximum, then speed might increase,
decrease, or remain the same as barrier
thickness increases.
Tunneling speed could vary with barrier
thickness if tunneling time could not reach
a maximum, but there is no basis in the
passage on which to conclude that this is
definitely so.
Correct. The tunneling particle could have
as low a speed in thicker barriers as in
thinner ones and simply take longer to make
its way through a thicker barrier.
The passage states that the probability
of successful tunneling declines as the
thickness of the barrier increases. However,
it does not address the issue of whether
the differences in probability of successful
tunneling are due to the greater time
required to go through thicker barriers.

The correct answer is D.
543. Which of the following statements about the earliest
scientific investigators of quantum tunneling can be
inferred from the passage?
(A)

They found it difficult to increase barrier
thickness continually.

(B)

They anticipated the later results of Chiao and
his colleagues.

(C)

They did not suppose that tunneling particles
could travel faster than light.

(D)

They were unable to observe instances of
successful tunneling.

(E)

They made use of photons to study the
phenomenon of tunneling.

7.6 Reading Comprehension Answer Explanations

Inference
This question asks about the earliest investigators
of quantum tunneling. The passage notes that
quantum tunneling’s extreme rapidity was
observed in 1932; thus, the earliest investigators
of this phenomenon knew of its existence at that
time. Not until 1955 did Wigner and Eisenbud
hypothesize that the particles traveled faster than
light. Thus, it is logical to infer that the earliest
investigators did not imagine such a speed.
A

B

C

D

E

There is nothing in the passage to suggest
that the earliest investigators of quantum
tunneling had difficulty manipulating
barrier thickness.
The passage states that Chiao and his
colleagues measured photons moving at 1.7
times the speed of light—but the passage
does not provide evidence that the earliest
investigators anticipated such speeds.
Correct. The passage suggests that prior to
1955, investigators of quantum tunneling
had not hypothesized that the particles
could travel faster than the speed of light.
The passage indicates that by 1932,
investigators had noted the rapidity of
quantum tunneling; although this does not
entail that they observed the phenomenon,
it is consistent with their having been able
to do so.
The passage indicates that Chiao’s work
involves photons, but it does not indicate
the type of particles used or observed
by the earliest investigators of the
phenomenon.

The correct answer is C.

503

8.0 Critical Reasoning

504

8.0 Critical Reasoning

8.0 Critical Reasoning
Critical reasoning questions appear in the Verbal section of the GMAT® exam. The Verbal section uses
multiple‐choice questions to measure your ability to read and comprehend written material, to reason
and to evaluate arguments, and to correct written material to conform to standard written English.
Because the Verbal section includes content from a variety of topics, you may be generally familiar with
some of the material; however, neither the passages nor the questions assume knowledge of the topics
discussed. Critical reasoning questions are intermingled with reading comprehension and sentence
correction questions throughout the Verbal section of the test.
You will have 75 minutes to complete the Verbal section, or about 1¾ minutes to answer each question.
Although critical reasoning questions are based on written passages, these passages are shorter than
reading comprehension passages. They tend to be less than 100 words in length and generally are
followed by one or two questions. For these questions, you will see a split computer screen. The written
passage will remain visible at the top of the screen. In turn, each associated question will appear either
below or above the relevant passage. You will see only one question at a time.
Critical reasoning questions are designed to test the reasoning skills involved in (1) making arguments,
(2) evaluating arguments, and (3) formulating or evaluating a plan of action. The materials on which
questions are based are drawn from a variety of sources. The GMAT exam does not expect any
familiarity with the subject matter of those materials.
In these questions, you are to analyze the situation on which each question is based, and then select the
answer choice that most appropriately answers the question. Begin by reading the passages carefully,
then reading the five answer choices. If the correct answer is not immediately obvious to you, see
whether you can eliminate some of the wrong answers. Reading the passage a second time may be
helpful in illuminating subtleties that were not immediately evident.
Answering critical reasoning questions requires no specialized knowledge of any particular field; you
don’t have to have knowledge of the terminology and conventions of formal logic. The sample critical
reasoning questions in this chapter illustrate the variety of topics the test may cover, the kinds of
questions it may ask, and the level of analysis it requires.
The following pages describe what critical reasoning questions are designed to measure and present the
directions that will precede questions of this type. Sample questions and explanations of the correct
answers follow.

505

GMAT® Official Guide 2018

8.1 What Is Measured
Critical reasoning questions are designed to provide one measure of your ability to reason effectively in
the following areas:
• Argument construction
Questions in this category may ask you to recognize such things as the basic structure of an
argument, properly drawn conclusions, underlying assumptions, well‐supported explanatory
hypotheses, and parallels between structurally similar arguments.
• Argument evaluation
These questions may ask you to analyze a given argument and to recognize such things as factors
that would strengthen or weaken the given argument; reasoning errors committed in making that
argument; and aspects of the method by which the argument proceeds.
• Formulating and evaluating a plan of action
This type of question may ask you to recognize such things as the relative appropriateness,
effectiveness, or efficiency of different plans of action, factors that would strengthen or weaken the
prospects of success of a proposed plan of action, and assumptions underlying a proposed plan of
action.

8.2 Test-Taking Strategies
1. Read very carefully the set of statements on which a question is based.

Pay close attention to
• what is put forward as factual information
• what is not said but necessarily follows from what is said
• what is claimed to follow from facts that have been put forward
• how well substantiated are any claims that a particular conclusion follows from the facts that have
been put forward
In reading the arguments, it is important to pay attention to the logical reasoning used; the actual
truth of statements portrayed as fact is not important.
2. Identify the conclusion.

The conclusion does not necessarily come at the end of the text; it may come somewhere in the
middle or even at the beginning. Be alert to clues in the text that an argument follows logically from
another statement or statements in the text.
3. Determine exactly what each question asks.

You might find it helpful to read the question first, before reading the material on which it is based;
don’t assume that you know what you will be asked about an argument. An argument may have
obvious flaws, and one question may ask you to detect them. But another question may direct you to
select the one answer choice that does NOT describe a flaw in the argument.
4. Read all the answer choices carefully.

Do not assume that a given answer is the best without first reading all the choices.

506

8.3 Critical Reasoning The Directions

8.3 The Directions
These are the directions you will see for critical reasoning questions when you take the GMAT exam. If
you read them carefully and understand them clearly before going to sit for the test, you will not need to
spend too much time reviewing them when you are at the test center and the test is under way.
For these questions, select the best of the answer choices given.

507

GMAT® Official Guide 2018

8.4 Practice Questions
Each of the critical reasoning questions is based on a short argument, a set of statements, or a plan of
action. For each question, select the best answer of the choices given.

544. Neuroscientist: Memory evolved to help animals react
appropriately to situations they encounter by drawing
on the past experience of similar situations. But this
does not require that animals perfectly recall every
detail of all their experiences. Instead, to function well,
memory should generalize from past experiences that
are similar to the current one.

545. Astronomer: Most stars are born in groups of
thousands, each star in a group forming from the
same parent cloud of gas. Each cloud has a unique,
homogeneous chemical composition. Therefore,
whenever two stars have the same chemical
composition as each other, they must have originated
from the same cloud of gas.

The neuroscientist’s statements, if true, most strongly
support which of the following conclusions?

Which of the following, if true, would most strengthen
the astronomer’s argument?

(A)

At least some animals perfectly recall every
detail of at least some past experiences.

(A)

In some groups of stars, not every star
originated from the same parent cloud of gas.

(B)

Perfectly recalling every detail of all their past
experiences could help at least some animals
react more appropriately than they otherwise
would to new situations they encounter.

(B)

Clouds of gas of similar or identical chemical
composition may be remote from each other.

(C)

Whenever a star forms, it inherits the chemical
composition of its parent cloud of gas.

(C)

Generalizing from past experiences requires
clear memories of most if not all the details of
those experiences.

(D)

Many stars in vastly different parts of the
universe are quite similar in their chemical
compositions.

(D)

Recalling every detail of all past experiences
would be incompatible with any ability to
generalize from those experiences.

(E)

Astronomers can at least sometimes precisely
determine whether a star has the same chemical
composition as its parent cloud of gas.

(E)

Animals can often react more appropriately
than they otherwise would to situations they
encounter if they draw on generalizations from
past experiences of similar situations.

546. With employer-paid training, workers have the potential
to become more productive not only in their present
employment but also in any number of jobs with
different employers. To increase the productivity of their
workforce, many firms are planning to maintain or even
increase their investments in worker training. But some
training experts object that if a trained worker is hired
away by another firm, the employer that paid for the
training has merely subsidized a competitor. They note
that such hiring has been on the rise in recent years.
Which of the following would, if true, contribute most
to defeating the training experts’ objection to the firms’
strategy?
(A)

508

Firms that promise opportunities for
advancement to their employees get, on
average, somewhat larger numbers of job
applications from untrained workers than do
firms that make no such promise.

8.4 Critical Reasoning Practice Questions

(B)

In many industries, employees who take
continuing-education courses are more
competitive in the job market.

(C)

More and more educational and training
institutions are offering reduced tuition fees to
firms that subsidize worker training.

(D)

Research shows that workers whose training is
wholly or partially subsidized by their employer
tend to get at least as much training as do
workers who pay for all their own training.

(E)

For most firms that invest in training their
employees, the value added by that investment
in employees who stay exceeds the value lost
through other employees’ leaving to work for
other companies.

547. Candle Corporation’s television stations are likely to
have more income from advertisers than previously.
This is because advertisers prefer to reach people
in the 18‐ to 49‐year‐old age group and the number
of people in that group watching Candle television is
increasing. Furthermore, among Candle viewers, the
percentage of viewers 18 to 49 years old is increasing.
Which of the following, if true, would most strengthen
the argument that Candle Corporation will receive more
income from advertisers?
(A)

Advertisers carefully monitor the demographic
characteristics of television audiences and
purchase advertising time to reach the audiences
they prefer to reach.

(B)

Among people over 49 years old, fewer viewers
of Candle stations buy products advertised on
television than do viewers of other stations.

(C)

There will be increasingly more advertisements
on television that are directed at viewers who are
over 49 years old.

(D)

Candle stations plan to show reruns of television
shows during hours when other stations run
shows for the first time.

(E)

People 18 to 49 years old generally have less
disposable income to spend than do people over
49 years old.

548. A provincial government plans to raise the gasoline
tax to give people an incentive to drive less, reducing
traffic congestion in the long term. However, skeptics
point out that most people in the province live in areas
where cars are the only viable transportation to jobs
and stores and therefore cannot greatly change their
driving habits in response to higher gasoline prices.
In light of the skeptics’ objection, which of the
following, if true, would most logically support the
prediction that the government’s plan will achieve its
goal of reducing traffic congestion?
(A)

The revenue from the tax will be used to
make public transportation a viable means of
transportation to jobs and stores for far more
people.

(B)

The tax will encourage many residents to switch
to more fuel‐efficient cars, reducing air pollution
and other problems.

(C)

Because gasoline has been underpriced for
decades, the province has many neighborhoods
where cars are the only viable means of
transportation.

(D)

Most residents who cannot greatly change
their driving habits could compensate for high
gasoline prices by reducing other expenses.

(E)

Traffic congestion is an especially serious
problem for people for whom cars are the only
viable means of transportation.

509

GMAT® Official Guide 2018

549. Editorial: The roof of Northtown’s municipal equipment‐
storage building collapsed under the weight of last
week’s heavy snowfall. The building was constructed
recently and met local building‐safety codes in every
particular, except that the nails used for attaching roof
supports to the building’s columns were of a smaller
size than the codes specify for this purpose. Clearly,
this collapse exemplifies how even a single, apparently
insignificant departure from safety standards can have
severe consequences.
Which of the following, if true, most seriously weakens
the editorial’s argument?
(A)

The only other buildings to suffer roof collapses
from the weight of the snowfall were older
buildings constructed according to less exacting
standards than those in the codes.

(B)

The amount of snow that accumulated on the
roof of the equipment‐storage building was
greater than the predicted maximum that was
used in drawing up the safety codes.

(C)

Because the equipment‐storage building was not
intended for human occupation, some safety‐
code provisions that would have applied to an
office building did not apply to it.

(D)

The municipality of Northtown itself has the
responsibility for ensuring that buildings
constructed within its boundaries meet the
provisions of the building‐safety codes.

(E)

Because the equipment‐storage building was
used for storing snow‐removal equipment, the
building was almost completely empty when the
roof collapsed.

550. Political theorist: Even with the best spies, area
experts, and satellite surveillance, foreign policy
assessments can still lack important information.
In such circumstances intuitive judgment is vital. A
national leader with such judgment can make good
decisions about foreign policy even when current
information is incomplete, since __________.
Which of the following, if true, most logically completes
the argument?
(A)

510

the central reason for failure in foreign policy
decision making is the absence of critical
information

(B)

those leaders whose foreign policy decisions
have been highly ranked have also been found to
have good intuitive judgment

(C)

both intuitive judgment and good information are
required for sound decision making

(D)

good foreign policy decisions often lead to
improved methods of gathering information

(E)

intuitive judgment can produce good decisions
based on past experience, even when there are
important gaps in current information

551. During the earliest period of industrialization in Britain,
steam engines were more expensive to build and
operate than either windmills or water mills, the other
practicable sources of power for factories. Yet despite
their significant cost disadvantage, steam‐powered
factories were built in large numbers well before technical
improvements brought their cost down. Furthermore,
they were built even in regions where geographical
conditions permitted the construction of wind‐ and water‐
powered factories close to major markets.
Which of the following, if true, most helps to explain
the proliferation of steam‐powered factories during the
earliest period of industrialization in Britain?
(A)

In many areas of Britain, there were fewer steam‐
powered factories than wind‐ or water‐powered
factories in the earliest period of industrialization.

(B)

Unlike wind‐ or water‐powered factories, steam‐
powered factories were fueled with coal, which
sometimes had to be transported significant
distances from the mine to the site of the factory.

(C)

It was both difficult and expensive to convert
a factory from wind power or water power to
steam power.

(D)

In the early period of industrialization, many
goods sold in towns and cities could not be
mass‐produced in factories.

(E)

In Britain, the number of sites where a wind‐
or water‐powered factory could be built was
insufficient to provide for all of the demand for
factory‐produced goods at the time.

552. Snowmaking machines work by spraying a mist that
freezes immediately on contact with cold air. Because
the sudden freezing kills bacteria, QuickFreeze is
planning to market a wastewater purification system

8.4 Critical Reasoning Practice Questions

that works on the same principle. The process
works only when temperatures are cold, however,
so municipalities using it will still need to maintain a
conventional system.
Which of the following, if true, provides the strongest
grounds for a prediction that municipalities will buy
QuickFreeze’s purification system despite the need to
maintain a conventional purification system as well?
(A)

Bacteria are not the only impurities that must be
removed from wastewater.

(B)

Many municipalities have old wastewater
purification systems that need to be replaced.

(C)

Conventional wastewater purification systems
have not been fully successful in killing bacteria
at cold temperatures.

(D)

During times of warm weather, when it is not in
use, QuickFreeze’s purification system requires
relatively little maintenance.

(E)

Places where the winters are cold rarely have a
problem of water shortage.

553. Suncorp, a new corporation with limited funds, has
been clearing large sections of the tropical Amazon
forest for cattle ranching. This practice continues
even though greater profits can be made from rubber
tapping, which does not destroy the forest, than from
cattle ranching, which does destroy the forest.
Which of the following, if true, most helps to explain
why Suncorp has been pursuing the less profitable of
the two economic activities mentioned above?
(A)

The soil of the Amazon forest is very rich in
nutrients that are important in the development
of grazing lands.

(B)

Cattle‐ranching operations that are located in
tropical climates are more profitable than cattle‐
ranching operations that are located in cold‐
weather climates.

(C)

In certain districts, profits made from cattle
ranching are more heavily taxed than profits
made from any other industry.

(D)

Some of the cattle that are raised on land
cleared in the Amazon are killed by wildcats.

(E)

The amount of money required to begin a rubber‐
tapping operation is twice as high as the amount
needed to begin a cattle ranch.

554. Archaeologists use technology to analyze ancient sites.
It is likely that this technology will advance considerably
in the near future, allowing archaeologists to gather
more information than is currently possible. If they
study certain sites now, they risk contaminating or
compromising them for future studies. Therefore, in
order to maximize the potential for gathering knowledge
in the long run, a team of archaeologists plans to delay
the examination of a newly excavated site.
Which of the following would be most useful to
investigate for the purpose of evaluating the plan’s
prospects for achieving its goal?
(A)

Whether any of the contents of the site will
significantly deteriorate before the anticipated
technology is available

(B)

Whether there will continue to be improvements
on the relevant technology

(C)

Whether the team can study a site other than the
newly excavated site for the time being

(D)

Whether the site was inhabited by a very ancient
culture

(E)

Whether the anticipated technology will damage
objects under study

555. A newly discovered painting seems to be the work
of one of two seventeenth‐century artists, either the
northern German Johannes Drechen or the Frenchman
Louis Birelle, who sometimes painted in the same
style as Drechen. Analysis of the carved picture frame,
which has been identified as the painting’s original
seventeenth‐century frame, showed that it is made of
wood found widely in northern Germany at the time, but
rare in the part of France where Birelle lived. This shows
that the painting is most likely the work of Drechen.
Which of the following is an assumption that the
argument requires?
(A)

The frame was made from wood local to the
region where the picture was painted.

(B)

Drechen is unlikely to have ever visited the home
region of Birelle in France.

(C)

Sometimes a painting so resembles others of its
era that no expert is able to confidently decide
who painted it.

(D)

The painter of the picture chose the frame for
the picture.

(E)

The carving style of the picture frame is not
typical of any specific region of Europe.
511

GMAT® Official Guide 2018

556. Beginning in 1966 all new cars sold in Morodia were
required to have safety belts and power steering.
Previously, most cars in Morodia were without these
features. Safety belts help to prevent injuries in
collisions, and power steering helps to avoid collisions
in the first place. But even though in 1966 one-seventh
of the cars in Morodia were replaced with new cars,
the number of car collisions and collision-related
injuries did not decline.
Which of the following, if true about Morodia, most helps
to explain why the number of collisions and collisionrelated injuries in Morodia failed to decline in 1966?

Most of the goods that TrueSave ships are
electronic products that are highly vulnerable to
being damaged in transit.

(E)

TrueSave has lost some of its regular customers
as a result of the high number of damaged items
they received.

558. Parland’s alligator population has been declining in
recent years, primarily because of hunting. Alligators
prey heavily on a species of freshwater fish that is highly
valued as food by Parlanders, who had hoped that
the decline in the alligator population would lead to an
increase in the numbers of these fish available for human
consumption. Yet the population of this fish species has
also declined, even though the annual number caught for
human consumption has not increased.

(A)

Because of a driver-education campaign, most
drivers and passengers in cars that did have
safety belts used them in 1966.

(B)

Most of the new cars bought in 1966 were
bought in the months of January and February.

(C)

In 1965, substantially more than one-seventh of
the cars in Morodia were replaced with new cars.

(A)

An excessive reliance on the new safety features
led many owners of new cars to drive less
cautiously in 1966 than before.

The decline in the alligator population has meant
that fishers can work in some parts of lakes and
rivers that were formerly too dangerous.

(B)

Over the last few years, Parland’s commercial
fishing enterprises have increased the number of
fishing boats they use.

(C)

The main predator of these fish is another
species of fish on which alligators also prey.

(D)

Many Parlanders who hunt alligators do so
because of the high market price of alligator
skins, not because of the threat alligators pose
to the fish population.

(E)

In several neighboring countries through which
Parland’s rivers also flow, alligators are at risk of
extinction as a result of extensive hunting.

(D)

(E)

The seat belts and power steering put into new
cars sold in 1966 had to undergo strict qualitycontrol inspections by manufacturers, whether the
cars were manufactured in Morodia or not.

557. In order to reduce the number of items damaged
while in transit to customers, packaging consultants
recommended that the TrueSave mail‐order company
increase the amount of packing material so as to fill
any empty spaces in its cartons. Accordingly, TrueSave
officials instructed the company’s packers to use
more packing material than before, and the packers
zealously acted on these instructions and used as
much as they could. Nevertheless, customer reports
of damaged items rose somewhat.
Which of the following, if true, most helps to explain
why acting on the consultants’ recommendation failed
to achieve its goal?

512

(D)

(A)

The change in packing policy led to an increase
in expenditure on packing material and labor.

(B)

When packing material is compressed too densely,
it loses some of its capacity to absorb shock.

(C)

The amount of packing material used in a carton
does not significantly influence the ease with
which a customer can unpack the package.

Which of the following, if true, most helps to explain
the decline in the population of the fish species?

559. A certain automaker aims to increase its market
share by deeply discounting its vehicles’ prices for
the next several months. The discounts will cut into
profits, but because they will be heavily advertised the
manufacturer hopes that they will attract buyers away
from rival manufacturers’ cars. In the longer term, the
automaker envisions that customers initially attracted
by the discounts may become loyal customers.
In assessing the plan’s chances of achieving its aim, it
would be most useful to know which of the following?
(A)

Whether the automaker’s competitors are likely
to respond by offering deep discounts on their
own products

8.4 Critical Reasoning Practice Questions

(A)

Whether some of the automaker’s models will be
more deeply discounted than others

The first is a premise used to support the
argument’s main conclusion; the second is the
argument’s main conclusion.

(B)

Whether the automaker will be able to cut costs
sufficiently to maintain profit margins even when
the discounts are in effect

The first is the argument’s main conclusion;
the second is a premise used to support that
conclusion.

(C)

Whether an alternative strategy might enable the
automaker to enhance its profitability while holding
a constant or diminishing share of the market

The first is one of two premises used to support
the argument’s main conclusion; the second is
the other of those two premises.

(D)

The first is a premise used to support the
argument’s main conclusion; the second is a
premise used to support another conclusion
drawn in the argument.

(E)

The first is inferred from another statement in the
argument; the second is inferred from the first.

(B)

Whether the advertisements will be created by
the manufacturer’s current advertising agency

(C)
(D)

(E)

560. In virtually any industry, technological improvements
increase labor productivity, which is the output of
goods and services per person‐hour worked. In
Parland’s industries, labor productivity is significantly
higher than it is in Vergia’s industries. Clearly,
therefore, Parland’s industries must, on the whole, be
further advanced technologically than Vergia’s are.
The argument is most vulnerable to which of the
following criticisms?

562. The Maxilux car company’s design for its new luxury
model, the Max 100, included a special design for the
tires that was intended to complement the model’s
image. The winning bid for supplying these tires was
submitted by Rubco. Analysts concluded that the
bid would only just cover Rubco’s costs on the tires,
but Rubco executives claim that winning the bid will
actually make a profit for the company.

(A)

It offers a conclusion that is no more than a
paraphrase of one of the pieces of information
provided in its support.

(B)

It presents as evidence in support of a claim
information that is inconsistent with other
evidence presented in support of the same claim.

Which of the following, if true, most strongly justifies
the claim made by Rubco’s executives?

(C)

It takes one possible cause of a condition to
be the actual cause of that condition without
considering any other possible causes.

(A)

In any Maxilux model, the spare tire is exactly
the same make and model as the tires that are
mounted on the wheels.

(D)

It takes a condition to be the effect of something
that happened only after the condition already
existed.

(B)

Rubco holds exclusive contracts to supply
Maxilux with the tires for a number of other
models made by Maxilux.

(E)

It makes a distinction that presupposes the truth
of the conclusion that is to be established.

(C)

The production facilities for the Max 100 and
those for the tires to be supplied by Rubco are
located very near each other.

(D)

When people who have purchased a carefully
designed luxury automobile need to replace a
worn part of it, they almost invariably replace it
with a part of exactly the same make and type.

(E)

When Maxilux awarded the tire contract to
Rubco, the only criterion on which Rubco’s bid
was clearly ahead of its competitors’ bids was
price.

561. Chaco Canyon, a settlement of the ancient Anasazi
culture in North America, had massive buildings.
It must have been a major Anasazi center.
Analysis of wood samples shows that some of the
timber for the buildings came from the Chuska and
San Mateo mountains, 50 miles from Chaco Canyon.
Only a major cultural center would have the
organizational power to import timber from 50
miles away.
In the argument given, the two portions in boldface
play which of the following roles?

513

GMAT® Official Guide 2018

563. Which of the following most logically completes the
passage?
Most bicycle helmets provide good protection for the
top and back of the head, but little or no protection for
the temple regions on the sides of the head. A study
of head injuries resulting from bicycle accidents showed
that a large proportion were caused by blows to the
temple area. Therefore, if bicycle helmets protected this
area, the risk of serious head injury in bicycle accidents
would be greatly reduced, especially since ________.
(A)

among the bicyclists included in the study’s
sample of head injuries, only a very small
proportion had been wearing a helmet at the
time of their accident

(B)

even those bicyclists who regularly wear helmets
have a poor understanding of the degree and
kind of protection that helmets afford

(C)

a helmet that included protection for the temples
would have to be somewhat larger and heavier
than current helmets

(D)

the bone in the temple area is relatively thin and
impacts in that area are thus very likely to cause
brain injury

(E)

bicyclists generally land on their arm or shoulder
when they fall to the side, which reduces the
likelihood of severe impacts on the side of the head

564. Which of the following most logically completes the
argument?
In a typical year, Innovair’s airplanes are involved in
35 collisions while parked or being towed in airports,
with a resulting yearly cost of $1,000,000 for repairs.
To reduce the frequency of ground collisions, Innovair
will begin giving its ground crews additional training,
at an annual cost of $500,000. Although this will cut
the number of ground collisions by about half at best,
the drop in repair costs can be expected to be much
greater, since __________.

514

(D)

the $500,000 cost figure for the additional
training of ground crews includes the wages that
those crews will earn during the time spent in
actual training

(E)

most ground collisions have been caused by the
least experienced ground‐crew members

565. Hunter: Many people blame hunters alone for
the decline in Greenrock National Forest’s deer
population over the past ten years. Yet clearly,
black bears have also played an important role
in this decline. In the past ten years, the forest’s
protected black bear population has risen sharply, and
examination of black bears found dead in the forest
during the deer hunting season showed that a number
of them had recently fed on deer.
In the hunter’s argument, the portion in boldface plays
which of the following roles?
(A)

It is the main conclusion of the argument.

(B)

It is a finding that the argument seeks to explain.

(C)

It is an explanation that the argument concludes
is correct.

(D)

It provides evidence in support of the main
conclusion of the argument.

(E)

It introduces a judgment that the argument
opposes.

566. Many agriculturally intensive areas of the world are
beginning to encounter water scarcity problems. As
a result, many farmers in these areas are likely to
reduce their output as the water supply they need in
order to maintain production shrinks. However, one
group of farmers in such a region plans to increase
their production by implementing techniques for water
conservation.
Which of the following, if true, would most strongly
support the prediction that the group’s plan will
succeed?

(A)

most ground collisions happen when ground
crews are rushing to minimize the time a delayed
airplane spends on the ground

(A)

Farmers that can gain a larger share of the food
market in their regions will be better positioned
to control more water resources.

(B)

a ground collision typically occurs when there
are no passengers on the airplane

(B)

Most agricultural practices in areas with water
shortages are water‐intensive.

(C)

the additional training will focus on helping
ground crews avoid those kinds of ground
collisions that cause the most costly damage

(C)

Other regions of the world not facing water
shortages are likely to make up for the reduction
in agricultural output.

8.4 Critical Reasoning Practice Questions

(D)

Demand for agricultural products in the group’s
region is not expected to decline.

(A)

One of the taller tables takes up less floor space
than one of the standard tables.

(E)

More than half the water used for agriculture
in the farmers’ region is lost to evaporation or
leakage from irrigation channels.

(B)

Diners seated on stools typically do not linger over
dinner as long as diners seated at standard tables.

(C)

Since the restaurant will replace only some of its
standard tables, it can continue to accommodate
customers who do not care for the taller tables.

(D)

Few diners are likely to avoid the restaurant
because of the new seating arrangement.

(E)

The standard tables being replaced by tall tables
would otherwise have to be replaced with new
standard tables at a greater expense.

567. Physician: The hormone melatonin has shown promise
as a medication for sleep disorders when taken in
synthesized form. Because the long‐term side effects
of synthetic melatonin are unknown, however, I cannot
recommend its use at this time.
Patient: Your position is inconsistent with your usual
practice. You prescribe many medications that you
know have serious side effects, so concern about side
effects cannot be the real reason you will not prescribe
melatonin.
The patient’s argument is flawed because it fails to
consider that
(A)

the side effects of synthetic melatonin might
be different from those of naturally produced
melatonin

(B)

it is possible that the physician does not believe
that melatonin has been conclusively shown to
be effective

(C)

sleep disorders, if left untreated, might lead to
serious medical complications

(D)

the side effects of a medication can take some
time to manifest themselves

(E)

known risks can be weighed against known
benefits, but unknown risks cannot

568. Hollywood restaurant is replacing some of its standard
tables with tall tables and stools. The restaurant
already fills every available seat during its operating
hours, and the change in seating arrangements will
not result in an increase in the restaurant’s seating
capacity. Nonetheless, the restaurant’s management
expects revenue to increase as a result of the seating
change without any concurrent change in menu,
prices, or operating hours.

569. A major network news organization experienced a
drop in viewership in the week following the airing of a
controversial report on the economy. The network also
received a very large number of complaints regarding
the report. The network, however, maintains that
negative reactions to the report had nothing to do with
its loss of viewers.
Which of the following, if true, most strongly supports
the network’s position?
(A)

The other major network news organizations
reported similar reductions in viewership during
the same week.

(B)

The viewers who registered complaints with
the network were regular viewers of the news
organization’s programs.

(C)

Major network news organizations publicly
attribute drops in viewership to their own reports
only when they receive complaints about those
reports.

(D)

This was not the first time that this network news
organization has aired a controversial report
on the economy that has inspired viewers to
complain to the network.

(E)

Most network news viewers rely on network
news broadcasts as their primary source of
information regarding the economy.

Which of the following, if true, provides the best reason
for the expectation?

515

GMAT® Official Guide 2018

570. Only a reduction of 10 percent in the number of
scheduled flights using Greentown’s airport will allow
the delays that are so common there to be avoided.
Hevelia airstrip, 40 miles away, would, if upgraded
and expanded, be an attractive alternative for fully
20 percent of the passengers using Greentown airport.
Nevertheless, experts reject the claim that turning
Hevelia into a full‐service airport would end the chronic
delays at Greentown.
Which of the following, if true, most helps to justify the
experts’ position?
(A)

Turning Hevelia into a full‐service airport would
require not only substantial construction at the
airport itself, but also the construction of new
access highways.

(B)

A second largely undeveloped airstrip close to
Greentown airport would be a more attractive
alternative than Hevelia for many passengers
who now use Greentown.

(C)

Hevelia airstrip lies in a relatively undeveloped
area but would, if it became a full‐service airport,
be a magnet for commercial and residential
development.

(D)

If an airplane has to wait to land, the extra jet
fuel required adds significantly to the airline’s
costs.

(E)

Several airlines use Greentown as a regional hub,
so that most flights landing at Greentown have
many passengers who then take different flights
to reach their final destinations.

571. For similar cars and comparable drivers, automobile
insurance for collision damage has always cost more
in Greatport than in Fairmont. Police studies, however,
show that cars owned by Greatport residents are, on
average, slightly less likely to be involved in a collision
than cars in Fairmont. Clearly, therefore, insurance
companies are making a greater profit on collision‐
damage insurance in Greatport than in Fairmont.
In evaluating the argument, it would be most useful to
compare

516

(A)

the level of traffic congestion in Greatport with
the level of traffic congestion in Fairmont

(B)

the cost of repairing collision damage in
Greatport with the cost of repairing collision
damage in Fairmont

(C)

the rates Greatport residents pay for other
forms of insurance with the rates paid for similar
insurance by residents of Fairmont

(D)

the condition of Greatport’s roads and streets with
the condition of Fairmont’s roads and streets

(E)

the cost of collision‐damage insurance in
Greatport and Fairmont with that in other cities

572. The air quality board recently informed Coffee Roast,
a small coffee roasting firm, of a complaint regarding
the smoke from its roaster. Recently enacted air quality
regulations require machines roasting more than
10 pounds of coffee to be equipped with expensive
smoke‐dissipating afterburners. The firm, however,
roasts only 8 pounds of coffee at a time. Nevertheless,
the company has decided to purchase and install an
afterburner.
Which of the following, if true, most strongly supports
the firm’s decision?
(A)

Until settling on the new air quality regulations,
the board had debated whether to require
afterburners for machines roasting more than
5 pounds of coffee at a time.

(B)

Coffee roasted in a machine equipped with an
afterburner has its flavor subtly altered.

(C)

The cost to the firm of an afterburner is less than
the cost of replacing its roaster with a smaller one.

(D)

Fewer complaints are reported in areas that
maintain strict rules regarding afterburners.

(E)

The firm has reason to fear that negative publicity
regarding the complaints could result in lost sales.

573. The tulu, a popular ornamental plant, does not
reproduce naturally, and is only bred and sold by
specialized horticultural companies. Unfortunately, the
tulu is easily devastated by a contagious fungal rot.
The government ministry plans to reassure worried
gardeners by requiring all tulu plants to be tested
for fungal rot before being sold. However, infected
plants less than 30 weeks old have generally not built
up enough fungal rot in their systems to be detected
reliably. And many tulu plants are sold before they are
24 weeks old.
Which of the following, if performed by the government
ministry, could logically be expected to overcome the
problem with their plan to test for the fungal rot?

8.4 Critical Reasoning Practice Questions

(A)

Releasing a general announcement that tulu
plants less than 30 weeks old cannot be
effectively tested for fungal rot

(B)

Requiring all tulu plants less than 30 weeks old
to be labeled as such

(C)

Researching possible ways to test tulu plants
less than 24 weeks old for fungal rot

(D)

Ensuring that tulu plants not be sold before they
are 30 weeks old

(E)

Quarantining all tulu plants from horticultural
companies at which any case of fungal rot has
been detected until those tulu plants can be
tested for fungal rot

574. People who do regular volunteer work tend to live longer,
on average, than people who do not. It has been found that
“doing good,” a category that certainly includes volunteer
work, releases endorphins, the brain’s natural opiates,
which induce in people a feeling of well‐being. Clearly, there
is a connection: Regular releases of endorphins must in
some way help to extend people’s lives.
Which of the following, if true, most seriously
undermines the force of the evidence given as support
for the hypothesis that endorphins promote longevity?
(A)

People who do regular volunteer work are
only somewhat more likely than others to
characterize the work they do for a living as
“doing good.”

(B)

Although extremely high levels of endorphins
could be harmful to health, such levels are never
reached as a result of the natural release of
endorphins.

(C)

There are many people who have done some
volunteer work but who do not do such work
regularly.

available for research, some legislators propose
requiring all fossils that are found in Mernia to be sold
only to universities or museums.
Which of the following, if true, most strongly indicates
that the legislators’ proposal will fail to achieve its goal?
(A)

Some fossil hunters in Mernia are not
commercial fossil hunters, but rather are
amateurs who keep the fossils that they find.

(B)

Most fossils found in Mernia are common types
that have little scientific interest.

(C)

Commercial fossil hunters in Mernia currently sell
some of the fossils they find to universities and
museums.

(D)

Many universities in Mernia do not engage in
fossil research.

(E)

Most fossils are found by commercial fossil
hunters, and they would give up looking for
fossils if they were no longer allowed to sell to
individual collectors.

576. The Eurasian ruffe, a fish species inadvertently
introduced into North America’s Great Lakes in recent
years, feeds on the eggs of lake whitefish, a native
species, thus threatening the lakes’ natural ecosystem.
To help track the ruffe’s spread, government agencies
have produced wallet‐sized cards about the ruffe. The
cards contain pictures of the ruffe and explain the
danger they pose; the cards also request anglers to
report any ruffe they catch.
Which of the following, if true, would provide most
support for the prediction that the agencies’ action will
have its intended effect?
(A)

The ruffe has spiny fins that make it unattractive
as prey.

(D)

People tend not to become involved in regular
volunteer work unless they are healthy and
energetic to begin with.

(B)

Ruffe generally feed at night, but most
recreational fishing on the Great Lakes is done
during daytime hours.

(E)

Releases of endorphins are responsible for the
sense of well‐being experienced by many long‐
distance runners while running.

(C)

Most people who fish recreationally on the Great
Lakes are interested in the preservation of the lake
whitefish because it is a highly prized game fish.

(D)

The ruffe is one of several nonnative species in
the Great Lakes whose existence threatens the
survival of lake whitefish populations there.

(E)

The bait that most people use when fishing for
whitefish on the Great Lakes is not attractive to
ruffe.

575. In Mernia commercial fossil hunters often sell
important fossils they have found, not to universities or
museums, but to individual collectors, who pay much
better but generally do not allow researchers access
to their collections. To increase the number of fossils

517

GMAT® Official Guide 2018

577. Which of the following most logically completes the
argument below?
Although the number of large artificial satellites orbiting
the Earth is small compared to the number of small
pieces of debris in orbit, the large satellites interfere more
seriously with telescope observations because of the
strong reflections they produce. Because many of those
large satellites have ceased to function, the proposal
has recently been made to eliminate interference from
nonfunctioning satellites by exploding them in space. This
proposal, however, is ill conceived, since ________.
(A)

many nonfunctioning satellites remain in orbit for
years

(B)

for satellites that have ceased to function,
repairing them while they are in orbit would be
prohibitively expensive

(C)

there are no known previous instances of
satellites’ having been exploded on purpose

(D)

the only way to make telescope observations
without any interference from debris in orbit is
to use telescopes launched into extremely high
orbits around the Earth

(E)

a greatly increased number of small particles
in Earth’s orbit would result in a blanket of
reflections that would make certain valuable
telescope observations impossible

578. Thyrian lawmaker: Thyria’s Cheese Importation Board
inspects all cheese shipments to Thyria and rejects
shipments not meeting specified standards. Yet only
1 percent is ever rejected. Therefore, since the health
consequences and associated economic costs of not
rejecting that 1 percent are negligible, whereas the
board’s operating costs are considerable, for economic
reasons alone the board should be disbanded.
Consultant: I disagree. The threat of having their
shipments rejected deters many cheese exporters
from shipping substandard product.
The consultant responds to the lawmaker’s argument by
(A)

(B)

518

(C)

objecting to the lawmaker’s introducing into the
discussion factors that are not strictly economic

(D)

pointing out a benefit of maintaining the board, which
the lawmaker’s argument has failed to consider

(E)

shifting the discussion from the argument at hand to
an attack on the integrity of the cheese inspectors

579. The growing popularity of computer‐based activities
was widely expected to result in a decline in television
viewing, since it had been assumed that people lack
sufficient free time to maintain current television viewing
levels while spending increasing amounts of free time
on the computer. That assumption, however, is evidently
false: In a recent mail survey concerning media use, a
very large majority of respondents who report increasing
time spent per week using computers report no change
in time spent watching television.
Which of the following would it be most useful to
determine in order to evaluate the argument?
(A)

Whether a large majority of the survey
respondents reported watching television regularly

(B)

Whether the amount of time spent watching
television is declining among people who report
that they rarely or never use computers

(C)

Whether the type of television programs a person
watches tends to change as the amount of time
spent per week using computers increases

(D)

Whether a large majority of the computer owners
in the survey reported spending increasing
amounts of time per week using computers

(E)

Whether the survey respondents’ reports of time
spent using computers included time spent using
computers at work

580. Although the school would receive financial benefits if
it had soft drink vending machines in the cafeteria, we
should not allow them. Allowing soft drink machines there
would not be in our students’ interest. If our students
start drinking more soft drinks, they will be less healthy.
The argument depends on which of the following?

rejecting the lawmaker’s argument while proposing
that the standards according to which the board
inspects imported cheese should be raised

(A)

providing evidence that the lawmaker’s argument
has significantly overestimated the cost of
maintaining the board

If the soft drink vending machines were placed in
the cafeteria, students would consume more soft
drinks as a result.

(B)

The amount of soft drinks that most students at
the school currently drink is not detrimental to
their health.

8.4 Critical Reasoning Practice Questions

(C)

Students are apt to be healthier if they do not
drink soft drinks at all than if they just drink small
amounts occasionally.

(D)

Students will not simply bring soft drinks from
home if the soft drink vending machines are not
placed in the cafeteria.

(E)

The school’s primary concern should be to
promote good health among its students.

581. Many athletes inhale pure oxygen after exercise in an
attempt to increase muscular reabsorption of oxygen.
Measured continuously after exercise, however, the
blood lactate levels of athletes who inhale pure oxygen
are practically identical, on average, to those of athletes
who breathe normal air. The lower the blood lactate level
is, the higher the muscular reabsorption of oxygen is.
If the statements above are all true, they most strongly
support which of the following conclusions?

(A)

the life of Prince Sviatov is not the subject of any
other medieval Moringian texts

(B)

the author of Sviatovin intended it to provide as
accurate a report about Prince Sviatov’s exploits
as possible

(C)

the diagram accurately represents the
composition of Sviatov’s family at the time
Sviatovin was written

(D)

Sviatovin is the earliest Moringian text whose
composition can be dated to within a few years

(E)

Sviatovin was not written by Sviatov’s father
himself

583. A study of ticket sales at a summer theater festival
found that people who bought tickets to individual
plays had a no‐show rate of less than 1 percent,
while those who paid in advance for all ten plays
being performed that summer had a no‐show rate
of nearly 30 percent. This may be at least in part
because the greater the awareness customers retain
about the cost of an item, the more likely they are to
use it.

(A)

Athletes’ muscular reabsorption of oxygen is not
increased when they inhale pure oxygen instead
of normal air.

(B)

High blood lactate levels cannot be reduced.

(C)

Blood lactate levels are a poor measure of
oxygen reabsorption by muscles.

(D)

The amount of oxygen reabsorbed by an
athlete’s muscles always remains constant.

(A)

The price per ticket was slightly cheaper for
those who bought all ten tickets in advance.

(E)

The inhaling of pure oxygen has no legitimate
role in athletics.

(B)

Many people who attended the theater festival
believed strongly that they should support it
financially.

(C)

Those who attended all ten plays became eligible
for a partial refund.

(D)

Usually, people who bought tickets to individual
plays did so immediately prior to each
performance that they attended.

(E)

People who arrived just before the performance
began could not be assured of obtaining seats in
a preferred location.

582. Which of the following most logically completes the
argument?
Sviatovin is a medieval Moringian text whose author
and exact date of composition are unknown. However,
the events in the life of Prince Sviatov that the text
describes occurred in 1165, and in the diagram of
Sviatov’s family that accompanies the text his father,
who died in 1167, is identified as still living. Thus
Sviatovin must have been written between 1165 and
1167, assuming that ________.

Which of the following would, if true, best serve as an
alternative explanation of the results of the study?

519

GMAT® Official Guide 2018

584. Although there is no record of poet Edmund Spenser’s
parentage, we do know that as a youth Spenser
attended the Merchant Tailors’ School in London for
a period between 1560 and 1570. Records from this
time indicate that the Merchant Tailors’ Guild then had
only three members named Spenser: Robert Spenser,
listed as a gentleman; Nicholas Spenser, elected the
Guild’s Warden in 1568; and John Spenser, listed as a
“journeyman cloth‐maker.” Of these, the last was likely
the least affluent of the three—and most likely Edmund’s
father, since school accounting records list Edmund as
a scholar who attended the school at a reduced fee.
Which of the following is an assumption on which the
argument depends?
(A)

Anybody in sixteenth‐century London who made
clothing professionally would have had to be a
member of the Merchant Tailors’ Guild.

(B)

The fact that Edmund Spenser attended the
Merchant Tailors’ School did not necessarily
mean that he planned to become a tailor.

(C)

(D)

(E)

No member of the Guild could become Guild
warden in sixteenth‐century London unless he
was a gentleman.
Most of those whose fathers were members of
the Merchant Tailors’ Guild were students at the
Merchant Tailors’ School.
The Merchant Tailors’ School did not reduce its fees
for the children of the more affluent Guild members.

585. Trancorp currently transports all its goods to Burland
Island by truck. The only bridge over the channel
separating Burland from the mainland is congested,
and trucks typically spend hours in traffic. Trains
can reach the channel more quickly than trucks, and
freight cars can be transported to Burland by barges
that typically cross the channel in an hour. Therefore,
to reduce shipping time, Trancorp plans to switch to
trains and barges to transport goods to Burland.
Which of the following would be most important
to know in determining whether Trancorp’s plan, if
implemented, is likely to achieve its goal?

520

(A)

Whether transportation by train and barge
would be substantially less expensive than
transportation by truck

(B)

Whether there are boats that can make the trip
between the mainland and Burland faster than
barges can

(C)

Whether loading the freight cars onto barges is
very time consuming

(D)

Whether the average number of vehicles
traveling over the bridge into Burland has been
relatively constant in recent years

(E)

Whether most trucks transporting goods into
Burland return to the mainland empty

586. Rainwater contains hydrogen of a heavy form called
deuterium. The deuterium content of wood reflects
the deuterium content of rainwater available to trees
during their growth. Wood from trees that grew
between 16,000 and 24,000 years ago in North
America contains significantly more deuterium than
wood from trees growing today. But water trapped
in several North American caves that formed during
that same early period contains significantly less
deuterium than rainwater in North America contains
today.
Which of the following, if true, most helps to reconcile
the two findings?
(A)

There is little deuterium in the North American
caves other than the deuterium in the water
trapped there.

(B)

Exposure to water after a tree has died does not
change the deuterium content of the wood.

(C)

Industrialization in North America over the past
100 years has altered the deuterium content of
rain.

(D)

Trees draw on shallow groundwater from rain
that falls during their growth, whereas water
trapped in caves may have fallen as rainwater
thousands of years before the caves formed.

(E)

Wood with a high deuterium content is no more
likely to remain preserved for long periods than
is wood with a low deuterium content.

587. Which of the choices most logically completes the
following argument?
NowNews, although still the most popular magazine
covering cultural events in Kalopolis, has recently
suffered a significant drop in advertising revenue
because of falling circulation. Many readers have
begun buying a competing magazine that, at 50
cents per copy, costs less than NowNews at $1.50
per copy. In order to boost circulation and thus
increase advertising revenue, NowNews’s publisher

8.4 Critical Reasoning Practice Questions

has proposed making it available at no charge.
However, this proposal has a serious drawback,
since ________.
(A)

those Kalopolis residents with the greatest
interest in cultural events are regular readers of
both magazines.

(B)

one reason NowNews’s circulation fell was that
its competitor’s reporting on cultural events was
superior.

(C)

the newsstands and stores that currently sell
NowNews will no longer carry it if it is being
given away for free.

(D)

at present, 10 percent of the total number of copies
of each issue of NowNews are distributed free to
students on college campuses in the Kalopolis area.

(E)

NowNews’s competitor would begin to lose large
amounts of money if it were forced to lower its
cover price.

588. Archaeologist: Researchers excavating a burial site
in Cyprus found a feline skeleton lying near a human
skeleton. Both skeletons were in the same sediment at
the same depth and equally well‐preserved, suggesting
that the feline and human were buried together
about 9,500 years ago. This shows that felines were
domesticated around the time farming began, when
they would have been useful in protecting stores of
grain from mice.

(D)

In Cyprus, there are many burial sites dating
from around 9,500 years ago in which the
remains of wild animals appear to have been
buried alongside human remains.

(E)

Before felines were domesticated, early farmers
had no effective way to protect stores of grain
from mice.

589. The heavy traffic in Masana is a growing drain on the
city’s economy—the clogging of the streets of the
central business district alone cost the economy more
than $1.2 billion over the past year. In order to address
this problem, officials plan to introduce congestion
pricing, by which drivers would pay to enter the city’s
most heavily trafficked areas during the busiest times
of the day.
Which of the following, if true, would most strongly
indicate that the plan will be a success?
(A)

Approximately one-fifth of the vehicles in the
central business district are in transit from one
side of the city to the other.

(B)

Planners expect that, without congestion pricing,
traffic in Masana is likely to grow by 6 percent in
the next five years.

(C)

In other urban areas, congestion pricing has
strongly encouraged carpooling (sharing of rides
by private commuters).

(D)

Several studies have shown that a reduction in
traffic of 15 percent in Masana could result in
5,500 or more new jobs.

(E)

Over 30 percent of the vehicles in the city’s
center are occupied by more than one person.

Which of the following, if true, would most seriously
weaken the archaeologist’s argument?
(A)

Archaeologists have not found any remains of stores
of grain in the immediate vicinity of the burial site.

(B)

The burial site in Cyprus is substantially older
than any other known burial site in which a feline
skeleton and a human skeleton appear to have
been buried together.

(C)

Paintings found near the burial site seem to show
people keeping felines as domestic companions,
but do not show felines hunting mice.

521

GMAT® Official Guide 2018

590. Economist: The most economically efficient way to
reduce emissions of air pollutants is to tax them in
proportion to the damage they are likely to cause. But
in Country Y, many serious pollutants are untaxed and
unregulated, and policy makers strongly oppose new
taxes. Therefore, the best way to achieve a reduction
in air pollutant emissions in Country Y would be to
institute fixed upper limits on them.
Which of the following is an assumption of the
economist’s argument?
(A)

Policy makers in Country Y oppose all new taxes
equally strongly, regardless of any benefits they
may provide.

(B)

Country Y’s air pollutant emissions would not fall
significantly if they were taxed in proportion to
the damage they are likely to cause.

(C)

Policy makers in Country Y strongly favor
reductions in air pollutant emissions.

(D)

Country Y’s policy makers believe that air
pollutant emissions should be reduced with
maximum economic efficiency.

(E)

Policy makers in Country Y do not oppose
setting fixed upper limits on air pollutant
emissions as strongly as they oppose new taxes.

591. Humans get Lyme disease from infected ticks. Ticks
get infected by feeding on animals with Lyme disease,
but the ease of transmission from host animal to tick
varies. With most species of host animal, transmission
of Lyme disease to ticks is extremely rare, but white‐
footed mice are an exception, readily passing Lyme
disease to ticks. And white‐footed mouse populations
greatly expand, becoming the main food source for
ticks, in areas where biodiversity is in decline.
The information in the passage most strongly supports
which of the following?

522

(A)

In areas where many humans are infected with
Lyme disease, the proportion of ticks infected
with Lyme disease is especially high.

(B)

Very few animals that live in areas where there
are no white‐footed mice are infected with Lyme
disease.

(C)

Humans are less at risk of contracting Lyme
disease in areas where biodiversity is high.

(D)

Ticks feed on white‐footed mice only when other
host species are not available to them.

(E)

The greater the biodiversity of an area, the more
likely any given host animal in that area is to
pass Lyme disease to ticks.

592. Many industrialized nations are trying to reduce
atmospheric concentrations of carbon dioxide, a gas
released by the burning of fossil fuels. One proposal
is to replace conventional cement, which is made with
calcium carbonate, by a new “eco‐cement.” This new
cement, made with magnesium carbonate, absorbs
large amounts of carbon dioxide when exposed to the
atmosphere. Therefore, using eco‐cement for new
concrete building projects will significantly help reduce
atmospheric concentrations of carbon dioxide.
Which of the following, if true, most strengthens the
argument?
(A)

The cost of magnesium carbonate, currently
greater than the cost of calcium carbonate,
probably will fall as more magnesium carbonate
is used in cement manufacture.

(B)

Eco‐cement is strengthened when absorbed
carbon dioxide reacts with the cement.

(C)

Before the development of eco‐cement,
magnesium‐based cement was considered too
susceptible to water erosion to be of practical use.

(D)

The manufacture of eco‐cement uses considerably
less fossil fuel per unit of cement than the
manufacture of conventional cement does.

(E)

Most building‐industry groups are unaware of the
development or availability of eco‐cement.

593. Which of the following most logically completes the
argument below?
Davison River farmers are currently deciding between
planting winter wheat this fall or spring wheat next
spring. Winter wheat and spring wheat are usually
about equally profitable. Because of new government
restrictions on the use of Davison River water for
irrigation, per acre yields for winter wheat, though not
for spring wheat, would be much lower than average.
Therefore, planting spring wheat will be more profitable
than planting winter wheat, since ________.

8.4 Critical Reasoning Practice Questions

(A)

the smaller‐than‐average size of a winter wheat
harvest this year would not be compensated for
by higher winter wheat prices

(B)

new crops of spring wheat must be planted
earlier than the time at which standing crops of
winter wheat are ready to be harvested

(C)

Which of the following, if true, would most support
the scholar’s hypothesis concerning the date of the
Testament?
(A)

The sole evidence that historians have had that
William Thorpe died no earlier than 1460 was the
presumed date of publication of the Testament.

the spring wheat that farmers in the Davison River
region plant is well adapted to the soil of the region

(B)

In the preface to the 1530 publication, the editor
attributes both works to William Thorpe.

(D)

spring wheat has uses that are different from
those of winter wheat

(C)

(E)

planting spring wheat is more profitable than
planting certain other crops, such as rye

Few writers in fifteenth‐century England
marked dates in their works using only Roman
numerals.

(D)

The Testament alludes to a date, “Friday,
September 20,” as apparently contemporaneous
with the writing of the Testament, and September
20 fell on a Friday in 1409 but not in 1460.

(E)

The Testament contains few references to
historical events that occurred later than 1406.

594. Advertisement: When your car’s engine is running at
its normal operating temperature, any major brand
of motor oil will protect it about as well as Tuff does.
When the engine is cold, it is a different story: Tuff
motor oil flows better at lower temperatures than its
major competitors do. So, if you want your car’s engine
to have maximum protection, you should use Tuff.
Which of the following, if true, most strengthens the
argument in the advertisement?

596. To reduce productivity losses from employees calling
in sick, Corporation X implemented a new policy
requiring employees to come into work unless they
were so sick that they had to go to a doctor. But a
year after the policy was implemented, a study found
that Corporation X’s overall productivity losses due to
reported employee illnesses had increased.

(A)

Tuff motor oil provides above‐average protection
for engines that happen to overheat.

(B)

Tuff motor oil is periodically supplied free of
charge to automobile manufacturers to use in
factory‐new cars.

Which of the following, if true, would best explain why
the policy produced the reverse of its intended effect?

(C)

Tuff motor oil’s share of the engine oil market
peaked three years ago.

(A)

(D)

Tuff motor oil, like any motor oil, is thicker and
flows less freely at cold temperatures than at hot
temperatures.

After the policy was implemented, employees
more frequently went to the doctor when they
felt sick.

(B)

Before the policy was implemented, employees
who were not sick at all often called in sick.

Tuff motor oil is manufactured at only one
refinery and shipped from there to all markets.

(C)

Employees coming into work when sick often
infect many of their coworkers.

(D)

Unusually few employees became genuinely
sick during the year after the policy was
implemented.

(E)

There are many other factors besides employee
illness that can adversely affect productivity.

(E)

595. The Testament of William Thorpe was published around
1530 as an appendix to Thorpe’s longer Examination.
Many scholars, however, doubt the attribution of
the Testament to Thorpe because, whereas the
Examination is dated 1406, the Testament is dated
1460. One scholar has recently argued that the 1460
date be amended to 1409, based on the observation
that when these numbers are expressed as Roman
numerals, MCCCCLX and MCCCCIX, it becomes easy
to see how the dates might have become confused
through scribal error.

523

GMAT® Official Guide 2018

597. Advertising by mail has become much less effective,
with fewer consumers responding. Because
consumers are increasingly overwhelmed by the sheer
amount of junk mail they receive, most discard almost
all offers without considering them. Thus, an effective
way for corporations to improve response rates would
be to more carefully target the individuals to whom
they mail advertising, thereby cutting down on the
amount of junk mail each consumer receives.

(C)

Despite major cutbacks in most other areas
of operation, the petrochemical industry has
devoted more of its resources to environmental
and safety measures in the last five years than in
the preceding five years.

(D)

There is evidence that the most damaging of the
recent oil spills would have been prevented had
cost‐cutting measures not been instituted.

(E)

Both the large fines and the adverse publicity
generated by the most recent oil spill have
prompted the petrochemical industry to increase
the resources devoted to oil‐spill prevention.

Which of the following, if true, would most support the
recommendation above?
(A)

There are cost‐effective means by which
corporations that currently advertise by mail
could improve response rates.

(B)

Many successful corporations are already
carefully targeting the individuals to whom they
mail advertising.

(C)

Any consumer who, immediately after receiving
an advertisement by mail, merely glances at it, is
very likely to discard it.

(D)

Improvements in the quality of the advertising
materials used in mail that is carefully targeted
to individuals can improve the response rate for
such mail.

(E)

Response rates to carefully targeted advertisements
by mail are considerably higher, on average, than
response rates to most other forms of advertising.

598. Petrochemical industry officials have said that
the extreme pressure exerted on plant managers
during the last five years to improve profits by
cutting costs has done nothing to impair the
industry’s ability to operate safely. However,
environmentalists contend that the recent rash of
serious oil spills and accidents at petrochemical
plants is traceable to cost‐cutting measures.
Which of the following, if true, would provide the
strongest support for the position held by industry
officials?

524

(A)

The petrochemical industry benefits if accidents do
not occur, since accidents involve risk of employee
injury as well as loss of equipment and product.

(B)

Petrochemical industry unions recently
demanded that additional money be spent on
safety and environmental protection measures,
but the unions readily abandoned those demands
in exchange for job security.

599. Economist: The price of tap water in our region should
be raised drastically. Supplies in local freshwater
reservoirs have been declining for years because
water is being used faster than it can be replenished.
Since the price of tap water has been low, few users
have bothered to adopt even easy conservation
measures.
The two sections in boldface play which of the
following roles in the economist’s argument?
(A)

The first is a conclusion for which support is
provided, and which in turn supports the main
conclusion; the second is the main conclusion.

(B)

The first is an observation for which the second
provides an explanation; the second is the main
conclusion but not the only conclusion.

(C)

The first is a premise supporting the argument’s
main conclusion; so is the second.

(D)

The first is the only conclusion; the second
provides an explanation for the first.

(E)

The first is the main conclusion; the second is
a conclusion for which support is provided, and
which in turn supports the first.

600. Politician: Hybrid cars use significantly less fuel
per kilometer than nonhybrids. And fuel produces
air pollution, which contributes to a number of
environmental problems. Motorists can save money by
driving cars that are more fuel efficient, and they will
be encouraged to drive hybrid cars if we make them
aware of that fact. Therefore, we can help reduce the
total amount of pollution emitted by cars in this country
by highlighting this advantage of hybrid cars.
Which of the following, if true, would most indicate a
vulnerability of the politician’s argument?

8.4 Critical Reasoning Practice Questions

(A)

People with more fuel‐efficient cars typically drive
more than do those with less fuel‐efficient cars.

(B)

Not all air pollution originates from automobiles.

(C)

Hybrid cars have already begun to gain popularity.

(D)

Fuel‐efficient alternatives to hybrid cars will likely
become available in the future.

(E)

The future cost of gasoline and other fuel
cannot be predicted with absolute precision or
certainty.

601. Which of the following most logically completes the
passage?
A recent government study links the high rates
of respiratory ailments in Groverston to airborne
pollutants released by the Woodco plywood
manufacturing plant there. To address the problem, the
government imposed strict regulations on emissions
which will go into effect in four years.
Although Woodco plans to cut its emissions in half two
years ahead of schedule, it is unlikely that the rate of
respiratory ailments will decline before the regulations
go into effect, since ________.
(A)

the number of facilities capable of treating
respiratory ailments is not likely to increase

(B)

reducing emissions even further than planned
would necessitate decreasing production at
Woodco

(C)

it is difficult to make accurate, long‐term
predictions about emissions

(D)

not all respiratory ailments are caused by
airborne pollutants

(E)

three new plywood manufacturing plants are
about to go into production in Groverston

602. One summer, floods covered low‐lying garlic fields
situated in a region with a large mosquito population.
Since mosquitoes lay their eggs in standing water,
flooded fields would normally attract mosquitoes, yet
no mosquitoes were found in the fields. Diallyl sulfide,
a major component of garlic, is known to repel several
species of insects, including mosquitoes, so it is
likely that diallyl sulfide from the garlic repelled the
mosquitoes.

Which of the following, if true, most strengthens the
argument?
(A)

Diallyl sulfide is also found in onions but at
concentrations lower than in garlic.

(B)

The mosquito population of the region as a
whole was significantly smaller during the year in
which the flooding took place than it had been in
previous years.

(C)

By the end of the summer, most of the garlic
plants in the flooded fields had been killed by
waterborne fungi.

(D)

Many insect species not repelled by diallyl sulfide
were found in the flooded garlic fields throughout
the summer.

(E)

Mosquitoes are known to be susceptible to
toxins in plants other than garlic, such as
marigolds.

603. Which of the following most logically completes the
passage?
Pecan growers get a high price for their crop when
pecans are comparatively scarce, but the price drops
sharply when pecans are abundant. Thus, in high‐yield
years, growers often hold back part of their crop in
refrigerated warehouses for one or two years, hoping
for higher prices in the future. This year’s pecan crop
was the smallest in five years. It is nonetheless quite
possible that a portion of this year’s crop will be held
back, since ________.
(A)

each of the last two years produced
recordbreaking pecan yields

(B)

the quality of this year’s pecan crop is no worse
than the quality of the pecan crops of the
previous five years

(C)

pecan prices have not been subject to sharp
fluctuations in recent years

(D)

for some pecan growers, this year’s crop was no
smaller than last year’s

(E)

the practice of holding back part of one year’s
crop had not yet become widespread the last time
the pecan crop was as small as it was this year

525

GMAT® Official Guide 2018

604. Coffee shop owner: A large number of customers will
pay at least the fair market value for a cup of coffee,
even if there is no formal charge. Some will pay more
than this out of appreciation of the trust that is placed
in them. And our total number of customers is likely to
increase. We could therefore improve our net cash flow
by implementing an honor system in which customers pay
what they wish for coffee by depositing money in a can.
Manager: We’re likely to lose money on this plan. Many
customers would cheat the system, paying a very
small sum or nothing at all.

606. Journalist: In physics journals, the number of articles
reporting the results of experiments involving particle
accelerators was lower last year than it had been in
previous years. Several of the particle accelerators
at major research institutions were out of service the
year before last for repairs, so it is likely that the low
number of articles was due to the decline in availability
of particle accelerators.
Which of the following, if true, most seriously
undermines the journalist’s argument?
(A)

Every article based on experiments with particle
accelerators that was submitted for publication
last year actually was published.

(B)

The average time scientists must wait for access
to a particle accelerator has declined over the
last several years.

(C)

The number of physics journals was the same
last year as in previous years.

(D)

Particle accelerators can be used for more than
one group of experiments in any given year.

(E)

Recent changes in the editorial policies of several
physics journals have decreased the likelihood
that articles concerning particle‐accelerator
research will be accepted for publication.

Which of the following, if true, would best support the
owner’s plan, in light of the manager’s concern?
(A)

The new system, if implemented, would increase
the number of customers.

(B)

By roasting its own coffee, the shop has
managed to reduce the difficulties (and cost)
of maintaining an inventory of freshly roasted
coffee.

(C)

Many customers stay in the cafe for long
stretches of time.

(D)

The shop makes a substantial profit from pastries
and other food bought by the coffee drinkers.

(E)

No other coffee shop in the area has such a system.

605. Which of the following most logically completes the
argument?
By competing with rodents for seeds, black ants help
control rodent populations that pose a public health
risk. However, a very aggressive species of black ant,
the Loma ant, which has recently invaded a certain
region, has a venomous sting that is often fatal to
humans. Therefore, the planned introduction into that
region of ant flies, which prey on Loma ants, would
benefit public health, since ________.

526

(A)

ant flies do not attack black ants other than
Loma ants

(B)

Loma ants are less effective than many bird
species in competing with rodents for seeds

(C)

certain other species of black ants are more
effective than Loma ants in competing with
rodents for seeds

(D)

the sting of Loma ants can also be fatal to rodents

(E)

some pesticides that could be used to control
Loma ants are harmful to the environment

607. Birds have been said to be descended from certain
birdlike dinosaur species with which they share
distinctive structural features. The fossil record,
however, shows that this cannot be so, since there
are bird fossils that are much older than the earliest
birdlike dinosaur fossils that have been found.
Which of the following is an assumption on which the
argument relies?
(A)

The birdlike dinosaurs have no living descendants.

(B)

There are no flightless dinosaur species that
have the distinctive structural features shared by
birds and birdlike dinosaurs.

(C)

There are no birdlike dinosaur fossils that are
older than the bird fossils but have not yet been
unearthed.

(D)

It could not have been the case that some
birds were descended from one of the birdlike
dinosaur species and other birds from another.

(E)

Birds cannot have been descended from
dinosaur species with which the birds do not
share the distinctive structural features.

8.4 Critical Reasoning Practice Questions

608. City council member: Demand for electricity has been
increasing by 1.5 percent a year, and there simply
is no more space to build additional power plants to
meet future demand increases. We must therefore
begin to curtail usage, which is why I propose passing
ordinances requiring energy‐conservation measures in
all city departments.
The city council member’s proposal assumes which of
the following?

(D)

are typically grown in large tracts devoted to a
single crop

(E)

are cultivated specifically for the seed they
produce rather than for their leaves or roots

610. Previously, Autoco designed all of its cars itself and
then contracted with specialized parts suppliers to
build parts according to its specifications. Now it plans
to include its suppliers in designing the parts they
are to build. Since many parts suppliers have more
designers with specialized experience than Autoco
has, Autoco expects this shift to reduce the overall
time and cost of the design of its next new car.

(A)

Existing power plants do not have the capacity
to handle all of the projected increase in demand
for electricity.

(B)

No city departments have implemented energy‐
conservation measures voluntarily.

(C)

Passing ordinances designed to curtail
electricity usage will not have negative economic
consequences for the city.

(A)

When suppliers provide their own designs,
Autoco often needs to modify its overall design.

(D)

Residential consumers are not responsible for
the recent increases in demand for electricity.

(B)

(E)

City departments that successfully conserve
energy will set a good example for residential
and industrial consumers of electricity.

In order to provide designs for Autoco, several
of the parts suppliers will have to add to their
existing staffs of designers.

(C)

Parts and services provided by outside suppliers
account for more than 50 percent of Autoco’s
total costs.

(D)

When suppliers built parts according to
specifications provided by Autoco, the suppliers
competed to win contracts.

(E)

Most of Autoco’s suppliers have on hand a wide
range of previously prepared parts designs that
can readily be modified for a new car.

609. Which of the following most logically completes the
argument below?
Using broad‐spectrum weed killers on weeds that are
competing with crops for sunlight, water, and nutrients
presents a difficulty: how to keep the crop from being
killed along with the weeds. For at least some food
crops, specially treated seed that produces plants
resistant to weed killers is under development. This
resistance wears off as the plants mature. Therefore,
the special seed treatment will be especially useful for
plants that ________.
(A)

produce their crop over an extended period of
time, as summer squash does

(B)

produce large seeds that are easy to treat
individually, as corn and beans do

(C)

provide, as they approach maturity, shade dense
enough to keep weeds from growing

Which of the following, if true, most strongly supports
Autoco’s expectation?

527

GMAT® Official Guide 2018

611. In Stenland, many workers have been complaining
that they cannot survive on minimum wage, the
lowest wage an employer is permitted to pay. The
government is proposing to raise the minimum wage.
Many employers who pay their workers the current
minimum wage argue that if it is raised, unemployment
will increase because they will no longer be able to
afford to employ as many workers.
Which of the following, if true in Stenland, most strongly
supports the claim that raising the minimum wage there
will not have the effects that the employers predict?

(C)

there are works that can conclusively be
attributed to Pescard that are not even
mentioned in the treatise from the early 1500s

(D)

the later treatises probably had no source for
their attribution other than the earlier treatise

(E)

no known treatises from the 1600s identify
Pescard as the composer of Lacrimae

613. When trying to identify new technologies that promise to
transform the marketplace, market researchers survey
the managers of those companies that are developing
new technologies. Such managers have an enormous
stake in succeeding, so they invariably overstate
the potential of their new technologies. Surprisingly,
however, market researchers typically do not survey
a new technology’s potential buyers, even though it
is the buyers—not the producers—who will ultimately
determine a technology’s commercial success.

(A)

For any position with wages below a living wage,
the difficulty of finding and retaining employees
adds as much to employment costs as would
raising wages.

(B)

Raising the minimum wage does not also
increase the amount employers have to
contribute in employee benefits.

(C)

When inflation is taken into account, the
proposed new minimum wage is not as high as
the current one was when it was introduced.

(A)

(D)

Many employees currently being paid wages at
the level of the proposed new minimum wage will
demand significant wage increases.

If a new technology succeeds, the commercial
benefits accrue largely to the producers, not to
the buyers, of that technology.

(B)

(E)

Many employers who pay some workers only the
minimum wage also pay other workers wages
that are much higher than the minimum.

People who promote the virtues of a new
technology typically fail to consider that the old
technology that is currently in use continues to
be improved, often substantially.

(C)

Investors are unlikely to invest substantial amounts
of capital in a company whose own managers are
skeptical about the commercial prospects of a
new technology they are developing.

(D)

The potential buyers for not‐yet‐available
technologies can seldom be reliably identified.

(E)

The developers of a new technology are
generally no better positioned than its potential
buyers to gauge how rapidly the new technology
can be efficiently mass‐produced.

Which of the following, if true, best accounts for the
typical survey practices among market researchers?

612. Which of the following most logically completes the
argument?
The attribution of the choral work Lacrimae to the
composer Pescard (1400–1474) has been regarded
as tentative, since it was based on a single treatise
from the early 1500s that named Pescard as the
composer. Recently, several musical treatises from
the late 1500s have come to light, all of which
name Pescard as the composer of Lacrimae.
Unfortunately, these newly discovered treatises lend
no support to the attribution of Lacrimae to Pescard,
since ________.

528

(A)

the treatise from the early 1500s misidentifies
the composers of some of the musical works it
considers

(B)

the author of the treatise from the early 1500s
had no very strong evidence on which to base
the identification of Pescard as the composer of
Lacrimae

614. Infotek, a computer manufacturer in Katrovia, has just
introduced a new personal computer model that sells
for significantly less than any other model. Market
research shows, however, that very few Katrovian
households without personal computers would buy
a computer, regardless of its price. Therefore,
introducing the new model is unlikely to increase the
number of computers in Katrovian homes.
Which of the following is an assumption on which the
argument depends?

8.4 Critical Reasoning Practice Questions

(A)

Infotek achieved the lower price of the new
model by using components of lower quality than
those used by other manufacturers.

(B)

The main reason cited by consumers in Katrovia
for replacing a personal computer is the desire
to have an improved model.

(C)

(D)

(E)

Katrovians in households that already have
computers are unlikely to purchase the new
Infotek model as an additional computer for
home use.
The price of other personal computers in
Katrovia is unlikely to drop below the price of
Infotek’s new model in the near future.

Which of the following, if true, most helps to explain
the surprising finding?
(A)

Tidal currents do not themselves dislodge
barnacles from the shells of horseshoe crabs.

(B)

Barnacles most readily attach themselves to
horseshoe crabs in areas where tidal currents
are weakest.

(C)

The strength of the tidal currents in a given
location varies widely over the course of a day.

(D)

A very large barnacle population can significantly
decrease the ability of a horseshoe crab to find food.

(E)

Until they are fully grown, horseshoe crabs shed
their shells and grow new ones several times a
year.

Most personal computers purchased in Katrovia
are intended for home use.

615. Fast‐food restaurants make up 45 percent of all
restaurants in Canatria. Customers at these restaurants
tend to be young; in fact, studies have shown that
the older people get, the less likely they are to eat in
fast‐food restaurants. Since the average age of the
Canatrian population is gradually rising and will continue
to do so, the number of fast‐food restaurants is likely to
decrease.
Which of the following, if true, most seriously weakens
the argument?
(A)

Fast‐food restaurants in Canatria are getting
bigger, so each one can serve more customers.

(B)

Some older people eat at fast‐food restaurants
more frequently than the average young person.

(C)

Many people who rarely eat in fast‐food restaurants
nevertheless eat regularly in restaurants.

(D)

The overall population of Canatria is growing
steadily.

(E)

As the population of Canatria gets older, more
people are eating at home.

616. In order to withstand tidal currents, juvenile horseshoe
crabs frequently burrow in the sand. Such burrowing
discourages barnacles from clinging to their shells.
When fully grown, however, the crabs can readily
withstand tidal currents without burrowing, and thus
they acquire substantial populations of barnacles.
Surprisingly, in areas where tidal currents are very
weak, juvenile horseshoe crabs are found not to have
significant barnacle populations, even though they
seldom burrow.

617. Last year a chain of fast‐food restaurants, whose
menu had always centered on hamburgers, added its
first vegetarian sandwich, much lower in fat than the
chain’s other offerings. Despite heavy marketing, the
new sandwich accounts for a very small proportion of
the chain’s sales. The sandwich’s sales would have to
quadruple to cover the costs associated with including it
on the menu. Since such an increase is unlikely, the chain
would be more profitable if it dropped the sandwich.
Which of the following, if true, most seriously weakens
the argument?
(A)

Although many of the chain’s customers have
never tried the vegetarian sandwich, in a market
research survey most of those who had tried it
reported that they were very satisfied with it.

(B)

Many of the people who eat at the chain’s
restaurants also eat at the restaurants of
competing chains and report no strong
preference among the competitors.

(C)

Among fast‐food chains in general, there has
been little or no growth in hamburger sales over
the past several years as the range of competing
offerings at other restaurants has grown.

(D)

When even one member of a group of diners is a
vegetarian or has a preference for low‐fat food,
the group tends to avoid restaurants that lack
vegetarian or low‐fat menu options.

(E)

An attempt by the chain to introduce a lower‐fat
hamburger failed several years ago, since it
attracted few new customers and most of the
chain’s regular customers greatly preferred the
taste of the regular hamburgers.
529

GMAT® Official Guide 2018

618. Transportation expenses accounted for a large portion
of the total dollar amount spent on trips for pleasure by
residents of the United States in 1997, and about half
of the total dollar amount spent on transportation was
for airfare. However, the large majority of United States
residents who took trips for pleasure in 1997 did not
travel by airplane but used other means of transportation.
If the statements above are true, which of the following
must also be true about United States residents who
took trips for pleasure in 1997?
(A)

(B)

(C)

Most of those who traveled by airplane did so
because the airfare to their destination was
lower than the cost of other available means of
transportation.
Most of those who traveled by airplane did so
because other means of transportation to their
destination were unavailable.
Per mile traveled, those who traveled by airplane
tended to spend more on transportation to their
destination than did those who used other means
of transportation.

(D)

Overall, people who did not travel by airplane
had lower average transportation expenses than
people who did.

(E)

Those who traveled by airplane spent about
as much, on average, on other means of
transportation as they did on airfare.

619. Voters commonly condemn politicians for being
insincere, but politicians often must disguise their
true feelings when they make public statements. If
they expressed their honest views—about, say, their
party’s policies—then achieving politically necessary
compromises would be much more difficult. Clearly,
the very insincerity that people decry shows that our
government is functioning well.
Which of the following, if true, most seriously
undermines this reasoning?
(A)

Some of the public statements made by
politicians about their party’s policies could in
fact be sincere.

620. To reduce waste of raw materials, the government of
Sperland is considering requiring household appliances
to be broken down for salvage when discarded. To
cover the cost of salvage, the government is planning
to charge a fee, which would be imposed when the
appliance is first sold. Imposing the fee at the time of
salvage would reduce waste more effectively, however,
because consumers tend to keep old appliances
longer if they are faced with a fee for discarding them.
Which of the following, if true, most seriously weakens
the argument?
(A)

Increasing the cost of disposing of an appliance
properly increases the incentive to dispose of it
improperly.

(B)

The fee provides manufacturers with no incentive
to produce appliances that are more durable.

(C)

For people who have bought new appliances
recently, the salvage fee would not need to be
paid for a number of years.

(D)

People who sell their used, working appliances
to others would not need to pay the salvage fee.

(E)

Many nonfunctioning appliances that are
currently discarded could be repaired at
relatively little expense.

621. When there is less rainfall than normal, the water
level of Australian rivers falls and the rivers flow more
slowly. Because algae whose habitat is river water
grow best in slow‐moving water, the amount of algae
per unit of water generally increases when there has
been little rain. By contrast, however, following a
period of extreme drought, algae levels are low even in
very slow‐moving river water.
Which of the following, if true, does most to explain the
contrast described above?
(A)

During periods of extreme drought, the
populations of some of the species that feed on
algae tend to fall.

(B)

Some political compromises are not in the best
long‐term interest of the government.

(B)

(C)

Voters often judge politicians by criteria other than
the sincerity with which they express their views.

The more slowly water moves, the more conducive
its temperature is to the growth of algae.

(C)

When algae populations reach very high levels,
conditions within the river can become toxic for
some of the other species that normally live there.

(D)
530

Achieving political compromises is not all that
is necessary for the proper functioning of a
government.

(E)

A political party’s policies could turn out to be
detrimental to the functioning of a government.

8.4 Critical Reasoning Practice Questions

(D)

Australian rivers dry up completely for short
intervals in periods of extreme drought.

(E)

Except during periods of extreme drought, algae
levels tend to be higher in rivers in which the flow
has been controlled by damming than in rivers
that flow freely.

622. Increased use of incineration is sometimes advocated
as a safe way to dispose of chemical waste. But
opponents of incineration point to the 40 incidents
involving unexpected releases of dangerous chemical
agents that were reported just last year at two existing
incinerators commissioned to destroy a quantity of
chemical waste material. Since designs for proposed
new incinerators include no additional means of
preventing such releases, leaks will only become more
prevalent if use of incineration increases.
Which of the following, if true, most seriously weakens
the argument?
(A)

At the two incinerators at which leaks were
reported, staff had had only cursory training on the
proper procedures for incinerating chemical waste.

(B)

Other means of disposing of chemical waste,
such as chemical neutralization processes, have
not been proven safer than incineration.

(C)

(D)

(E)

The capacity of existing incinerators is sufficient
to allow for increased incineration of chemical
waste without any need for new incinerators.
The frequency of reports of unexpected releases
of chemical agents at newly built incinerators
is about the same as the frequency at older
incinerators.
In only three of the reported incidents of
unexpected chemical leaks did the releases
extend outside the property on which the
incinerators were located.

623. Public health expert: Increasing the urgency of a
public health message may be counterproductive.
In addition to irritating the majority who already behave
responsibly, it may undermine all government
pronouncements on health by convincing people
that such messages are overly cautious. And there
is no reason to believe that those who ignore measured
voices will listen to shouting.

The two sections in boldface play which of the
following roles in the public health expert’s argument?
(A)

The first is a conclusion for which support
is provided, but is not the argument’s main
conclusion; the second is an unsupported premise
supporting the argument’s main conclusion.

(B)

The first is a premise supporting the only explicit
conclusion; so is the second.

(C)

The first is the argument’s main conclusion; the
second supports that conclusion and is itself a
conclusion for which support is provided.

(D)

The first is a premise supporting the argument’s
only conclusion; the second is that conclusion.

(E)

The first is the argument’s only explicit conclusion;
the second is a premise supporting that conclusion.

624. Several industries have recently switched at least
partly from older technologies powered by fossil fuels
to new technologies powered by electricity. It is thus
evident that less fossil fuel is being used as a result
of the operations of these industries than would have
been used if these industries had retained their older
technologies.
Which of the following, if true, most strengthens the
argument above?
(A)

Many of the industries that have switched
at least partly to the new technologies have
increased their output.

(B)

Less fossil fuel was used to manufacture the
machinery employed in the new technologies
than was originally used to manufacture the
machinery employed in the older technologies.

(C)

More electricity is used by those industries that have
switched at least partly to the new technologies than
by those industries that have not switched.

(D)

Some of the industries that have switched at least
partly to the new technologies still use primarily
technologies that are powered by fossil fuels.

(E)

The amount of fossil fuel used to generate the
electricity needed to power the new technologies
is less than the amount that would have been
used to power the older technologies.

531

GMAT® Official Guide 2018

625. The difference in average annual income in favor of
employees who have college degrees, compared with
those who do not have such degrees, doubled between
1980 and 1990. Some analysts have hypothesized
that increased competition between employers for
employees with college degrees drove up income for
such employees.
Which of the following, if true, most seriously
undermines the explanation described above?
(A)

During the 1980s a growing percentage of
college graduates, unable to find jobs requiring a
college degree, took unskilled jobs.

(B)

The average age of all employees increased
slightly during the 1980s.

(C)

The unemployment rate changed very little
throughout most of the 1980s.

(D)

From 1980 to 1990 the difference in average
income between employees with advanced
degrees and those with bachelor’s degrees also
increased.

(E)

During the 1980s there were some employees
with no college degree who earned incomes
comparable to the top incomes earned by
employees with a college degree.

626. Which of the following most logically completes the
passage?
According to the last pre-election poll in
Whippleton, most voters believe that the three
problems government needs to address, in
order of importance, are pollution, crime, and
unemployment. Yet in the election, candidates from
parties perceived as strongly against pollution were
defeated, while those elected were all from parties
with a history of opposing legislation designed to
reduce pollution. These results should not be taken
to indicate that the poll was inaccurate, however,
since __________.

532

(A)

some voters in Whippleton do not believe that
pollution needs to be reduced

(B)

every candidate who was defeated had a strong
antipollution record

(C)

there were no issues other than crime,
unemployment, and pollution on which the
candidates had significant differences of opinion

(D)

all the candidates who were elected were
perceived as being stronger against both crime
and unemployment than the candidates who
were defeated

(E)

many of the people who voted in the election
refused to participate in the poll

627. Manufacturing plants in Arundia have recently been
acquired in substantial numbers by investors from
abroad. Arundian politicians are proposing legislative
action to stop such investment, justifying the proposal
by arguing that foreign investors, opportunistically
exploiting a recent fall in the value of the Arundian
currency, were able to buy Arundian assets at less than
their true value.
Which of the following, if true, casts the most serious
doubt on the adequacy of the Arundian politicians’
justification for the proposed legislation?
(A)

The Arundian government originally welcomed
the fall in the value of the Arundian currency
because the fall made Arundian exports more
competitive on international markets.

(B)

Foreign investors who acquired Arundian
manufacturing plants generally did so with no
intention of keeping and running those plants
over the long term.

(C)

Without the recent fall in the value of the
Arundian currency, many of the Arundian assets
bought by foreign investors would have been
beyond the financial reach of those investors.

(D)

In Concordia, a country broadly similar to
Arundia, the share of manufacturing assets that
is foreign-controlled is 60 percent higher than it
is in Arundia.

(E)

The true value of an investment is determined by
the value of the profits from it, and the low value
of the Arundian currency has depressed the
value of any profits earned by foreign investors
from Arundian assets.

628. Proposed new safety rules for Beach City airport would
lengthen considerably the minimum time between
takeoffs from the airport. In consequence, the airport
would be able to accommodate 10 percent fewer
flights than currently use the airport daily. The city’s
operating budget depends heavily on taxes generated

8.4 Critical Reasoning Practice Questions

by tourist spending, and most of the tourists come
by plane. Therefore, the proposed new safety rules,
if adopted, will reduce the revenue available for the
operating budget.
The argument depends on assuming which of the
following?
(A)

There are no periods of the day during which
the interval between flights taking off from the
airport is significantly greater than the currently
allowed minimum.

(B)

Few, if any, of the tourists who use Beach City
airport do so when their main destination is a
neighboring community and not Beach City itself.

(C)

If the proposed safety rules are adopted, the
reduction in tourist numbers will not result mainly
from a reduction in the number of tourists who
spend relatively little in Beach City.

(D)

Increasing the minimum time between takeoffs
is the only way to achieve necessary safety
improvements without a large expenditure by the
city government on airport enhancements.

(E)

The response to the adoption of the new safety
rules would not include a large increase in the
number of passengers per flight.

629. The introduction of new drugs into the market is
frequently prevented by a shortage of human subjects
for the clinical trials needed to show that the drugs are
safe and effective. Since the lives and health of people
in future generations may depend on treatments that
are currently experimental, practicing physicians are
morally in the wrong when, in the absence of any
treatment proven to be effective, they fail to encourage
suitable patients to volunteer for clinical trials.
Which of the following, if true, casts most doubt on the
conclusion of the argument?
(A)

Many drugs undergoing clinical trials are
intended for the treatment of conditions for
which there is currently no effective treatment.

(B)

Patients do not share the physician’s professional
concern for public health, but everyone has a
moral obligation to alleviate suffering when able
to do so.

(C)

Usually, half the patients in a clinical trial serve
as a control group and receive a nonactive drug
in place of the drug being tested.

(D)

An experimental drug cannot legally be made
available to patients unless those patients are
subjects in clinical trials of the drug.

(E)

Physicians have an overriding moral and legal
duty to care for the health and safety of their
current patients.

630. As a construction material, bamboo is as strong as
steel and sturdier than concrete. Moreover, in tropical
areas bamboo is a much less expensive construction
material than either steel or concrete and is always
readily available. In tropical areas, therefore, building with
bamboo makes better economic sense than building with
steel or concrete, except where land values are high.
Which of the following, if true, most helps to explain
the exception noted above?
(A)

Buildings constructed of bamboo are less likely
to suffer earthquake damage than are steel and
concrete buildings.

(B)

Bamboo is unsuitable as a building material for
multistory buildings.

(C)

In order to protect it from being damaged by
termites and beetles, bamboo must be soaked,
at some expense, in a preservative.

(D)

In some tropical areas, bamboo is used to
make the scaffolding that is used during large
construction projects.

(E)

Bamboo growing in an area where land values
are increasing is often cleared to make way for
construction.

533

GMAT® Official Guide 2018

631. Newspaper editors should not allow reporters to write
the headlines for their own stories. The reason for this
is that, while the headlines that reporters themselves
write are often clever, what typically makes them
clever is that they allude to little-known information
that is familiar to the reporter but that never appears
explicitly in the story itself.

(C)

Whether the insecticides typically used on feed
corn tend to be more expensive than insecticides
typically used on other crops

(D)

Whether most of the farmers who tried the
genetically modified corn last season applied
more insecticide than was actually necessary

(E)

Whether, for most farmers who plant feed corn,
it is their most profitable crop

Which of the following, if true, most strengthens the
argument?
(A)

The reporter who writes a story is usually better
placed than the reporter’s editor is to judge what
the story’s most newsworthy features are.

(B)

To write a headline that is clever, a person must
have sufficient understanding of the story that
the headline accompanies.

(C)

Most reporters rarely bother to find out how
other reporters have written stories and
headlines about the same events that they
themselves have covered.

(D)

For virtually any story that a reporter writes,
there are at least a few people who know more
about the story’s subject matter than does the
reporter.

(E)

The kind of headlines that newspaper editors want
are those that anyone who has read a reporter’s
story in its entirety will recognize as clever.

632. Scientists have modified feed corn genetically,
increasing its resistance to insect pests. Farmers
who tried out the genetically modified corn last
season applied less insecticide to their corn fields
and still got yields comparable to those they would
have gotten with ordinary corn. Ordinary corn seed,
however, costs less, and what these farmers saved
on insecticide rarely exceeded their extra costs
for seed. Therefore, for most feed‐corn farmers,
switching to genetically modified seed would be
unlikely to increase profits.

633. Debater: The average amount of overtime per month
worked by an employee in the manufacturing division
of the Haglut Corporation is 14 hours. Most employees
of the Haglut Corporation work in the manufacturing
division. Furthermore, the average amount of overtime
per month worked by any employee in the company
generally does not fluctuate much from month to
month. Therefore, each month, most employees of the
Haglut Corporation almost certainly work at least some
overtime.
The debater’s argument is most vulnerable to criticism
on which of these grounds?
(A)

It takes for granted that the manufacturing
division is a typical division of the corporation
with regard to the average amount of overtime
its employees work each month.

(B)

It takes for granted that if a certain average
amount of overtime is worked each month by
each employee of the Haglut Corporation, then
approximately the same amount of overtime
must be worked each month by each employee
of the manufacturing division.

(C)

It confuses a claim from which the argument’s
conclusion about the Haglut Corporation would
necessarily follow with a claim that would follow
from the argument’s conclusion only with a high
degree of probability.

(D)

It overlooks the possibility that even if, on
average, a certain amount of overtime is worked
by the members of some group, many members
of that group may work no overtime at all.

(E)

It overlooks the possibility that even if most
employees of the corporation work some
overtime each month, any one corporate
employee may, in some months, work no
overtime.

Which of the following would it be most useful to know
in order to evaluate the argument?

534

(A)

Whether there are insect pests that sometimes
reduce feed‐corn yields, but against which
commonly used insecticides and the genetic
modification are equally ineffective

(B)

Whether the price that farmers receive for feed
corn has remained steady over the past few
years

8.4 Critical Reasoning Practice Questions

634. Proponents of the recently introduced tax on sales
of new luxury boats had argued that a tax of this sort
would be an equitable way to increase government
revenue because the admittedly heavy tax burden
would fall only on wealthy people and neither they nor
anyone else would suffer any economic hardship. In
fact, however, 20 percent of the workers employed by
manufacturers of luxury boats have lost their jobs as a
direct result of this tax.

(B)

Drivers 65 and older do not constitute a
significantly larger percentage of licensed drivers
in Wareland than drivers ages 18–24 do.

(C)

Drivers 65 and older are less likely than are
drivers 24 and younger to drive during weather
conditions that greatly increase the risk of
accidents.

(D)

The difference between the accident rate of
drivers under 21 and of those ages 21–24 is
attributable to the greater driving experience of
those in the older group.

(E)

There is no age bracket for which the accident
rate is lower than it is for licensed drivers 65 and
older.

The information given, if true, most strongly supports
which of the following?
(A)

The market for luxury boats would have
collapsed even if the new tax on luxury boats had
been lower.

(B)

The new tax would produce a net gain in tax
revenue for the government only if the yearly
total revenue that it generates exceeds the total
of any yearly tax-revenue decrease resulting from
the workers’ loss of jobs.

(C)

Because many people never buy luxury items,
imposing a sales tax on luxury items is the kind
of legislative action that does not cost incumbent
legislators much popular support.

(D)

Before the tax was instituted, luxury boats were
largely bought by people who were not wealthy.

(E)

Taxes can be equitable only if their burden is
evenly distributed over the entire population.

635. In Wareland last year, 16 percent of licensed drivers
under 21 and 11 percent of drivers ages 21–24 were
in serious accidents. By contrast, only 3 percent
of licensed drivers 65 and older were involved in
serious accidents. These figures clearly show that the
greater experience and developed habits of caution
possessed by drivers in the 65-and-older group make
them far safer behind the wheel than the younger
drivers are.
Which of the following is an assumption on which the
argument depends?
(A)

636. In the past the country of Malvernia has relied heavily
on imported oil. Malvernia recently implemented a
program to convert heating systems from oil to natural
gas. Malvernia currently produces more natural gas
each year than it uses, and oil production in Malvernian
oil fields is increasing at a steady pace. If these trends
in fuel production and usage continue, therefore,
Malvernian reliance on foreign sources for fuel is likely
to decline soon.
Which of the following would it be most useful to
establish in evaluating the argument?
(A)

When, if ever, will production of oil in Malvernia
outstrip production of natural gas?

(B)

Is Malvernia among the countries that rely most
on imported oil?

(C)

What proportion of Malvernia’s total energy
needs is met by hydroelectric, solar, and nuclear
power?

(D)

Is the amount of oil used each year in
Malvernia for generating electricity and fuel for
transportation increasing?

(E)

Have any existing oil‐burning heating systems
in Malvernia already been converted to natural‐
gas‐burning heating systems?

Drivers 65 and older do not, on average, drive
very many fewer miles per year than drivers 24
and younger.

535

GMAT® Official Guide 2018

637. Exposure to certain chemicals commonly used in
elementary schools as cleaners or pesticides causes
allergic reactions in some children. Elementary
school nurses in Renston report that the proportion
of schoolchildren sent to them for treatment of
allergic reactions to those chemicals has increased
significantly over the past ten years. Therefore, either
Renston’s schoolchildren have been exposed to greater
quantities of the chemicals, or they are more sensitive
to them than schoolchildren were ten years ago.
Which of the following is an assumption on which the
argument depends?
(A)

The number of school nurses employed by
Renston’s elementary schools has not decreased
over the past ten years.

(B)

Children who are allergic to the chemicals are no
more likely than other children to have allergies
to other substances.

(C)

Children who have allergic reactions to the
chemicals are not more likely to be sent to a
school nurse now than they were ten years ago.

(D)

The chemicals are not commonly used as
cleaners or pesticides in houses and apartment
buildings in Renston.

(E)

Children attending elementary school do not
make up a larger proportion of Renston’s
population now than they did ten years ago.

638. Normally, the pineal gland governs a person’s
sleep-wake cycle by secreting melatonin in response to
the daily cycle of light and darkness as detected by the
eye. Nonetheless, many people who are totally blind due
to lesions in the visual cortex of the brain easily maintain
a 24‐hour sleep‐wake cycle. So the neural pathway by
which the pineal gland receives information from the eye
probably does not pass through the visual cortex.
For purposes of evaluating the argument it would be
most useful to establish which of the following?
(A)

(B)

536

Whether melatonin supplements help people who
have difficulty maintaining a 24‐hour sleep cycle
to establish such a pattern
Whether the melatonin levels of most totally
blind people who successfully maintain a 24‐hour
sleep‐wake cycle change in response to changes
in exposure to light and darkness

(C)

Whether melatonin is the only substance
secreted by the pineal gland

(D)

Whether most people who do not have a 24‐hour
sleep‐wake cycle nevertheless have a cycle of
consistent duration

(E)

Whether there are any people with normal vision
whose melatonin levels respond abnormally to
periods of light and darkness

639. In countries where automobile insurance includes
compensation for whiplash injuries sustained in
automobile accidents, reports of having suffered
such injuries are twice as frequent as they are
in countries where whiplash is not covered.
Presently, no objective test for whiplash exists, so it is
true that spurious reports of whiplash injuries cannot
be readily identified. Nevertheless, these facts do not
warrant the conclusion drawn by some commentators
that in the countries with the higher rates of reported
whiplash injuries, half of the reported cases are
spurious. Clearly, in countries where automobile
insurance does not include compensation for
whiplash, people often have little incentive to
report whiplash injuries that they actually have
suffered.
In the argument given, the two boldfaced portions play
which of the following roles?
(A)

The first is a claim that the argument disputes;
the second is a conclusion that has been based
on that claim.

(B)

The first is a claim that has been used to support
a conclusion that the argument accepts; the
second is that conclusion.

(C)

The first is evidence that has been used to
support a conclusion for which the argument
provides further evidence; the second is the
main conclusion of the argument.

(D)

The first is a finding whose implications are at
issue in the argument; the second is a claim
presented in order to argue against deriving
certain implications from that finding.

(E)

The first is a finding whose accuracy is evaluated
in the argument; the second is evidence
presented to establish that the finding is accurate.

8.4 Critical Reasoning Practice Questions

640. Last year Comfort Airlines had twice as many
delayed flights as the year before, but the number of
complaints from passengers about delayed flights went
up three times. It is unlikely that this disproportionate
increase in complaints was rooted in an increase in
overall dissatisfaction with the service Comfort Airlines
provides, since the airline made a special effort to
improve other aspects of its service last year.
Which of the following, if true, most helps to explain
the disproportionate increase in customer complaints?
(A)

Comfort Airlines had more flights last year than
the year before.

(B)

Last year a single period of unusually bad
weather caused a large number of flights to be
delayed.

(C)

Some of the improvements that Comfort Airlines
made in its service were required by new
government regulations.

(D)

The average length of a flight delay was greater
last year than it was the year before.

(E)

The average number of passengers per flight
was no higher last year than the year before.

641. Last year a global disturbance of weather patterns
disrupted harvests in many of the world’s important
agricultural areas. Worldwide production of soybeans,
an important source of protein for people and livestock
alike, was not adversely affected, however. Indeed, last
year’s soybean crop was actually slightly larger than
average. Nevertheless, the weather phenomenon is
probably responsible for a recent increase in the world
price of soybeans.

(C)

The world price of soybeans also rose
several years ago, immediately after an
earlier occurrence of a similar global weather
disturbance.

(D)

Heavy rains attributable to the weather
phenomenon improved grazing pastures last
year, allowing farmers in many parts of the world
to reduce their dependence on supplemental
feed.

(E)

Prior to last year, soybean prices had been
falling for several years.

642. Most of the year, the hermit thrush, a North American
songbird, eats a diet consisting mainly of insects, but
in autumn, as the thrushes migrate to their Central and
South American wintering grounds, they feed almost
exclusively on wild berries. Wild berries, however, are
not as rich in calories as insects, yet thrushes need to
consume plenty of calories in order to complete their
migration. One possible explanation is that berries
contain other nutrients that thrushes need for migration
and that insects lack.
Which of the following, if true, most seriously calls into
question the explanation given for the thrush’s diet
during migration?
(A)

Hermit thrushes, if undernourished, are unable
to complete their autumn migration before the
onset of winter.

(B)

Insect species contain certain nutrients that are
not found in wild berries.

(C)

For songbirds, catching insects requires the
expenditure of significantly more calories than
eating wild berries does.

Which of the following, if true, provides the strongest
justification for the attribution of the increase in
soybean prices to the weather phenomenon?

(D)

Along the hermit thrushes’ migration routes,
insects are abundant throughout the migration
season.

(A)

Last year’s harvest of anchovies, which provide
an important protein source for livestock,
was disrupted by the effects of the weather
phenomenon.

(E)

There are some species of wild berries that
hermit thrushes generally do not eat, even
though these berry species are exceptionally
rich in calories.

(B)

Most countries that produce soybeans for export
had above‐average harvests of a number of food
crops other than soybeans last year.

537

GMAT® Official Guide 2018

643. The kinds of hand and wrist injuries that result from
extended use of a computer while maintaining an
incorrect posture are common among schoolchildren
in Harnville. Computers are important to the school
curriculum there, so instead of reducing the amount
their students use computers, teachers plan to bring
about a sharp reduction in the number of these
injuries by carefully monitoring their
students’ posture when using computers in the
classroom.
Which of the following would it be most useful to know
in order to assess the likelihood that the teachers’ plan
will be successful?
(A)

Whether extended use of a computer while
maintaining incorrect posture can cause injuries
other than hand and wrist injuries

(B)

Whether hand and wrist injuries not caused
by computer use are common among
schoolchildren in Harnville

(C)

What proportion of schoolchildren in Harnville
with hand and wrist injuries use computers
extensively outside the classroom

(D)

Whether changes in the curriculum could reduce
the schools’ dependence on computers

(E)

What proportion of schoolchildren in Harnville
already use correct posture while using a
computer

644. A certain cultivated herb is one of a group of
closely related plants that thrive in soil with high
concentrations of metals that are toxic to most other
plants. Agronomists studying the growth of this herb
have discovered that it produces large amounts of
histidine, an amino acid that, in test‐tube solutions,
renders these metals chemically inert. Hence, the
herb’s high histidine production must be the key
feature that allows it to grow in metal‐rich soils.
In evaluating the argument, it would be most important
to determine which of the following?

538

(A)

Whether the herb can thrive in soil that does
not have high concentrations of the toxic
metals

(B)

Whether others of the closely related group
of plants also produce histidine in large
quantities

(C)

Whether the herb’s high level of histidine
production is associated with an unusually low
level of production of some other amino acid

(D)

Whether growing the herb in soil with high
concentrations of the metals will, over time,
reduce their concentrations in the soil

(E)

Whether the concentration of histidine in the
growing herb declines as the plant approaches
maturity

645. Many people suffer an allergic reaction to certain
sulfites, including those that are commonly added
to wine as preservatives. However, since there are
several winemakers who add sulfites to none of
the wines they produce, people who would like to
drink wine but are allergic to sulfites can drink wines
produced by these winemakers without risking an
allergic reaction to sulfites.
Which of the following is an assumption on which the
argument depends?
(A)

These winemakers have been able to duplicate
the preservative effect produced by adding
sulfites by means that do not involve adding any
potentially allergenic substances to their wine.

(B)

Not all forms of sulfite are equally likely to
produce the allergic reaction.

(C)

Wine is the only beverage to which sulfites are
commonly added.

(D)

Apart from sulfites, there are no substances
commonly present in wine that give rise to an
allergic reaction.

(E)

Sulfites are not naturally present in the wines
produced by these winemakers in amounts
large enough to produce an allergic reaction in
someone who drinks these wines.

646. A new law gives ownership of patents—documents
providing exclusive right to make and sell an
invention—to universities, not the government, when
those patents result from government‐sponsored
university research. Administrators at Logos University
plan to sell any patents they acquire to corporations
in order to fund programs to improve undergraduate
teaching.

8.4 Critical Reasoning Practice Questions

Which of the following, if true, would cast the most
doubt on the viability of the college administrators’
plan described above?
(A)

Profit‐making corporations interested in
developing products based on patents held by
universities are likely to try to serve as exclusive
sponsors of ongoing university research
projects.

(B)

Corporate sponsors of research in university
facilities are entitled to tax credits under new
federal tax‐code guidelines.

(C)

Research scientists at Logos University have few
or no teaching responsibilities and participate
little if at all in the undergraduate programs in
their field.

(D)

Government‐sponsored research conducted at
Logos University for the most part duplicates
research already completed by several
profitmaking corporations.

(E)

Logos University is unlikely to attract corporate
sponsorship of its scientific research.

647. Since it has become known that several of a bank’s
top executives have been buying shares in their
own bank, the bank’s depositors, who had been
worried by rumors that the bank faced impending
financial collapse, have been greatly relieved. They
reason that, since top executives evidently have faith
in the bank’s financial soundness, those worrisome
rumors must be false. Such reasoning might well
be overoptimistic, however, since corporate
executives have been known to buy shares in
their own company in a calculated attempt to
dispel negative rumors about the company’s
health.
In the argument given, the two boldfaced portions play
which of the following roles?
(A)

The first describes evidence that has been taken
as supporting a conclusion; the second gives a
reason for questioning that support.

(B)

The first describes evidence that has been taken
as supporting a conclusion; the second states a
contrary conclusion that is the main conclusion
of the argument.

(C)

The first provides evidence in support of the
main conclusion of the argument; the second
states that conclusion.

(D)

The first describes the circumstance that the
argument as a whole seeks to explain; the
second gives the explanation that the argument
seeks to establish.

(E)

The first describes the circumstance that
the argument as a whole seeks to explain;
the second provides evidence in support of
the explanation that the argument seeks to
establish.

648. Between 1980 and 2000 the sea otter population
of the Aleutian Islands declined precipitously. There
were no signs of disease or malnutrition, so there
was probably an increase in the number of otters
being eaten by predators. Orcas will eat otters when
seals, their normal prey, are unavailable, and the
Aleutian Islands seal population declined dramatically
in the 1980s. Therefore, orcas were most likely the
immediate cause of the otter population decline.
Which of the following, if true, most strengthens the
argument?
(A)

The population of sea urchins, the main food of
sea otters, has increased since the sea otter
population declined.

(B)

Seals do not eat sea otters, nor do they
compete with sea otters for food.

(C)

Most of the surviving sea otters live in a bay that
is inaccessible to orcas.

(D)

The population of orcas in the Aleutian Islands
has declined since the 1980s.

(E)

An increase in commercial fishing near the
Aleutian Islands in the 1980s caused a slight
decline in the population of the fish that seals
use for food.

539

GMAT® Official Guide 2018

649. Studies in restaurants show that the tips left by
customers who pay their bill in cash tend to be larger
when the bill is presented on a tray that bears a
credit-card logo. Consumer psychologists hypothesize
that simply seeing a credit‐card logo makes many
credit-card holders willing to spend more because it
reminds them that their spending power exceeds the
cash they have immediately available.

(C)

There were at least some volunteers who were
assigned to do the hard task and felt that the
assignment was unfair.

(D)

On average, the volunteers to whom the scenario
was described were more accurate in their moral
judgments than the other volunteers were.

(E)

At least some volunteers given the choice
between assigning the tasks themselves and
having the computer assign them felt that they
had made the only fair choice available to them.

Which of the following, if true, most strongly supports
the psychologists’ interpretation of the studies?
(A)

The effect noted in the studies is not limited to
patrons who have credit cards.

(B)

Patrons who are under financial pressure from
their credit‐card obligations tend to tip less when
presented with a restaurant bill on a tray with a
credit‐card logo than when the tray has no logo.

(C)

In virtually all of the cases in the studies, the
patrons who paid bills in cash did not possess
credit cards.

(D)

In general, restaurant patrons who pay their bills
in cash leave larger tips than do those who pay
by credit card.

(E)

The percentage of restaurant bills paid with a
given brand of credit card increases when that
credit card’s logo is displayed on the tray with
which the bill is presented.

650. In an experiment, each volunteer was allowed to
choose between an easy task and a hard task and was
told that another volunteer would do the other task.
Each volunteer could also choose to have a computer
assign the two tasks randomly. Most volunteers chose
the easy task for themselves and under questioning
later said they had acted fairly. But when the scenario
was described to another group of volunteers, almost
all said choosing the easy task would be unfair. This
shows that most people apply weaker moral standards
to themselves than to others.
Which of the following is an assumption required by
this argument?

540

(A)

At least some volunteers who said they had
acted fairly in choosing the easy task would have
said that it was unfair for someone else to do so.

(B)

The most moral choice for the volunteers would
have been to have the computer assign the two
tasks randomly.

651. Country X’s recent stock-trading scandal should not
diminish investors’ confidence in the country’s stock
market. For one thing, the discovery of the scandal
confirms that Country X has a strong regulatory
system, as the following considerations show. In any
stock market, some fraudulent activity is inevitable.
If a stock market is well regulated, any significant
stock-trading fraud in it will very likely be discovered.
This deters potential perpetrators and facilitates
improvement in regulatory processes.
In the argument, the portion in boldface plays which of
the following roles?
(A)

It is the argument’s only conclusion.

(B)

It is a conclusion for which the argument
provides support and which itself is used to
support the argument’s main conclusion.

(C)

It is the argument’s main conclusion and is
supported by another explicitly stated conclusion
for which further support is provided.

(D)

It is an assumption for which no explicit
support is provided and is used to support the
argument’s only conclusion.

(E)

It is a compound statement containing both the
argument’s main conclusion and an assumption
used to support that conclusion.

652. Delta Products Inc. has recently switched at least
partly from older technologies using fossil fuels
to new technologies powered by electricity. The
question has been raised whether it can be concluded
that for a given level of output Delta’s operation
now causes less fossil fuel to be consumed than
it did formerly. The answer, clearly, is yes, since the
amount of fossil fuel used to generate the electricity
needed to power the new technologies is less than
the amount needed to power the older technologies,
provided level of output is held constant.

8.4 Critical Reasoning Practice Questions

In the argument given, the two boldfaced portions play
which of the following roles?
(A)

The first identifies the content of the conclusion
of the argument; the second provides support
for that conclusion.

(B)

The first provides support for the conclusion of
the argument; the second identifies the content
of that conclusion.

(C)

The first states the conclusion of the argument;
the second calls that conclusion into question.

(D)

(E)

locations will not stay vacant for long. In the five
years since the opening of Colson’s, a nondiscount
department store, a new store has opened at the
location of every store in the shopping district that
closed because it could not compete with Colson’s.
Which of the following, if true, most seriously weakens
the argument?
(A)

Many customers of Colson’s are expected to
do less shopping there than they did before the
SpendLess store opened.

The first provides support for the conclusion of
the argument; the second calls that conclusion
into question.

(B)

Increasingly, the stores that have opened in the
central shopping district since Colson’s opened
have been discount stores.

Each provides support for the conclusion of the
argument.

(C)

At present, the central shopping district has as
many stores operating in it as it ever had.

(D)

Over the course of the next five years, it is
expected that Goreville’s population will grow at a
faster rate than it has for the past several decades.

(E)

Many stores in the central shopping district sell
types of merchandise that are not available at
either SpendLess or Colson’s.

653. Theater Critic: The play La Finestrina, now at Central
Theater, was written in Italy in the eighteenth century.
The director claims that this production is as similar
to the original production as is possible in a modern
theater. Although the actor who plays Harlequin the
clown gives a performance very reminiscent of the
twentieth‐century American comedian Groucho Marx,
Marx’s comic style was very much within the comic
acting tradition that had begun in sixteenth‐century Italy.
The considerations given best serve as part of an
argument that

655. Last year all refuse collected by Shelbyville city services
was incinerated. This incineration generated a large
quantity of residual ash. In order to reduce the amount of
residual ash Shelbyville generates this year to half of last
year’s total, the city has revamped its collection program.
This year city services will separate for recycling enough
refuse to reduce the number of truckloads of refuse to be
incinerated to half of last year’s number.

(A)

modern audiences would find it hard to tolerate
certain characteristics of a historically accurate
performance of an eighteenth‐century play

(B)

Groucho Marx once performed the part of the
character Harlequin in La Finestrina

Which of the following is required for the revamped
collection program to achieve its aim?

(C)

in the United States the training of actors in the
twentieth century is based on principles that do
not differ radically from those that underlay the
training of actors in eighteenth‐century Italy

(A)

This year, no materials that city services could
separate for recycling will be incinerated.

(B)

Separating recyclable materials from materials to
be incinerated will cost Shelbyville less than half
what it cost last year to dispose of the residual ash.

(C)

Refuse collected by city services will contain
a larger proportion of recyclable materials this
year than it did last year.

(D)

The refuse incinerated this year will generate no
more residual ash per truckload incinerated than
did the refuse incinerated last year.

(E)

The total quantity of refuse collected by
Shelbyville city services this year will be no
greater than that collected last year.

(D)

(E)

the performance of the actor who plays
Harlequin in La Finestrina does not serve as
evidence against the director’s claim
the director of La Finestrina must have advised the
actor who plays Harlequin to model his performance
on comic performances of Groucho Marx

654. Although the discount stores in Goreville’s central
shopping district are expected to close within five
years as a result of competition from a SpendLess
discount department store that just opened, those

541

GMAT® Official Guide 2018

656. Veterinarians generally derive some of their income
from selling several manufacturers’ lines of pet-care
products. Knowing that pet owners rarely throw
away mail from their pet’s veterinarian unread, one
manufacturer of pet-care products offered free
promotional materials on its products to veterinarians
for mailing to their clients. Very few veterinarians
accepted the offer, however, even though the
manufacturer’s products are of high quality.
Which of the following, if true, most helps to explain
the veterinarians’ reaction to the manufacturer’s
promotional scheme?
(A)

Most of the veterinarians to whom the free
promotional materials were offered were already
selling the manufacturer’s pet-care products to
their clients.

(B)

The special promotional materials were
intended as a supplement to the manufacturer’s
usual promotional activities rather than as a
replacement for them.

(C)

The manufacturer’s products, unlike most
equally good competing products sold by
veterinarians, are also available in pet stores and
in supermarkets.

(D)

(E)

Many pet owners have begun demanding quality
in products they buy for their pets that is as high
as that in products they buy for themselves.
Veterinarians sometimes recommend that pet
owners use products formulated for people when
no suitable product specially formulated for
animals is available.

657. The average hourly wage of television assemblers in
Vernland has long been significantly lower than that in
neighboring Borodia. Since Borodia dropped all tariffs
on Vernlandian televisions three years ago, the number
of televisions sold annually in Borodia has not changed.
However, recent statistics show a drop in the number
of television assemblers in Borodia. Therefore, updated
trade statistics will probably indicate that the number
of televisions Borodia imports annually from Vernland
has increased.
Which of the following is an assumption on which the
argument depends?
(A)

542

The number of television assemblers in Vernland
has increased by at least as much as the number
of television assemblers in Borodia has decreased.

(B)

Televisions assembled in Vernland have
features that televisions assembled in
Borodia do not have.

(C)

The average number of hours it takes a
Borodian television assembler to assemble a
television has not decreased significantly during
the past three years.

(D)

The number of televisions assembled annually in
Vernland has increased significantly during the
past three years.

(E)

The difference between the hourly wage of
television assemblers in Vernland and the hourly
wage of television assemblers in Borodia is likely
to decrease in the next few years.

658. Guidebook writer: I have visited hotels throughout the
country and have noticed that in those built before
1930 the quality of the original carpentry work is
generally superior to that in hotels built afterward.
Clearly carpenters working on hotels before 1930
typically worked with more skill, care, and effort
than carpenters who have worked on hotels built
subsequently.
Which of the following, if true, most seriously weakens
the guidebook writer’s argument?
(A)

The quality of original carpentry in hotels is
generally far superior to the quality of original
carpentry in other structures, such as houses
and stores.

(B)

Hotels built since 1930 can generally
accommodate more guests than those built
before 1930.

(C)

The materials available to carpenters working
before 1930 were not significantly different in
quality from the materials available to carpenters
working after 1930.

(D)

The better the quality of original carpentry in a
building, the less likely that building is to fall into
disuse and be demolished.

(E)

The average length of apprenticeship for
carpenters has declined significantly since
1930.

8.4 Critical Reasoning Practice Questions

659. Scientists typically do their most creative work
before the age of forty. It is commonly thought
that this happens because aging by itself brings
about a loss of creative capacity. However, studies
show that of scientists who produce highly
creative work beyond the age of forty, a
disproportionately large number entered their
field at an older age than is usual. Since by the
age of forty the large majority of scientists have
been working in their field for at least fifteen years,
the studies’ finding strongly suggests that the real
reason why scientists over forty rarely produce
highly creative work is not that they have aged but
rather that scientists over forty have generally
spent too long in their field.

(A)

None of NorthAir’s competitors offers
significantly better seating and service to
economy-class passengers than NorthAir
does.

(B)

On many of the routes that NorthAir flies, it is the
only airline to offer direct flights.

(C)

A few of NorthAir’s economy-class passengers
are satisfied with the service they receive, given
the low price they pay.

(D)

Very few people avoid flying on NorthAir because
of the cramped seating and poor service offered
in economy class.

(E)

The number of people who would be willing
to pay the high fares NorthAir charges for its
business-class seats would decrease if its
economy-class seating were more acceptable.

In the argument given, the two portions in boldface
play which of the following roles?
(A)

The first is a claim, the accuracy of which
is at issue in the argument; the second is a
conclusion drawn on the basis of that claim.

(B)

The first is an objection that has been raised
against a position defended in the argument; the
second is that position.

(C)

The first is evidence that has been used to
support an explanation that the argument
challenges; the second is that explanation.

(D)

The first is evidence that has been used to
support an explanation that the argument
challenges; the second is a competing
explanation that the argument favors.

(E)

The first provides evidence to support an
explanation that the argument favors; the second
is that explanation.

660. NorthAir charges low fares for its economy-class
seats, but it provides very cramped seating and few
amenities. Market research shows that economy
passengers would willingly pay more for wider seating
and better service, and additional revenue provided
by these higher ticket prices would more than cover
the additional cost of providing these amenities. Even
though NorthAir is searching for ways to improve
its profitability, it has decided not to make these
improvements.
Which of the following, if true, would most help to
explain NorthAir’s decision in light of its objectives?

661. Which of the following most logically completes the
argument given?
Asthma, a chronic breathing disorder, is significantly
more common today among adult competitive
swimmers than it is among competitive athletes who
specialize in other sports. Although chlorine is now
known to be a lung irritant and swimming pool water
is generally chlorinated, it would be rash to assume
that frequent exposure to chlorine is the explanation of
the high incidence of asthma among these swimmers,
since __________.
(A)

young people who have asthma are no more
likely to become competitive athletes than are
young people who do not have asthma

(B)

competitive athletes who specialize in sports
other than swimming are rarely exposed to
chlorine

(C)

competitive athletes as a group have a
significantly lower incidence of asthma than do
people who do not participate in competitive
athletics

(D)

until a few years ago, physicians routinely
recommended competitive swimming to children
with asthma, in the belief that this form of
exercise could alleviate asthma symptoms

(E)

many people have asthma without knowing
they have it and thus are not diagnosed with
the condition until they begin engaging in very
strenuous activities, such as competitive athletics

543

GMAT® Official Guide 2018

662. In the country of Marut, the Foreign Trade Agency’s
records were reviewed in 1994 in light of information
then newly available about neighboring Goro. The
review revealed that in every year since 1963, the
agency’s projection of what Goro’s gross national
product (GNP) would be five years later was a serious
underestimate. The review also revealed that in every
year since 1963, the agency estimated Goro’s GNP
for the previous year—a Goro state secret—very
accurately.
Of the following claims, which is most strongly
supported by the statements given?
(A)

Goro’s GNP fluctuated greatly between 1963 and
1994.

(B)

Prior to 1995, Goro had not released data
intended to mislead the agency in making its five‐
year projections.

(C)

The amount by which the agency underestimated
the GNP it projected for Goro tended to increase
over time.

(D)

Even before the new information came to light,
the agency had reason to think that at least
some of the five‐year projections it had made
were inaccurate.

(E)

The agency’s five‐year projections of Goro’s GNP
had no impact on economic planning in Marut.

663. Vargonia has just introduced a legal requirement that
student‐teacher ratios in government‐funded schools
not exceed a certain limit. All Vargonian children are
entitled to education, free of charge, in these schools.
When a recession occurs and average incomes fall,
the number of children enrolled in government‐funded
schools tends to increase. Therefore, though most
employment opportunities contract in economic
recessions, getting a teaching job in Vargonia’s
government‐funded schools will not be made more
difficult by a recession.
Which of the following would be most important to
determine in order to evaluate the argument?

544

(A)

Whether in Vargonia there are any schools not
funded by the government that offer children an
education free of charge

(B)

Whether the number of qualified applicants
for teaching positions in government‐funded
schools increases significantly during economic
recessions

(C)

What the current student‐teacher ratio in
Vargonia’s government‐funded schools is

(D)

What proportion of Vargonia’s workers currently
hold jobs as teachers in government‐funded
schools

(E)

Whether in the past a number of governmentfunded schools in Vargonia have had studentteacher ratios well in excess of the new limit

664. In Colorado subalpine meadows, nonnative dandelions
co‐occur with a native flower, the larkspur. Bumblebees
visit both species, creating the potential for
interactions between the two species with respect to
pollination. In a recent study, researchers selected
16 plots containing both species; all dandelions were
removed from eight plots; the remaining eight control
plots were left undisturbed. The control plots yielded
significantly more larkspur seeds than the dandelion‐
free plots, leading the researchers to conclude that
the presence of dandelions facilitates pollination
(and hence seed production) in the native species by
attracting more pollinators to the mixed plots.
Which of the following, if true, most seriously
undermines the researchers’ reasoning?
(A)

Bumblebees preferentially visit dandelions over
larkspurs in mixed plots.

(B)

In mixed plots, pollinators can transfer pollen
from one species to another to augment seed
production.

(C)

If left unchecked, nonnative species like
dandelions quickly crowd out native species.

(D)

Seed germination is a more reliable measure of
a species’ fitness than seed production.

(E)

Soil disturbances can result in fewer blooms, and
hence lower seed production.

665. An experiment was done in which human subjects
recognize a pattern within a matrix of abstract designs
and then select another design that completes that
pattern. The results of the experiment were surprising.
The lowest expenditure of energy in neurons in the
brain was found in those subjects who performed most
successfully in the experiments.
Which of the following hypotheses best accounts for
the findings of the experiment?

3.2 Diagnostic Test Verbal Questions

(A)

The neurons of the brain react less when a
subject is trying to recognize patterns than when
the subject is doing other kinds of reasoning.

(B)

Those who performed best in the experiment
experienced more satisfaction when working with
abstract patterns than did those who performed
less well.

(C)

People who are better at abstract pattern
recognition have more energy‐efficient neural
connections.

(D)

The energy expenditure of the subjects’ brains
increases when a design that completes the
initially recognized pattern is determined.

(E)

The task of completing a given design is more
capably performed by athletes, whose energy
expenditure is lower when they are at rest.

666. With seventeen casinos, Moneyland operates the
most casinos in a certain state. Although intent on
expanding, it was outmaneuvered by Apex Casinos
in negotiations to acquire the Eldorado chain. To
complete its acquisition of Eldorado, Apex must sell
five casinos to comply with a state law forbidding any
owner to operate more than one casino per county.
Since Apex will still be left operating twenty casinos
in the state, it will then have the most casinos in the
state.

(C)

Some of the state’s counties do not permit
casinos.

(D)

Moneyland already operates casinos in the
majority of the state’s counties.

(E)

Apex will use funds it obtains from the sale of
the five casinos to help fund its acquisition of the
Eldorado chain.

667. It is widely assumed that people need to engage
in intellectual activities such as solving crossword
puzzles or mathematics problems in order to maintain
mental sharpness as they age. In fact, however,
simply talking to other people—that is, participating
in social interaction, which engages many mental and
perceptual skills—suffices. Evidence to this effect
comes from a study showing that the more social
contact people report, the better their mental skills.
Which of the following, if true, most seriously weakens
the force of the evidence cited?
(A)

As people grow older, they are often advised
to keep exercising their physical and mental
capacities in order to maintain or improve them.

(B)

Many medical conditions and treatments that
adversely affect a person’s mental sharpness
also tend to increase that person’s social
isolation.

Which of the following, if true, most seriously
undermines the prediction?

(C)

Many people are proficient both in social
interactions and in solving mathematical
problems.

(A)

Apex, Eldorado, and Moneyland are the only
organizations licensed to operate casinos in the
state.

(D)

The study did not itself collect data but analyzed
data bearing on the issue from prior studies.

(E)

(B)

The majority of Eldorado’s casinos in the state
will need extensive renovations if they are to
continue to operate profitably.

The tasks evaluating mental sharpness for which
data were compiled by the study were more akin
to mathematics problems than to conversation.

545

GMAT® Official Guide 2018

8.5 Answer Key
544. E

575. E

606. E

637. C

545. C

576. C

607. C

638. B

546. E

577. E

608. A

639. D

547. A

578. D

609. C

640. D

548. A

579. E

610. E

641. A

549. B

580. A

611. A

642. C

550. E

581. A

612. D

643. C

551. E

582. C

613. D

644. B

552. C

583. D

614. C

645. E

553. E

584. E

615. D

646. D

554. A

585. C

616. E

647. A

555. A

586. D

617. D

648. C

556. D

587. C

618. D

649. B

557. B

588. D

619. A

650. A

558. C

589. C

620. A

651. B

559. A

590. E

621. D

652. B

560. C

591. C

622. A

653. D

561. B

592. D

623. E

654. B

562. D

593. A

624. E

655. D

563. D

594. A

625. A

656. C

564. C

595. D

626. D

657. C

565. E

596. C

627. E

658. D

566. E

597. E

628. E

659. E

567. E

598. C

629. E

660. E

568. B

599. C

630. B

661. D

569. A

600. A

631. E

662. D

570. E

601. E

632. D

663. B

571. B

602. D

633. D

664. E

572. E

603. A

634. B

665. C

573. D

604. D

635. A

666. A

574. D

605. A

636. D

667. B

546

8.6 Critical Reasoning Answer Explanations

8.6 Answer Explanations
The following discussion is intended to familiarize you with the most efficient and effective approaches
to critical reasoning questions. The particular questions in this chapter are generally representative of
the kinds of critical reasoning questions you will encounter on the GMAT exam. Remember that it is the
problem solving strategy that is important, not the specific details of a particular question.

544. Neuroscientist: Memory evolved to help animals react appropriately to situations they encounter by drawing on the past
experience of similar situations. But this does not require that animals perfectly recall every detail of all their experiences.
Instead, to function well, memory should generalize from past experiences that are similar to the current one.
The neuroscientist’s statements, if true, most strongly support which of the following conclusions?
(A)

At least some animals perfectly recall every detail of at least some past experiences.

(B)

Perfectly recalling every detail of all their past experiences could help at least some animals react more appropriately
than they otherwise would to new situations they encounter.

(C)

Generalizing from past experiences requires clear memories of most if not all the details of those experiences.

(D)

Recalling every detail of all past experiences would be incompatible with any ability to generalize from those
experiences.

(E)

Animals can often react more appropriately than they otherwise would to situations they encounter if they draw on
generalizations from past experiences of similar situations.

Argument Construction
Situation

A neuroscientist claims that memory evolved to help animals learn how to react appropriately by
generalizing from past experiences but that this does not require animals to remember all details of
those experiences.

Reasoning

What conclusion would the neuroscientist’s theory about memory most strongly support? The
neuroscientist asserts that the evolutionary function of memory is to help animals learn to react
appropriately by drawing on generalizations from similar experiences they have had. If memory is
to serve this function, drawing on generalizations must actually help animals learn to react more
appropriately than they otherwise would, even when they do not remember all the details of past
experiences.

A
B

C
D
E

Even if no animal ever recalls all the details of any past experience, animals could still learn through
generalizations, as the neuroscientist claims.
This statement could be false even if all of what the neuroscientist says is true. Even if it were never helpful
for any animal to recall every detail of all its past experiences, animals could still benefit by learning
through generalizations.
Generalizations from experiences might be made while the experiences are occurring, so that only the
generalizations and not the details need to be remembered.
The neuroscientist only claims that remembering perfect details is not required for memory to serve its
function, not that such perfect recall is incompatible with memory serving its function.
Correct. If the evolutionary function of memory is to help animals react more appropriately by drawing
on generalizations from past experiences, it follows that animal memories can often successfully serve this
function in this manner.

The correct answer is E.

547

GMAT® Official Guide 2018

545. Astronomer: Most stars are born in groups of thousands, each star in a group forming from the same parent cloud of gas.
Each cloud has a unique, homogeneous chemical composition. Therefore, whenever two stars have the same chemical
composition as each other, they must have originated from the same cloud of gas.
Which of the following, if true, would most strengthen the astronomer’s argument?
(A)

In some groups of stars, not every star originated from the same parent cloud of gas.

(B)

Clouds of gas of similar or identical chemical composition may be remote from each other.

(C)

Whenever a star forms, it inherits the chemical composition of its parent cloud of gas.

(D)

Many stars in vastly different parts of the universe are quite similar in their chemical compositions.

(E)

Astronomers can at least sometimes precisely determine whether a star has the same chemical composition as its
parent cloud of gas.

Argument Evaluation
Situation

Most stars are born in groups, any one of which forms from a parent gas cloud with a unique,
homogenous chemical composition.

Reasoning

What would be additional evidence that any two stars with the same chemical composition
originated from the same gas cloud? The implicit reasoning is that since the chemical composition
of each gas cloud is unique and homogenous, any two stars that formed from gas with the same
chemical composition must have originated from the same cloud. The astronomer then infers that
if two stars have the same composition now, they must have originated from the same cloud. This
inference requires the assumption that the composition each star has now depends only on the
composition of the cloud in which it originated. Any evidence that supports this assumption will
strengthen the argument.

A
B

C
D
E

Whether or not stars born in different clouds of gas are ever in the same “group” is not clearly relevant to
whether or not they ever have the same chemical composition.
How remote clouds of similar compositions are from each other is not clearly relevant to whether
stars that have the same chemical composition may have formed from different clouds of gas. Also, the
suggestion that different gas clouds may have identical compositions conflicts with the astronomer’s
premise that the composition of each cloud from which stars form is unique.
Correct. If each star’s composition is identical to that of its parent cloud, and each cloud’s composition is
unique, then any two stars identical in composition must have formed from the same parent cloud.
If anything, this would suggest that stars with the same composition might have formed from different
clouds, so it would weaken rather than strengthen the argument.
If astronomers could do this, they might be able to obtain additional evidence for or against the position
taken in the argument, but this, in itself, provides no reason to suppose that the evidence would support,
rather than weaken, that position. They might find that the stars’ compositions do not precisely correlate
with the compositions of the stars’ parent gas clouds.

The correct answer is C.

548

8.6 Critical Reasoning Answer Explanations

546. With employer-paid training, workers have the potential to become more productive not only in their present employment but
also in any number of jobs with different employers. To increase the productivity of their workforce, many firms are planning to
maintain or even increase their investments in worker training. But some training experts object that if a trained worker is hired
away by another firm, the employer that paid for the training has merely subsidized a competitor. They note that such hiring
has been on the rise in recent years.
Which of the following would, if true, contribute most to defeating the training experts’ objection to the firms’ strategy?
(A)

Firms that promise opportunities for advancement to their employees get, on average, somewhat larger numbers of
job applications from untrained workers than do firms that make no such promise.

(B)

In many industries, employees that take continuing-education courses are more competitive in the job market.

(C)

More and more educational and training institutions are offering reduced tuition fees to firms that subsidize worker
training.

(D)

Research shows that workers whose training is wholly or partially subsidized by their employer tend to get at least as
much training as do workers who pay for all their own training.

(E)

For most firms that invest in training their employees, the value added by that investment in employees who stay
exceeds the value lost through other employees’ leaving to work for other companies.

Evaluation of a Plan
Situation

Many firms pay to train their workers in order to increase their workforces’ productivity. But in
recent years firms have been increasingly hiring away from each other workers who have had such
training.

Reasoning

What would most help address the concern that firms that pay to train workers are thereby subsidizing
competitors that hire away those workers? In order for the employer-paid training to be worthwhile
for a given firm despite the risk of subsidizing competitors that may hire away the trained workers,
that firm has to gain more benefits from the training than it loses by subsidizing such competitors.
Any evidence that this is true for most firms would help to address the experts’ concern.

A

A typical firm does not necessarily want larger numbers of applications from unqualified workers. And if
hired, those workers can still be hired away by competitors after the firm has paid to train them, just as the
experts warned.
B
This suggests that in many industries, companies, rather than investing in employee training, prefer to
hire employees who already have specifically relevant training (perhaps funded by other companies). If
anything, this slightly supports, rather than defeats, the training experts’ view. No firm has an interest in
making its own employees more competitive in the job market unless the firm is likely to benefit from
their being so.
C Even firms that pay reduced tuition fees for worker training may lose the money they pay for those fees
and effectively subsidize competitors that hire the trained employees away. So this does not defeat the
training experts’ objection.
D The more highly trained workers—regardless of whether their training was company subsidized or not—
would presumably be prime targets for recruitment by competing firms, just as the experts warned. The
research finding in question does not help defeat the experts’ objection.
E Correct. This explicitly indicates that most firms gain more than they lose from the general practice of
firms paying to train their workers.
The correct answer is E.

549

GMAT® Official Guide 2018

547. Candle Corporation’s television stations are likely to have more income from advertisers than previously. This is because
advertisers prefer to reach people in the 18‐ to 49‐year‐old age group and the number of people in that group watching
Candle television is increasing. Furthermore, among Candle viewers, the percentage of viewers 18 to 49 years old is
increasing.
Which of the following, if true, would most strengthen the argument that Candle Corporation will receive more income from
advertisers?
(A)

Advertisers carefully monitor the demographic characteristics of television audiences and purchase advertising time
to reach the audiences they prefer to reach.

(B)

Among people over 49 years old, fewer viewers of Candle stations buy products advertised on television than do
viewers of other stations.

(C)

There will be increasingly more advertisements on television that are directed at viewers who are over 49 years old.

(D)

Candle stations plan to show reruns of television shows during hours when other stations run shows for the first time.

(E)

People 18 to 49 years old generally have less disposable income to spend than do people over 49 years old.

Argument Evaluation
Situation

Both the number and the percentage of Candle television viewers who are 18 to 49 years old are
increasing. Advertisers prefer to reach people in this age group.

Reasoning

What evidence, when combined with the cited facts, would most support the prediction that Candle
will receive more income from advertisers? The argument assumes that the increasing number and
percentage of Candle viewers in the age group that advertisers prefer to reach will probably
encourage advertisers to spend more on advertising with Candle. This assumption could be
supported by evidence that the advertisers realize that Candle is getting more viewers in that
preferred age range or by evidence that this awareness will influence the advertisers’ purchase of
advertising time.

A

B
C

D

E

Correct. Advertisers monitoring demographics will probably realize that Candle has increasing numbers
of viewers in their preferred age range. If they purchase advertising to reach viewers in that age range, then
they will probably purchase more advertising time with Candle.
This gives advertisers less reason to advertise on Candle to reach viewers over 49 years old. Other things
being equal, that makes Candle likely to receive less income from advertisers, not more income.
Since the percentage of Candle viewers 18 to 49 years old is growing, the percentage over 49 years old is
probably shrinking. This could make advertisers seeking to reach older viewers less inclined to advertise on
Candle even as they increase their overall television advertising.
Advertisers are not necessarily inclined to purchase more advertising during showings of reruns than
during original airings of television shows and may even be inclined to purchase less advertising during
such showings.
This gives advertisers less incentive to try to reach audiences between 18 and 49 years old and hence less
reason to purchase advertising on Candle.

The correct answer is A.

550

8.6 Critical Reasoning Answer Explanations

548. A provincial government plans to raise the gasoline tax to give people an incentive to drive less, reducing traffic congestion
in the long term. However, skeptics point out that most people in the province live in areas where cars are the only viable
transportation to jobs and stores and therefore cannot greatly change their driving habits in response to higher gasoline
prices.
In light of the skeptics’ objection, which of the following, if true, would most logically support the prediction that the
government’s plan will achieve its goal of reducing traffic congestion?
(A)

The revenue from the tax will be used to make public transportation a viable means of transportation to jobs and
stores for far more people.

(B)

The tax will encourage many residents to switch to more fuel‐efficient cars, reducing air pollution and other problems.

(C)

Because gasoline has been underpriced for decades, the province has many neighborhoods where cars are the only
viable means of transportation.

(D)

Most residents who cannot greatly change their driving habits could compensate for high gasoline prices by reducing
other expenses.

(E)

Traffic congestion is an especially serious problem for people for whom cars are the only viable means of
transportation.

Evaluation of a Plan
Situation

A provincial government plans to raise the gasoline tax in order to reduce traffic congestion by
discouraging people from driving. But skeptics point out that most people in the province have no
viable form of transportation other than driving.

Reasoning

What would suggest that raising the gasoline tax will reduce traffic congestion even though most people
in the province have no viable form of transportation other than driving? The skeptics point out that
since most people in the province have no way to reach jobs or stores except by car, they will not
be able to reduce their driving much even if the gasoline tax increases. Any evidence that raising
the gasoline tax would reduce traffic congestion despite this obstacle would help to support the
plan in light of the skeptics’ objection.

A
B
C

D
E

Correct. If the tax will fund these public transit improvements, then far more people will have a viable
means of transportation other than driving, undermining the basis of the skeptics’ objection.
People switching to fuel‐efficient cars would not reduce traffic congestion.
This essentially only tends to support the skeptics’ objection. Unless the plan somehow helps to alleviate
the necessity of driving (by, for example, making alternative transportation available), the information
provided gives no reason to suppose that the higher costs would significantly reduce traffic congestion.
If residents cannot greatly change their driving habits, then the tax will not reduce traffic congestion.
This suggests that many residents in the province could benefit if the plan did reduce traffic congestion,
but it does not provide a reason to believe the plan will have that effect.

The correct answer is A.

551

GMAT® Official Guide 2018

549. Editorial: The roof of Northtown’s municipal equipment‐storage building collapsed under the weight of last week’s heavy
snowfall. The building was constructed recently and met local building‐safety codes in every particular, except that the nails
used for attaching roof supports to the building’s columns were of a smaller size than the codes specify for this purpose.
Clearly, this collapse exemplifies how even a single, apparently insignificant departure from safety standards can have
severe consequences.
Which of the following, if true, most seriously weakens the editorial’s argument?
(A)

The only other buildings to suffer roof collapses from the weight of the snowfall were older buildings constructed
according to less exacting standards than those in the codes.

(B)

The amount of snow that accumulated on the roof of the equipment‐storage building was greater than the predicted
maximum that was used in drawing up the safety codes.

(C)

Because the equipment‐storage building was not intended for human occupation, some safety‐code provisions that
would have applied to an office building did not apply to it.

(D)

The municipality of Northtown itself has the responsibility for ensuring that buildings constructed within its boundaries
meet the provisions of the building‐safety codes.

(E)

Because the equipment‐storage building was used for storing snow‐removal equipment, the building was almost
completely empty when the roof collapsed.

Argument Evaluation
Situation

The roof of a recently constructed building collapsed under heavy snowfall. The only way the
building did not meet safety standards was that some nails for the roof supports were smaller than
prescribed by the building codes.

Reasoning

What would make it less likely that the building’s collapse resulted from a single, apparently minor
departure from safety standards? The building met safety standards except for the size of the nails.
So if the collapse exemplifies how a departure from safety standards can have severe consequences,
as the conclusion claims, then the size of the nails had to be responsible for the collapse. Thus,
evidence that a factor other than the size of the nails could fully account for the collapse would
weaken the argument.

A
B
C

D
E

This suggests that the snow would not have been heavy enough to collapse the roof if the construction had
completely met the safety standards, so it strengthens, rather than weakens, the argument.
Correct. This suggests that the snow could have collapsed the roof even if the nails had met the safety
standards, thus casting doubt on the assumption that the nails’ inadequacy was responsible for the collapse.
The claim that the safety requirements for this building were weaker than some others tends slightly to
strengthen, rather than weaken, the hypothesis that the bad consequences resulted partly from a failure to
comply. Even if safety‐code provisions for an equipment‐storage building differ from those for an office
building, they may still be adequate to ensure the roof ’s stability.
The question of who was responsible for ensuring compliance with the safety codes is irrelevant to whether
a failure to comply was responsible for the roof ’s collapse.
This suggests that the alleged consequences of failing to meet safety standards were less severe than they
could have been, but it is irrelevant to determining the cause of the collapse.

The correct answer is B.

552

8.6 Critical Reasoning Answer Explanations

550. Political theorist: Even with the best spies, area experts, and satellite surveillance, foreign policy assessments can still lack
important information. In such circumstances intuitive judgment is vital. A national leader with such judgment can make
good decisions about foreign policy even when current information is incomplete, since __________.
Which of the following, if true, most logically completes the argument?
(A)

the central reason for failure in foreign policy decision making is the absence of critical information

(B)

those leaders whose foreign policy decisions have been highly ranked have also been found to have good intuitive
judgment

(C)

both intuitive judgment and good information are required for sound decision making

(D)

good foreign policy decisions often lead to improved methods of gathering information

(E)

intuitive judgment can produce good decisions based on past experience, even when there are important gaps in
current information

Argument Construction
Situation

National leaders sometimes must make foreign policy decisions while lacking important
information.

Reasoning

What would most help support the claim that a national leader with intuitive judgment can make good
foreign policy decisions without complete information? The word since preceding the blank indicates
that the blank should be filled with a premise supporting the statement immediately before the
blank. So an observation that supports the claim that a national leader with intuitive judgment can
make good foreign policy decisions without complete information would logically complete the
argument.

A
B

C

D
E

This gives us no reason to suppose that intuitive judgment helps national leaders avoid such failures.
This does not specify who ranked the foreign policy decisions, nor how they determined the rankings, so
it gives us no reason to accept those rankings. For all we know, the anonymous rankers may have used the
dubious rankings they created as the sole evidence for their so-called findings about which leaders have
good intuitive judgment.
This implies that intuitive judgment alone is inadequate without good information, so it undermines
rather than supports the claim that national leaders can make good foreign policy decisions with intuitive
judgment while lacking complete information.
This gives us no reason to suppose that good foreign policy decisions can be made in the first place by
leaders lacking important information.
Correct. This suggests that national leaders can make good foreign policy decisions using intuitive
judgment based on their past foreign policy experience, even without complete information about the
current situations they’re facing.

The correct answer is E.

553

GMAT® Official Guide 2018

551. During the earliest period of industrialization in Britain, steam engines were more expensive to build and operate than
either windmills or water mills, the other practicable sources of power for factories. Yet despite their significant cost
disadvantage, steam‐powered factories were built in large numbers well before technical improvements brought their cost
down. Furthermore, they were built even in regions where geographical conditions permitted the construction of wind‐ and
water‐powered factories close to major markets.
Which of the following, if true, most helps to explain the proliferation of steam‐powered factories during the earliest period
of industrialization in Britain?
(A)

In many areas of Britain, there were fewer steam‐powered factories than wind‐ or water‐powered factories in the
earliest period of industrialization.

(B)

Unlike wind‐ or water‐powered factories, steam‐powered factories were fueled with coal, which sometimes had to be
transported significant distances from the mine to the site of the factory.

(C)

It was both difficult and expensive to convert a factory from wind power or water power to steam power.

(D)

In the early period of industrialization, many goods sold in towns and cities could not be mass‐produced in factories.

(E)

In Britain, the number of sites where a wind‐ or water‐powered factory could be built was insufficient to provide for all
of the demand for factory‐produced goods at the time.

Argument Construction
Situation

Although steam engines were more expensive than windmills and water mills in early industrial
Britain, many steam‐powered factories were built even in regions where the construction of wind‐
and water‐powered factories was geographically feasible.

Reasoning

Why might steam‐powered factories have proliferated despite their cost disadvantage? Early
industrialists would have needed some positive reason to choose steam over less expensive power
sources for their factories. For example, steam engines might have operated faster or more
effectively than windmills or water mills. Or steam engines might have received government
subsidies. Or conditions restricting the number or locations of windmills and water mills might
have forced industrialists to use steam power instead.

A
B
C
D
E

This suggests that the steam‐powered factories did not initially proliferate as widely as they might have,
but it does not explain why they proliferated to the extent that they did.
The inconvenience of transporting coal for steam‐powered factories would have made those factories less
likely to proliferate, not more likely.
The difficulty of converting factories to steam power would have made steam‐powered factories less likely
to proliferate, not more likely.
The technological inability to mass‐produce popular products in factories would have made factories in
general less likely to proliferate, including steam‐powered factories.
Correct. The inadequate number of sites for wind‐ and water‐powered factories might have encouraged
early industrialists to build steam‐powered factories instead, since the high demand for factory‐produced
goods could have made these factories profitable despite their cost disadvantage.

The correct answer is E.

554

8.6 Critical Reasoning Answer Explanations

552. Snowmaking machines work by spraying a mist that freezes immediately on contact with cold air. Because the sudden
freezing kills bacteria, QuickFreeze is planning to market a wastewater purification system that works on the same principle.
The process works only when temperatures are cold, however, so municipalities using it will still need to maintain a
conventional system.
Which of the following, if true, provides the strongest grounds for a prediction that municipalities will buy QuickFreeze’s
purification system despite the need to maintain a conventional purification system as well?
(A)

Bacteria are not the only impurities that must be removed from wastewater.

(B)

Many municipalities have old wastewater purification systems that need to be replaced.

(C)

Conventional wastewater purification systems have not been fully successful in killing bacteria at cold temperatures.

(D)

During times of warm weather, when it is not in use, QuickFreeze’s purification system requires relatively little
maintenance.

(E)

Places where the winters are cold rarely have a problem of water shortage.

Evaluation of a Plan
Situation

QuickFreeze is planning to market wastewater purification systems that work by spraying a mist
that freezes on contact with cold air. The sudden freezing kills bacteria. Because the system works
only at cold temperatures, municipalities using it will still need to maintain a conventional system.

Reasoning

Which statement provides the strongest grounds for thinking that at least some municipalities will buy the
purification system despite the need to maintain a conventional purification system as well? The passage
tells us why a municipality using a QuickFreeze wastewater purification system would still need
a conventional system. But why would a municipality want the QuickFreeze system in addition
to a conventional system? If conventional systems are not fully effective at cold temperatures, the
QuickFreeze system would allow municipalities that sometimes experience cold temperatures to
purify their wastewater more effectively.

A

B

C
D

E

There is no basis in the passage for determining whether the QuickFreeze system will help remove
impurities other than bacteria from wastewater. If it does not, this answer choice implies that the
QuickFreeze system would not be sufficient for purifying wastewater. This would actually undermine the
prediction.
The passage states that municipalities using the QuickFreeze system would still need a conventional
system. Thus, the old conventional wastewater systems would still need to be replaced with new
conventional systems. This answer choice provides no reason to think municipalities would buy the
QuickFreeze system.
Correct. This statement, if true, would strengthen the prediction, because it provides a valid reason why
the QuickFreeze system could be needed alongside conventional ones: it is more effective in cold weather.
Although this claim does undercut one reason for thinking municipalities might not be likely to purchase
the QuickFreeze system, it provides little reason to think that they will purchase such a system. Perhaps in
times of cold weather, the QuickFreeze system is very expensive to maintain.
The issue of whether or not there are water shortages in places where winters are cold is not directly
relevant. If conventional wastewater systems are sufficient to purify water in such places, municipalities
would not need the QuickFreeze system (as they would still need to maintain a conventional purification
system).

The correct answer is C.

555

GMAT® Official Guide 2018

553. Suncorp, a new corporation with limited funds, has been clearing large sections of the tropical Amazon forest for cattle
ranching. This practice continues even though greater profits can be made from rubber tapping, which does not destroy the
forest, than from cattle ranching, which does destroy the forest.
Which of the following, if true, most helps to explain why Suncorp has been pursuing the less profitable of the two economic
activities mentioned above?
(A)

The soil of the Amazon forest is very rich in nutrients that are important in the development of grazing lands.

(B)

Cattle‐ranching operations that are located in tropical climates are more profitable than cattle‐ranching operations that
are located in cold‐weather climates.

(C)

In certain districts, profits made from cattle ranching are more heavily taxed than profits made from any other
industry.

(D)

Some of the cattle that are raised on land cleared in the Amazon are killed by wildcats.

(E)

The amount of money required to begin a rubber‐tapping operation is twice as high as the amount needed to begin a
cattle ranch.

Argument Construction
Situation

Suncorp is a new corporation with limited funds. It has been clearing large sections of the tropical
Amazon forest for ranching, even though rubber‐tapping would be more profitable.

Reasoning

What would explain why Suncorp is clearing sections of the rain forest for ranching, even though rubber
tapping would be more profitable? Because Suncorp has limited funds, if rubber tapping has much
higher start‐up costs, Suncorp might not have enough money to start rubber‐tapping operations.
If cattle ranching has much lower start‐up costs than rubber tapping, Suncorp might be able to
afford such an operation.

A

B

C
D
E

This statement gives a reason why cattle ranching in the Amazon might be more profitable than one might
otherwise think it would be. However, we already know from the passage that rubber tapping would be
more profitable than cattle ranching. So, this answer choice does not help explain why cattle ranching
might be preferable to rubber tapping.
The comparison between the profitableness of cattle ranching in tropical climates and in cold‐weather
climates is irrelevant. The passage only covers cattle ranching in the tropical Amazon forest. This answer
choice would at most explain why Suncorp is undertaking cattle ranching in the Amazon rather than in
some cold‐weather location.
This statement makes what needs to be explained harder to understand, for it indicates that cattle ranching
in the Amazon might be less profitable than one would otherwise think.
Like answer choice (C), this statement indicates a disadvantage of cattle ranching in the Amazon. So, it
does not explain why cattle ranching would be preferred to some other economic activity.
Correct. Because it costs less to begin cattle ranching than it does to begin rubber tapping, Suncorp—
which has limited funds—would have a reason to pursue cattle ranching over a potentially more profitable
activity.

The correct answer is E.

556

8.6 Critical Reasoning Answer Explanations

554. Archaeologists use technology to analyze ancient sites. It is likely that this technology will advance considerably in the near
future, allowing archaeologists to gather more information than is currently possible. If they study certain sites now, they
risk contaminating or compromising them for future studies. Therefore, in order to maximize the potential for gathering
knowledge in the long run, a team of archaeologists plans to delay the examination of a newly excavated site.
Which of the following would be most useful to investigate for the purpose of evaluating the plan’s prospects for achieving
its goal?
(A)

Whether any of the contents of the site will significantly deteriorate before the anticipated technology is available

(B)

Whether there will continue to be improvements on the relevant technology

(C)

Whether the team can study a site other than the newly excavated site for the time being

(D)

Whether the site was inhabited by a very ancient culture

(E)

Whether the anticipated technology will damage objects under study

Evaluation of a Plan
Situation

To avoid prematurely compromising a newly excavated site, an archaeological team plans to
postpone examining it until more advanced technology is developed that will let them gather more
information from it. Their goal is to maximize the potential for gathering knowledge.

Reasoning

What would be most helpful to investigate in order to assess how likely it is that delaying examination of
the site will maximize the potential for gathering knowledge from it? In order to maximize (or even
increase) the potential for gathering knowledge from the site by delaying its examination, the risk
of compromising the site by examining it now has to be greater than the risk that the site will
be compromised as much or more by delaying the examination. The delay might also increase
the risk that the site will never be examined at all—for example, the team might lose its funding
while it delays, or changes in local political conditions might prevent the site’s future examination.
Investigating any of these risks could be helpful in assessing the likelihood that the team’s plan will
achieve its goal.

A

B

C

D
E

Correct. If any of the site’s contents will significantly deteriorate before the technology becomes available,
that could reduce the ability to gather future information from the site even more than examining and
compromising the site now would.
The passage already tells us that it is likely the technology will advance considerably in the near future.
Given this information, further inquiry into whether there will be any ongoing (perhaps minor)
improvements is somewhat redundant and probably of minimal value with respect to evaluating the plan’s
likelihood of success.
Even if the team can study a second site in the meanwhile, they might maximize the overall potential
for gathering knowledge by delaying the examination of either site, both sites, or neither site until more
advanced technology is available.
The age of the culture that inhabited the site is irrelevant to assessing the risks of delaying the site’s
examination until more advanced technology is available.
Even if the anticipated technology will damage or destroy the objects under study, it might still maximize
the amount of knowledge that can be gathered from those objects. Without any comparison between the
damage risk that would be incurred by proceeding with the current technology and the damage risk that
would be incurred by waiting, the mere fact that some damage would occur is irrelevant.

The correct answer is A.

557

GMAT® Official Guide 2018

555. A newly discovered painting seems to be the work of one of two seventeenth‐century artists, either the northern German
Johannes Drechen or the Frenchman Louis Birelle, who sometimes painted in the same style as Drechen. Analysis of the
carved picture frame, which has been identified as the painting’s original seventeenth‐century frame, showed that it is made
of wood found widely in northern Germany at the time, but rare in the part of France where Birelle lived. This shows that the
painting is most likely the work of Drechen.
Which of the following is an assumption that the argument requires?
(A)

The frame was made from wood local to the region where the picture was painted.

(B)

Drechen is unlikely to have ever visited the home region of Birelle in France.

(C)

Sometimes a painting so resembles others of its era that no expert is able to confidently decide who painted it.

(D)

The painter of the picture chose the frame for the picture.

(E)

The carving style of the picture frame is not typical of any specific region of Europe.

Argument Construction
Situation

The original frame of a seventeenth‐century painting that seems to be by either the northern
German Johannes Drechen or the Frenchman Louis Birelle is made of a type of wood much more
common in northern Germany than in France, suggesting that Drechen was the painter.

Reasoning

What must be true in order for the facts presented to support the conclusion that the painting is by
Drechen? The argument is that in the seventeenth century, the type of wood in the frame was more
common in northern Germany where Drechen was from than in France where Birelle was from,
so probably Drechen painted the picture. In order for this inference to be plausible, the argument
must implicitly assume that the frame’s wood was from the same region the painter was from. And
in order to justify this assumed connection between the wood and the region the painter was from,
the argument must also assume that the painter painted the picture in that region.

A

B
C
D
E

Correct. If the frame were not made of wood local to the region where the picture was painted, or if that
region in turn were not where the painter was from, then the cited fact about where the wood was more
common would be irrelevant to the conclusion about who painted the picture.
The argument is compatible with the plausible hypothesis that Drechen visited France at some point
during his lifetime but did not frame this or any painting with French wood.
Even if experts always felt confident in deciding who painted any picture, examining the wood in the
picture frame might help them decide correctly.
The argument is compatible with the plausible hypothesis that the picture was sold to a local customer
who then chose a frame of local wood.
The argument would be even stronger if this were false and the carving style of the frame were typical of
northern Germany specifically.

The correct answer is A.

558

8.6 Critical Reasoning Answer Explanations

556. Beginning in 1966 all new cars sold in Morodia were required to have safety belts and power steering. Previously, most
cars in Morodia were without these features. Safety belts help to prevent injuries in collisions, and power steering helps to
avoid collisions in the first place. But even though in 1966 one-seventh of the cars in Morodia were replaced with new cars,
the number of car collisions and collision-related injuries did not decline.
Which of the following, if true about Morodia, most helps to explain why the number of collisions and collision-related injuries in
Morodia failed to decline in 1966?
(A)

Because of a driver-education campaign, most drivers and passengers in cars that did have safety belts used them in
1966.

(B)

Most of the new cars bought in 1966 were bought in the months of January and February.

(C)

In 1965, substantially more than one-seventh of the cars in Morodia were replaced with new cars.

(D)

An excessive reliance on the new safety features led many owners of new cars to drive less cautiously in 1966 than
before.

(E)

The seat belts and power steering put into new cars sold in 1966 had to undergo strict quality-control inspections by
manufacturers, whether the cars were manufactured in Morodia or not.

Argument Construction
Situation

Starting in 1966, new cars sold in Morodia were required to have safety belts and power steering.
But the numbers of car collisions and collision-related injuries did not decline that year.

Reasoning

What could explain why the newly required safety features did not reduce the numbers of collisions and
collision-related injuries in 1966? The passage says that power steering helps to prevent collisions
and that safety belts help to prevent collision-related injuries. Since most Morodian cars
previously lacked these features, and one-seventh of them were replaced with new cars in 1966,
the proportion of cars with these features must have increased that year. This should have reduced
the numbers of collisions and collision-related injuries unless some other factor counteracted the
reductions. Evidence of any such countervailing factor would help to explain why the numbers did
not decrease.

A
B

C

D

E

Increased usage of safety belts should have reduced the number of collision-related injuries, so it would not
help explain why this number did not decrease.
If the new cars bought in 1966 were mostly purchased early in the year, the increased proportion of cars
with the newly required safety features should have started more significantly reducing the numbers of
collisions and collision-related injuries early in the year, producing greater reductions for the year as a
whole.
However, many cars were replaced in the year before the safety features were required, in 1966 the
replacement of one-seventh of all Morodian cars should still have increased the overall proportion of
Morodian cars with the safety features and thus reduced the numbers of collisions and collision-related
injuries.
Correct. If many owners of the cars with the new safety features drove less cautiously, their recklessness
could have increased the overall numbers of collisions and collision-related injuries despite any benefits
from the safety features.
Strict quality-control inspections should have made the safety features more reliable, further reducing the
numbers of collisions and collision-related injuries.

The correct answer is D.

559

GMAT® Official Guide 2018

557. In order to reduce the number of items damaged while in transit to customers, packaging consultants recommended
that the TrueSave mail‐order company increase the amount of packing material so as to fill any empty spaces in its
cartons. Accordingly, TrueSave officials instructed the company’s packers to use more packing material than before, and
the packers zealously acted on these instructions and used as much as they could. Nevertheless, customer reports of
damaged items rose somewhat.
Which of the following, if true, most helps to explain why acting on the consultants’ recommendation failed to achieve its
goal?
(A)

The change in packing policy led to an increase in expenditure on packing material and labor.

(B)

When packing material is compressed too densely, it loses some of its capacity to absorb shock.

(C)

The amount of packing material used in a carton does not significantly influence the ease with which a customer can
unpack the package.

(D)

Most of the goods that TrueSave ships are electronic products that are highly vulnerable to being damaged in transit.

(E)

TrueSave has lost some of its regular customers as a result of the high number of damaged items they received.

Evaluation of a Plan
Situation

Mail‐order company TrueSave wants to reduce the number of items damaged while in transit to
customers. Packaging consultants recommended that to achieve this goal, the company should use
more packing material to fill empty spaces in its cartons. The company’s packers began using as
much packing material as they could, yet reports of damaged items rose rather than fell.

Reasoning

What would help explain why the company’s acting on the recommendation did not achieve its goal? The
recommendation involved increasing the amount of packing material, so there must have been
something about that increase that led to more damage. More damage would be likely to result if
stuffing more packing material into shipping boxes made the packaging less effective.

A
B
C

D

E

An increase in expenditure on packing material and labor might affect the company’s profitability, but it
would have no effect on whether items were damaged in transit.
Correct. This statement adequately explains why more items, rather than fewer, were damaged in transit.
If customers were able to remove their items just as easily from boxes filled with more packing material
as from boxes using less packing material, the items would be unaffected by an increase in the amount of
packing material used.
The kind of goods TrueSave ships most frequently is not relevant to the question of why increasing the
amount of packing material failed to reduce the number of items damaged in transit, since they most likely
shipped this same kind of goods both before and after making the recommended change.
The loss of regular customers helps explain why TrueSave turned to the packaging consultants for help,
but it does not help explain why those consultants’ recommendation failed to reduce the number of items
damaged in transit.

The correct answer is B.

560

8.6 Critical Reasoning Answer Explanations

558. Parland’s alligator population has been declining in recent years, primarily because of hunting. Alligators prey heavily on
a species of freshwater fish that is highly valued as food by Parlanders, who had hoped that the decline in the alligator
population would lead to an increase in the numbers of these fish available for human consumption. Yet the population of
this fish species has also declined, even though the annual number caught for human consumption has not increased.
Which of the following, if true, most helps to explain the decline in the population of the fish species?
(A)

The decline in the alligator population has meant that fishers can work in some parts of lakes and rivers that were
formerly too dangerous.

(B)

Over the last few years, Parland’s commercial fishing enterprises have increased the number of fishing boats they
use.

(C)

The main predator of these fish is another species of fish on which alligators also prey.

(D)

Many Parlanders who hunt alligators do so because of the high market price of alligator skins, not because of the
threat alligators pose to the fish population.

(E)

In several neighboring countries through which Parland’s rivers also flow, alligators are at risk of extinction as a result
of extensive hunting.

Argument Construction
Situation

Due to hunting, Parland’s alligator population has been declining. Parlanders had hoped that
the population of a certain prized species of freshwater fish that alligators prey on would have
increased as a result, but the population of this freshwater fish has actually declined.

Reasoning

What would explain why the population of the freshwater species has declined? Suppose alligators prey
not only on the prized freshwater fish but also on another species of fish that is the main predator
of those fish. If there are fewer alligators to prey on the predator fish, there may well be more of
the predator fish. An increase in the population of the predator fish could help explain why the
population of the prized freshwater fish has declined: there are now more of the predator fish
around to prey on them.

A

B
C

D
E

Though this statement suggests that the population of the prized fish species may have declined due to
their being caught in greater numbers by fishers, the passage tells us that the number of these fish caught
for human consumption has not increased. Therefore, this statement cannot explain the species’ decline in
population.
The passage tells us that the number of fish caught for human consumption has not increased. This answer
choice does not provide any explanation for why the population of these fish has declined.
Correct. Since alligators prey on the predator fish, a decline in the alligator population could result in an
increase in the population of the predator fish, which could lead to an increase in nonhuman consumption
of the prized freshwater fish.
We already know from the passage that hunting has led to a decline in the population of alligators. It is
irrelevant what motivated the hunting of alligators.
The fact that alligators are at risk of extinction due to hunting in neighboring countries does not help
explain why the prized freshwater fish species is declining in population. If it is puzzling why the fish
population is declining despite the reduction in Parland’s alligator population, it would be just as puzzling
if the alligator population elsewhere was declining.

The correct answer is C.

561

GMAT® Official Guide 2018

559. A certain automaker aims to increase its market share by deeply discounting its vehicles’ prices for the next several
months. The discounts will cut into profits, but because they will be heavily advertised the manufacturer hopes that they
will attract buyers away from rival manufacturers’ cars. In the longer term, the automaker envisions that customers initially
attracted by the discounts may become loyal customers.
In assessing the plan’s chances of achieving its aim, it would be most useful to know which of the following?
(A)

Whether the automaker’s competitors are likely to respond by offering deep discounts on their own products

(B)

Whether the advertisements will be created by the manufacturer’s current advertising agency

(C)

Whether some of the automaker’s models will be more deeply discounted than others

(D)

Whether the automaker will be able to cut costs sufficiently to maintain profit margins even when the discounts are in
effect

(E)

Whether an alternative strategy might enable the automaker to enhance its profitability while holding a constant or
diminishing share of the market

Evaluation of a Plan
Situation

An automaker is planning to offer deep discounts on its vehicles’ prices in order to increase its
market share. The automaker’s profit margins will be reduced by this action. By advertising the
discounts, the automaker hopes to attract customers who might otherwise be inclined to buy rival
manufacturers’ cars. These customers would ideally then develop loyalty to the automaker’s cars.

Reasoning

What would it be most useful to know in assessing whether offering deep discounts will enable the
automaker to increase its market share? To achieve an increase in market share, the automaker would
have to take customers away from other automakers. Under what circumstances would other
automakers be able to retain their customers, if those customers are more likely to purchase cars
from automakers that offer deep discounts (and then remain loyal to those automakers)? The other
automakers might try to retain their customers by matching the discounts. Thus it would be useful
to know whether the other automakers would indeed offer such discounts.

A
B
C

D

E

Correct. If the answer to this question were yes, the plan would probably not achieve its aim of increasing
market share. If the answer were no, the plan would have a good chance of succeeding.
Since there is no information about the effectiveness of the automaker’s current advertising, it would not
be useful to know whether the same advertising agency will produce the ads publicizing the discount.
Knowing whether some models will be more deeply discounted than others might help in assessing
which of the automaker’s models will sell best, but it would not help in assessing the overall chance of the
automaker increasing its market share.
The discounts the automaker plans to offer will cut into profits, according to the information given, so
the question of whether the automaker can maintain profit margins while the discounts are in effect has
already been answered.
While it might be useful to the automaker to know about alternative strategies, such knowledge does not
help in assessing the likelihood that the plan under discussion will achieve its aim.

The correct answer is A.

562

8.6 Critical Reasoning Answer Explanations

560. In virtually any industry, technological improvements increase labor productivity, which is the output of goods and services
per person‐hour worked. In Parland’s industries, labor productivity is significantly higher than it is in Vergia’s industries.
Clearly, therefore, Parland’s industries must, on the whole, be further advanced technologically than Vergia’s are.
The argument is most vulnerable to which of the following criticisms?
(A)

It offers a conclusion that is no more than a paraphrase of one of the pieces of information provided in its support.

(B)

It presents as evidence in support of a claim information that is inconsistent with other evidence presented in support
of the same claim.

(C)

It takes one possible cause of a condition to be the actual cause of that condition without considering any other
possible causes.

(D)

It takes a condition to be the effect of something that happened only after the condition already existed.

(E)

It makes a distinction that presupposes the truth of the conclusion that is to be established.

Argument Evaluation
Situation

Technological improvements in nearly every industry increase labor productivity, which is the
output of goods and services per person‐hour worked. Because labor productivity is significantly
higher in Parland than Vergia, Parland’s industries are, in general, more technologically advanced
than Vergia’s.

Reasoning

To which criticism is the argument most vulnerable? Though one factor, such as technological
advancements, may lead to greater labor productivity, it may not be the only such factor, or even
a necessary factor, leading to great labor productivity. Therefore, the mere fact that one region’s
labor is more productive than another’s is not sufficient to establish that the former region is more
technologically advanced than the latter region is.

A
B
C

D
E

The conclusion is not merely a paraphrase of the pieces of information provided in its support. Indeed, the
problem with the argument is that the conclusion goes too far beyond what the premises merit.
The premises of the argument are not inconsistent with one another.
Correct. This accurately describes the flaw in the argument because the reasons given in the argument
for its conclusion would be good reasons only if there were no other plausible explanations for Parland’s
greater labor productivity.
The argument does not mention how long Parland has had more productive labor, or when technological
improvements would have occurred.
Neither of the premises contains anything that presupposes the conclusion to be true.

The correct answer is C.

563

GMAT® Official Guide 2018

561. Chaco Canyon, a settlement of the ancient Anasazi culture in North America, had massive buildings. It must have been a
major Anasazi center. Analysis of wood samples shows that some of the timber for the buildings came from the Chuska
and San Mateo mountains, 50 miles from Chaco Canyon. Only a major cultural center would have the organizational
power to import timber from 50 miles away.
In the argument given, the two portions in boldface play which of the following roles?
(A)

The first is a premise used to support the argument’s main conclusion; the second is the argument’s main conclusion.

(B)

The first is the argument’s main conclusion; the second is a premise used to support that conclusion.

(C)

The first is one of two premises used to support the argument’s main conclusion; the second is the other of those two
premises.

(D)

The first is a premise used to support the argument’s main conclusion; the second is a premise used to support
another conclusion drawn in the argument.

(E)

The first is inferred from another statement in the argument; the second is inferred from the first.

Argument Construction
Situation

The ancient Anasazi settlement at Chaco Canyon had massive buildings, for which some of the
timber came from mountains 50 miles away.

Reasoning

What roles do the statement that Chaco Canyon must have been a major Anasazi center and the statement
that only a major center would have the organizational power to import timber from 50 miles away play
in the argument? The first and third sentences in the passage are both factual observations. Since
no further support is provided for either of them, neither can be a conclusion in the argument.
The fourth sentence is a speculative generalization about major cultural centers. None of the other
statements gives us any reason to think this generalization is true, so it cannot be a conclusion in
the argument, either. However, the third and fourth sentences together imply that Chaco Canyon
was a major cultural center, and the first sentence indicates that it was Anasazi. So together, the
first, third, and fourth sentences all support the claim that Chaco Canyon was a major Anasazi
cultural center and thus more generally a major Anasazi center, as the second sentence asserts.
Therefore, the first, third, and fourth sentences are all premises that jointly support the second
sentence as a conclusion.

A

As explained above, the first boldface sentence is a conclusion supported by the latter, not the other way
around.
B Correct. As explained above, the first boldface sentence is the argument’s only stated conclusion, and in
that sense its main conclusion, while all the other sentences are premises used to support it.
C As explained above, the first boldface sentence is the argument’s conclusion, not a premise used to support
the conclusion.
D As explained above, the first boldface sentence is the argument’s only stated conclusion, and there is no
reason to suppose that the argument is intended to lead to any other tacit conclusion that the second
boldface sentence is intended to support.
E As explained above, the first boldface sentence is inferred from the three other statements in the argument
together, not from any one of them alone. The second boldface sentence is a speculative generalization that
cannot be, and is not meant to be, inferred from the former or from any other statement in the argument.
The correct answer is B.

564

8.6 Critical Reasoning Answer Explanations

562. The Maxilux car company’s design for its new luxury model, the Max 100, included a special design for the tires that was
intended to complement the model’s image. The winning bid for supplying these tires was submitted by Rubco. Analysts
concluded that the bid would only just cover Rubco’s costs on the tires, but Rubco executives claim that winning the bid will
actually make a profit for the company.
Which of the following, if true, most strongly justifies the claim made by Rubco’s executives?
(A)

In any Maxilux model, the spare tire is exactly the same make and model as the tires that are mounted on the wheels.

(B)

Rubco holds exclusive contracts to supply Maxilux with the tires for a number of other models made by Maxilux.

(C)

The production facilities for the Max 100 and those for the tires to be supplied by Rubco are located very near each
other.

(D)

When people who have purchased a carefully designed luxury automobile need to replace a worn part of it, they
almost invariably replace it with a part of exactly the same make and type.

(E)

When Maxilux awarded the tire contract to Rubco, the only criterion on which Rubco’s bid was clearly ahead of its
competitors’ bids was price.

Argument Construction
Situation

Rubco won a bid for supplying tires for the Max 100, a new luxury model by Maxilux. The bid
would barely cover the cost of the tires, but Rubco executives claim that winning the bid will be
profitable.

Reasoning

What would support the executives’ claim? Rubco is not expected to make a profit from supplying
the tires for the new cars, so we must look for some other way that Rubco could derive a profit
as a result of winning the bid. If by winning the bid Rubco created an inevitable market for itself
in replacement tires—on which Rubco could earn a profit—then the executives’ claim may be
justified.

A

B

C

D
E

We have already been told that the bid is expected to barely cover the costs of supplying the tires on the
new cars, so the analysts mentioned in the passage have presumably already taken into account that there
is a spare tire supplied for the Max 100.
If winning the bid led Rubco to win more exclusive contracts with the Maxilux, that might help support
the executives’ claim. But this statement indicates only that Rubco already has several exclusive contracts to
supply Maxilux with tires, not that winning the bid has led to, or will lead to, more such contracts, which is
what would be needed.
As in answer choice (A), this is relevant to the costs of supplying the tires for the Max 100, but presumably
this was taken into account by the analysts when they concluded that the bid would barely cover Rubco’s
costs on the tires.
Correct. This indicates that by winning the bid Rubco has created a way to profit from the contract with
Maxilux, specifically, by creating a market for replacement tires.
This is likely one of the reasons that Rubco’s bid only just covers Rubco’s costs on the tires; it does nothing
to justify the executives’ claims that the bid will lead to a profit for Rubco.

The correct answer is D.

565

GMAT® Official Guide 2018

563. Which of the following most logically completes the passage?
Most bicycle helmets provide good protection for the top and back of the head, but little or no protection for the temple
regions on the sides of the head. A study of head injuries resulting from bicycle accidents showed that a large proportion
were caused by blows to the temple area. Therefore, if bicycle helmets protected this area, the risk of serious head injury in
bicycle accidents would be greatly reduced, especially since ________.
(A)

among the bicyclists included in the study’s sample of head injuries, only a very small proportion had been wearing a
helmet at the time of their accident

(B)

even those bicyclists who regularly wear helmets have a poor understanding of the degree and kind of protection that
helmets afford

(C)

a helmet that included protection for the temples would have to be somewhat larger and heavier than current helmets

(D)

the bone in the temple area is relatively thin and impacts in that area are thus very likely to cause brain injury

(E)

bicyclists generally land on their arm or shoulder when they fall to the side, which reduces the likelihood of severe
impacts on the side of the head

Argument Construction
Situation

Bicycle helmets protect the top and back of the head, but not the sides or temples. A study found
that a large proportion of head injuries caused by biking accidents were caused by blows to the
temple area.

Reasoning

Why would the risk of serious head injury in bicycle accidents be greatly reduced if bicycle helmets protected
the temple regions? If for some reason a serious head injury is particularly likely when there is
impact to the temple area, then bicycle helmets that protect that area would be apt to reduce the
number of serious head injuries from bicycle accidents. One such reason is that the bone in the
temple area is relatively thin.

A
B

C

D
E

This point is irrelevant because it gives us no information about the seriousness or the likelihood of injuries
due to impact to the temple area.
Whether bicyclists who regularly wear helmets have a good understanding of what protection their
helmets afford is not relevant as to whether serious head injuries are particularly likely to occur from
impact to the temple area.
This point is relevant only to what a helmet that protected the temple area would be like, not to the
seriousness of injuries resulting from impact to that area. If anything, this point counts as a reason against
the conclusion, not for it. If such helmets are heavier and larger, they may be used less than they otherwise
would be. If fewer helmets are used, then improvements to helmet design will have less of an effect in
reducing serious head injuries.
Correct. This statement provides a reason why the temple area of the rider’s head needs protection:
impacts to this area are very likely to cause brain injuries.
This is largely irrelevant. Even if it suggests that head injuries do not generally result from bicyclists falling
to the side, it does not indicate that such injuries are rare or that there is not great risk of serious injury in
those cases in which there is impact to the temple area.

The correct answer is D.

566

8.6 Critical Reasoning Answer Explanations

564. Which of the following most logically completes the argument?
In a typical year, Innovair’s airplanes are involved in 35 collisions while parked or being towed in airports, with a resulting
yearly cost of $1,000,000 for repairs.
To reduce the frequency of ground collisions, Innovair will begin giving its ground crews additional training, at an annual cost
of $500,000. Although this will cut the number of ground collisions by about half at best, the drop in repair costs can be
expected to be much greater, since __________.
(A)

most ground collisions happen when ground crews are rushing to minimize the time a delayed airplane spends on the
ground

(B)

a ground collision typically occurs when there are no passengers on the airplane

(C)

the additional training will focus on helping ground crews avoid those kinds of ground collisions that cause the most
costly damage

(D)

the $500,000 cost figure for the additional training of ground crews includes the wages that those crews will earn
during the time spent in actual training

(E)

most ground collisions have been caused by the least experienced ground‐crew members

Evaluation of a Plan
Situation

An airline will give its ground crews additional training to reduce the frequency of the collisions
its airplanes are involved in while parked or being towed in airports.

Reasoning

What premise would most logically support the conclusion that the additional training will reduce repair
costs from ground collisions much more than it reduces the number of such collisions? The key word since
before the blank shows that the argument should be completed with a premise that supports the
preceding claim that the drop in repair costs can be expected to be much greater. A suitable premise
might provide evidence that the training will disproportionately help to prevent the ground
collisions that result in the higher repair costs as opposed to the less serious collisions that result in
lower repair costs.

A
B
C

D
E

We are given no reason to believe that the additional training would affect how much ground crews rush
to minimize delays.
The number of passengers is not clearly relevant to the repair costs resulting from a ground collision and in
any case would not be affected by additional ground crew training.
Correct. If the training especially helps the ground crews avoid those kinds of collisions that cause the
most costly damage, then it will probably reduce repair costs even more than it reduces the number of
collisions.
Whether the cited expense for training includes wages is irrelevant to whether the training will reduce
repair costs more than it reduces the number of collisions.
This suggests that the additional training may help reduce the number of collisions, not that it will reduce
repair costs more than it reduces the number of collisions.

The correct answer is C.

567

GMAT® Official Guide 2018

565. Hunter: Many people blame hunters alone for the decline in Greenrock National Forest’s deer population over
the past ten years. Yet clearly, black bears have also played an important role in this decline. In the past ten years, the
forest’s protected black bear population has risen sharply, and examination of black bears found dead in the forest during
the deer hunting season showed that a number of them had recently fed on deer.
In the hunter’s argument, the portion in boldface plays which of the following roles?
(A)

It is the main conclusion of the argument.

(B)

It is a finding that the argument seeks to explain.

(C)

It is an explanation that the argument concludes is correct.

(D)

It provides evidence in support of the main conclusion of the argument.

(E)

It introduces a judgment that the argument opposes.

Argument Construction
Situation

The hunter claims that hunters have been identified by many people as the sole cause of the
decline in Greenrock National Forest’s deer population. But the hunter argues that black bears
have also contributed to the deer population decline. Black bears are protected and have increased
in number, and they have been found to have fed recently on deer.

Reasoning

What role in the argument is played by the hunter’s statement that many people blame hunters alone for
the decline in the national forest’s deer population? In this statement, the hunter claims that many
people have judged hunters responsible for the decline. The hunter then goes on to offer evidence
supporting a different judgment: that hunters are not solely responsible, but that black bears are
also to blame.

A
B
C
D
E

The hunter’s main conclusion is that black bears have also contributed to the decline in the deer
population.
The argument seeks to offer a reason for the finding that the deer population has declined, not the finding
that people blame hunters for that decline.
The hunter does not conclude that blaming hunters for the decline in the deer population is correct; rather,
the hunter suggests that black bears should also be blamed.
The hunter believes that hunters are not solely responsible for the decline in the deer population, so
people’s suggestion that they are responsible does not support the hunter’s main conclusion.
Correct. The boldfaced statement cites a judgment that the hunter attributes to many people, and that
the hunter argues is incorrect. The hunter opposes the judgment that hunters alone are responsible for the
decline in the deer population.

The correct answer is E.

568

8.6 Critical Reasoning Answer Explanations

566. Many agriculturally intensive areas of the world are beginning to encounter water scarcity problems. As a result, many
farmers in these areas are likely to reduce their output as the water supply they need in order to maintain production
shrinks. However, one group of farmers in such a region plans to increase their production by implementing techniques for
water conservation.
Which of the following, if true, would most strongly support the prediction that the group’s plan will succeed?
(A)

Farmers who can gain a larger share of the food market in their regions will be better positioned to control more
water resources.

(B)

Most agricultural practices in areas with water shortages are water‐intensive.

(C)

Other regions of the world not facing water shortages are likely to make up for the reduction in agricultural output.

(D)

Demand for agricultural products in the group’s region is not expected to decline.

(E)

More than half the water used for agriculture in the farmers’ region is lost to evaporation or leakage from irrigation
channels.

Evaluation of a Plan
Situation

Farmers in many agriculturally intensive regions will probably reduce their output because the
regions’ water supplies are dwindling, but one group of farmers in such a region plans to use water
conservation techniques to increase their output.

Reasoning

What would provide evidence that water conservation techniques will help the farmers increase
production despite their region’s dwindling water supplies? In order for the water conservation
techniques to be effective, they must result in significantly more water becoming available for the
farmers to use. Because overall supplies are shrinking, rather than growing, that can only happen
if the farmers are currently losing or wasting a great deal of water in ways that could be prevented
with water conservation techniques.

A
B

C

D

E

This suggests an advantage the farmers will gain if their water conservation plan enables them to increase
production, but it provides no evidence that the plan actually will enable them to increase production.
This suggests that the plan would have to yield quite a lot of conserved water in order for the farmers to
increase production, but it offers no evidence that the plan will do so. Thus, it provides some reason to
question whether the plan will succeed.
Whether regions without water shortages will increase production is not directly relevant to the question
of whether a particular measure would lead to increased production in one region that does have a water
shortage.
This has some slight, indirect relevance to the question of whether the farmers’ plan will succeed: it
suggests that if the farmers do manage to increase production, they will continue to have a market for what
they produce. However, it does not address the issue of whether they will be able to increase production.
Furthermore, even if demand for agricultural products in the group’s region were expected to decline, it
could still remain high enough to support the farmers’ increased output from their water conservation plan.
Correct. This suggests that the farmers are losing a lot of water in ways that the water conservation
techniques might prevent, so it provides evidence that employing some such techniques could enable the
farmers to save enough water to increase their output.

The correct answer is E.

569

GMAT® Official Guide 2018

567. Physician: The hormone melatonin has shown promise as a medication for sleep disorders when taken in synthesized form.
Because the long‐term side effects of synthetic melatonin are unknown, however, I cannot recommend its use at this time.
Patient: Your position is inconsistent with your usual practice. You prescribe many medications that you know have serious
side effects, so concern about side effects cannot be the real reason you will not prescribe melatonin.
The patient’s argument is flawed because it fails to consider that
(A)

the side effects of synthetic melatonin might be different from those of naturally produced melatonin

(B)

it is possible that the physician does not believe that melatonin has been conclusively shown to be effective

(C)

sleep disorders, if left untreated, might lead to serious medical complications

(D)

the side effects of a medication can take some time to manifest themselves

(E)

known risks can be weighed against known benefits, but unknown risks cannot

Argument Evaluation
Situation

The physician refuses to prescribe synthetic melatonin to treat sleep disorders despite this
medication’s promise. The reason the physician offers for this refusal is that the long‐term side
effects of synthetic melatonin are unknown. The patient responds that because the physician
prescribes other medications that are known to have serious side effects, it cannot be a concern
for synthetic melatonin’s side effects that is prompting the physician’s refusal to prescribe that
medication.

Reasoning

What does the patient’s argument fail to consider? The patient says that the inconsistency in the
physician’s position lies in the physician’s unwillingness to prescribe synthetic melatonin coupled
with a willingness to prescribe other medications that are known to have serious side effects. But
notice that the physician does not say that synthetic melatonin has serious side effects; rather, the
physician points out that the long‐term side effects of synthetic melatonin are unknown. The
physician most likely prescribes medications that have serious side effects because the medications’
benefits outweigh the risks posed by their side effects. In the case of synthetic melatonin, however,
this kind of decision cannot be made.

A

B

C

D
E

The patient’s argument has to do with whether the physician’s refusal to prescribe synthetic melatonin is
consistent with the physician’s usual prescription practices. The question of whether naturally produced
melatonin has different side effects than synthetic melatonin has no bearing on that argument.
It is quite reasonable for the patient’s argument not to mention this possibility, especially since the
physician expresses a belief that synthetic melatonin may be effective—but expresses no belief about
whether or not it has been conclusively shown to be effective.
Awareness that sleep disorders can lead to serious medical complications most likely prompts the patient’s
desire for treatment—but the patient’s not mentioning this possible consequence of sleep disorders does
not indicate a flaw in the argument.
The patient makes clear that the physician prescribes medications that have serious side effects; the time
those side effects take to manifest themselves is not relevant to the argument.
Correct. The patient’s argument is flawed in failing to consider this key difference between known risks
and unknown risks. If the patient had considered this key difference, the patient would have realized that
the physician’s position is not at all inconsistent, and that the physician’s refusal to prescribe is genuinely
based on a concern about an unknown risk.

The correct answer is E.

570

8.6 Critical Reasoning Answer Explanations

568. Hollywood restaurant is replacing some of its standard tables with tall tables and stools. The restaurant already fills every
available seat during its operating hours, and the change in seating arrangements will not result in an increase in the
restaurant’s seating capacity. Nonetheless, the restaurant’s management expects revenue to increase as a result of the
seating change without any concurrent change in menu, prices, or operating hours.
Which of the following, if true, provides the best reason for the expectation?
(A)

One of the taller tables takes up less floor space than one of the standard tables.

(B)

Diners seated on stools typically do not linger over dinner as long as diners seated at standard tables.

(C)

Since the restaurant will replace only some of its standard tables, it can continue to accommodate customers who do
not care for the taller tables.

(D)

Few diners are likely to avoid the restaurant because of the new seating arrangement.

(E)

The standard tables being replaced by tall tables would otherwise have to be replaced with new standard tables at a
greater expense.

Argument Construction
Situation

Hollywood restaurant is replacing some of its tables with taller tables and stools, and the
management expects this will increase revenue, despite the fact that the restaurant already fills all
of its available seats and that this change will not increase seating capacity. Furthermore, there will
not be any change in menu, prices, or operating hours.

Reasoning

What would strongly support the management’s expectation? Since the new seating will not increase
the restaurant’s seating capacity, the management’s expectations must be based on a belief that the
change to taller tables and stools will somehow change diners’ behavior, perhaps by leading them
to order more food, or to stay at their tables for a shorter time, thereby allowing the restaurant to
serve more diners during its operating hours without increasing seating capacity. If diners seated
at tall tables and on tall stools spend less time lingering over their dinners, then they will leave
sooner, opening up the tables for more diners. Because the restaurant, before the change, already
fills every available seat during its operating hours, it is reasonable to think that it will be able to
serve more diners than it currently does, thereby selling more food and thus increasing revenue.

A
B
C

D
E

This would be relevant if we could infer from it that seating capacity will increase. However, the passage
indicates that the new seating arrangement will not result in greater capacity.
Correct. Because the restaurant will be able to serve more meals during its operating hours, the
restaurant’s revenue can be expected to increase.
This may indicate that the restaurant is less likely to alienate customers who do not care for tall tables
and stools, but that only supports the claim that the restaurant will not lose customers and therefore lose
revenue; it does not indicate that the restaurant will see revenue increase.
Again, this merely indicates that there will not be a loss—or much loss—of revenue, not that there will be
an increase in revenue.
Less expensive tables will decrease the restaurant’s costs, but it will not increase the restaurant’s revenue.

The correct answer is B.

571

GMAT® Official Guide 2018

569. A major network news organization experienced a drop in viewership in the week following the airing of a controversial
report on the economy. The network also received a very large number of complaints regarding the report. The network,
however, maintains that negative reactions to the report had nothing to do with its loss of viewers.
Which of the following, if true, most strongly supports the network’s position?
(A)

The other major network news organizations reported similar reductions in viewership during the same week.

(B)

The viewers who registered complaints with the network were regular viewers of the news organization’s programs.

(C)

Major network news organizations publicly attribute drops in viewership to their own reports only when they receive
complaints about those reports.

(D)

This was not the first time that this network news organization has aired a controversial report on the economy that
has inspired viewers to complain to the network.

(E)

Most network news viewers rely on network news broadcasts as their primary source of information regarding the
economy.

Argument Construction
Situation

A major network news organization aired a controversial report on the economy, and the following
week the network’s viewership declined. The network claims that the loss of viewers was not
connected with negative reactions to the report.

Reasoning

Which statement most strongly supports the network’s position? If other major news network
organizations had similar drops in viewership, it is implausible to think that the controversial
report accounted for the other organizations’ drops in viewership. On the other hand, it is not
implausible to suppose that whatever did cause the drop in the viewership experienced by other
network news organizations—e.g., holidays, weather, popular non‐news programming—also had
that effect on the organization that ran the controversial report. This would give some reason to
believe that it was not the report that accounts for the organization’s drop in viewership.

A
B
C
D

E

Correct. This statement indicates that something other than the airing of the report could account for the
subsequent drop in the organization’s viewership.
If anything, this statement tends to undermine the network’s claim, because it suggests that the report
offended people who otherwise might have continued to watch the organization’s programming.
Since the network did in fact receive complaints about the report, this statement is irrelevant.
The fact that the network has received complaints before about controversial reports on the economy
that the network’s news organization has aired tells us nothing about whether this recent report caused a
subsequent drop in viewership.
The fact that viewers turn to network news broadcasts as their primary source of information about
the economy tells us nothing about whether viewers might stop watching a particular network news
organization’s programs as a result of its airing a controversial report on the economy.

The correct answer is A.

572

8.6 Critical Reasoning Answer Explanations

570. Only a reduction of 10 percent in the number of scheduled flights using Greentown’s airport will allow the delays that are so
common there to be avoided. Hevelia airstrip, 40 miles away, would, if upgraded and expanded, be an attractive alternative
for fully 20 percent of the passengers using Greentown airport. Nevertheless, experts reject the claim that turning Hevelia
into a full‐service airport would end the chronic delays at Greentown.
Which of the following, if true, most helps to justify the experts’ position?
(A)

Turning Hevelia into a full‐service airport would require not only substantial construction at the airport itself, but also
the construction of new access highways.

(B)

A second largely undeveloped airstrip close to Greentown airport would be a more attractive alternative than Hevelia
for many passengers who now use Greentown.

(C)

Hevelia airstrip lies in a relatively undeveloped area but would, if it became a full‐service airport, be a magnet for
commercial and residential development.

(D)

If an airplane has to wait to land, the extra jet fuel required adds significantly to the airline’s costs.

(E)

Several airlines use Greentown as a regional hub, so that most flights landing at Greentown have many passengers
who then take different flights to reach their final destinations.

Evaluation of a Plan
Situation

To avoid the delays now common at Greentown’s airport, the number of scheduled flights
there would need to be reduced by 10 percent. If the nearby Hevelia airstrip were expanded and
upgraded, it would be an attractive alternative for 20 percent of Greentown airport’s passengers.
Still, experts do not believe that the delays at Greentown would end even if Hevelia were turned
into a full‐service airport.

Reasoning

Which statement most supports the experts’ position? If the number of flights at Greentown’s airport
did not drop by at least 10 percent, despite the fact that 20 percent of the passengers who currently
use Greentown’s airport would find nearby Hevelia airstrip an attractive alternative, then the
delays would not be avoided. Airlines generally use certain airports as regional hubs—an airport
through which an airline routes most of its traffic—so, even if many passengers would be willing
to use Hevelia airstrip, the number of flights at Greentown may not decline significantly, or at all.

A

B

C
D

E

The experts’ position concerns what would happen to the flight delays at Greentown airport if the Hevelia
airstrip were converted into a full‐service airport. So the fact that there are great costs involved in making
such a conversion—possibly making such a conversion unlikely—has no bearing on the effects such a
conversion would have on flight delays at Greentown if the conversion were to be carried out.
This statement indicates that the undeveloped airstrip near Greentown might be a better way to alleviate
flight delays at Greentown, but it tells us nothing about the effects that converting the Hevelia airstrip to a
full‐service airport would have were it to be carried out.
This in no way explains why converting the Hevelia airstrip into a full‐service airport would not alleviate
the problem with flight delays at Greentown.
This provides a reason to think that reducing the number of flights at Greentown might make the airport
more efficient. But that has no bearing on the effect that converting the Hevelia airstrip to a full‐service
airport might have on flight delays at Greentown.
Correct. This statement provides support for the experts’ position because it gives a reason for thinking
that the number of scheduled flights at Greentown would not be reduced, even if Hevelia airstrip became
an attractive alternative for some 20 percent of Greentown’s passengers.

The correct answer is E.

573

GMAT® Official Guide 2018

571. For similar cars and comparable drivers, automobile insurance for collision damage has always cost more in Greatport than
in Fairmont. Police studies, however, show that cars owned by Greatport residents are, on average, slightly less likely to be
involved in a collision than cars in Fairmont. Clearly, therefore, insurance companies are making a greater profit on collision‐
damage insurance in Greatport than in Fairmont.
In evaluating the argument, it would be most useful to compare
(A)

the level of traffic congestion in Greatport with the level of traffic congestion in Fairmont

(B)

the cost of repairing collision damage in Greatport with the cost of repairing collision damage in Fairmont

(C)

the rates Greatport residents pay for other forms of insurance with the rates paid for similar insurance by residents of
Fairmont

(D)

the condition of Greatport’s roads and streets with the condition of Fairmont’s roads and streets

(E)

the cost of collision‐damage insurance in Greatport and Fairmont with that in other cities

Argument Evaluation
Situation

A particular kind of insurance, that for collision damage, costs more in Greatport than in
Fairmont. The cars of Greatport residents are, however, less likely to be involved in collisions
than are cars of Fairmont residents. So insurance companies must be making a greater profit on
collision‐damage insurance in Greatport than in Fairmont.

Reasoning

What would it help to consider in evaluating the argument? Insurance companies would make greater
profits on collision‐damage insurance in Greatport than they make in Fairmont if they pay out
less money in response to Greatport residents’ claims than they do in response to Fairmont’s
residents’ claims. That Greatport residents’ cars are involved in fewer collisions than are Fairmont’s
residents’ cars supports this—if there are fewer collisions overall, then the insurance companies
might pay out less money overall. But the number of collisions is only one factor contributing
to how much money an insurance company pays out in response to claims; another factor is the
amount of damage inflicted on the cars involved in collisions and how much it costs to repair that
damage. These costs would need to be considered before concluding that insurance companies’
profits on collision‐damage insurance are greater in Greatport than in Fairmont.

A

B
C
D

E

The level of traffic congestion probably contributes to the frequency of collisions in each town. The
information given, however, includes the statement that Greatport cars are less likely to be involved
in collisions than are Fairmont cars. Why this occurs—whether, for example, traffic congestion is a
contributory factor—is not relevant.
Correct. This is clearly a factor that would affect the profitability of insurance in the two towns—and is
therefore highly relevant to evaluating the argument, especially its conclusion.
The argument’s conclusion is about insurance companies’ profits on collision‐damage insurance alone, so
other types of insurance, and the rates paid for them, are not relevant.
The condition of the roads and streets in each town probably contributes to the frequency of collisions in
each town. The information given, however, includes the statement that Greatport cars are less likely to be
involved in collisions than are Fairmont cars. Why this is so—whether, for example, the condition of the
roads is a contributory cause—is not relevant.
Since the argument is concerned solely with collision‐insurance costs and profits in Greatport and
Fairmont, comparing the cost of insurance in those towns with the cost of insurance elsewhere would
provide no useful insight.

The correct answer is B.

574

8.6 Critical Reasoning Answer Explanations

572. The air quality board recently informed Coffee Roast, a small coffee roasting firm, of a complaint regarding the smoke
from its roaster. Recently enacted air quality regulations require machines roasting more than 10 pounds of coffee to be
equipped with expensive smoke‐dissipating afterburners. The firm, however, roasts only 8 pounds of coffee at a time.
Nevertheless, the company has decided to purchase and install an afterburner.
Which of the following, if true, most strongly supports the firm’s decision?
(A)

Until settling on the new air quality regulations, the board had debated whether to require afterburners for machines
roasting more than 5 pounds of coffee at a time.

(B)

Coffee roasted in a machine equipped with an afterburner has its flavor subtly altered.

(C)

The cost to the firm of an afterburner is less than the cost of replacing its roaster with a smaller one.

(D)

Fewer complaints are reported in areas that maintain strict rules regarding afterburners.

(E)

The firm has reason to fear that negative publicity regarding the complaints could result in lost sales.

Evaluation of a Plan
Situation

After being informed of a complaint about smoke from its coffee roaster, a firm decided to
purchase and install an afterburner to reduce or eliminate emissions of smoke, even though the
roaster roasts too little coffee at a time for an afterburner to be legally required.

Reasoning

What would have been a good reason for the firm to buy and install the afterburner? The only factors
mentioned that might give the firm reason to buy an afterburner are the complaint about smoke
and the regulations requiring an afterburner. Since the regulations do not apply in this case, the
complaint is more likely to have motivated the firm’s decision. Any serious potential consequences
the firm might have faced from failure to address the complaint could have provided a good reason
to buy and install the afterburner.

A

B
C
D

E

If this debate had still been ongoing when the firm made its decision, uncertainty about the pending
regulations might have justified the decision. But the debate had already been settled before the firm
decided to purchase the afterburner, and the regulations clearly did not require one.
An unspecified alteration in flavor is not clearly a good reason to use an afterburner—the afterburner
might worsen the flavor.
The firm’s roaster was already small enough that the regulations did not require it to be replaced, even
without an afterburner.
This reason relates only to rules regarding afterburners, not to Coffee Roast’s purchase of an afterburner,
which was not mandated by regulations. Furthermore, it could be that the air quality regulations recently
enacted are among the strictest in any region, which could result in fewer complaints regardless of whether
Coffee Roast installs an afterburner.
Correct. Since installing an afterburner is a plausible way to address the complaint and prevent future
complaints, the firm has plausible reasons to believe this strategy will help it avoid the negative publicity
and lost sales it fears. These considerations could have reasonably justified its decision.

The correct answer is E.

575

GMAT® Official Guide 2018

573. The tulu, a popular ornamental plant, does not reproduce naturally, and is only bred and sold by specialized horticultural
companies. Unfortunately, the tulu is easily devastated by a contagious fungal rot. The government ministry plans to
reassure worried gardeners by requiring all tulu plants to be tested for fungal rot before being sold. However, infected
plants less than 30 weeks old have generally not built up enough fungal rot in their systems to be detected reliably. And
many tulu plants are sold before they are 24 weeks old.
Which of the following, if performed by the government ministry, could logically be expected to overcome the problem with
their plan to test for the fungal rot?
(A)

Releasing a general announcement that tulu plants less than 30 weeks old cannot be effectively tested for fungal rot

(B)

Requiring all tulu plants less than 30 weeks old to be labeled as such

(C)

Researching possible ways to test tulu plants less than 24 weeks old for fungal rot

(D)

Ensuring that tulu plants not be sold before they are 30 weeks old

(E)

Quarantining all tulu plants from horticultural companies at which any case of fungal rot has been detected until those
tulu plants can be tested for fungal rot

Evaluation of a Plan
Situation

There is a contagious fungal rot that devastates the tulu, a popular ornamental plant. To reassure
worried gardeners, the government ministry plans to require that tulu plants be tested for the rot
before being sold. However, many tulu plants are sold before they are 24 weeks old, yet fungal rot
in plants less than 30 weeks old generally cannot be detected reliably.

Reasoning

What could the government ministry do to overcome the problem? The problem arises from the fact
that tulu plants are frequently sold before they are 24 weeks old, which is too soon for any fungal
rot that is present to have built up enough in their root systems to be detected. Since the goal
of the testing is to ensure that infected tulu plants not be sold, an obvious solution would be to
make sure that no plants are sold before they are old enough for fungal rot to have built up to a
detectable level. Thus, tulu plants should not be sold before they are 30 weeks old.

A

B
C
D
E

Releasing such an announcement would help overcome the problem if it guaranteed that no one
would buy or sell tulu plants before the plants were 30 weeks old, but it is far from certain that such an
announcement would guarantee this.
Since some people may not be aware of the significance of such labeling, such labeling might have very
little effect.
There is no guarantee that such research will be successful at reducing the age at which tulu plants can be
reliably tested.
Correct. If the government ensures that no tulu plants less than 30 weeks of age are sold, then the specific
problem mentioned in the passage would be overcome.
This will not help overcome the problem. Such a quarantine program might lead horticultural companies
to start selling tulu plants only if they are less than 24 weeks old, thereby minimizing the chance of
quarantine by minimizing the chance of detection.

The correct answer is D.

576

8.6 Critical Reasoning Answer Explanations

574. People who do regular volunteer work tend to live longer, on average, than people who do not. It has been found that “doing
good,” a category that certainly includes volunteer work, releases endorphins, the brain’s natural opiates, which induce
in people a feeling of well‐being. Clearly, there is a connection: Regular releases of endorphins must in some way help to
extend people’s lives.
Which of the following, if true, most seriously undermines the force of the evidence given as support for the hypothesis that
endorphins promote longevity?
(A)

People who do regular volunteer work are only somewhat more likely than others to characterize the work they do for
a living as “doing good.”

(B)

Although extremely high levels of endorphins could be harmful to health, such levels are never reached as a result of
the natural release of endorphins.

(C)

There are many people who have done some volunteer work but who do not do such work regularly.

(D)

People tend not to become involved in regular volunteer work unless they are healthy and energetic to begin with.

(E)

Releases of endorphins are responsible for the sense of well‐being experienced by many long‐distance runners while
running.

Argument Evaluation
Situation

People who volunteer regularly live longer on average than people who do not. Doing good work,
including volunteer work, releases endorphins, which induce a feeling of well‐being.

Reasoning

What additional findings would suggest that the cited evidence does not indicate that endorphins increase
longevity? The argument implicitly assumes that the reason regular volunteers tend to live longer
is that volunteering lengthens their lives. It further assumes that no factor that is correlated with
volunteering, other than the endorphin release, would plausibly explain how volunteering could
have this effect. Findings that cast doubt on either of these assumptions would undermine the
connection between the cited evidence and the conclusion that endorphins promote longevity.

A
B
C

D
E

Volunteering might greatly boost volunteers’ endorphin levels even if the work the volunteers do for a
living is no different from other people’s work.
Even if unnaturally high endorphin levels could harm health, the levels attainable through volunteer work
may promote health.
The argument is about an observed correlation in a certain group of people (those who regularly do
volunteer work). How many people are outside that group (i.e., do not regularly do volunteer work) is
independent of the question of what causes the observed correlation. Even if some people volunteer only
occasionally, volunteering regularly may promote longevity by causing regular releases of endorphins.
Correct. This suggests that the initially better health of people who choose to volunteer could fully explain
the cited correlation between volunteering and longevity.
Unless we are also given evidence that long‐distance runners tend not to live longer than other people,
this does not undermine the purported evidence in the argument. Endorphins might promote longevity in
both regular volunteers and long‐distance runners.

The correct answer is D.

577

GMAT® Official Guide 2018

575. In Mernia commercial fossil hunters often sell important fossils they have found, not to universities or museums, but to
individual collectors, who pay much better but generally do not allow researchers access to their collections. To increase
the number of fossils available for research, some legislators propose requiring all fossils that are found in Mernia to be
sold only to universities or museums.
Which of the following, if true, most strongly indicates that the legislators’ proposal will fail to achieve its goal?
(A)

Some fossil hunters in Mernia are not commercial fossil hunters, but rather are amateurs who keep the fossils that
they find.

(B)

Most fossils found in Mernia are common types that have little scientific interest.

(C)

Commercial fossil hunters in Mernia currently sell some of the fossils they find to universities and museums.

(D)

Many universities in Mernia do not engage in fossil research.

(E)

Most fossils are found by commercial fossil hunters, and they would give up looking for fossils if they were no longer
allowed to sell to individual collectors.

Evaluation of a Plan
Situation

Fossil hunters in Mernia often sell important fossils to collectors who do not make them
accessible to researchers. To increase the number of fossils available for research, some legislators
propose requiring all fossils found in Mernia to be sold only to universities or museums.

Reasoning

What would most strongly suggest that requiring all fossils found in Mernia to be sold only to universities
or museums would not increase the number of fossils available for research? To increase the number of
fossils available for research, the proposed requirement will have to be implemented and effectively
enforced. It will presumably have to increase the total number of fossils sold to universities and
museums. And those institutions will have to make more of the fossils in their collections available
to researchers than the private collectors do. Evidence that any of those conditions will not be
fulfilled would suggest that the legislators’ proposal will fail to achieve its goal.

A

B
C
D

E

Even if the legislation does not affect fossils kept by amateurs, it might still result in many more fossils
being sold to universities or museums rather than to private collectors, and thus might still increase the
number of fossils available for research.
Even if few Mernian fossils are interesting to researchers, the legislation could still achieve its goal of
making more fossils available for research.
Even if commercial fossil hunters already sell a few fossils to universities and museums, the legislation
could encourage them to sell many more fossils.
The universities that do not engage in fossil research presumably will not be interested in buying fossils
even if the legislation passes. But the fossil hunters can just sell their fossils to other universities and
museums that do engage in fossil research.
Correct. This suggests that if the legislation passes, fossils will simply be left in the ground rather than
sold to private collectors. That would not increase the total number of fossils available for research.

The correct answer is E.

578

8.6 Critical Reasoning Answer Explanations

576. The Eurasian ruffe, a fish species inadvertently introduced into North America’s Great Lakes in recent years, feeds on the
eggs of lake whitefish, a native species, thus threatening the lakes’ natural ecosystem. To help track the ruffe’s spread,
government agencies have produced wallet‐sized cards about the ruffe. The cards contain pictures of the ruffe and explain
the danger they pose; the cards also request anglers to report any ruffe they catch.
Which of the following, if true, would provide most support for the prediction that the agencies’ action will have its intended
effect?
(A)

The ruffe has spiny fins that make it unattractive as prey.

(B)

Ruffe generally feed at night, but most recreational fishing on the Great Lakes is done during daytime hours.

(C)

Most people who fish recreationally on the Great Lakes are interested in the preservation of the lake whitefish because
it is a highly prized game fish.

(D)

The ruffe is one of several nonnative species in the Great Lakes whose existence threatens the survival of lake
whitefish populations there.

(E)

The bait that most people use when fishing for whitefish on the Great Lakes is not attractive to ruffe.

Evaluation of a Plan
Situation

The Eurasian ruffe, a species not native to the Great Lakes, is threatening the native lake whitefish.
Government agencies hope that wallet‐sized cards identifying the ruffe, explaining the danger
they pose, and asking anglers to report their ruffe catches will help them track the ruffe’s spread.

Reasoning

What point would support the idea that the agencies’ action will have its intended effect? The cards are
intended to help government agencies track the ruffe’s spread. They will be useful for this purpose
only if anglers actually report the ruffe they catch. Thus anything that increases the odds of anglers’
doing such reporting would make it more likely that the cards will have their intended effect.

A
B

C

D

E

If ruffe are unattractive as prey, they will probably spread more quickly in the Great Lakes. This will most
likely have little effect on whether the wallet‐sized cards will help government agencies track the ruffe.
If ruffe feed at night, while fishing is done in the daytime, it is unlikely that anglers would catch ruffe. Thus
few catches would be reported to government agencies, making it more difficult for those agencies to track
the spread of ruffe.
Correct. This statement properly identifies a point that supports the prediction that the agencies’ action
will have its intended effect—that is, those who are interested in preserving the lake whitefish will be likely
to report catches of ruffe, which threaten whitefish, thus enabling the agencies’ tracking of the spread of
ruffe.
That the ruffe is one of several nonnative species threatening the Great Lakes lessens the odds that the
whitefish will survive, but this has no effect on the question of whether the wallet‐sized cards will help
government agencies track the ruffe’s spread.
This would make it likely that anglers would catch few ruffe. If anglers do not catch many ruffe, there
will not be many to report to government agencies, which would in turn make it more difficult for those
agencies to track the ruffe’s spread.

The correct answer is C.

579

GMAT® Official Guide 2018

577. Which of the following most logically completes the argument below?
Although the number of large artificial satellites orbiting the Earth is small compared to the number of small pieces of debris in
orbit, the large satellites interfere more seriously with telescope observations because of the strong reflections they produce.
Because many of those large satellites have ceased to function, the proposal has recently been made to eliminate interference
from nonfunctioning satellites by exploding them in space. This proposal, however, is ill conceived, since ________.
(A)

many nonfunctioning satellites remain in orbit for years

(B)

for satellites that have ceased to function, repairing them while they are in orbit would be prohibitively expensive

(C)

there are no known previous instances of satellites’ having been exploded on purpose

(D)

the only way to make telescope observations without any interference from debris in orbit is to use telescopes
launched into extremely high orbits around the Earth

(E)

a greatly increased number of small particles in Earth’s orbit would result in a blanket of reflections that would make
certain valuable telescope observations impossible

Evaluation of a Plan
Situation

Many large artificial satellites orbiting Earth no longer function. Reflections from these satellites
interfere with telescope observations. A proposal has been made to eliminate this interference by
exploding these satellites in space.

Reasoning

Why is the proposal ill conceived? If exploding the large artificial satellites—thereby creating a large
amount of debris—would result in an increase in interference with telescope observation, the
proposal would be self‐defeating, and therefore would be ill conceived.

A

B

C

D
E

The fact that many nonfunctioning satellites remain in orbit for years would seem to make the proposal
more attractive. If large nonfunctioning artificial satellites generally do not remain in orbit for years—e.g.,
if they fall out of orbit—then there might be no need to explode the satellites. Therefore this statement is
not correct.
This statement, too, makes the proposal more attractive—the opposite of what is called for. If
nonfunctioning satellites cannot be repaired, then that eliminates one possible reason for not blowing
them up.
The fact that there are no known instances of intentional explosions of satellites does not show that the
plan is ill conceived. At most this might suggest that the consequences of such an explosion are not well
understood, but even that is not very strongly suggested. For instance, if large artificial satellites have
accidentally blown up, scientists may have studied the results and have good reason to believe that no
ill effects will result from exploding the satellites.
This is not a cogent reason for thinking the proposal to be ill conceived. Even if the proposal would not
eliminate all interference with ground based telescopes, it might still greatly reduce interference.
Correct. If exploding large nonfunctioning satellites would lead to more interference, then we have a
reason to think that the proposal is ill conceived.

The correct answer is E.

580

8.6 Critical Reasoning Answer Explanations

578. Thyrian lawmaker: Thyria’s Cheese Importation Board inspects all cheese shipments to Thyria and rejects shipments not
meeting specified standards. Yet only 1 percent is ever rejected. Therefore, since the health consequences and associated
economic costs of not rejecting that 1 percent are negligible, whereas the board’s operating costs are considerable, for
economic reasons alone the board should be disbanded.
Consultant: I disagree. The threat of having their shipments rejected deters many cheese exporters from shipping
substandard product.
The consultant responds to the lawmaker’s argument by
(A)

rejecting the lawmaker’s argument while proposing that the standards according to which the board inspects imported
cheese should be raised

(B)

providing evidence that the lawmaker’s argument has significantly overestimated the cost of maintaining the board

(C)

objecting to the lawmaker’s introducing into the discussion factors that are not strictly economic

(D)

pointing out a benefit of maintaining the board, which the lawmaker’s argument has failed to consider

(E)

shifting the discussion from the argument at hand to an attack on the integrity of the cheese inspectors

Argument Construction
Situation

The Thyrian lawmaker argues that the Cheese Importation Board should be disbanded, because
its operating costs are high and it rejects only a small percentage of the cheese it inspects. The
consultant disagrees, pointing out that the board’s inspections deter those who export cheese to
Thyria from shipping substandard cheese.

Reasoning

What strategy does the consultant use in the counterargument? The consultant indicates to the
lawmaker that there is a reason to retain the board that the lawmaker has not considered. The
benefit the board provides is not that it identifies a great deal of substandard cheese and rejects
it (thus keeping the public healthy), but that the possibility that their cheese could be found
substandard is what keeps exporters from attempting to export low‐quality cheese to Thyria.

A

B
C
D

E

The consultant does reject the lawmaker’s argument, but the consultant does not propose higher standards.
Indeed, in suggesting that the board should be retained, the consultant implies that the board’s standards
are appropriate.
The consultant does not provide any evidence related to the board’s cost.
The only point the lawmaker raises that is not strictly economic is about the health consequences of
disbanding the board, but the consultant does not address this point at all.
Correct. This statement properly identifies the strategy the consultant employs in his or her
counterargument. The consultant points out that the board provides a significant benefit that the lawmaker
did not consider.
The consultant does not attack the integrity of the cheese inspectors; to the contrary, the consultant says
that their inspections deter the cheese exporters from shipping substandard cheese.

The correct answer is D.

581

GMAT® Official Guide 2018

579. The growing popularity of computer‐based activities was widely expected to result in a decline in television viewing, since
it had been assumed that people lack sufficient free time to maintain current television‐viewing levels while spending
increasing amounts of free time on the computer. That assumption, however, is evidently false: In a recent mail survey
concerning media use, a very large majority of respondents who report increasing time spent per week using computers
report no change in time spent watching television.
Which of the following would it be most useful to determine in order to evaluate the argument?
(A)

Whether a large majority of the survey respondents reported watching television regularly

(B)

Whether the amount of time spent watching television is declining among people who report that they rarely or never
use computers

(C)

Whether the type of television programs a person watches tends to change as the amount of time spent per week
using computers increases

(D)

Whether a large majority of the computer owners in the survey reported spending increasing amounts of time per
week using computers

(E)

Whether the survey respondents’ reports of time spent using computers included time spent using computers at work

Argument Evaluation
Situation

The argument is intended to debunk the assumption that people lack sufficient free time to
maintain television‐viewing levels while spending increasing amounts of free time on the
computer. To do so, it cites a survey of media use in which a large majority of respondents who
spend increasing amounts of time using computers also claim to have not altered the amount of
time they spend watching television.

Reasoning

What would it be most useful to know in order to evaluate the argument? The argument uses the survey
results to claim that people have enough free time to both maintain their television viewing levels
and spend increasing amounts of free time on the computer. But the survey, as reported here, did
not address whether people are spending their free time on the computer; the respondents reported
increasing time spent per week using computers. Since the argument is about free time, it is
important to know whether this is actually what the respondents were reporting.

A
B

C
D
E

The argument is concerned with the change in the amount of television watched by those whose computer
use increased, so whether the survey’s respondents reported watching television regularly is irrelevant.
The argument is concerned with the change in the amount of television watched by those whose computer
use has increased, so it does not matter whether the amount of time spent watching television among
people who do not use computers is declining, remaining the same, or increasing.
The argument is concerned with the amount of television watched by those whose computer use has
increased, not the type of television programs such a person does or does not watch.
The argument here is concerned with people who report spending increasing amounts of time on the
computer; what computer owners do is a separate question.
Correct. This statement properly identifies something that would be useful to know in evaluating the
argument: whether the survey data included time spent using computers at work—if it did, this would
make the data misleading as evidence for the argument’s conclusion.

The correct answer is E.

582

8.6 Critical Reasoning Answer Explanations

580. Although the school would receive financial benefits if it had soft drink vending machines in the cafeteria, we should not
allow them. Allowing soft drink machines there would not be in our students’ interest. If our students start drinking more soft
drinks, they will be less healthy.
The argument depends on which of the following?
(A)

If the soft drink vending machines were placed in the cafeteria, students would consume more soft drinks as a result.

(B)

The amount of soft drinks that most students at the school currently drink is not detrimental to their health.

(C)

Students are apt to be healthier if they do not drink soft drinks at all than if they just drink small amounts occasionally.

(D)

Students will not simply bring soft drinks from home if the soft drink vending machines are not placed in the cafeteria.

(E)

The school’s primary concern should be to promote good health among its students.

Argument Construction
Situation

Allowing soft drink vending machines in a school cafeteria would financially benefit the school,
but students who drink more soft drinks would become less healthy.

Reasoning

What must be true in order for the claim that students drinking more soft drinks would cause them to
become less healthy to justify the conclusion that soft drink vending machines should not be allowed in
the cafeteria? The argument is that because drinking more soft drinks would be unhealthy for the
students, allowing the vending machines would not be in the students’ interest, so the vending
machines should not be allowed. This reasoning depends on the implicit factual assumption that
allowing the vending machines would result in the students drinking more soft drinks. It also
depends on the implicit value judgment that receiving financial benefits should be less important
to the school than preventing a situation that would make the students less healthy.

A
B
C
D
E

Correct. If the cafeteria vending machines would not result in students consuming more soft drinks, then
allowing the machines would not harm the students’ health in the way the argument assumes.
Even if the amount of soft drinks the students currently drink were unhealthy, enabling the students to
drink more could make them even less healthy.
Even if drinking small amounts of soft drinks occasionally would not harm the students, vending
machines in the cafeteria could lead the students to drink excessive amounts.
Even if students who cannot buy soft drinks in the cafeteria sometimes bring them from home instead,
adding vending machines in the cafeteria could increase the students’ overall soft drink consumption.
A concern does not have to be the primary one in order to be valid and important. It could be held that
promoting students’ good health should not be the schools’ primary concern but should still be a more
important concern than the financial benefits from the vending machines.

The correct answer is A.

583

GMAT® Official Guide 2018

581. Many athletes inhale pure oxygen after exercise in an attempt to increase muscular reabsorption of oxygen. Measured
continuously after exercise, however, the blood lactate levels of athletes who inhale pure oxygen are practically identical,
on average, to those of athletes who breathe normal air. The lower the blood lactate level is, the higher the muscular
reabsorption of oxygen is.
If the statements above are all true, they most strongly support which of the following conclusions?
(A)

Athletes’ muscular reabsorption of oxygen is not increased when they inhale pure oxygen instead of normal air.

(B)

High blood lactate levels cannot be reduced.

(C)

Blood lactate levels are a poor measure of oxygen reabsorption by muscles.

(D)

The amount of oxygen reabsorbed by an athlete’s muscles always remains constant.

(E)

The inhaling of pure oxygen has no legitimate role in athletics.

Argument Construction
Situation

Blood lactate levels after exercise are practically identical in athletes who breathe normal air and
in those who inhale pure oxygen after exercise. The lower the blood lactate level, the higher the
muscular reabsorption of oxygen.

Reasoning

What conclusion do the stated facts most strongly support? We are told that lower blood lactate levels
correspond consistently to higher muscular reabsorption of oxygen. Since athletes who breathe
pure oxygen after exercise have blood lactate levels practically identical to those in athletes who
breathe normal air, probably muscular reabsorption of oxygen does not differ significantly between
athletes who breathe pure oxygen and those who breathe pure air.

A

B
C

D
E

Correct. As explained above, the stated facts suggest that muscular reabsorption of oxygen does not differ
significantly between athletes who breathe pure oxygen and those who breathe pure air. So breathing pure
oxygen instead of normal air after exercise probably does not increase athletes’ muscular reabsorption of
oxygen.
None of the statements indicates that blood lactate levels cannot be reduced by means other than inhaling
pure oxygen.
We are told that blood lactate levels are negatively correlated with muscular reabsorption of oxygen. This
negative correlation might allow muscular reabsorption of oxygen to be precisely determined by measuring
blood lactate levels.
Muscular reabsorption of oxygen might vary for reasons unrelated to whether an athlete has been inhaling
pure oxygen.
Inhaling pure oxygen might have some legitimate role unrelated to muscular reabsorption of oxygen.

The correct answer is A.

584

8.6 Critical Reasoning Answer Explanations

582. Which of the following most logically completes the argument?
Sviatovin is a medieval Moringian text whose author and exact date of composition are unknown. However, the events in the
life of Prince Sviatov that the text describes occurred in 1165, and in the diagram of Sviatov’s family that accompanies the
text his father, who died in 1167, is identified as still living. Thus Sviatovin must have been written between 1165 and 1167,
assuming that ________.
(A)

the life of Prince Sviatov is not the subject of any other medieval Moringian texts

(B)

the author of Sviatovin intended it to provide as accurate a report about Prince Sviatov’s exploits as possible

(C)

the diagram accurately represents the composition of Sviatov’s family at the time Sviatovin was written

(D)

Sviatovin is the earliest Moringian text whose composition can be dated to within a few years

(E)

Sviatovin was not written by Sviatov’s father himself

Argument Construction
Situation

A medieval Moringian text was written about Prince Sviatov. It is not known exactly when the
text was written, but the events described in it occurred in 1165 and a diagram in the text indicates
that Sviatov’s father—who died in 1167—was alive at the time it was composed.

Reasoning

What completion of the blank would provide the best reason for believing the argument’s conclusion?
The argument’s conclusion is that the book Sviatovin was written between 1165 and 1167. The
reasoning given is this: It could not have been written before 1165 because it includes events that
took place in that year. It could not have been written after 1167 because Sviatov’s father died
in that year. A diagram in the book suggests he was alive when the book was written. Was the
diagram correct in that suggestion? The argument depends on assuming that it was.

A
B

C

D
E

The argument focuses on one text and its date of composition. The argument does not need to make any
assumptions about other texts.
The argument does not assume this. Issues about the accurate reporting of Sviatov’s exploits (or about the
author’s intention to report them accurately) are irrelevant to the argument, which hinges on the accuracy
of the diagram that accompanies the text. The diagram does not report Sviatov’s exploits.
Correct. The reasoning in the argument assumes that the diagram was correct in representing the Prince’s
father as still living when the text was composed. If his father was not living when the text was written,
then the information that his father actually died in 1167 is no guarantee that the text was composed by
then.
See the explanation for answer choice (A) above. The degree of precision with which scholars have
succeeded in dating other Moringian texts is entirely irrelevant.
The conclusion concerns when Sviatovin was written and not by whom it was written. If, contrary to
answer choice (E), it were known that the author was Sviatov’s father, this would actually provide strong
support for the argument’s conclusion. Thus answer choice (E) does not provide a good reason for the
argument’s conclusion.

The correct answer is C.

585

GMAT® Official Guide 2018

583. A study of ticket sales at a summer theater festival found that people who bought tickets to individual plays had a no‐show
rate of less than 1 percent, while those who paid in advance for all ten plays being performed that summer had a no‐show
rate of nearly 30 percent. This may be at least in part because the greater the awareness customers retain about the cost
of an item, the more likely they are to use it.
Which of the following would, if true, best serve as an alternative explanation of the results of the study?
(A)

The price per ticket was slightly cheaper for those who bought all ten tickets in advance.

(B)

Many people who attended the theater festival believed strongly that they should support it financially.

(C)

Those who attended all ten plays became eligible for a partial refund.

(D)

Usually, people who bought tickets to individual plays did so immediately prior to each performance that they
attended.

(E)

People who arrived just before the performance began could not be assured of obtaining seats in a preferred
location.

Argument Construction
Situation

People who bought tickets to individual plays at a theater festival had a much lower no‐show rate
than did people who paid in advance for all ten plays.

Reasoning

What factor other than greater awareness of the ticket costs could explain why people who bought tickets
individually were more likely to attend the plays? The passage suggests that people who bought
tickets individually were more likely to attend the plays because they were more vividly aware of
what they had paid for each ticket. But there are other possible explanations—perhaps the people
who bought the tickets individually were more eager to attend each play for its own sake, or had
other characteristics or incentives that made them more likely to attend the plays.

A
B
C
D

E

A slight price difference would not plausibly explain why the no‐show rate was thirty times greater among
those who bought all the tickets in advance than among those who bought them individually.
This could be true of many people who bought their tickets individually as well as many who bought them
in advance.
This would provide an added incentive for those who bought tickets in advance to attend all the plays.
Correct. If people who bought individual tickets usually did so right before each performance, they would
have much less time after buying the tickets to change their minds about whether to attend than would
people who bought all the tickets in advance.
If anything, this might present an additional difficulty for those who bought individual tickets without
advance planning, so it would not help to explain the lower no‐show rate among buyers of individual
tickets.

The correct answer is D.

586

8.6 Critical Reasoning Answer Explanations

584. Although there is no record of poet Edmund Spenser’s parentage, we do know that as a youth Spenser attended the
Merchant Tailors’ School in London for a period between 1560 and 1570. Records from this time indicate that the Merchant
Tailors’ Guild then had only three members named Spenser: Robert Spenser, listed as a gentleman; Nicholas Spenser,
elected the Guild’s Warden in 1568; and John Spenser, listed as a “journeyman cloth‐maker.” Of these, the last was likely
the least affluent of the three—and most likely Edmund’s father, since school accounting records list Edmund as a scholar
who attended the school at a reduced fee.
Which of the following is an assumption on which the argument depends?
(A)

Anybody in sixteenth century London who made clothing professionally would have had to be a member of the
Merchant Tailors’ Guild.

(B)

The fact that Edmund Spenser attended the Merchant Tailors’ School did not necessarily mean that he planned to
become a tailor.

(C)

No member of the Guild could become Guild warden in sixteenth century London unless he was a gentleman.

(D)

Most of those whose fathers were members of the Merchant Tailors’ Guild were students at the Merchant Tailors’
School.

(E)

The Merchant Tailors’ School did not reduce its fees for the children of the more affluent Guild members.

Argument Construction
Situation

Records indicate that the poet Edmund Spenser attended the Merchant Tailors’ School for a
reduced fee as a youth. There is no record of his parentage, but at the time the Merchant Tailors’
Guild had only three members named Spenser, of whom the least affluent was probably John
Spenser.

Reasoning

What must be true in order for the cited facts to support the conclusion that John Spenser was probably
Edmund Spenser’s father? The implicit reasoning is that since Edmund Spenser attended the
Merchant Tailors’ School at a reduced fee, his father must have been poor. And since John Spenser
was probably the poorest of the three men named Spenser in the Merchant Tailors’ Guild, he was
probably Edmund Spenser’s father. This reasoning assumes that only the children of poor parents
had reduced fees at the Merchant Tailors’ School, that the children at the school generally had
fathers in the Merchant Tailors’ Guild, that children in that time and place generally shared their
fathers’ surnames, and that the two other Spensers in the Merchant Tailors’ Guild were not poor
enough for their children to qualify for reduced fees.

A
B
C
D
E

John Spenser, as a tailor and member of the guild, could have been Edmund Spenser’s father even if some
other professional tailors did not belong to the guild and did not have children at the school.
Although Edmund Spenser became a poet as an adult, he and all his classmates might have attended the
school as children because they planned to become tailors.
The argument assumes that a Guild’s Warden probably would have been wealthier than a journeyman
cloth‐maker, but that might have been probable even if the Guild’s Warden were not a “gentleman.”
Even if most children of fathers in the guild did not attend the school, all the children who did attend the
school might have had fathers in the guild.
Correct. If the school reduced its fees for children of wealthier guild members, then the fact that Edmund
Spenser’s fees were reduced would not provide evidence that his father was the poorest of the three
Spensers in the guild, as the argument requires.

The correct answer is E.

587

GMAT® Official Guide 2018

585. Trancorp currently transports all its goods to Burland Island by truck. The only bridge over the channel separating Burland
from the mainland is congested, and trucks typically spend hours in traffic. Trains can reach the channel more quickly than
trucks, and freight cars can be transported to Burland by barges that typically cross the channel in an hour. Therefore, to
reduce shipping time, Trancorp plans to switch to trains and barges to transport goods to Burland.
Which of the following would be most important to know in determining whether Trancorp’s plan, if implemented, is likely to
achieve its goal?
(A)

Whether transportation by train and barge would be substantially less expensive than transportation by truck

(B)

Whether there are boats that can make the trip between the mainland and Burland faster than barges can

(C)

Whether loading the freight cars onto barges is very time consuming

(D)

Whether the average number of vehicles traveling over the bridge into Burland has been relatively constant in
recent years

(E)

Whether most trucks transporting goods into Burland return to the mainland empty

Evaluation of a Plan
Situation

Transporting goods to Burland Island by truck takes many hours, because the trucks must take
the congested single bridge that reaches the island. Trains can get goods to the channel separating
Burland from the mainland more quickly than trucks can, and the freight cars can then be loaded
onto barges that can cross the channel in an hour. Trancorp plans to reduce shipping time by
switching from trucks to trains and barges.

Reasoning

What would it be most important to know in determining whether Trancorp will achieve its goal of
reducing shipping time? Trancorp’s plan could fail to reduce shipping time if there were some
aspect of the new shipping process, involving the trains and the barges, that took more time than
anticipated.

A
B

C

D

E

The goal of Trancorp’s plan is to reduce shipping time. This might lower costs—but whether or not it does
so is not directly relevant to whether or not the plan’s goal is achieved.
Trancorp’s plan involves the use of barges. If some boats can make the trip between the mainland and
Burland faster than barges can, that might be something to consider for the future, but it has nothing to
do with whether the current plan will reduce shipping time.
Correct. This statement properly identifies something that would be important in determining whether
Trancorp’s plan for reducing shipping time will achieve its goal—that is, whether loading the freight cars
onto the barges will use up all the time saved by not using trucks.
Regardless of variation in traffic, the bridge, according to the information provided in the passage, is
congested and typically causes trucking delays. Given this information, the degree of variation is not
helpful in evaluating Transcorp’s plan.
The state of the trucks returning to the mainland has nothing to do with whether Trancorp’s plan for
reducing shipping time will achieve its goal.

The correct answer is C.

588

8.6 Critical Reasoning Answer Explanations

586. Rainwater contains hydrogen of a heavy form called deuterium. The deuterium content of wood reflects the deuterium
content of rainwater available to trees during their growth. Wood from trees that grew between 16,000 and 24,000 years
ago in North America contains significantly more deuterium than wood from trees growing today. But water trapped in
several North American caves that formed during that same early period contains significantly less deuterium than rainwater
in North America contains today.
Which of the following, if true, most helps to reconcile the two findings?
(A)

There is little deuterium in the North American caves other than the deuterium in the water trapped there.

(B)

Exposure to water after a tree has died does not change the deuterium content of the wood.

(C)

Industrialization in North America over the past 100 years has altered the deuterium content of rain.

(D)

Trees draw on shallow groundwater from rain that falls during their growth, whereas water trapped in caves may have
fallen as rainwater thousands of years before the caves formed.

(E)

Wood with a high deuterium content is no more likely to remain preserved for long periods than is wood with a low
deuterium content.

Argument Construction
Situation

In North America, wood from trees that grew 16,000 to 24,000 years ago contains more
deuterium than wood from trees growing today. But water in caves that formed during that same
period contains less deuterium than rainwater contains today.

Reasoning

What could explain the puzzling discrepancy between the observed deuterium levels in wood and in
caves? Since the deuterium content of wood from trees reflects the deuterium content of rainwater
available to the trees while they grew, the deuterium levels observed in wood suggests that North
American rainwater contained more deuterium 16,000 to 24,000 years ago than it contains today.
But this conclusion seems at odds with the low deuterium levels in water in caves that formed
16,000 to 24,000 years ago. Several factors might explain the discrepancy: the water in those caves
might not be rainwater from the period when the caves formed; or some natural process might
have altered the deuterium levels in the cave water or the wood; or the wood or caves in which
deuterium levels were measured might be statistically abnormal somehow.

A

B

C
D

E

If the caves had absorbed deuterium out of the rainwater trapped in them, there would probably be
deuterium in the cave walls. So the observation that there is little deuterium in the caves apart from that in
the water eliminates one possible explanation for the oddly low deuterium levels in the cave water.
This suggests that the deuterium levels in the wood accurately reflect higher deuterium levels in rainwater
that fell 16,000 to 24,000 years ago, but it does not explain why the deuterium levels are so low in water in
the caves that formed then.
This could explain why deuterium levels in rainwater have changed, but it does not help explain the
discrepancy between the high deuterium levels in the wood and the low deuterium levels in the cave water.
Correct. If the water in the caves fell as rainwater thousands of years before the caves formed, it may date
from a period when rainwater contained much less deuterium than during the period 16,000 to 24,000
years ago, and much less than today.
If wood with high deuterium content were more likely to be preserved, then wood from 16,000 to 24,000
years ago might have a high deuterium content even if the rainwater then had a low deuterium content. So
the observation that wood with more deuterium is not more likely to be preserved eliminates one possible
explanation for the discrepancy.

The correct answer is D.

589

GMAT® Official Guide 2018

587. Which of the following most logically completes the argument below?
NowNews, although still the most popular magazine covering cultural events in Kalopolis, has recently suffered a significant
drop in advertising revenue because of falling circulation. Many readers have begun buying a competing magazine that, at
50 cents per copy, costs less than NowNews at $1.50 per copy. In order to boost circulation and thus increase advertising
revenue, NowNews’s publisher has proposed making it available at no charge, but this proposal has a serious drawback,
since ________.
(A)

Those Kalopolis residents with the greatest interest in cultural events are regular readers of both magazines.

(B)

One reason NowNews’s circulation fell was that its competitor’s reporting on cultural events was superior.

(C)

The newsstands and stores that currently sell NowNews will no longer carry it if it is being given away for free.

(D)

At present, 10 percent of the total number of copies of each issue of NowNews are distributed free to students on
college campuses in the Kalopolis area.

(E)

NowNews’s competitor would begin to lose large amounts of money if it were forced to lower its cover price.

Argument Construction
Situation

NowNews is suffering declines in circulation and advertising revenue due to competition from
a lower‐priced magazine. The publisher proposes offering NowNews for free to reverse these
declines.

Reasoning

What would suggest that the publisher’s proposal will fail to increase circulation and advertising revenue?
The proposal’s intended effect is simply to increase advertising revenue by increasing circulation.
Any evidence that offering the magazine for free will not result in more copies being circulated
or will not attract advertisers would therefore be evidence of a drawback in the proposal. So a
statement offering such evidence would logically complete the argument.

A
B
C
D
E

The fact that certain highly motivated Kalopolis residents still read NowNews even at a cost of $1.50 per
issue leaves open the possibility that providing the magazine free might still boost readership.
This suggests that improving its cultural reporting might help NowNews increase its circulation, not that
the publisher’s proposal will fail to do so.
Correct. If the proposal leads newsstands and stores to stop carrying NowNews, circulation and
advertising revenue would probably decline as a result.
Even if 10 percent of the copies of NowNews are already distributed for free, distributing the remaining
90 percent for free could still increase circulation and advertising revenue as the publisher intends.
Forcing a competing magazine to lower its cover price and lose lots of money would be an advantage
rather than a drawback of the proposal, as far as the publisher of NowNews was concerned.

The correct answer is C.

590

8.6 Critical Reasoning Answer Explanations

588. Archaeologist: Researchers excavating a burial site in Cyprus found a feline skeleton lying near a human skeleton. Both
skeletons were in the same sediment at the same depth and equally well‐preserved, suggesting that the feline and human
were buried together about 9,500 years ago. This shows that felines were domesticated around the time farming began,
when they would have been useful in protecting stores of grain from mice.
Which of the following, if true, would most seriously weaken the archaeologist’s argument?
(A)

Archaeologists have not found any remains of stores of grain in the immediate vicinity of the burial site.

(B)

The burial site in Cyprus is substantially older than any other known burial site in which a feline skeleton and a human
skeleton appear to have been buried together.

(C)

Paintings found near the burial site seem to show people keeping felines as domestic companions, but do not show
felines hunting mice.

(D)

In Cyprus, there are many burial sites dating from around 9,500 years ago in which the remains of wild animals
appear to have been buried alongside human remains.

(E)

Before felines were domesticated, early farmers had no effective way to protect stores of grain from mice.

Argument Evaluation
Situation

A human skeleton and a feline skeleton were apparently buried together in Cyprus about 9,500
years ago.

Reasoning

What would most strongly suggest that the skeletons do not show that felines were domesticated around
the time farming began? The argument implicitly assumes that farming in Cyprus began around
9,500 years ago, so evidence against that assumption would weaken the argument. The argument
could also be weakened by evidence that felines were domesticated much earlier, that the feline
skeleton was not from a domesticated cat, or that the two skeletons were not actually buried
together around 9,500 years ago.

A
B

C
D
E

Even if archaeologists searched for evidence of a grain store, the fact that no such evidence was found near
the burial site is at best only weak evidence that no grain store existed there or slightly farther away.
The lack of corroborating evidence from other burial sites would weaken the argument slightly but would
still be compatible with the hypothesis that this site revealed one of the very first burials of a domesticated
cat.
This would cast doubt on the hypothesis that cats were domesticated mainly to protect stores of grain, but
not on the argument’s conclusion that cats were domesticated around the time farming began.
Correct. If many wild animals were buried alongside humans in Cyprus around 9,500 years ago, then the
feline skeleton is just as likely to be that of a wild animal than that of a domesticated cat.
Since this would provide an additional reason why early farmers might have domesticated the local cats, it
would strengthen rather than weaken the argument.

The correct answer is D.

591

GMAT® Official Guide 2018

589. The heavy traffic in Masana is a growing drain on the city’s economy—the clogging of the streets of the central business
district alone cost the economy more than $1.2 billion over the past year. In order to address this problem, officials plan to
introduce congestion pricing, by which drivers would pay to enter the city’s most heavily trafficked areas during the busiest
times of the day.
Which of the following, if true, would most strongly indicate that the plan will be a success?
(A)

Approximately one-fifth of the vehicles in the central business district are in transit from one side of the city to the
other.

(B)

Planners expect that, without congestion pricing, traffic in Masana is likely to grow by 6 percent in the next five years.

(C)

In other urban areas, congestion pricing has strongly encouraged carpooling (sharing of rides by private commuters).

(D)

Several studies have shown that a reduction in traffic of 15 percent in Masana could result in 5,500 or more new
jobs.

(E)

Over 30 percent of the vehicles in the city’s center are occupied by more than one person.

Evaluation of a Plan
Situation

Traffic congestion in Masana has been harming the city’s economy. To address the problem,
officials plan to make drivers pay to enter the city’s most heavily trafficked areas during the busiest
times of day.

Reasoning

What would most strongly suggest that the plan will reduce the harm to Masana’s economy from traffic
congestion? In order to succeed, the plan will have to be implemented and effectively enforced.
Furthermore, the prices drivers pay will have to be high enough to significantly change their
behavior in ways that reduce the amount of traffic congestion in the city. Finally, the economic
benefits from the reduced traffic congestion will have to substantially outweigh any economically
damaging side effects of the congestion pricing. Any evidence that any of these conditions will
hold would provide at least some support for the prediction that the plan will succeed.

A

B
C

D
E

This provides no evidence that the congestion pricing would affect the behavior of either the one-fifth of
drivers whose vehicles traverse the city or of the other four-fifths of drivers, nor does it give any evidence
that the plan would produce overriding economic benefits.
This indicates that the traffic problem will grow worse if the plan is not implemented, but it does not
provide any evidence that the plan will help address the problem.
Correct. This indicates that similar plans have successfully changed drivers’ behavior in other cities in
a way likely to reduce the number of cars on the road in heavily trafficked areas at busy times of day
without producing harmful economic side effects. Thus, it provides evidence that the strategy could also be
successful in Masana.
Although this suggests that reducing traffic congestion would be economically beneficial, it doesn’t provide
any evidence that the plan will succeed in reducing traffic congestion.
This suggests that many drivers in the city center are already carpooling, which, if anything, indicates that
the plan will be less able to further affect those drivers’ behavior and thus could be less effective than it
might otherwise be.

The correct answer is C.

592

8.6 Critical Reasoning Answer Explanations

590. Economist: The most economically efficient way to reduce emissions of air pollutants is to tax them in proportion to the
damage they are likely to cause. But in Country Y, many serious pollutants are untaxed and unregulated, and policy makers
strongly oppose new taxes. Therefore, the best way to achieve a reduction in air pollutant emissions in Country Y would be
to institute fixed upper limits on them.
Which of the following is an assumption of the economist’s argument?
(A)

Policy makers in Country Y oppose all new taxes equally strongly, regardless of any benefits they may provide.

(B)

Country Y’s air pollutant emissions would not fall significantly if they were taxed in proportion to the damage they are
likely to cause.

(C)

Policy makers in Country Y strongly favor reductions in air pollutant emissions.

(D)

Country Y’s policy makers believe that air pollutant emissions should be reduced with maximum economic efficiency.

(E)

Policy makers in Country Y do not oppose setting fixed upper limits on air pollutant emissions as strongly as they
oppose new taxes.

Argument Construction
Situation

Although taxing air pollution emissions in proportion to the damage they cause is the most
economically efficient way to reduce those emissions, many serious pollutants in Nation Y are
untaxed and unregulated, and the nation’s policy makers strongly oppose new taxes. Therefore,
fixed upper limits on such emissions would more effectively reach this goal.

Reasoning

What must be true in order for the factors the economist cites to support the claim that fixing upper
limits on air pollutant emissions in Nation Y would be the best way to reduce those emissions? Political
opposition to taxation in Nation Y is the only factor the economist cites to support the argument’s
conclusion that it would be best to institute fixed upper limits on air pollutants. In order for
the premise to support the conclusion, there must be less political opposition in Nation Y to
instituting such limits than there would be to the proportional taxation approach the economist
prefers.

A
B
C

D

E

Even if the policy makers oppose some new taxes less than others, they could still oppose the proportional
taxation approach strongly enough for it to be utterly infeasible.
Even if the proportional taxation scheme would significantly reduce emissions, it still might not be the
best approach for Nation Y if it would generate too much political opposition to be viable there.
Even if policy makers in Nation Y do not strongly favor reducing emissions, fixing upper limits on
emissions might still be a better and more politically feasible way to reduce emissions than any alternative
is.
Since fixing upper emissions limits would be no more economically efficient than the proportional taxation
scheme, the policy makers’ support for economic efficiency would not make the former approach any more
politically feasible than the latter.
Correct. If the policy makers opposed fixing upper emissions limits as strongly as they oppose new taxes,
then their opposition to new taxes would no longer support the conclusion that fixing the emissions limits
is a better way to reduce emissions.

The correct answer is E.

593

GMAT® Official Guide 2018

591. Humans get Lyme disease from infected ticks. Ticks get infected by feeding on animals with Lyme disease, but the ease
of transmission from host animal to tick varies. With most species of host animal, transmission of Lyme disease to ticks
is extremely rare, but white‐footed mice are an exception, readily passing Lyme disease to ticks. And white‐footed mouse
populations greatly expand, becoming the main food source for ticks, in areas where biodiversity is in decline.
The information in the passage most strongly supports which of the following?
(A)

In areas where many humans are infected with Lyme disease, the proportion of ticks infected with Lyme disease is
especially high.

(B)

Very few animals that live in areas where there are no white‐footed mice are infected with Lyme disease.

(C)

Humans are less at risk of contracting Lyme disease in areas where biodiversity is high.

(D)

Ticks feed on white‐footed mice only when other host species are not available to them.

(E)

The greater the biodiversity of an area, the more likely any given host animal in that area is to pass Lyme disease to
ticks.

Argument Construction
Situation

White‐footed mice readily pass Lyme disease to ticks, which pass it to humans. White‐footed
mouse populations expand where biodiversity is declining.

Reasoning

What conclusion do the stated facts support? Since declining biodiversity causes white‐footed mouse
populations to increase, and white‐footed mice are especially likely to pass Lyme disease to ticks,
and ticks pass it to humans, declining biodiversity could reasonably be expected to increase the
incidence of Lyme disease in both ticks and humans.

A

B
C

D
E

In areas where many humans are infected with Lyme disease, the total number of ticks may be unusually
high, so even if the number of infected ticks is unusually high, the proportion of infected ticks may not be
unusually high
Most animals with Lyme disease may get it from sources other than ticks that have fed on infected mice.
Correct. If biodiversity is high, then any biodiversity decline that has already begun has likely not yet
reached a point where white‐footed mouse populations have greatly expanded, so the risk of people
contracting Lyme disease is still relatively less than in areas where biodiversity is low and where significant
decline in biodiversity has likely already occurred.
Even if ticks feed on white‐footed mice when few other species are available for them to feed on, they may
also sometimes feed on white‐footed mice when there are many other species for them to feed on.
The passage suggests that the overall incidence of Lyme disease is probably lower in more biodiverse areas,
so any given host animal in those areas would probably be less likely to pass Lyme disease to a tick.

The correct answer is C.

594

8.6 Critical Reasoning Answer Explanations

592. Many industrialized nations are trying to reduce atmospheric concentrations of carbon dioxide, a gas released by the
burning of fossil fuels. One proposal is to replace conventional cement, which is made with calcium carbonate, by a
new “eco‐cement.” This new cement, made with magnesium carbonate, absorbs large amounts of carbon dioxide when
exposed to the atmosphere. Therefore, using eco‐cement for new concrete building projects will significantly help reduce
atmospheric concentrations of carbon dioxide.
Which of the following, if true, most strengthens the argument?
(A)

The cost of magnesium carbonate, currently greater than the cost of calcium carbonate, probably will fall as more
magnesium carbonate is used in cement manufacture.

(B)

Eco‐cement is strengthened when absorbed carbon dioxide reacts with the cement.

(C)

Before the development of eco‐cement, magnesium‐based cement was considered too susceptible to water erosion
to be of practical use.

(D)

The manufacture of eco‐cement uses considerably less fossil fuel per unit of cement than the manufacture of
conventional cement does.

(E)

Most building‐industry groups are unaware of the development or availability of eco‐cement.

Argument Evaluation
Situation

Many nations are trying to reduce atmospheric concentrations of carbon dioxide. One proposed
method is to use a new type of "eco‐cement" that absorbs carbon dioxide from air.

Reasoning

What evidence, combined with the cited facts, would most support the prediction that using eco‐cement
will significantly help reduce atmospheric concentrations of carbon dioxide? The prediction assumes that
the use of eco‐cement would be an effective way to reduce carbon dioxide levels. Any evidence
supporting this assumption will support the prediction.

A

B

C

D

E

Since eco‐cement uses magnesium carbonate, the prediction that magnesium carbonate prices will fall
suggests that a potential financial barrier to widespread eco‐cement use will diminish. However, those
prices may not fall enough to make eco‐cement cost‐competitive with regular cement.
Even if absorbed carbon dioxide strengthens eco‐cement, the strengthened eco‐cement might still be
much weaker than regular cement and thus might never become widely used, in which case it will not
significantly help reduce atmospheric concentrations of carbon dioxide.
Even if eco‐cement is less susceptible to water erosion than earlier forms of magnesium‐based cement
were, it might still be much more susceptible to water erosion than regular cement is, and thus might never
become widely used.
Correct. This suggests that manufacturing eco‐cement produces much less carbon dioxide than
manufacturing regular cement does, so it supports the claim that widespread use of eco‐cement would be
an effective way to reduce carbon dioxide levels.
If anything, this lack of awareness makes it less likely that eco‐cement will become widely used, which
in turn makes it less likely that eco‐cement will significantly help reduce atmospheric concentrations of
carbon dioxide.

The correct answer is D.

595

GMAT® Official Guide 2018

593. Which of the following most logically completes the argument below?
Davison River farmers are currently deciding between planting winter wheat this fall or spring wheat next spring. Winter
wheat and spring wheat are usually about equally profitable. Because of new government restrictions on the use of Davison
River water for irrigation, per acre yields for winter wheat, though not for spring wheat, would be much lower than average.
Therefore, planting spring wheat will be more profitable than planting winter wheat, since ________.
(A)

the smaller‐than‐average size of a winter wheat harvest this year would not be compensated for by higher winter
wheat prices

(B)

new crops of spring wheat must be planted earlier than the time at which standing crops of winter wheat are ready to
be harvested

(C)

the spring wheat that farmers in the Davison River region plant is well adapted to the soil of the region

(D)

spring wheat has uses that are different from those of winter wheat

(E)

planting spring wheat is more profitable than planting certain other crops, such as rye

Argument Construction
Situation

Farmers in the Davison River region must choose between planting winter wheat in the fall and
planting spring wheat next spring. The crops tend to be equally profitable. This year’s winter wheat
crop yield is likely to be lower than average. The spring wheat yield should not be lower than
average. Thus, for these reasons (plus one that the argument omits), spring wheat will be more
profitable than winter wheat.

Reasoning

Which point would logically complete the argument? What would ensure spring wheat’s profitability
over winter wheat? Since the yield per acre of winter wheat is likely to be lower than usual, there
will most likely be less winter wheat to sell. Winter wheat could match its usual profitability if the
price farmers receive for it were to rise. If its price does not rise, however, it is unlikely to match its
usual profitability. It would thus be unlikely to match spring wheat’s profitability.

A

B
C
D

E

Correct. If this is true, it would mean that smaller‐than‐average winter wheat yields would translate into
lower‐than‐usual profits on winter wheat (while spring wheat would be as profitable as winter wheat
would normally be). This would justify the conclusion that spring wheat will be more profitable than
winter wheat.
This provides support for the idea that farmers must choose between planting winter wheat and planting
spring wheat, but it does not help determine which would be more profitable to plant.
This does not help explain why spring wheat is likely to be more profitable than winter wheat, because it
gives no information about how well winter wheat is adapted to the soil of the region.
That spring wheat and winter wheat have different uses is not helpful in supporting a conclusion about
which kind of wheat will be more profitable. It might help to know which of their uses are more profitable
than others.
Since the question is whether it will be more profitable to plant winter wheat or to plant spring wheat, the
fact that spring wheat is more profitable than nonwheat crops is not relevant.

The correct answer is A.

596

8.6 Critical Reasoning Answer Explanations

594. Advertisement: When your car’s engine is running at its normal operating temperature, any major brand of motor oil will
protect it about as well as Tuff does. When the engine is cold, it is a different story: Tuff motor oil flows better at lower
temperatures than its major competitors do. So, if you want your car’s engine to have maximum protection, you should use
Tuff.
Which of the following, if true, most strengthens the argument in the advertisement?
(A)

Tuff motor oil provides above‐average protection for engines that happen to overheat.

(B)

Tuff motor oil is periodically supplied free of charge to automobile manufacturers to use in factory‐new cars.

(C)

Tuff motor oil’s share of the engine oil market peaked three years ago.

(D)

Tuff motor oil, like any motor oil, is thicker and flows less freely at cold temperatures than at hot temperatures.

(E)

Tuff motor oil is manufactured at only one refinery and shipped from there to all markets.

Argument Evaluation
Situation

An advertisement argues that since Tuff motor oil flows better than its major competitors at low
temperatures and works about as well as they do at normal temperatures, it provides maximum
protection for car engines.

Reasoning

What additional evidence would suggest that Tuff motor oil provides the best available protection for
car engines? The argument requires the assumptions that no type of motor oil other than the
“major brands” provides superior protection, that flowing better at lower temperatures ensures
superior protection at those temperatures, and that Tuff protects car engines at least as well as its
competitors do at above‐normal temperatures. Any evidence supporting any of these assumptions
would strengthen the argument.

A

B
C
D
E

Correct. If Tuff provides above‐average protection when engines overheat, in addition to the solid
protection it provides at normal and low temperatures, it may well provide the best available protection
overall.
The company that makes Tuff might give automobile manufacturers free motor oil as a promotional
gimmick even if Tuff is an inferior product.
Tuff ’s sales might have declined over the past three years because consumers have realized that Tuff is an
inferior product.
The similar responses of Tuff and other motor oils to temperature changes do not suggest that Tuff
provides better protection overall than those other motor oils do.
Even if Tuff is manufactured at only one refinery, it may still be an inferior product.

The correct answer is A.

597

GMAT® Official Guide 2018

595. The Testament of William Thorpe was published around 1530 as an appendix to Thorpe’s longer Examination. Many
scholars, however, doubt the attribution of the Testament to Thorpe because, whereas the Examination is dated 1406,
the Testament is dated 1460. One scholar has recently argued that the 1460 date be amended to 1409, based on the
observation that when these numbers are expressed as Roman numerals, MCCCCLX and MCCCCIX, it becomes easy to see
how the dates might have become confused through scribal error.
Which of the following, if true, would most support the scholar’s hypothesis concerning the date of the Testament?
(A)

The sole evidence that historians have had that William Thorpe died no earlier than 1460 was the presumed date of
publication of the Testament.

(B)

In the preface to the 1530 publication, the editor attributes both works to William Thorpe.

(C)

Few writers in fifteenth‐century England marked dates in their works using only Roman numerals.

(D)

The Testament alludes to a date, “Friday, September 20,” as apparently contemporaneous with the writing of the
Testament, and September 20 fell on a Friday in 1409 but not in 1460.

(E)

The Testament contains few references to historical events that occurred later than 1406.

Argument Construction
Situation

The Testament of William Thorpe, dated 1460, was published around 1530 as an appendix to Thorpe’s
Examination, dated 1406. But when expressed in Roman numerals, 1460 could easily be confused
with 1409.

Reasoning

Given the facts cited, what would provide additional evidence that Thorpe’s Testament dates from
1409 rather than 1460? The scholar’s hypothesis that the work dates from 1409 is based on the
observation that in Roman numerals, 1409 might easily have been improperly transcribed as
1460. This hypothesis could be supported by evidence that the manuscripts were dated in Roman
numerals, or by any independent evidence that 1409 is a more likely date for the Testament than
1460.

A

B

C
D
E

This suggests that scholars have no biographical evidence that the Testament was published in 1460, but
they could still have abundant evidence of other types to support that date, such as the text’s cultural
allusions or references to other works.
The editor of the 1530 publication could easily have been mistaken about the authorship of one or both
works. And even if the editor were correct, Thorpe might have lived long enough to write one work in
1406 and the other in 1460.
This would cast doubt on the scholar’s argument by providing evidence that the original manuscripts were
not dated only in Roman numerals.
Correct. This provides strong evidence directly supporting the hypothesis that the Testament dates from
1409 specifically.
Even if the Testament contained only one reference to a historical event that occurred later than 1406 (for
example, one event in 1459), that reference alone could provide strong evidence that the work dates from
1460 rather than 1409.

The correct answer is D.

598

8.6 Critical Reasoning Answer Explanations

596. To reduce productivity losses from employees calling in sick, Corporation X implemented a new policy requiring employees
to come into work unless they were so sick that they had to go to a doctor. But a year after the policy was implemented, a
study found that Corporation X’s overall productivity losses due to reported employee illnesses had increased.
Which of the following, if true, would best explain why the policy produced the reverse of its intended effect?
(A)

After the policy was implemented, employees more frequently went to the doctor when they felt sick.

(B)

Before the policy was implemented, employees who were not sick at all often called in sick.

(C)

Employees coming into work when sick often infect many of their coworkers.

(D)

Unusually few employees became genuinely sick during the year after the policy was implemented.

(E)

There are many other factors besides employee illness that can adversely affect productivity.

Evaluation of a Plan
Situation

After a company started requiring employees to come to work unless they were sick enough to
have to go to a doctor, the company’s productivity losses from reported employee illness increased.

Reasoning

What would explain why the policy increased productivity losses from reported employee illness? Any
factors that could have plausibly caused the policy to increase employee absenteeism from reported
illness or to reduce the employees’ productivity at work as a result of reported illness could explain
why the policy increased productivity losses from reported illness.

A

B
C
D
E

Even though the policy required sick employees to consult a doctor, there is no reason to think that
employees’ doing so would have made them less productive than they would otherwise have been when
absent from work.
This suggests that the policy made it more difficult for employees to falsely claim illness as an excuse for a
work absence. Reduction in absences should result in productivity gains rather than losses.
Correct. This could have been a result of the policy and would have led to productivity losses possibly
greater than those seen before the policy was introduced.
This would help to explain lower productivity losses from reported illness after the policy was
implemented, not higher productivity losses.
The question is what could explain how the policy increased productivity losses from reported employee
illness specifically, not productivity losses from any other factors.

The correct answer is C.

599

GMAT® Official Guide 2018

597. Advertising by mail has become much less effective, with fewer consumers responding. Because consumers are
increasingly overwhelmed by the sheer amount of junk mail they receive, most discard almost all offers without considering
them. Thus, an effective way for corporations to improve response rates would be to more carefully target the individuals to
whom they mail advertising, thereby cutting down on the amount of junk mail each consumer receives.
Which of the following, if true, would most support the recommendation above?
(A)

There are cost‐effective means by which corporations that currently advertise by mail could improve response rates.

(B)

Many successful corporations are already carefully targeting the individuals to whom they mail advertising.

(C)

Any consumer who, immediately after receiving an advertisement by mail, merely glances at it is very likely to discard
it.

(D)

Improvements in the quality of the advertising materials used in mail that is carefully targeted to individuals can
improve the response rate for such mail.

(E)

Response rates to carefully targeted advertisements by mail are considerably higher, on average, than response rates
to most other forms of advertising.

Evaluation of a Plan
Situation

Advertising by mail has become less effective because consumers overwhelmed with the amount of
junk mail they receive discard almost all of it without considering it.

Reasoning

What would most help to support the claim that making mail advertising more carefully targeted would
improve response rates? The passage recommends targeted advertising, reasoning that since targeted
advertising would reduce the total amount of junk mail consumers receive, it would generate
higher response rates. Any additional evidence for the claim that carefully targeted advertising
would improve response rates would support this recommendation.

A
B
C
D
E

Even if targeted advertising and every other means of improving response rates were too expensive to be
cost‐effective, targeted advertising could still be effective for any corporation willing to pay the expense.
If many corporations already mail targeted advertising, and mail advertising is nonetheless yielding
declining response rates, that suggests that targeted mail is an ineffective way to increase response rates.
This could be equally true for targeted and untargeted mail advertising, so it does not suggest that the
former is more effective.
The question under consideration is whether more carefully targeted mail advertising would in itself
increase response rates, not whether higher quality advertising would do so.
Correct. This provides some evidence that carefully targeted mail advertising is associated with higher
response rates than untargeted mail advertising is, and therefore that targeting mail advertising more
carefully would improve response rates.

The correct answer is E.

600

8.6 Critical Reasoning Answer Explanations

598. Petrochemical industry officials have said that the extreme pressure exerted on plant managers during the last five
years to improve profits by cutting costs has done nothing to impair the industry’s ability to operate safely. However,
environmentalists contend that the recent rash of serious oil spills and accidents at petrochemical plants is traceable to
cost‐cutting measures.
Which of the following, if true, would provide the strongest support for the position held by industry officials?
(A)

The petrochemical industry benefits if accidents do not occur, since accidents involve risk of employee injury as well
as loss of equipment and product.

(B)

Petrochemical industry unions recently demanded that additional money be spent on safety and environmental
protection measures, but the unions readily abandoned those demands in exchange for job security.

(C)

Despite major cutbacks in most other areas of operation, the petrochemical industry has devoted more of its
resources to environmental and safety measures in the last five years than in the preceding five years.

(D)

There is evidence that the most damaging of the recent oil spills would have been prevented had cost‐cutting
measures not been instituted.

(E)

Both the large fines and the adverse publicity generated by the most recent oil spill have prompted the petrochemical
industry to increase the resources devoted to oil‐spill prevention.

Argument Evaluation
Situation

Petrochemical industry officials claim that pressure on plant managers to cut costs over the past
five years has not made the industry’s operations any less safe. Environmentalists claim that recent
oil spills and accidents show otherwise.

Reasoning

What evidence would most strongly suggest that the cost‐cutting pressure was not responsible for the
recent rash of oil spills and accidents? Evidence that the plant managers did not cut costs in any
specific ways likely to have increased the likelihood of oil spills and accidents would support the
industry officials’ position that the cost‐cutting pressure has not made petrochemical operations
any less safe.

A
B

C

D
E

Even if the petrochemical industry has good reasons to try to prevent accidents, the recent rash of serious
accidents suggests that it is failing to do so and that the cost‐cutting pressure might be responsible.
This suggests that the unions, whose members could directly observe the cost‐cutting pressure’s effects,
share the environmentalists’ belief that this pressure contributed to the oil spills and accidents. Because the
unions abandoned their demands, their concerns probably have not been addressed.
Correct. This suggests that, as the industry officials claim, the cost‐cutting pressure has not in itself
reduced the industry’s effectiveness at preventing oil spills and accidents. Thus, it suggests that other factors
are probably responsible for the recent problems.
This clearly suggests that the cost‐cutting measures have indeed caused the industry to operate less safely,
as the environmentalists claim.
Although this suggests that the industry is now trying to address the recent problems, the cost‐cutting
measures might nonetheless have caused all those problems.

The correct answer is C.

601

GMAT® Official Guide 2018

599. Economist: The price of tap water in our region should be raised drastically. Supplies in local freshwater reservoirs
have been declining for years because water is being used faster than it can be replenished. Since the price of tap
water has been low, few users have bothered to adopt even easy conservation measures.
The two sections in boldface play which of the following roles in the economist’s argument?
(A)

The first is a conclusion for which support is provided, and which in turn supports the main conclusion; the second is
the main conclusion.

(B)

The first is an observation for which the second provides an explanation; the second is the main conclusion but not
the only conclusion.

(C)

The first is a premise supporting the argument’s main conclusion; so is the second.

(D)

The first is the only conclusion; the second provides an explanation for the first.

(E)

The first is the main conclusion; the second is a conclusion for which support is provided, and which in turn supports
the first.

Argument Construction
Situation

Local water supplies have been declining for years because of excessive water use and low prices.
Few users have adopted even easy conservation measures.

Reasoning

What roles do the two boldface statements play in the argument? Both are factual observations. Since
no further evidence or support is provided for either, neither can be a conclusion in the argument.
However, interconnected causal explanations, signaled by because and since, are provided for both.
The observation in the first boldface statement is causally explained by the further observation
that water is being used faster than it can be replenished, which in turn is causally explained by the
entire final sentence. The observation in the second boldface statement is causally explained by the
observation that the price of tap water has been low. The only remaining portion of the argument
is the initial sentence, a recommendation supported by these four observations together, and by the
causal claims in which they are embedded. Thus, the four observations (including the two boldface
statements) and the causal claims containing them are all premises, and the initial statement is the
argument’s only conclusion.

A
B

C

D

E

As explained above, the two boldface statements are premises of the argument. Although causal
explanations are provided for both, no support or evidence is provided for either.
As explained above, the second boldface statement does provide part of the causal explanation for the
observation in the first boldface statement. But no support is provided for either statement, so neither is a
conclusion.
Correct. As explained above, each of the statements is a premise that serves along with other claims to
support the recommendation in the initial sentence, which is the argument’s only conclusion, and in that
sense its main conclusion.
As explained above, the second boldface statement does provide part of the causal explanation for the
observation in the first boldface statement. But no support is provided for either statement, so neither is a
conclusion in the argument.
As explained above, the two boldface statements are premises of the argument. Although causal
explanations are provided for both, no support or evidence is provided for either.

The correct answer is C.

602

8.6 Critical Reasoning Answer Explanations

600. Politician: Hybrid cars use significantly less fuel per kilometer than nonhybrids. And fuel produces air pollution, which
contributes to a number of environmental problems. Motorists can save money by driving cars that are more fuel efficient,
and they will be encouraged to drive hybrid cars if we make them aware of that fact. Therefore, we can help reduce the
total amount of pollution emitted by cars in this country by highlighting this advantage of hybrid cars.
Which of the following, if true, would most indicate a vulnerability of the politician’s argument?
(A)

People with more fuel‐efficient cars typically drive more than do those with less fuel‐efficient cars.

(B)

Not all air pollution originates from automobiles.

(C)

Hybrid cars have already begun to gain popularity.

(D)

Fuel‐efficient alternatives to hybrid cars will likely become available in the future.

(E)

The future cost of gasoline and other fuel cannot be predicted with absolute precision or certainty.

Argument Evaluation
Situation

According to a politician, hybrid cars use less fuel per kilometer than nonhybrids, and fuel
produces air pollution. Motorists can save money by driving fuel‐efficient cars, and will be
encouraged to do so if made aware of the fact. The politician concludes that highlighting this fact
will result in a reduction in air pollution.

Reasoning

What would suggest that telling motorists they can save money by driving fuel‐efficient cars would
not reduce automotive air pollution, despite the facts cited by the politician? The politician’s implicit
reasoning is that since hybrid cars use less fuel per kilometer, and fuel produces air pollution,
motorists who drive hybrid cars must produce less air pollution than those who drive nonhybrids.
The politician concludes that encouraging motorists to drive hybrid cars by telling them they
would save money on fuel will therefore reduce automotive air pollution. Evidence that motorists
who drive hybrid cars produce just as much automotive air pollution as those who drive
nonhybrids would undermine this argument.

A

B
C
D
E

Correct. If drivers of hybrid cars tend to drive more kilometers than drivers of nonhybrids, then they may
consume just as much fuel and produce just as much air pollution as the nonhybrid car drivers do, despite
their lower fuel use per kilometer.
The politician’s argument is only about air pollution from cars specifically, not air pollution from all
sources.
Even if hybrid cars are beginning to gain popularity, informing motorists of the cost savings from fuel
efficiency could help these cars become more popular than they would otherwise be.
Encouraging motorists to switch to hybrid cars now could reduce fuel use and automotive air pollution in
the near future even if other, more fuel‐efficient vehicles will become available further in the future.
Even if the future cost of fuel cannot be predicted accurately, encouraging motorists to switch to hybrid
cars could reduce air pollution as the politician argues.

The correct answer is A.

603

GMAT® Official Guide 2018

601. Which of the following most logically completes the passage?
A recent government study links the high rates of respiratory ailments in Groverston to airborne pollutants released by
the Woodco plywood manufacturing plant there. To address the problem, the government imposed strict regulations on
emissions which will go into effect in four years. Although Woodco plans to cut its emissions in half two years ahead of
schedule, it is unlikely that the rate of respiratory ailments will decline before the regulations go into effect, since ________.
(A)

the number of facilities capable of treating respiratory ailments is not likely to increase

(B)

reducing emissions even further than planned would necessitate decreasing production at Woodco

(C)

it is difficult to make accurate, long‐term predictions about emissions

(D)

not all respiratory ailments are caused by airborne pollutants

(E)

three new plywood manufacturing plants are about to go into production in Groverston

Argument Construction
Situation

A government study linked high rates of respiratory illness in Groverston to air pollutants released
by a plywood manufacturing plant. A government‐imposed restriction on emissions will go into
effect in four years. But the Woodco manufacturing plant plans to cut its emissions in half two
years ahead of schedule.

Reasoning

Which of the possible completions of the passage will provide the most support for the argument’s
conclusion? The argument’s conclusion is that the rate of respiratory illness is unlikely to decline
before the regulations go into effect four years from now. That is a surprising claim, given that the
existing plywood factory is going to halve its emissions two years from now—so this claim requires
additional support (to be provided by completing the blank). Such additional support is provided
by the information that three new plywood manufacturing plants will soon be starting production
in Groverston. This suggests that even if Groverston reduces its emissions by half over the next
two years—and even if each of the three new plants have only, let’s say, one quarter of the emissions
of the existing plant as currently operated—the total amount of airborne pollutants from plywood
manufacture in the Groverston area will not decline. Since the total airborne pollution from
the plants will not decline, we should not expect that the rate of respiratory illnesses from such
pollution would decline either.

A

B

C

D

E

The argument focuses on the incidence of respiratory ailments in Groverston, not on the availability
of treatment facilities for such ailments once they occur. So this statement is largely irrelevant to the
argument.
This information provides no support for the argument’s conclusion. It merely indicates a practical
difficulty Woodco would have in making more than a 50 percent reduction in its emissions. However,
even with the 50 percent emissions reduction that Woodco plans to implement in two years, the rate of
respiratory ailments should decline.
First, it is not clear that a prediction about levels of emissions in Groverston three or four years from now
is a long‐term prediction. More to the point, if it would be very difficult to predict the emissions even two
years from now, that would not support a conclusion that rates of respiratory ailments are unlikely to
decline. Rather, it would support the conclusion that we do not know whether rates will decline.
This information raises the possibility that within the next four years, an increase in the occurrence of
factors other than airborne pollutants might make a decline in respiratory ailments unlikely. However, the
passage provides no information whatsoever that implies an increase in the occurrence of such “other”
factors.
Correct. This helps to support the conclusion that rates of respiratory ailments in Groverston are unlikely
to decline before the new government regulations go into effect.

The correct answer is E.
604

8.6 Critical Reasoning Answer Explanations

602. One summer, floods covered low‐lying garlic fields situated in a region with a large mosquito population. Since mosquitoes
lay their eggs in standing water, flooded fields would normally attract mosquitoes, yet no mosquitoes were found in the
fields. Diallyl sulfide, a major component of garlic, is known to repel several species of insects, including mosquitoes, so it
is likely that diallyl sulfide from the garlic repelled the mosquitoes.
Which of the following, if true, most strengthens the argument?
(A)

Diallyl sulfide is also found in onions but at concentrations lower than in garlic.

(B)

The mosquito population of the region as a whole was significantly smaller during the year in which the flooding took
place than it had been in previous years.

(C)

By the end of the summer, most of the garlic plants in the flooded fields had been killed by waterborne fungi.

(D)

Many insect species not repelled by diallyl sulfide were found in the flooded garlic fields throughout the summer.

(E)

Mosquitoes are known to be susceptible to toxins in plants other than garlic, such as marigolds.

Argument Evaluation
Situation

When summer floods covered garlic fields in an area with many mosquitoes, no mosquitoes were
found in the fields, even though flooded fields would normally attract mosquitoes to lay their eggs
in the water. Diallyl sulfide, which is found in garlic, repels mosquitoes and some other insect
species, and likely accounts for the lack of mosquitoes in the area.

Reasoning

Given the facts cited, what would provide additional evidence that diallyl sulfide from the garlic made
mosquitoes avoid the flooded fields? The argument would be strengthened by any independent
evidence suggesting that diallyl sulfide pervaded the flooded fields or excluding other factors that
might explain the absence of mosquitoes in the fields.

A
B
C
D
E

This could strengthen the argument if mosquitoes also avoid flooded onion fields, but we do not know
whether they do.
This would weaken the argument by suggesting that the general mosquito population decline, rather than
the diallyl sulfide, could explain the absence of mosquitoes from the fields.
It is not clear how this would affect the amount of diallyl sulfide in the flooded fields, so this does not
provide evidence that the diallyl sulfide repelled the mosquitoes.
Correct. This provides evidence that there was no factor other than diallyl sulfide that reduced insect
populations in the flooded garlic fields.
If anything, this would weaken the argument, since it is at least possible that some of these toxins were
present in the flooded fields.

The correct answer is D.

605

GMAT® Official Guide 2018

603. Which of the following most logically completes the passage?
Pecan growers get a high price for their crop when pecans are comparatively scarce, but the price drops sharply when
pecans are abundant. Thus, in high‐yield years, growers often hold back part of their crop in refrigerated warehouses
for one or two years, hoping for higher prices in the future. This year’s pecan crop was the smallest in five years. It is
nonetheless quite possible that a portion of this year’s crop will be held back, since ________.
(A)

each of the last two years produced record‐breaking pecan yields

(B)

the quality of this year’s pecan crop is no worse than the quality of the pecan crops of the previous five years

(C)

pecan prices have not been subject to sharp fluctuations in recent years

(D)

for some pecan growers, this year’s crop was no smaller than last year’s

(E)

the practice of holding back part of one year’s crop had not yet become widespread the last time the pecan crop was
as small as it was this year

Argument Construction
Situation

The price of pecans tends to drop sharply in years when pecans are abundant. So in high-yield
years, growers often hold back part of the harvest in refrigerated warehouses. This year’s harvest
was the smallest in five years.

Reasoning

What would provide the best completion of the argument? The argument’s conclusion is that some of
this year’s crop might be held back. The blank to be completed should provide a reason in support
of that conclusion. What would lead us to believe that some of this year’s crop might go into cold
storage even though the crop was unusually small? Only in high‐yield years does this usually
happen. But suppose there is already a large quantity of pecans in cold storage from previous
harvests. Given this information, it would make perfect sense to expect that the pecans already in
cold storage would be marketed first, while some of the latest crop would be stored. This would
avoid the market oversupply and lower producer prices that might result if both all of this year’s
crop and all of the already stored pecans were marketed this year.

A
B
C

D

E

Correct. This answer choice provides information that makes it more probable that the conclusion is true.
The argument provides no information whatsoever that would suggest the decision to store or not to store
pecans is based on evaluation of the crop’s quality.
This information is of little or no relevance. It is reasonable to think that predictions about pecan prices
this year would affect the decision to store or not to store. But the information in this answer choice sheds
little or no light on what this year’s pecan prices might be, given that, as the passage tells us, this year’s crop
is exceptionally small.
It is not surprising that some growers had crops this year that were as big as their crops the year before.
But what matters, what affects the price of pecans, is the overall size of total pecan production and the
abundance or scarcity of pecans at the time.
This piece of history about marketing and storage practices explains why pecans were not placed in storage
in previous small‐yield years, but it provides no reason to believe that some of the new pecan crop will be
stored this year.

The correct answer is A.

606

8.6 Critical Reasoning Answer Explanations

604. Coffee shop owner: A large number of customers will pay at least the fair market value for a cup of coffee, even if there is
no formal charge. Some will pay more than this out of appreciation of the trust that is placed in them. And our total number
of customers is likely to increase. We could therefore improve our net cash flow by implementing an honor system in which
customers pay what they wish for coffee by depositing money in a can.
Manager: We’re likely to lose money on this plan. Many customers would cheat the system, paying a very small sum or
nothing at all.
Which of the following, if true, would best support the owner’s plan, in light of the manager’s concern?
(A)

The new system, if implemented, would increase the number of customers.

(B)

By roasting its own coffee, the shop has managed to reduce the difficulties (and cost) of maintaining an inventory of
freshly roasted coffee.

(C)

Many customers stay in the cafe for long stretches of time.

(D)

The shop makes a substantial profit from pastries and other food bought by the coffee drinkers.

(E)

No other coffee shop in the area has such a system.

Evaluation of a Plan
Situation

The owner and the manager of a coffee shop disagree about whether allowing customers to pay for
coffee on an honor system would increase or decrease profits.

Reasoning

What would be the best evidence that the honor‐system plan would increase profits even if many customers
cheated the system? The owner argues that profits would increase because many customers will
choose to pay as much or more than before and the total number of customers will likely increase.
But the manager points out that many customers would also choose to pay little or nothing.
Assuming that the manager is correct about that, what further support could the owner present for
the claim that the plan would still be profitable?

A
B
C
D
E

Since the owner has already basically asserted this, asserting it again would not provide any significant
additional support for the plan.
This suggests that the shop is already profitable, not that the honor‐system plan would make it more
profitable.
Customers who stay in the cafe for long stretches would not necessarily pay any more per cup on the
honor‐system plan than other customers would.
Correct. If the customer base increases (as both the owner and the manager seem to agree), more
customers will likely purchase highly profitable pastries and other foods, thus boosting profits.
The reason no other coffee shop in the area has an honor system may be that their owners and managers
have determined that it would not be profitable.

The correct answer is D.

607

GMAT® Official Guide 2018

605. Which of the following most logically completes the argument?
By competing with rodents for seeds, black ants help control rodent populations that pose a public health risk. However, a
very aggressive species of black ant, the Loma ant, which has recently invaded a certain region, has a venomous sting that
is often fatal to humans. Therefore, the planned introduction into that region of ant flies, which prey on Loma ants, would
benefit public health, since ________.
(A)

ant flies do not attack black ants other than Loma ants

(B)

Loma ants are less effective than many bird species in competing with rodents for seeds

(C)

certain other species of black ants are more effective than Loma ants in competing with rodents for seeds

(D)

the sting of Loma ants can also be fatal to rodents

(E)

some pesticides that could be used to control Loma ants are harmful to the environment

Argument Construction
Situation

Black ants help to control populations of rodents by competing with them for seeds. But a very
aggressive species of black ant, the Loma ant, has a sting that can be fatal to humans. Ant flies
prey on Loma ants and their presence can thereby benefit public health.

Reasoning

Which of the possible completions of the passage provides the most support for the conclusion? The
argument’s conclusion is that introducing ant flies into the region where Loma ants have recently
invaded would benefit public health. We know from the passage that black ants, generally, benefit
public health by keeping down rodent populations. However, the sting of Loma ants, a species of
black ant, can be fatal to humans. Ant flies prey on Loma ants. To that extent their introduction
in the region would tend to benefit public health by making fatal Loma stinging of humans less
likely. But if these ant flies also prey on black ants other than the Loma ants, then to that extent
they would undermine another public health benefit associated with controlling rodents. Thus, the
information that ant flies do not prey on black ants other than Loma ants would provide strong
logical support for the conclusion.

A

B
C
D

E

Correct. This most logically completes the argument because it addresses a potential downside of
introducing the ant flies into the region. The potential downside is that it might reduce the desirable effect
that other species of black ants have in keeping down the rodent populations.
We have no idea whether the bird species that are more effective than Loma ants at competing with
rodents for seeds are even present in the region in question.
This does not help the conclusion very much because we do not know from the passage whether ant flies
prey on other species of black ants besides Loma ants.
If anything, this is a reason not to introduce ant flies into the region. This answer choice at least suggests
that Loma ants might have some positive effect on public health because they might keep down rodent
populations by reducing their survival chances.
This provides very little support for the conclusion. It does not exclude the possibility that there are
pesticides—perhaps several—that would control Loma ants effectively without harming the environment.
So it is not a strong reason for introducing ant flies.

The correct answer is A.

608

8.6 Critical Reasoning Answer Explanations

606. Journalist: In physics journals, the number of articles reporting the results of experiments involving particle accelerators
was lower last year than it had been in previous years. Several of the particle accelerators at major research institutions
were out of service the year before last for repairs, so it is likely that the low number of articles was due to the decline in
availability of particle accelerators.
Which of the following, if true, most seriously undermines the journalist’s argument?
(A)

Every article based on experiments with particle accelerators that was submitted for publication last year actually was
published.

(B)

The average time scientists must wait for access to a particle accelerator has declined over the last several years.

(C)

The number of physics journals was the same last year as in previous years.

(D)

Particle accelerators can be used for more than one group of experiments in any given year.

(E)

Recent changes in the editorial policies of several physics journals have decreased the likelihood that articles
concerning particle‐accelerator research will be accepted for publication.

Argument Evaluation
Situation

A journalist attributes the low number of articles about particle accelerators in physics journals to
the fact that several accelerators at major research institutions had been out of service the previous
year.

Reasoning

What point undermines the journalist’s argument? The journalist assumes that the researchers’ lack of
access to the accelerators is responsible for the decline in the number of articles. What else could
explain fewer articles? What if the decline is due, not to the availability of the accelerators for
experiments, but to policies regarding publishing articles related to such experiments? An alternate
explanation is that changes in the editorial policies of physics journals, rather than the effect of the
out‐of‐service accelerators, could well be responsible for the lower number of published articles
about particle‐accelerator research.

A
B
C
D
E

This statement rules out the possibility that submitted articles were not published, and eliminating this
alternate explanation tends to support the argument.
A decline in waiting time would seem to promote more articles about accelerator research being written
and published, not fewer.
While the decline in articles could be explained by a decline in the number of journals, this statement
eliminates that alternate explanation.
If the accelerators can be used for multiple experiments, then it is reasonable to expect more articles related
to them, not fewer.
Correct. This statement properly identifies a point that undermines the journalist’s reasoning.

The correct answer is E.

609

GMAT® Official Guide 2018

607. Birds have been said to be descended from certain birdlike dinosaur species with which they share distinctive structural
features. The fossil record, however, shows that this cannot be so, since there are bird fossils that are much older than the
earliest birdlike dinosaur fossils that have been found.
Which of the following is an assumption on which the argument relies?
(A)

The birdlike dinosaurs have no living descendants.

(B)

There are no flightless dinosaur species that have the distinctive structural features shared by birds and birdlike
dinosaurs.

(C)

There are no birdlike dinosaur fossils that are older than the bird fossils but have not yet been unearthed.

(D)

It could not have been the case that some birds were descended from one of the birdlike dinosaur species and other
birds from another.

(E)

Birds cannot have been descended from dinosaur species with which the birds do not share the distinctive structural
features.

Argument Construction
Situation

Although birds have been said to be descended from birdlike dinosaurs, some bird fossils predate
the earliest known birdlike dinosaur fossils.

Reasoning

What must be true in order for the premise that some bird fossils predate the earliest known birdlike
dinosaur fossils to support the conclusion that birds are not descended from birdlike dinosaurs? The
argument implicitly reasons that since the cited bird fossils predate the earliest known birdlike
dinosaur fossils, they must be from birds that lived before the earliest birdlike dinosaurs, and
which therefore could not have been descended from birdlike dinosaurs. This reasoning assumes
that any birdlike dinosaurs that lived before the first birds would have left fossils that still exist. It
also assumes that no undiscovered birdlike dinosaur fossils predate the cited bird fossils.

A
B
C
D

E

The argument is only about whether birds are descended from birdlike dinosaurs. Whether birdlike
dinosaurs have any living descendants other than birds is irrelevant.
The argument is only about birds and birdlike dinosaurs. It is not about other types of dinosaurs that were
not birdlike.
Correct. If any undiscovered birdlike dinosaur fossils predate the cited bird fossils, then the latter fossils’
age does not support the conclusion that birds are not descended from birdlike dinosaurs.
The argument purports to establish that the relative ages of bird fossils and birdlike dinosaur fossils
show that birds cannot be descended from any of the known birdlike dinosaur species. In doing this, it
acknowledges multiple birdlike dinosaur species and leaves open the question of whether some birds may
be descended from one such species and other birds from another such species.
The argument does not claim that the known fossil record shows that birds cannot be descended from
dinosaurs. It only claims that the record shows that they cannot be descended from the birdlike dinosaurs
that shared their distinctive structural features.

The correct answer is C.

610

8.6 Critical Reasoning Answer Explanations

608. City council member: Demand for electricity has been increasing by 1.5 percent a year, and there simply is no more space
to build additional power plants to meet future demand increases. We must therefore begin to curtail usage, which is why I
propose passing ordinances requiring energy‐conservation measures in all city departments.
The city council member’s proposal assumes which of the following?
(A)

Existing power plants do not have the capacity to handle all of the projected increase in demand for electricity.

(B)

No city departments have implemented energy‐conservation measures voluntarily.

(C)

Passing ordinances designed to curtail electricity usage will not have negative economic consequences for the city.

(D)

Residential consumers are not responsible for the recent increases in demand for electricity.

(E)

City departments that successfully conserve energy will set a good example for residential and industrial consumers
of electricity.

Argument Construction
Situation

A city council member proposes energy‐conservation measures for all city government
departments because there is no room to build new power plants to meet future increases in the
demand for electricity.

Reasoning

What must be true in order for the factors the city council member cites to help justify the proposal? The
city council member says electricity usage must be curtailed on account of an increasing demand
for electricity and a lack of space for new power plants that could meet future demand increases.
In order for this reasoning to help justify the proposal, the cited factors must actually establish a
need to curtail electricity usage.

A

B
C
D
E

Correct. If current power plants could satisfy the projected increased demand for electricity, then
the increasing demand and the lack of room to build new plants would not establish a need to curtail
electricity usage.
The proposed ordinances could still be necessary even if one city department had voluntarily implemented
energy‐conservation measures.
Passing the ordinances could still be necessary even if they would have some negative economic effects.
No matter who is responsible for the recent increases in demand, curtailing the city government’s
electricity usage could still help to reduce demand.
Ordinances to curtail the city government’s energy usage could be economically necessary regardless of
whether or not departments that obey the ordinances set a good example.

The correct answer is A.

611

GMAT® Official Guide 2018

609. Which of the following most logically completes the argument below?
Using broad‐spectrum weed killers on weeds that are competing with crops for sunlight, water, and nutrients presents a
difficulty: how to keep the crop from being killed along with the weeds. For at least some food crops, specially treated
seed that produces plants resistant to weed killers is under development. This resistance wears off as the plants mature.
Therefore, the special seed treatment will be especially useful for plants that ________.
(A)

produce their crop over an extended period of time, as summer squash does

(B)

produce large seeds that are easy to treat individually, as corn and beans do

(C)

provide, as they approach maturity, shade dense enough to keep weeds from growing

(D)

are typically grown in large tracts devoted to a single crop

(E)

are cultivated specifically for the seed they produce rather than for their leaves or roots

Argument Construction
Situation

A difficulty in using broad‐spectrum weed killers is keeping them from killing the food crops
along with the weeds. Specially treated seed is being developed that will protect certain food crop
plants in their earlier stages of growth.

Reasoning

Which is the best completion for the conclusion? The conclusion is incompletely stated as “Therefore,
the special seed treatment will be especially useful for plants that ________.” The question is what
sorts of plants does the passage suggest the seed treatment would be especially useful for. We have
been told that this treatment makes the plants resistant to weed killer, but that this resistance
wears off when the plant matures. So the treatment will be most useful with plants that are not
harmed by weed killer and that suffer no significant disadvantage when the resistance wears off as
the plant matures. Choice (C) is the correct answer choice because it describes a sort of plant that
can combat weeds and requires no weed killer once the plant matures.

A
B

C

D

E

Given that the seed treatment wears off as the plant matures, it would not be especially useful for plants
that produce their crops over an extended period.
We have not been told whether small seeds are more difficult to treat, and so we have no basis to conclude
that the special seed treatment would be especially useful for plants that have large seeds that are easy to
treat individually. We have also been given no reason to think that it is better to treat seeds individually.
Correct. Plants that, as they approach maturity, produce shade dense enough to keep weeds from growing,
would benefit from resistance to weed killer when young and would not need weed killer when they have
matured and lost their resistance.
We have been given no reason to think that the seed treatment would be especially useful for plants grown
in a large tract devoted to a single crop. For example, why would it be less useful for small tracts with a
variety of crops?
A plant harvested for its roots, fruits, or leaves, rather than for its seeds, would derive no less an advantage
from resistance to weed killers in earlier stages of growth.

The correct answer is C.

612

8.6 Critical Reasoning Answer Explanations

610. Previously, Autoco designed all of its cars itself and then contracted with specialized parts suppliers to build parts
according to its specifications. Now it plans to include its suppliers in designing the parts they are to build. Since many
parts suppliers have more designers with specialized experience than Autoco has, Autoco expects this shift to reduce the
overall time and cost of the design of its next new car.
Which of the following, if true, most strongly supports Autoco’s expectation?
(A)

When suppliers provide their own designs, Autoco often needs to modify its overall design.

(B)

In order to provide designs for Autoco, several of the parts suppliers will have to add to their existing staffs of
designers.

(C)

Parts and services provided by outside suppliers account for more than 50 percent of Autoco’s total costs.

(D)

When suppliers built parts according to specifications provided by Autoco, the suppliers competed to win contracts.

(E)

Most of Autoco’s suppliers have on hand a wide range of previously prepared parts designs that can readily be
modified for a new car.

Evaluation of a Plan
Situation

A car manufacturer plans to have its parts suppliers start helping to design the parts they build for
the manufacturer. Many parts suppliers have more designers with specialized experience than the
manufacturer has.

Reasoning

What would make it more likely that having the parts suppliers help design the parts will reduce the
time and cost of designing the manufacturer’s next new car? In order for the change to reduce the
time and cost, the parts suppliers involved in designing the next car will probably have to do their
portion of the design process faster and cheaper than the manufacturer would have, and the design
collaboration process will have to avoid producing substantial new inefficiencies.

A
B
C
D

E

The additional need to modify the overall design would probably make the design process slower and more
expensive, not faster and cheaper.
The additional need to hire more designers would probably increase design costs, not reduce them.
Although this suggests that the change is likely to substantially affect the design’s expense, it does not
indicate whether the expense will increase or decrease.
If anything, this competition probably made Autoco’s previous design process cheaper. It does not suggest
that the new design process, which may involve less competition, will be faster or cheaper than the
previous one.
Correct. Modifying the previously prepared parts designs will probably be faster and cheaper than
creating new designs from scratch.

The correct answer is E.

613

GMAT® Official Guide 2018

611. In Stenland, many workers have been complaining that they cannot survive on minimum wage, the lowest wage an employer
is permitted to pay. The government is proposing to raise the minimum wage. Many employers who pay their workers the
current minimum wage argue that if it is raised, unemployment will increase because they will no longer be able to afford to
employ as many workers.
Which of the following, if true in Stenland, most strongly supports the claim that raising the minimum wage there will not
have the effects that the employers predict?
(A)

For any position with wages below a living wage, the difficulty of finding and retaining employees adds as much to
employment costs as would raising wages.

(B)

Raising the minimum wage does not also increase the amount employers have to contribute in employee benefits.

(C)

When inflation is taken into account, the proposed new minimum wage is not as high as the current one was when it
was introduced.

(D)

Many employees currently being paid wages at the level of the proposed new minimum wage will demand significant
wage increases.

(E)

Many employers who pay some workers only the minimum wage also pay other workers wages that are much higher
than the minimum.

Argument Evaluation
Situation

Stenland’s government proposes to raise the minimum wage because many workers have
complained they cannot survive on it. But many employers claim that raising the minimum wage
will increase unemployment.

Reasoning

What evidence would most strongly suggest that raising the minimum wage will not increase
unemployment? The employers with minimum‐wage workers implicitly reason that because raising
the minimum wage will increase the wages they have to pay each worker, it will reduce the number
of workers they can afford to employ, and thus will increase unemployment. Evidence that the
increased wage would not actually increase the employers’ expenses per employee would cast doubt
on their prediction, as would evidence that reducing the number of minimum‐wage workers would
not increase the nation’s overall unemployment rate.

A

B
C
D
E

Correct. This suggests that raising the minimum wage would make it easier for employers to find and
retain minimum‐wage employees, and that the savings would fully offset the cost of paying the higher
wages. If there were such offsetting savings, the employers should still be able to afford to employ as many
workers as they currently do.
Even if raising the minimum wage does not increase employers’ costs for employee benefits, paying the
higher wage might still in itself substantially increase employers’ overall costs per employee.
For all we know, the current minimum wage might have substantially increased unemployment when it
was introduced.
These additional demands would probably raise employers’ overall costs per employee, making it more
likely that increasing the minimum wage would increase overall unemployment.
Even if some workers receive more than the minimum wage, raising that wage could still raise employers’
expenses for employing low‐wage workers, making it too expensive for the employers to employ as many
workers overall.

The correct answer is A.

614

8.6 Critical Reasoning Answer Explanations

612. Which of the following most logically completes the argument?
The attribution of the choral work Lacrimae to the composer Pescard (1400–1474) has been regarded as tentative, since
it was based on a single treatise from the early 1500s that named Pescard as the composer. Recently, several musical
treatises from the late 1500s have come to light, all of which name Pescard as the composer of Lacrimae. Unfortunately,
these newly discovered treatises lend no support to the attribution of Lacrimae to Pescard, since ________.
(A)

the treatise from the early 1500s misidentifies the composers of some of the musical works it considers

(B)

the author of the treatise from the early 1500s had no very strong evidence on which to base the identification of
Pescard as the composer of Lacrimae

(C)

there are works that can conclusively be attributed to Pescard that are not even mentioned in the treatise from the
early 1500s

(D)

the later treatises probably had no source for their attribution other than the earlier treatise

(E)

no known treatises from the 1600s identify Pescard as the composer of Lacrimae

Argument Construction
Situation

A choral work has been tentatively attributed to Pescard based on a single treatise from the early
1500s. But several treatises from the late 1500s have recently been discovered, and all of them
attribute the work to Pescard.

Reasoning

Which of the answer choices provides the strongest reason for the conclusion? The argument’s conclusion
is that the newly discovered late‐1500 treatises lend no support to the attribution of Lacrimae to
Pescard. It is worth noting that prior to the conclusion the passage provides information which
suggests that these newly discovered treatises do lend support to the attribution. So the question
is: Why don’t they? A good reason for thinking they do not is that the newly discovered treatises
probably derive solely from the attribution given in the earlier text. Thus the attributions in the
later treatises are only as reliable as the attribution in the earlier treatise—and the argument
suggests that that reliability has not been conclusively established.

A
B
C
D

E

This makes the treatise from the early 1500s less reliable, but it does not explain why the newly discovered
treatises are unreliable.
Like answer choice (A), this is irrelevant. The question is not why the treatise from the early 1500s fails to
lend support to the attribution but why the treatises from the late 1500s fail to do so.
This is irrelevant because it does not refer to the newly discovered treatises whose attribution of Lacrimae
is at issue.
Correct. The question is whether these newly discovered treatises lend additional support. Lacrimae
has already been tentatively attributed to Pescard based on the text from the early 1500s. So, if the later
treatises base their attribution solely on the earlier treatise, then they provide no additional support beyond
that already provided by the earlier treatise.
This leaves open the possibility that there was no treatise at all in the 1600s that discussed Pescard or
Lacrimae. Also, it fails to provide significant evidence either for or against Pescard’s having composed
Lacrimae. But even if it did provide such evidence, it would be irrelevant because the issue is why the
late‐1500 treatises fail to provide significant support for the attribution of Lacrimae to Pescard, not
whether Pescard composed the work.

The correct answer is D.

615

GMAT® Official Guide 2018

613. When trying to identify new technologies that promise to transform the marketplace, market researchers survey the
managers of those companies that are developing new technologies. Such managers have an enormous stake in
succeeding, so they invariably overstate the potential of their new technologies. Surprisingly, however, market researchers
typically do not survey a new technology’s potential buyers, even though it is the buyers—not the producers—who will
ultimately determine a technology’s commercial success.
Which of the following, if true, best accounts for the typical survey practices among market researchers?
(A)

If a new technology succeeds, the commercial benefits accrue largely to the producers, not to the buyers, of that
technology.

(B)

People who promote the virtues of a new technology typically fail to consider that the old technology that is currently
in use continues to be improved, often substantially.

(C)

Investors are unlikely to invest substantial amounts of capital in a company whose own managers are skeptical about
the commercial prospects of a new technology they are developing.

(D)

The potential buyers for not‐yet‐available technologies can seldom be reliably identified.

(E)

The developers of a new technology are generally no better positioned than its potential buyers to gauge how rapidly
the new technology can be efficiently mass‐produced.

Argument Construction
Situation

Market researchers seeking to identify new technologies that have the potential to transform the
marketplace survey managers of companies developing new technologies, but typically not the
potential buyers of new technologies, even though managers tend to overstate the potential of
their new technologies and it is the buyers who determine the products’ commercial success.

Reasoning

What best explains why it is managers, not buyers, that the market researchers survey? Why, despite the
information in the passage, are managers of technology companies surveyed while potential buyers
are typically not? A partial explanation would be that it is difficult to reliably determine who
the potential buyers of new technologies will be. If market researchers cannot identify who the
potential buyers of as‐yet unavailable technologies will be, that explains why they are not typically
surveyed—and why the next best alternative may be to survey managers.

A

B

C

D
E

This answer choice tells us who would benefit from commercial success of new technologies. But it
says nothing about whose opinion would be most valuable in predicting the commercial success of new
technologies.
At most, this could help explain why managers overstate the potential of their new technologies. But it
does not explain the motives of market researchers in relying on the managers’ rather than buyers’ opinions
about new technologies.
Given that managers of technology companies will want to attract investors, this helps to explain why the
managers would tend to overstate the potential of their new technologies. But it does not help to explain
the survey practices.
Correct. This accounts for why potential buyers of new technologies are not typically sought out in surveys
by market researchers: It is difficult to determine in advance who they are.
This, like answer choice (C), tends to make the practices of market researchers more difficult rather
than easier to understand. If developers of new technologies are no better at gauging how rapidly a new
technology can be mass‐produced (a factor affecting commercial success), then all the more reason to
survey potential buyers rather than the managers.

The correct answer is D.

616

8.6 Critical Reasoning Answer Explanations

614. Infotek, a computer manufacturer in Katrovia, has just introduced a new personal computer model that sells for significantly
less than any other model. Market research shows, however, that very few Katrovian households without personal
computers would buy a computer, regardless of its price. Therefore, introducing the new model is unlikely to increase the
number of computers in Katrovian homes.
Which of the following is an assumption on which the argument depends?
(A)

Infotek achieved the lower price of the new model by using components of lower quality than those used by other
manufacturers.

(B)

The main reason cited by consumers in Katrovia for replacing a personal computer is the desire to have an improved
model.

(C)

Katrovians in households that already have computers are unlikely to purchase the new Infotek model as an additional
computer for home use.

(D)

The price of other personal computers in Katrovia is unlikely to drop below the price of Infotek’s new model in the
near future.

(E)

Most personal computers purchased in Katrovia are intended for home use.

Argument Construction
Situation

In Katrovia, a new personal computer model costs less than any other model. But market research
shows that very few Katrovian households without personal computers would buy even cheap
ones.

Reasoning

What must be true in order for the stated facts to support the conclusion that introducing the new
computer model is unlikely to increase the overall number of computers in Katrovian homes? The market
research supports the conclusion that no new computer model is likely to significantly increase the
number of computers in Katrovian homes that currently lack computers. But the overall number of
computers in Katrovian homes will still increase if Katrovian homes that already have computers
buy additional computers while keeping their existing ones. So the argument has to assume that
the new computer model will not increase the number of additional computers purchased for
Katrovian homes that already have computers.

A
B
C

D

E

Even if Infotek used high‐quality components in the new computer model, Katrovians might still refuse to
buy it.
Replacing a personal computer does not change the overall number of personal computers in homes, so
Katrovians’ motives for replacing their computers are irrelevant to the argument.
Correct. As explained above, unless computers of the new model are purchased as additional computers
for Katrovian homes that already have computers, the new model’s introduction is unlikely to increase the
overall number of computers in Katrovian homes.
The assumption that other personal computer prices would stay relatively high does not help establish the
link between its premises and its conclusion. If answer choice D were false, the argument would be no
weaker than it is without any consideration of other computers’ potential prices.
If most personal computers purchased in Katrovia were not intended for home use, then the new model’s
introduction would be even less likely to increase the number of personal computers in Katrovian homes.
So the argument does not depend on assuming that most of the computers purchased are for home use.

The correct answer is C.

617

GMAT® Official Guide 2018

615. Fast‐food restaurants make up 45 percent of all restaurants in Canatria. Customers at these restaurants tend to be young;
in fact, studies have shown that the older people get, the less likely they are to eat in fast‐food restaurants. Since the
average age of the Canatrian population is gradually rising and will continue to do so, the number of fast‐food restaurants is
likely to decrease.
Which of the following, if true, most seriously weakens the argument?
(A)

Fast‐food restaurants in Canatria are getting bigger, so each one can serve more customers.

(B)

Some older people eat at fast‐food restaurants more frequently than the average young person.

(C)

Many people who rarely eat in fast‐food restaurants nevertheless eat regularly in restaurants.

(D)

The overall population of Canatria is growing steadily.

(E)

As the population of Canatria gets older, more people are eating at home.

Argument Evaluation
Situation

In Canatria, the older people get, the less likely they are to eat in fast‐food restaurants. The average
age of Canatrians is increasing.

Reasoning

What evidence would most weaken the support provided by the cited facts for the prediction that the
number of fast‐food restaurants in Canatria is likely to decrease? The argument implicitly reasons that
since studies have shown that Canatrians tend to eat in fast‐food restaurants less as they get older,
and since Canatrians are getting older on average, the proportion of Canatrians eating in fast‐food
restaurants will decline. The argument assumes that this means the overall number of fast‐food
restaurant customers will decline and that demand will decrease enough to reduce the number of
fast‐food restaurants that can sustain profitability. Consequently, fewer new fast‐food restaurants
will open or more old ones will close, or both. Thus, the number of fast‐food restaurants in
Canatria will fall. Any evidence casting doubt on any inference in this chain of implicit reasoning
will weaken the argument.

A

B
C
D

E

This strengthens the argument by providing additional evidence that the total number of fast‐food
restaurants will decrease. If the average number of customers per fast‐food restaurant is increasing, then
fewer fast‐food restaurants will be needed to serve the same—or a lesser—number of customers.
Even if a few individuals do not follow the general trends described, those trends could still reduce the
overall demand for and number of fast‐food restaurants.
The argument is only about fast‐food restaurants, not restaurants of other types.
Correct. This suggests that even if the proportion of Canatrians eating at fast‐food restaurants declines,
the total number doing so may not decline. Thus, the total demand for and profitability of fast‐food
restaurants may not decline either, so the total number of fast‐food restaurants in Canatria may not
decrease.
If anything, this strengthens the argument by pointing out an additional trend likely to reduce the demand
for, and thus the number of, fast‐food restaurants in Canatria.

The correct answer is D.

618

8.6 Critical Reasoning Answer Explanations

616. In order to withstand tidal currents, juvenile horseshoe crabs frequently burrow in the sand. Such burrowing discourages
barnacles from clinging to their shells. When fully grown, however, the crabs can readily withstand tidal currents without
burrowing, and thus they acquire substantial populations of barnacles. Surprisingly, in areas where tidal currents are very
weak, juvenile horseshoe crabs are found not to have significant barnacle populations, even though they seldom burrow.
Which of the following, if true, most helps to explain the surprising finding?
(A)

Tidal currents do not themselves dislodge barnacles from the shells of horseshoe crabs.

(B)

Barnacles most readily attach themselves to horseshoe crabs in areas where tidal currents are weakest.

(C)

The strength of the tidal currents in a given location varies widely over the course of a day.

(D)

A very large barnacle population can significantly decrease the ability of a horseshoe crab to find food.

(E)

Until they are fully grown, horseshoe crabs shed their shells and grow new ones several times a year.

Argument Construction
Situation

Juvenile horseshoe crabs withstand tidal currents by burrowing in the sand. This action makes
barnacles less likely to cling to their shells. Adult horseshoe crabs can withstand currents, so they
do not burrow, and barnacles become more likely to cling to their shells. Surprisingly, however,
juvenile horseshoe crabs that do not burrow, because tidal currents do not threaten them, do not
have significant numbers of barnacles clinging to their shells.

Reasoning

What would most help explain the finding that nonburrowing juvenile horseshoe crabs do not
have significant barnacle populations? The finding suggests that there is some way in which
nonburrowing juvenile horseshoe crabs either discourage barnacles from clinging to their shells,
or get rid of the barnacles that do cling to their shells. Identifying how this is accomplished will
explain the finding.

A
B
C
D

E

This gives a reason why juvenile horseshoe crabs that do not burrow would have significant barnacle
populations.
If barnacles in areas of weak tidal currents readily attach themselves to horseshoe crabs, then it would be
more likely for juvenile horseshoe crabs in such areas to have significant barnacle populations.
The areas under discussion are those where tidal currents are very weak. The strength of currents may vary
widely there, but presumably they are still weak compared to other areas.
The surprising finding under discussion is why certain juvenile horseshoe crabs do not have significant
barnacle populations, despite failing to engage in behavior that dislodges barnacles. That a very large
barnacle population can hurt a horseshoe crab does not help explain such a finding.
Correct. This statement properly identifies something that helps explain the surprising finding: If juvenile
horseshoe crabs regularly shed their shells, they also regularly shed the barnacles that cling to those shells.
Thus juvenile horseshoe crabs would most likely be found not to have significant barnacle populations.

The correct answer is E.

619

GMAT® Official Guide 2018

617. Last year a chain of fast‐food restaurants, whose menu had always centered on hamburgers, added its first vegetarian
sandwich, much lower in fat than the chain’s other offerings. Despite heavy marketing, the new sandwich accounts for a
very small proportion of the chain’s sales. The sandwich’s sales would have to quadruple to cover the costs associated with
including it on the menu. Since such an increase is unlikely, the chain would be more profitable if it dropped the sandwich.
Which of the following, if true, most seriously weakens the argument?
(A)

Although many of the chain’s customers have never tried the vegetarian sandwich, in a market research survey most
of those who had tried it reported that they were very satisfied with it.

(B)

Many of the people who eat at the chain’s restaurants also eat at the restaurants of competing chains and report no
strong preference among the competitors.

(C)

Among fast‐food chains in general, there has been little or no growth in hamburger sales over the past several years
as the range of competing offerings at other restaurants has grown.

(D)

When even one member of a group of diners is a vegetarian or has a preference for low‐fat food, the group tends to
avoid restaurants that lack vegetarian or low‐fat menu options.

(E)

An attempt by the chain to introduce a lower‐fat hamburger failed several years ago, since it attracted few new
customers and most of the chain’s regular customers greatly preferred the taste of the regular hamburgers.

Argument Evaluation
Situation

Last year a fast‐food restaurant chain specializing in hamburgers started offering a low‐fat
vegetarian sandwich and marketed it heavily. The new sandwich’s sales are far too low to cover the
costs associated with including it on the menu.

Reasoning

What evidence would most weaken the support provided by the cited facts for the prediction that it
would be more profitable for the chain to drop the sandwich? The implicit argument is that since the
new sandwich’s sales are too low to cover the costs associated with including it on the menu,
offering the sandwich diminishes the chain’s profitability and will continue to do so if the
sandwich continues to be offered. This reasoning assumes that the sandwich provides the chain
no substantial indirect financial benefits except through its direct sales. It also assumes that the
sandwich’s sales will not increase sufficiently to make the sandwich a viable product. Any evidence
casting doubt on either of these assumptions will weaken the argument.

A

B

C

D
E

This gives information only about the respondents to the survey who had tried the sandwich (possibly very
few), who were probably already more open to liking a vegetarian sandwich than any of the chain’s other
customers. So their responses are probably unrepresentative of the chain’s customers in general and do not
suggest that the sandwich has enough market potential.
Although the issue of competition with other restaurants is not raised in the information provided, this
new information, if anything, strengthens the argument, by suggesting that the introduction of the new
sandwich has not significantly enhanced customer preference for eating at the restaurants that offer the
new sandwich.
This suggests that the cause of stagnation in fast‐food restaurants’ hamburger sales has been competition
from non‐fast‐food restaurants, but not that the non‐fast‐food restaurants competed by offering vegetarian
options.
Correct. This suggests that even if the sandwich’s sales are low, it may indirectly increase the chain’s overall
profits by encouraging large groups to eat at the chain.
This strengthens the argument by suggesting that the chain’s customers are generally not interested in
low‐fat menu options such as the new sandwich.

The correct answer is D.

620

8.6 Critical Reasoning Answer Explanations

618. Transportation expenses accounted for a large portion of the total dollar amount spent on trips for pleasure by residents
of the United States in 1997, and about half of the total dollar amount spent on transportation was for airfare. However,
the large majority of United States residents who took trips for pleasure in 1997 did not travel by airplane but used other
means of transportation.
If the statements above are true, which of the following must also be true about United States residents who took trips for
pleasure in 1997?
(A)

Most of those who traveled by airplane did so because the airfare to their destination was lower than the cost of other
available means of transportation.

(B)

Most of those who traveled by airplane did so because other means of transportation to their destination were
unavailable.

(C)

Per mile traveled, those who traveled by airplane tended to spend more on transportation to their destination than did
those who used other means of transportation.

(D)

Overall, people who did not travel by airplane had lower average transportation expenses than people who did.

(E)

Those who traveled by airplane spent about as much, on average, on other means of transportation as they did on
airfare.

Argument Construction
Situation

In 1997, about half of total transportation spending by U.S. residents taking trips for pleasure was
for airfare. But the large majority of U.S. residents who took trips for pleasure in 1997 did not
travel by airplane.

Reasoning

What can be deduced from the stated facts? The information provided indicates that among U.S.
residents who took trips for pleasure in 1997, those who traveled by airplane were a small minority.
Yet this small minority’s spending for airfare accounted for half of all transportation spending
among residents taking trips for pleasure. It follows that on average, those who traveled by airplane
must have spent far more per person on transportation than those who did not travel by airplane.

A

B

C

D

E

This does not follow logically from the information given. Most of those who traveled by airplane may
have done so even if flying was more expensive than other modes of transportation—for example, because
flying was faster or more comfortable.
This does not follow from the information given. Most of those who traveled by airplane may have done
so even if many other modes of transportation were available—the other modes may all have been less
desirable.
This does not follow from the information given. Those who traveled by airplane may have traveled much
farther on average than those who used other means of transportation, so their transportation spending per
mile traveled need not have been greater.
Correct. As explained above, those who traveled by airplane must have spent more per person on
transportation than those who did not travel by airplane, on average. In other words, those who did not
travel by airplane must have had lower average transportation expenses than those who did.
This does not follow from the information given. Although half the total dollar spending on transportation
was for airfare, much of the transportation spending that was not for airfare was by the large majority of
U.S. residents who did not travel by airplane.

The correct answer is D.

621

GMAT® Official Guide 2018

619. Voters commonly condemn politicians for being insincere, but politicians often must disguise their true feelings when they
make public statements. If they expressed their honest views—about, say, their party’s policies—then achieving politically
necessary compromises would be much more difficult. Clearly, the very insincerity that people decry shows that our
government is functioning well.
Which of the following, if true, most seriously undermines this reasoning?
(A)

Achieving political compromises is not all that is necessary for the proper functioning of a government.

(B)

Some political compromises are not in the best long‐term interest of the government.

(C)

Voters often judge politicians by criteria other than the sincerity with which they express their views.

(D)

A political party’s policies could turn out to be detrimental to the functioning of a government.

(E)

Some of the public statements made by politicians about their party’s policies could in fact be sincere.

Argument Evaluation
Situation

Politicians must often make insincere public statements because expressing their true feelings
would make it harder for them to achieve politically necessary compromises.

Reasoning

What would suggest that the argument’s premises do not establish that politicians’ insincerity shows our
government is functioning well? The implicit reasoning is that insincerity helps politicians achieve
politically necessary compromises, and these compromises help our government to function well,
so insincerity must show that our government is functioning well. Evidence that these necessary
compromises do not ensure that our government functions well would undermine the argument’s
reasoning, as would evidence that politicians’ insincerity has other substantial effects that hinder
the government’s functioning.

A

B
C
D
E

Correct. If governments may function poorly even when insincerity allows necessary political
compromises to be made, then the argument’s premises do not establish that politicians’ insincerity shows
our government is functioning well.
The argument does not require that all political compromises help government to function well, only that
politically necessary compromises do.
Even if voters often judge politicians by criteria other than their sincerity, they may also often decry
politicians’ insincerity, not realizing or caring that such insincerity helps the government function well.
Even if a political party’s policies impair the government’s functioning, politically necessary compromises
by politicians in that party could improve the government’s functioning.
Even if politicians sometimes speak sincerely about their party’s policies, their general willingness to be
insincere as needed to achieve politically necessary compromises could be a sign that the government is
functioning well.

The correct answer is A.

622

8.6 Critical Reasoning Answer Explanations

620. To reduce waste of raw materials, the government of Sperland is considering requiring household appliances to be broken
down for salvage when discarded. To cover the cost of salvage, the government is planning to charge a fee, which would
be imposed when the appliance is first sold. Imposing the fee at the time of salvage would reduce waste more effectively,
however, because consumers tend to keep old appliances longer if they are faced with a fee for discarding them.
Which of the following, if true, most seriously weakens the argument?
(A)

Increasing the cost of disposing of an appliance properly increases the incentive to dispose of it improperly.

(B)

The fee provides manufacturers with no incentive to produce appliances that are more durable.

(C)

For people who have bought new appliances recently, the salvage fee would not need to be paid for a number of
years.

(D)

People who sell their used, working appliances to others would not need to pay the salvage fee.

(E)

Many nonfunctioning appliances that are currently discarded could be repaired at relatively little expense.

Evaluation of a Plan
Situation

A government is considering requiring household appliances to be broken down for salvage when
discarded. To cover the salvage costs, the government plans to charge a fee on appliance sales.

Reasoning

What would suggest that charging the fee at the time of salvage would less effectively reduce waste than
charging the fee at the time of sale would? The argument is that charging the fee at the time of
salvage would reduce waste of raw materials because it would encourage consumers to keep their
appliances longer before salvaging them. This argument could be weakened by pointing out other
factors that might increase waste if the fee is charged at the time of salvage or reduce waste if the
fee is charged at the time of sale.

A

B
C

D
E

Correct. This suggests that charging the fee at the time of salvage rather than the time of sale would
encourage consumers to discard their appliances illegally, thereby increasing waste of raw materials by
reducing the proportion of discarded appliances that are salvaged.
This factor would remain the same regardless of whether the fee was charged at the time of sale or the
time of salvage.
This might be a reason for consumers to prefer the fee be charged at the time of salvage rather than the
time of sale, but it does not suggest that charging the fee at the time of salvage would reduce waste less
effectively.
This provides an additional reason to expect that charging the fee at the time of salvage would help reduce
waste, so it strengthens rather than weakens the argument.
This would give consumers an additional reason to keep using their old appliances and postpone paying a
fee at the time of salvage, so it strengthens rather than weakens the argument.

The correct answer is A.

623

GMAT® Official Guide 2018

621. When there is less rainfall than normal, the water level of Australian rivers falls and the rivers flow more slowly. Because
algae whose habitat is river water grow best in slow‐moving water, the amount of algae per unit of water generally increases
when there has been little rain. By contrast, however, following a period of extreme drought, algae levels are low even in
very slow‐moving river water.
Which of the following, if true, does most to explain the contrast described above?
(A)

During periods of extreme drought, the populations of some of the species that feed on algae tend to fall.

(B)

The more slowly water moves, the more conducive its temperature is to the growth of algae.

(C)

When algae populations reach very high levels, conditions within the river can become toxic for some of the other
species that normally live there.

(D)

Australian rivers dry up completely for short intervals in periods of extreme drought.

(E)

Except during periods of extreme drought, algae levels tend to be higher in rivers in which the flow has been
controlled by damming than in rivers that flow freely.

Argument Construction
Situation

When Australian rivers flow slowly due to little rain, algae populations in those rivers increase. But
after periods of extreme drought, algae levels are low even in water moving at speeds that would
normally show population increases.

Reasoning

What would explain the contrast between algae levels in slow‐moving water resulting from little rain
and slow‐moving water after a drought? There must be some difference between what happens
during periods in which there is simply less rainfall than normal and periods in which there is
extreme drought, a difference that affects the algae population.

A
B
C
D

E

This indicates one of the consequences of drought, and slightly suggests that this might be due to a lower
algae level. But it does nothing to explain why algae levels might be lower after a drought.
This could explain why some rivers that are slow‐moving and have little water might have a high algae
level—but not why the algae level is low in such rivers after a period of drought.
This explains why levels of other species might be low when algae populations are high, not why algae
populations are high when there is little rain, but low following a period of extreme drought.
Correct. This statement properly identifies something that helps explain the contrast. According to the
information given, the habitat of the algae under discussion is river water. If the river dries up, the algae
will probably not survive. Then after the drought, algae population levels would likely take a while to rise
again.
This emphasizes that there is a contrast between what happens to algae during periods of extreme drought
and what happens to them at other times, but it does not help explain that contrast.

The correct answer is D.

624

8.6 Critical Reasoning Answer Explanations

622. Increased use of incineration is sometimes advocated as a safe way to dispose of chemical waste. But opponents of
incineration point to the 40 incidents involving unexpected releases of dangerous chemical agents that were reported just
last year at two existing incinerators commissioned to destroy a quantity of chemical waste material. Since designs for
proposed new incinerators include no additional means of preventing such releases, leaks will only become more prevalent
if use of incineration increases.
Which of the following, if true, most seriously weakens the argument?
(A)

At the two incinerators at which leaks were reported, staff had had only cursory training on the proper procedures for
incinerating chemical waste.

(B)

Other means of disposing of chemical waste, such as chemical neutralization processes, have not been proven safer
than incineration.

(C)

The capacity of existing incinerators is sufficient to allow for increased incineration of chemical waste without any
need for new incinerators.

(D)

The frequency of reports of unexpected releases of chemical agents at newly built incinerators is about the same as
the frequency at older incinerators.

(E)

In only three of the reported incidents of unexpected chemical leaks did the releases extend outside the property on
which the incinerators were located.

Argument Evaluation
Situation

Last year, at two chemical waste incinerators, there were forty reported incidents involving
unexpected releases of dangerous chemicals. Designs for proposed new incinerators include no
additional safeguards against such releases. Therefore, increased use of incineration will likely make
such releases more prevalent.

Reasoning

What would undermine the support provided for the conclusion that leaks will become more prevalent
if more chemical waste is disposed of through incineration? The argument draws a general conclusion
about chemical waste incineration from evidence about only two particular incinerators. This
reasoning would be undermined by any evidence that the leaks at those two incinerators were the
result of something other than insufficient safeguards against such releases.

A

B
C
D

E

Correct. If the staff training at the two incinerators was cursory, then the leaks may have been the results
of staff not knowing how to use safeguards with which the incinerators are equipped that, if properly used,
would have prevented the release of dangerous chemicals. Therefore, if staff at newer incinerators will be
better trained, leaks might not become more prevalent even if chemical waste incineration becomes more
common.
Other chemical waste disposal methods may be safer than incineration even if no one has proven so; and
even if they’re not safer overall, they may involve fewer leaks.
Continuing to use existing incinerators might well produce just as many leaks as switching to new
incinerators would.
This suggests that new incinerators produce as many leaks as older incinerators do, a finding that provides
additional evidence that increased incineration even with proposed new incinerators would lead to more
leaks.
The argument is not about how far the releases from leaks extend, only about how many of them are likely
to occur.

The correct answer is A.

625

GMAT® Official Guide 2018

623. Public health expert: Increasing the urgency of a public health message may be counterproductive. In addition to
irritating the majority who already behave responsibly, it may undermine all government pronouncements on health
by convincing people that such messages are overly cautious. And there is no reason to believe that those who
ignore measured voices will listen to shouting.
The two sections in boldface play which of the following roles in the public health expert’s argument?
(A)

The first is a conclusion for which support is provided, but is not the argument’s main conclusion; the second is an
unsupported premise supporting the argument’s main conclusion.

(B)

The first is a premise supporting the only explicit conclusion; so is the second.

(C)

The first is the argument’s main conclusion; the second supports that conclusion and is itself a conclusion for which
support is provided.

(D)

The first is a premise supporting the argument’s only conclusion; the second is that conclusion.

(E)

The first is the argument’s only explicit conclusion; the second is a premise supporting that conclusion.

Argument Construction
Situation

A public health expert argues against increasing the urgency of public health messages by pointing
out negative effects that may arise from such an increase, as well as by questioning its efficacy.

Reasoning

What roles are played in the argument by the two claims in boldface? The first claim in boldface states
that increasing the urgency of public health messages may be counterproductive. After making
this claim, the public health expert mentions two specific reasons this could be so: it could
irritate people who already behave responsibly, and it could convince people that all public health
messages are too cautious. (The latter reason in the second claim in boldface). The phrase [i]n
addition to indicates that neither claim in the second sentence is intended to support or explain
the other. However, since each claim in the second sentence gives a reason to believe the claim in
the first sentence, each independently supports the first sentence as a conclusion. The word [a]nd
beginning the third sentence reveals that its intended role in the argument is the same as that of
the two claims in the second sentence.

A
B
C

D
E

Everything stated after the first sentence is intended to help support it, so the first sentence is the
argument’s main conclusion.
Everything stated after the first sentence is intended to help support it, so the first sentence is a conclusion,
not a premise.
Each of the three claims in the second and third sentences is presented as an independent reason to accept
the general claim in the first sentence. Therefore, nothing in the passage is intended to support the second
statement in boldface as a conclusion.
Everything stated after the first sentence is intended to help support it, so the first sentence is a conclusion,
not a premise.
Correct. Each of the three claims in the second and third sentences is presented as an independent reason
to accept the general claim in the first sentence. Thus, each of those claims is a premise supporting the
claim in the first sentence as the argument’s only conclusion.

The correct answer is E.

626

8.6 Critical Reasoning Answer Explanations

624. Several industries have recently switched at least partly from older technologies powered by fossil fuels to new
technologies powered by electricity. It is thus evident that less fossil fuel is being used as a result of the operations of
these industries than would have been used if these industries had retained their older technologies.
Which of the following, if true, most strengthens the argument above?
(A)

Many of the industries that have switched at least partly to the new technologies have increased their output.

(B)

Less fossil fuel was used to manufacture the machinery employed in the new technologies than was originally used to
manufacture the machinery employed in the older technologies.

(C)

More electricity is used by those industries that have switched at least partly to the new technologies than by those
industries that have not switched.

(D)

Some of the industries that have switched at least partly to the new technologies still use primarily technologies that
are powered by fossil fuels.

(E)

The amount of fossil fuel used to generate the electricity needed to power the new technologies is less than the
amount that would have been used to power the older technologies.

Argument Evaluation
Situation

Several industries have now switched, at least partly, to technologies using electricity rather than
fossil fuels. Thus, less fossil fuel will be consumed as a result of the operation of these industries
than otherwise would have been.

Reasoning

Which option most strengthens the argument? One way to strengthen an argument is to eliminate or
minimize one of its flaws or weaknesses. Because the conclusion is stated in terms of “fossil fuel
consumed as a result of the operation of these industries,” the claim would encompass even any
fossil fuel that might be used to generate the electricity that the newer technologies use. Yet the
premise of the argument does not address this issue. So the argument is strengthened if it turns
out that less fossil fuel was used to produce the electricity than would have been used to power the
older technologies.

A

B

C
D

E

In an indirect way, this option slightly weakens rather than strengthens the argument. For if fossil fuels are
used to produce the electricity now used by the industries and if it is because of these newer technologies
that output has increased, the argument’s conclusion is less likely.
It does not matter how much fossil fuel was used to manufacture the older technologies originally. That has
no bearing on whether more fossil fuel would have been expended as a result of the continued operation of
the industries if the partial switch to newer technologies had not occurred.
This is what we would expect, but it in no way strengthens the argument.
This may seem to weaken the argument by indicating that the switch from older technologies will have
less of an impact on fossil fuel consumption by these industries than we might have assumed. But since the
conclusion makes no claim about how much consumption has been reduced, it is not clear that this option
has any bearing on the strength of the argument one way or the other.
Correct. This is the option that most strengthens the argument.

The correct answer is E.

627

GMAT® Official Guide 2018

625. The difference in average annual income in favor of employees who have college degrees, compared with those who do
not have such degrees, doubled between 1980 and 1990. Some analysts have hypothesized that increased competition
between employers for employees with college degrees drove up income for such employees.
Which of the following, if true, most seriously undermines the explanation described above?
(A)

During the 1980s a growing percentage of college graduates, unable to find jobs requiring a college degree, took
unskilled jobs.

(B)

The average age of all employees increased slightly during the 1980s.

(C)

The unemployment rate changed very little throughout most of the 1980s.

(D)

From 1980 to 1990 the difference in average income between employees with advanced degrees and those with
bachelor’s degrees also increased.

(E)

During the 1980s there were some employees with no college degree who earned incomes comparable to the top
incomes earned by employees with a college degree.

Argument Evaluation
Situation

The amount by which average annual income for employees with college degrees exceeds that for
employees without such degrees doubled between 1980 and 1990.

Reasoning

What evidence would most strongly suggest that increased competition among employers for employees
with college degrees does not explain the relative increase in those employees’ incomes? Such increased
competition could not explain the relative increase in income for employees with college degrees
if the competition did not actually increase, or if such competition occurred but did not result
in employers paying higher wages or salaries, or if the increase in competition to hire employees
without college degrees was even greater. So evidence that any of those conditions existed would
undermine the analysts’ explanation.

A
B

C
D

E

Correct. This suggests that the supply of college graduates grew relative to employers’ demand for them,
and hence that employers’ competition for college‐educated employees did not actually increase.
The average age might have increased equally for employees with college degrees and for those without
them, so the increase is not clearly relevant to explaining why the difference between these two groups’
average incomes grew.
Even if the overall unemployment rate did not change, competition for college‐educated employees could
have increased while competition for other employees decreased.
This statement gives information comparing income trends among two groups of those with college
degrees, and is irrelevant to the comparison of income trends for those with college degrees and those
without college degrees.
Even if there was strong competition and high pay for certain unusual types of employees without college
degrees, increasing competition for employees with college degrees might have explained the overall
growing difference in average pay between employees with college degrees and those without.

The correct answer is A.

628

8.6 Critical Reasoning Answer Explanations

626. Which of the following most logically completes the passage?
According to the last pre-election poll in Whippleton, most voters believe that the three problems government needs to address,
in order of importance, are pollution, crime, and unemployment. Yet in the election, candidates from parties perceived as strongly
against pollution were defeated, while those elected were all from parties with a history of opposing legislation designed to reduce
pollution. These results should not be taken to indicate that the poll was inaccurate, however, since __________.
(A)

some voters in Whippleton do not believe that pollution needs to be reduced

(B)

every candidate who was defeated had a strong antipollution record

(C)

there were no issues other than crime, unemployment, and pollution on which the candidates had significant
differences of opinion

(D)

all the candidates who were elected were perceived as being stronger against both crime and unemployment than the
candidates who were defeated

(E)

many of the people who voted in the election refused to participate in the poll

Argument Construction
Situation

A pre-election poll indicated that most voters believed the three problems government needs
to address, in order of importance, are pollution, crime, and unemployment. But in the election,
candidates from parties with a history of opposing anti-pollution legislation beat candidates from
parties perceived as more strongly against pollution.

Reasoning

What would most help explain how the poll might have been accurate despite the election results? Since
the poll indicated that voters were most concerned about pollution, it suggested that candidates
from anti-pollution parties would be more likely to be elected, other things being equal—and yet
those candidates were not elected. There are many possible explanations for this outcome that
are compatible with the poll having been accurate. For example, voters might have been swayed
by the candidates’ personalities, qualifications, or advertising more than by their positions on the
issues. Or some candidates might have convinced voters that their personal positions on the issues
were different from those of their parties. Or voters might have chosen candidates based on their
positions on crime and unemployment, considering those issues together more important than
pollution alone. Any statement suggesting that any such factors explained the election results
would logically complete the passage by providing a reason to believe that the poll could have been
accurate despite those results.

A

If the number of voters who did not believe that pollution needed to be reduced was large enough to
explain the election results, then the poll was probably inaccurate. So this does not explain how the poll
might have been accurate despite those results.
B
This eliminates the possibility that candidates were defeated for having weak antipollution records
conflicting with their parties’ antipollution stances, so it eliminates one explanation of how the poll might
have been accurate despite the election results. Thus, it slightly weakens the conclusion of the argument
instead of providing a premise to support it.
C This eliminates the possibility that differences of opinion among the candidates on these other issues
might explain the election results, but it does not explain how the poll could have been accurate despite the
election results.
D Correct. The poll indicated that voters believed that the government needs to address crime and
unemployment as well as pollution. So if the poll was accurate, the election outcome might have resulted
from voters considering candidates’ positions on crime and unemployment to be jointly more important
than their positions on pollution.
E If anything, this provides a reason to doubt that the poll accurately reflected voters’ opinions. It does not
explain how the poll might have accurately reflected those opinions despite the election results.
The correct answer is D.
629

GMAT® Official Guide 2018

627. Manufacturing plants in Arundia have recently been acquired in substantial numbers by investors from abroad. Arundian
politicians are proposing legislative action to stop such investment, justifying the proposal by arguing that foreign investors,
opportunistically exploiting a recent fall in the value of the Arundian currency, were able to buy Arundian assets at less than
their true value.
Which of the following, if true, casts the most serious doubt on the adequacy of the Arundian politicians’ justification for the
proposed legislation?
(A)

The Arundian government originally welcomed the fall in the value of the Arundian currency because the fall made
Arundian exports more competitive on international markets.

(B)

Foreign investors who acquired Arundian manufacturing plants generally did so with no intention of keeping and
running those plants over the long term.

(C)

Without the recent fall in the value of the Arundian currency, many of the Arundian assets bought by foreign investors
would have been beyond the financial reach of those investors.

(D)

In Concordia, a country broadly similar to Arundia, the share of manufacturing assets that is foreign-controlled is
60 percent higher than it is in Arundia.

(E)

The true value of an investment is determined by the value of the profits from it, and the low value of the Arundian
currency has depressed the value of any profits earned by foreign investors from Arundian assets.

Argument Evaluation
Situation

After a recent fall in the value of Arundian currency, foreign investors have been acquiring many
Arundian manufacturing plants. Arundian politicians are proposing legislation to stop such
investment.

Reasoning

What would most undermine the Arundian politicians’ justification for the proposed legislation? The
politicians are justifying their proposal by claiming that foreign investors have been exploiting the
fall in the currency’s value by buying Arundian assets at less than their true value (whatever that
means). Any evidence that their claim is false or meaningless would undermine their justification
for the proposal, as would any evidence that the claim, even if true, does not provide a good reason
to stop the foreign investments.

A

This suggests that the foreign investors got a good deal on the manufacturing plants, since it provides
evidence that those plants will now be more competitive and profitable. So, if anything, it supports the
politicians’ justification for their proposal rather than undermining it.
B
This suggests that the foreign investors generally believe the manufacturing plants are undervalued,
and intend to sell them at a profit as soon as the currency rises enough. So it supports the politicians’
justification for their proposal rather than undermining it.
C This suggests that the recent fall in the currency’s value made Arundian assets cost less than usual for
foreign investors, thus arguably allowing the investors to buy the assets at less than their true value. So, if
anything, it supports the politicians’ justification for their proposal rather than undermining it.
D The Arundian politicians might consider the example of Concordia to be a warning of the disaster that
could befall Arundia unless the legislation is enacted. So the situation in Concordia might be cited as
support for the politicians’ justification of their proposal.
E Correct. This implies that the fall in the Arundian currency’s value has reduced the true value of Arundian
manufacturing plants and any profits they may make, so it undermines the politicians’ claim that the
foreign investors exploited the fall in the currency’s value to acquire the plants for less than their true value.
The correct answer is E.

630

8.6 Critical Reasoning Answer Explanations

628. Proposed new safety rules for the Beach City airport would lengthen considerably the minimum time between takeoffs from
the airport. In consequence, the airport would be able to accommodate 10 percent fewer flights than currently use the
airport daily. The city’s operating budget depends heavily on taxes generated by tourist spending, and most of the tourists
come by plane. Therefore, the proposed new safety rules, if adopted, will reduce the revenue available for the operating
budget.
The argument depends on assuming which of the following?
(A)

There are no periods of the day during which the interval between flights taking off from the airport is significantly
greater than the currently allowed minimum.

(B)

Few, if any, of the tourists who use the Beach City airport do so when their main destination is a neighboring
community and not Beach City itself.

(C)

If the proposed safety rules are adopted, the reduction in tourist numbers will not result mainly from a reduction in the
number of tourists who spend relatively little in Beach City.

(D)

Increasing the minimum time between takeoffs is the only way to achieve necessary safety improvements without a
large expenditure by the city government on airport enhancements.

(E)

The response to the adoption of the new safety rules would not include an increase in the number of passengers per
flight.

Argument Construction
Situation

Proposed safety rules for a city airport would reduce the number of daily flights the airport can
accommodate. The city’s operating budget depends heavily on taxes generated by tourists, who
mostly come by plane. Therefore, adopting the safety rules will result in lower revenue available for
the operating budget.

Reasoning

What must be true in order for the cited facts to support the conclusion that the proposed rules would
reduce the revenue for the operating budget? The implicit reasoning is that since the rules would
reduce the number of flights that can be accommodated, they would thereby reduce the number
of tourists arriving by plane, which in turn would reduce the tax revenue that tourist spending
generates for the operating budget. This assumes that the actual number of daily flights would fall
along with the number that the airport can accommodate; that fewer daily flights would mean
fewer people flying into the airport; that fewer people flying into the airport would mean fewer
tourists flying into the airport; that fewer tourists flying into the airport would mean fewer tourists
visiting the city; that fewer tourists visiting the city would mean less taxable spending by tourists;
and that less taxable spending by tourists would mean less revenue overall for the operating
budget.

A
B
C

D

E

Even if flights depart the airport less frequently during some periods of the day, increasing the minimum
time between flights at busy times of day could reduce the total number of daily flights from the airport.
Even if half the tourists flying into the airport were bound for other nearby towns, the other half could still
spend enough in town to generate lots of revenue for the operating budget.
It is possible that most tourists spend relatively little in the city, but a few spend a lot. In that case, even if
a reduction in tourist numbers resulted mainly from a declining number of tourists who spend relatively
little, it could also greatly reduce the already small number of tourists who spend a lot.
This suggests that the proposed rules might be financially better for the city than any alternative
way to improve safety, whereas the argument’s conclusion is that the proposed rules are financially
disadvantageous.
Correct. If adopting the proposed rules would result in a large increase in the number of passengers per
flight, fewer daily flights would not necessarily mean fewer passengers or fewer tourists overall.

The correct answer is E.
631

GMAT® Official Guide 2018

629. The introduction of new drugs into the market is frequently prevented by a shortage of human subjects for the clinical trials
needed to show that the drugs are safe and effective. Since the lives and health of people in future generations may depend
on treatments that are currently experimental, practicing physicians are morally in the wrong when, in the absence of any
treatment proven to be effective, they fail to encourage suitable patients to volunteer for clinical trials.
Which of the following, if true, casts most doubt on the conclusion of the argument?
(A)

Many drugs undergoing clinical trials are intended for the treatment of conditions for which there is currently no
effective treatment.

(B)

Patients do not share the physician’s professional concern for public health, but everyone has a moral obligation to
alleviate suffering when able to do so.

(C)

Usually, half the patients in a clinical trial serve as a control group and receive a nonactive drug in place of the drug
being tested.

(D)

An experimental drug cannot legally be made available to patients unless those patients are subjects in clinical trials
of the drug.

(E)

Physicians have an overriding moral and legal duty to care for the health and safety of their current patients.

Argument Evaluation
Situation

A shortage of human subjects for clinical trials needed to show that new drugs are safe and
effective often prevents those drugs from being introduced into the market. The lives and health of
future generations may depend on treatments that are now experimental.

Reasoning

What would cast doubt on the judgment that doctors are morally obligated to encourage their patients
to volunteer for clinical trials? Note that the argument’s conclusion, unlike its premises, is a moral
judgment. This judgment could be cast into doubt by a moral principle that would be likely to
conflict with it under the conditions described. For example, a principle suggesting that it is
sometimes morally unacceptable for doctors to encourage their patients to volunteer for clinical
trials would also suggest that they are not morally obligated to encourage their patients to
volunteer for clinical trials, since anything morally obligatory must also be morally acceptable.

A
B

C

D
E

If anything, this highlights how important it is to ensure that these drugs undergo clinical trials to benefit
future generations, so it supports rather than casts doubt on the argument’s conclusion.
This suggests that patients are morally obligated to volunteer for clinical trials to help prevent suffering
in future generations. If anything, this supports the claim that doctors are morally obligated to encourage
their patients to volunteer.
The clinical trial will probably not harm any patients in the control group, yet their participation will
benefit future generations. So, if anything, this supports the claim that doctors should encourage their
patients to volunteer.
This legal barrier makes it even more essential for the drugs to undergo clinical trials in order to benefit
patients, so it supports rather than casts doubt on the argument’s conclusion.
Correct. Since the experimental drugs’ safety is being tested during the trials, the drugs may prove unsafe
for subjects in the trials. If doctors have an overriding moral duty to keep their current patients safe, then it
may be morally unacceptable for them to encourage those patients to volunteer for the trials.

The correct answer is E.

632

8.6 Critical Reasoning Answer Explanations

630. As a construction material, bamboo is as strong as steel and sturdier than concrete. Moreover, in tropical areas bamboo is
a much less expensive construction material than either steel or concrete and is always readily available. In tropical areas,
therefore, building with bamboo makes better economic sense than building with steel or concrete, except where land
values are high.
Which of the following, if true, most helps to explain the exception noted above?
(A)

Buildings constructed of bamboo are less likely to suffer earthquake damage than are steel and concrete buildings.

(B)

Bamboo is unsuitable as a building material for multistory buildings.

(C)

In order to protect it from being damaged by termites and beetles, bamboo must be soaked, at some expense, in a
preservative.

(D)

In some tropical areas, bamboo is used to make the scaffolding that is used during large construction projects.

(E)

Bamboo growing in an area where land values are increasing is often cleared to make way for construction.

Argument Construction
Situation

Bamboo is as strong as steel and sturdier than concrete when used as a construction material. In
tropical areas, bamboo is much less expensive and is always readily available.

Reasoning

What explains the exception specified in the conclusion? The argument’s conclusion is that in tropical
areas bamboo is a more economical building material than steel or concrete, except where land
values are high. The information in the passage makes clear why bamboo is a more economical
building material in tropical areas than are concrete or steel. So the question is: Why must an
exception be made for areas where land values are high? Multistory buildings are particularly
desirable in areas where land values are high, but bamboo may not be suitable for such buildings.

A

B

C
D

E

This explains why bamboo would be preferable to steel or concrete in tropical areas especially prone to
earthquakes. However, there is no clear connection to be made between areas where land values are high
and areas especially prone to earthquakes.
Correct. Multistory buildings provide a greater area of floor space for a given site area, and in that sense
are more economical. A single‐story building with the same floor space will occupy a much bigger site, so
the higher the land values, the more likely it is that a multistory building will be built on that land. Thus,
given this information, bamboo is less suitable for areas where land values are high.
This undermines, to some extent, the claim that bamboo is an economical building material. But it does
nothing to explain why it would be less economical specifically in areas where land values are high.
This is irrelevant. Bamboo is used to build scaffolding for construction projects and as a building material
for permanent structures. There is no way to infer from this that bamboo is less economical specifically in
areas where land values are high.
The fact that bamboo is cleared from an area to make room for construction in no way implies that
bamboo would not be a suitable and economical building material for the area once it has been cleared.

The correct answer is B.

633

GMAT® Official Guide 2018

631. Newspaper editors should not allow reporters to write the headlines for their own stories. The reason for this is that, while
the headlines that reporters themselves write are often clever, what typically makes them clever is that they allude to littleknown information that is familiar to the reporter but that never appears explicitly in the story itself.
Which of the following, if true, most strengthens the argument?
(A)

The reporter who writes a story is usually better placed than the reporter’s editor is to judge what the story’s most
newsworthy features are.

(B)

To write a headline that is clever, a person must have sufficient understanding of the story that the headline
accompanies.

(C)

Most reporters rarely bother to find out how other reporters have written stories and headlines about the same events
that they themselves have covered.

(D)

For virtually any story that a reporter writes, there are at least a few people who know more about the story’s subject
matter than does the reporter.

(E)

The kind of headlines that newspaper editors want are those that anyone who has read a reporter’s story in its entirety
will recognize as clever.

Argument Evaluation
Situation

The headlines newspaper reporters write for their own stories are often clever only because they
allude to little-known information that never appears explicitly in the stories themselves.

Reasoning

What would most help the argument support the conclusion that newspaper editors should not allow
reporters to write headlines for their own stories? The argument’s only explicit premise is that the
headlines newspaper reporters write for their own stories are often clever only because they allude
to little-known information that never appears explicitly in the stories themselves. In order for
this premise to support the conclusion that newspaper editors should not allow reporters to write
their own headlines, it would be helpful to be given a reason why editors should avoid headlines
alluding to such little-known information.

A

This suggests that reporters are likely to write better headlines for their stories than editors are, so it
weakens the argument that editors should not allow reporters to write their own headlines.
B
Since a reporter who wrote a story is likely to understand that story well, this does not provide a reason
why editors should not allow reporters to write their own headlines.
C If most reporters did what is suggested, they could perhaps hone their headline-writing skills—unless
almost all reporters are weak in such skills, as suggested in the given information. The fact that they do not
bother to do so may help explain why reporters’ headline-writing skills are weak. An explanation of why
this is so does not provide additional support for the argument’s conclusion.
D The people who know more about a story’s subject matter than the reporter writing the story might be just
as likely to see the cleverness of allusions to little-known information as the reporters are. So, to the extent
that this is relevant at all, it slightly weakens the argument by suggesting that obscurely clever headlines
sometimes function as intended.
E Correct. The argument’s explicit premise suggests that typically a reporter’s headline for his or her own
story cannot be recognized as clever by a reader who has read the whole story. So if editors want headlines
that anyone who has read the accompanying stories would recognize as clever, they have a reason not to let
reporters write the headlines.
The correct answer is E.

634

8.6 Critical Reasoning Answer Explanations

632. Scientists have modified feed corn genetically, increasing its resistance to insect pests. Farmers who tried out the
genetically modified corn last season applied less insecticide to their corn fields and still got yields comparable to those
they would have gotten with ordinary corn. Ordinary corn seed, however, costs less, and what these farmers saved on
insecticide rarely exceeded their extra costs for seed. Therefore, for most feed‐corn farmers, switching to genetically
modified seed would be unlikely to increase profits.
Which of the following would it be most useful to know in order to evaluate the argument?
(A)

Whether there are insect pests that sometimes reduce feed‐corn yields, but against which commonly used
insecticides and the genetic modification are equally ineffective

(B)

Whether the price that farmers receive for feed corn has remained steady over the past few years

(C)

Whether the insecticides typically used on feed corn tend to be more expensive than insecticides typically used on
other crops

(D)

Whether most of the farmers who tried the genetically modified corn last season applied more insecticide than was
actually necessary

(E)

Whether, for most farmers who plant feed corn, it is their most profitable crop

Argument Evaluation
Situation

Farmers who grew feed corn genetically engineered to be pest resistant got yields comparable
to those of farmers growing ordinary feed corn, but did so while using less pesticide. Since the
amount saved on pesticide was rarely in excess of the extra costs for the genetically modified corn,
most farmers will probably not increase profits by choosing the genetically engineered variety.

Reasoning

Which would be most useful to know in evaluating the argument? To answer a question such as this,
one should look for information that would strengthen or weaken the argument. If one had
information that the farmers growing the genetically modified corn could have increased their
yields last year at lower cost, this would be helpful in evaluating the argument, because this would
show that the argument is weak.

A

B
C
D

E

It does not matter to the argument whether there are pests against which pesticides and genetic resistance
are equally ineffective, because that is compatible with there being pests against which they are not equally
effective.
Whether prices of feed corn go up or down affects the comparison groups equally.
The relative cost of insecticides for other crops has no bearing on the argument because the argument is
concerned with only feed corn.
Correct. This option provides the information that it would be most useful to know in evaluating the
argument. It shows that farmers growing genetically modified corn last year could have attained higher
profits than they in fact did.
The argument concerns only the relative profitability of growing one variety of feed corn versus another.

The correct answer is D.

635

GMAT® Official Guide 2018

633. Debater: The average amount of overtime per month worked by an employee in the manufacturing division of the Haglut
Corporation is 14 hours. Most employees of the Haglut Corporation work in the manufacturing division. Furthermore, the
average amount of overtime per month worked by any employee in the company generally does not fluctuate much from
month to month. Therefore, each month, most employees of the Haglut Corporation almost certainly work at least some
overtime.
The debater’s argument is most vulnerable to criticism on which of these grounds?
(A)

It takes for granted that the manufacturing division is a typical division of the corporation with regard to the average
amount of overtime its employees work each month.

(B)

It takes for granted that if a certain average amount of overtime is worked each month by each employee of the
Haglut Corporation, then approximately the same amount of overtime must be worked each month by each employee
of the manufacturing division.

(C)

It confuses a claim from which the argument’s conclusion about the Haglut Corporation would necessarily follow with a
claim that would follow from the argument’s conclusion only with a high degree of probability.

(D)

It overlooks the possibility that even if, on average, a certain amount of overtime is worked by the members of some
group, many members of that group may work no overtime at all.

(E)

It overlooks the possibility that even if most employees of the corporation work some overtime each month, any one
corporate employee may, in some months, work no overtime.

Argument Evaluation
Situation

Most of the employees of the Haglut Corporation work in the manufacturing division, where
employees average 14 hours per month in overtime. The average amount of overtime per month
for employees at Haglut does not fluctuate much from month to month.

Reasoning

What is the argument’s greatest weakness? The argument’s conclusion is that almost certainly each
month most of the employees of Haglut work at least some overtime. Answer choice (D) identifies
the argument’s greatest weakness because it points out how the conclusion of the argument could
be false even if all of the supporting information were true. For example, it could be that less than
half of the employees work any overtime at all, but those that do work overtime work much more
than 14 hours per month.

A

B
C

D

E

The argument leaves open the possibility that in some divisions of the corporation, the average monthly
overtime of its employees is quite different from 14 hours, even if (as the argument states) that average
does not change much from month to month.
The argument does not assume that there is a monthly amount of overtime worked by each employee of
the manufacturing division equivalent to the company‐wide average monthly overtime per employee.
This does not identify a weakness that can be detected in the argument. Since the claims mentioned here
are not specified, the passage provides no evidence that clearly indicates that this type of confusion is
playing a role in the argument.
Correct. The argument ignores the possibility that most of the employees of Haglut work no overtime at
all in a particular month—which is quite consistent with the argument’s assertion that the average number
of monthly overtime hours per employee within the manufacturing division is 14.
The possibility described by this is not overlooked by the argument, because this possibility is consistent
with the conclusion. It could easily be that most employees of the corporation work some overtime each
month—as the conclusion envisions—but that there are always some employees who do not work any
overtime.

The correct answer is D.

636

8.6 Critical Reasoning Answer Explanations

634. Proponents of the recently introduced tax on sales of new luxury boats had argued that a tax of this sort would be an
equitable way to increase government revenue because the admittedly heavy tax burden would fall only on wealthy
people and neither they nor anyone else would suffer any economic hardship. In fact, however, 20 percent of the workers
employed by manufacturers of luxury boats have lost their jobs as a direct result of this tax.
The information given, if true, most strongly supports which of the following?
(A)

The market for luxury boats would have collapsed even if the new tax on luxury boats had been lower.

(B)

The new tax would produce a net gain in tax revenue for the government only if the yearly total revenue that it
generates exceeds the total of any yearly tax-revenue decrease resulting from the workers’ loss of jobs.

(C)

Because many people never buy luxury items, imposing a sales tax on luxury items is the kind of legislative action that
does not cost incumbent legislators much popular support.

(D)

Before the tax was instituted, luxury boats were largely bought by people who were not wealthy.

(E)

Taxes can be equitable only if their burden is evenly distributed over the entire population.

Argument Construction
Situation

Proponents of a recently introduced tax on sales of new luxury boats argued that it would be an
equitable way to increase government revenue because the tax would fall only on the wealthy and
cause no economic hardship. But because of the tax, 20 percent of luxury-boat manufacturing
workers have lost their jobs.

Reasoning

What conclusion do the statements about the proponents’ argument and the tax’s effects support? Since the
tax caused many workers to lose their jobs, apparently the proponents were incorrect in asserting
that it would cause no one to suffer any economic hardship. Thus, their justification for concluding
that the tax is an equitable way to increase government revenue is factually inaccurate, casting
doubt on that conclusion.

A

B

C
D

E

The passage indicates that the tax directly caused a significant decrease (though not necessarily a collapse)
in the market for luxury boats. But the passage contains no evidence about whether such a decrease might
not have occurred if the new tax had been somewhat lower.
Correct. Since the tax caused the workers to lose their jobs, it might have made the government lose
revenue from payroll taxes that the laid-off workers would have paid if they had kept their jobs. So if the
yearly total revenue generated directly and indirectly by the tax were less than those total yearly payroll
taxes and any other tax revenue that was lost as a result of the tax, the tax would have caused a net loss in
tax revenue.
The passage contains no information about what types of legislative actions cost, or do not cost, incumbent
legislators popular support.
Although the passage suggests that some of the tax proponents’ assumptions were wrong, it contains no
information suggesting that those proponents were wrong in thinking that luxury boats are purchased
mainly by wealthy people.
The passage does not provide any basis for determining what makes a tax equitable or about whether the
luxury boat tax is equitable. The tax’s proponents evidently felt that a tax whose burden falls only on the
wealthy rather than evenly on the entire population can be equitable.

The correct answer is B.

637

GMAT® Official Guide 2018

635. In Wareland last year, 16 percent of licensed drivers under 21 and 11 percent of drivers ages 21–24 were in serious
accidents. By contrast, only 3 percent of licensed drivers 65 and older were involved in serious accidents. These figures
clearly show that the greater experience and developed habits of caution possessed by drivers in the 65-and-older group
make them far safer behind the wheel than the younger drivers are.
Which of the following is an assumption on which the argument depends?
(A)

Drivers 65 and older do not, on average, drive very many fewer miles per year than drivers 24 and younger.

(B)

Drivers 65 and older do not constitute a significantly larger percentage of licensed drivers in Wareland than drivers
ages 18–24 do.

(C)

Drivers 65 and older are less likely than are drivers 24 and younger to drive during weather conditions that greatly
increase the risk of accidents.

(D)

The difference between the accident rate of drivers under 21 and of those ages 21–24 is attributable to the greater
driving experience of those in the older group.

(E)

There is no age bracket for which the accident rate is lower than it is for licensed drivers 65 and older.

Argument Evaluation
Situation

Last year in Wareland, a much higher percentage of drivers 24 and under than of drivers 65 and
older were in serious accidents.

Reasoning

What must be true for the observation about the accident rates to support the conclusion that the greater
experience and caution of drivers 65 and older make them safer behind the wheel than the younger
drivers? Several factors other than greater experience and caution could explain the lower accident
rate among the older drivers. For example, the older drivers might simply drive much less than the
younger ones, but still get in just as many accidents per mile driven. Or perhaps because the older
drivers are more often retired, their schedules less often lead them to drive at times of day when
accident rates are greater for everyone. Or they might be more likely to live in rural areas with less
traffic and lower accident rates. The argument depends on assuming that none of these factors fully
explains the difference in accident rates.

A

B

C

D

E

Correct. Although we are given no information about the possible extent of any difference in average
miles driven, the (somewhat vague) information that drivers 65 and older drive very many fewer miles per
year, on average, than drivers 24 and younger would cast serious doubt on the statistical argument given.
The argument assumes that the difference in miles driven is not sufficiently substantial to undermine the
argument.
The argument is only about the discrepancy between the percentages of the drivers in two specific age
groups who were in serious accidents last year. The percentages of licensed drivers who fall in these age
groups are irrelevant.
Even if drivers 65 and older are just as likely as younger drivers to drive in inclement weather, they may do
so far more carefully than the younger drivers, so the older drivers’ greater experience and caution could
still explain their lower accident rates.
Even if greater experience does not explain the difference between the accident rates of the two younger
groups of drivers, it might still explain the differences between the accident rate of those two younger
groups taken together and that of drivers aged 65 and older.
The accident rate could be lower for drivers in late middle age than for those 65 and older because drivers
in late middle age are also cautious and experienced, but their reflexes and vision tend to be less impaired.
Even if that were true, the experience and caution of the drivers 65 and older might still make them safer
than drivers 24 and under.

The correct answer is A.

638

8.6 Critical Reasoning Answer Explanations

636. In the past the country of Malvernia has relied heavily on imported oil. Malvernia recently implemented a program to convert
heating systems from oil to natural gas. Malvernia currently produces more natural gas each year than it uses, and oil
production in Malvernian oil fields is increasing at a steady pace. If these trends in fuel production and usage continue,
therefore, Malvernian reliance on foreign sources for fuel is likely to decline soon.
Which of the following would it be most useful to establish in evaluating the argument?
(A)

When, if ever, will production of oil in Malvernia outstrip production of natural gas?

(B)

Is Malvernia among the countries that rely most on imported oil?

(C)

What proportion of Malvernia’s total energy needs is met by hydroelectric, solar, and nuclear power?

(D)

Is the amount of oil used each year in Malvernia for generating electricity and fuel for transportation increasing?

(E)

Have any existing oil‐burning heating systems in Malvernia already been converted to natural‐gas‐burning heating
systems?

Argument Evaluation
Situation

Malvernia has relied heavily on imported oil, but recently began a program to convert heating
systems from oil to natural gas. Malvernia produces more natural gas than it uses, so it will
probably reduce its reliance on imported oils if these trends continue.

Reasoning

Which option provides the information that it would be most useful to know in evaluating the argument?
In other words, we are looking for the option which—depending on whether it was answered
yes or no—would either most weaken or most strengthen the argument. The argument indicates
that Malvernia will be using less oil for heating and will be producing more oil domestically. But
the conclusion that Malvernia’s reliance on foreign oil will decline, assuming the current trends
mentioned continue, would be seriously undermined if there was something in the works that was
bound to offset these trends, for instance, if it turned out that the country’s need for oil was going
to rise drastically in the coming years.

A
B
C
D
E

Since both counteract the need for imported oil, it makes little difference to the argument whether
domestic oil production exceeds domestic natural gas.
Whether there are many countries that rely more on foreign oil than Malvernia would have little impact
on whether Malvernia’s need for foreign oil can be expected to decline.
Since there is no information in the argument about whether Malvernia can expect an increase or decrease
from these other energy sources, it does not matter how much they now provide.
Correct. This option provides the information that it would be most useful to know in evaluating the
argument.
The argument tells us that a program has begun recently to convert heating systems from oil to gas. So,
even if no such conversions have been completed, the argument still indicates that they can be expected to
occur.

The correct answer is D.

639

GMAT® Official Guide 2018

637. Exposure to certain chemicals commonly used in elementary schools as cleaners or pesticides causes allergic reactions
in some children. Elementary school nurses in Renston report that the proportion of schoolchildren sent to them for
treatment of allergic reactions to those chemicals has increased significantly over the past ten years. Therefore, either
Renston’s schoolchildren have been exposed to greater quantities of the chemicals, or they are more sensitive to them
than schoolchildren were ten years ago.
Which of the following is an assumption on which the argument depends?
(A)

The number of school nurses employed by Renston’s elementary schools has not decreased over the past ten years.

(B)

Children who are allergic to the chemicals are no more likely than other children to have allergies to other substances.

(C)

Children who have allergic reactions to the chemicals are not more likely to be sent to a school nurse now than they
were ten years ago.

(D)

The chemicals are not commonly used as cleaners or pesticides in houses and apartment buildings in Renston.

(E)

Children attending elementary school do not make up a larger proportion of Renston’s population now than they did
ten years ago.

Argument Construction
Situation

Some children have allergic reactions to some of the chemicals commonly used in elementary
schools as cleaners and pesticides. The number of children sent to elementary school nurses in
Renston for allergic reactions to such chemicals has risen significantly over the past ten years.

Reasoning

What must the argument assume? The argument’s conclusion presents just two alternatives: either
the children are exposed to more of the chemicals than children in earlier years or they are more
sensitive. But there is a third possible explanation for the significant increase in school‐nurse visits
that the school nurses have reported: that children are just more inclined to go to the school nurse
when they experience an allergic reaction than were children several years ago. For the conclusion
to follow from its premises, the argument must assume that this is not the correct explanation.

A

B

C

D

E

If the number of elementary school nurses in Renston elementary schools had decreased over the past ten
years, that would in no way explain the rise in the proportion of children reporting to school nurses for
allergic reactions.
Only school‐nurse visits for allergic reactions to the cleaners and pesticides used in elementary schools
are in question in the argument. Of course there could be school‐nurse visits for allergic reactions to other
things, but that issue does not arise in the argument.
Correct. This can be seen by considering whether the argument would work if we assume that this were
false, i.e., that a school‐nurse visit is more likely in such cases. As noted above, this provides an alternative
to the two explanations that the conclusion claims are the sole possibilities.
This does not need to be assumed by the argument. The argument’s conclusion suggests that children may
in recent years have had greater exposure to the chemicals, not that this exposure has occurred exclusively
in the schools. The argument does not rely on this latter assumption.
The argument does not need to make this assumption. The argument is framed in terms of proportions
of children having school‐nurse visits for certain allergic reactions. How many children there are or what
proportion such children are of Renston’s total population is not directly relevant to the argument.

The correct answer is C.

640

8.6 Critical Reasoning Answer Explanations

638. Normally, the pineal gland governs a person’s sleep‐wake cycle by secreting melatonin in response to the daily cycle of light
and darkness as detected by the eye. Nonetheless, many people who are totally blind due to lesions in the visual cortex of
the brain easily maintain a 24‐hour sleep‐wake cycle. So the neural pathway by which the pineal gland receives information
from the eye probably does not pass through the visual cortex.
For purposes of evaluating the argument it would be most useful to establish which of the following?
(A)

Whether melatonin supplements help people who have difficulty maintaining a 24‐hour sleep cycle to establish such a
pattern

(B)

Whether the melatonin levels of most totally blind people who successfully maintain a 24‐hour sleep‐wake cycle
change in response to changes in exposure to light and darkness

(C)

Whether melatonin is the only substance secreted by the pineal gland

(D)

Whether most people who do not have a 24‐hour sleep‐wake cycle nevertheless have a cycle of consistent duration

(E)

Whether there are any people with normal vision whose melatonin levels respond abnormally to periods of light and
darkness

Argument Evaluation
Situation

Normally, a person’s sleep‐wake cycle is governed by the pineal gland secreting melatonin in
response to the daily cycle of light and darkness as detected by the eye. Yet many people who are
totally blind due to lesions of the visual cortex easily maintain a 24‐hour sleep‐wake cycle.

Reasoning

What additional information would be most helpful in evaluating the argument? The argument’s
conclusion is that the neural pathway by which the pineal gland receives information probably
does not pass through the visual cortex. This is suggested by the fact that people without a
well‐functioning visual cortex (e.g., people with a certain type of blindness) can nonetheless
maintain a 24‐hour sleep‐wake cycle. Is it by the pineal gland’s secretion of melatonin that they
do so? The argument tells us that normally (i.e., in sighted people), this is the mechanism for
sleep regulation. But the argument depends on assuming that a similar mechanism is operating
in people who are blind but have well‐regulated sleep cycles. The best choice will be the one that
helps us decide whether that assumption is correct.

A

B

C

D
E

This question would not give us an answer that would help in evaluating the argument. A “no” answer
would not clarify whether the pineal gland‐melatonin mechanism operates in people who are blind. A
“yes” answer would do no better. The question refers only to people who have sleep dysfunctions (which
the argument does not address).
Correct. Answering this question would provide the most useful information for evaluating the argument.
A “yes” answer would help confirm a key assumption of the argument: that blind people rely on the pineal
gland‐melatonin mechanism for sleep regulation. A “no” answer would help disconfirm that assumption.
Whether or not there are other substances secreted by the pineal gland makes no difference to the
reasoning. The argument relies on the premise that the pineal gland governs the sleep cycle by secreting
melatonin. For example, if the pineal gland sometimes secreted adrenaline, that would still have no bearing
on the argument.
The consistency or inconsistency of the duration of some people’s sleep patterns has no relevance to the
reasoning. Their sleep patterns could be due to any of a number of factors.
This does not help, for there could be sighted people whose melatonin levels respond abnormally simply
because of a pineal‐gland abnormality.

The correct answer is B.

641

GMAT® Official Guide 2018

639. In countries where automobile insurance includes compensation for whiplash injuries sustained in automobile
accidents, reports of having suffered such injuries are twice as frequent as they are in countries where
whiplash is not covered. Presently, no objective test for whiplash exists, so it is true that spurious reports of whiplash
injuries cannot be readily identified. Nevertheless, these facts do not warrant the conclusion drawn by some commentators
that in the countries with the higher rates of reported whiplash injuries, half of the reported cases are spurious. Clearly,
in countries where automobile insurance does not include compensation for whiplash, people often have little
incentive to report whiplash injuries that they actually have suffered.
In the argument given, the two boldfaced portions play which of the following roles?
(A)

The first is a claim that the argument disputes; the second is a conclusion that has been based on that claim.

(B)

The first is a claim that has been used to support a conclusion that the argument accepts; the second is that
conclusion.

(C)

The first is evidence that has been used to support a conclusion for which the argument provides further evidence;
the second is the main conclusion of the argument.

(D)

The first is a finding whose implications are at issue in the argument; the second is a claim presented in order to
argue against deriving certain implications from that finding.

(E)

The first is a finding whose accuracy is evaluated in the argument; the second is evidence presented to establish that
the finding is accurate.

Argument Evaluation
Situation

Reported whiplash injuries are twice as common in countries where car insurance companies
pay compensation for such injuries as they are in countries where insurance companies do not.
Although there is no objective test for whiplash, this does not mean, as some suggest, that half of
the reports of such injuries are fake. It could simply be that where insurance will not pay for such
injuries, people are less inclined to report them.

Reasoning

What roles do the two boldfaced portions play in the argument? The first portion tells us about the
correlation between reported cases of whiplash in countries and the willingness of insurance
companies in those countries to compensate for whiplash injuries. The argument next states that
whiplash is difficult to objectively verify. The argument then asserts that although this last fact,
taken together with the first boldfaced portion, has led some to infer that over half of the reported
cases in countries with the highest whiplash rates are spurious, such an inference is unwarranted.
The second boldfaced portion then helps to explain why such an inference is not necessarily
warranted by offering an alternative explanation.

A
B

C
D
E

The claim made in the first boldfaced portion is never disputed in the argument; at dispute is how to
account for the fact that this claim is true. The second is not the argument’s conclusion.
In a manner of speaking, perhaps, the argument uses the first portion to support its conclusion; but there is
no indication that it has been used elsewhere to do so. In any case, the second boldfaced portion is not the
argument’s conclusion.
The first has been used to support a conclusion that the argument rejects; the second boldfaced portion is
not the argument’s conclusion.
Correct. This option correctly identifies the roles played in the argument by the boldfaced portions.
The accuracy of the first boldfaced portion is never questioned in the argument; nor is the second intended
to somehow help show that the first is accurate. Rather, the argument assumes that the first portion is
accurate.

The correct answer is D.

642

8.6 Critical Reasoning Answer Explanations

640. Last year Comfort Airlines had twice as many delayed flights as the year before, but the number of complaints from
passengers about delayed flights went up three times. It is unlikely that this disproportionate increase in complaints was
rooted in an increase in overall dissatisfaction with the service Comfort Airlines provides, since the airline made a special
effort to improve other aspects of its service last year.
Which of the following, if true, most helps to explain the disproportionate increase in customer complaints?
(A)

Comfort Airlines had more flights last year than the year before.

(B)

Last year a single period of unusually bad weather caused a large number of flights to be delayed.

(C)

Some of the improvements that Comfort Airlines made in its service were required by new government regulations.

(D)

The average length of a flight delay was greater last year than it was the year before.

(E)

The average number of passengers per flight was no higher last year than the year before.

Argument Construction
Situation

Last year Comfort Airlines had twice as many delayed flights as it did the year before, but three
times as many passenger complaints about delayed flights. The airline made a special effort to
improve other aspects of its service last year.

Reasoning

What could explain why the number of complaints about delayed flights increased disproportionately to
the number of delayed flights last year? In other words, why did the average number of passenger
complaints per delayed flight go up last year? One obvious possibility is that the average number
of passengers per delayed flight was greater last year than it had been the year before. Another is
that the flight delays tended to cause worse problems for passengers last year than they had the
year before, so that on average each delay was more upsetting for the passengers.

A
B

C

D
E

This helps explain why the airline had more delayed flights last year, but not why the increase in
complaints about delayed flights was disproportionate to the increase in delayed flights.
This helps explain why the airline had more delayed flights last year. But, if anything, the situation should
have reduced the number of passenger complaints per delayed flight, since many passengers should have
realized that the unusually bad weather was not the airline’s fault.
If any of the improvements concerned handling of flight delays, for example, and passengers were aware
that government regulations addressed this, then passengers might have complained more than previously.
But the information we are given here is too general and too vague to explain the disproportionate increase
in complaints.
Correct. Longer flight delays would have more severely inconvenienced passengers and thus would
probably have generated more passenger complaints per delay.
This rules out the possibility that an increased number of passengers per delayed flight could have caused
the disproportionate increase in the number of complaints about delayed flights. But no alternative
explanation is offered.

The correct answer is D

643

GMAT® Official Guide 2018

641. Last year a global disturbance of weather patterns disrupted harvests in many of the world’s important agricultural areas.
Worldwide production of soybeans, an important source of protein for people and livestock alike, was not adversely
affected, however. Indeed, last year’s soybean crop was actually slightly larger than average. Nevertheless, the weather
phenomenon is probably responsible for a recent increase in the world price of soybeans.
Which of the following, if true, provides the strongest justification for the attribution of the increase in soybean prices to the
weather phenomenon?
(A)

Last year’s harvest of anchovies, which provide an important protein source for livestock, was disrupted by the
effects of the weather phenomenon.

(B)

Most countries that produce soybeans for export had above‐average harvests of a number of food crops other than
soybeans last year.

(C)

The world price of soybeans also rose several years ago, immediately after an earlier occurrence of a similar global
weather disturbance.

(D)

Heavy rains attributable to the weather phenomenon improved grazing pastures last year, allowing farmers in many
parts of the world to reduce their dependence on supplemental feed.

(E)

Prior to last year, soybean prices had been falling for several years.

Argument Construction
Situation

A weather disturbance last year disrupted harvests worldwide but did not reduce production of
soybeans, a protein source for both people and livestock. Soybean prices increased nonetheless,
likely a result of the weather.

Reasoning

What evidence would suggest that the weather disturbance caused the increase in soybean prices even
though it did not reduce soybean production? Prices tend to increase when the supply of a product
falls relative to the demand for the product. But the production of soybeans did not fall. Evidence
that the weather disturbance either hindered the global distribution of soybeans or increased
global demand for soybeans could support the claim that the weather disturbance caused the
increase in soybean prices.

A

B

C
D
E

Correct. If the weather disturbance reduced the anchovy harvest, and anchovies provide protein for
livestock just as soybeans do, then more soybeans for livestock feed would be needed to compensate for the
lack of anchovies. The resulting increase in demand for soybeans could thus have increased global soybean
prices.
This is not surprising, given that the weather disturbance did not severely affect the soybean‐producing
countries, but it does not explain how the weather disturbance could have caused soybean prices to
increase.
The rise in soybean prices after the earlier weather disturbance could easily have been a coincidence. Or,
unlike last year’s disturbance, the earlier disturbance could have reduced soybean production.
This suggests that demand for soybeans should have fallen as a result of the weather disturbance, so it does
not explain why soybean prices rose.
If soybean prices were unusually low for some temporary reason when the weather disturbance occurred,
they might have been likely to rise back to normal levels even without the weather disturbance.

The correct answer is A.

644

8.6 Critical Reasoning Answer Explanations

642. Most of the year, the hermit thrush, a North American songbird, eats a diet consisting mainly of insects, but in autumn, as
the thrushes migrate to their Central and South American wintering grounds, they feed almost exclusively on wild berries.
Wild berries, however, are not as rich in calories as insects, yet thrushes need to consume plenty of calories in order to
complete their migration. One possible explanation is that berries contain other nutrients that thrushes need for migration
and that insects lack.
Which of the following, if true, most seriously calls into question the explanation given for the thrush’s diet during migration?
(A)

Hermit thrushes, if undernourished, are unable to complete their autumn migration before the onset of winter.

(B)

Insect species contain certain nutrients that are not found in wild berries.

(C)

For songbirds, catching insects requires the expenditure of significantly more calories than eating wild berries does.

(D)

Along the hermit thrushes’ migration routes, insects are abundant throughout the migration season.

(E)

There are some species of wild berries that hermit thrushes generally do not eat, even though these berry species
are exceptionally rich in calories.

Argument Evaluation
Situation

Hermit thrushes are songbirds that usually eat insects but switch to eating berries when migrating.
The thrushes need lots of calories to migrate, but berries contain fewer calories than insects do.
Perhaps the berries contain nutrients that insects do not provide.

Reasoning

What would cast doubt on the claim that the thrushes switch to berries because berries contain nutrients
that insects lack and that the thrushes need for their migration? Evidence that berries do not contain
such nutrients or that thrushes do not decrease their net calorie consumption by eating berries
would cast doubt on the proposed explanation. So would any evidence that supported an
alternative explanation for the diet change during migration—for example, seasonal or regional
differences in the amount or quality of berries or insects available for the thrushes to consume.

A
B
C
D
E

Even if thrushes need to be well‐nourished to finish migrating before winter, extra nutrients found in
berries but not insects might help provide the nourishment they need.
Even if insects contain certain nutrients not found in wild berries, those specific nutrients may not be the
ones the thrushes need for their migration.
Correct. This suggests that the thrushes might gain more net calories from eating berries than from eating
insects, which could explain why they switch to eating berries even if the berries contain no extra nutrients.
By ruling out a lack of insects to eat while migrating as an alternative explanation for why the thrushes
switch to eating berries, this would support the proposed explanation.
The calorie‐rich species of berries the thrushes do not eat might be poisonous or indigestible for them,
even if the species of berries the thrushes do eat contain nutrients they need to migrate.

The correct answer is C.

645

GMAT® Official Guide 2018

643. The kinds of hand and wrist injuries that result from extended use of a computer while maintaining an incorrect posture
are common among schoolchildren in Harnville. Computers are important to the school curriculum there, so instead
of reducing the amount their students use computers, teachers plan to bring about a sharp reduction in the number of
these injuries by carefully monitoring their students’ posture when using computers in the classroom.
Which of the following would it be most useful to know in order to assess the likelihood that the teachers’ plan will be
successful?
(A)

Whether extended use of a computer while maintaining incorrect posture can cause injuries other than hand and wrist
injuries

(B)

Whether hand and wrist injuries not caused by computer use are common among schoolchildren in Harnville

(C)

What proportion of schoolchildren in Harnville with hand and wrist injuries use computers extensively outside the
classroom

(D)

Whether changes in the curriculum could reduce the schools’ dependence on computers

(E)

What proportion of schoolchildren in Harnville already use correct posture while using a computer

Evaluation of a Plan
Situation

Hand and wrist injuries from using computers while maintaining poor posture are common
among schoolchildren in Harnville. Teachers plan to greatly reduce the number of such injuries by
monitoring their students’ posture while the students use computers in the classroom.

Reasoning

What would be most helpful to know to determine the likelihood that the teachers’ plan will succeed? The
primary concern is the posture students adopt while using computers. To succeed, the teachers’
plan must reduce the time students spend with poor posture while using computers and reduce it
enough to greatly reduce the number of injuries. To know how likely this is, it would help to know
how effectively the teachers will be able to monitor and improve their students’ posture inside the
classroom. But how many of the students use computers outside of school while maintaining poor
posture and how often do they do so? If many students do so quite often, they may develop hand
and wrist injuries regardless of what happens in school.

A

The teachers do not plan to reduce any injuries other than hand and wrist injuries, so whether computer
use with poor posture causes any such other injuries is irrelevant to the likelihood that their plan will
produce its intended effect.
B
The plan being discussed concerns only the reduction of hand and wrist injuries caused specifically by
computer use with poor posture, so the frequency of hand and wrist injuries from other causes is irrelevant
to the likelihood that the plan will produce its intended effect.
C Correct. If the students’ school use of computers is a large part of their overall computer use, any retraining that
accompanies the monitoring might have some effect on their posture and related injury rates overall. However,
the greater the proportion of children with hand and wrist injuries who use computers extensively outside the
classroom, the more children are likely to keep developing the injuries regardless of any monitoring at school, so
the less effective the teachers’ plan involving only computer use at school is likely to be.
D Knowing whether this is the case might help in developing a potential alternative to the teachers’ plan, but
if it did, this would not help significantly toward assessing the likelihood that the actual plan will succeed.
The teachers’ actual plan involves monitoring computer use in school without reducing such use. Other
possible means of achieving the plan’s goal are not part of the plan and are therefore irrelevant to the
likelihood that the teachers’ actual plan will succeed.
E The passage indicates that the proportion of the schoolchildren maintaining poor posture while using
computers is high enough for many to develop hand and wrist injuries as a result. Whatever the exact
proportion is, the teachers’ plan may or may not succeed in reducing it.
The correct answer is C.
646

8.6 Critical Reasoning Answer Explanations

644. A certain cultivated herb is one of a group of closely related plants that thrive in soil with high concentrations of metals
that are toxic to most other plants. Agronomists studying the growth of this herb have discovered that it produces large
amounts of histidine, an amino acid that, in test‐tube solutions, renders these metals chemically inert. Hence, the herb’s
high histidine production must be the key feature that allows it to grow in metal‐rich soils.
In evaluating the argument, it would be most important to determine which of the following?
(A)

Whether the herb can thrive in soil that does not have high concentrations of the toxic metals

(B)

Whether others of the closely related group of plants also produce histidine in large quantities

(C)

Whether the herb’s high level of histidine production is associated with an unusually low level of production of some
other amino acid

(D)

Whether growing the herb in soil with high concentrations of the metals will, over time, reduce their concentrations in
the soil

(E)

Whether the concentration of histidine in the growing herb declines as the plant approaches maturity

Argument Evaluation
Situation

A certain herb and closely related species thrive in soil full of metals toxic to most plants. The
herb produces much histidine, which makes those metals chemically inert. Histidine production,
therefore, is largely what accounts for the herb’s thriving in metal‐rich soils.

Reasoning

What evidence would help determine whether the herb’s histidine production is what enables it to thrive
in metal‐rich soils? The argument is that since the herb’s histidine chemically neutralizes the
metals that are toxic to most plants, it must explain why the herb can thrive in metal‐rich soils. To
evaluate this argument, it would be helpful to know about the relationship between other closely
related plant species’ histidine production and the ability to thrive in metal‐rich soils. It would also
be helpful to know about any other factors that might plausibly explain why the herb can thrive in
those soils.

A
B
C
D
E

Whether or not the herb thrives in metal‐free soils, histidine production could enable it to thrive in soils
that contain toxic metals.
Correct. If the closely related plants do not produce much histidine, whatever other factor allows them to
thrive in metal‐rich soils would likely account for why the herb thrives in those soils as well.
The given information suggests no particular reason to suppose that a low level of some unspecified amino
acid would enable a plant to thrive in metal‐rich soils.
The herb might absorb metals from any metal‐rich soil it grows in, regardless of why it thrives in that soil.
Whether or not histidine concentrations in the herb decline as it approaches maturity, there could still be
enough histidine in the growing herb to neutralize the metals and explain why it can grow in metal‐rich
soil.

The correct answer is B.

647

GMAT® Official Guide 2018

645. Many people suffer an allergic reaction to certain sulfites, including those that are commonly added to wine as
preservatives. However, since there are several winemakers who add sulfites to none of the wines they produce, people
who would like to drink wine but are allergic to sulfites can drink wines produced by these winemakers without risking an
allergic reaction to sulfites.
Which of the following is an assumption on which the argument depends?
(A)

These winemakers have been able to duplicate the preservative effect produced by adding sulfites by means that do
not involve adding any potentially allergenic substances to their wine.

(B)

Not all forms of sulfite are equally likely to produce the allergic reaction.

(C)

Wine is the only beverage to which sulfites are commonly added.

(D)

Apart from sulfites, there are no substances commonly present in wine that give rise to an allergic reaction.

(E)

Sulfites are not naturally present in the wines produced by these winemakers in amounts large enough to produce an
allergic reaction in someone who drinks these wines.

Argument Construction
Situation

People who are allergic to certain sulfites can avoid risking an allergic reaction by drinking wine
from one of the several producers that does not add sulfites.

Reasoning

On what assumption does the argument depend? Drinking wine to which no sulfites have been added
will not prevent exposure to sulfites if, for instance, sulfites occur naturally in wines. In particular,
if the wines that do not have sulfites added have sulfites present naturally in quantities sufficient
to produce an allergic reaction, drinking these wines will not prevent an allergic reaction. The
argument therefore depends on assuming that this is not the case.

A
B
C

D
E

The argument does not require this because the conclusion does not address allergic reactions to substances
other than sulfites.
The argument specifically refers to “certain sulfites” producing allergic reactions. It is entirely compatible
with certain other forms of sulfites not producing allergic reactions in anyone.
This is irrelevant. The argument does not claim that one can avoid having an allergic reaction to sulfites
from any source just by restricting one’s wine consumption to those varieties to which no sulfites have been
added.
Once again, the argument’s conclusion does not address allergic reactions to substances other than sulfites
in wine.
Correct. The argument relies on this assumption.

The correct answer is E.

648

8.6 Critical Reasoning Answer Explanations

646. A new law gives ownership of patents—documents providing exclusive right to make and sell an invention—to universities,
not the government, when those patents result from government‐sponsored university research. Administrators at Logos
University plan to sell any patents they acquire to corporations in order to fund programs to improve undergraduate
teaching.
Which of the following, if true, would cast the most doubt on the viability of the college administrators’ plan described
above?
(A)

Profit‐making corporations interested in developing products based on patents held by universities are likely to try to
serve as exclusive sponsors of ongoing university research projects.

(B)

Corporate sponsors of research in university facilities are entitled to tax credits under new federal tax‐code guidelines.

(C)

Research scientists at Logos University have few or no teaching responsibilities and participate little if at all in the
undergraduate programs in their field.

(D)

Government‐sponsored research conducted at Logos University for the most part duplicates research already
completed by several profit‐making corporations.

(E)

Logos University is unlikely to attract corporate sponsorship of its scientific research.

Evaluation of a Plan
Situation

Universities own the patents resulting from government‐sponsored research at their institutions.
One university plans to sell its patents to corporations to fund a program to improve teaching.

Reasoning

Which point casts doubt on the university’s plan? The university’s plan assumes there will be a market
for its patents, and that the corporations will want to buy them. What might make this untrue?
If some of the corporations have already done the same or similar research, they will not be
prospective buyers of the university’s patents.

A
B
C
D
E

This point is irrelevant to the plan to sell patents in order to fund a program.
The university plans to sell the patents to the corporations, not to invite the corporations to sponsor
research.
This point is irrelevant to the university’s plan to sell off patents since the plan does not specify that the
research scientists will be involved in the programs to improve undergraduate teaching.
Correct. This statement properly identifies a factor that casts doubt on the university’s plan to sell its
patents to corporations.
The plan concerns selling patents resulting from government‐sponsored research, not attracting corporate
sponsorship for research.

The correct answer is D.

649

GMAT® Official Guide 2018

647. Since it has become known that several of a bank’s top executives have been buying shares in their own bank, the
bank’s depositors, who had been worried by rumors that the bank faced impending financial collapse, have been greatly
relieved. They reason that, since top executives evidently have faith in the bank’s financial soundness, those worrisome
rumors must be false. Such reasoning might well be overoptimistic, however, since corporate executives have been
known to buy shares in their own company in a calculated attempt to dispel negative rumors about the
company’s health.
In the argument given, the two boldfaced portions play which of the following roles?
(A)

The first describes evidence that has been taken as supporting a conclusion; the second gives a reason for
questioning that support.

(B)

The first describes evidence that has been taken as supporting a conclusion; the second states a contrary conclusion
that is the main conclusion of the argument.

(C)

The first provides evidence in support of the main conclusion of the argument; the second states that conclusion.

(D)

The first describes the circumstance that the argument as a whole seeks to explain; the second gives the explanation
that the argument seeks to establish.

(E)

The first describes the circumstance that the argument as a whole seeks to explain; the second provides evidence in
support of the explanation that the argument seeks to establish.

Argument Evaluation
Situation

Top executives at a bank that has been rumored to be in financial trouble have been buying shares
in the bank. Bank depositors see this as a good sign, because they believe that it indicates that the
executives have faith in the bank. However, corporate executives sometimes do this just to dispel
rumors about a company’s health.

Reasoning

What is the role that the two boldfaced portions play in the argument? The first boldfaced portion
states that bank executives are buying bank shares, which the passage indicates is taken by bank
depositors to be evidence of the executives’ faith in the bank. The passage then tells us what
some have inferred from this, and finally offers in the second boldfaced statement evidence that
undermines this inference.

A
B
C
D

E

Correct. This option correctly identifies the roles played by the boldfaced portions.
This correctly describes the first statement’s role, but the second statement is not offered as a conclusion—
no evidence is given for it; rather it is evidence for something else.
The second statement is not offered as a conclusion; no evidence is given for it.
The second statement is not itself offered as an explanation of why these bank executives are investing in
the bank; if it were, that would mean that the bank executives are doing so because corporate executives
are known to do such things in a calculated effort to dispel worries. Furthermore the argument does not
conclude that this other explanation (which the boldfaced portion points to) is correct, only that the one
inferred by depositors may not be.
The argument is not so much seeking to establish an explanation of its own as it is trying to undermine
that inferred by the depositors.

The correct answer is A.

650

8.6 Critical Reasoning Answer Explanations

648. Between 1980 and 2000 the sea otter population of the Aleutian Islands declined precipitously. There were no signs of
disease or malnutrition, so there was probably an increase in the number of otters being eaten by predators. Orcas will eat
otters when seals, their normal prey, are unavailable, and the Aleutian Islands seal population declined dramatically in the
1980s. Therefore, orcas were most likely the immediate cause of the otter population decline.
Which of the following, if true, most strengthens the argument?
(A)

The population of sea urchins, the main food of sea otters, has increased since the sea otter population declined.

(B)

Seals do not eat sea otters, nor do they compete with sea otters for food.

(C)

Most of the surviving sea otters live in a bay that is inaccessible to orcas.

(D)

The population of orcas in the Aleutian Islands has declined since the 1980s.

(E)

An increase in commercial fishing near the Aleutian Islands in the 1980s caused a slight decline in the population of
the fish that seals use for food.

Argument Evaluation
Situation

A sea otter population declined even though there were no signs of disease or malnutrition.
The local seal population also declined. Orcas eat otters when seals are unavailable, and thus are
probably the cause of the decline in the otter population.

Reasoning

What would be evidence that predation by orcas reduced the sea otter population? Disease and
malnutrition are ruled out as alternative explanations of the decline in the sea otter population.
The argument could be further strengthened by casting doubt on other possible explanations,
such as predation by other animals, or by presenting observations that predation of otters by orcas
would help to explain.

A
B
C
D
E

Regardless of whether or not orcas ate the sea otters, the sea urchin population would most likely have
increased when the population of sea otters preying on them decreased.
Because the seal population declined during the initial years of the otter population decline, predation by
and competition with seals were already implausible explanations of the otter population decline.
Correct. Orcas eating most of the accessible otters could plausibly explain this observation, which
therefore provides additional evidence that orca predation reduced the sea otter population.
If the orca population declined at the same time as the sea otter population, it would be less likely that
increasing predation by orcas reduced the otter population.
Since the sea otters showed no signs of malnutrition, they were probably getting enough fish. But if they
were not, commercial fishing rather than orcas might have caused the otter population decline.

The correct answer is C.

651

GMAT® Official Guide 2018

649. Studies in restaurants show that the tips left by customers who pay their bill in cash tend to be larger when the bill is
presented on a tray that bears a credit‐card logo. Consumer psychologists hypothesize that simply seeing a credit‐card logo
makes many credit‐card holders willing to spend more because it reminds them that their spending power exceeds the cash
they have immediately available.
Which of the following, if true, most strongly supports the psychologists’ interpretation of the studies?
(A)

The effect noted in the studies is not limited to patrons who have credit cards.

(B)

Patrons who are under financial pressure from their credit‐card obligations tend to tip less when presented with a
restaurant bill on a tray with a credit‐card logo than when the tray has no logo.

(C)

In virtually all of the cases in the studies, the patrons who paid bills in cash did not possess credit cards.

(D)

In general, restaurant patrons who pay their bills in cash leave larger tips than do those who pay by credit card.

(E)

The percentage of restaurant bills paid with a given brand of credit card increases when that credit card’s logo is
displayed on the tray with which the bill is presented.

Argument Evaluation
Situation

Studies have found that restaurant customers give more generous tips when their bills are brought
on trays bearing a credit‐card logo. Psychologists speculate that this is because the logo reminds
customers of their ability to spend more money than they have.

Reasoning

Which of the options most helps to support the psychologists’ explanation of the studies? The psychologists’
hypothesis is that the credit‐card logos on the trays bring to the minds of those who tip more
the fact that they have more purchasing power than merely the cash that they have at hand. This
explanation would not be valid even if those people who are not reminded of their own excess
purchasing power—if in fact they have any such power—when they see such a logo nonetheless
tip more in such trays. Thus, if restaurant patrons who are under financial pressure from their
credit‐card obligations do not tip more when their bills are presented on trays bearing credit‐card
logos, then the psychologists’ interpretation of the studies is supported.

A
B
C
D
E

This undermines the psychologists’ interpretation, for it shows that the same phenomenon occurs even
when the alleged cause has been removed.
Correct. This option identifies the result that would most strengthen the psychologists’ interpretation.
This undermines the psychologists’ interpretation by showing that the same phenomenon occurs even
when the alleged cause has been removed; patrons cannot be reminded of something that is not there.
To the extent that this bears on the interpretation of the study, it weakens it. Patrons using credit cards are
surely aware that they have credit, and yet they spend less generously.
This does not support the idea that being reminded that one has a credit card induces one to be more
generous, only that it induces one to use that credit card.

The correct answer is B.

652

8.6 Critical Reasoning Answer Explanations

650. In an experiment, each volunteer was allowed to choose between an easy task and a hard task and was told that another
volunteer would do the other task. Each volunteer could also choose to have a computer assign the two tasks randomly.
Most volunteers chose the easy task for themselves and under questioning later said they had acted fairly. But when the
scenario was described to another group of volunteers, almost all said choosing the easy task would be unfair. This shows
that most people apply weaker moral standards to themselves than to others.
Which of the following is an assumption required by this argument?
(A)

At least some volunteers who said they had acted fairly in choosing the easy task would have said that it was unfair
for someone else to do so.

(B)

The most moral choice for the volunteers would have been to have the computer assign the two tasks randomly.

(C)

There were at least some volunteers who were assigned to do the hard task and felt that the assignment was unfair.

(D)

On average, the volunteers to whom the scenario was described were more accurate in their moral judgments than
the other volunteers were.

(E)

At least some volunteers given the choice between assigning the tasks themselves and having the computer assign
them felt that they had made the only fair choice available to them.

Argument Construction
Situation

In an experiment, most volunteers chose to do an easy task themselves and leave a hard task for
someone else. They later said they had acted fairly, but almost all volunteers in another group to
which the scenario was described said choosing the easy task would be unfair, indicating that most
people apply weaker moral standards to themselves.

Reasoning

What must be true in order for the facts presented to support the conclusion that most people apply weaker
moral standards to themselves than to others? One set of volunteers said they had acted fairly in
taking the easy task, whereas different volunteers said that doing so would be unfair. In neither
case did any of the volunteers actually judge their own behavior differently from how they judged
anyone else’s. So the argument implicitly infers from the experimental results that most of the
volunteers would judge their own behavior differently from someone else’s if given the chance.
This inference assumes that the volunteers in the second group would have applied the same moral
standards that those in the first group did if they had been in the first group’s position, and vice
versa.

A

B
C

D
E

Correct. If none of the volunteers who said their own behavior was fair would have judged someone else’s
similar behavior as unfair, then their relaxed moral judgment of themselves would not suggest that they
applied weaker moral standards to themselves than to others.
Even if this is so, the experimental results could still suggest that the volunteers would apply weaker moral
standards to themselves than to others.
The argument would be equally strong even if volunteers who were assigned the hard task did not know
that someone else had gotten an easier task—or even if no volunteers were actually assigned the hard task
at all.
Even if the moral standards applied by the volunteers who judged themselves were as accurate as those
applied by the volunteers to whom the scenario was described, the former standards were still weaker.
Even if all the volunteers in the first group had felt that all the choices available to them would have been
fair for them to make personally, they might have applied stricter moral standards to someone else in the
same position.

The correct answer is A.

653

GMAT® Official Guide 2018

651. Country X’s recent stock-trading scandal should not diminish investors’ confidence in the country’s stock market. For one
thing, the discovery of the scandal confirms that Country X has a strong regulatory system, as the following
considerations show. In any stock market, some fraudulent activity is inevitable. If a stock market is well regulated,
any significant stock-trading fraud in it will very likely be discovered. This deters potential perpetrators and facilitates
improvement in regulatory processes.
In the argument, the portion in boldface plays which of the following roles?
(A)

It is the argument’s only conclusion.

(B)

It is a conclusion for which the argument provides support and which itself is used to support the argument’s main
conclusion.

(C)

It is the argument’s main conclusion and is supported by another explicitly stated conclusion for which further support
is provided.

(D)

It is an assumption for which no explicit support is provided and is used to support the argument’s only conclusion.

(E)

It is a compound statement containing both the argument’s main conclusion and an assumption used to support that
conclusion.

Argument Construction
Situation

Country X recently had a stock-trading scandal.

Reasoning

What role does the statement that the scandal’s discovery confirms that Country X has a strong regulatory
system play in the argument? In the sentence containing the boldface statement, the phrase For
one thing indicates that the statement is being used to justify the claim in the preceding sentence.
Thus, the boldface statement must support that preceding sentence as a conclusion. Directly after
the boldface statement, the phrase as the following considerations show indicates that the subsequent
sentences are being used to support the boldface statement. Thus, the boldface statement is a
conclusion supported by the sentences following it, and this statement itself supports the sentence
preceding it, which must be the argument’s main conclusion.

A
B
C
D
E

As explained above, the boldface statement supports the claim in the preceding sentence, so it cannot be
the argument’s only conclusion.
Correct. As explained above, the boldface statement is supported by the statements following it and in
turn is used to support the argument’s main conclusion in the statement preceding it.
As explained above, the boldface statement cannot be the argument’s main conclusion, because it supports
a further conclusion presented in the sentence preceding it.
As explained above, the sentences following the boldface statement are the explicit support provided for it.
As explained above, the argument’s main conclusion is stated only in the first sentence, which precedes the
boldface statement. It is not repeated anywhere in the boldface statement.

The correct answer is B.

654

8.6 Critical Reasoning Answer Explanations

652. Delta Products Inc. has recently switched at least partly from older technologies using fossil fuels to new
technologies powered by electricity. The question has been raised whether it can be concluded that for a given level
of output Delta’s operation now causes less fossil fuel to be consumed than it did formerly. The answer, clearly, is
yes, since the amount of fossil fuel used to generate the electricity needed to power the new technologies is less than the
amount needed to power the older technologies, provided level of output is held constant.
In the argument given, the two boldfaced portions play which of the following roles?
(A)

The first identifies the content of the conclusion of the argument; the second provides support for that conclusion.

(B)

The first provides support for the conclusion of the argument; the second identifies the content of that conclusion.

(C)

The first states the conclusion of the argument; the second calls that conclusion into question.

(D)

The first provides support for the conclusion of the argument; the second calls that conclusion into question.

(E)

Each provides support for the conclusion of the argument.

Argument Evaluation
Situation

Delta switched from technologies using fossil fuels to ones using electricity. It has been asked
whether this results in less fossil fuel used per level of output. The answer is that it does.

Reasoning

What roles do the two boldfaced portions play in the argument? The first boldfaced statement is simply
asserted by the passage. But the second boldfaced statement, when it is first introduced, is not
asserted to be true, but rather is identified as something that might be inferred from the first
statement. By the end of the passage the argument concludes that the second statement is true.

A
B
C
D

E

This option simply reverses the roles that the statements play in the argument.
Correct. This option identifies the roles the boldfaced portions play.
Nothing in the passage is intended to support the first statement; and the second statement is not
supposed to call the first into question.
This correctly identifies the role of the first statement, but the second boldfaced portion does not call the
argument’s conclusion into question—it is part of a sentence that refers to the question whether that
conclusion can be drawn from the first statement.
Again, this is only half right. The second boldfaced portion is not offered as support for the conclusion;
if it were offered as such support, the argument would be guilty of circular reasoning, since the second
boldfaced portion states exactly what the argument concludes.

The correct answer is B.

655

GMAT® Official Guide 2018

653. Theater Critic: The play La Finestrina, now at Central Theater, was written in Italy in the eighteenth century. The director
claims that this production is as similar to the original production as is possible in a modern theater. Although the actor who
plays Harlequin the clown gives a performance very reminiscent of the twentieth‐century American comedian Groucho Marx,
Marx’s comic style was very much within the comic acting tradition that had begun in sixteenth‐century Italy.
The considerations given best serve as part of an argument that
(A)

modern audiences would find it hard to tolerate certain characteristics of a historically accurate performance of an
eighteenth‐century play

(B)

Groucho Marx once performed the part of the character Harlequin in La Finestrina

(C)

in the United States the training of actors in the twentieth century is based on principles that do not differ radically
from those that underlay the training of actors in eighteenth‐century Italy

(D)

the performance of the actor who plays Harlequin in La Finestrina does not serve as evidence against the director’s
claim

(E)

the director of La Finestrina must have advised the actor who plays Harlequin to model his performance on comic
performances of Groucho Marx

Argument Construction
Situation

The director of the local production of La Finestrina says it is as similar to the original production
as is possible in a modern theater. The actor playing Harlequin gives a performance reminiscent
of Groucho Marx, whose comic style falls within an acting tradition which began in sixteenth‐
century Italy.

Reasoning

For which of the options would the consideration given best serve as an argument? The actor’s
performance was reminiscent of someone who fell within a tradition going back to sixteenth‐
century Italy. The play was written, and therefore was likely first performed, in eighteenth‐century
Italy. All of this suggests that there could be a similarity between the performances of Harlequin
in the local production and in the original production. While the two performances might have
been quite dissimilar, there is nothing here that supports that.

A

B
C
D
E

Regardless of how plausible this option might be on its own merits, the passage provides no support
for it because the passage provides no information about the characteristics of a historically accurate
performance of an eighteenth‐century play.
The passage neither says this nor implies it.
The passage says nothing about the training of actors, so this option would be supported by the passage
only in a very roundabout, indirect way.
Correct. This is the option that the considerations most support.
That the performance reminded the theater critic of Groucho Marx hardly shows that the similarity was
intentional, let alone that it was at the director’s instruction.

The correct answer is D.

656

8.6 Critical Reasoning Answer Explanations

654. Although the discount stores in Goreville’s central shopping district are expected to close within five years as a result of
competition from a SpendLess discount department store that just opened, those locations will not stay vacant for long.
In the five years since the opening of Colson’s, a nondiscount department store, a new store has opened at the location of
every store in the shopping district that closed because it could not compete with Colson’s.
Which of the following, if true, most seriously weakens the argument?
(A)

Many customers of Colson’s are expected to do less shopping there than they did before the SpendLess store
opened.

(B)

Increasingly, the stores that have opened in the central shopping district since Colson’s opened have been discount
stores.

(C)

At present, the central shopping district has as many stores operating in it as it ever had.

(D)

Over the course of the next five years, it is expected that Goreville’s population will grow at a faster rate than it has
for the past several decades.

(E)

Many stores in the central shopping district sell types of merchandise that are not available at either SpendLess or
Colson’s.

Argument Evaluation
Situation

Due to competition from a recently opened SpendLess discount department store, discount stores
in Goreville’s central shopping district are expected to close within five years. But those locations
will not be vacant long, for new stores have replaced all those that closed because of the opening
five years ago of a Colson’s nondiscount department store.

Reasoning

The question is which option would most weaken the argument? The arguer infers that stores that leave
because of the SpendLess will be replaced in their locations by other stores because that is what
happened after the Colson’s department store came in. Since the reasoning relies on a presumed
similarity between the two cases, any information that brings to light a relevant dissimilarity
would weaken the argument. If the stores that were driven out by Colson’s were replaced mostly by
discount stores, that suggests that the stores were replaced because of a need that no longer exists
after the opening of SpendLess.

A
B
C
D
E

The fact that Colson’s may be seeing fewer customers does not mean that the discount stores that close will
not be replaced; they might be replaced by stores that in no way compete with Colson’s or SpendLess.
Correct. This option most seriously weakens the argument.
If anything, this strengthens the argument by indicating that Goreville’s central shopping district is
thriving.
This, too, strengthens the argument because one is more likely to open a new store in an area with a
growing population.
Because this statement does not indicate whether any of these stores that offer goods not sold at
SpendLess or Colson’s will be among those that are closing, it is not possible to determine what effect it
has on the strength of the argument.

The correct answer is B.

657

GMAT® Official Guide 2018

655. Last year all refuse collected by Shelbyville city services was incinerated. This incineration generated a large quantity of
residual ash. In order to reduce the amount of residual ash Shelbyville generates this year to half of last year’s total, the city
has revamped its collection program. This year city services will separate for recycling enough refuse to reduce the number
of truckloads of refuse to be incinerated to half of last year’s number.
Which of the following is required for the revamped collection program to achieve its aim?
(A)

This year, no materials that city services could separate for recycling will be incinerated.

(B)

Separating recyclable materials from materials to be incinerated will cost Shelbyville less than half what it cost last
year to dispose of the residual ash.

(C)

Refuse collected by city services will contain a larger proportion of recyclable materials this year than it did last year.

(D)

The refuse incinerated this year will generate no more residual ash per truckload incinerated than did the refuse
incinerated last year.

(E)

The total quantity of refuse collected by Shelbyville city services this year will be no greater than that collected last
year.

Argument Construction
Situation

To cut in half the residual ash produced at its incinerator, the city will separate for recycling
enough refuse to cut in half the number of truckloads of refuse going to the incinerator.

Reasoning

Which option is required if the city’s revamped collection program is to achieve its aim? Cutting the
number of truckloads of refuse in half must reduce the amount of residual ash to half last year’s
level. But if removal of the recycled refuse does not proportionately reduce the amount of ash,
this will not happen. So if the amount of residual ash produced per truckload increases after
recycling, then the amount of ash produced will not be cut in half by cutting in half the number of
truckloads.

A
B
C
D
E

This merely indicates that no further reduction of ash through recycling could be achieved this year; it
indicates nothing about how much the ash will be reduced.
This suggests a further benefit from recycling, but does not bear on the amount of ash that will be
produced.
Since no information is provided about how much, if any, recyclable materials were removed from the
refuse last year, this does not affect the reasoning.
Correct. This states a requirement for the collection program to achieve its aim.
This is not a requirement because even if the city collects more refuse this year, it could still cut in half the
amount of residual ash by cutting in half the number of truckloads going to the incinerator.

The correct answer is D.

658

8.6 Critical Reasoning Answer Explanations

656. Veterinarians generally derive some of their income from selling several manufacturers’ lines of pet-care products. Knowing
that pet owners rarely throw away mail from their pet’s veterinarian unread, one manufacturer of pet-care products offered
free promotional materials on its products to veterinarians for mailing to their clients. Very few veterinarians accepted the
offer, however, even though the manufacturer’s products are of high quality.
Which of the following, if true, most helps to explain the veterinarians’ reaction to the manufacturer’s promotional scheme?
(A)

Most of the veterinarians to whom the free promotional materials were offered were already selling the manufacturer’s
pet-care products to their clients.

(B)

The special promotional materials were intended as a supplement to the manufacturer’s usual promotional activities
rather than as a replacement for them.

(C)

The manufacturer’s products, unlike most equally good competing products sold by veterinarians, are also available in
pet stores and in supermarkets.

(D)

Many pet owners have begun demanding quality in products they buy for their pets that is as high as that in products
they buy for themselves.

(E)

Veterinarians sometimes recommend that pet owners use products formulated for people when no suitable product
specially formulated for animals is available.

Evaluation of a Plan
Situation

Veterinarians generally derive some income from selling various manufacturers’ pet-care products,
but very few veterinarians accepted free promotional materials from one such manufacturer to mail
to their clients.

Reasoning

What would most help explain why so few veterinarians accepted the free promotional materials to mail
to their clients? The passage says that veterinarians generally derive income from selling pet-care
products, which suggests that it should have been in many veterinarians’ financial interest to accept
and mail out the free promotional materials to increase sales. Any evidence that mailing out these
specific promotional materials from this manufacturer would not actually have been in many
veterinarians’ financial interest could help explain why so few veterinarians accepted the materials.

A
B

C

D

E

This suggests that most of the veterinarians should have had a financial interest in accepting and mailing
out the promotional materials in order to increase their sales of the manufacturer’s products.
Even if the promotional materials supplemented the manufacturer’s usual promotional activities, they
could still have increased the veterinarians’ sales of the manufacturer’s products and thus generated more
income for the veterinarians.
Correct. If this manufacturer’s products are available in pet stores and supermarkets but most other
products sold by veterinarians are not, then distributing the manufacturer’s promotional materials could
have encouraged customers to buy this manufacturer’s products from pet stores and supermarkets rather
than to buy competing products from the veterinarians. Thus, the veterinarians may have been concerned
that the promotions would reduce their profits.
The passage says the manufacturer’s products are of high quality, so we have no reason to suppose that
clients’ demand for quality products would discourage veterinarians from accepting the manufacturer’s
promotional materials.
Presumably the manufacturer’s products are specially formulated for pets, so any products veterinarians
recommend only when no specially formulated pet-care products are available would not reduce the
veterinarians’ interest in promoting the manufacturer’s products.

The correct answer is C.

659

GMAT® Official Guide 2018

657. The average hourly wage of television assemblers in Vernland has long been significantly lower than that in neighboring
Borodia. Since Borodia dropped all tariffs on Vernlandian televisions three years ago, the number of televisions sold annually
in Borodia has not changed. However, recent statistics show a drop in the number of television assemblers in Borodia.
Therefore, updated trade statistics will probably indicate that the number of televisions Borodia imports annually from
Vernland has increased.
Which of the following is an assumption on which the argument depends?
(A)

The number of television assemblers in Vernland has increased by at least as much as the number of television
assemblers in Borodia has decreased.

(B)

Televisions assembled in Vernland have features that televisions assembled in Borodia do not have.

(C)

The average number of hours it takes a Borodian television assembler to assemble a television has not decreased
significantly during the past three years.

(D)

The number of televisions assembled annually in Vernland has increased significantly during the past three years.

(E)

The difference between the hourly wage of television assemblers in Vernland and the hourly wage of television
assemblers in Borodia is likely to decrease in the next few years.

Argument Construction
Situation

Television assemblers in Vernland are paid less than those in neighboring Borodia. The number
of televisions sold in Borodia has not dropped since its tariffs on Vernlandian TVs were lowered
three years ago, but the number of TV assemblers in Borodia has. So TV imports from Vernland
have likely increased.

Reasoning

What assumption does the argument depend on? The fact that fewer individuals in Borodia are
working as TV assemblers is offered as evidence that TV imports from Vernland into Borodia
have likely increased. That piece of evidence is relevant only as an indication that the number of
TVs being produced within Borodia has decreased. But a drop in the number of TV assemblers
does not indicate a drop in the number of TVs being assembled if the number of TVs an average
assembler puts together has increased. Thus, the argument must be assuming that the average time
it takes an assembler to put together a TV has not significantly decreased.

A
B

C
D
E

The argument does not rely on any information about the number of television assemblers in Vernland nor
for that matter on the number of TVs assembled in Vernland.
The argument need not assume there is any difference in the features of the TVs produced in the two
countries. Increased sales of Vernlandian TVs in Borodia could be due to any number of other reasons,
such as price or quality.
Correct. This option states an assumption on which the argument depends.
The argument does not depend upon this being so: Vernland’s domestic TV sales (or perhaps its exports to
countries other than Borodia) may have decreased by more than its imports into Borodia have increased.
The argument’s conclusion addresses what has happened; the argument in no way relies on any
assumptions about what may or may not happen in the coming years.

The correct answer is C.

660

8.6 Critical Reasoning Answer Explanations

658. Guidebook writer: I have visited hotels throughout the country and have noticed that in those built before 1930 the quality of
the original carpentry work is generally superior to that in hotels built afterward. Clearly carpenters working on hotels before
1930 typically worked with more skill, care, and effort than carpenters who have worked on hotels built subsequently.
Which of the following, if true, most seriously weakens the guidebook writer’s argument?
(A)

The quality of original carpentry in hotels is generally far superior to the quality of original carpentry in other
structures, such as houses and stores.

(B)

Hotels built since 1930 can generally accommodate more guests than those built before 1930.

(C)

The materials available to carpenters working before 1930 were not significantly different in quality from the materials
available to carpenters working after 1930.

(D)

The better the quality of original carpentry in a building, the less likely that building is to fall into disuse and be
demolished.

(E)

The average length of apprenticeship for carpenters has declined significantly since 1930.

Argument Evaluation
Situation

The original carpentry in hotels built before 1930 shows superior care, skill, and effort to that in
hotels built after 1930. This leads to the conclusion that carpenters working on hotels before 1930
were superior in skill, care, and effort to those that came after.

Reasoning

Which option most seriously weakens the argument? The argument draws an inference from a
comparison between carpentry in hotels of different eras to a judgment about the carpenters
working on hotels in those eras. One way to weaken this inference is by finding some way in
which the carpentry in the hotels may be unrepresentative of the skill, care, and effort of the
carpenters working in the eras. The comparison is between the carpentry evident in hotels of
the two eras that still exist. Thus, if there is some reason to think that hotels with good carpentry
survive longer than those with bad carpentry, then still‐existing hotels from the older era will have
disproportionately more good carpentry, even assuming no difference between the skill, care, and
effort of the carpenters from the two eras.

A
B
C
D
E

This option applies equally to both eras, so it has no bearing on the argument.
It is not clear whether carpenters working on larger hotels would exercise more, less, or the same skill and
care as those working on smaller hotels; thus this option does not weaken the argument.
The argument does not rely, even implicitly, on there being any difference in the quality of materials used
in the two eras, so it does not weaken the argument to point out that no such difference exists.
Correct. This weakens the reasoning in the argument by showing a respect in which the comparison
between existing hotels is unrepresentative.
The longer a carpenter works as an apprentice, the more skill he or she is apt to have upon becoming a
full‐fledged carpenter. So this option would tend to slightly strengthen rather than weaken the argument.

The correct answer is D.

661

GMAT® Official Guide 2018

659. Scientists typically do their most creative work before the age of forty. It is commonly thought that this happens because
aging by itself brings about a loss of creative capacity. However, studies show that of scientists who produce highly
creative work beyond the age of forty, a disproportionately large number entered their field at an older age
than is usual. Since by the age of forty the large majority of scientists have been working in their field for at least fifteen
years, the studies’ finding strongly suggests that the real reason why scientists over forty rarely produce highly creative
work is not that they have aged but rather that scientists over forty have generally spent too long in their field.
In the argument given, the two portions in boldface play which of the following roles?
(A)

The first is a claim, the accuracy of which is at issue in the argument; the second is a conclusion drawn on the basis
of that claim.

(B)

The first is an objection that has been raised against a position defended in the argument; the second is that position.

(C)

The first is evidence that has been used to support an explanation that the argument challenges; the second is that
explanation.

(D)

The first is evidence that has been used to support an explanation that the argument challenges; the second is a
competing explanation that the argument favors.

(E)

The first provides evidence to support an explanation that the argument favors; the second is that explanation.

Argument Evaluation
Situation

It is generally thought that the reason scientists tend to do their most creative work before age
forty is that creative capacity declines with age. Yet those scientists who do creative work after
forty tend, disproportionately, to have started their careers in science later in life. So a better
explanation is that many scientists over forty have just been at it too long.

Reasoning

What roles do the two portions of the argument that are in boldface play? The argument describes a
phenomenon and what is commonly thought to explain it. Then, the first boldfaced statement
introduces evidence that suggests that there may be another explanation. After this evidence is
further developed, the argument then concludes that there is indeed a better explanation for the
phenomenon; that explanation is stated in the second boldfaced portion.

A
B
C
D
E

The accuracy of the first statement is never called into question by the argument; rather, it is relied upon as
the basis for the argument’s conclusion.
The first statement is not an objection against the position the argument defends; instead, it is a basis for
that position.
The first statement is not used to support a position the argument challenges, and the second statement is
the explanation the argument supports, not the one it challenges.
The second statement is indeed an explanation that the argument favors; but the first statement is not used
to support a competing explanation that the argument challenges.
Correct. This option correctly identifies the roles played by the boldfaced portions of the argument.

The correct answer is E.

662

8.6 Critical Reasoning Answer Explanations

660. NorthAir charges low fares for its economy-class seats, but it provides very cramped seating and few amenities. Market
research shows that economy passengers would willingly pay more for wider seating and better service, and additional
revenue provided by these higher ticket prices would more than cover the additional cost of providing these amenities. Even
though NorthAir is searching for ways to improve its profitability, it has decided not to make these improvements.
Which of the following, if true, would most help to explain NorthAir’s decision in light of its objectives?
(A)

None of NorthAir’s competitors offers significantly better seating and service to economy-class passengers than
NorthAir does.

(B)

On many of the routes that NorthAir flies, it is the only airline to offer direct flights.

(C)

A few of NorthAir’s economy-class passengers are satisfied with the service they receive, given the low price they
pay.

(D)

Very few people avoid flying on NorthAir because of the cramped seating and poor service offered in economy class.

(E)

The number of people who would be willing to pay the high fares NorthAir charges for its business-class seats would
decrease if its economy-class seating were more acceptable.

Evaluation of a Plan
Situation

Market research shows that improving some amenities for economy-class passengers would allow
NorthAir to raise its economy ticket prices more than enough to cover the additional cost of
providing those amenities. But NorthAir has decided not to improve those amenities, even though
it is looking for ways to improve its profitability.

Reasoning

What would most help explain why NorthAir decided not to improve the seating and other amenities,
even though the resulting increase in economy-class ticket prices would more than cover the expense?
NorthAir is looking for ways to improve its profitability. Making improvements that would
increase ticket prices enough to generate more revenue than they cost should improve profitability,
other things being equal. But if improving the amenities would generate side effects that reduced
profitability, those side effects would provide a good reason for NorthAir’s decision not to improve
the amenities and hence would help explain why NorthAir made that decision.

A

The passage says that for NorthAir, the cost of providing better economy seating and other amenities
would be more than met by the increased revenue from the higher ticket prices that passengers would be
willing to pay. This could give NorthAir a competitive edge, with improved profitability.
B
Even if NorthAir faces little or no competition on certain routes, offering extra amenities might increase
passengers’ interest in flying those routes. It might also lead passengers to choose NorthAir on other routes
that competing airlines also serve. Both of these effects could improve NorthAir’s profitability.
C Even if a few NorthAir economy passengers would not pay more for extra amenities, the market research
indicates that most of them would, so offering the amenities could still improve NorthAir’s profits
attributable to economy-class seating.
D This suggests that improving the amenities would not increase the total number of NorthAir passengers.
But improving the amenities might still enable the airline to increase its ticket prices per passenger enough
to improve its profitability.
E Correct. This suggests that improving the economy-class amenities would reduce NorthAir’s revenue
from sales of business-class tickets, which are likely much more expensive than economy-class tickets. This
reduction in revenue could be enough to reduce NorthAir’s total profitability despite the increased revenue
from economy-class ticket sales.
The correct answer is E.

663

GMAT® Official Guide 2018

661. Which of the following most logically completes the argument given?
Asthma, a chronic breathing disorder, is significantly more common today among adult competitive swimmers than it
is among competitive athletes who specialize in other sports. Although chlorine is now known to be a lung irritant and
swimming pool water is generally chlorinated, it would be rash to assume that frequent exposure to chlorine is the
explanation of the high incidence of asthma among these swimmers, since __________.
(A)

young people who have asthma are no more likely to become competitive athletes than are young people who do not
have asthma

(B)

competitive athletes who specialize in sports other than swimming are rarely exposed to chlorine

(C)

competitive athletes as a group have a significantly lower incidence of asthma than do people who do not participate
in competitive athletics

(D)

until a few years ago, physicians routinely recommended competitive swimming to children with asthma, in the belief
that this form of exercise could alleviate asthma symptoms

(E)

many people have asthma without knowing they have it and thus are not diagnosed with the condition until they begin
engaging in very strenuous activities, such as competitive athletics

Argument Construction
Situation

Asthma is more common among competitive swimmers than among other competitive athletes.
Chlorine is a lung irritant generally present in swimming pool water.

Reasoning

What would cast doubt on the hypothesis that exposure to chlorine in swimming pools accounts for the
high incidence of asthma among adult competitive swimmers? Evidence of any other factor that would
provide an alternative explanation of why asthma is more common among adult competitive
swimmers than among other competitive athletes would make it rash to assume that frequent
exposure to chlorine explains the high incidence of asthma among these swimmers, so a statement
providing such evidence would logically fill in the blank at the end of the passage to complete the
argument.

A

This might help explain why competitive athletes in general are not especially likely to have asthma, but it
does not explain why adult competitive swimmers are more likely to have asthma than other competitive
athletes are.
B
This provides additional evidence that exposure to chlorine explains why adult competitive swimmers
are more likely to have asthma than other competitive athletes are, so it does not cast doubt on that
hypothesis.
C A lower incidence of asthma among competitive athletes than among nonathletes does not help explain
the higher incidence of asthma among adult competitive swimmers than among other competitive athletes.
D Correct. Routinely encouraging children with asthma to take up competitive swimming would likely
have made the proportion of adult competitive swimmers with asthma exceed the proportion of other
competitive athletes with asthma, even if chlorine in swimming pool water never causes asthma in
swimmers.
E This might help explain why people with asthma are just as likely as other people to become competitive
athletes, but it does not help explain why adult competitive swimmers are more likely to have asthma than
other competitive athletes are.
The correct answer is D.

664

8.6 Critical Reasoning Answer Explanations

662. In the country of Marut, the Foreign Trade Agency’s records were reviewed in 1994 in light of information then newly
available about neighboring Goro. The review revealed that in every year since 1963, the agency’s projection of what Goro’s
gross national product (GNP) would be five years later was a serious underestimate. The review also revealed that in every
year since 1963, the agency estimated Goro’s GNP for the previous year—a Goro state secret—very accurately.
Of the following claims, which is most strongly supported by the statements given?
(A)

Goro’s GNP fluctuated greatly between 1963 and 1994.

(B)

Prior to 1995, Goro had not released data intended to mislead the agency in making its five‐year projections.

(C)

The amount by which the agency underestimated the GNP it projected for Goro tended to increase over time.

(D)

Even before the new information came to light, the agency had reason to think that at least some of the five‐year
projections it had made were inaccurate.

(E)

The agency’s five‐year projections of Goro’s GNP had no impact on economic planning in Marut.

Argument Construction
Situation

A review in 1994 revealed that every year since 1963, Marut’s Foreign Trade Agency had seriously
underestimated what Goro’s GNP would be five years later, but accurately estimated what Goro’s
GNP had been the previous year.

Reasoning

What conclusion do the stated facts most strongly support? Goro’s GNP in each year at least from 1969
through 1993 had been seriously underestimated by the agency five years in advance, yet was then
accurately estimated by the agency one year after the fact. It follows that for each of these years,
the agency’s earlier projection of Goro’s GNP must have been much lower than its later estimate.

A

B

C
D

E

This is not supported by the information given. The fact that the agency consistently underestimated each
year’s GNP in its five‐year projections and then correctly estimated it after the fact does not indicate that
Goro’s GNP fluctuated greatly.
This is not supported by the information given. The reason the agency’s five‐year projections were
inaccurate might well have been that Goro deliberately released data intended to mislead the agency in
making those projections.
This is not supported by the information given. The fact that the underestimates remained large
throughout the years in question does not indicate that the underestimates increased over time.
Correct. As explained above, for many years there were serious discrepancies between the agency’s
five‐year projections of Goro’s GNP and its retrospective estimates of each previous year’s trade. In any
year at least from 1970 through 1993, these discrepancies, if noticed, would have given the agency reason
to doubt some of the five‐year projections.
This is not supported by the information given. Even though at least some of the five‐year projections were
eventually known to be serious underestimates, they could still have affected Marut’s economic planning.
The economic planners might have retained an unreasonable faith in the accuracy of the most recent
projections.

The correct answer is D.

665

GMAT® Official Guide 2018

663. Vargonia has just introduced a legal requirement that student‐teacher ratios in government‐funded schools not exceed a
certain limit. All Vargonian children are entitled to education, free of charge, in these schools. When a recession occurs and
average incomes fall, the number of children enrolled in government‐funded schools tends to increase. Therefore, though
most employment opportunities contract in economic recessions, getting a teaching job in Vargonia’s government‐funded
schools will not be made more difficult by a recession.
Which of the following would be most important to determine in order to evaluate the argument?
(A)

Whether in Vargonia there are any schools not funded by the government that offer children an education free of
charge

(B)

Whether the number of qualified applicants for teaching positions in government‐funded schools increases significantly
during economic recessions

(C)

What the current student‐teacher ratio in Vargonia’s government‐funded schools is

(D)

What proportion of Vargonia’s workers currently hold jobs as teachers in government‐funded schools

(E)

Whether in the past a number of government‐funded schools in Vargonia have had student‐teacher ratios well in
excess of the new limit

Argument Evaluation
Situation

During a recession, the number of children in government‐funded schools in Vargonia tends to
increase. Vargonian children are entitled to a free education in these schools. A new law requires
student‐teacher ratios in these schools to remain below a certain limit.

Reasoning

Which of the five questions would provide us with the best information for evaluating the argument?
The argument’s conclusion is that recessions do not make teaching jobs in Vargonia’s
government‐funded schools harder to get. During recessions, the reasoning goes, more students
will enroll in Vargonia’s government‐funded schools than in nonrecession times. Implicit in the
argument is the thought that, because the new law sets an upper limit on the average number of
students per teacher, schools that get an influx of new students would have to hire more teachers.
During a recession, however, there might be much more competition in the labor market for
teachers because many more qualified people are applying for teaching jobs.

A
B

C

D

E

This information is not significant in the context of the argument, which does not need to assume that
only government‐funded schools provide free education.
Correct. Getting an answer to this question would provide us with specific information useful in
evaluating the argument. A “yes” answer to this question would suggest that competition for teaching jobs
in Vargonian government‐funded schools would be keener during recessions. A “no” answer would suggest
that the level of competition would decrease during recessions.
Discovering the current student‐teacher ratio in Vargonia’s schools would be of no value, by itself, in
evaluating the argument. We do not know what the new upper limit on the student‐teacher ratio is, and
we do not know whether Vargonia is currently in a recession.
Finding out whether the proportion this refers to is 1 percent, for example, or 4 percent, would tell us
nothing about whether getting teaching jobs at government‐funded schools in Vargonia becomes more
difficult during a recession. Among other things, we do not know whether Vargonia is currently in a
recession, and we do not know what proportion of Vargonia’s workers would be qualified candidates for
teaching jobs.
This is of no relevance in evaluating the argument because, presumably, the new limit on student-teacher
ratios will be complied with. Thus, even if student‐teacher ratios in the past would have exceeded the new
limit, the argument concerns whether, in the future, getting a teaching job in Vargonia’s government‐funded
schools will be made more difficult by a recession.

The correct answer is B.
666

8.6 Critical Reasoning Answer Explanations

664. In Colorado subalpine meadows, nonnative dandelions co‐occur with a native flower, the larkspur. Bumblebees visit both
species, creating the potential for interactions between the two species with respect to pollination. In a recent study,
researchers selected 16 plots containing both species; all dandelions were removed from eight plots; the remaining eight
control plots were left undisturbed. The control plots yielded significantly more larkspur seeds than the dandelion‐free plots,
leading the researchers to conclude that the presence of dandelions facilitates pollination (and hence seed production) in
the native species by attracting more pollinators to the mixed plots.
Which of the following, if true, most seriously undermines the researchers’ reasoning?
(A)

Bumblebees preferentially visit dandelions over larkspurs in mixed plots.

(B)

In mixed plots, pollinators can transfer pollen from one species to another to augment seed production.

(C)

If left unchecked, nonnative species like dandelions quickly crowd out native species.

(D)

Seed germination is a more reliable measure of a species’ fitness than seed production.

(E)

Soil disturbances can result in fewer blooms, and hence lower seed production.

Argument Evaluation
Situation

Bumblebees visit both larkspur and dandelions in certain meadows. A study found that more
larkspur seeds were produced in meadow plots in which both larkspur and dandelions grew
than in similar plots from which all dandelions had been removed. The researchers inferred that
dandelions facilitate larkspur pollination.

Reasoning

What evidence would cast the most doubt on the inference from the study’s findings to the conclusion that
dandelions facilitate larkspur pollination by attracting more pollinators? The argument assumes that
the only relevant difference between the two types of plots was whether dandelions were present.
Evidence that the plots differed in some other way that could provide a plausible alternative
explanation of why more larkspur seeds were produced in the plots with dandelions would weaken
the argument.

A
B

C
D
E

This would suggest that the larkspur pollination should have been lower in the plots with dandelions, so it
does not provide a plausible alternative explanation for the study’s findings.
This is fully compatible with the claim that the dandelions attracted more pollinators to the mixed plots,
and it would also help to support the argument’s conclusion that dandelions facilitated larkspur pollination
in those plots.
Although this suggests that the mixed plots won’t remain mixed for long, it does not provide a plausible
alternative explanation for the study’s finding that larkspur seed production was higher in the mixed plots.
The argument is not about how fit larkspurs are as a species, but about why they produced different
numbers of seeds in the different plots.
Correct. This provides a plausible alternative explanation for why larkspur seed production was lower in
the plots from which dandelions had been removed, since digging them out would have disturbed the soil.

The correct answer is E.

667

GMAT® Official Guide 2018

665. An experiment was done in which human subjects recognize a pattern within a matrix of abstract designs and then select
another design that completes that pattern. The results of the experiment were surprising. The lowest expenditure of energy
in neurons in the brain was found in those subjects who performed most successfully in the experiments.
Which of the following hypotheses best accounts for the findings of the experiment?
(A)

The neurons of the brain react less when a subject is trying to recognize patterns than when the subject is doing other
kinds of reasoning.

(B)

Those who performed best in the experiment experienced more satisfaction when working with abstract patterns than
did those who performed less well.

(C)

People who are better at abstract pattern recognition have more energy‐efficient neural connections.

(D)

The energy expenditure of the subjects’ brains increases when a design that completes the initially recognized pattern
is determined.

(E)

The task of completing a given design is more capably performed by athletes, whose energy expenditure is lower
when they are at rest.

Argument Construction
Situation

Experimental subjects worked with pattern recognition and completion. The subjects who
performed best showed the lowest expenditure of energy in neurons in the brain.

Reasoning

Which hypothesis best accounts for the findings? In order to account for the findings, the hypothesis
must suggest a plausible link between successful performance and the energy expenditure of
neurons in the brain. Consider each answer choice, and evaluate its plausibility and logic. Where is
there a reasonably direct relationship between the given factors and the conclusion that is drawn?
Understand that hypotheses based on factors not included in the experiment cannot be used to
account for the findings.

A
B
C
D
E

The experiment did not compare types of reasoning so this hypothesis does not account for the results.
No information is provided about subjects’ satisfaction, so this hypothesis is not warranted.
Correct. This statement properly identifies a hypothesis that connects subjects’ performance with their
energy expenditure and so could account for the experiment’s results.
The most successful subjects would presumably not have completed fewer patterns than average, so the
posited increase in energy would likely lead to higher energy expenditures for them, not lower.
No information is offered on the subjects, so no hypothesis about athletes is warranted.

The correct answer is C.

668

8.6 Critical Reasoning Answer Explanations

666. With seventeen casinos, Moneyland operates the most casinos in a certain state. Although intent on expanding, it was
outmaneuvered by Apex Casinos in negotiations to acquire the Eldorado chain. To complete its acquisition of Eldorado,
Apex must sell five casinos to comply with a state law forbidding any owner to operate more than one casino per county.
Since Apex will still be left operating twenty casinos in the state, it will then have the most casinos in the state.
Which of the following, if true, most seriously undermines the prediction?
(A)

Apex, Eldorado, and Moneyland are the only organizations licensed to operate casinos in the state.

(B)

The majority of Eldorado’s casinos in the state will need extensive renovations if they are to continue to operate
profitably.

(C)

Some of the state’s counties do not permit casinos.

(D)

Moneyland already operates casinos in the majority of the state’s counties.

(E)

Apex will use funds it obtains from the sale of the five casinos to help fund its acquisition of the Eldorado chain.

Argument Evaluation
Situation

Moneyland operates seventeen casinos, the most in a certain state, and is intent on expanding.
Another operator, Apex Casinos, is acquiring the Eldorado casino chain, but must sell five casinos
to comply with a state law forbidding any owner to operate more than one casino per county.
After these transactions, Apex will operate twenty casinos in the state.

Reasoning

What observation would cast the most doubt on the prediction that Apex will have the most casinos in
the state after the transactions? Apex will operate twenty casinos, whereas Moneyland now operates
just seventeen, and no one else operates even that many. It follows that Apex will operate more
casinos after its transactions than Moneyland or any other one owner now operates. However, if
Moneyland also acquires three or more casinos during the transactions, then Apex will not have
the most casinos in the state afterward. Thus, any observation suggesting that Moneyland is about
to acquire several casinos would undermine the prediction.

A

B
C

D

E

Correct. Since Apex is acquiring Eldorado, Moneyland and Apex will be the only remaining licensed
casino operators in the state. Therefore, Moneyland is the only likely buyer for the five casinos Apex needs
to sell. So Moneyland is likely to acquire the five casinos during the sale and end up with twenty‐two
casinos—more than Apex.
This does not undermine the prediction. Even if the Eldorado casinos cannot operate profitably for long
without extensive renovations, Apex will still have twenty casinos immediately after its transactions.
This supports rather than undermines the prediction. If fewer counties permit casinos, there will be fewer
opportunities for Moneyland or any other operator to acquire more casinos to surpass the twenty Apex
will own.
This supports rather than undermines the prediction. If Moneyland’s seventeen casinos are in most of the
state’s counties already, then there are fewer counties in which Moneyland could acquire additional casinos
to surpass the twenty Apex will own.
This supports rather than undermines the prediction. Apex’s use of the funds from selling the five casinos
to acquire the Eldorado chain will not help anyone else to acquire more casinos to surpass the twenty Apex
will own.

The correct answer is A.

669

GMAT® Official Guide 2018

667. It is widely assumed that people need to engage in intellectual activities such as solving crossword puzzles or mathematics
problems in order to maintain mental sharpness as they age. In fact, however, simply talking to other people—that is,
participating in social interaction, which engages many mental and perceptual skills—suffices. Evidence to this effect
comes from a study showing that the more social contact people report, the better their mental skills.
Which of the following, if true, most seriously weakens the force of the evidence cited?
(A)

As people grow older, they are often advised to keep exercising their physical and mental capacities in order to
maintain or improve them.

(B)

Many medical conditions and treatments that adversely affect a person’s mental sharpness also tend to increase that
person’s social isolation.

(C)

Many people are proficient both in social interactions and in solving mathematical problems.

(D)

The study did not itself collect data but analyzed data bearing on the issue from prior studies.

(E)

The tasks evaluating mental sharpness for which data were compiled by the study were more akin to mathematics
problems than to conversation.

Argument Evaluation
Situation

A study shows that the more social contact people report, the better their mental skills are, so
engaging in social interaction is sufficient for maintaining mental sharpness.

Reasoning

What would suggest that the study does not establish the truth of the conclusion? The study shows
a correlation between mental sharpness and social interaction but does not indicate why this
correlation exists. Evidence that mental sharpness contributes to social interaction or that some
third factor affects both mental sharpness and social interaction, could provide an alternative
explanation for the correlation and thus cast doubt on the explanation that social interaction
contributes to mental sharpness.

A

B

C
D
E

People are often wrongly advised to do things that are not actually beneficial. And even if exercising
mental capacities does help to maintain them, the passage says that social interaction provides such
exercise.
Correct. This provides evidence that the correlation observed in the study results from mental sharpness
facilitating social interaction, in which case the study results do not indicate that social interaction
facilitates mental sharpness.
This would be expected, given the argument’s conclusion that social interaction helps to maintain better
mental skills overall.
A study that analyzes data from prior studies can provide evidence just as well as a study that collects its
own data can.
The argument’s conclusion would be compatible with this observation, and would then suggest that social
interaction contributes to the mental sharpness needed for tasks similar to math problems.

The correct answer is B.

670

8.6 Critical Reasoning Answer Explanations

To register for the GMAT exam go to www.mba.com
671

9.0

672

Sentence Correction

9.0 Sentence Correction

9.0 Sentence Correction
Sentence correction questions appear in the Verbal section of the GMAT® exam. The Verbal section
uses multiple‐choice questions to measure your ability to read and comprehend written material, to
reason and evaluate arguments, and to correct written material to express ideas effectively in standard
written English. Because the Verbal section includes passages from several different content areas, you
may be generally familiar with some of the material; however, neither the passages nor the questions
assume detailed knowledge of the topics discussed. Sentence correction questions are intermingled with
critical reasoning and reading comprehension questions throughout the Verbal section of the test. You
will have 75 minutes to complete the Verbal section or about 1¾ minutes to answer each question.
Sentence correction questions present a statement in which words are underlined. The questions
ask you to select the best expression of the idea or relationship described in the underlined section
from the answer options. The first answer choice always repeats the original phrasing, whereas the
other four provide alternatives. In some cases, the original phrasing is the best choice. In other cases,
the underlined section has obvious or subtle errors that require correction. These questions require
you to be familiar with the stylistic conventions and grammatical rules of standard written English
and to demonstrate your ability to improve incorrect or ineffective expressions. Sentence correction
questions may include English‐language idioms, which are standard constructions not derived from the
most basic rules of grammar and vocabulary, but idioms are not intended to measure any specialized
knowledge of colloquialisms or regionalisms.
You should begin these questions by reading the sentence carefully. Note whether there are any obvious
grammatical errors as you read the underlined section. Then read the five answer choices carefully.
If there was a subtle error you did not recognize the first time you read the sentence, it may become
apparent after you have read the answer choices. If the error is still unclear, see whether you can
eliminate some of the answers as being incorrect. Remember that in some cases, the original selection
may be the best answer.

9.1 Basic English Grammar Rules
Sentence correction questions ask you to recognize and potentially correct at least one of the following
grammar rules. However, these rules are not exhaustive. If you are interested in learning more about
English grammar as a way to prepare for the GMAT exam, there are several resources available on the
Internet.

Agreement
Standard English requires elements within a sentence to be consistent. There are two types of
agreement: noun‐verb and pronoun.
Noun‐verb agreement: Singular subjects take singular verbs, whereas plural subjects take plural verbs.
Examples:
Correct: “I walk to the store.” Incorrect: “I walks to the store.”
Correct: “We go to school.” Incorrect: “We goes to school.”
Correct: “The number of residents has grown.” Incorrect: “The number of residents have grown.”
Correct: “The masses have spoken.” Incorrect: “The masses has spoken.”

673

GMAT® Official Guide 2018

Pronoun agreement: A pronoun must agree with the noun or pronoun it refers to in person, number, and
gender.
Examples:
Correct: “When you dream, you are usually asleep.” Incorrect: “When one dreams, you are usually
asleep.”
Correct: “When the kids went to sleep, they slept like logs.” Incorrect: “When the kids went to sleep, he
slept like a log.”

Diction
Words should be chosen to reflect correctly and effectively the appropriate part of speech. There are
several words that are commonly used incorrectly. When answering sentence correction questions, pay
attention to the following conventions.
Among/between: Among is used to refer to relationships involving more than two objects. Between is used
to refer to relationships involving only two objects.
Examples:
Correct: “We divided our winnings among the three of us.” Incorrect: “We divided our winnings
between the three of us.”
Correct: “She and I divided the cake between us.” Incorrect: “She and I divided the cake among us.”
As/like: As can be a preposition meaning “in the capacity of,” but more often is a conjunction of manner
and is followed by a verb. Like is generally used as a preposition, and therefore is followed by a noun, an
object pronoun, or a verb ending in ‐ing.
Examples:
Correct: “I work as a librarian.” Incorrect: “I work like a librarian.”
Correct: “Do as I say, not as I do.” Incorrect: “Do like I say, not like I do.”
Correct: “It felt like a dream.” Incorrect: “It felt as a dream.”
Correct: “People like you inspire me.” Incorrect: “People as you inspire me.”
Correct: “There’s nothing like biking on a warm, autumn day.” Incorrect: “There’s nothing as biking on a
warm, autumn day.”
Mass and count words: Mass words are nouns quantified by an amount rather than by a number. Count
nouns can be quantified by a number.
Examples:
Correct: “We bought a loaf of bread.” Incorrect: “We bought one bread.”
Correct: “He wished me much happiness.” Incorrect: “He wished me many happinesses.”
Correct: “We passed many buildings.” Incorrect: “We passed much buildings.”
Pronouns: Myself should not be used as a substitute for I or me.
Examples:
Correct: “Mom and I had to go to the store.” Incorrect: “Mom and myself had to go to the store.”
Correct: “He gave the present to Dad and me.” Incorrect: “He gave the present to Dad and myself.”

674

9.1 Sentence Correction Basic English Grammar Rules

Grammatical Construction
Good grammar requires complete sentences. Be on the lookout for improperly formed constructions.
Fragments: Parts of a sentence that are disconnected from the main clause are called fragments.
Examples:
Correct: “We saw the doctor and his nurse at the party.” Incorrect: “We saw the doctor at the party. And
his nurse.”
Run‐on sentences: A run‐on sentence is two or more independent clauses that run together without
proper punctuation.
Examples:
Correct: “Jose Canseco is still a feared batter; most pitchers don’t want to face him.” Incorrect: “Jose
Canseco is still a feared batter most pitchers don’t want to face him.”
Constructions: Avoid wordy, redundant constructions.
Examples:
Correct: “We could not come to the meeting because of a conflict.” Incorrect: “The reason we could not
come to the meeting is because of a conflict.”

Idiom
It is important to avoid nonstandard expressions, although English idioms sometimes do not follow
conventional grammatical rules. Be careful to use the correct idiom when using the constructions and
parts of speech.
Prepositions: Specific prepositions have specific purposes.
Examples:
Correct: “She likes to jog in the morning.” Incorrect: “She likes to jog on the morning.”
Correct: “They ranged in age from 10 to 15.” Incorrect: “They ranged in age from 10 up to 15.”
Correlatives: Word combinations such as “not only … but also” should be followed by an element of the
same grammatical type.
Examples:
Correct: “I have called not only to thank her but also to tell her about the next meeting.” Incorrect: “I
have called not only to thank her but also I told her about the next meeting.”
Forms of comparison: Many forms follow precise constructions. Fewer refers to a specific number, whereas
less than refers to a continuous quantity. Between … and is the correct form to designate a choice. Farther
refers to distance, whereas further refers to degree.
Examples:
Correct: “There were fewer children in my class this year.” Incorrect: “There were less children in my
class this year.”
Correct: “There was less devastation than I was told.” Incorrect: “There was fewer devastation than I was
told.”
Correct: “We had to choose between chocolate and vanilla.” Incorrect: “We had to choose between
chocolate or vanilla.” (It is also correct to say, “We had to choose chocolate or vanilla.”)
Correct: “I ran farther than John, but he took his weight training further than I did.”
Incorrect: “I ran further than John, but he took his weight training farther than I did.”

675

GMAT® Official Guide 2018

Logical Predication
Watch out for phrases that detract from the logical argument.
Modification problems: Modifiers should be positioned so it is clear what word or words they are meant
to modify. If modifiers are not positioned clearly, they can cause illogical references or comparisons, or
distort the meaning of the statement.
Examples:
Correct: “I put the cake that I baked by the door.” Incorrect: “I put the cake by the door that I baked.”
Correct: “Reading my mind, she gave me the delicious cookie.” Incorrect: “Reading my mind, the cookie
she gave me was delicious.”
Correct: “In the Middle Ages, the world was believed to be flat.” Incorrect: “In the Middle Ages, the
world was flat.”

Parallelism
Constructing a sentence that is parallel in structure depends on making sure that the different
elements in the sentence balance each other; this is a little bit like making sure that the two sides of a
mathematical equation are balanced. To make sure that a sentence is grammatically correct, check to
ensure that phrases, clauses, verbs, and other sentence elements parallel each other.
Examples:
Correct: “I took a bath, went to sleep, and woke up refreshed.” Incorrect: “I took a bath, sleeping, and
waking up refreshed.”
Correct: “The only way to know is to take the plunge.” Incorrect: “The only way to know is taking the
plunge.”

Rhetorical Construction
Good sentence structure avoids constructions that are awkward, wordy, redundant, imprecise, or unclear,
even when they are free of grammatical errors.
Examples:
Correct: “Before we left on vacation, we watered the plants, checked to see that the stove was off, and
set the burglar alarm.” Incorrect: “Before we left to go on vacation, we watered, checked to be sure that
the stove had been turned off, and set it.”

Verb Form
In addition to watching for problems of agreement or parallelism, make sure that verbs are used in the
correct tense. Be alert to whether a verb should reflect past, present, or future tense.
Examples:
Correct: “I went to school yesterday.” “I go to school every weekday.” “I will go to school tomorrow.”
Each tense also has a perfect form (used with the past participle—e.g., walked, ran), a progressive form
(used with the present participle—e.g., walking, running), and a perfect progressive form (also used with
the present participle—e.g., walking, running).
Present perfect: Used with has or have, the present perfect tense describes an action that occurred at an
indefinite time in the past or that began in the past and continues into the present.
Examples:
Correct: “I have traveled all over the world.” (at an indefinite time)
Correct: “He has gone to school since he was five years old.” (continues into the present)
676

9.1 Sentence Correction Basic English Grammar Rules

Past perfect: This verb form is used with had to show the order of two events that took place in the past.
Example:
Correct: “By the time I left for school, the cake had been baked.”
Future perfect: Used with will have, this verb form describes an event in the future that will precede
another event.
Example:
Correct: “By the end of the day, I will have studied for all my tests.”
Present progressive: Used with am, is, or are, this verb form describes an ongoing action that is happening
now.
Examples:
Correct: “I am studying for exams.” “The student is studying for exams.” “We are studying for exams.”
Past progressive: Used with was or were, this verb form describes something that was happening when
another action occurred.
Examples:
Correct: “The student was studying when the fire alarm rang.” “They were studying when the fire broke
out.”
Future progressive: Used with will be or shall be, this verb tense describes an ongoing action that will
continue into the future.
Example:
Correct: “The students will be studying for exams throughout the month of December.”
Present perfect progressive: Used with have been or has been, this verb tense describes something that
began in the past, continues into the present, and may continue into the future.
Example:
Correct: “The student has been studying hard in the hope of acing the test.”
Past perfect progressive: Used with had been, this verb form describes an action of some duration that was
completed before another past action occurred.
Example:
Correct: “Before the fire alarm rang, the student had been studying.”
Future perfect progressive: Used with will have been, this verb form describes a future, ongoing action that
will occur before a specified time.
Example:
Correct: “By the end of next year, the students will have been studying math for five years.”

677

GMAT® Official Guide 2018

9.2 Study Suggestions
There are two basic ways you can study for sentence correction questions:
• Read material that reflects standard usage.
One way to gain familiarity with the basic conventions of standard written English is simply to
read. Suitable material will usually be found in good magazines and nonfiction books, editorials in
outstanding newspapers, and the collections of essays used by many college and university writing
courses.
• Review basic rules of grammar and practice with writing exercises.
Begin by reviewing the grammar rules laid out in this chapter. Then, if you have school
assignments (such as essays and research papers) that have been carefully evaluated for
grammatical errors, it may be helpful to review the comments and corrections.

9.3 What Is Measured
Sentence correction questions test three broad aspects of language proficiency:
• Correct expression
A correct sentence is grammatically and structurally sound. It conforms to all the rules of standard
written English, including noun‐verb agreement, noun‐pronoun agreement, pronoun consistency,
pronoun case, and verb tense sequence. A correct sentence will not have dangling, misplaced,
or improperly formed modifiers; unidiomatic or inconsistent expressions; or faults in parallel
construction.
• Effective expression
An effective sentence expresses an idea or relationship clearly and concisely as well as
grammatically. This does not mean that the choice with the fewest and simplest words is
necessarily the best answer. It means that there are no superfluous words or needlessly complicated
expressions in the best choice.
• Proper diction
An effective sentence also uses proper diction. (Diction refers to the standard dictionary meanings
of words and the appropriateness of words in context.) In evaluating the diction of a sentence, you
must be able to recognize whether the words are well chosen, accurate, and suitable for the context.

9.4 Test-Taking Strategies
1. Read the entire sentence carefully.
Try to understand the specific idea or relationship that the sentence should express.
2.

Evaluate the underlined passage for errors and possible corrections before reading
the answer choices.
This strategy will help you discriminate among the answer choices. Remember, in some cases the
underlined passage is correct.

3. Read each answer choice carefully.
The first answer choice always repeats the underlined portion of the original sentence. Choose this
answer if you think that the sentence is best as originally written, but do so only after examining all
the other choices.
678

9.5 Sentence Correction The Directions

4.

Try to determine how to correct what you consider to be wrong with the original
sentence.
Some of the answer choices may change things that are not wrong, whereas others may not
change everything that is wrong.

5.

Make sure that you evaluate the sentence and the choices thoroughly.
Pay attention to general clarity, grammatical and idiomatic usage, economy and precision of
language, and appropriateness of diction.

6.

Read the whole sentence, substituting the choice that you prefer for the
underlined passage.
A choice may be wrong because it does not fit grammatically or structurally with the rest of the
sentence. Remember that some sentences will require no correction. When the given sentence
requires no correction, choose the first answer.

9.5 The Directions
These are the directions that you will see for sentence correction questions when you take the GMAT
exam. If you read them carefully and understand them clearly before going to sit for the test, you
will not need to spend too much time reviewing them once you are at the test center and the test is
under way.
Sentence correction questions present a sentence, part or all of which is underlined. Beneath the
sentence, you will find five ways of phrasing the underlined passage. The first answer choice repeats
the original underlined passage; the other four are different. If you think the original phrasing is best,
choose the first answer; otherwise, choose one of the others.
This type of question tests your ability to recognize the correctness and effectiveness of expression in
standard written English. In choosing your answer, follow the requirements of standard written English;
that is, pay attention to grammar, choice of words, and sentence construction. Choose the answer that
produces the most effective sentence; this answer should be clear and exact, without awkwardness,
ambiguity, redundancy, or grammatical error.

679

GMAT® Official Guide 2018

9.6 Practice Questions
Each of the sentence correction questions presents a sentence, part or all of which is underlined.
Beneath the sentence you will find five ways of phrasing the underlined part. The first of these repeats
the original; the other four are different. Follow the requirements of standard written English to choose
your answer, paying attention to grammar, word choice, and sentence construction. Select the answer
that produces the most effective sentence; your answer should make the sentence clear, exact, and free
of grammatical error. It should also minimize awkwardness, ambiguity, and redundancy.

668. In a review of 2,000 studies of human behavior that
date back to the 1940s, two Swiss psychologists,
declaring that since most of the studies had failed to
control for such variables as social class and family
size, none could be taken seriously.
(A)

(B)

psychologists, declaring that since most of the
studies had failed to control for such variables as
social class and family size,
psychologists, declaring that most of the studies
failed in not controlling for such variables like
social class and family size, and

(C)

psychologists declared that since most of
the studies, having failed to control for such
variables as social class and family size,

(D)

psychologists declared that since most of the
studies fail in controlling for such variables like
social class and family size,

(E)

psychologists declared that since most of the
studies had failed to control for variables such as
social class and family size,

669. In her presentation, the head of the Better Business
Bureau emphasized that companies should think of the
cost of conventions and other similar gatherings as not
an expense, but as an investment in networking that
will pay dividends.

680

(A)

as not an expense, but as

(B)

as not expense but

(C)

not an expense, rather

(D)

not as an expense, but as

(E)

not in terms of expense, but

670. Recent interdisciplinary studies advance the argument
that emotions, including those deemed personal
or private is a social phenomenon, though one
inseparable from bodily response.
(A)

private is a social phenomenon, though one
inseparable

(B)

private, are social phenomena that are inseparable

(C)

private are a social phenomenon but are not
those separable

(D)

private—are social phenomena but not separable

(E)

also as private emotions, are social phenomena
not inseparable

671. Manufacturers rate batteries in watt‐hours; if they rate
the watt‐hour higher, the longer the battery can be
expected to last.
(A)

if they rate the watt‐hour higher, the longer

(B)

rating the watt‐hour higher, it is that much longer

(C)

the higher the watt‐hour rating, the longer

(D)

the higher the watt‐hour rating, it is that much
longer that

(E)

when the watt‐hour rating is higher, the longer it is

672. At the end of the 1930s, Duke Ellington was looking
for a composer to assist him—someone not only who
could arrange music for his successful big band, but
mirroring his eccentric writing style as well in order
to finish the many pieces he had started but never
completed.
(A)

someone not only who could arrange music
for his successful big band, but mirroring his
eccentric writing style as well in order to finish

(B)

someone who could not only arrange music
for his successful big band, but also mirror his
eccentric writing style in order to finish

9.6 Sentence Correction Practice Questions

(C)

someone who not only could arrange music for
his successful big band, but also to mirror his
eccentric writing style in finishing

(D)

that being someone who could not only arrange
music for his successful big band, but mirroring
his eccentric writing style for finishing

(E)

being someone not only who could arrange
music for his successful big band, but mirror his
eccentric writing style as well, finishing

673. While Noble Sissle may be best known for his
collaboration with Eubie Blake, as both a vaudeville
performer and as a lyricist for songs and Broadway
musicals, also enjoying an independent career as a
singer with such groups as Hahn’s Jubilee Singers.
(A)

and as a lyricist for songs and Broadway
musicals, also enjoying

(B)

and writing lyrics for songs and Broadway
musicals, also enjoying

(C)

and a lyricist for songs and Broadway musicals,
he also enjoyed

(D)

as well as writing lyrics for songs and Broadway
musicals, he also enjoyed

(E)

as well as a lyricist for songs and Broadway
musicals, he had also enjoyed

674. Air traffic routes over the North Pole are currently used
by only two or three planes a day, but it was found
by a joint Canadian–Russian study to be both feasible
as well as desirable if those routes are opened to
thousands more commercial planes a year.
(A)

(B)

(C)

Air traffic routes over the North Pole are
currently used by only two or three planes a day,
but it was found by a joint Canadian–Russian
study to be both feasible as well as desirable
if those routes are opened to thousands more
commercial planes a year.
Currently used by only two or three planes a day,
a joint Canadian–Russian study has found that if
air traffic routes over the North Pole are opened
to thousands more commercial planes a year, it
would be both feasible and desirable.

(D)

Although air traffic routes over the North Pole
are currently used by only two or three planes a
day, a joint Canadian–Russian study has found
that opening those routes to thousands more
commercial planes a year is both feasible and
desirable.

(E)

With air traffic routes over the North Pole
currently used by only two or three planes a
day, opening those routes to thousands more
commercial planes a year has been found by a
joint Canadian—Russian study as both feasible
and desirable.

675. From an experiment using special extrasensory
perception cards, each bearing one of a set of
symbols, parapsychologist Joseph Banks Rhine
claimed statistical proof for subjects who could use
thought transference to identify a card in the dealer’s
hand.
(A)

for subjects who could use thought transference
to identify a card in the dealer’s hand

(B)

for a card in the dealer’s hand to be identified by
subjects with thought transference

(C)

of subjects able to identify with thought
transference a card in the dealer’s hand

(D)

that subjects could identify a card in the dealer’s
hand by using thought transference

(E)

that subjects are capable to use thought
transference for identifying a card in the dealer’s
hand

676. A long‐term study of some 1,000 physicians indicates
that the more coffee these doctors drank, the more
they had a likelihood of coronary disease.
(A)

more they had a likelihood of coronary disease

(B)

more was their likelihood of having coronary
disease

(C)

more they would have a likelihood to have
coronary disease

(D)

greater was their likelihood of having coronary
disease

(E)

greater was coronary disease likely

A joint Canadian–Russian study, finding it to be
both feasible as well as desirable to open air
traffic routes over the North Pole, which are
currently used by only two or three planes a day,
to thousands more commercial planes a year.
681

GMAT® Official Guide 2018

677. The intricate structure of the compound insect eye,
having hundreds of miniature eyes called ommatidia,
help explain why scientists have assumed that it
evolved independently of the vertebrate eye.
(A)

having hundreds of miniature eyes called
ommatidia, help explain why scientists have
assumed that it

(B)

having hundreds of miniature eyes that are called
ommatidia, helps explain why scientists have
assumed that they

(C)

(D)
(E)

with its hundreds of miniature eyes that are
called ommatidia, helps explain scientists’
assuming that they
with its hundreds of miniature eyes called
ommatidia, help explain scientists’ assuming that it

(A)

less than those of the 1978 harvest

(B)

less than the 1978 harvest

(C)

less than 1978

(D)

fewer than 1978

(E)

fewer than that of India’s 1978 harvest

679. Hurricanes at first begin traveling from east to west,
because that direction is the way the prevailing winds
in the tropics blow, but they then veer off toward
higher latitudes, in many cases changing direction
toward the east before dissipating over the colder,
more northerly waters or over land.

(B)

(C)

(D)

682

Hurricanes, beginning by traveling from east
to west, because this is the direction of the
prevailing winds in the tropics,

680. Unlike the virginal, whose single set of strings runs
parallel to the front edge of the instrument, the
harpsichord’s several sets of strings are placed at right
angles to its front edge.
(A)

whose single set of strings runs parallel to the
front edge of the instrument, the harpsichord’s
several sets of strings are

(B)

with a single set of strings running parallel to the
front edge of the instrument, the several sets of
strings of the harpsichord are

(C)

which has a single set of strings that runs
parallel to the front edge of the instrument, in the
case of the harpsichord, several sets of strings
are

(D)

which has a single set of strings that run
parallel to the front edge of the instrument, the
harpsichord has several sets of strings

(E)

in which a single set of strings run parallel to the
front edge of the instrument, the harpsichord’s
several sets of strings are

with its hundreds of miniature eyes called
ommatidia, helps explain why scientists have
assumed that it

678. In 1979 lack of rain reduced India’s rice production
to about 41 million tons, nearly 25 percent less than
those of the 1978 harvest.

(A)

(E)

681. Although Alice Walker published a number of essays,
poetry collections, and stories during the 1970s, her
third novel, The Color Purple, which was published in
1982, brought her the widest acclaim in that it won
both the National Book Award as well as the Pulitzer
Prize.
(A)

which was published in 1982, brought her the
widest acclaim in that it won both the National
Book Award as well as the Pulitzer Prize

(B)

Hurricanes at first begin traveling from east
to west, because that direction is the way the
prevailing winds in the tropics blow, but

published in 1982, bringing her the widest
acclaim by winning both the National Book Award
and the Pulitzer Prize

(C)

At first, hurricanes travel from east to west,
because that is the direction of the prevailing
winds in the tropics, but

published in 1982, brought her the widest
acclaim, winning both the National Book Award
and the Pulitzer Prize

(D)

While hurricanes travel from east to west at first,
the direction of the prevailing winds blowing in
the tropics, and

was published in 1982 and which, winning both
the National Book Award and the Pulitzer Prize,
brought her the widest acclaim

(E)

was published in 1982, winning both the National
Book Award as well as the Pulitzer Prize, and
bringing her the widest acclaim

Because hurricanes at first travel from east to
west, since it is the direction of the prevailing
winds in the tropics,

9.6 Sentence Correction Practice Questions

682. Many experts regarded the large increase in credit
card borrowing in March not as a sign that households
were pressed for cash and forced to borrow, rather
a sign of confidence by households that they could
safely handle new debt.
(A)

rather a sign of confidence by households that
they could safely

(B)

yet as a sign of households’ confidence that it
was safe for them to

(C)

but a sign of confidence by households that they
could safely

(D)

but as a sign that households were confident
they could safely

(E)

but also as a sign that households were
confident in their ability safely to

683. A pioneer journalist, Nellie Bly’s exploits included
circling the globe faster than Jules Verne’s fictional
Phileas Fogg.
(A)

A pioneer journalist, Nellie Bly’s exploits included

(B)

The exploits of Nellie Bly, a pioneer journalist,
included

(C)

Nellie Bly was a pioneer journalist including in her
exploits the

(D)

Included in the pioneer journalist Nellie Bly’s
exploits are

(E)

The pioneer journalist’s exploits of Nellie Bly
included

684. In 1913, the largely self-taught Indian mathematician
Srinivasa Ramanujan mailed 120 of his theorems to
three different British mathematicians; only one, G. H.
Hardy, recognized the brilliance of these theorems,
but thanks to Hardy’s recognition, Ramanujan was
eventually elected to the Royal Society of London.

(D)

but, only one, G. H. Hardy, recognizing their
brilliance,

(E)

only one G. H. Hardy recognized, but these
theorems were brilliant

685. Cost cutting and restructuring has allowed the
manufacturing company to lower its projected losses
for the second quarter, and they are forecasting a
profit before the end of the year.
(A)

has allowed the manufacturing company to lower
its projected losses for the second quarter, and
they are forecasting

(B)

has allowed for the manufacturing company to
lower its projected losses in the second quarter
and to forecast

(C)

have allowed that the manufacturing company
can lower the projected losses for the second
quarter, and to forecast

(D)

have allowed the manufacturing company to
lower its projected second-quarter losses and to
forecast

(E)

have allowed for the manufacturing company to
lower the projected losses in the second quarter,
as well as forecasting

686. The Life and Casualty Company hopes that by
increasing its environmental fund reserves to $1.2
billion, that it has set aside enough to pay for
environmental claims and no longer has to use its
profits and capital to pay those claims bit by bit, year
by year.
(A)

that it has set aside enough to pay for
environmental claims and no longer has

(B)

enough has been set aside with which environmental
claims can be paid and it will have no longer

(C)
(A)

only one, G. H. Hardy, recognized the brilliance
of these theorems, but

it has set aside enough for payment of
environmental claims and thus no longer having

(D)

(B)

they were brilliant, G. H. Hardy alone recognized,
but

enough has been set aside to pay for
environmental claims, thus no longer having

(E)

(C)

these theorems were brilliant, but only one, G. H.
Hardy recognized;

it has set aside enough to pay for environmental
claims and will no longer have

683

GMAT® Official Guide 2018

687. Like ancient Egyptian architectural materials that
were recycled in the construction of ancient Greek
Alexandria, so ancient Greek materials from the
construction of that city were reused in subsequent
centuries by Roman, Muslim, and modern builders.
(A)

Like ancient Egyptian architectural materials that
were recycled in the construction of

(B)

Like recycling ancient Egyptian architectural
materials to construct

(C)

Just as ancient Egyptian architectural materials
were recycled in the construction of

(D)
(E)

fasts, due to having the ability of surviving

(D)

fasts because they are able to survive

(E)

fasts because of having the ability of surviving

691. Thai village crafts, as with other cultures, have
developed through the principle that form follows
function and incorporate readily available materials
fashioned using traditional skills.
(A)

as with

(B)

as did those of

Just as they recycled ancient Egyptian
architectural materials in constructing

(C)

as they have in

(D)

like in

Just like ancient Egyptian architectural materials
that were recycled in constructing

(E)

like those of

688. Especially in the early years, new entrepreneurs may
need to find resourceful ways, like renting temporary
office space or using answering services, that make
their company seem large and more firmly established
than they may actually be.
(A)

that make their company seem large

(B)

to make their companies seem larger

(C)

thus making their companies seem larger

(D)

so that the companies seem larger

(E)

of making their company seem large

689. Unlike the nests of leaf cutters and most other ants,
situated underground or in pieces of wood, raider
ants make a portable nest by entwining their long
legs to form “curtains” of ants that hang from logs or
boulders, providing protection for the queen and the
colony larvae and pupae.
(A)

the nests of leaf cutters and most other ants,

(B)

the nests of leaf cutters and most other ants,
which are

(C)

leaf cutters and most other ants, whose nests are

(D)

leaf cutters and most other ants in having nests

(E)

those of leaf cutters and most other ants with nests

690. Turtles, like other reptiles, can endure long fasts,
in their ability to survive on weekly or even monthly
feedings; however, when food is readily available, they
may eat frequently and grow very fat.

684

(C)

(A)

fasts, in their ability to survive

(B)

fasts, having their ability to survive

692. Tropical bats play important roles in the rain forest
ecosystem, aiding in the dispersal of cashew, date,
and fig seeds; pollinating banana, breadfruit, and
mango trees; and indirectly help produce tequila by
pollinating agave plants.
(A)

pollinating banana, breadfruit, and mango trees;
and indirectly help produce

(B)

pollinating banana, breadfruit, and mango trees;
and indirectly helping to produce

(C)

pollinating banana, breadfruit, and mango trees;
and they indirectly help to produce

(D)

they pollinate banana, breadfruit, and mango
trees; and indirectly help producing

(E)

they pollinate banana, breadfruit, and mango
trees; indirectly helping the producing of

693. Discussion of greenhouse effects have usually had
as a focus the possibility of Earth growing warmer
and to what extent it might, but climatologists have
indicated all along that precipitation, storminess, and
temperature extremes are likely to have the greatest
impact on people.
(A)

Discussion of greenhouse effects have usually
had as a focus the possibility of Earth growing
warmer and to what extent it might,

(B)

Discussion of greenhouse effects has usually
had as its focus whether Earth would get warmer
and what the extent would be,

(C)

Discussion of greenhouse effects has usually
focused on whether Earth would grow warmer
and to what extent,

9.6 Sentence Correction Practice Questions

(D)

The discussion of greenhouse effects have
usually focused on the possibility of Earth getting
warmer and to what extent it might,

(E)

The discussion of greenhouse effects has usually
focused on whether Earth would grow warmer
and the extent that is,

694. In the seventh century B.C., the Roman alphabet was
adapted from the Etruscan alphabet, which in turn had
been adapted in the previous century from a western
Greek alphabet, which itself had been adapted earlier
in the same century from the Phoenician alphabet.
(A)

which itself had been adapted earlier

(B)

adapting itself earlier

(C)

itself being adapted earlier

(D)

having been earlier adapted itself

(E)

earlier itself having been adapted

695. With corn, soybean, and wheat reserves being low
enough so a poor harvest would send prices skyrocketing, grain futures brokers and their clients are
especially interested in weather that could affect crops.
(A)

being low enough so

(B)

so low such that

(C)

so low that

(D)

that are low enough so

(E)

that are so low such that

696. By 1940, the pilot Jacqueline Cochran held seventeen
official national and international speed records,
and she earned them at a time when aviation was
still so new for many of the planes she flew to be of
dangerously experimental design.
(A)
(B)
(C)
(D)
(E)

697. The foundation works to strengthen local and
regional agricultural markets and cooperating with
governments, improving access for farmers for
productive resources such as land and credit.
(A)

cooperating with governments, improving access
for farmers for

(B)

cooperates with governments to improve access
for farmers to

(C)

cooperate with governments for improvements
of access for farmers to

(D)

cooperate with governments and improve
accessibility for farmers for their

(E)

in cooperation with governments to improve
access for farmers for

698. A professor at the university has taken a sabbatical to
research on James Baldwin’s books that Baldwin wrote
in France while he was living there.
(A)

on James Baldwin’s books that Baldwin wrote in
France while he was living there

(B)

about the books James Baldwin wrote in France

(C)

into James Baldwin’s books written while in
France

(D)

on the books of James Baldwin, written while he
lived in France

(E)

the books James Baldwin wrote while he lived in
France

699. Researchers now regard interferon as not a single
substance, but it is rather a biological family of
complex molecules that play an important, though not
entirely defined, role in the immune system.
(A)

and she earned them at a time when aviation was
still so new for many of the planes she flew to be

as not a single substance, but it is rather a
biological family of complex molecules that play

(B)

earning them at a time that aviation was still so
new for many of the planes she flew to be

as not a single substance but as a biological
family of complex molecules playing

(C)

earning these at a time where aviation was still
so new that many of the planes she flew were

not as a single substance but as a biological
family of complex molecules that play

(D)

earned at a time in which aviation was still so
new such that many of the planes she flew were

not to be a single substance but rather a
biological family of complex molecules playing

(E)

not as a single substance but instead as being a
biological family of complex molecules that play

earned at a time when aviation was still so new
that many of the planes she flew were

685

GMAT® Official Guide 2018

700. The remarkable similarity of Thule artifacts throughout
a vast region can, in part, be explained as a very rapid
movement of people from one end of North America to
the other.
(A)

The remarkable similarity of Thule artifacts
throughout a vast region can, in part, be
explained as

(B)

Thule artifacts being remarkably similar
throughout a vast region, one explanation is

(C)

That Thule artifacts are remarkably similar
throughout a vast region is, in part, explainable as

(D)

One explanation for the remarkable similarity of
Thule artifacts throughout a vast region is that
there was

(E)

Throughout a vast region Thule artifacts are
remarkably similar, with one explanation for this
being

701. The widely accepted big bang theory holds that the
universe began in an explosive instant ten to twenty
billion years ago and has been expanding ever since.
(A)

(B)

(C)

that the universe began in an explosive instant
ten to twenty billion years ago and has been
expanding
that the universe had begun in an explosive
instant ten to twenty billion years ago and had
been expanding
that the beginning of the universe was an
explosive instant ten to twenty billion years ago
that has expanded

(D)

the beginning of the universe to have been an
explosive instant ten to twenty billion years ago
that is expanding

(E)

the universe to have begun in an explosive
instant ten to twenty billion years ago and has
been expanding

702. Between 14,000 and 8,000 B.C. the ice cap that
covered northern Asia, Europe, and America began
to melt, uncovering vast new areas that were to be
occupied by migrating peoples moving northward.

686

(A)

began to melt, uncovering vast new areas that
were to be occupied

(B)

began melting, to uncover vast new areas to be
occupied

(C)

began, by melting, to uncover vast new areas for
occupation

(D)

began, after melting, uncovering vast new areas
which are to be occupied

(E)

would begin to uncover, through melting, vast
new areas for occupation

703. Bengal‐born writer, philosopher, and educator
Rabindranath Tagore had the greatest admiration
for Mohandas K. Gandhi the person and also as a
politician, but Tagore had been skeptical of Gandhi’s
form of nationalism and his conservative opinions
about India’s cultural traditions.
(A)

for Mohandas K. Gandhi the person and also as
a politician, but Tagore had been

(B)

for Mohandas K. Gandhi as a person and as a
politician, but Tagore was also

(C)

for Mohandas K. Gandhi not only as a person
and as a politician, but Tagore was also

(D)

of Mohandas K. Gandhi as a person and as also
a politician, but Tagore was

(E)

of Mohandas K. Gandhi not only as a person and
as a politician, but Tagore had also been

704. Traffic safety officials predict that drivers will be
equally likely to exceed the proposed speed limit as
the current one.
(A)

equally likely to exceed the proposed speed limit
as

(B)

equally likely to exceed the proposed speed limit
as they are

(C)

equally likely that they will exceed the proposed
speed limit as

(D)

as likely that they will exceed the proposed
speed limit as

(E)

as likely to exceed the proposed speed limit as
they are

705. Written early in the French Revolution, Mary
Wollstonecraft’s A Vindication of the Rights of Man
(1790) and A Vindication of the Rights of Woman
(1792) attributed Europe’s social and political ills to be
the result of the dominance of aristocratic values and
patriarchal hereditary privilege.
(A)

Mary Wollstonecraft’s A Vindication of the Rights
of Man (1790) and A Vindication of the Rights
of Woman (1792) attributed Europe’s social and
political ills to be the result of

9.6 Sentence Correction Practice Questions

(B)

Mary Wollstonecraft’s A Vindication of the Rights
of Man (1790) and A Vindication of the Rights
of Woman (1792) attributed Europe’s social and
political ills to result from

(C)

Mary Wollstonecraft’s A Vindication of the Rights
of Man (1790) and A Vindication of the Rights
of Woman (1792) attributed Europe’s social and
political ills to

(D)

in A Vindication of the Rights of Man (1790) and
A Vindication of the Rights of Woman (1792),
Mary Wollstonecraft attributed Europe’s social
and political ills to have been the result of

(E)

Mary Wollstonecraft, in A Vindication of the
Rights of Man (1790) and A Vindication of the
Rights of Woman (1792), attributed Europe’s
social and political ills to

706. Using study groups managed by the principal popular
organizations and political parties, the Swedish public
was informed by the government about energy and
nuclear power.
(A)

the Swedish public was informed by the
government about energy and nuclear power

(B)

the government informed the Swedish public
about energy and nuclear power

(C)

energy and nuclear power information was given
to the Swedish public by the government

(D)

information about energy and nuclear power was
given to the Swedish public by the government

(E)

the public of Sweden was given energy and
nuclear power information by the government

707. The use of the bar code, or Universal Product Code,
which was created in part to enable supermarkets to
process customers at a faster rate, has expanded
beyond supermarkets to other retail outlets and
have become readily accepted despite some initial
opposition when it was first introduced in 1974.
(A)
(B)

(C)

(D)

has become readily accepted despite some
initial opposition when the bar code was first
introduced in 1974

(E)

bar codes have become readily accepted
despite some initial opposition when it was first
introduced in 1974

708. The Organization of Petroleum Exporting Countries
(OPEC) had long been expected to announce a
reduction in output to bolster sagging oil prices, but
officials of the organization just recently announced
that the group will pare daily production by 1.5 million
barrels by the beginning of next year, but only if
non‐OPEC nations, including Norway, Mexico, and
Russia, were to trim output by a total of 500,000
barrels a day.
(A)

year, but only if non‐OPEC nations, including
Norway, Mexico, and Russia, were to trim output

(B)

year, but only if the output of non‐OPEC nations,
which includes Norway, Mexico, and Russia, is
trimmed

(C)

year only if the output of non‐OPEC nations,
including Norway, Mexico, and Russia, would be
trimmed

(D)

year only if non‐OPEC nations, which includes
Norway, Mexico, and Russia, were trimming
output

(E)

year only if non‐OPEC nations, including Norway,
Mexico, and Russia, trim output

709. Not trusting themselves to choose wisely among the
wide array of investment opportunities on the market,
stockbrokers are helping many people who turn to them
to buy stocks that could be easily bought directly.
(A)

stockbrokers are helping many people who turn
to them to buy stocks that could be easily

(B)

have become readily accepted despite some initial
opposition when it was first introduced in 1974

stockbrokers are helping many people who are
turning to them for help in buying stocks that
they could easily have

(C)

has become readily accepted despite some
initial opposition when they were first introduced
in 1974

many people are turning to stockbrokers for help
from them to buy stocks that could be easily

(D)

many people are turning to stockbrokers for help
to buy stocks that easily could have been

(E)

many people are turning to stockbrokers for help
in buying stocks that could easily be

have become readily accepted despite some
initial opposition when first introduced in 1974

687

GMAT® Official Guide 2018

710. There are several ways to build solid walls using just
mud or clay, but the most extensively used method
has been the forming of bricks out of mud or clay, and,
after some preliminary air drying or sun drying, they
are laid in the wall in mud mortar.
(A)

(B)

the forming of bricks out of mud or clay, and,
after some preliminary air drying or sun drying,
they are laid
forming the mud or clay into bricks, and, after
some preliminary air drying or sun drying, to lay
them

(C)

having bricks formed from mud or clay, and,
after some preliminary air drying or sun drying,
they were laid

(D)

to form the mud or clay into bricks, and, after
some preliminary air drying or sun drying, to lay
them

(E)

that bricks were formed from mud or clay, which,
after some preliminary air drying or sun drying,
were laid

711. A surge in new home sales and a drop in weekly
unemployment claims suggest that the economy
might not be as weak as some analysts previously
thought.
(A)

claims suggest that the economy might not be
as weak as some analysts previously thought

(B)

claims suggests that the economy might not
be so weak as some analysts have previously
thought

(C)

claims suggest that the economy might not be
as weak as have been previously thought by
some analysts

(D)

claims, suggesting about the economy that it
might not be so weak as previously thought by
some analysts

(E)

claims, suggesting the economy might not be
as weak as previously thought to be by some
analysts

712. Retail sales rose 0.8 of 1 percent in August,
intensifying expectations that personal spending in the
July–September quarter more than doubled that of the
1.4 percent growth rate in personal spending for the
previous quarter.
(A)
688

that personal spending in the July–September
quarter more than doubled that of

(B)

that personal spending in the July–September
quarter would more than double

(C)

of personal spending in the July–September
quarter, that it more than doubled

(D)

of personal spending in the July–September
quarter more than doubling that of

(E)

of personal spending in the July–September
quarter, that it would more than double that of

713. In the early part of the twentieth century, many
vacationers found that driving automobiles and sleeping
in tents allowed them to enjoy nature close at hand and
tour at their own pace, with none of the restrictions of
passenger trains and railroad timetables or with the
formalities, expenses, and impersonality of hotels.
(A)

with none of the restrictions of passenger trains
and railroad timetables or with the

(B)

with none of the restrictions of passenger trains,
railroad timetables, nor

(C)

without the restrictions of passenger trains and
railroad timetables nor

(D)

without the restrictions of passenger trains and
railroad timetables or with the

(E)

without the restrictions of passenger trains and
railroad timetables or the

714. Over the next few years, increasing demands on the
Chattahoochee River, which flows into the Apalachicola
River, could alter the saline content of Apalachicola
Bay, which would rob the oysters there of their flavor,
and to make them decrease in size, less distinctive,
and less in demand.
(A)

which would rob the oysters there of their flavor,
and to make them decrease in size,

(B)

and it would rob the oysters there of their flavor,
make them smaller,

(C)

and rob the oysters there of their flavor, making
them decrease in size,

(D)

robbing the oysters there of their flavor and
making them smaller,

(E)

robbing the oysters there of their flavor, and
making them decrease in size,

9.6 Sentence Correction Practice Questions

715. Elizabeth Barber, the author of both Prehistoric
Textiles, a comprehensive work on cloth in the
early cultures of the Mediterranean, and also of
Women’s Work, a more general account of early cloth
manufacture, is an expert authority on textiles in
ancient societies.
(A)

also of Women’s Work, a more general account
of early cloth manufacture, is an expert authority
on

(B)

also Women’s Work, a more general account of
cloth manufacture, is an expert authority about

(C)

of Women’s Work, a more general account about
early cloth manufacture, is an authority on

(D)

of Women’s Work, a more general account about
early cloth manufacture, is an expert authority
about

(E)

Women’s Work, a more general account of early
cloth manufacture, is an authority on

716. Digging in sediments in northern China, evidence
has been gathered by scientists suggesting that
complex life‐forms emerged much earlier than they had
previously thought.
(A)

evidence has been gathered by scientists
suggesting that complex life‐forms emerged
much earlier than they had

(B)

evidence gathered by scientists suggests a
much earlier emergence of complex life‐forms
than had been

(C)

scientists have gathered evidence suggesting that
complex life‐forms emerged much earlier than

(D)

scientists have gathered evidence that suggests
a much earlier emergence of complex life‐forms
than that which was

(E)

scientists have gathered evidence which
suggests a much earlier emergence of complex
life‐forms than that

717. Employing many different techniques throughout his
career, Michelangelo produced a great variety of art
works, including paintings, for example, in the Sistine
Chapel, to sculpture, for example, the statue of David.
(A)

including paintings, for example, in the Sistine
Chapel, to sculpture, for example,

(B)

including paintings, for example, in the Sistine
Chapel, to sculpture, like

(C)

including paintings, such as those in the Sistine
Chapel, and sculpture, as

(D)

ranging from paintings, such as those in the
Sistine Chapel, to sculpture, such as

(E)

ranging from paintings, such as in the Sistine
Chapel, and sculpture, such as

718. Outlining his strategy for nursing the troubled
conglomerate back to health, the chief executive’s
plans were announced on Wednesday for cutting the
company’s huge debt by selling nearly $12 billion in
assets over the next 18 months.
(A)

executive’s plans were announced on Wednesday
for cutting the company’s huge debt by selling
nearly $12 billion in assets over the next
18 months

(B)

executive’s plans, which are to cut the company’s
huge debt by selling nearly $12 billion in assets
over the next 18 months, were announced on
Wednesday

(C)

executive’s plans for cutting the company’s
huge debt by selling nearly $12 billion in assets
over the next 18 months were announced on
Wednesday

(D)

executive announced plans Wednesday to cut the
company’s huge debt by selling nearly $12 billion
in assets over the next 18 months

(E)

executive announced plans Wednesday that are
to cut the company’s huge debt by selling nearly
$12 billion in assets over the next 18 months

719. It is called a sea, but the landlocked Caspian is actually
the largest lake on Earth, which covers more than four
times the surface area of its closest rival in size, North
America’s Lake Superior.
(A)

It is called a sea, but the landlocked Caspian is
actually the largest lake on Earth, which covers

(B)

Although it is called a sea, actually the
landlocked Caspian is the largest lake on Earth,
which covers

(C)

Though called a sea, the landlocked Caspian is
actually the largest lake on Earth, covering

(D)

Though called a sea but it actually is the largest
lake on Earth, the landlocked Caspian covers

(E)

Despite being called a sea, the largest lake on
Earth is actually the landlocked Caspian, covering

689

GMAT® Official Guide 2018

720. Neuroscientists, having amassed a wealth of
knowledge over the past twenty years about the brain
and its development from birth to adulthood, are now
drawing solid conclusions about how the human brain
grows and how babies acquire language.
(A)

(B)

Neuroscientists, having amassed a wealth of
knowledge over the past twenty years about
the brain and its development from birth to
adulthood, are

(A)

good growing conditions both encouraged
farming traditions that, in places, endure in

(B)

good growing conditions encouraged farming
traditions that have, in places, endured for

Neuroscientists, having amassed a wealth of
knowledge about the brain and its development
from birth to adulthood over the past twenty
years, and are

(C)

of good growing conditions have encouraged
farming traditions that, in places, endured for

(D)

of good growing conditions both encouraged
farming traditions that have, in places, endured

(E)

of good growing conditions encouraged farming
traditions that have, in places, been enduring
for

(C)

Neuroscientists amassing a wealth of knowledge
about the brain and its development from birth to
adulthood over the past twenty years, and are

(D)

Neuroscientists have amassed a wealth of
knowledge over the past twenty years about the
brain and its development from birth to adulthood,

(E)

Neuroscientists have amassed, over the past
twenty years, a wealth of knowledge about the
brain and its development from birth to adulthood,

721. According to a recent study of consumer spending on
prescription medications, increases in the sales of the
50 drugs that were advertised most heavily accounts
for almost half of the $20.8 billion increase in drug
spending last year, the remainder of which came
from sales of the 9,850 prescription medicines that
companies did not advertise or advertised very little.

690

722. Along the major rivers that traverse the deserts of
northeast Africa, the Middle East, and northwest India,
the combination of a reliable supply of water and good
growing conditions both encouraged farming traditions
that, in places, endure in at least 6,000 years.

723. Despite its covering the entire planet, Earth has a
crust that is not seamless or stationary, rather it is
fragmented into mobile semirigid plates.
(A)

Despite its covering the entire planet, Earth has
a crust that is not seamless or stationary, rather
it is

(B)

Despite the fact that it covers the entire planet,
Earth’s crust is neither seamless nor is it
stationary, but is

(C)

Despite covering the entire planet, Earth’s crust
is neither seamless nor is it stationary, but
rather

(A)

heavily accounts for almost half of the $20.8
billion increase in drug spending last year, the
remainder of which came

(D)

Although it covers the entire planet, Earth’s
crust is neither seamless nor stationary, but
rather

(B)

heavily were what accounted for almost half of
the $20.8 billion increase in drug spending last
year; the remainder of the increase coming

(E)

Although covering the entire planet, Earth has a
crust that is not seamless or stationary, but

(C)

heavily accounted for almost half of the $20.8
billion increase in drug spending last year, the
remainder of the increase coming

(D)

heavily, accounting for almost half of the $20.8
billion increase in drug spending last year, while
the remainder of the increase came

(E)

heavily, which accounted for almost half of the
$20.8 billion increase in drug spending last year,
with the remainder of it coming

724. Emily Dickinson’s letters to Susan Huntington Dickinson
were written over a period beginning a few years
before Susan’s marriage to Emily’s brother and ending
shortly before Emily’s death in 1886, outnumbering her
letters to anyone else.
(A)

Dickinson were written over a period beginning
a few years before Susan’s marriage to Emily’s
brother and ending shortly before Emily’s death
in 1886, outnumbering

9.6 Sentence Correction Practice Questions

(B)

Dickinson were written over a period that begins
a few years before Susan’s marriage to Emily’s
brother and ended shortly before Emily’s death in
1886, outnumber

(C)

general market inactivity, if not paralysis, has
lately provided

(D)

lately, general market inactivity, if not paralysis,
has provided

(C)

Dickinson, written over a period beginning a few
years before Susan’s marriage to Emily’s brother
and that ends shortly before Emily’s death in
1886 and outnumbering

(E)

is that lately general market inactivity, if not
paralysis, which provides

(D)

Dickinson, which were written over a period
beginning a few years before Susan’s marriage
to Emily’s brother, ending shortly before Emily’s
death in 1886, and outnumbering

(E)

Dickinson, which were written over a period
beginning a few years before Susan’s marriage
to Emily’s brother and ending shortly before
Emily’s death in 1886, outnumber

725. At the end of 2001, motion picture industry
representatives said that there were about a million
copies of Hollywood movies available online and
expected piracy to increase with high‐speed Internet
connections that become more widely available.

727. Unlike the conviction held by many of her colleagues
that genes were relatively simple and static, Barbara
McClintock adhered to her own more complicated
ideas about how genes might operate, and in 1983,
at the age of 81, was awarded a Nobel Prize for her
discovery that the genes in corn are capable of moving
from one chromosomal site to another.
(A)

Unlike the conviction held by many of her
colleagues that genes were

(B)

Although many of her colleagues were of the
conviction of genes being

(C)

Contrary to many of her colleagues being
convinced that genes were

(D)

Even though many of her colleagues were
convinced that genes were

(E)

Even with many of her colleagues convinced of
genes being

(A)

online and expected piracy to increase with high‐
speed Internet connections that become more
widely available

(B)

online and expect the increase of piracy with
the wider availability of high‐speed Internet
connections

(C)

online, and they expect more piracy to increase
with the wider availability of high‐speed Internet
connections

(A)

(D)

online, and that they expected the increase of
piracy as high‐speed Internet connections would
become more widely available

that of Japan were instrumental in his
development as a writer, informing his literary
style as much as

(B)

(E)

online, and that they expected piracy to increase
as high‐speed Internet connections became
more widely available

that of Japan was instrumental in his
development as a writer, and it informed both his
literary style as well as

(C)

Japan was instrumental in his development as a
writer, informing both his literary style and

(D)

Japan was instrumental in his development as a
writer, as it informed his literary style as much
as

(E)

Japan were instrumental in his development as a
writer, informing both his literary style in addition
to

726. Making things even more difficult has been general
market inactivity lately, if not paralysis, which has
provided little in the way of pricing guidance.
(A)

has been general market inactivity lately, if not
paralysis, which has provided

(B)

there is general market inactivity, if not paralysis,
lately it has provided

728. Ry–u nosuke Akutagawa’s knowledge of the literatures of
Europe, China, and that of Japan were instrumental in
his development as a writer, informing his literary style
as much as the content of his fiction.

691

GMAT® Official Guide 2018

729. According to scientists who monitored its path,
an expanding cloud of energized particles ejected
from the Sun recently triggered a large storm in the
magnetic field that surrounds Earth, which brightened
the Northern Lights and also possibly knocking out a
communications satellite.
(A)

(B)

(C)

an expanding cloud of energized particles
ejected from the Sun recently triggered a large
storm in the magnetic field that surrounds Earth,
which brightened the Northern Lights and also
possibly knocking
an expanding cloud of energized particles
ejected from the Sun was what recently
triggered a large storm in the magnetic field that
surrounds Earth, and it brightened the Northern
Lights and also possibly knocked
an expanding cloud of energized particles
ejected from the Sun recently triggered a large
storm in the magnetic field that surrounds Earth,
brightening the Northern Lights and possibly
knocking

(D)

a large storm in the magnetic field that surrounds
Earth, recently triggered by an expanding cloud
of energized particles, brightened the Northern
Lights and it possibly knocked

(E)

a large storm in the magnetic field surrounding
Earth was recently triggered by an expanding
cloud of energized particles, brightening the
Northern Lights and it possibly knocked

730. In 1850, Lucretia Mott published her Discourse on
Women, arguing in a treatise for women to have equal
political and legal rights and for changes in the married
women’s property laws.

692

(A)

arguing in a treatise for women to have equal
political and legal rights

(B)

arguing in a treatise for equal political and legal
rights for women

(C)

a treatise that advocates women’s equal political
and legal rights

(D)

a treatise advocating women’s equal political and
legal rights

(E)

a treatise that argued for equal political and legal
rights for women

731. To develop more accurate population forecasts,
demographers have to know a great deal more than now
about the social and economic determinants of fertility.
(A)

have to know a great deal more than now about
the social and economic

(B)

have to know a great deal more than they do
now about the social and economical

(C)

would have to know a great deal more than they
do now about the social and economical

(D)

would have to know a great deal more than they
do now about the social and economic

(E)

would have to know a great deal more than now
about the social and economic

732. Laos has a land area about the same as Great Britain
but only four million in population, where many are
members of hill tribes ensconced in the virtually
inaccessible mountain valleys of the north.
(A)

about the same as Great Britain but only four
million in population, where many

(B)

of about the same size as Great Britain is, but
in Laos there is a population of only four million,
and many

(C)

that is about the same size as Great Britain’s
land area, but in Laos with a population of only
four million people, many of them

(D)

comparable to the size of Great Britain, but only
four million in population, and many

(E)

comparable to that of Great Britain but a
population of only four million people, many of
whom

733. Having been named for a mythological nymph who
cared for the infant Jupiter, the asteroid named Ida, in
the middle of the belt of asteroids that orbit the Sun
between Mars and Jupiter, was discovered in 1884.
(A)

Having been named for a mythological nymph
who cared for the infant Jupiter, the asteroid
named Ida, in the middle of the belt of asteroids
that orbit the Sun between Mars and Jupiter, was
discovered in 1884.

(B)

Discovered in 1884, the asteroid Ida, named for
a mythological nymph who cared for the infant
Jupiter, is in the middle of the belt of asteroids
that orbit the Sun between Mars and Jupiter.

9.6 Sentence Correction Practice Questions

(C)

(D)

(E)

In the middle of the belt of asteroids that orbit the
Sun between Mars and Jupiter, the asteroid Ida,
discovered in 1884 and named for a mythological
nymph who cared for the infant Jupiter.

(B)

causing only a mild illness, they become immune
from

(C)

which causes only a mild illness, they are
immune to

The asteroid Ida, named for a mythological
nymph who cared for the infant Jupiter and
discovered in 1884, is in the middle of the belt
of asteroids to orbit the Sun between Mars and
Jupiter.

(D)

causing only a mild illness, they became immune
from

(E)

which caused only a mild illness, they would
become immune to

Ida, an asteroid discovered in 1884 and which
was named for a mythological nymph who cared
for the infant Jupiter, is in the middle of the belt
of asteroids to orbit the Sun between Mars and
Jupiter.

734. In ancient Thailand, much of the local artisans’ creative
energy was expended for the creation of Buddha
images and when they constructed and decorated the
temples that enshrined them.
(A)

much of the local artisans’ creative energy was
expended for the creation of Buddha images
and when they constructed and decorated the
temples that enshrined them

(B)

much of the local artisans’ creative energy was
expended on the creation of Buddha images and
on construction and decoration of the temples in
which they were enshrined

(C)

much of the local artisans’ creative energy was
expended on the creation of Buddha images
as well as constructing and decoration of the
temples in which they were enshrined

(D)

creating images of Buddha accounted for
much of the local artisans’ creative energy, and
also constructing and decorating the temples
enshrining them

(E)

the creation of Buddha images accounted for
much of the local artisans’ creative energy
as well as construction and decoration of the
temples that enshrined them

736. Covering 71 percent of Earth’s surface, the oceans
play an essential role in maintaining the conditions for
human existence on land, moderating temperature
by the absorption of heat and carbon dioxide, and
giving pure water back to the atmosphere through
evaporation.
(A)

Covering 71 percent of Earth’s surface, the
oceans play an essential role in maintaining
the conditions for human existence on land,
moderating

(B)

Covering 71 percent of Earth’s surface and
playing an essential role in maintaining the
conditions for human existence on land, the
oceans moderate

(C)

The oceans cover 71 percent of Earth’s
surface and play an essential role in maintaining
conditions for human existence on land, and by
moderating

(D)

The oceans cover 71 percent of Earth’s
surface, play an essential role in maintaining
the conditions for human existence on land, and
moderate

(E)

The oceans cover 71 percent of Earth’s surface,
playing an essential role in maintaining the
conditions for human existence on land, and they
moderate

735. The English physician Edward Jenner found that if
experimental subjects were deliberately infected with
cowpox, which caused only a mild illness, they are
immune from smallpox.
(A)

which caused only a mild illness, they are
immune from

693

GMAT® Official Guide 2018

737. Some anthropologists believe that the genetic
homogeneity evident in the world’s people is the result
of a “population bottleneck”—at some time in the past
our ancestors suffered an event, greatly reducing their
numbers and thus our genetic variation.
(A)

at some time in the past our ancestors suffered
an event, greatly reducing their numbers

(B)

that at some time in the past our ancestors
suffered an event that greatly reduced their
numbers

(C)

that some time in the past our ancestors
suffered an event so that their numbers were
greatly reduced,

(D)

some time in the past our ancestors suffered
an event from which their numbers were greatly
reduced

(E)

some time in the past, that our ancestors
suffered an event so as to reduce their numbers
greatly,

738. Foraging at all times of the day and night, but
interspersing their feeding with periods of rest that last
between one and eight hours, a sperm whale could eat
so much as a ton of squid a day.
(A)

between one and eight hours, a sperm whale
could eat so

(B)

between one and eight hours, sperm whales can
eat as

(C)

between one to eight hours, sperm whales could
eat as

(D)

from one to eight hours, sperm whales could eat
so

(E)

from one to eight hours, a sperm whale can eat
so

739. In some types of pine tree, a thick layer of needles
protects the buds from which new growth proceeds;
consequently they are able to withstand forest fires
relatively well.

694

(A)

a thick layer of needles protects the buds from
which new growth proceeds; consequently they
are able to withstand forest fires relatively well

(B)

a thick needle layer protects buds from where
new growth proceeds, so that they can withstand
forest fires relatively well

(C)

a thick layer of needles protect the buds from
which new growth proceeds; thus, they are able
to withstand relatively well any forest fires

(D)

since the buds from which new growth
proceeds are protected by a thick needle layer,
consequently they can therefore withstand forest
fires relatively well

(E)

because the buds where new growth happens
are protected by a thick layer of needles, they
are able to withstand forest fires relatively easily
as a result

740. The tourism commission has conducted surveys of
hotels in the most popular resorts, with the ultimate
goal of reducing the guests who end up expressing
overall dissatisfaction with the service in the hotels.
(A)

with the ultimate goal of reducing the guests who
end up expressing overall dissatisfaction with the
service in the hotels

(B)

with the goal to ultimately reduce the number
of guests who end up expressing overall
dissatisfaction with the hotels’ service

(C)

ultimately with the goal to reduce expressions
of overall dissatisfaction by the guests with the
hotel service

(D)

in an ultimate attempt to reduce the number
of guests that ends up expressing overall
dissatisfaction with the hotels’ service

(E)

with the ultimate goal of reducing the number of
guests who express overall dissatisfaction with
the hotels’ service

741. A new study suggests that the conversational pace of
everyday life may be so brisk it hampers the ability of
some children for distinguishing discrete sounds and
words and, the result is, to make sense of speech.
(A)

it hampers the ability of some children for
distinguishing discrete sounds and words and,
the result is, to make

(B)

that it hampers the ability of some children to
distinguish discrete sounds and words and, as a
result, to make

(C)

that it hampers the ability of some children to
distinguish discrete sounds and words and, the
result of this, they are unable to make

9.6 Sentence Correction Practice Questions

(D)

that it hampers the ability of some children to
distinguish discrete sounds and words, and
results in not making

(E)

as to hamper the ability of some children for
distinguishing discrete sounds and words,
resulting in being unable to make

742. The nineteenth‐century chemist Humphry Davy
presented the results of his early experiments in his
“Essay on Heat and Light,” a critique of all chemistry
since Robert Boyle as well as a vision of a new
chemistry that Davy hoped to found.
(A)

a critique of all chemistry since Robert Boyle as
well as a vision of a

(B)

a critique of all chemistry following Robert Boyle
and also his envisioning of a

(C)

a critique of all chemistry after Robert Boyle and
envisioning as well

(D)

critiquing all chemistry from Robert Boyle
forward and also a vision of

(E)

critiquing all the chemistry done since Robert
Boyle as well as his own envisioning of

743. To attract the most talented workers, some companies
are offering a wider range of benefits, letting
employees pick those most important to them.
(A)

benefits, letting employees pick those most
important to them

(B)

benefits, letting employees pick the most
important of them to themselves

(C)

benefits and letting employees pick the most
important to themselves

(D)

benefits and let employees pick the most
important to them

(E)

benefits and let employees pick those that are
most important to themselves

(B)

Empire, fashioned from either the spotted
sandstone of Mathura or from

(C)

Empire, either fashioned from the spotted
sandstone of Mathura or

(D)

Empire and either fashioned from the spotted
sandstone of Mathura or from

(E)

Empire and were fashioned either from the
spotted sandstone of Mathura or from

745. Tides typically range from three to six feet, but while
some places show no tides at all, some others, such
as the Bay of Fundy, have tides of at least thirty feet
and more.
(A)

some others, such as the Bay of Fundy, have
tides of at least thirty feet and more

(B)

the others, such as the Bay of Fundy, that have
tides of more than thirty feet

(C)

others, such as the Bay of Fundy, have tides of
more than thirty feet

(D)

those at the Bay of Fundy, which has tides of
more than thirty feet

(E)

the ones at the Bay of Fundy have tides of at
least thirty feet and more

746. A leading figure in the Scottish Enlightenment, Adam
Smith’s two major books are to democratic capitalism
what Marx’s Das Kapital is to socialism.
(A)

Adam Smith’s two major books are to
democratic capitalism what

(B)

Adam Smith’s two major books are to
democratic capitalism like

(C)

Adam Smith’s two major books are to
democratic capitalism just as

(D)

Adam Smith wrote two major books that are to
democratic capitalism similar to

(E)

Adam Smith wrote two major books that are to
democratic capitalism what

744. Many of the earliest known images of Hindu deities
in India date from the time of the Kushan Empire,
fashioned either from the spotted sandstone of
Mathura or Gandharan grey schist.
(A)

Empire, fashioned either from the spotted
sandstone of Mathura or

695

GMAT® Official Guide 2018

747. Researchers studying the brain scans of volunteers
who pondered ethical dilemmas have found that the
basis for making tough moral judgments is emotion,
not logic or analytical reasoning.
(A)

the brain scans of volunteers who pondered
ethical dilemmas have found that the basis for
making tough moral judgments is

(B)

the brain scans of volunteers who pondered
ethical dilemmas and found the basis to make
tough moral decisions to be

(C)

the brain scans of volunteers pondering ethical
dilemmas and found that the basis for making
tough moral decisions is

(D)

volunteers’ brain scans while pondering ethical
dilemmas have found the basis to make tough
moral judgments to be

(E)

volunteers’ brain scans while they pondered
ethical dilemmas have found that the basis for
making tough moral judgments is

748. Rivaling the pyramids of Egypt or even the ancient
cities of the Maya as an achievement, the army of
terra‐cotta warriors created to protect Qin Shi Huang,
China’s first emperor, in his afterlife is more than 2,000
years old and took 700,000 artisans more than 36
years to complete.
(A)

the army of terra‐cotta warriors created to protect
Qin Shi Huang, China’s first emperor, in his afterlife
is more than 2,000 years old and took 700,000
artisans more than 36 years to complete

(B)

Qin Shi Huang, China’s first emperor, was
protected in his afterlife by an army of terracotta warriors that was created more than
2,000 years ago by 700,000 artisans who took
more than 36 years to complete it

(C)

696

it took 700,000 artisans more than 36 years
to create an army of terra‐cotta warriors more
than 2,000 years ago that would protect
Qin Shi Huang, China’s first emperor, in his afterlife

(D)

more than 2,000 years ago, 700,000 artisans
worked more than 36 years to create an army
of terra‐cotta warriors to protect Qin Shi Huang,
China’s first emperor, in his afterlife

(E)

more than 36 years were needed to complete
the army of terra‐cotta warriors that 700,000
artisans created 2,000 years ago to protect Qin
Shi Huang, China’s first emperor, in his afterlife

749. In California, a lack of genetic variation in the Argentine
ant has allowed the species to spread widely; due to
their being so genetically similar to one another, the ants
consider all their fellows to be a close relative and thus
do not engage in the kind of fierce intercolony struggles
that limits the spread of this species in its native Argentina.
(A)

due to their being so genetically similar to one
another, the ants consider all their fellows to be
a close relative and thus do not engage in the
kind of fierce intercolony struggles that limits

(B)

due to its being so genetically similar, the ant
considers all its fellows to be a close relative
and thus does not engage in the kind of fierce
intercolony struggles that limit

(C)

because it is so genetically similar, the ant
considers all its fellows to be close relatives
and thus does not engage in the kind of fierce
intercolony struggles that limits

(D)

because they are so genetically similar to one
another, the ants consider all their fellows to be
close relatives and thus do not engage in the
kind of fierce intercolony struggles that limit

(E)

because of being so genetically similar to one
another, the ants consider all their fellows to be
a close relative and thus do not engage in the
kind of fierce intercolony struggles that limits

750. Next month, state wildlife officials are scheduled to
take over the job of increasing the wolf population in
the federally designated recovery area, the number
of which will however ultimately be dictated by the
number of prey in the area.
(A)

area, the number of which will however

(B)

area; the size of the population, however, will

(C)

area, however the number of wolves will

(D)

area; the number of which will, however,

(E)

area, when the size of the population will, however,

751. About 5 million acres in the United States have been
invaded by leafy spurge, a herbaceous plant from
Eurasia with milky sap that gives mouth sores to cattle,
displacing grasses and other cattle food and rendering
rangeland worthless.
(A)

States have been invaded by leafy spurge, a
herbaceous plant from Eurasia with milky sap
that gives mouth sores to cattle, displacing
grasses and other cattle food and rendering

9.6 Sentence Correction Practice Questions

(B)

States have been invaded by leafy spurge, a
herbaceous plant from Eurasia, with milky sap,
that gives mouth sores to cattle and displaces
grasses and other cattle food, rendering

(C)

States have been invaded by leafy spurge, a
herbaceous plant from Eurasia having milky sap
that gives mouth sores to cattle and displacing
grasses and other cattle food, rendering

(D)

(E)

(B)

(C)

(A)

include the animal known as the killer whale,
which can grow to be 30 feet long and is

(B)

States, having been invaded by leafy spurge,
a herbaceous plant from Eurasia with milky
sap that gives mouth sores to cattle, displaces
grasses and other cattle food, and renders

include the animal known as the killer whale,
growing as big as 30 feet long and

(C)

include the animal known as the killer whale,
growing up to 30 feet long and being

(D)

States, having been invaded by leafy spurge, a
herbaceous plant from Eurasia that has milky
sap giving mouth sores to cattle and displacing
grasses and other cattle food, rendering

includes the animal known as the killer whale,
which can grow as big as 30 feet long and is

(E)

includes the animal known as the killer whale,
which can grow to be 30 feet long and it is

752. While it costs about the same to run nuclear plants as
other types of power plants, it is the fixed costs that
stem from building nuclear plants that makes it more
expensive for them to generate electricity.
(A)

753. The 32 species that make up the dolphin family are
closely related to whales and in fact include the animal
known as the killer whale, which can grow to be 30
feet long and is famous for its aggressive hunting
pods.

While it costs about the same to run nuclear plants
as other types of power plants, it is the fixed costs
that stem from building nuclear plants that makes it
more expensive for them to generate electricity.

754. The first trenches that were cut into a 500‐acre site
at Tell Hamoukar, Syria, have yielded strong evidence
for centrally administered complex societies in
northern regions of the Middle East that were arising
simultaneously with but independently of the more
celebrated city‐states of southern Mesopotamia, in
what is now southern Iraq.
(A)

While the cost of running nuclear plants is about the
same as for other types of power plants, the fixed
costs that stem from building nuclear plants make
the electricity they generate more expensive.

that were cut into a 500‐acre site at Tell
Hamoukar, Syria, have yielded strong evidence
for centrally administered complex societies in
northern regions of the Middle East that were
arising simultaneously with but

(B)

Even though it costs about the same to run
nuclear plants as for other types of power
plants, it is the fixed costs that stem from
building nuclear plants that makes the electricity
they generate more expensive.

that were cut into a 500‐acre site at Tell
Hamoukar, Syria, yields strong evidence that
centrally administered complex societies in
northern regions of the Middle East were arising
simultaneously with but also

(C)

having been cut into a 500‐acre site at Tell
Hamoukar, Syria, have yielded strong evidence
that centrally administered complex societies in
northern regions of the Middle East were arising
simultaneously but

(D)

cut into a 500‐acre site at Tell Hamoukar, Syria,
yields strong evidence of centrally administered
complex societies in northern regions of the
Middle East arising simultaneously but also

(E)

cut into a 500‐acre site at Tell Hamoukar, Syria,
have yielded strong evidence that centrally
administered complex societies in northern
regions of the Middle East arose simultaneously
with but

(D)

It costs about the same to run nuclear plants as for
other types of power plants, whereas the electricity
they generate is more expensive, stemming from
the fixed costs of building nuclear plants.

(E)

The cost of running nuclear plants is about the
same as other types of power plants, but the
electricity they generate is made more expensive
because of the fixed costs stemming from
building nuclear plants.

697

GMAT® Official Guide 2018

755. Companies are relying more and more on networked
computers for such critical tasks as inventory
management, electronic funds transfer, and electronic
data interchange, in which standard business
transactions are handled via computer rather than on
paper.

interior and most rapidly travel through cold,
dense regions, and slower

(E)

interior and that travel most rapidly through cold,
dense regions and slower

758. Prices at the producer level are only 1.3 percent
higher now than a year ago and are going down,
even though floods in the Midwest and drought in the
South are hurting crops and therefore raised corn and
soybean prices.

(A)

in which standard business transactions are
handled via computer rather than on paper

(B)

where computers handle standard business
transactions rather than on paper

(C)

in which computers handle standard business
transactions instead of on paper

(A)

(D)

where standard business transactions are
handled, not with paper, but instead via
computer

than a year ago and are going down, even
though floods in the Midwest and drought in the
South are hurting crops and therefore raised

(B)

than those of a year ago and are going down,
even though floods in the Midwest and drought in
the South are hurting crops and therefore raising

(C)

than a year ago and are going down, despite
floods in the Midwest and drought in the South,
and are hurting crops and therefore raising

(D)

as those of a year ago and are going down, even
though floods in the Midwest and drought in the
South hurt crops and therefore raise

(E)

as they were a year ago and are going down,
despite floods in the Midwest and drought in
the South, and are hurting crops and therefore
raising

(E)

in which standard business transactions are being
handled via computer, in place of on paper

756. Combining enormous physical strength with higher
intelligence, the Neanderthals appear as equipped for
facing any obstacle the environment could put in their
path, but their relatively sudden disappearance during
the Paleolithic era indicates that an inability to adapt to
some environmental change led to their extinction.
(A)

appear as equipped for facing any obstacle the
environment could put in their path,

(B)

appear to have been equipped to face any
obstacle the environment could put in their path,

(C)

appear as equipped to face any obstacle the
environment could put in their paths,

(D)

appeared as equipped to face any obstacle the
environment could put in their paths,

(E)

appeared to have been equipped for facing any
obstacle the environment could put in their path,

757. To map Earth’s interior, geologists use a network of
seismometers to chart seismic waves that originate
in the earth’s crust and ricochet around its interior,
most rapidly traveling through cold, dense regions and
slower through hotter rocks.

698

(D)

(A)

interior, most rapidly traveling through cold,
dense regions and slower

(B)

interior, which travel most rapidly through cold,
dense regions, and more slowly

(C)

interior, traveling most rapidly through cold,
dense regions and more slowly

759. Fossils of the arm of a sloth found in Puerto Rico in
1991, and dated at 34 million years old, made it the
earliest known mammal of the Greater Antilles Islands.
(A)

sloth found in Puerto Rico in 1991, and dated at
34 million years old, made it the earliest known
mammal of

(B)

sloth, that they found in Puerto Rico in 1991, has
been dated at 34 million years old, thus making
it the earliest mammal known on

(C)

sloth that was found in Puerto Rico in 1991, was
dated at 34 million years old, making this the
earliest known mammal of

(D)

sloth, found in Puerto Rico in 1991, have been
dated at 34 million years old, making the sloth
the earliest known mammal on

(E)

sloth which, found in Puerto Rico in 1991, was
dated at 34 million years old, made the sloth the
earliest known mammal of

9.6 Sentence Correction Practice Questions

760. Recently physicians have determined that stomach
ulcers are not caused by stress, alcohol, or rich foods,
but a bacterium that dwells in the mucous lining of the
stomach.
(A)

not caused by stress, alcohol, or rich foods, but

(B)

not caused by stress, alcohol, or rich foods, but
are by

(C)

caused not by stress, alcohol, or rich foods, but
by

(D)

caused not by stress, alcohol, and rich foods, but

(E)

caused not by stress, alcohol, and rich foods,
but are by

761. The eyes of the elephant seal adapt to darkness more
quickly than any other animal yet tested, thus allowing
it to hunt efficiently under the gloomy conditions at its
feeding depth of between 300 and 700 meters.
(A)

The eyes of the elephant seal adapt to darkness
more quickly than any other animal yet tested,
thus allowing it

(B)

The eyes of the elephant seal adapt to darkness
more quickly than does any other animal yet
tested, allowing them

(C)

The eyes of the elephant seal adapt to darkness
more quickly than do those of any other animal
yet tested, allowing it

(D)

Because they adapt to darkness more quickly
than any other animal yet tested, the eyes of the
elephant seal allow it

(E)

Because the eyes of the elephant seal adapt
to darkness more quickly than do those of any
other animal yet tested, it allows them

762. A mutual fund having billions of dollars in assets will
typically invest that money in hundreds of companies,
rarely holding more than one percent of the shares of
any particular corporation.
(A)

companies, rarely holding more than one percent

(B)

companies, and it is rare to hold at least one
percent or more

(C)

companies and rarely do they hold more than
one percent

(D)

companies, so that they rarely hold more than
one percent

(E)

companies; rarely do they hold one percent or
more

763. Positing an enormous volcanic explosion at the end
of the Permian period would explain the presence of a
buried crater, account for the presence of the element
iridium (originating deep within the earth), and the
presence of quartz having been shattered by highimpact shock waves.
(A)

account for the presence of the element iridium
(originating deep within the earth), and the
presence of quartz having been

(B)

of the element iridium (originating deep within the
earth), and of quartz

(C)

the element iridium (originating deep within the
earth), and explain the presence of quartz having
been

(D)

the presence of the element iridium (originating
deep within the earth), and explain the presence
of quartz

(E)

explain the element iridium (originating deep
within the earth), and the presence of quartz

764. The 19‐year‐old pianist and composer performed his
most recent work all over Europe, Asia, and North
America last year, winning prestigious awards in both
London as well as Tokyo for his achievement at so
young an age, and he is hoping to continue composing
now that he has returned to Chicago.
(A)

winning prestigious awards in both London as
well as Tokyo for his achievement at so young an
age, and he is hoping

(B)

winning prestigious awards both in London and
Tokyo for his achievement at such a young age,
and hoping

(C)

having won prestigious awards both in London
and Tokyo for his achievement at so young an
age, hoping

(D)

winning prestigious awards in both London and
Tokyo for his achievement at such a young age,
and he hopes

(E)

having won prestigious awards both in London as
well as Tokyo for his achievement at so young an
age, and he hopes

699

GMAT® Official Guide 2018

765. Starfish, with anywhere from five to eight arms,
have a strong regenerative ability, and if one arm is
lost it quickly replaces it, sometimes by the animal
overcompensating and growing an extra one or two.
(A)

one arm is lost it quickly replaces it, sometimes
by the animal overcompensating and

(B)

one arm is lost it is quickly replaced, with the
animal sometimes overcompensating and

(C)

they lose one arm they quickly replace it,
sometimes by the animal overcompensating,

(D)

they lose one arm they are quickly replaced, with
the animal sometimes overcompensating,

(E)

they lose one arm it is quickly replaced,
sometimes with the animal overcompensating,

766. In 2000, a mere two dozen products accounted for
half the increase in spending on prescription drugs,
a phenomenon that is explained not just because of
more expensive drugs but by the fact that doctors are
writing many more prescriptions for higher‐cost drugs.
(A)

a phenomenon that is explained not just because
of more expensive drugs but by the fact that
doctors are writing

(B)

a phenomenon that is explained not just by the
fact that drugs are becoming more expensive
but also by the fact that doctors are writing

(C)

a phenomenon occurring not just because of
drugs that are becoming more expensive but
because of doctors having also written

(D)

which occurred not just because drugs are
becoming more expensive but doctors are also
writing

(E)

which occurred not just because of more expensive
drugs but because doctors have also written

767. Similar to other early Mississippi Delta blues singers,
the music of Robert Johnson arose from an oral
tradition beginning with a mixture of chants, fiddle
tunes, and religious music and only gradually evolved
into the blues.
(A)

(B)

700

Similar to other early Mississippi Delta blues
singers, the music of Robert Johnson arose from
an oral tradition beginning with
Similar to that of other early Mississippi Delta
blues singers, Robert Johnson made music that
arose from an oral tradition that began with

(C)

As with other early Mississippi Delta blues
singers, Robert Johnson made music that arose
from an oral tradition beginning as

(D)

Like other early Mississippi Delta blues singers,
Robert Johnson’s music arose from an oral
tradition beginning with

(E)

Like the music of other early Mississippi Delta
blues singers, the music of Robert Johnson
arose from an oral tradition that began as

768. Thelonious Monk, who was a jazz pianist and
composer, produced a body of work both rooted in
the stride‐piano tradition of Willie (The Lion) Smith and
Duke Ellington, yet in many ways he stood apart from
the mainstream jazz repertory.
(A)

Thelonious Monk, who was a jazz pianist and
composer, produced a body of work both rooted

(B)

Thelonious Monk, the jazz pianist and composer,
produced a body of work that was rooted both

(C)

Jazz pianist and composer Thelonious Monk,
who produced a body of work rooted

(D)

Jazz pianist and composer Thelonious Monk
produced a body of work that was rooted

(E)

Jazz pianist and composer Thelonious Monk
produced a body of work rooted both

769. Nobody knows exactly how many languages there
are in the world, partly because of the difficulty
of distinguishing between a language and the
sublanguages or dialects within it, but those who
have tried to count typically have found about five
thousand.
(A)

and the sublanguages or dialects within it, but
those who have tried to count typically have
found

(B)

and the sublanguages or dialects within them,
with those who have tried counting typically
finding

(C)

and the sublanguages or dialects within it, but
those who have tried counting it typically find

(D)

or the sublanguages or dialects within them, but
those who tried to count them typically found

(E)

or the sublanguages or dialects within them, with
those who have tried to count typically finding

9.6 Sentence Correction Practice Questions

770. Although a number of excellent studies narrate
the development of domestic technology and
its impact on housewifery, these works do not
discuss the contributions of the women employed
by manufacturers and utility companies as product
demonstrators and publicists, who initially promoted
new and unfamiliar technology to female consumers.
(A)

by manufacturers and utility companies as
product demonstrators and publicists,

(B)

to be product demonstrators and publicists by
manufacturers and utility companies,

(C)

to demonstrate and publicize their products by
manufacturers and utility companies

(D)

by manufacturers and utility companies to be
demonstrators and publicists of their products

(E)

by manufacturers and utility companies to
demonstrate and publicize their products

771. The absence from business and financial records of
the nineteenth century of statistics about women leave
us with no record of the jobs that were performed by
women and how they survived economically.
(A)

from business and financial records of the
nineteenth century of statistics about women
leave us with no record of the jobs that were
performed by women and

(B)

from business and financial records of statistics
about women from the nineteenth century leave us
with no record of what jobs women performed or

(C)

of statistics for women from business and
financial records in the nineteenth century leaves
us with no record of either the jobs that women
were performing and of

(D)

of statistics on women from business and financial
records in the nineteenth century leave us with no
record of the jobs that women performed or of

(E)

of statistics about women from business and
financial records of the nineteenth century leaves
us with no record of either what jobs women
performed or

772. Heating‐oil prices are expected to be higher this year
than last because refiners are paying about $5 a barrel
more for crude oil than they were last year.
(A)

(B)

Heating‐oil prices are expected to rise higher this
year over last because refiners pay about $5 a
barrel for crude oil more than they did

(C)

Expectations are for heating‐oil prices to be
higher this year than last year’s because refiners
are paying about $5 a barrel for crude oil more
than they did

(D)

It is the expectation that heating‐oil prices will be
higher for this year over last because refiners
are paying about $5 a barrel more for crude oil
now than what they were

(E)

It is expected that heating‐oil prices will rise higher
this year than last year’s because refiners pay
about $5 a barrel for crude oil more than they did

773. Even though Clovis points, spear points with
longitudinal grooves chipped onto their faces, have
been found all over North America, they are named for
the New Mexico site where they were first discovered
in 1932.
(A)

Even though Clovis points, spear points with
longitudinal grooves chipped onto their faces,
have been found all over North America, they are
named for the New Mexico site where they were
first discovered in 1932.

(B)

Although named for the New Mexico site where
first discovered in 1932, Clovis points are spear
points of longitudinal grooves chipped onto
their faces and have been found all over North
America.

(C)

Named for the New Mexico site where they have
been first discovered in 1932, Clovis points,
spear points of longitudinal grooves chipped
onto the faces, have been found all over North
America.

(D)

Spear points with longitudinal grooves that are
chipped onto the faces, Clovis points, even
though named for the New Mexico site where
first discovered in 1932, but were found all over
North America.

(E)

While Clovis points are spear points whose faces
have longitudinal grooves chipped into them,
they have been found all over North America,
and named for the New Mexico site where they
have been first discovered in 1932.

Heating‐oil prices are expected to be higher this
year than last because refiners are paying about
$5 a barrel more for crude oil than they were
701

GMAT® Official Guide 2018

774. Heavy commitment by an executive to a course of
action, especially if it has worked well in the past,
makes it likely to miss signs of incipient trouble or
misinterpret them when they do appear.
(A)

(B)

Heavy commitment by an executive to a course
of action, especially if it has worked well in the
past, makes it likely to miss signs of incipient
trouble or misinterpret them when they do
appear.
An executive who is heavily committed to a
course of action, especially one that worked well
in the past, makes missing signs of incipient
trouble or misinterpreting ones likely when they
do appear.

(C)

An executive who is heavily committed to a
course of action is likely to miss or misinterpret
signs of incipient trouble when they do appear,
especially if it has worked well in the past.

(D)

Executives’ being heavily committed to a course
of action, especially if it has worked well in the
past, makes them likely to miss signs of incipient
trouble or misinterpreting them when they do
appear.

(E)

Being heavily committed to a course of action,
especially one that has worked well in the past,
is likely to make an executive miss signs of
incipient trouble or misinterpret them when they
do appear.

775. According to recent studies comparing the nutritional
value of meat from wild animals and meat from
domesticated animals, wild animals have less total fat
than do livestock fed on grain and more of a kind of fat
they think is good for cardiac health.
(A)

wild animals have less total fat than do livestock
fed on grain and more of a kind of fat they think
is

(B)

wild animals have less total fat than livestock fed
on grain and more of a kind of fat thought to be

(C)

(D)

(E)

702

776. Yellow jackets number among the 900 or so species
of the world’s social wasps, wasps living in a highly
cooperative and organized society where they consist
almost entirely of females—the queen and her sterile
female workers.
(A)

wasps living in a highly cooperative and
organized society where they consist almost
entirely of

(B)

wasps that live in a highly cooperative and
organized society consisting almost entirely of

(C)

which means they live in a highly cooperative and
organized society, almost all

(D)

which means that their society is highly
cooperative, organized, and it is almost entirely

(E)

living in a society that is highly cooperative,
organized, and it consists of almost all

777. Before 1988, insurance companies in California were
free to charge whatever rates the market would bear,
needing no approval from regulators before raising
rates.
(A)

needing no approval from regulators before
raising

(B)

and it needed no approval by regulators before
raising

(C)

and needing no approval from regulators before
they raised

(D)

with approval not needed by regulators before
they raised

(E)

with no approval needed from regulators before
the raising of

778. Marconi’s conception of the radio was as a substitute
for the telephone, a tool for private conversation;
instead, it is precisely the opposite, a tool for
communicating with a large, public audience.
(A)

wild animals have less total fat than that of
livestock fed on grain and have more fat of a
kind thought to be

Marconi’s conception of the radio was as a
substitute for the telephone, a tool for private
conversation; instead, it is

(B)

total fat of wild animals is less than livestock
fed on grain and they have more fat of a kind
thought to be

Marconi conceived of the radio as a substitute
for the telephone, a tool for private conversation,
but which is

(C)

Marconi conceived of the radio as a tool for
private conversation that could substitute for the
telephone; instead, it has become

total fat is less in wild animals than that of
livestock fed on grain and more of their fat is of
a kind they think is

9.6 Sentence Correction Practice Questions

(D)

Marconi conceived of the radio to be a tool
for private conversation, a substitute for the
telephone, which has become

(E)

Marconi conceived of the radio to be a substitute
for the telephone, a tool for private conversation,
other than what it is,

779. Because there are provisions of the new maritime code
that provide that even tiny islets can be the basis for
claims to the fisheries and oil fields of large sea areas,
they have already stimulated international disputes
over uninhabited islands.
(A)

Because there are provisions of the new
maritime code that provide that even tiny islets
can be the basis for claims to the fisheries and
oil fields of large sea areas, they have already
stimulated

(B)

Because the new maritime code provides that
even tiny islets can be the basis for claims to the
fisheries and oil fields of large sea areas, it has
already stimulated

(C)

Even tiny islets can be the basis for claims to the
fisheries and oil fields of large sea areas under
provisions of the new maritime code, already
stimulating

(D)

Because even tiny islets can be the basis for
claims to the fisheries and oil fields of large
sea areas under provisions of the new maritime
code, this has already stimulated

(E)

Because even tiny islets can be the basis for
claims to the fisheries and oil fields of large
sea areas under provisions of the new maritime
code, which is already stimulating

780. Unlike the automobile company, whose research was
based on crashes involving sport utility vehicles, the
research conducted by the insurance company took
into account such factors as a driver’s age, sex, and
previous driving record.
(A)

company, whose research was based on

(B)

company, which researched

(C)

company, in its research of

(D)

company’s research, having been based on

(E)

company’s research on

781. Gusty westerly winds will continue to usher in a
seasonably cool air mass into the region, as a broad
area of high pressure will build and bring fair and dry
weather for several days.
(A)

to usher in a seasonably cool air mass into the
region, as a broad area of high pressure will
build and

(B)

ushering in a seasonably cool air mass into the
region and a broad area of high pressure will
build that

(C)

to usher in a seasonably cool air mass to the
region, a broad area of high pressure building,
and

(D)

ushering a seasonably cool air mass in the
region, with a broad area of high pressure
building and

(E)

to usher a seasonably cool air mass into the
region while a broad area of high pressure
builds, which will

782. With the patience of its customers and with its network
strained to the breaking point, the on-line service
company announced a series of new initiatives trying
to relieve the congestion that has led to at least four
class-action lawsuits and thousands of complaints from
frustrated customers.
(A)

the patience of its customers and with its
network strained to the breaking point, the online service company announced a series of new
initiatives trying to relieve

(B)

the patience of its customers and its network
strained to the breaking point, the on-line service
company announced a series of new initiatives
that try to relieve

(C)

its network and the patience of its customers
strained to the breaking point, the on-line service
company announced a series of new initiatives to
try to relieve

(D)

its network and with the patience of its
customers strained to the breaking point, the
on-line service company announced a series of
initiatives to try relieving

(E)

its network and its customers’ patience strained
to the breaking point, the on-line service
company announced a series of new initiatives to
try relieving

703

GMAT® Official Guide 2018

783. November is traditionally the strongest month for sales
of light trucks, but sales this past November, even when
compared with sales in previous Novembers, accounted
for a remarkably large share of total vehicle sales.
(A)

but sales this past November, even when
compared with sales in previous Novembers,

(B)

but even when it is compared with previous
Novembers, this past November’s sales

(C)

but even when they are compared with previous
Novembers, sales of light trucks this past
November

(D)

so that compared with previous Novembers,
sales of light trucks this past November

(E)

so that this past November’s sales, even
compared with previous Novembers’ sales,

784. Most of the country’s biggest daily newspapers had
lower circulation in the six months from October 1995
through March 1996 than a similar period a year earlier.
(A)

a similar period

(B)

a similar period’s

(C)

in a similar period

(D)

that in a similar period

(E)

that of a similar period

785. Mauritius was a British colony for almost 200 years,
excepting for the domains of administration and
teaching, the English language was never really spoken
on the island.
(A)

excepting for

(B)

except in

(C)

but except in

(D)

but excepting for

(E)

with the exception of

786. Although appearing less appetizing than most of
their round and red supermarket cousins, heirloom
tomatoes, grown from seeds saved during the previous
year—they are often green and striped, or have plenty
of bumps and bruises—heirlooms are more flavorful
and thus in increasing demand.
(A)

704

Although appearing less appetizing than most
of their round and red supermarket cousins,
heirloom tomatoes, grown from seeds saved
during the previous year

(B)

Although heirloom tomatoes, grown from seeds
saved during the previous year, appear less
appetizing than most of their round and red
supermarket cousins

(C)

Although they appear less appetizing than most
of their round and red supermarket cousins,
heirloom tomatoes, grown from seeds saved
during the previous year

(D)

Grown from seeds saved during the previous
year, heirloom tomatoes appear less appetizing
than most of their round and red supermarket
cousins

(E)

Heirloom tomatoes, grown from seeds saved
during the previous year, although they appear
less appetizing than most of their round and red
supermarket cousins

787. The World Wildlife Fund has declared that global
warming, a phenomenon most scientists agree to be
caused by human beings in burning fossil fuels, will
create havoc among migratory birds by altering the
environment in ways harmful to their habitats.
(A)

a phenomenon most scientists agree to be
caused by human beings in burning fossil fuels,

(B)

a phenomenon most scientists agree that is
caused by fossil fuels burned by human beings,

(C)

a phenomenon that most scientists agree is
caused by human beings’ burning of fossil fuels,

(D)

which most scientists agree on as a
phenomenon caused by human beings who burn
fossil fuels,

(E)

which most scientists agree to be a phenomenon
caused by fossil fuels burned by human beings,

788. The largest of all the planets, not only is Jupiter three
times so massive as Saturn, the next larger planet, but
also possesses four of the largest satellites, or moons,
in our solar system.
(A)

not only is Jupiter three times so massive as
Saturn, the next larger

(B)

not only is Jupiter three times as massive as
Saturn, the next largest

(C)

Jupiter, not only three times as massive as
Saturn, the next largest

9.6 Sentence Correction Practice Questions

(D)

Jupiter not only is three times as massive as
Saturn, the next largest

(E)

Jupiter is not only three times so massive as
Saturn, the next larger

789. While many of the dinosaur fossils found recently
in northeast China seem to provide evidence of the
kinship between dinosaurs and birds, the wealth of
enigmatic fossils seem more likely at this stage that
they will inflame debates over the origin of birds rather
than settle them.
(A)
(B)

She was less successful after she had emigrated
to New York compared to

(B)

Being less successful after she had emigrated to
New York as compared to

(C)

Less successful after she emigrated to New York
than she had been in

(D)

seem more likely that it will inflame debates over
the origin of birds at this stage than

Although she was less successful after
emigrating to New York when compared to

(E)

She had been less successful after emigrating to
New York than in

seems more likely to inflame debates on the
origin of birds at this stage rather than

(D)

seems more likely at this stage to inflame
debates over the origin of birds than to

(E)

seems more likely that it will inflame debates on
the origin of birds at this stage than to

790. Found only in the Western Hemisphere and surviving
through extremes of climate, hummingbirds’ range
extends from Alaska to Tierra del Fuego, from sea‐level
rain forests to the edges of Andean snowfields and ice
fields at altitudes of 15,000 feet.
Found only in the Western Hemisphere
and surviving through extremes of climate,
hummingbirds’ range extends

(B)

Found only in the Western Hemisphere,
hummingbirds survive through extremes of
climate, their range extending

(C)

Hummingbirds, found only in the Western
Hemisphere and surviving through extremes of
climate, with their range extending

(D)

Hummingbirds, found only in the Western
Hemisphere and surviving through extremes of
climate, their range extends

(E)

(A)

seem more likely at this stage that they will inflame
debates over the origin of birds rather than

(C)

(A)

791. She was less successful after she had emigrated
to New York compared to her native Germany,
photographer Lotte Jacobi nevertheless earned a small
group of discerning admirers, and her photographs
were eventually exhibited in prestigious galleries
across the United States.

Hummingbirds are found only in the Western
Hemisphere, survive through extremes of
climate, and their range extends

792. Scientists have recently found evidence that black
holes—regions of space in which matter is so
concentrated and the pull of gravity so powerful that
nothing, not even light, can emerge from them—probably
exist at the core of nearly all galaxies and the mass of
each black hole is proportional to its host galaxy.
(A)

exist at the core of nearly all galaxies and the
mass of each black hole is proportional to

(B)

exist at the core of nearly all galaxies and that the
mass of each black hole is proportional to that of

(C)

exist at the core of nearly all galaxies, and that
the mass of each black hole is proportional to

(D)

exists at the core of nearly all galaxies, and that the
mass of each black hole is proportional to that of

(E)

exists at the core of nearly all galaxies and the
mass of each black hole is proportional to that of

793. The use of lie detectors is based on the assumption that
lying produces emotional reactions in an individual that,
in turn, create unconscious physiological responses.
(A)

that, in turn, create unconscious physiological
responses

(B)

that creates unconscious physiological
responses in turn

(C)

creating, in turn, unconscious physiological
responses

(D)

to create, in turn, physiological responses that
are unconscious

(E)

who creates unconscious physiological
responses in turn
705

GMAT® Official Guide 2018

794. Australian embryologists have found evidence that
suggests that the elephant is descended from an
aquatic animal, and its trunk originally evolving as a
kind of snorkel.

(A)

(A)

that suggests that the elephant is descended from
an aquatic animal, and its trunk originally evolving

the cheetah species’ homogeneity makes them
especially vulnerable to disease

(B)

(B)

that has suggested the elephant descended from
an aquatic animal, its trunk originally evolving

the cheetah species is especially vulnerable to
disease because of its homogeneity

(C)

(C)

suggesting that the elephant had descended from
an aquatic animal with its trunk originally evolved

the homogeneity of the cheetah species makes it
especially vulnerable to disease

(D)

(D)

to suggest that the elephant had descended
from an aquatic animal and its trunk originally
evolved

homogeneity makes members of the cheetah
species especially vulnerable to disease

(E)

members of the cheetah species are especially
vulnerable to disease because of their
homogeneity

(E)

to suggest that the elephant is descended from an
aquatic animal and that its trunk originally evolved

795. Most efforts to combat such mosquito‐borne diseases
like malaria and dengue have focused either on the
vaccination of humans or on exterminating mosquitoes
with pesticides.

798. As sources of electrical power, windmills now account
for only about 2,500 megawatts nationwide, but
production is almost expected to double by the end
of the year, which would provide enough electricity for
1.3 million households.

(A)

like malaria and dengue have focused either on
the vaccination of humans or on exterminating

(A)

almost expected to double by the end of the
year, which would provide

(B)

like malaria and dengue have focused either on
vaccinating of humans or on the extermination of

(B)

almost expected that it will double by the end of
the year, thus providing

(C)

as malaria and dengue have focused on either
vaccinating humans or on exterminating

(C)

expected that it will almost double by the end of
the year to provide

(D)

as malaria and dengue have focused on either
vaccinating of humans or on extermination of

(D)

expected almost to double by the end of the
year and thus to provide

(E)

as malaria and dengue have focused on either
vaccinating humans or exterminating

(E)

expected almost to double by the end of the
year, which would thus be providing

796. Among the Tsonga, a Bantu-speaking group of tribes
in southeastern Africa, dance teams represent
their own chief at the court of each other, providing
entertainment in return for food, drink, and lodging.

706

797. Almost like clones in their similarity to one another, the
cheetah species’ homogeneity makes them especially
vulnerable to disease.

(A)

the court of each other, providing entertainment
in return for

(B)

the court of another and provide entertainment in
return for

(C)

the court of the other, so as to provide
entertainment as a return on

(D)

each other’s court, entertainment being provided
in return for

(E)

another’s court and provide entertainment as a
return on

799. While most of the earliest known ball courts in
Mesoamerica date to 900–400 B.C., waterlogged
latex balls found at El Manati and representations of
ballplayers painted on ceramics found at San Lorenzo
attest to the fact that the Mesoamerican ballgame was
well established by the mid-thirteenth century B.C.
(A)

waterlogged latex balls found at El Manati
and representations of ballplayers painted on
ceramics found at San Lorenzo attest

(B)

waterlogged latex balls found at El Manati and
the painting of representations of ballplayers on
ceramics found at San Lorenzo attests

(C)

waterlogged latex balls found at El Manati
and ceramics painted with representations of
ballplayers found at San Lorenzo attests

9.6 Sentence Correction Practice Questions

(D)

the finding of waterlogged latex balls at El Manati
and the painting of representations of ballplayers
on ceramics found at San Lorenzo attests

(E)

the finding of waterlogged latex balls at El Manati
and of representations of ballplayers painted on
ceramics at San Lorenzo attest

800. As criminal activity on the Internet becomes more and
more sophisticated, not only are thieves able to divert
cash from company bank accounts, they can also pilfer
valuable information such as business development
strategies, new product specifications, and contract
bidding plans, and sell the data to competitors.
(A)

(B)

they can also pilfer valuable information such as
business development strategies, new product
specifications, and contract bidding plans, and
sell
they can also pilfer valuable information that
includes business development strategies, new
product specifications, and contract bidding
plans, and selling

(C)

also pilfering valuable information including
business development strategies, new product
specifications, and contract bidding plans, selling

(D)

but also pilfer valuable information such as
business development strategies, new product
specifications, and contract bidding plans to sell

(E)

but also pilfering valuable information such as
business development strategies, new product
specifications, and contract bidding plans and
selling

801. Last week local shrimpers held a news conference to
take some credit for the resurgence of the rare Kemp’s
ridley turtle, saying that their compliance with laws
requiring that turtle‐excluder devices be on shrimp nets
protect adult sea turtles.
(A)

requiring that turtle‐excluder devices be on
shrimp nets protect

(B)

requiring turtle‐excluder devices on shrimp nets
is protecting

(C)

that require turtle‐excluder devices on shrimp
nets protect

(D)

to require turtle‐excluder devices on shrimp nets
are protecting

(E)

to require turtle‐excluder devices on shrimp nets
is protecting

802. A ruined structure found at Aqaba, Jordan, was
probably a church, as indicated in its eastward
orientation and by its overall plan, as well as artifacts,
such as glass oil‐lamp fragments, found at the site.
(A)

A ruined structure found at Aqaba, Jordan, was
probably a church, as indicated in its eastward
orientation and by its overall plan, as well as

(B)

A ruined structure found at Aqaba, Jordan, once
probably being a church, was indicated by its
eastward orientation, overall plan, and

(C)

Indicating that a ruined structure found at Aqaba,
Jordan, was probably a church were its eastward
orientation and overall plan, but also the

(D)

A ruined structure found at Aqaba, Jordan, was
probably a church, as indicates its eastward
orientation and overall plan, as well as the

(E)

That a ruined structure found at Aqaba, Jordan,
was probably a church is indicated by its
eastward orientation and overall plan, as well as
by the

803. In the major cities of industrialized countries at the
end of the nineteenth century, important public places
such as theaters, restaurants, shops, and banks had
installed electric lighting, but electricity was in less
than 1 percent of homes, where lighting was still
provided mainly by candles or gas.
(A)

electricity was in less than 1 percent of homes,
where lighting was still

(B)

electricity was in less than 1 percent of homes
and lighting still

(C)

there had been less than 1 percent of homes
with electricity, where lighting was still being

(D)

there was less than 1 percent of homes that had
electricity, having lighting that was still

(E)

less than 1 percent of homes had electricity,
where lighting had still been

707

GMAT® Official Guide 2018

804. By 1999, astronomers had discovered 17 nearby stars
that are orbited by planets about the size of Jupiter.
(A)

had discovered 17 nearby stars that are orbited
by planets

(A)

did its return in 1910–1911

(B)

had discovered 17 nearby stars with planets
orbiting them that were

(B)

had its 1910–1911 return

(C)

in its return of 1910–1911

(C)

had discovered that there were 17 nearby stars
that were orbited by planets

(D)

its return of 1910–1911 did

(E)

its return in 1910–1911

(D)

have discovered 17 nearby stars with planets
orbiting them that are

(E)

have discovered that 17 nearby stars are orbited
by planets

805. Although she was considered among her
contemporaries to be the better poet than her
husband, later Elizabeth Barrett Browning was
overshadowed by his success.
(A)

708

806. In no other historical sighting did Halley’s Comet
cause such a worldwide sensation as did its return in
1910–1911.

Although she was considered among her
contemporaries to be the better poet than her
husband, later Elizabeth Barrett Browning was
overshadowed by his success.

(B)

Although Elizabeth Barrett Browning was
considered among her contemporaries as a
better poet than her husband, she was later
overshadowed by his success.

(C)

Later overshadowed by the success of her
husband, Elizabeth Barrett Browning’s poetry had
been considered among her contemporaries to
be better than that of her husband.

(D)

Although Elizabeth Barrett Browning’s success
was later overshadowed by that of her husband,
among her contemporaries she was considered
the better poet.

(E)

Elizabeth Barrett Browning’s poetry was
considered among her contemporaries as better
than her husband, but her success was later
overshadowed by his.

807. Rock samples taken from the remains of an asteroid
about twice the size of the 6‐mile‐wide asteroid that
eradicated the dinosaurs has been dated to be 3.47
billion years old and thus is evidence of the earliest
known asteroid impact on Earth.
(A)

has been dated to be 3.47 billion years old and
thus is

(B)

has been dated at 3.47 billion years old and thus

(C)

have been dated to be 3.47 billion years old and
thus are

(D)

have been dated as being 3.47 billion years old
and thus

(E)

have been dated at 3.47 billion years old and
thus are

9.7 Sentence Correction Answer Key

9.7 Answer Key
668. E

703. B

738. B

773. A

669. D

704. E

739. A

774. E

670. B

705. C

740. E

775. B

671. C

706. B

741. B

776. B

672. B

707. D

742. A

777. A

673. C

708. E

743. A

778. C

674. D

709. E

744. E

779. B

675. D

710. D

745. C

780. E

676. D

711. A

746. E

781. E

677. E

712. B

747. A

782. C

678. B

713. E

748. A

783. A

679. B

714. D

749. D

784. C

680. D

715. E

750. B

785. C

681. C

716. C

751. B

786. B

682. D

717. D

752. B

787. C

683. B

718. D

753. A

788. D

684. A

719. C

754. E

789. D

685. D

720. A

755. A

790. B

686. E

721. C

756. B

791. C

687. C

722. B

757. C

792. B

688. B

723. D

758. B

793. A

689. C

724. E

759. D

794. E

690. D

725. E

760. C

795. E

691. E

726. D

761. C

796. B

692. B

727. D

762. A

797. E

693. C

728. C

763. B

798. D

694. A

729. C

764. D

799. A

695. C

730. E

765. B

800. A

696. E

731. D

766. B

801. B

697. B

732. E

767. E

802. E

698. E

733. B

768. D

803. A

699. C

734. B

769. A

804. A

700. D

735. E

770. A

805. D

701. A

736. A

771. E

806. C

702. A

737. B

772. A

807. E

709

GMAT® Official Guide 2018

9.8 Answer Explanations
The following discussion of sentence correction is intended to familiarize you with the most efficient
and effective approaches to these kinds of questions. The particular questions in this chapter are
generally representative of the kinds of sentence correction questions you will encounter on the
GMAT exam.

668. In a review of 2,000 studies of human behavior that
date back to the 1940s, two Swiss psychologists,
declaring that since most of the studies had failed to
control for such variables as social class and family
size, none could be taken seriously.
(A)

psychologists, declaring that since most of the
studies had failed to control for such variables as
social class and family size,

(B)

psychologists, declaring that most of the studies
failed in not controlling for such variables like
social class and family size, and

(C)

psychologists declared that since most of
the studies, having failed to control for such
variables as social class and family size,

(D)

psychologists declared that since most of the
studies fail in controlling for such variables like
social class and family size,

(E)

psychologists declared that since most of the
studies had failed to control for variables such as
social class and family size,

Verb form; Diction
The subject of the sentence, two Swiss
psychologists, needs a main verb. The ‐ing verb
form declaring cannot, on its own, be the main
verb of a correct English sentence. Furthermore,
the clause that addresses the reason for not taking
the studies also seriously needs a subject (most of
the studies) and a verb (had failed).
A

B

710

The sentence needs a verb form that agrees
in person and number with the subject
two Swiss psychologists and that is in the
appropriate tense.
In addition to the problem with declaring,
explained above, in this version of the
sentence the phrase failed in not controlling
for is awkward and does not mean the
same thing as failed to control for. Also,
the expression such X like Y is incorrect in
English; the correct usage is such X as Y.

C

D

E

The correct form for the sentence’s main
verb declared is used here, but having failed is
a participial form and as such cannot be the
main verb in the clause.
English has a rule of sequence of tenses:
once a verb form is marked for past tense,
the following verb forms that describe
the same object or event have to be in the
past tense as well. Thus, fail is the wrong
verb form. The expression such X like Y is
incorrect in English; the correct usage is
such X as Y.
Correct. The subject two Swiss psychologists
is followed by a verb in the past tense
(declared); the dependent clause since …
family size also has the correct verb form
(had failed).

The correct answer is E.
669. In her presentation, the head of the Better Business
Bureau emphasized that companies should think of the
cost of conventions and other similar gatherings as not
an expense, but as an investment in networking that
will pay dividends.
(A)

as not an expense, but as

(B)

as not expense but

(C)

not an expense, rather

(D)

not as an expense, but as

(E)

not in terms of expense, but

Parallelism; Idiom
This sentence is constructed around not as X,
but as Y, which must start with the word not
in accordance with this idiomatic pattern, and
express both parts in a parallel way.
A
B

This sentence improperly places as before not.
This sentence improperly places as before not.

9.8 Sentence Correction Answer Explanations

C
D

E

This version lacks the required words as and
but.
Correct. The idiom has all of its parts and
expresses the two opposed concepts in
parallel terms.
Although in terms of is an acceptable
substitute for as, the construction is no
longer parallel due to the lack of a second in
terms of (or as) after but.

D

E

The correct answer is D.
670. Recent interdisciplinary studies advance the argument
that emotions, including those deemed personal
or private is a social phenomenon, though one
inseparable from bodily response.
(A)

private is a social phenomenon, though one
inseparable

(B)

private, are social phenomena that are
inseparable

(C)

private are a social phenomenon but are not
those separable

The dash would be correct to set off the
parenthetical expression only if including
had been immediately preceded by a dash;
otherwise a comma is needed. The phrase
not separable is awkward; it would be better
as are not separable or are inseparable.
The phrase also as private emotions is
wordy and unidiomatic. The phrase social
phenomena not inseparable not only does
not express the intended meaning, but is
also awkward without a verb and a relative
pronoun such as that: a better phrasing is
social phenomena that are not separable.

The correct answer is B.
671. Manufacturers rate batteries in watt‐hours; if they rate
the watt‐hour higher, the longer the battery can be
expected to last.
(A)

if they rate the watt‐hour higher, the longer

(B)

rating the watt‐hour higher, it is that much longer

(C)

the higher the watt‐hour rating, the longer

(D)

private—are social phenomena but not separable

(D)

(E)

also as private emotions, are social phenomena
not inseparable

the higher the watt‐hour rating, it is that much
longer that

(E)

when the watt‐hour rating is higher, the longer it is

Agreement; Rhetorical construction
The main problem is one of agreement: in the
subordinate clause starting with that, the subject
is the plural emotions, which demands the verb
are, not is. Also, the phrase starting with including
is a parenthetical expression that needs to be
set off from the rest of the clause, with some
punctuation to indicate a pause at its end (after
private).
A

B

C

The verb form is is incorrect, and should
instead be are; the parenthetical expression
is not separated at its end from the rest of
the clause.
Correct. Are is the correct agreeing verb
form, and the comma after private correctly
sets off the parenthetical expression.
Are is correct, but nothing after private sets
off the parenthetical expression from the
subsequent material. In addition, not those
separable is awkwardly phrased; it would be
better as not separable or inseparable as in the
correct answer choice B.

Logical predication; Grammatical
construction; Parallelism
There is a cause‐and‐effect connection between
the watt‐hour rating and the battery life; therefore,
the phrases describing the cause and effect have
to be expressed in two parallel grammatical
constructions using the two comparative forms of
the modifiers higher and longer.
A

B

C

Because of the way this version is
constructed, the pronoun they, referring to
manufacturers, illogically suggests that the
manufacturers can make the batteries last
longer simply by rating them higher. The
cause and effect is also unparallel.
Rating the watt‐hour higher needs a subject;
this faulty construction is known as a
dangling modifier. It is possible to construe
it as referring to manufacturers, which would
be confusing. See explanation of (A) above.
Correct. This version uses the correct logical
and grammatical structure the higher X, …
the longer Y.
711

GMAT® Official Guide 2018

D

E

This version is unparallel and
ungrammatical. The correct form is the
higher X, the longer Y.
This version is unparallel and
ungrammatical. The correct form is the
higher X, the longer Y.

The correct answer is C.

C

D

672. At the end of the 1930s, Duke Ellington was looking for
a composer to assist him—someone not only who could
arrange music for his successful big band, but mirroring
his eccentric writing style as well in order to finish the
many pieces he had started but never completed.
(A)

someone not only who could arrange music
for his successful big band, but mirroring his
eccentric writing style as well in order to finish

(B)

someone who could not only arrange music
for his successful big band, but also mirror his
eccentric writing style in order to finish

(C)

someone who not only could arrange music for
his successful big band, but also to mirror his
eccentric writing style in finishing

(D)

that being someone who could not only arrange
music for his successful big band, but mirroring
his eccentric writing style for finishing

(E)

being someone not only who could arrange
music for his successful big band, but mirror his
eccentric writing style as well, finishing

Rhetorical construction; Parallelism
In describing the qualities Duke Ellington sought
in a composer, the sentence uses the construction
not only … but also.… In this construction the
clauses need to be parallel—for example, not only
subject verb, but also subject verb.
A

B

712

This version is wordy, awkward, and
unparallel. Furthermore, the two verbs in
the not only … but also… construction have
to be in the same form, while here we have
arrange and mirroring.
Correct. This version exemplifies a truly
parallel construction. Here, not only and
but also are directly in front of the verbs
they introduce. The verbs denoting the two
qualities in the sought‐after composer are
the same form: arrange, mirror.

E

This version is awkward and unparallel.
The two verbs have to be in the same
form, while here we have arrange and to
mirror. Finally, the best way to express the
causal relationship between the composer’s
qualities and finishing Ellington’s pieces is
to use the conjunction in order to.
Instead of the shorter and clearer someone
who, this version uses the longer phrase that
being someone who, which is overly wordy.
Furthermore, the verbs are not in the same
form and so the construction is unparallel.
Finally, the best way to express the causal
relationship between the composer’s
qualities and finishing Ellington’s pieces is
to use the conjunction in order to.
Placing being in front of someone is
potentially misleading; it could illogically
refer to Ellington himself. The not only …
but also … construction is also unparallel.
Finally, the best way to express the causal
relationship between this composer’s
qualities and finishing Ellington’s pieces is
to use the conjunction in order to.

The correct answer is B.
673. While Noble Sissle may be best known for his
collaboration with Eubie Blake, as both a vaudeville
performer and as a lyricist for songs and Broadway
musicals, also enjoying an independent career as a
singer with such groups as Hahn’s Jubilee Singers.
(A)

and as a lyricist for songs and Broadway
musicals, also enjoying

(B)

and writing lyrics for songs and Broadway
musicals, also enjoying

(C)

and a lyricist for songs and Broadway musicals,
he also enjoyed

(D)

as well as writing lyrics for songs and Broadway
musicals, he also enjoyed

(E)

as well as a lyricist for songs and Broadway
musicals, he had also enjoyed

Grammatical construction; Idiom; Parallelism
As worded, this sentence opens with a dependent
clause (a clause that cannot stand on its own),
which requires a main clause (also known as an
independent clause) to complete the sentence;

9.8 Sentence Correction Answer Explanations

however, there is no main clause. Also, given the
placement of as before both, the as before a lyricist
is incorrect. It would be acceptable to write as both
a vaudeville performer and a lyricist or to write both
as a vaudeville performer and as a lyricist; it is not
acceptable to mix the two forms, as is done here.
A

B

C

D

E

The dependent clause, While . . . Broadway
musicals, is followed by a participial phrase
rather than a main clause and is therefore
ungrammatical. Furthermore, the word as
before a lyricist violates the parallel structure
required by the phrase both . . . and.
The construction as both a performer and
writing lyrics is incorrect. Also, like (A), this
version of the sentence does not supply a
main clause.
Correct. Unlike (A) and (B), this version
has a main clause. Also, unlike the other
version, it correctly uses the both x and
y form.
Although this version does supply the main
clause anticipated by While . . ., its use as
both a vaudeville performer as well as writing
lyrics is incorrect.
This version’s use of both x as well as y
instead of both x and y is incorrect. It also
introduces an inexplicable past perfect verb,
had . . . enjoyed, in the main clause.

The correct answer is C.
674. Air traffic routes over the North Pole are currently used
by only two or three planes a day, but it was found
by a joint Canadian–Russian study to be both feasible
as well as desirable if those routes are opened to
thousands more commercial planes a year.
(A)

Air traffic routes over the North Pole are
currently used by only two or three planes a day,
but it was found by a joint Canadian–Russian
study to be both feasible as well as desirable
if those routes are opened to thousands more
commercial planes a year.

(B)

Currently used by only two or three planes a day,
a joint Canadian–Russian study has found that if
air traffic routes over the North Pole are opened
to thousands more commercial planes a year, it
would be both feasible and desirable.

(C)

A joint Canadian–Russian study, finding it to be
both feasible as well as desirable to open air
traffic routes over the North Pole, which are
currently used by only two or three planes a day,
to thousands more commercial planes a year.

(D)

Although air traffic routes over the North Pole
are currently used by only two or three planes a
day, a joint Canadian–Russian study has found
that opening those routes to thousands more
commercial planes a year is both feasible and
desirable.

(E)

With air traffic routes over the North Pole
currently used by only two or three planes a
day, opening those routes to thousands more
commercial planes a year has been found by a
joint Canadian—Russian study as both feasible
and desirable.

Rhetorical construction; Verb form; Logical
predication
The point of the sentence is to share the results
of a study about air routes over the North Pole,
but the wording is confusing and verbose. The
passive construction it was found by a joint
Canadian–Russian study to be could be expressed
more directly and clearly in active voice: a joint
Canadian–Russian study has found.
The passive construction it was found by a
joint Canadian–Russian study to be, especially
followed by the conditional if, is wordy,
awkward, and unclear.
B
Used by only two or three planes illogically
modifies a joint Canadian–Russian study; the
pronoun it has no clear antecedent.
C The subject A joint Canadian–Russian study
has no verb; to thousands more commercial
planes is located too far away from to open,
which it is intended to modify.
D Correct. The sentence uses correctly placed
modifiers and the active voice to explain
clearly what a joint Canadian–Russian study
has found.
E The passive construction opening those routes
. . . has been found by a joint Canadian–
Russian study as both is wordy and
unidiomatic.
The correct answer is D.
A

713

GMAT® Official Guide 2018

675. From an experiment using special extrasensory
perception cards, each bearing one of a set of
symbols, parapsychologist Joseph Banks Rhine claimed
statistical proof for subjects who could use thought
transference to identify a card in the dealer’s hand.
(A)

for subjects who could use thought transference
to identify a card in the dealer’s hand

(B)

for a card in the dealer’s hand to be identified by
subjects with thought transference

(C)

of subjects able to identify with thought
transference a card in the dealer’s hand

(D)

that subjects could identify a card in the dealer’s
hand by using thought transference

(E)

that subjects are capable to use thought
transference for identifying a card in the dealer’s
hand

E

The correct answer is D.
676. A long‐term study of some 1,000 physicians indicates
that the more coffee these doctors drank, the more
they had a likelihood of coronary disease.
(A)

more they had a likelihood of coronary disease

(B)

more was their likelihood of having coronary
disease

(C)

more they would have a likelihood to have
coronary disease

(D)

greater was their likelihood of having coronary
disease

This sentence is meant to indicate that Joseph
Banks Rhine claimed that a certain experiment
statistically proved that subjects could identify
what symbol was on a card in a dealer’s hand by
using thought transference. The present version
of the sentence does not convey the intended
meaning well, however. What should follow proof
is a statement of the assertion that Rhine claims
the experiment has statistically proved, linked
to the word proof by the word that. Instead proof
is followed by a prepositional phrase for subjects
who. . . .

(E)

greater was coronary disease likely

A

A

Idiom; Rhetorical construction

B

C

D

714

While this version of the sentence correctly
follows proof with that followed by an
assertion, it fails to use the appropriate
idiom with capable; instead of capable to use,
it should have capable of using.

This version of the sentence inappropriately
attempts to describe the claim by using a
prepositional phrase, for subjects who. . . .
Like (A), this version of the sentence
inappropriately attempts to describe the
claim by using a prepositional phrase, for a
card in. . . .
Like (A) and (B), this version of the sentence
inappropriately attempts to describe the
claim by using a prepositional phrase. While
proof might reasonably be followed by of,
the phrase that follows the preposition is
ungrammatical, requiring a participle to
modify subjects, such as being able to. . . .
Correct. This version correctly uses the
idiom proof that followed by an assertion.

Idiom; Rhetorical construction
This sentence describes the results of a study in
which researchers found a correlation between
the amounts of coffee that people drank and their
likelihood of coronary disease. It most eloquently
expresses this correlation as a comparison of
parallel forms, using the idiom the + comparative
adjective phrase . . . the + comparative adjective
phrase. The two adjective phrases should have the
same grammatical form.

B

This version obscures the intended
correlation between coffee and likelihood. The
phrase the more they had a likelihood somewhat
illogically indicates that the research subjects
had likelihood to a greater degree rather than
that their likelihood was greater.
Although the adverb more is used to
indicate a greater degree or extent in phrases
such as more likely, the adjective more
normally indicates greater quantity. Thus,
the use of more as an adjective modifying
their likelihood is nonstandard. Differences in
a particular type of likelihood are normally
thought of as matters of degree, not of
quantity. Greater is the preferred adjective
for indicating such differences.

9.8 Sentence Correction Answer Explanations

C

D
E

The conditional verb phrase would have a
likelihood to have is redundant, wordy, and not
comparable to the simple past tense drank.
Correct. This version uses proper wording
and is clear and concise.
This completion of the comparison is not
idiomatic; moreover, it is ungrammatical,
attempting to use an adjective greater to
describe an adverb, likely.

B

C

The correct answer is D.
677. The intricate structure of the compound insect eye,
having hundreds of miniature eyes called ommatidia,
help explain why scientists have assumed that it
evolved independently of the vertebrate eye.
(A)

having hundreds of miniature eyes called
ommatidia, help explain why scientists have
assumed that it

(B)

having hundreds of miniature eyes that are called
ommatidia, helps explain why scientists have
assumed that they

(C)

with its hundreds of miniature eyes that are
called ommatidia, helps explain scientists’
assuming that they

(D)

with its hundreds of miniature eyes called
ommatidia, help explain scientists’ assuming that it

(E)

with its hundreds of miniature eyes called
ommatidia, helps explain why scientists have
assumed that it

D

E

This wording misleads as in (A).
Furthermore, the pronoun at the end of the
underlined part of the sentence should refer
back to the compound insect eye, so it cannot
be plural.
The phrase helps explain scientists’ assuming
correctly agrees with the subject structure.
However, it is nonstandard and awkward;
the use of assuming with the possessive is
unwarranted. Finally, the pronoun at the
end of the underlined part of the sentence
should refer back to the compound insect eye,
so it cannot be plural.
The subject structure and the verb help
do not agree in number. As in (C),
explain scientists’ assuming is verbose and
awkward, and the use of the assuming with
the possessive makes the construction
unnecessarily complex.
Correct. The phrase with its hundreds
correctly refers back to the compound insect
eye. The verb helps correctly agrees with
the subject structure. Finally, the pronoun
it correctly refers to the singular noun the
compound insect eye.

The correct answer is E.
678. In 1979 lack of rain reduced India’s rice production
to about 41 million tons, nearly 25 percent less than
those of the 1978 harvest.

Agreement; Logical predication

(A)

less than those of the 1978 harvest

This sentence addresses why scientists have
decided that the vertebrate eye and the insect
eye evolved independently of each other. The
insect eye is much more intricate, with hundreds
of miniature eyes. The sentence needs to be
clear as to what has hundreds of miniature eyes.
The structure or the insect eye? Furthermore, the
singular subject intricate structure requires the
singular verb helps.

(B)

less than the 1978 harvest

(C)

less than 1978

(D)

fewer than 1978

(E)

fewer than that of India’s 1978 harvest

A

This wording misleadingly suggests that the
intricate structure has miniature eyes, while
the correct subject of the predicate having
is the compound insect eye. Subject‐verb
agreement is violated.

Logical predication; Diction
The crucial part of this sentence is the latter part,
where India’s 1979 production of 41 million
tons of rice is compared to the production of the
previous year. What was produced in 1979 was
25 percent less than what was produced in 1978.
The grammatical means of comparison have to
express correctly this relationship between the
two harvests. It helps if you think of 41 million
tons of rice as the collective equivalent of the
1979 harvest.
715

GMAT® Official Guide 2018

A

B
C
D

E

This version exhibits redundant word choice.
The pronoun those refers to the tons of the
1978 harvest. Both harvests are measured
in tons, and it is clearer and simpler to
compare with the harvest itself, not with the
tons of the harvest.
Correct. This version correctly compares the
1978 harvest with the 1979 harvest.
This version is illogical. It compares rice
production in tons with a year, 1978.
In addition to the problem described in
(C) above, this version uses the wrong
comparative adjective. Fewer refers to
countable nouns, while less refers to
quantities. Since the comparative adjective
should refer to rice production, fewer is not
appropriate.
Since the comparative adjective should refer
to rice production, fewer is not appropriate.
Furthermore, the pronoun that refers back
to India’s rice production [fewer than the
rice production of India’s 1978 harvest] and
is redundant. The word harvest is sufficient
here to express the comparison.

Rhetorical construction; Grammatical
construction
Hurricanes at first begin traveling is redundant.
The sentence could start with At first, hurricanes
travel or with Hurricanes begin traveling; there is
no need to have both at first and begin. A concise
version of the sentence would also avoid that
direction is the way the prevailing winds in the
tropics blow. The meaning of way here is already
conveyed by direction.
A

B
C

The correct answer is B.
679. Hurricanes at first begin traveling from east to west,
because that direction is the way the prevailing winds
in the tropics blow, but they then veer off toward
higher latitudes, in many cases changing direction
toward the east before dissipating over the colder,
more northerly waters or over land.

716

(A)

Hurricanes at first begin traveling from east
to west, because that direction is the way the
prevailing winds in the tropics blow, but

(B)

At first, hurricanes travel from east to west,
because that is the direction of the prevailing
winds in the tropics, but

(C)

While hurricanes travel from east to west at first,
the direction of the prevailing winds blowing in
the tropics, and

(D)

Because hurricanes at first travel from east to
west, since it is the direction of the prevailing
winds in the tropics,

(E)

Hurricanes, beginning by traveling from east
to west, because this is the direction of the
prevailing winds in the tropics,

D

E

At first and begin are redundant. Also, that
direction is the way the prevailing winds blow
is unnecessarily wordy.
Correct. This version of the sentence is
grammatically correct and lacks redundancy.
This sentence begins with a dependent
clause (that is, a clause that cannot stand
on its own), anticipating a main clause
(also known as an independent clause) to
complete the sentence; a main clause does
follow, but it is connected to the initial
dependent clause by the coordinating
conjunction and, which would be
appropriate here only if the initial clause
was of the same grammatical type (that is,
was also a main clause).
The causal conjunction Because suggests that
the direction in which hurricanes initially
travel causes them later to veer off to the
north, but this make little logical sense.
This sentence opens with a main subject,
Hurricanes, but this subject has no verb. The
sentence’s main verb, veer, has as a subject
they.

The correct answer is B.
680. Unlike the virginal, whose single set of strings runs
parallel to the front edge of the instrument, the
harpsichord’s several sets of strings are placed at right
angles to its front edge.
(A)

whose single set of strings runs parallel to the
front edge of the instrument, the harpsichord’s
several sets of strings are

(B)

with a single set of strings running parallel to the
front edge of the instrument, the several sets of
strings of the harpsichord are

9.8 Sentence Correction Answer Explanations

(C)

which has a single set of strings that runs parallel
to the front edge of the instrument, in the case of
the harpsichord, several sets of strings are

(D)

was published in 1982 and which, winning both
the National Book Award and the Pulitzer Prize,
brought her the widest acclaim

(D)

which has a single set of strings that run
parallel to the front edge of the instrument, the
harpsichord has several sets of strings

(E)

was published in 1982, winning both the National
Book Award as well as the Pulitzer Prize, and
bringing her the widest acclaim

(E)

in which a single set of strings run parallel to the
front edge of the instrument, the harpsichord’s
several sets of strings are

Idiom; Grammatical construction

Parallelism; Agreement
The point of the sentence is to contrast two
instruments, but the sentence has been written to
contrast the virginal with the sets of strings on the
harpsichord. The proper contrast is between the
virginal and the harpsichord.
A

B
C
D
E

The virginal is illogically contrasted with
the sets of strings on the harpsichord. Note
the possessive form harpsichord’s in contrast
to virginal.
The virginal is illogically contrasted with
the sets of strings on the harpsichord.
In the case of the harpsichord is not parallel to
the virginal.
Correct. The contrast is properly drawn
between the virginal and the harpsichord.
The virginal is illogically contrasted with
the sets of strings on the harpsichord; the
verb run does not agree with the singular
subject set.

The correct answer is D.
681. Although Alice Walker published a number of essays,
poetry collections, and stories during the 1970s, her
third novel, The Color Purple, which was published in
1982, brought her the widest acclaim in that it won both
the National Book Award as well as the Pulitzer Prize.
(A)

(B)

(C)

which was published in 1982, brought her the
widest acclaim in that it won both the National
Book Award as well as the Pulitzer Prize
published in 1982, bringing her the widest
acclaim by winning both the National Book Award
and the Pulitzer Prize
published in 1982, brought her the widest
acclaim, winning both the National Book Award
and the Pulitzer Prize

This sentence claims that the 1982 novel The
Color Purple brought Alice Walker more acclaim
than her many publications in the 1970s.
A

B

C

D

E

The construction both the American Book
Award as well as the Pulitzer Prize is
unidiomatic; the correct idiomatic form is
both x and y.
Because this sentence uses only participial
phrases in the clause following the initial,
dependent clause, the sentence lacks a main
verb and is therefore ungrammatical.
Correct. This version correctly uses the
form both x and y and is grammatically
correct.
The use of which is inappropriate here;
although it would still be rhetorically
inferior to the correct answer (C), this
version would be acceptable if the word
which were deleted.
Like (A), this version of the sentence uses
the unidiomatic form both x as well as y.

The correct answer is C.
682. Many experts regarded the large increase in credit
card borrowing in March not as a sign that households
were pressed for cash and forced to borrow, rather
a sign of confidence by households that they could
safely handle new debt.
(A)

rather a sign of confidence by households that
they could safely

(B)

yet as a sign of households’ confidence that it
was safe for them to

(C)

but a sign of confidence by households that they
could safely

(D)

but as a sign that households were confident
they could safely

(E)

but also as a sign that households were
confident in their ability safely to

717

GMAT® Official Guide 2018

Idiom; Logical predication; Parallelism
The structure of this sentence expresses negation
and affirmation: Many experts regarded X not as a
sign that Y, but as a sign of Z. The idiomatic choice
of words for this construction should involve
two clauses introduced by not as … but as…. It is
also important to connect logically the adjective
confident with the noun it describes, that is,
households. If the adjective confident or its equivalent
noun confidence is related to other words in the
sentence, the message loses clarity and power.
A

B

C

D

E

The idiomatic choice of words to express the
negation of one clause and the affirmation
of another is not as … but as…. The affirmed
clause is incorrectly introduced by rather.
Furthermore, the preposition by does
not express the idea that households are
confident. The placement of the restrictive
clause that they could safely handle new debt
after households could also cause confusion.
The idiomatic choice of words to express the
negation of one clause and the affirmation
of another is not as … but as…. The affirmed
clause is incorrectly introduced by yet as.
The wording it was safe for them [households]
to handle new debt changes the intended
meaning, making the households less clearly
an agent in the handling.
The idiomatic choice of words to express the
negation of one clause and the affirmation
of another is not as … but as…. The affirmed
clause is incorrectly introduced by but
without the as. Furthermore, the preposition
by does not express the idea that households
are confident.
Correct. This version provides the correct
logical structure. It also expresses the
confidence of households using the most
succinct and direct wording.
The use of the word also is misleading here.
If the first embedded clause is negated, the
affirmed second clause cannot also be true.
In addition, in their ability, while expressing
the same idea as could, is redundant and
safely to handle is awkward; the wording in
(D) is superior.

683. A pioneer journalist, Nellie Bly’s exploits included
circling the globe faster than Jules Verne’s fictional
Phileas Fogg.
(A)

A pioneer journalist, Nellie Bly’s exploits included

(B)

The exploits of Nellie Bly, a pioneer journalist,
included

(C)

Nellie Bly was a pioneer journalist including in her
exploits the

(D)

Included in the pioneer journalist Nellie Bly’s
exploits are

(E)

The pioneer journalist’s exploits of Nellie Bly
included

Logical predication; Modification
This sentence intends to discuss Nellie Bly, a
pioneer journalist who counts among her exploits
a very fast trip around the globe. The current
construction is misleading because it suggests
that Bly’s exploits, not Bly herself, were a pioneer
journalist.
A

B
C

D

E

In this answer choice, a pioneer journalist
incorrectly modifies exploits, rather than
Nellie Bly.
Correct. This version correctly attributes the
modifier a pioneer journalist to Nellie Bly.
The wording of this version is illogical
because it makes Nellie Bly the subject of the
verb including.
Since this event clearly happened in the
past, the tense of the verb are is wrong.
Piling on too many descriptive words for
the noun exploits (the pioneer journalist Nellie
Bly’s exploits) makes the phrase unwieldy and
awkward.
This version can be construed as entailing
that the pioneer journalist and Nellie Bly are
two different people, one exploiting the other.

The correct answer is B.
684. In 1913, the largely self-taught Indian mathematician
Srinivasa Ramanujan mailed 120 of his theorems to
three different British mathematicians; only one, G. H.
Hardy, recognized the brilliance of these theorems,
but thanks to Hardy’s recognition, Ramanujan was
eventually elected to the Royal Society of London.

The correct answer is D.
(A)

718

only one, G. H. Hardy, recognized the brilliance
of these theorems, but

9.8 Sentence Correction Answer Explanations

(B)

they were brilliant, G. H. Hardy alone recognized,
but

(C)

these theorems were brilliant, but only one, G. H.
Hardy recognized;

(D)

but, only one, G. H. Hardy, recognizing their
brilliance,

(E)

only one G. H. Hardy recognized, but these
theorems were brilliant

685. Cost cutting and restructuring has allowed the
manufacturing company to lower its projected losses
for the second quarter, and they are forecasting a
profit before the end of the year.
(A)

has allowed the manufacturing company to lower
its projected losses for the second quarter, and
they are forecasting

(B)

has allowed for the manufacturing company to
lower its projected losses in the second quarter
and to forecast

(C)

have allowed that the manufacturing company
can lower the projected losses for the second
quarter, and to forecast

(D)

have allowed the manufacturing company to
lower its projected second-quarter losses and to
forecast

(E)

have allowed for the manufacturing company to
lower the projected losses in the second quarter,
as well as forecasting

Logical predication; Grammatical
construction
The point of the sentence is that only one of the
British mathematicians with whom Srinivasa
Ramanujan initially shared his theorems
recognized their brilliance, but that recognition
was sufficient to earn Ramanujan acclaim. The
sentence has been correctly constructed, with
the phrase only one referring clearly to British
mathematicians and the noun theorems repeated to
avoid confusion.
A

B

C

D

E

Correct. The phrase only one refers clearly
to British mathematicians, and the noun
theorems is repeated to avoid confusion.
They is intended to refer to theorems but
instead refers to mathematicians, causing
confusion.
Structured in this way, the sentence does
not make sense. Only one is intended to refer
to G. H. Hardy but instead seems to refer
to theorems, causing confusion. If only one is
taken to refer to G. H. Hardy, the absence of
a comma after the name and the absence of
an object for recognized make the sentence
ungrammatical.
The sentence is ungrammatical because
G. H. Hardy serves as the subject of a clause
but is not paired with a verb.
Only one G. H. Hardy recognized incorrectly
implies (in an awkwardly inverted wording
structure) that Hardy recognized only one
theorem; these theorems were brilliant thanks
to Hardy’s recognition illogically suggests
that Hardy’s recognition is what made the
theorems brilliant.

The correct answer is A.

Agreement; Idiom; Verb form
The point of the sentence is to explain the
two main effects of the changes made by
the company. However, the singular verb has
allowed does not agree with the compound
subject cost cutting and restructuring, which are
far more plausibly understood as two actions
rather than as two facets of a single action. In
principle, the manufacturing company could be
construed as plural (referring collectively to
the decision makers and spokespeople who are
projecting losses and forecasting a profit), but
the plural pronoun they does not agree with
the earlier its, which treats the antecedent as
singular.
A

B

C

The singular verb has allowed does not agree
with the compound subject cost cutting
and restructuring; the plural pronoun they
does not agree with the intended singular
antecedent company.
The singular verb has allowed does not agree
with the compound subject cost cutting and
restructuring; allowed for is not the correct
idiom.
Allowed that is not the correct idiom; can
lower and to forecast are not grammatically
parallel.

719

GMAT® Official Guide 2018

Correct. The sentence uses the correct
subject–verb combination cost cutting
and restructuring have allowed; the two
occurrences of the pronoun its agree with
each other in treating their antecedent
company as singular; and the two effects
to lower and to forecast are parallel and
idiomatic.
E Allowed for is not the correct idiom; to lower
and as well as forecasting are not parallel.
The correct answer is D.
D

686. The Life and Casualty Company hopes that by
increasing its environmental fund reserves to $1.2
billion, that it has set aside enough to pay for
environmental claims and no longer has to use its
profits and capital to pay those claims bit by bit, year
by year.
(A)

that it has set aside enough to pay for
environmental claims and no longer has

(B)

enough has been set aside with which
environmental claims can be paid and it will have
no longer

(C)

it has set aside enough for payment of
environmental claims and thus no longer
having

(D)

enough has been set aside to pay for
environmental claims, thus no longer having

(E)

it has set aside enough to pay for environmental
claims and will no longer have

Grammatical construction; Logical
predication
All predicates need a proper logical subject. Here,
the relevant predicates are the verbs increase,
set aside, and have. With it as the subject for set
aside—referring back to the Life and Casualty
Company—all three verbs should have this
as their understood subject. With a different
subject for set aside, at least one of the other
verbs lacks a proper logical subject. Also, this
sentence uses the word that after hope to start
the subordinate clause, but then incorrectly
repeats the that after the initial adverbial phrase
(by increasing . . . billion). Only the first that is
grammatically correct.

720

A

B

C

D

E

The additional that makes the sentence
ungrammatical. Also, because in this context
the hope is forward‐looking (bit by bit, year
by year), it would be preferable to use the
future tense, will no longer have to, instead of
the present no longer has to.
With enough as the subject of set aside, the
next subject (it) is naturally interpreted as
enough, but this is not a logical choice for
the subject of use.
The phrase no longer having is
ungrammatical when connected to the
rest of the sentence by and, which should
connect two regular clauses; no longer
having . . . is a mere phrase, not a clause,
and no longer has would be correct.
For no longer having, illogically, the implied
subject is enough; the implied subject,
instead, should be the company referred to
at the beginning of the sentence.
Correct. There is only one that, and will
no longer have to use has its proper logical
subject (it) from the clause preceding it.

The correct answer is E.
687. Like ancient Egyptian architectural materials that
were recycled in the construction of ancient Greek
Alexandria, so ancient Greek materials from the
construction of that city were reused in subsequent
centuries by Roman, Muslim, and modern builders.
(A)

Like ancient Egyptian architectural materials that
were recycled in the construction of

(B)

Like recycling ancient Egyptian architectural
materials to construct

(C)

Just as ancient Egyptian architectural materials
were recycled in the construction of

(D)

Just as they recycled ancient Egyptian
architectural materials in constructing

(E)

Just like ancient Egyptian architectural materials
that were recycled in constructing

Diction; Parallelism
When two situations are asserted to be similar,
the proper way to express this is with the paired
expressions just as . . . so . . ., not like . . . so.
Moreover, the two compared situations should
be expressed as clauses, not as noun phrases. Thus
the clause ancient Egyptian architectural materials

9.8 Sentence Correction Answer Explanations

were recycled . . . is correct, as opposed to a noun
phrase like ancient Egyptian architectural materials
that were recycled . . .
A

B

C

D

E

Just as and a following clause with a passive
verb are preferred, but instead this option
has like and a following noun phrase (ancient
Egyptian materials . . .). It appears, somewhat
implausibly, to say that the ancient Greek
materials were similar to the earlier ancient
Egyptian ones in that both were used by
Roman, Muslim, and modern builders.
Just as and a following clause with a passive
verb are preferred, but instead this option
has like and a following noun phrase
(recycling ancient Egyptian materials . . .).
It appears illogically to say that the action
of recycling was similar to the ancient
Greek materials in that both were reused in
subsequent centuries.
Correct. The expressions just as and so are
paired to link the two clauses in parallel, and
both clauses use the passive construction.
Just as is used to connect two clauses, which
is good, but the first clause employs the
active construction rather than the preferred
passive, so there is a failure of parallelism.
Also, it is unclear what they refers to. The
sentence appears to say illogically that some
unidentified group’s action of recycling was
similar to the ancient Greek materials’ being
reused.
Just as introducing a clause with a passive
verb is preferable, but this sentence uses
like and a following noun phrase (ancient
Egyptian materials . . .). It appears, somewhat
implausibly, to say that the ancient Greek
materials were similar to the earlier ancient
Egyptian ones in that both were used by
Roman, Muslim, and modern builders.

The correct answer is C.
688. Especially in the early years, new entrepreneurs may
need to find resourceful ways, like renting temporary
office space or using answering services, that make
their company seem large and more firmly established
than they may actually be.
(A)

that make their company seem large

(B)

to make their companies seem larger

(C)

thus making their companies seem larger

(D)

so that the companies seem larger

(E)

of making their company seem large

Grammatical construction; Diction
The clause beginning with that suggests that a
company can be made to seem better than it
actually is. The comparison signaled by than appears
to be intended to apply to both the size of the
company and the degree to which the company is
firmly established. In that case, it is a mistake to use
large instead of larger. If, on the other hand, large is
not intended to be part of the comparison, it would
be better to clarify this by separating the two topics
in a construction such as make their company seem
large and also make it seem more firmly established. .
. . The phrase resourceful ways suggests purpose, so
ways should connect with to or of, rather than a that
clause to capture the intended meaning.
A

B

C
D

E

Large is the incorrect form to express
comparison; it should be larger; the that
clause does not adequately capture the idea
of purpose implicit in ways. The singular
company does not agree with the plural
pronoun they. Thus, the sentence either
commits an agreement mistake or illogically
says that entrepreneurs need to find ways
to make themselves seem large and more
firmly established.
Correct. Larger is the correct form to
express the comparative meaning, and to
correctly follows way. Companies agrees with
the plural they.
The phrase beginning thus making does not
capture the idea of purpose implicit in ways.
Clauses beginning so that can express purpose,
but do not fit with ways in the manner
required here: to or of is needed after ways.
Large is the incorrect form to express
comparison; it should be larger. The singular
company does not agree with the plural
pronoun they. Thus, the sentence either
commits an agreement mistake or illogically
says that entrepreneurs need to find ways
to make themselves seem large and more
firmly established.

The correct answer is B.

721

GMAT® Official Guide 2018

689. Unlike the nests of leaf cutters and most other ants,
situated underground or in pieces of wood, raider
ants make a portable nest by entwining their long
legs to form “curtains” of ants that hang from logs or
boulders, providing protection for the queen and the
colony larvae and pupae.

The referent of those is unclear; presumably
it refers to nests, but grammatically it has
no clear antecedent. If it is taken to refer to
nests, those creates an illogical comparison
with raider ants.

The correct answer is C.

(A)

the nests of leaf cutters and most other ants,

(B)

the nests of leaf cutters and most other ants,
which are

(C)

leaf cutters and most other ants, whose nests
are

(D)

leaf cutters and most other ants in having nests

(A)

fasts, in their ability to survive

(E)

those of leaf cutters and most other ants with
nests

(B)

fasts, having their ability to survive

(C)

fasts, due to having the ability of surviving

(D)

fasts because they are able to survive

(E)

fasts because of having the ability of surviving

Logical predication
As worded, this sentence draws a contrast
between raider ants and the nests of leaf cutters
and most other ants. The appropriate contrast
would be with leaf cutters and most other ants
themselves, not their nests.
A

B

C

D

722

E

As indicated above, the appropriate contrast
is between raider ants and other kinds of
ants (namely leaf cutters and most ants). As
worded, this version of the sentence says
something obviously true: of course the
nests of leaf cutters and most other ants do
not make portable nests.
Like (A), this sentence compares nests with
raider ants. Also, the referent of the relative
pronoun which is ambiguous, possibly
modifying ants, and possibly modifying nests.
Correct. This version correctly draws the
contrast between raider ants and other kinds
of ants. Furthermore, unlike in (B) and (D),
it is clear here that situated underground or
in pieces of wood applies to the nests of leaf
cutters and most other ants.
This sentence correctly compares leaf cutters
and other ants with raider ants, but the
prepositional phrase in having nests suggests
that it is raider ants, not leaf cutters and
most other ants, that have nests situated
underground or in pieces of wood; however,
the rest of the sentence indicates that in fact
raider ants’ nests are not situated in such
locations.

690. Turtles, like other reptiles, can endure long fasts,
in their ability to survive on weekly or even monthly
feedings; however, when food is readily available, they
may eat frequently and grow very fat.

Idiom; Diction
To express why turtles can endure long fasts—
their ability to survive on only occasional
feedings—it is clearer and more idiomatic to use
because than to use in or having. Also, the noun
ability here requires to introducing a noun phrase
denoting the nature of the ability.
A
B
C

D
E

In does not express the causal relationship
clearly, whereas because does.
Having does not express the causal
relationship clearly.
Ability of is incorrect; ability must be followed
by to in order to express the intended
meaning. Due to having the ability is awkward,
nonstandard, and unnecessarily wordy.
Correct. This option uses because to express
the causal relation and uses to after ability.
Ability of is incorrect; ability must be
followed by to in order to express the
intended meaning. Because of having the
ability is awkward, nonstandard, and
unnecessarily wordy.

The correct answer is D.
691. Thai village crafts, as with other cultures, have
developed through the principle that form follows
function and incorporate readily available materials
fashioned using traditional skills.
(A)

as with

(B)

as did those of

9.8 Sentence Correction Answer Explanations

(C)

as they have in

(D)

like in

(E)

like those of

Logical predication; Diction; Verb form
The phrase as with other cultures is initially
confusing given that no culture has been
specifically mentioned; Thai village culture is
indirectly referenced by the mention of Thai village
crafts, so perhaps that is what is meant. But then,
looking at the phrase in context, it becomes clear
that the sentence is intended to indicate that the
crafts of other cultures are similar in certain ways
to Thai village crafts. Thus, the sentence should say
those of other cultures. Furthermore, the use here of
as with is questionable. To do the job it is supposed
to do here (to indicate that the crafts of Thai
villages are like the crafts of other cultures in a
particular way), as with should be at the beginning
of the sentence: As with the crafts of other cultures,
Thai village crafts have developed. . . . Alternatively,
like could be used here instead of as with.
A

B

C

D

The appropriate comparison is between Thai
village crafts and those (i.e., crafts) of other
cultures, not the other cultures themselves.
Additionally, like would be more appropriate
here than as with.
The verb form here is incorrect. [D]id is not
parallel to have later in the sentence. The
phrase as have those of other cultures would be
parallel, but it is in the wrong position. To be
correct, it would need to occur after the main
verb is introduced: Thai village crafts have
developed, as have those of other cultures, . . .
The use of the pronoun they is inaccurate;
the reader is likely to take its antecedent
to be Thai village crafts, not crafts.
Furthermore, even if the pronoun here
were not problematic, if the construction as
they have in other cultures is used, it should
occur after the main verb (have developed)
is introduced.
The comparative term like compares two
nouns or noun phrases, but in this version
of the sentence, like compares a noun
(crafts) with a prepositional phrase (in other
cultures).

E

Correct. [T]hose of other cultures clearly
refers to the crafts of other cultures; like
is appropriate for making a comparison
between two sorts of things (crafts).

The correct answer is E.
692. Tropical bats play important roles in the rain forest
ecosystem, aiding in the dispersal of cashew, date,
and fig seeds; pollinating banana, breadfruit, and
mango trees; and indirectly help produce tequila by
pollinating agave plants.
(A)

pollinating banana, breadfruit, and mango trees;
and indirectly help produce

(B)

pollinating banana, breadfruit, and mango trees;
and indirectly helping to produce

(C)

pollinating banana, breadfruit, and mango trees;
and they indirectly help to produce

(D)

they pollinate banana, breadfruit, and mango
trees; and indirectly help producing

(E)

they pollinate banana, breadfruit, and mango
trees; indirectly helping the producing of

Logical predication; Parallelism
This sentence expresses a list of the roles tropical
bats play in the rain forest ecosystem. Since
these roles are enumerated in a list, and since
the first member of the list is already provided,
it is necessary to maintain the same structure
for the rest of the members of the list in order
to maintain parallelism and clarity. Note that
semicolons separate the members of the list,
leaving the commas to mark series of items
within each member of the list.
A

B

C

In this version, the third member of the list
does not maintain the ‐ing verb form that
the two previous members use.
Correct. This version correctly maintains
the parallel structure (aiding in …;
pollinating …; and helping …).
In this version, the third member of the
list does not maintain the ‐ing verb form
of the two previous members of the list. In
addition, this member of the list includes a
subject (they) while the other members do
not, again violating parallelism.

723

GMAT® Official Guide 2018

D

E

In order to maintain parallelism the verb
that is the member of the list has to be in
the ‐ing form, not its complement. Thus,
the ‐ing has to be on the verb help, not on
produce.
Although this version maintains parallelism
throughout, the phrase helping the producing
is an incorrect construction in English.

C

D

E

The correct answer is B.
693. Discussion of greenhouse effects have usually had as
a focus the possibility of Earth growing warmer and to
what extent it might, but climatologists have indicated
all along that precipitation, storminess, and temperature
extremes are likely to have the greatest impact on people.
(A)

Discussion of greenhouse effects have usually
had as a focus the possibility of Earth growing
warmer and to what extent it might,

(B)

Discussion of greenhouse effects has usually
had as its focus whether Earth would get warmer
and what the extent would be,

(C)

Discussion of greenhouse effects has usually
focused on whether Earth would grow warmer
and to what extent,

(D)

The discussion of greenhouse effects have
usually focused on the possibility of Earth getting
warmer and to what extent it might,

(E)

The discussion of greenhouse effects has usually
focused on whether Earth would grow warmer
and the extent that is,

Agreement; Parallelism
The sentence contrasts climatologists’ views
concerning greenhouse effects with other views
that emphasize global warming. The main subject
of the sentence is discussion . . ., which is singular,
so the main verb should be singular. The two
things that are said to be the focus of discussion
should be in parallel form.
A

B

724

The plural verb have . . . had does not agree
with the singular subject discussion. The
phrases the possibility . . . warmer and to what
. . . might are not parallel.
The verb form has had as its focus is
unnecessarily wordy; the noun clauses are
parallel in form, but it is not clear what the
extent refers to.

Correct. This has correct subject‐verb
agreement, eliminates the wordiness of the
original sentence, and the phrases whether . . .
warmer and to what extent are parallel.
The singular subject discussion does not agree
with the plural verb have focused. The possibility
of . . . is not parallel with to what extent. . . .
The two phrases following on are not in
parallel form. What that refers to in the
extent that is is unclear.

The correct answer is C.
694. In the seventh century B.C., the Roman alphabet
was adapted from the Etruscan alphabet, which
in turn had been adapted in the previous century
from a western Greek alphabet, which itself had
been adapted earlier in the same century from the
Phoenician alphabet.
(A)

which itself had been adapted earlier

(B)

adapting itself earlier

(C)

itself being adapted earlier

(D)

having been earlier adapted itself

(E)

earlier itself having been adapted

Rhetorical construction; Verb form; Logical
predication
This sentence describes a string of adaptations of
the alphabet, tracing back from the seventh century
BC through two points in the eighth century
BC. Because the latest of the three adaptations is
temporally located in past tense, earlier adaptations
should be located in the past perfect tense.
A

B

Correct. This sentence is properly
constructed and uses the appropriate verb
forms for the relationships among the
events that it describes.
It is unclear what the participial phrase
adapting itself earlier . . . refers to. The
present participle could be used for an
event that was simultaneous with, or
part of, the event mentioned before
the underlined portion. However, it is
inappropriate for describing an event
that took place even earlier than the
seventh century BC, which has already
been designated in the sentence as past
tense. The reflexive form adapting itself is
nonsensical in this sentence.

9.8 Sentence Correction Answer Explanations

C

D

E

The present tense of the reflexive participial
phrase is inappropriate for describing an
event prior to the seventh century BC, given
that the seventh century BC has already
been designated in the sentence as past
tense.
Without commas around itself, the pronoun
appears to be reflexive. The combination of
passive and reflexive then makes no sense.
If itself is intended simply for emphasis,
rather than as a reflexive pronoun, it would
be preferable, in this context, to set it off
with commas. But if it were intended in that
way, it would seem to refer to the Roman
alphabet, and the claim made in the sentence
would be confusing and nearly nonsensical.
This modifier is confusingly placed before
the designation of time (in the same century)
and source (from the Phoenician alphabet).
The adverb earlier is misplaced before the
pronoun itself; it should be immediately
before the phrase in the same century.

The correct answer is A.
695. With corn, soybean, and wheat reserves being
low enough so a poor harvest would send prices
skyrocketing, grain futures brokers and their clients are
especially interested in weather that could affect crops.
(A)

being low enough so

(B)

so low such that

(C)

so low that

(D)

that are low enough so

(E)

that are so low such that

Idiom; Rhetorical construction
This sentence opens with a long participial
phrase (With . . . skyrocketing) that describes
conditions within which the action of the main
clause (grain futures brokers . . . crops) occurs.
The opening phrase compares one economic
condition (grain reserve levels) with another
(prices) using the idiomatic expression so low that.
Some of the phrases used in expressing this kind
of comparison are wordy and indirect (being low
enough so). Thus, they are not standard in written
English, even though they may be accepted in
some informal speaking contexts.

A
B

C
D

E

This phrase is wordy and unidiomatic.
This phrase combines two idioms (so . . . that
and such that) in a way that does not clearly
make sense. It is wordy and redundant; both
so . . . that and such signal comparison.
Correct. This wording is standard, clear, and
direct.
The relative clause introduced by that are
makes this phrase unnecessarily wordy and
cumbersome; enough and so are redundant.
This version of the phrase is the most
redundant of all—piling comparative terms
one upon the other (that are so and such).
The repetition obscures instead of develops
meaning.

The correct answer is C.
696. By 1940, the pilot Jacqueline Cochran held seventeen
official national and international speed records,
and she earned them at a time when aviation was
still so new for many of the planes she flew to be of
dangerously experimental design.
(A)

and she earned them at a time when aviation was
still so new for many of the planes she flew to be

(B)

earning them at a time that aviation was still so
new for many of the planes she flew to be

(C)

earning these at a time where aviation was still
so new that many of the planes she flew were

(D)

earned at a time in which aviation was still so
new such that many of the planes she flew were

(E)

earned at a time when aviation was still so new
that many of the planes she flew were

Rhetorical construction; Diction
Since this is a complex sentence with a lot of
elements, its rhetorical construction is crucial
in order to maintain effective communication.
The underlined part emphasizes that on top
of earning seventeen speed records, the pilot
Jacqueline Cochran did so at a time when planes
were of dangerous experimental design because
aviation was still so new.
A

This version is too wordy and does not
contribute to clarity. Economical, clear
expression connects earned directly with the
noun speed records, which it modifies.

725

GMAT® Official Guide 2018

B

C

D

E

Here, the word earning takes the pilot
herself, not the records, as its subject.
However, earning is close to the records, not
to Jacqueline Cochran, making this sentence
hard to process. In addition, the causal
relationship between aviation being new and
planes being of experimental design is more
effectively communicated by the structure so
new that many X were Y, not by the structure
so new for many X to be Y.
The word earning takes the pilot herself, not
the records, as its subject. However, earning
is close to the records, not to Jacqueline
Cochran, making this sentence hard to
understand. Furthermore, the use of where to
modify time is wrong; where refers to place.
The correct pronoun for time is when. The
word such is redundant within so new that
many X were Y.
Correct. The word earned is close to the
noun it modifies, speed records. This version
uses the clearest expression of causal
relationship: so new that many X were Y.

The correct answer is E.
697. The foundation works to strengthen local and
regional agricultural markets and cooperating with
governments, improving access for farmers for
productive resources such as land and credit.

726

(A)

cooperating with governments, improving access
for farmers for

(B)

cooperates with governments to improve access
for farmers to

(C)

cooperate with governments for improvements
of access for farmers to

(D)
(E)

plausible because of the redundancy of works to
cooperate. In the former, the proper verb form,
parallel with the present-tense works, would be
cooperates. In the latter, the proper verb form,
parallel with the infinitive to strengthen, would
be to cooperate or simply cooperate. Only one of
the answer choices resolves the uncertainty of
meaning in a coherent and well-formed way.
A
B

C

D

E

Cooperating is incorrect, since it is not in the
same form as either works or to strengthen.
Correct. This version correctly represents
the foundation’s two actions by putting
the verbs works and cooperates in parallel
form. Cooperates . . . to improve is the most
concise phrasing to express the purpose of
improving access.
Cooperate . . . for improvements of access
for farmers is awkwardly phrased and
unnecessarily wordy.
Cooperate . . . and improve accessibility
for farmers is awkwardly phrased and
unnecessarily wordy.
In cooperation destroys the parallelism required
by works to: there should be a verb following
works to, not this prepositional phrase.

The correct answer is B.
698. A professor at the university has taken a sabbatical to
research on James Baldwin’s books that Baldwin wrote
in France while he was living there.
(A)

on James Baldwin’s books that Baldwin wrote in
France while he was living there

(B)

about the books James Baldwin wrote in France

(C)

into James Baldwin’s books written while in France

cooperate with governments and improve
accessibility for farmers for their

(D)

on the books of James Baldwin, written while he
lived in France

in cooperation with governments to improve
access for farmers for

(E)

the books James Baldwin wrote while he lived in
France

Parallelism; Rhetorical construction

Diction; Rhetorical construction

As written, this sentence does not clearly
indicate whether and is intended to conjoin
two things that the foundation does (working
to strengthen markets and cooperating with
governments) or two things that the foundation
works to accomplish (strengthening markets and
cooperating with governments). The latter is less

The phrasing of this sentence is wordy and
redundant: books that Baldwin wrote in France
while he was living there could more concisely be
expressed with books James Baldwin wrote while
he lived in France. The verb research requires a
direct object, not a preposition followed by its
object.

9.8 Sentence Correction Answer Explanations

A

B
C

D

E

On incorrectly follows to research, and books
that Baldwin wrote in France while he was
living there includes redundant information.
About incorrectly follows to research.
Into incorrectly follows to research. Given
this sentence structure, while in France
seems to say, illogically, that the books were
written while they were in France.
On incorrectly follows to research; also, there
is no need to make written while he lived in
France into an independent phrase instead
of a relative clause.
Correct. Research takes a direct object,
which describes the books directly without
redundancy.

The correct answer is E.
699. Researchers now regard interferon as not a single
substance, but it is rather a biological family of
complex molecules that play an important, though not
entirely defined, role in the immune system.
(A)

as not a single substance, but it is rather a
biological family of complex molecules that play

(B)

as not a single substance but as a biological
family of complex molecules playing

(C)

not as a single substance but as a biological
family of complex molecules that play

(D)

not to be a single substance but rather a
biological family of complex molecules playing

(E)

not as a single substance but instead as being a
biological family of complex molecules that play

Parallelism; Rhetorical construction
This sentence draws a contrast between how
interferon is and is not regarded by researchers.
However, the two parts of the contrast are not
expressed in parallel form since the first is a
prepositional phrase (as not a single substance),
and the second is a clause (it is rather a biological
family). Furthermore, the wording regard
interferon as not is awkward and confusing. For
clarity and proper parallelism, the contrast should
be constructed using the expression not as X but
as Y, where X and Y are both noun phrases.
A

The contrast is not expressed using parallel
grammatical structure, and regard interferon
as not is awkward and confusing.

B
C

D

E

The wording regard interferon as not is awkward
and confusing and violates proper parallelism.
Correct. The contrast is expressed using the
parallel structure not as a single substance but
as a biological family.
The main defect here is that regard interferon
not to be a single substance is unidiomatic; regard
. . . as . . . is the correct form; and completing
the parallelism correctly would require the
repetition of as, in the phrase but rather as . . .
The contrast is not expressed using parallel
grammatical structure since a single substance
is a noun phrase, while being a biological
family is a participial phrase; omitting the
unnecessary words being and instead would
improve the sentence.

The correct answer is C.
700. The remarkable similarity of Thule artifacts throughout
a vast region can, in part, be explained as a very rapid
movement of people from one end of North America to
the other.
(A)

The remarkable similarity of Thule artifacts
throughout a vast region can, in part, be
explained as

(B)

Thule artifacts being remarkably similar
throughout a vast region, one explanation is

(C)

That Thule artifacts are remarkably similar
throughout a vast region is, in part, explainable as

(D)

One explanation for the remarkable similarity of
Thule artifacts throughout a vast region is that
there was

(E)

Throughout a vast region Thule artifacts are
remarkably similar, with one explanation for this
being

Logical predication; Grammatical
construction; Rhetorical construction
The intended meaning of the sentence is that
the rapid movement of people across North
America is one explanation of the similarity of
Thule artifacts throughout a vast region. As worded,
however, the sentence is illogical: The sentence
indicates that the similarity in artifacts was a
rapid movement of people, which makes no sense.
Instead of equating similarity with movement,
the sentence needs to identify this movement of
people as a cause of similarity among artifacts.
727

GMAT® Official Guide 2018

A

B

C

D
E

As worded, this version of the sentence makes
the illogical statement that the similarity
among artifacts is explainable as a very rapid
movement. It should specify that the similarity
of artifacts may be a consequence of the rapid
population movement.
This version of the sentence is syntactically
awkward, and leaves unclear what the main
subject, one explanation, is supposed to be an
explanation of.
Like (A), this version of the sentence
equates the similarity of artifacts with
the movement of people, when a causal
connection is what is intended.
Correct. This version adequately expresses
the intended causal connection.
This version is awkward, introducing the
causal connection with the unnecessarily
wordy and indirect string of prepositional
phrases, with one explanation for this. . . .

The correct answer is D.
701. The widely accepted big bang theory holds that the
universe began in an explosive instant ten to twenty
billion years ago and has been expanding ever since.
(A)

that the universe began in an explosive instant
ten to twenty billion years ago and has been
expanding

(B)

that the universe had begun in an explosive
instant ten to twenty billion years ago and had
been expanding

(C)

that the beginning of the universe was an
explosive instant ten to twenty billion years ago
that has expanded

(D)

the beginning of the universe to have been an
explosive instant ten to twenty billion years ago
that is expanding

(E)

the universe to have begun in an explosive
instant ten to twenty billion years ago and has
been expanding

Logical predication; Verb form
The sentence describes the central tenet of a
theory about how the universe began. The focus
of the second clause should be consistently on
the subject the universe, and all verbs in the clause
beginning with that must describe what the
universe did at the initial explosive moment.
728

A
B

C
D

E

Correct. Both verbs in the second clause
correctly take universe as their subject.
Had begun is the wrong tense because it
describes action that occurred farther in the
past than some other, specified past action.
The relative clause that has expanded
describes instant, which makes no sense.
The beginning of the universe to have been … is
unnecessarily indirect and wordy; illogically
suggests that beginning is expanding, not the
universe.
The verb phrases to have begun and has been
expanding both reference the same subject
of the clause, universe, and therefore need to
be parallel.

The correct answer is A.
702. Between 14,000 and 8,000 B.C. the ice cap that
covered northern Asia, Europe, and America began
to melt, uncovering vast new areas that were to be
occupied by migrating peoples moving northward.
(A)

began to melt, uncovering vast new areas that
were to be occupied

(B)

began melting, to uncover vast new areas to be
occupied

(C)

began, by melting, to uncover vast new areas for
occupation

(D)

began, after melting, uncovering vast new areas
which are to be occupied

(E)

would begin to uncover, through melting, vast
new areas for occupation

Verb form; Logical predication
The sentence explains what happened when
an ice cap began to melt. The participial phrase
uncovering vast new areas succinctly describes
the immediate effects of the melting. The verb
form were to be occupied is used to indicate that
occupation would take place at a time in the
future from the time of the melting.
A

B

Correct. The sentence succinctly expresses
immediate and future effects of the melting
of an ice cap.
The infinitive to uncover incorrectly implies
that the ice cap melted for the purpose of
uncovering new areas.

9.8 Sentence Correction Answer Explanations

C

D

E

Since melting is what caused new areas to be
uncovered, that word should be part of the
main verb, not placed in the nonrestrictive
prepositional phrase by melting.
It does not make sense to say that the ice
cap began . . . uncovering new areas after it
had melted—since the ice cap no longer
existed in the areas where it had melted.
Since melting is what caused new areas to be
uncovered, that word should be part of the
main verb, not placed in the nonrestrictive
prepositional phrase through melting.

B

C

D

The correct answer is A.
703. Bengal‐born writer, philosopher, and educator
Rabindranath Tagore had the greatest admiration
for Mohandas K. Gandhi the person and also as a
politician, but Tagore had been skeptical of Gandhi’s
form of nationalism and his conservative opinions
about India’s cultural traditions.

E

Correct. This version correctly conjoins two
parallel phrases, Gandhi as a person and as
a politician, and, in using two simple past
tenses to introduce the two emotions, marks
them as holding at the same time.
The phrase not only X but also Y matches
the meaning of this sentence: Tagore had
not only admiration but also skepticism.
However not only has to precede admiration
for this rhetorical construction to be parallel.
The noun admiration as it is positioned in
this sentence should take the preposition for,
not of, since it refers to a person. The adverb
also is redundant because it expresses the
same meaning as the conjunction and.
As in (D), the noun admiration should take
the preposition for. As in (C), the rhetorical
structure of not only X but also Y is violated.
Finally, the use of the past perfect tense had
been is misleading with respect to the time
line.

(A)

for Mohandas K. Gandhi the person and also as
a politician, but Tagore had been

(B)

for Mohandas K. Gandhi as a person and as a
politician, but Tagore was also

(C)

for Mohandas K. Gandhi not only as a person
and as a politician, but Tagore was also

(D)

of Mohandas K. Gandhi as a person and as also
a politician, but Tagore was

(A)

equally likely to exceed the proposed speed limit
as

(E)

of Mohandas K. Gandhi not only as a person and
as a politician, but Tagore had also been

(B)

equally likely to exceed the proposed speed limit
as they are

(C)

equally likely that they will exceed the proposed
speed limit as

(D)

as likely that they will exceed the proposed
speed limit as

(E)

as likely to exceed the proposed speed limit as
they are

Rhetorical construction; Parallelism
This sentence describes the writer and philosopher
Tagore’s two types of feelings for Gandhi. The
underlined part of the sentence has to express
correctly the time line of these two feelings (they
happened simultaneously). The underlined part
also has to express the correct relationship between
the complements of admiration and skepticism.
A

To maintain parallelism, it is important for
two conjoined phrases to be of the same
grammatical type. Thus, it is appropriate to
conjoin Gandhi the person and the politician, or
Gandhi as a person and as a politician, but it is
nonstandard in English to mix and match. In
addition, the use of the past perfect tense had
been places the skepticism earlier on the time
line than the admiration, which is misleading.

The correct answer is B.
704. Traffic safety officials predict that drivers will be
equally likely to exceed the proposed speed limit as
the current one.

Idiom; Parallelism
This sentence reports on a prediction that compares
the likelihood of drivers exceeding a proposed new
speed limit with the likelihood of drivers exceeding
the current speed limit. The idiom as x as y, rather
than the incorrect form equally . . . as, should be
used to express the comparison.

729

GMAT® Official Guide 2018

A
B
C

D

E

Equally likely . . . as is not an idiomatic form
of comparison.
This also offers a nonidiomatic form of
comparison.
The comparison is expressed nonidiomatically.
Also, the drivers will be equally likely should
be followed by to exceed rather than by that
they will exceed. The resulting sentence is
wordy and structurally flawed.
The resulting sentence is wordy and
structurally flawed. The idiomatic phrase
as x as y is somewhat in use, but as likely
that they is awkward, and the comparison is
unclear and not parallel.
Correct. The idiomatic phrase as x as y is
properly used, and the comparison is clear
and parallel.

Logical predication; Idiom
The phrase at the beginning needs a subject
for written; most logically here it would
be the books mentioned written by Mary
Wollstonecraft; any other main‐clause subject
is therefore incorrect. The verb attribute
idiomatically requires the preposition to
followed by a noun phrase, not the infinitive
marker to followed by a verb.
A

B

C

The correct answer is E.
705. Written early in the French Revolution, Mary
Wollstonecraft’s A Vindication of the Rights of Man
(1790) and A Vindication of the Rights of Woman
(1792) attributed Europe’s social and political ills to be
the result of the dominance of aristocratic values and
patriarchal hereditary privilege.
(A)

(B)

Mary Wollstonecraft’s A Vindication of the Rights
of Man (1790) and A Vindication of the Rights
of Woman (1792) attributed Europe’s social and
political ills to result from

(C)

Mary Wollstonecraft’s A Vindication of the Rights
of Man (1790) and A Vindication of the Rights
of Woman (1792) attributed Europe’s social and
political ills to

(D)

(E)

730

Mary Wollstonecraft’s A Vindication of the Rights
of Man (1790) and A Vindication of the Rights
of Woman (1792) attributed Europe’s social and
political ills to be the result of

in A Vindication of the Rights of Man (1790)
and A Vindication of the Rights of Woman
(1792), Mary Wollstonecraft attributed
Europe’s social and political ills to have been
the result of
Mary Wollstonecraft, in A Vindication of the
Rights of Man (1790) and A Vindication of the
Rights of Woman (1792), attributed Europe’s
social and political ills to

D

E

The main subject is correctly predicated of
written, but attribute is incorrectly followed
by the infinitive to plus a verb (be).
The main subject is correctly predicated of
written, but attribute is incorrectly followed
by the infinitive to plus a verb (result).
Correct. The main subject is correctly
predicated of written, and attribute is
correctly followed by the preposition to.
The subject of the main clause is Mary
Wollstonecraft, and so this phrase is illogically
forced to be taken as the subject of written.
Also, attribute is incorrectly followed by to
plus a verb (have been).
The subject of the main clause is Mary
Wollstonecraft, and so this phrase is
illogically forced to be taken as the subject
of written.

The correct answer is C.
706. Using study groups managed by the principal popular
organizations and political parties, the Swedish public
was informed by the government about energy and
nuclear power.
(A)

the Swedish public was informed by the
government about energy and nuclear power

(B)

the government informed the Swedish public
about energy and nuclear power

(C)

energy and nuclear power information was given
to the Swedish public by the government

(D)

information about energy and nuclear power was
given to the Swedish public by the government

(E)

the public of Sweden was given energy and
nuclear power information by the government

9.8 Sentence Correction Answer Explanations

Logical predication; Rhetorical construction
This sentence tries to describe a situation in which
the government used study groups to inform the
Swedish public. Therefore, it is incorrect to use
the Swedish public as the subject of inform in this
sentence, because doing so in this case illogically
makes the Swedish public the subject of using as
well. Additionally, inform is a more concise and
direct way to express the idea in give information.
A

B

C

D

E

Using the Swedish public as the subject of the
main clause incorrectly makes it the subject
of using as well.
Correct. Using the government as the
main subject correctly allows it to count
as the subject of using; inform is a concise
phrasing for the main action of the sentence.
Energy and nuclear power information
does not work as the subject of the main
clause, since this also, illogically, makes it
the subject of using. Also, this phrase delays
the reader’s understanding of the important
noun information (a clearer phrasing is
information about energy and nuclear power)
and employs give information rather than
the more concise inform.
Energy and nuclear power information does
not work as the main clause subject, since
this also, illogically, makes it the subject of
using. In addition, this version awkwardly
uses give information to instead of the more
concise inform.
The public of Sweden is awkward compared
to the Swedish public, and in any case is
illogically taken as the subject of using;
given . . . information could be phrased more
concisely with inform.

The correct answer is B.
707. The use of the bar code, or Universal Product Code,
which was created in part to enable supermarkets to
process customers at a faster rate, has expanded
beyond supermarkets to other retail outlets and
have become readily accepted despite some initial
opposition when it was first introduced in 1974.
(A)

have become readily accepted despite some
initial opposition when it was first introduced
in 1974

(B)

has become readily accepted despite some
initial opposition when they were first introduced
in 1974

(C)

have become readily accepted despite some
initial opposition when first introduced in 1974

(D)

has become readily accepted despite some
initial opposition when the bar code was first
introduced in 1974

(E)

bar codes have become readily accepted
despite some initial opposition when it was first
introduced in 1974

Agreement; Rhetorical construction
The subject of this sentence is the use of the bar
code, the main noun of which is the singular use;
thus, the corresponding main verb should be in
the singular form has, not the plural have. The
actual subject for this verb is merely understood,
but when it is present, any pronoun that refers
back to it must agree with it in number.
A

B

C
D

E

The verb form have does not agree with
the sentence’s subject. The referent of it is
potentially unclear.
Has is correct; however, the rest of its clause
is badly worded, because its subject they
does not clearly refer back to the singular
Universal Product Code (or bar code); a better
choice is it.
The plural have does not agree with the
singular subject use.
Correct. The verb is in the correct form has,
and using bar code as the last clause’s subject
avoids an agreement problem.
Restating the subject as the plural bar codes
allows the following verb to be have, but it is
then incorrect to use it later in the sentence,
since it does not agree in number with bar
codes.

The correct answer is D.
708. The Organization of Petroleum Exporting Countries
(OPEC) had long been expected to announce a
reduction in output to bolster sagging oil prices, but
officials of the organization just recently announced
that the group will pare daily production by 1.5 million
barrels by the beginning of next year, but only if non‐
OPEC nations, including Norway, Mexico, and Russia,
were to trim output by a total of 500,000 barrels a day.
731

GMAT® Official Guide 2018

(A)

year, but only if non‐OPEC nations, including
Norway, Mexico, and Russia, were to trim output

(B)

year, but only if the output of non‐OPEC nations,
which includes Norway, Mexico, and Russia, is
trimmed

(A)

(C)

year only if the output of non‐OPEC nations,
including Norway, Mexico, and Russia, would be
trimmed

stockbrokers are helping many people who turn
to them to buy stocks that could be easily

(B)

year only if non‐OPEC nations, which includes
Norway, Mexico, and Russia, were trimming
output

stockbrokers are helping many people who are
turning to them for help in buying stocks that
they could easily have

(C)

many people are turning to stockbrokers for help
from them to buy stocks that could be easily

year only if non‐OPEC nations, including Norway,
Mexico, and Russia, trim output

(D)

many people are turning to stockbrokers for help
to buy stocks that easily could have been

(E)

many people are turning to stockbrokers for help
in buying stocks that could easily be

(D)

(E)

Rhetorical construction; Logical predication
The underlined part of this sentence deals with
the conditions under which OPEC members
will lower their own oil production by 1.5
million barrels by the beginning of next year. The
important thing to notice here is the following
logical relation: X will do something only if Y does
something else.
A

B

C

D

E

This version has redundant words, were to in
front of trim, that do not add anything more
in meaning. Furthermore, were to trim is
not the proper verb form to accompany will
pare. The addition of but before only is also
redundant.
This version uses the passive construction in
the conditional clause only if the output … is
trimmed. This use of the passive voice makes
this sentence vague; it is now unclear who
needs to trim the output of non‐OPEC
nations. Finally, the addition of but before
only is redundant.
As in (B), this version also introduces
vagueness by using the passive construction.
In addition, would in front of the passive
verb be trimmed is redundant.
This version uses an active verb, but in the
past progressive form, were trimming. The
progressive tense denotes actions in progress,
so its use is not normally warranted in
conditional sentences such as this one.
Correct. This version uses the correct and
most concise conditional structure, without
redundancies.

The correct answer is E.
732

709. Not trusting themselves to choose wisely among the
wide array of investment opportunities on the market,
stockbrokers are helping many people who turn to them
to buy stocks that could be easily bought directly.

Logical predication; Grammatical
construction; Verb form
This sentence is intended to be about people who,
because they do not trust themselves to make
wise investment decisions, turn to stockbrokers
for advice. As the sentence is worded, however,
it is stockbrokers who do not trust themselves to
choose wisely. The sentence is made even more
incomprehensible by the peculiar placement of the
adverbs in the phrase, could be easily bought directly.
A

B

C

D

This version of the sentence incorrectly
identifies the subject described by the opening
modifier as stockbrokers; the adverb easily is
misplaced in the phrase could be easily bought.
As in (A), the opening clause illogically
modifies stockbrokers rather than many people.
The tense of the verb phrase could easily
have bought does not match the tense of are
helping earlier in the main clause.
Although the opening modifier is correctly
attached to people rather than stockbrokers,
the sentence is unnecessarily wordy (for help
from them).
To buy is not idiomatic in this context—in
buying would be correct—and the tense
of the verb could have been does not match
the tense of the verb earlier in the clause;
the point is not that people are turning to
stockbrokers for help in buying stocks that
at some earlier time could have been bought
directly, but rather that the stocks could be
bought by the people directly at the very time
they are seeking help from the stockbrokers.

9.8 Sentence Correction Answer Explanations

E

Correct. The opening clause correctly
modifies many people, and the adverb is
correctly placed.

The correct answer is E.
710. There are several ways to build solid walls using just
mud or clay, but the most extensively used method
has been the forming of bricks out of mud or clay, and,
after some preliminary air drying or sun drying, they
are laid in the wall in mud mortar.
(A)

the forming of bricks out of mud or clay, and,
after some preliminary air drying or sun drying,
they are laid

(B)

forming the mud or clay into bricks, and, after
some preliminary air drying or sun drying, to lay
them

(C)

having bricks formed from mud or clay, and,
after some preliminary air drying or sun drying,
they were laid

(D)

to form the mud or clay into bricks, and, after
some preliminary air drying or sun drying, to lay
them

(E)

that bricks were formed from mud or clay, which,
after some preliminary air drying or sun drying,
were laid

Parallelism; Verb form
The purpose of the sentence is to describe the
historically most popular method of building
walls. The first clause announces this topic and
the second clause describes the particular method.
The clearest, most efficient way to accomplish
these two pieces of business is to use a parallel
structure. The ways to build in the first clause is
narrowed to the single way to form and to lay in
the second clause. There is no need to alternate
the verb phrases between active and passive voice
or to shift tenses.
A

B
C

The active gerund phrase the forming of
bricks does not fit with the passive verb
phrase that follows (they are laid).
The verb phrases forming the mud … and to
lay them are not parallel.
In addition to faulty parallelism between
having bricks formed and they were laid, the
tense in the second half of the sentence
unaccountably shifts from present to past.

D

E

Correct. The phrases to form and to lay in
the second clause are parallel to to build in
the first clause.
The relative clause beginning with which
apparently (but nonsensically) describes the
closest nouns, mud or clay, rather than bricks.

The correct answer is D.
711. A surge in new home sales and a drop in weekly
unemployment claims suggest that the economy might
not be as weak as some analysts previously thought.
(A)

claims suggest that the economy might not be
as weak as some analysts previously thought

(B)

claims suggests that the economy might not be
so weak as some analysts have previously thought

(C)

claims suggest that the economy might not be
as weak as have been previously thought by
some analysts

(D)

claims, suggesting about the economy that it
might not be so weak as previously thought by
some analysts

(E)

claims, suggesting the economy might not be as
weak as previously thought to be by some analysts

Agreement; Grammatical construction
The plural subject of this sentence (surge and
drop) requires a plural verb, suggest. The object of
this verb, the clause beginning with that, should
be presented in as clear and direct a manner as
possible.
A
B
C
D

E

Correct. The plural subject is matched with
a plural verb.
The singular verb suggests does not match
the plural subject of the sentence.
The sentence offers no plural subject to fit
the passive verb have been thought.
This construction is awkward, wordy, and
imprecise; it also lacks a main verb; there is
no reason to use passive voice, and suggesting
about the economy that it might … introduces
extra words that contribute nothing to the
meaning of this sentence fragment.
The passive construction makes this
unnecessarily wordy; the lack of a main verb
makes this a sentence fragment.

The correct answer is A.
733

GMAT® Official Guide 2018

712. Retail sales rose 0.8 of 1 percent in August,
intensifying expectations that personal spending in the
July–September quarter more than doubled that of the
1.4 percent growth rate in personal spending for the
previous quarter.
(A)
(B)

(B)

with none of the restrictions of passenger trains,
railroad timetables, nor

(C)

without the restrictions of passenger trains and
railroad timetables nor

(D)

that personal spending in the July–September
quarter more than doubled that of

without the restrictions of passenger trains and
railroad timetables or with the

(E)

that personal spending in the July–September
quarter would more than double

without the restrictions of passenger trains and
railroad timetables or the

Rhetorical construction; Diction

(C)

of personal spending in the July–September
quarter, that it more than doubled

(D)

of personal spending in the July–September
quarter more than doubling that of

(E)

of personal spending in the July–September
quarter, that it would more than double that of

Verb form; Logical predication
The sentence explains the expectations that resulted
from a past retail sales trend. Since expectations
look to the future but are not yet realized, the
relative clause explaining these expectations should
be conditional, employing the auxiliary verb would.
A

B

C

D
E

The simple past‐tense verb form does
not express the forward‐looking sense of
expectations.
Correct. By using the verb would double,
this concise sentence indicates that the
expectation has not yet been realized.
This construction is awkward, announcing the
topic (personal spending) and then elaborating
in a relative clause that restates this topic as it.
Although this option is not technically
wrong, it is less clear and graceful than (B).
Like (C), this sentence is awkward and
unnecessarily wordy, announcing the topic
and then using an additional clause to
elaborate on it.

The correct answer is B.
713. In the early part of the twentieth century, many
vacationers found that driving automobiles and sleeping
in tents allowed them to enjoy nature close at hand and
tour at their own pace, with none of the restrictions of
passenger trains and railroad timetables or with the
formalities, expenses, and impersonality of hotels.
(A)
734

with none of the restrictions of passenger trains
and railroad timetables or with the

The sentence lays out some advantages that car
travel and tent camping were perceived to offer
over rail travel. The sentence attempts to describe
these advantages in terms of the absence of any of
a series of annoyances accompanying rail travel.
But the sentence fails because of the mismatch
between with none of . . . and or with. . . . One way
to successfully convey the intended meaning is to
use the preposition without governing all the items
in the series, expressed as nouns or noun phrases.
A

B
C

D

E

By using none of . . . to introduce the first
drawback of rail travel, and or to introduce
the rest of them, this sentence suggests that
the presence of drawbacks on the final list is
an alternative to the absence of restrictions.
With none . . . nor . . . is nonidiomatic (as
opposed to neither . . . nor . . .).
The structure without . . . nor . . . in the way
used here is nonidiomatic. The negative
without governs the whole list of drawbacks
at the end of the sentence.
The coupling of without and or with is
confusing, suggesting, as in (A), that the
drawbacks on the final list are an alternative
to the absence of restrictions.
Correct. The sentence is unambiguous and
constructed in a way that without clearly
distributes over all the items in the series.

The correct answer is E.
714. Over the next few years, increasing demands on the
Chattahoochee River, which flows into the Apalachicola
River, could alter the saline content of Apalachicola
Bay, which would rob the oysters there of their flavor,
and to make them decrease in size, less distinctive,
and less in demand.
(A)

which would rob the oysters there of their flavor,
and to make them decrease in size,

9.8 Sentence Correction Answer Explanations

(B)

and it would rob the oysters there of their flavor,
make them smaller,

(B)

also Women’s Work, a more general account of
cloth manufacture, is an expert authority about

(C)

and rob the oysters there of their flavor, making
them decrease in size,

(C)

of Women’s Work, a more general account about
early cloth manufacture, is an authority on

(D)

robbing the oysters there of their flavor and
making them smaller,

(D)

(E)

robbing the oysters there of their flavor, and
making them decrease in size,

of Women’s Work, a more general account about
early cloth manufacture, is an expert authority
about

(E)

Women’s Work, a more general account of early
cloth manufacture, is an authority on

The sentence claims that demands for river water may
change the saline content of the bay, possibly altering
the flavor and size of oysters there and diminishing the
oysters’ marketability. The sentence is not parallel. It
should read which would rob the oysters . . . and make
them decrease to be parallel and grammatical. The series
of three phrases after make them that describes what will
happen to the oysters also needs to be parallel.

A

B

C

D

E

The referent of the relative pronoun, which,
is ambiguous; the two effects of altered
saline content are not expressed in parallel
form, with a relative clause expressing
the first effect, and an infinitive phrase
expressing the second.
The referent of the pronoun it is ambiguous.
Also, the effects of the bay’s altered saline
content are not expressed in parallel form—
the first being an independent clause and
the second a verb phrase.
The comma before the conjunction and
signals that an independent clause will
follow and, but a verb phrase follows
instead. The series of phrases following
making them lacks appropriate parallelism.
Correct. The potential effects on the oysters
are expressed by two parallel participial
phrases, the second of which lists three
adjectives correctly in a series.
The series of phrases following making them
lacks appropriate parallelism.

The correct answer is D.
715. Elizabeth Barber, the author of both Prehistoric Textiles,
a comprehensive work on cloth in the early cultures
of the Mediterranean, and also of Women’s Work, a
more general account of early cloth manufacture, is an
expert authority on textiles in ancient societies.
(A)

Rhetorical construction; Idiom; Parallelism
Using a lot of parenthetical elements, this
sentence communicates the main idea that
Elizabeth Barber is an authority on textiles in
ancient societies. It is the main rhetorical goal
of the sentence to position the parenthetical
elements so that they do not obscure the main
idea. The parenthetical descriptions need to be
streamlined enough to be informative, but not too
long. In addition, several versions repeat of before
the title Women’s Work and doing so makes the
sentence unparallel; the first of comes before both
and so should distribute over both clauses.
A

B

C

D

The use of also of before Women’s Work is
redundant and unparallel. It is sufficient to
connect the two book titles like this: both X
and Y. The meanings of the two nouns expert
and authority largely overlap, so there is no
need to modify one with the other.
The use of also before Women’s Work is
redundant. It is sufficient to connect the
two book titles like this: both X and Y. The
meanings of the two nouns expert and
authority largely overlap, so there is no need to
modify one with the other. Finally, the noun
authority takes the preposition on, not about.
As in (A), repeating the preposition of
before Women’s Work makes the sentence
unparallel. The noun account takes the
preposition of, not about.
As in (A) and (C), repeating the preposition
of before Women’s Work makes the sentence
unparallel. The noun account takes the
preposition of, not about. It is redundant to
modify authority with expert because they
express the same idea. Finally, the noun
authority takes the preposition on, not about.

also of Women’s Work, a more general account of
early cloth manufacture, is an expert authority on
735

GMAT® Official Guide 2018

E

Correct. This version is parallel, uses the
most concise structure of the parenthetical
descriptions, eschews the redundant
modification of authority, and employs the
correct prepositions.

B

C

The correct answer is E.
716. Digging in sediments in northern China, evidence
has been gathered by scientists suggesting that
complex life‐forms emerged much earlier than they had
previously thought.
(A)

evidence has been gathered by scientists
suggesting that complex life‐forms emerged
much earlier than they had

(B)

evidence gathered by scientists suggests a
much earlier emergence of complex life‐forms
than had been

(C)

scientists have gathered evidence suggesting
that complex life‐forms emerged much earlier
than

(D)

scientists have gathered evidence that suggests
a much earlier emergence of complex life‐forms
than that which was

(E)

scientists have gathered evidence which
suggests a much earlier emergence of complex
life‐forms than that

D

E

The correct answer is C.
717. Employing many different techniques throughout his
career, Michelangelo produced a great variety of art
works, including paintings, for example, in the Sistine
Chapel, to sculpture, for example, the statue of David.
(A)

including paintings, for example, in the Sistine
Chapel, to sculpture, for example,

(B)

including paintings, for example, in the Sistine
Chapel, to sculpture, like

(C)

including paintings, such as those in the Sistine
Chapel, and sculpture, as

(D)

ranging from paintings, such as those in the
Sistine Chapel, to sculpture, such as

(E)

ranging from paintings, such as in the Sistine
Chapel, and sculpture, such as

Logical predication; Modification
In principle, the relationship described in the first
part of the underlined portion could be expressed
with scientists as the subject (scientists gathered
evidence) or with evidence as the subject (evidence
was gathered by scientists). The latter construction
could be effective in some contexts, but here its
relationship to the rest of the sentence appears to
commit the writer to the claim that the evidence
was digging in China.
A

736

This version has a dangling participle,
digging. … Digging in sediments in northern
China must modify scientists, not evidence.
The passive structure of the main clause also
creates an inadvisable distance between the
words evidence and suggesting. Furthermore,
the dependent clause starting with
suggesting may be construed with either the
evidence or the scientists, which makes this
version unnecessarily ambiguous.

This version has a dangling participle,
digging. … Digging in sediments in northern
China must modify scientists, not evidence.
Correct. Choosing scientists as the subject
of gathered, this version corrects the
dangling participle. It also uses a parallel
active form of the verb emerge, and does not
use redundant material.
In this context it would be preferable to use
a verb (emerged). The phrasing used here
(suggests a much earlier emergence of ) sounds
more stilted and is less clear and direct.
In addition, inserting that which before
previously thought is not only redundant but
incorrect English.
The problems described in (D) above are
also in evidence here.

Parallelism; Rhetorical construction
This sentence names painting and sculpture
as two of the many kinds of art created by
Michelangelo and provides examples of his work
in those two art forms. Although the two sets
of examples are expressed in parallel form, the
position of the phrase for example that introduces
them creates a choppy and awkward sentence
since it must be surrounded by commas. A more
concise way to construct parallel sets of examples
is to express each using a phrase introduced by
such as.

9.8 Sentence Correction Answer Explanations

A

The sentence is choppy and awkward due
to the twofold use of the phrase for example,
which must be surrounded by commas. The
preposition to, in context, suggests a range,
but it would then be needed to be preceded
by from.
B
The examples of art are not in parallel form
since the first is introduced by for example
followed by the prepositional phrase in the
Sistine Chapel, and the second is introduced
by the prepositional phrase like. . . The
preposition to is unidiomatic and awkwardly
used here.
C The examples of art are not in parallel form
since the first is introduced by such as, and
the second is introduced by as.
D Correct. The examples of art are in parallel
form, each introduced with the words such
as.
E Although the examples of art are both
introduced by such as, the form is not
parallel since the first is a prepositional
phrase (in the Sistine Chapel) and the second
is a noun phrase (the statue of David).
The correct answer is D.
718. Outlining his strategy for nursing the troubled
conglomerate back to health, the chief executive’s
plans were announced on Wednesday for cutting the
company’s huge debt by selling nearly $12 billion in
assets over the next 18 months.
(A)

executive’s plans were announced on Wednesday
for cutting the company’s huge debt by selling
nearly $12 billion in assets over the next
18 months

(B)

executive’s plans, which are to cut the company’s
huge debt by selling nearly $12 billion in assets
over the next 18 months, were announced on
Wednesday

(C)

(D)

(E)

executive’s plans for cutting the company’s huge
debt by selling nearly $12 billion in assets over the
next 18 months were announced on Wednesday

Logical predication; Verb form
In this sentence, the opening dependent clause
beginning Outlining his strategy is a dangling
modifier. Furthermore, the verb form for
announce should make it clear that the chief
executive is doing the announcing. In addition,
to cut is a clearer phrase than for cutting in this
sentence.
A

B

C

D

E

The subject of the opening clause should be
executive, not plans. The passive verb form
were announced suggests that someone other
than the chief executive is outlining the
strategy. Putting the phrase were announced
between plans and for cutting makes it
somewhat unclear whether for cutting … is
intended to modify announced or plans.
In addition to having a dangling modifier
and the wrong form of the verb announce,
this sentence is made less clear by separating
the subject and verb with the long clause
beginning with which are.
This version has the same issues as in (A)
and (B) and is made less clear by separating
the subject and verb with the long clause
beginning with for cutting.
Correct. The opening clause properly
modifies chief executive and the verb form
announced makes it clear that the chief
executive is doing the announcing.
Although the opening clause correctly
modifies chief executive in this version,
the words that are are extraneous and also
suggest that the plans themselves are doing
the cutting and selling.

The correct answer is D.
719. It is called a sea, but the landlocked Caspian is actually
the largest lake on Earth, which covers more than four
times the surface area of its closest rival in size, North
America’s Lake Superior.
(A)

executive announced plans Wednesday to cut the
company’s huge debt by selling nearly $12 billion
in assets over the next 18 months

It is called a sea, but the landlocked Caspian is
actually the largest lake on Earth, which covers

(B)

executive announced plans Wednesday that are
to cut the company’s huge debt by selling nearly
$12 billion in assets over the next 18 months

Although it is called a sea, actually the
landlocked Caspian is the largest lake on Earth,
which covers

(C)

Though called a sea, the landlocked Caspian is
actually the largest lake on Earth, covering
737

GMAT® Official Guide 2018

(D)

Though called a sea but it actually is the largest
lake on Earth, the landlocked Caspian covers

(E)

Despite being called a sea, the largest lake
on Earth is actually the landlocked Caspian,
covering

(B)

Neuroscientists, having amassed a wealth of
knowledge about the brain and its development
from birth to adulthood over the past twenty
years, and are

(C)

Neuroscientists amassing a wealth of knowledge
about the brain and its development from birth to
adulthood over the past twenty years, and are

(D)

Neuroscientists have amassed a wealth of
knowledge over the past twenty years about
the brain and its development from birth to
adulthood,

(E)

Neuroscientists have amassed, over the past
twenty years, a wealth of knowledge about
the brain and its development from birth to
adulthood,

Logical predication; Grammatical
construction
The topic of this sentence is a single large body
of water, the Caspian Sea. The wording needs to
make it clear that being the largest lake on Earth and
covering more than four times the surface area of …
Lake Superior are both predicated of this one subject.
A

B

C
D
E

The referent of which is unclear.
Grammatically, its antecedent cannot be
the landlocked Caspian, so it must be either
Earth or the largest lake on Earth. The latter
is a little odd, because the sentence has
already said that the lake in question is the
Caspian, so one would expect and instead of
which. For these reasons and because Earth
immediately precedes which, the sentence
appears to say, illogically, that Earth covers
more than four times the surface area of
Lake Superior.
As in (A), this appears to say, illogically,
that Earth covers more than four times the
surface area of Lake Superior.
Correct. The wording is direct,
unambiguous, and grammatically correct.
The structure here is grammatically incoherent.
Despite being called a sea indicates, somewhat
illogically, that the largest lake being called a
sea would lead one to expect it not to be the
Caspian. This makes little sense, especially
to those who are familiar with the name
Caspian Sea.

Grammatical construction; Logical
predication
This sentence introduces the subject
(Neuroscientists), pauses to explain what
neuroscientists have accomplished in the past
twenty years, and then concludes by explaining
what neuroscientists are presently doing as
a result of their past accomplishments. The
second part of the sentence—the explanation—
interrupts the flow of the sentence from the
subject (Neuroscientists) to the predicate (are
now drawing solid conclusions …); it should
therefore be bracketed by commas. The sentence
construction should provide a main verb for the
subject neuroscientists.
A

B

The correct answer is C.
720. Neuroscientists, having amassed a wealth of
knowledge over the past twenty years about the brain
and its development from birth to adulthood, are now
drawing solid conclusions about how the human brain
grows and how babies acquire language.
(A)

738

Neuroscientists, having amassed a wealth of
knowledge over the past twenty years about
the brain and its development from birth to
adulthood, are

C

Correct. The explanatory phrase between
the subject and predicate is set off by
commas, and the main clause contains
both a subject (Neuroscientists) and a
corresponding verb (are now drawing).
And are indicates that are follows a previous
verb, but in fact the sentence has not yet
provided a first main verb for the subject
Neuroscientists; the sentence is therefore
incomplete; over the … years appears to be
modifying adulthood.
Amassing, like having amassed, functions
as an adjective, not a verb; the sentence
therefore lacks the first main verb implied
by the compound verb construction and are
now drawing. …

9.8 Sentence Correction Answer Explanations

D

E

The final descriptor in present tense, now
drawing conclusions … does not fit the
opening clause, which is in present‐perfect
tense (have amassed a wealth …) and seems
to modify adulthood.
Like (D), this sentence attempts to attach a
present‐tense descriptor to a present‐perfect
clause.

The correct answer is A.
721. According to a recent study of consumer spending on
prescription medications, increases in the sales of the
50 drugs that were advertised most heavily accounts
for almost half of the $20.8 billion increase in drug
spending last year, the remainder of which came
from sales of the 9,850 prescription medicines that
companies did not advertise or advertised very little.
(A)

heavily accounts for almost half of the $20.8
billion increase in drug spending last year, the
remainder of which came

(B)

heavily were what accounted for almost half of
the $20.8 billion increase in drug spending last
year; the remainder of the increase coming

(C)

heavily accounted for almost half of the $20.8
billion increase in drug spending last year, the
remainder of the increase coming

(D)

heavily, accounting for almost half of the $20.8
billion increase in drug spending last year, while
the remainder of the increase came

(E)

heavily, which accounted for almost half of the
$20.8 billion increase in drug spending last year,
with the remainder of it coming

Grammatical construction; Verb form
The sentence indicates that according to research,
increases in sales of the relatively small number
of the most heavily advertised drugs accounted
for nearly half of last year’s total increase in drug
spending. The sentence is flawed because of
subject‐verb disagreement, and an ambiguity in
the referent of which.
A

The singular verb form accounts fails to agree
in number with the plural subject increases.
What the relative pronoun which refers to is
unclear; to make clear sense, it should refer
to the . . . increase.

B

C

D
E

The phrase were what accounted is
unnecessarily wordy; the semicolon before
the remainder signals that a complete
clause will follow, but what follows is not a
complete clause.
Correct. The sentence is clear and
grammatically correct. The subject and verb
agree.
In the resulting sentence, no main verb
follows the main subject increases.
The resulting sentence lacks a main verb for
the main subject increases.

The correct answer is C.
722. Along the major rivers that traverse the deserts of
northeast Africa, the Middle East, and northwest India,
the combination of a reliable supply of water and good
growing conditions both encouraged farming traditions
that, in places, endure in at least 6,000 years.
(A)

good growing conditions both encouraged
farming traditions that, in places, endure in

(B)

good growing conditions encouraged farming
traditions that have, in places, endured for

(C)

of good growing conditions have encouraged
farming traditions that, in places, endured for

(D)

of good growing conditions both encouraged
farming traditions that have, in places, endured

(E)

of good growing conditions encouraged farming
traditions that have, in places, been enduring for

Logical predication; Rhetorical construction
The time line of this sentence, captured by the
use of verb tenses, is of utmost importance. A
combination of factors (in the past) encouraged
farming traditions that are still with us today. The
conditions for the use of the present perfect tense
have endured are in place.
A

B

The word both repeats the meaning of
combination and is thus redundant. The use
of the present tense (endure) is not justified
by the time line of the whole sentence.
The correct preposition for this type of
construction is for (an amount of time), not in.
Correct. This version correctly employs the
present perfect tense with the appropriate
adverbial for at least 6,000 years.

739

GMAT® Official Guide 2018

C

D

E

The repetition of the preposition of before
good growing conditions makes no sense.
It seems to indicate that there is both a
combination of a reliable supply of water and
a combination of good growing conditions.
The preposition of should not be repeated in
front of good growing conditions. The word
both repeats the meaning of combination and
is thus redundant.
The preposition of should not be repeated
in front of good growing conditions. The use
of the present perfect progressive have been
enduring is not grammatically incorrect, but
it is rhetorically inappropriate and sounds
exaggerated.

B

C
D

E

The correct answer is B.
723. Despite its covering the entire planet, Earth has a
crust that is not seamless or stationary, rather it is
fragmented into mobile semirigid plates.
(A)

Despite its covering the entire planet, Earth has
a crust that is not seamless or stationary, rather
it is

(B)

Despite the fact that it covers the entire planet,
Earth’s crust is neither seamless nor is it
stationary, but is

The correct answer is D.
724. Emily Dickinson’s letters to Susan Huntington Dickinson
were written over a period beginning a few years
before Susan’s marriage to Emily’s brother and ending
shortly before Emily’s death in 1886, outnumbering her
letters to anyone else.
(A)

Dickinson were written over a period beginning
a few years before Susan’s marriage to Emily’s
brother and ending shortly before Emily’s death
in 1886, outnumbering

(B)

Dickinson were written over a period that begins
a few years before Susan’s marriage to Emily’s
brother and ended shortly before Emily’s death in
1886, outnumber

Idiom; Parallelism; Logical predication

(C)

A dangling modifier is an error in sentence
structure whereby a participle is associated with
a word other than the one intended or with no
particular word at all. In this sentence, Earth is
the closest word to the participial clause, and so
the latter means that Earth is covering the entire
planet (itself ), which is a contradiction.

Dickinson, written over a period beginning a few
years before Susan’s marriage to Emily’s brother
and that ends shortly before Emily’s death in
1886 and outnumbering

(D)

Dickinson, which were written over a period
beginning a few years before Susan’s marriage
to Emily’s brother, ending shortly before Emily’s
death in 1886, and outnumbering

A

(E)

Dickinson, which were written over a period
beginning a few years before Susan’s marriage
to Emily’s brother and ending shortly before
Emily’s death in 1886, outnumber

(C)

Despite covering the entire planet, Earth’s crust
is neither seamless nor is it stationary, but rather

(D)

Although it covers the entire planet, Earth’s crust
is neither seamless nor stationary, but rather

(E)

740

Parallel structure is disrupted by the
addition of is it after nor and by the addition
of is after but.
Parallel structure is disrupted by the
addition of is it after nor.
Correct. Despite and although are very close
in meaning. However, despite is a preposition
and needs to be followed by a noun or noun
phrase, while although is a conjunction and
should be followed by a finite clause. This
version uses although correctly. The parallel
structure is also clear and correct.
Although is a conjunction and should be
followed by a finite clause with a subject, not
by a participle. In addition, the first clause
represents a dangling modifier. Omitting
rather from the parallel structure neither X
nor Y but rather Z is possible, but not optimal.

Although covering the entire planet, Earth has a
crust that is not seamless or stationary, but

This version has a dangling participle.
The addition of it is before fragmented
is unwarranted and makes the sentence
ungrammatical. Neither … nor … but rather
would make the intended relationship among
seamless, stationary, and fragmented clearer
and more precise than not … or … rather.

9.8 Sentence Correction Answer Explanations

Parallelism; Grammatical construction
The main point of the sentence is that
Dickinson’s letters to her sister‐in‐law outnumber
her letters to anyone else. To emphasize this
point, outnumber should be the main verb, and the
description introduced by the passive verb were
written needs to be changed from a main clause
to an adjectival phrase.
A

B
C

D
E

The long, wordy opening clause gives
too much emphasis to the period when
Dickinson’s letters were written; it is unclear
what outnumbering refers to.
The verbs describing the letter‐writing
period (begins and ended) are not parallel.
The verbs describing the letter‐writing
period need to be in parallel form and agree
in tense—e.g., beginning and ending or that
began and that ended; this is a fragment
because it lacks a main verb for letters.
The lack of a main verb for the subject of
the sentence, letters, makes this a fragment.
Correct. The information about the period
when Dickinson’s letters were written is
contained in an adjectival phrase set off by
commas, and the main verb outnumber refers
clearly to letters.

(E)

Rhetorical construction; Grammatical
construction
Every clause needs a subject, either an overt
subject or an understood subject (whose
interpretation can come from a coordinated clause
or some other nearby clause). In this sentence, the
clause containing expected lacks a clear subject.
The intended subject is motion picture industry
representatives, but to clearly indicate that, the
subject should either be repeated or be replaced
with the pronoun they. Furthermore, piracy to
increase with high‐speed Internet connections that
become more widely available is awkward, and it
fails to clearly communicate the idea that piracy
will increase as a result of high‐speed Internet
connections becoming available.
A
B
C
D

The correct answer is E.
725. At the end of 2001, motion picture industry
representatives said that there were about a million
copies of Hollywood movies available online and
expected piracy to increase with high‐speed Internet
connections that become more widely available.
(A)

online and expected piracy to increase with high‐
speed Internet connections that become more
widely available

(B)

online and expect the increase of piracy with
the wider availability of high‐speed Internet
connections

(C)

(D)

online, and they expect more piracy to increase
with the wider availability of high‐speed Internet
connections
online, and that they expected the increase of
piracy as high‐speed Internet connections would
become more widely available

online, and that they expected piracy to increase
as high‐speed Internet connections became
more widely available

E

The second clause is awkward and unclear;
there is no clear subject for expected.
There is no clear subject for expect.
It is redundant to use both more and increase.
This wording makes the meaning very
unclear. They expected the increase in
piracy appears to refer to a particular
(past) increase, but this does not clearly
make sense with the ensuing use of the
conditional verb form would become, which
is inappropriate here.
Correct. In this version the verb expect
has an overt subject, and the following
phrasing clearly indicates that the expected
increase in piracy is the result of high‐speed
Internet connections becoming more widely
available.

The correct answer is E.
726. Making things even more difficult has been general
market inactivity lately, if not paralysis, which has
provided little in the way of pricing guidance.
(A)

has been general market inactivity lately, if not
paralysis, which has provided

(B)

there is general market inactivity, if not paralysis,
lately it has provided

(C)

general market inactivity, if not paralysis, has
lately provided
741

GMAT® Official Guide 2018

(D)

lately, general market inactivity, if not paralysis,
has provided

(E)

is that lately general market inactivity, if not
paralysis, which provides

Grammatical construction; Rhetorical
construction
This sentence uses a special inverted structure,
putting the predicate (Making things even more
difficult) before the verb (has been) and the
subject. In this construction, the subject (general
market inactivity) can be directly compared with
paralysis. Such contrasts are best made using
phrases that are adjacent, not separated by other
material (here, the adverb lately). If a more
normal clause structure is used, making things
even more difficult becomes a modifier, not the
main predicate, so it should be clearly set off from
the rest of the clause with a comma.
A

B

C

D

E

This inverted structure makes general market
activity the subject of Making things even
more difficult. This would be legitimate
by itself, but it requires general market
inactivity to be next to both if not paralysis
(for contrast) and which (marking a relative
clause modifying inactivity). It cannot be
next to both of these simultaneously.
This is a run‐on sentence, with two
independent clauses (Making things
even more difficult there is general market
inactivity, if not paralysis, and lately it has
provided little in the way of pricing guidance)
conjoined merely by a comma, rather than
by a coordinating conjunction, such as and.
Also, the initial topic phrase (Making things
even more difficult) is awkward without a
following comma.
The initial topic phrase (Making things
even more difficult) should be followed by a
comma.
Correct. The topic phrase (Making things
even more difficult) is properly separated
from the subject by a comma, and inactivity
and if not paralysis are adjacent for the
clearest connection between them.
This is a sentence fragment.

The correct answer is D.

742

727. Unlike the conviction held by many of her colleagues
that genes were relatively simple and static, Barbara
McClintock adhered to her own more complicated
ideas about how genes might operate, and in 1983,
at the age of 81, was awarded a Nobel Prize for her
discovery that the genes in corn are capable of moving
from one chromosomal site to another.
(A)

Unlike the conviction held by many of her
colleagues that genes were

(B)

Although many of her colleagues were of the
conviction of genes being

(C)

Contrary to many of her colleagues being
convinced that genes were

(D)

Even though many of her colleagues were
convinced that genes were

(E)

Even with many of her colleagues convinced of
genes being

Rhetorical construction; Idiom; Logical
predication
The sentence compares a widely held conviction
about genes with McClintock’s adherence to her
own ideas, then goes on to describe McClintock’s
accomplishments. The sentence must not compare
widespread convictions with McClintock herself.
The clearest and most efficient way to make
the comparison is to introduce McClintock’s
colleagues’ convictions in a dependent clause,
followed by a main clause that introduces
McClintock’s different way of doing things and
goes on to explain how successful she was.
A
B
C
D

E

Incorrect comparison between conviction
and Barbara McClintock.
Were of the conviction of genes being relatively
simple is wordy and awkward.
Contrary to many of her colleagues being
convinced is wordy and awkward.
Correct. A dependent clause describing
the beliefs of McClintock’s colleagues is
followed by the main clause presenting the
contrasting beliefs of McClintock.
Even with many of her colleagues … is wordy
and indirect.

The correct answer is D.

9.8 Sentence Correction Answer Explanations

728. Ryu– nosuke Akutagawa’s knowledge of the literatures of
Europe, China, and that of Japan were instrumental in
his development as a writer, informing his literary style
as much as the content of his fiction.
(A)

that of Japan were instrumental in his development
as a writer, informing his literary style as much as

(B)

that of Japan was instrumental in his
development as a writer, and it informed both his
literary style as well as

(C)

Japan was instrumental in his development as a
writer, informing both his literary style and

(D)

Japan was instrumental in his development as a
writer, as it informed his literary style as much as

(E)

Japan were instrumental in his development
as a writer, informing both his literary style in
addition to

E

The correct answer is C.
729. According to scientists who monitored its path,
an expanding cloud of energized particles ejected
from the Sun recently triggered a large storm in the
magnetic field that surrounds Earth, which brightened
the Northern Lights and also possibly knocking out a
communications satellite.
(A)

an expanding cloud of energized particles
ejected from the Sun recently triggered a large
storm in the magnetic field that surrounds Earth,
which brightened the Northern Lights and also
possibly knocking

(B)

an expanding cloud of energized particles
ejected from the Sun was what recently
triggered a large storm in the magnetic field that
surrounds Earth, and it brightened the Northern
Lights and also possibly knocked

(C)

an expanding cloud of energized particles
ejected from the Sun recently triggered a large
storm in the magnetic field that surrounds Earth,
brightening the Northern Lights and possibly
knocking

(D)

a large storm in the magnetic field that surrounds
Earth, recently triggered by an expanding cloud
of energized particles, brightened the Northern
Lights and it possibly knocked

(E)

a large storm in the magnetic field surrounding
Earth was recently triggered by an expanding
cloud of energized particles, brightening the
Northern Lights and it possibly knocked

Logical predication; Agreement
When a verb follows a complex noun phrase made
up of several parts, it agrees with the first noun in
the phrase. In this case, knowledge of the literatures
of Europe, China, and Japan is a singular noun and
the correct verb form is was, not were. The various
parts of an enumeration have to be alike: the
literatures of Europe, China, and Japan. The logical
relationship between the predicates is important.
A

B

C

D

This version of the sentence violates the
correct subject‐verb agreement, and the
correct structure of enumeration is disrupted
by the addition of that of in front of Japan.
The correct structure of enumeration is
disrupted by the addition of that of in front of
Japan. Both … as well as … is incorrect usage.
Correct. The structure of the enumeration
(Europe, China, and Japan) as well as the
conjunction structure (both X and Y) are
correct. The logical relationships among the
parts of the sentence are clearly expressed.
This phrasing makes it unclear what the
writer is claiming. It appears to indicate that
the effect of Akutagawa’s knowledge on his
development as a writer was due to the fact
that both of the aspects of his writing were
influenced to the same extent. However, it is
implausible to suppose that this is what the
writer intends. Furthermore, the comparison
is ambiguous: did his knowledge inform his
style as much as it informed the content,

or did it inform his style as much as the
content informed his style?
The subject‐verb agreement in this version is
incorrect. Both X in addition to Y is incorrect
usage.

Logical predication; Rhetorical
construction; Verb form
The timing and logical relationships among the
events described in this sentence are of utmost
importance. The scientists monitored a cloud
ejected from the Sun. The cloud triggered a large
storm, whose consequences were the brightening
of the Northern Lights and the possible knocking
out of a satellite. The latter two events are in a
conjunction, so they should be represented by
similar verb forms.
743

GMAT® Official Guide 2018

A

B

C

D

E

In this context, the shift in verb form from
which brightened to and also possibly knocking
is ungrammatical. The two verbs should be in
the same verb form for parallel construction.
X was what triggered Y is wordy and
awkward, and its meaning is unclear in
this context. Given the most plausible
intended meaning of the sentence, the two
conjunctions and … and … in the last clause
are redundant. The comma after Earth
turns the final part of the sentence into an
independent clause, and it is unclear whether
this is part of what the scientists claimed or a
separate claim made by the writer.
Correct. The conjoined elements are of
parallel forms, and the logical relations
between the events are clear and concisely
communicated.
The wording in this answer choice makes
the intended meaning unclear. The
information that the cloud particles were
ejected from the Sun is lost. The sentence is
ungrammatical; the second conjoined main
verb, knocked, needs no pronoun subject it
because its subject is a large storm.
The wording in this answer choice
makes the intended meaning unclear.
The information that the cloud particles
were ejected from the Sun is lost. The
two conjoined verbs are of different form;
the second conjoined verb includes an
unnecessary pronoun subject.

The correct answer is C.
730. In 1850, Lucretia Mott published her Discourse on
Women, arguing in a treatise for women to have equal
political and legal rights and for changes in the married
women’s property laws.
(A)

744

arguing in a treatise for women to have equal
political and legal rights

(B)

arguing in a treatise for equal political and legal
rights for women

(C)

a treatise that advocates women’s equal political
and legal rights

(D)

a treatise advocating women’s equal political and
legal rights

(E)

a treatise that argued for equal political and legal
rights for women

Parallelism; Rhetorical construction
Mott’s Discourse was a treatise, and it is redundant
and confusing to present her as both publishing
her Discourse and arguing in a treatise, as though
they were two separate things. The verb arguing
must be followed by a prepositional phrase
beginning with for, but the verb advocating simply
takes a direct object.
A
B
C

D

E

After published her Discourse … arguing in a
treatise is wordy and imprecise.
Arguing in a treatise is redundant and awkward.
The verb advocates does not work
idiomatically with the prepositional phrase
for changes. …
The verbal advocating does not work
idiomatically with the prepositional phrase
for changes. …
Correct. The title of Mott’s publication is
followed by a phrase describing the treatise,
and argued is followed by for.

The correct answer is E.
731. To develop more accurate population forecasts,
demographers have to know a great deal more than now
about the social and economic determinants of fertility.
(A)

have to know a great deal more than now about
the social and economic

(B)

have to know a great deal more than they do
now about the social and economical

(C)

would have to know a great deal more than they
do now about the social and economical

(D)

would have to know a great deal more than they
do now about the social and economic

(E)

would have to know a great deal more than now
about the social and economic

Verb form; Logical predication
This sentence explains a hypothetical situation
and therefore calls for a conditional—or contrary‐
to‐fact—construction, because in order to more
accurately predict population, demographers
would have to know more than they presently
know. A present‐tense verb is required to describe
the current state of demographers’ knowledge, and
the comparison made by the sentence must be
between current and conditional knowledge, not
between knowledge and time of knowing (now).

9.8 Sentence Correction Answer Explanations

A

B
C
D

E

Wrong comparison—between knowledge
and time (now); conditional verb is
needed.
Conditional verb is needed; economical is the
wrong adjective.
Economical is the wrong adjective.
Correct. Conditional knowledge, indicated
by would have to know, is correctly compared
to current knowledge.
Wrong comparison—between what
demographers need to know and now.

The correct answer is D.
732. Laos has a land area about the same as Great Britain
but only four million in population, where many are
members of hill tribes ensconced in the virtually
inaccessible mountain valleys of the north.
(A)

about the same as Great Britain but only four
million in population, where many

(B)

of about the same size as Great Britain is, but
in Laos there is a population of only four million,
and many

(C)

that is about the same size as Great Britain’s
land area, but in Laos with a population of only
four million people, many of them

(D)

comparable to the size of Great Britain, but only
four million in population, and many

(E)

comparable to that of Great Britain but a
population of only four million people, many of
whom

A

B

C
D

E

The correct answer is E.
733. Having been named for a mythological nymph who
cared for the infant Jupiter, the asteroid named Ida, in
the middle of the belt of asteroids that orbit the Sun
between Mars and Jupiter, was discovered in 1884.
(A)

Having been named for a mythological nymph
who cared for the infant Jupiter, the asteroid
named Ida, in the middle of the belt of asteroids
that orbit the Sun between Mars and Jupiter, was
discovered in 1884.

(B)

Discovered in 1884, the asteroid Ida, named for
a mythological nymph who cared for the infant
Jupiter, is in the middle of the belt of asteroids
that orbit the Sun between Mars and Jupiter.

(C)

In the middle of the belt of asteroids that orbit
the Sun between Mars and Jupiter, the asteroid
Ida, discovered in 1884 and named for a
mythological nymph who cared for the infant
Jupiter.

(D)

The asteroid Ida, named for a mythological
nymph who cared for the infant Jupiter and
discovered in 1884, is in the middle of the belt
of asteroids to orbit the Sun between Mars and
Jupiter.

(E)

Ida, an asteroid discovered in 1884 and which
was named for a mythological nymph who cared
for the infant Jupiter, is in the middle of the belt
of asteroids to orbit the Sun between Mars and
Jupiter

Logical predication; Grammatical
construction
The comparison in this sentence is between the
land area of Laos and the land area of Great
Britain, not between the land area of Laos and
Great Britain. The phrase about the population
of Laos is most clearly and efficiently expressed
in an appositive using a relative pronoun
to refer back to people rather than the more
abstract population. Using this construction
keeps the appropriate emphasis on the two
main claims being made about Laos, one
describing its land area and the other its sparse
population.

The comparison between land area and
Great Britain is incorrect; where is an
inappropriate referent to population, which
does not designate a place.
Inappropriate comparison between land
area and Great Britain; the there is …
construction is wordy and imprecise.
The reference of them is unclear and the
expression is generally awkward.
The coordinating conjunction and gives
undue emphasis to the claim that many of
the people in Laos live in inaccessible places.
Correct. The land area of Laos is correctly
compared to that of Great Britain; whom
refers appropriately to people.

745

GMAT® Official Guide 2018

Rhetorical construction; Logical
predication; Grammatical construction
This sentence describes a discovery that
occurred in 1884 and provides some additional
information about the object that was discovered.
The most effectively worded answer choice opens
with a past‐participial phrase (discovered . . .)
describing the subject of the sentence, the asteroid
Ida. Ida’s discovery is logically prior to its naming,
described in a second past, following the subject
(named . . .). The sentence is then completed with
a present tense linking verb is + prepositional
phrase to explain Ida’s location.
A

B
C

D

E

Opening with a past perfect passive verb,
Having been named, this version of the
sentence illogically suggests that being
named for a mythological nymph preceded
the discovery of Ida.
Correct. This version is clear, logically
coherent, and grammatically correct.
This version of the sentence is
ungrammatical; it has no main verb for the
subject the asteroid Ida.
The sequence of events is obscured by the
placement of named before discovered in the
compound participial phrases. The infinitive
form to orbit is ungrammatical in place of
the relative clause.
This sentence awkwardly attempts to use a
compound conjunction and to join the past
participial phrase discovered in 1884 with
the relative clause which was named. . . . The
infinitive form to orbit is ungrammatical in
place of the relative clause.

The correct answer is B.
734. In ancient Thailand, much of the local artisans’ creative
energy was expended for the creation of Buddha
images and when they constructed and decorated the
temples that enshrined them.

746

(A)

much of the local artisans’ creative energy was
expended for the creation of Buddha images
and when they constructed and decorated the
temples that enshrined them

(B)

much of the local artisans’ creative energy was
expended on the creation of Buddha images and
on construction and decoration of the temples in
which they were enshrined

(C)

much of the local artisans’ creative energy was
expended on the creation of Buddha images
as well as constructing and decoration of the
temples in which they were enshrined

(D)

creating images of Buddha accounted for much
of the local artisans’ creative energy, and also
constructing and decorating the temples enshrining
them

(E)

the creation of Buddha images accounted for
much of the local artisans’ creative energy
as well as construction and decoration of the
temples that enshrined them

Idiom; Parallelism; Rhetorical construction
The main point of the sentence is that artisans
in ancient Thailand spent most of their creative
energy on three tasks: creating Buddha images,
and constructing and decorating temples to
enshrine the Buddhas. These three tasks must
be described in parallel forms. The verb expended
should be followed by the preposition on, not for.
A

B

C

D

E

For is the wrong preposition; the and when
… clause introduces faulty parallelism; it is
unclear what they refers to.
Correct. The three activities are presented
in parallel form: creation of Buddha images
and construction and decoration of the temples.
Constructing violates the parallelism
otherwise maintained by creation and
decoration.
The and also phrase is awkwardly set apart
from the main claim of the sentence—
which is that all three tasks consumed much
of the artisans’ energy.
This construction is awkward and
unnecessarily wordy and says something
different—that the images accounted for the
construction and decoration of temples.

The correct answer is B.
735. The English physician Edward Jenner found that if
experimental subjects were deliberately infected with
cowpox, which caused only a mild illness, they are
immune from smallpox.
(A)

which caused only a mild illness, they are
immune from

(B)

causing only a mild illness, they become immune
from

9.8 Sentence Correction Answer Explanations

(C)

which causes only a mild illness, they are
immune to

(D)

causing only a mild illness, they became immune
from

(E)

which caused only a mild illness, they would
become immune to

(A)

Covering 71 percent of Earth’s surface, the
oceans play an essential role in maintaining
the conditions for human existence on land,
moderating

(B)

Covering 71 percent of Earth’s surface and
playing an essential role in maintaining the
conditions for human existence on land, the
oceans moderate

(C)

The oceans cover 71 percent of Earth’s surface
and play an essential role in maintaining conditions
for human existence on land, and by moderating

(D)

The oceans cover 71 percent of Earth’s
surface, play an essential role in maintaining
the conditions for human existence on land, and
moderate

(E)

The oceans cover 71 percent of Earth’s surface,
playing an essential role in maintaining the
conditions for human existence on land, and they
moderate

Verb form
This sentence describes the result of infecting
volunteers with cowpox. A conditional verb
form is used to describe the cowpox infection: if
experimental subjects were deliberately infected. The
sentence then incorrectly uses simple present tense
for the effects of that infection: they are immune.
However, since the effects are dependent on an
action that may or may not occur, the correct
way to express those effects is by using another
conditional verb form: they would become immune.
A

The effects of a conditional situation are
incorrectly expressed using the simple
present tense verb are.
B
The participle causing suggests that infecting
the subjects with cowpox caused a mild
illness only in some of the cases—but this
does not seem to be the intended meaning.
The effects of a conditional situation are
incorrectly expressed using the simple present
tense verb become. The preposition from is
incorrect with immune; it should be to.
C The effects of a conditional situation are
incorrectly expressed using the simple
present tense verb are.
D See above the comment on causing in (B).
The effects of a conditional situation are
incorrectly expressed using the simple past
tense verb became. The preposition with
immune should be to.
E Correct. The effects of a conditional situation
are correctly expressed using the conditional
verb would become. The preposition to (rather
than from) is correct with immune.
The correct answer is E.
736. Covering 71 percent of Earth’s surface, the oceans
play an essential role in maintaining the conditions for
human existence on land, moderating temperature
by the absorption of heat and carbon dioxide, and
giving pure water back to the atmosphere through
evaporation.

Parallelism; Rhetorical construction;
Grammatical construction
This sentence about the essential role oceans play
in maintaining conditions for human existence on
land begins with a participial phrase describing
the vastness of the oceans, followed by the
main clause (oceans play an essential role . . .). The
object, role, is modified by a prepositional phrase
that indicates what kind of role the oceans play
(it plays a role in maintaining the conditions for
human existence on land). This statement about the
ocean’s role in maintaining terrestrial conditions
is elucidated by two parallel participial phrases
that describe the oceans, explaining how they
maintain conditions essential for human existence
on land (moderating . . . and giving . . .).
A

B

C

Correct. This version of the sentence
effectively conveys the means by which the
ocean plays an essential role in maintaining
the conditions for human existence on land,
using parallel verb forms.
This version of the sentence is ungrammatical;
and giving . . . suggests that this is part of a
series of participial phrases, but it is not.
The conjunction followed by a prepositional
phrase and by moderating . . . anticipates a
new subject—an agent that does something
by moderating—but this subject never
appears.
747

GMAT® Official Guide 2018

D

E

This version of the sentence is structured as a
series of verbs—cover, play, and moderate—all
describing things the oceans do; the final
participial phrase giving . . . water . . . violates
the parallel structure set up in the sentence.
This version of the sentence correctly makes
playing an essential role . . . a function of
the oceans’ coverage of Earth’s surface,
but the introduction of a new main verb
moderate fails to indicate that what follows
(moderating temperature and returning
pure water) identifies the role oceans play in
maintaining conditions for human existence
on land. Furthermore, the participial phrase
and giving pure water . . . violates the parallel
structure set up by the series of main verbs
that appear earlier in this version of the
sentence.

A

B
C

D

The correct answer is A.
737. Some anthropologists believe that the genetic
homogeneity evident in the world’s people is the result
of a “population bottleneck”—at some time in the past
our ancestors suffered an event, greatly reducing their
numbers and thus our genetic variation.
(A)

at some time in the past our ancestors suffered
an event, greatly reducing their numbers

(B)

that at some time in the past our ancestors
suffered an event that greatly reduced their
numbers

(C)

that some time in the past our ancestors
suffered an event so that their numbers were
greatly reduced,

(D)

some time in the past our ancestors suffered
an event from which their numbers were greatly
reduced

(E)

some time in the past, that our ancestors
suffered an event so as to reduce their numbers
greatly,

Grammatical construction; Parallelism
The underlined part of this sentence is an
explanatory rewording of the clause that follows
believe. Scientists believe that X—[in other words,]
that Y. In this construction, X and Y are parallel
clauses.

748

E

The omission of that after the dash makes
the function of the final clause unclear.
The structure makes that clause appear to
be an awkward and rhetorically puzzling
separate assertion that the writer has
appended to the prior claim about what the
anthropologists believe. The agent or cause
of reducing is unclear.
Correct. Repetition of that effectively
signals the paraphrasing of the belief.
The preposition at before some time is
missing; without at the adverb sometime
would be needed instead of this two‐word
noun phrase. The modifier of event is
expressed with a wordy passive construction,
which destroys the parallelism between it
and what follows.
Repetition of that signals the paraphrasing
of the belief and is therefore needed. The
preposition at before some time is missing.
The modifier of event is expressed with a
wordy passive construction, which destroys
the parallelism between it and what
follows.
That is repeated in the paraphrase, but in
the wrong place. A possible, and absurd,
reading of this version is that our ancestors
suffered an event in order to willfully
reduce their own numbers and thus our
genetic variation.

The correct answer is B.
738. Foraging at all times of the day and night, but
interspersing their feeding with periods of rest that last
between one and eight hours, a sperm whale could eat
so much as a ton of squid a day.
(A)

between one and eight hours, a sperm whale
could eat so

(B)

between one and eight hours, sperm whales can
eat as

(C)

between one to eight hours, sperm whales could
eat as

(D)

from one to eight hours, sperm whales could eat
so

(E)

from one to eight hours, a sperm whale can
eat so

9.8 Sentence Correction Answer Explanations

Agreement; Diction
Although this sentence, as presented, uses the
conditional or past verb form could, it is more
plausibly intended to make a general statement
about the actual behavior of a species, a statement
that holds in the present day. For that purpose,
the present indicative can is preferable. So much
as is not the correct wording to express the upper
level of a variable amount; as much as should
be used instead. Also, although the singular a
sperm whale can be used to refer to sperm whales
generally, the plural their needs to refer to the
plural sperm whales.
A

B
C

D
E

This sentence incorrectly uses so. Also, the
plural their does not agree with the singular
sperm whale.
Correct. Both can and as are used; also,
sperm whales agrees with the plural their.
To is the wrong word to use with between.
The proper construction would be between . . .
and or from . . . to.
This sentence incorrectly uses so.
So is not the correct form; also the use of the
singular a sperm whale does not agree with
the plural their.

The correct answer is B.
739. In some types of pine tree, a thick layer of needles
protects the buds from which new growth proceeds;
consequently they are able to withstand forest fires
relatively well.
(A)

a thick layer of needles protects the buds from
which new growth proceeds; consequently they
are able to withstand forest fires relatively well

(B)

a thick needle layer protects buds from where
new growth proceeds, so that they can withstand
forest fires relatively well

(C)

a thick layer of needles protect the buds from
which new growth proceeds; thus, they are able
to withstand relatively well any forest fires

(D)

(E)

since the buds from which new growth
proceeds are protected by a thick needle layer,
consequently they can therefore withstand forest
fires relatively well
because the buds where new growth happens are
protected by a thick layer of needles, they are able
to withstand forest fires relatively easily as a result

Grammatical construction; Rhetorical
construction
This sentence is fine as written. It uses the
correct from which to introduce the relative clause
modifying buds and avoids redundant expressions
of causation, such as consequently . . . therefore, or
because . . . as a result.
A

B

C

D

E

Correct. The relative clause starting with
from which is in the correct form, and the
causality is expressed efficiently and clearly
with one word, consequently.
In this context, needle layer is less precise
than the more standard layer of needles,
which makes it clear that the layer is
composed of needles rather than being, for
example, a layer of a needle. From where is
not the correct form, because it is redundant
in using two words that express the idea of
location (from and where) instead of one.
The short direct object any forest fires is
separated from its verb withstand by an
adverb phrase; this word order is awkward,
and is acceptable only with very long direct
objects and in some cases where there is no
other reasonable way to eliminate ambiguity.
This version is unnecessarily redundant
in expressing causation, using all of since,
consequently, and therefore. As in answer
choice B, layer of needles would be more
precise than needle layer.
This version is unnecessarily redundant in
expressing causation, using both because and
as a result.

The correct answer is A.
740. The tourism commission has conducted surveys of
hotels in the most popular resorts, with the ultimate
goal of reducing the guests who end up expressing
overall dissatisfaction with the service in the hotels.
(A)

with the ultimate goal of reducing the guests who
end up expressing overall dissatisfaction with the
service in the hotels

(B)

with the goal to ultimately reduce the number
of guests who end up expressing overall
dissatisfaction with the hotels’ service

749

GMAT® Official Guide 2018

(C)

ultimately with the goal to reduce expressions
of overall dissatisfaction by the guests with the
hotel service

(D)

in an ultimate attempt to reduce the number
of guests that ends up expressing overall
dissatisfaction with the hotels’ service

(E)

with the ultimate goal of reducing the number of
guests who express overall dissatisfaction with
the hotels’ service

741. A new study suggests that the conversational pace
of everyday life may be so brisk it hampers the ability
of some children for distinguishing discrete sounds
and words and, the result is, to make sense of
speech.
(A)

it hampers the ability of some children for
distinguishing discrete sounds and words and,
the result is, to make

(B)

that it hampers the ability of some children to
distinguish discrete sounds and words and, as a
result, to make

(C)

that it hampers the ability of some children to
distinguish discrete sounds and words and, the
result of this, they are unable to make

(D)

that it hampers the ability of some children to
distinguish discrete sounds and words, and
results in not making

(E)

as to hamper the ability of some children for
distinguishing discrete sounds and words,
resulting in being unable to make

Verb form; Rhetorical construction
This sentence seems to be saying something
absurd: that the goal is to reduce the guests
themselves, instead of to reduce the number of
guests or the expressions of dissatisfaction. It is
also awkward in introducing the superfluous end
up; but at least, if it does so, the correct form to
agree with the subject guests is end up, not ends
up, which is used in answer choice D. In general,
direct modifiers (such as with the (hotel) service)
should not be separated from the word they
modify (such as dissatisfaction) if possible.
A

B
C

D

E

Reducing the guests is clearly not the intent
of the sentence; it should be phrased as
reducing the number of guests or the
expressions of dissatisfaction.
With the goal to ultimately reduce is awkward
and unidiomatic.
With the hotel service should be placed next
to dissatisfaction; also, ultimately with the
goal is awkward, better phrased as with the
ultimate goal.
The relative clause that ends up . . . modifies
the plural guests (not, in this context,
number), so the correct verb form is end up.
Correct. The with phrase is concise, and
it is the number of guests, not the guests
themselves, that is to be reduced. With the
hotel’s service is adjacent to dissatisfaction.
Also, in the relative clause starting with who,
the implicit subject of express is guests, so
this verb correctly agrees with its subject.

Rhetorical construction; Parallelism;
Diction
The sentence describes a hypothesized causal
series: The fast conversational pace impairs
children’s ability to distinguish individual sounds
and words, and this, in turn, impairs their ability
to make sense of speech. These two consequences,
both impaired abilities, are most clearly and
efficiently expressed in parallel infinitive phrases
(to distinguish and to make). The explanatory
phrase as a result before the second infinitive
clarifies the sequence. The term ability should be
followed by the preposition to, not for.
A

B

The correct answer is E.
C

750

For is the wrong preposition to follow
ability; the phrase and, the result is,
introduces a new clause which indicates
that children’s inability to distinguish
sounds enables them to make sense of
speech.
Correct. The two abilities hampered by the
fast pace of conversation are described with
the parallel infinitive phrases to distinguish
and to make.
The result of this is a new subject that
grammatically requires a new verb; the
phrase is wordy and unclear.

9.8 Sentence Correction Answer Explanations

D

E

This version of the sentence nonsensically
suggests that the pace of speech results in
not making sense of speech, removing the
children from the picture as the ones who
are affected.
The phrase is awkward, wordy, and unclear;
for is the incorrect preposition to follow ability.

The correct answer is B.
742. The nineteenth‐century chemist Humphry Davy
presented the results of his early experiments in his
“Essay on Heat and Light,” a critique of all chemistry
since Robert Boyle as well as a vision of a new
chemistry that Davy hoped to found.

743. To attract the most talented workers, some companies
are offering a wider range of benefits, letting
employees pick those most important to them.
(A)

benefits, letting employees pick those most
important to them

(B)

benefits, letting employees pick the most
important of them to themselves

(C)

benefits and letting employees pick the most
important to themselves

(D)

benefits and let employees pick the most
important to them

(E)

benefits and let employees pick those that are
most important to themselves

Diction; Parallelism; Verb form
(A)

a critique of all chemistry since Robert Boyle as
well as a vision of a

(B)

a critique of all chemistry following Robert Boyle
and also his envisioning of a

(C)

a critique of all chemistry after Robert Boyle and
envisioning as well

(D)

critiquing all chemistry from Robert Boyle
forward and also a vision of

(E)

critiquing all the chemistry done since Robert
Boyle as well as his own envisioning of

The sentence describes the benefit options offered
by some companies, which allow employees
to pick those most important to them. Letting
maintains the progressive sense of are offering;
those refers clearly and concisely to benefits; and
them is the correct pronoun to serve as the object
of the preposition to.
A

Parallelism; Rhetorical construction
The main objective of the sentence is to describe
“Essay on Heat and Light” as Davy’s presentation
of his own experiments and to further explain
that the essay served as both a critique of
previous chemistry and a vision of a new kind of
chemistry. The clearest, most effective form for
providing this explanation of the essay’s function
is to make critique and vision both appositives of
“Essay on Heat and Light,” and to present them
in a parallel structure.
A
B

C
D
E

Correct. The phrases describing the essay’s
function are presented in parallel form.
Critique and his envisioning are not parallel;
the phrase and also his envisioning is
unnecessarily wordy; it is also unclear to
whom his refers.
The two descriptors are not parallel.
The two descriptors are not parallel.
The meaning is confused in the assertion that
Davy critiqued his own vision of chemistry.

B

C

D

Correct. The sentence clearly and concisely
explains benefit options that allow
employees to pick those most important to
them.
The most important of them to themselves
is wordy, and the function of themselves
is unclear. Normally, themselves would be
either reflexive or emphatic, but in this
case it cannot reasonably be taken in either
of those ways. This nonstandard use of
the pronoun makes it unclear whether to
themselves is supposed to modify pick or most
important of them.
The pronoun themselves is used incorrectly,
and its intended function is unclear.
Normally, themselves would be either
reflexive or emphatic, but in this case it
cannot reasonably be taken in either of
those ways. This nonstandard use of the
pronoun makes it unclear whether to
themselves is supposed to modify pick or most
important of them.
The present tense verb let incorrectly shifts
tense from the present progressive are
offering.

The correct answer is A.
751

GMAT® Official Guide 2018

The present tense verb let incorrectly
shifts tense from the present progressive
are offering; the function of themselves is
unclear. Normally, themselves would be
either reflexive or emphatic, but in this
case it cannot reasonably be taken in either
of those ways. This nonstandard use of
the pronoun makes it unclear whether to
themselves is supposed to modify pick or most
important of them.
The correct answer is A.

E

744. Many of the earliest known images of Hindu deities
in India date from the time of the Kushan Empire,
fashioned either from the spotted sandstone of
Mathura or Gandharan grey schist.

D
E

As in (A) and (B), the placement of the
modifier after Empire is misleading;
parallelism requires that the phrase fashioned
from, or another comparable verb and
preposition, follow or.
Parallelism requires that a verb follow or,
since a verb follows either.
Correct. Two verbs, date and were fashioned,
introduce parallel predicates for the subject,
earliest known images; the choices of media
are correctly presented with the structure
either from … or from.

The correct answer is E.
745. Tides typically range from three to six feet, but while
some places show no tides at all, some others, such
as the Bay of Fundy, have tides of at least thirty feet
and more.

(A)

Empire, fashioned either from the spotted
sandstone of Mathura or

(B)

Empire, fashioned from either the spotted
sandstone of Mathura or from

(A)

(C)

Empire, either fashioned from the spotted
sandstone of Mathura or

some others, such as the Bay of Fundy, have
tides of at least thirty feet and more

(B)

(D)

Empire and either fashioned from the spotted
sandstone of Mathura or from

the others, such as the Bay of Fundy, that have
tides of more than thirty feet

(C)

(E)

Empire and were fashioned either from the
spotted sandstone of Mathura or from

others, such as the Bay of Fundy, have tides of
more than thirty feet

(D)

those at the Bay of Fundy, which has tides of
more than thirty feet

(E)

the ones at the Bay of Fundy have tides of at
least thirty feet and more

Logical predication; Parallelism
The sentence makes two claims about the earliest
known images of Hindu deities in India: They
date from the Kushan Empire, and they are
made from sandstone or schist. The clearest, most
effective way to incorporate these two claims
into a single sentence is to provide two parallel
predicates for the single subject, the earliest known
images of Hindu deities in India. The two options
of media, presented as either/or choices, must
also be given in parallel structure: either from … or
from … or from either … or. …
A

B

752

C

Placement of the modifier fashioned …
suggests that the Empire (the closest noun),
not the images of the deities, was fashioned
out of these materials; to parallel either
from, the preposition from should also
follow or.
Parallelism requires that either precede the
first appearance of from or that the second
appearance of from be eliminated.

Idiom; Grammatical construction
This sentence defines typical tides and then draws
a contrast between locations with tides lower
than that norm and locations with tides higher
than the norm. The proper idiom for drawing this
contrast is some places and others—not some places
and some others as written. The height of tides
in places such as the Bay of Fundy is expressed
in a confusing manner since at least thirty feet
sets a lower limit on the height. This wording
is pointlessly redundant with the phrase and
more, which follows it. At least would normally
be used to indicate that the writer does not
know, or prefers not to say, whether the tides are
sometimes higher. And more rhetorically conflicts
with this by signaling a definite commitment
to the claim that they are (at least sometimes)
higher. A charitable reading suggests that tides of
more than thirty feet is the intended meaning.

9.8 Sentence Correction Answer Explanations

A

B

C

D

E

The sentence contains repetitive and
redundant wording—some places and some
others, and at least thirty feet and more.
The final clause is incomplete because that
introduces a subordinate clause, leaving the
subject others with no main verb.
Correct. A contrast is drawn between places
with low tides and places with high tides
using the expression some places and others,
and the height of the high tides is expressed
clearly and without redundancy.
The pronoun those, which refers to places,
does not make sense along with at the Bay
of Fundy, which names a single place; the
final clause is incomplete because which
introduces a subordinate clause, leaving the
subject those with no verb.
The word ones, which refers to places, does
not make sense along with at the Bay of
Fundy, which names a single place; at
least thirty feet and more is redundant and
confusing.

The correct answer is C.
746. A leading figure in the Scottish Enlightenment, Adam
Smith’s two major books are to democratic capitalism
what Marx’s Das Kapital is to socialism.
(A)

Adam Smith’s two major books are to
democratic capitalism what

(B)

Adam Smith’s two major books are to
democratic capitalism like

(C)

Adam Smith’s two major books are to
democratic capitalism just as

(D)

Adam Smith wrote two major books that are to
democratic capitalism similar to

(E)

Adam Smith wrote two major books that are to
democratic capitalism what

Idiom; Logical predication
A leading figure in the Scottish Enlightenment
describes Adam Smith, not his two books, so the
name of Adam Smith must immediately follow
the opening phrase. The comparison between
Smith’s books and Marx’s book is expressed as a
ratio, so the correct idiomatic expression is x is to
y what a is to b.
A

The opening phrase is a dangling modifier
because it describes Smith, not his books.

B

C

D
E

The opening phrase is a dangling modifier;
like is an incorrect word for making the
comparison.
The opening phrase is a dangling modifier;
just as is an incorrect term for the
comparison.
Similar to is an incorrect conclusion to the
comparison introduced by are to.
Correct. The opening phrase is followed by
the subject that it modifies, Adam Smith,
and the comparison of the two men’s work
is presented idiomatically.

The correct answer is E.
747. Researchers studying the brain scans of volunteers
who pondered ethical dilemmas have found that the
basis for making tough moral judgments is emotion,
not logic or analytical reasoning.
(A)

the brain scans of volunteers who pondered
ethical dilemmas have found that the basis for
making tough moral judgments is

(B)

the brain scans of volunteers who pondered
ethical dilemmas and found the basis to make
tough moral decisions to be

(C)

the brain scans of volunteers pondering ethical
dilemmas and found that the basis for making
tough moral decisions is

(D)

volunteers’ brain scans while pondering ethical
dilemmas have found the basis to make tough
moral judgments to be

(E)

volunteers’ brain scans while they pondered
ethical dilemmas have found that the basis for
making tough moral judgments is

Logical predication; Grammatical
construction
The sentence reports that researchers got volunteers
to ponder ethical dilemmas and make moral
judgments. Brain scans revealed that the volunteers’
judgments were based on emotion rather than
logical analysis. The main clause of this sentence
is Researchers . . . have found that . . .; embedded
within this sentence, the present participial phrase
studying . . . describes the researchers, the relative
clause who pondered . . . describes the volunteers,
and the object of the main verb appears as a noun
clause that the basis . . . is. . . .
753

GMAT® Official Guide 2018

A
B

C

D

E

Correct. The sentence is coherent and
grammatically correct.
The use of the conjunction and immediately
before found indicates that the past tense
verbs pondered and found both have
volunteers as subject, but this changes the
original sentence, making it a long noun
phrase rather than a complete sentence.
The conjunction and leaves the verb
found without a subject, and this changes
the original sentence into a sequence of
incoherently connected phrases rather than
a complete sentence.
The phrase the basis to make is unidiomatic,
a sufficient reason for rejecting this option.
The placement of the modifier while
pondering appears in a form parallel to
studying and means that the researchers,
not the volunteers, were pondering ethical
dilemmas. This does not make the sentence
incoherent, but creates a sentence that fails
to capture the meaning clearly intended in
the original sentence.
Because the word volunteers’ is a possessive
form, and functions adjectivally as a
modifier of brain scans, they must refer back
to researchers rather than to volunteers’. This
is not incorrect in itself, but, as with (D),
the resulting sentence fails to capture the
intended meaning of the original sentence.

The correct answer is A.
748. Rivaling the pyramids of Egypt or even the ancient
cities of the Maya as an achievement, the army of
terra‐cotta warriors created to protect Qin Shi Huang,
China’s first emperor, in his afterlife is more than 2,000
years old and took 700,000 artisans more than 36
years to complete.

754

(A)

the army of terra‐cotta warriors created to protect
Qin Shi Huang, China’s first emperor, in his afterlife
is more than 2,000 years old and took 700,000
artisans more than 36 years to complete

(B)

Qin Shi Huang, China’s first emperor, was
protected in his afterlife by an army of terracotta warriors that was created more than
2,000 years ago by 700,000 artisans who took
more than 36 years to complete it

(C)

it took 700,000 artisans more than 36 years to
create an army of terra‐cotta warriors more than
2,000 years ago that would protect Qin Shi Huang,
China’s first emperor, in his afterlife

(D)

more than 2,000 years ago, 700,000 artisans
worked more than 36 years to create an army
of terra‐cotta warriors to protect Qin Shi Huang,
China’s first emperor, in his afterlife

(E)

more than 36 years were needed to complete
the army of terra‐cotta warriors that 700,000
artisans created 2,000 years ago to protect
Qin Shi Huang, China’s first emperor, in his
afterlife

Logical predication; Rhetorical
construction
The opening modifier, Rivaling the pyramids
… describes the army of terra‐cotta warriors,
which must immediately follow the modifier.
The placement of the predicates that follow is
important; they must clarify two things about the
army of terra‐cotta warriors: how old it is and
how long it took to complete. The clearest and
most effective way to express these two assertions
is as parallel verb phrases, is more than 2,000 years
old and took … more than 36 years to complete.
A

B

C

D

E

Correct. The opening phrase correctly
modifies the subject, the army of terra‐cotta
warriors; the placement of modifiers and
predicates in the main clause makes the
meaning of the sentence clear.
Opening phrase is a dangling modifier
because it does not describe the subject
Qin Shi Huang; in addition, the sentence is
awkward and unclear.
Opening phrase is a dangling modifier
because it does not describe the subject it;
the sequence of information presented is
confusing and unclear.
Opening phrase is a dangling modifier
because it does not describe the subject
700,000 artisans.
Opening phrase is a dangling modifier
because it does not describe the subject
more than 36 years.

The correct answer is A.

9.8 Sentence Correction Answer Explanations

749. In California, a lack of genetic variation in the Argentine
ant has allowed the species to spread widely; due to
their being so genetically similar to one another, the
ants consider all their fellows to be a close relative and
thus do not engage in the kind of fierce intercolony
struggles that limits the spread of this species in its
native Argentina.

D

E

Correct. The clause with similar uses the
plural they and an explicit to one another, and
agreement is respected between ants and
fellows.
The plural fellows and singular a close relative
do not agree.

The correct answer is D.
(A)

(B)

(C)

(D)

(E)

due to their being so genetically similar to one
another, the ants consider all their fellows to be
a close relative and thus do not engage in the
kind of fierce intercolony struggles that limits
due to its being so genetically similar, the ant
considers all its fellows to be a close relative
and thus does not engage in the kind of fierce
intercolony struggles that limit
because it is so genetically similar, the ant
considers all its fellows to be close relatives
and thus does not engage in the kind of fierce
intercolony struggles that limits
because they are so genetically similar to one
another, the ants consider all their fellows to be
close relatives and thus do not engage in the
kind of fierce intercolony struggles that limit
because of being so genetically similar to one
another, the ants consider all their fellows to be
a close relative and thus do not engage in the
kind of fierce intercolony struggles that limits

Diction; Agreement
Words that express comparisons, such as similar,
require either a plural object, with an optional
expression of the entities being compared, or
a singular object, in which case this explicit
comparison is required. Thus its being so genetically
similar, without this explicit comparison, is
incorrect. Also, the two sides of the construction
consider . . . to be must agree in number (fellows . . .
close relatives, not fellows . . . a close relative).
A

B

C

Consider all their fellows to be a close relative
shows incorrect agreement, with plural
fellows and singular a close relative.
Its being so genetically similar is incorrect
because there is no explicit statement of what
the ant is similar to; also, the plural fellows
and singular a close relative do not agree.
It is so genetically similar is incorrect because
there is no explicit statement of what the
ant is similar to.

750. Next month, state wildlife officials are scheduled to
take over the job of increasing the wolf population in
the federally designated recovery area, the number
of which will however ultimately be dictated by the
number of prey in the area.
(A)

area, the number of which will however

(B)

area; the size of the population, however, will

(C)

area, however the number of wolves will

(D)

area; the number of which will, however,

(E)

area, when the size of the population will,
however,

Grammatical construction; Diction
The point of the sentence is that the ultimate
size of the wolf population will be determined
according to the number of prey in the area.
However, the phrase the number of which has
no referent since it cannot logically refer to the
noncount noun population or to the singular
wolf, which is used adjectivally here to modify
population. The idea can be expressed clearly by
making the size of the population the subject of a
new independent clause: the size of the population
will be dictated by the number of prey.
A

B

C

The number of which cannot logically refer
to the noncount noun population or to the
singular wolf, which is used adjectivally here
to modify population.
Correct. The idea is expressed clearly with an
independent clause: the size of the population
will be dictated by the number of prey.
However is intended to serve as a
conjunctive adverb between the two
independent clauses, but the punctuation of
the sentence creates confusion by suggesting
that however is modifying are scheduled. A
semicolon after area and a comma after
however would make the intended function
of however clear.
755

GMAT® Official Guide 2018

The number of which cannot logically refer
to the noncount noun population; the
semicolon creates confusion since it is not
followed by an independent clause.
E When illogically suggests that the size of
the population will be determined at the
moment wildlife officials take over the task.
This conflicts with the ensuing claim that
the determination will ultimately depend on
a long-term condition (the number of prey in
the area).
The correct answer is B.
D

751. About 5 million acres in the United States have been
invaded by leafy spurge, a herbaceous plant from
Eurasia with milky sap that gives mouth sores to cattle,
displacing grasses and other cattle food and rendering
rangeland worthless.
(A)

States have been invaded by leafy spurge, a
herbaceous plant from Eurasia with milky sap
that gives mouth sores to cattle, displacing
grasses and other cattle food and rendering

(B)

States have been invaded by leafy spurge, a
herbaceous plant from Eurasia, with milky sap,
that gives mouth sores to cattle and displaces
grasses and other cattle food, rendering

(C)

States have been invaded by leafy spurge, a
herbaceous plant from Eurasia having milky sap
that gives mouth sores to cattle and displacing
grasses and other cattle food, rendering

(D)

States, having been invaded by leafy spurge,
a herbaceous plant from Eurasia with milky
sap that gives mouth sores to cattle, displaces
grasses and other cattle food, and renders

(E)

States, having been invaded by leafy spurge, a
herbaceous plant from Eurasia that has milky
sap giving mouth sores to cattle and displacing
grasses and other cattle food, rendering

Logical predication; Grammatical
construction
The sentence explains that leafy spurge causes
mouth sores in cattle and also displaces other
plants eaten by cattle. However, the structure of
the sentence seems, illogically, to indicate that
displacing grasses modifies either the immediately
preceding phrase (that gives mouth sores to cattle)
or the main subject of the sentence (about 5
756

million acres in the United States). A clearer way
to express the effects of the leafy spurge invasion
is with a compound predicate in the subordinate
clause: that gives mouth sores . . . and displaces
grasses.
A

Displacing grasses appears illogically to
modify either about 5 million acres in the
United States or that gives mouth sores to
cattle.
B Correct. The effects of the leafy spurge
invasion are expressed clearly with a
compound predicate in the subordinate
clause: that gives mouth sores . . . and
displaces grasses. The parenthetical commas
around with milky sap make it clear that the
entire phrase that gives . . . and displaces
. . . rendering . . . is intended to modify a
herbaceous plant from Eurasia. Although
the sap may well be the means by which
the plant gives mouth sores to cattle, the
sentence can be well formed and meaningful
without making a definite commitment to
whether that is the case.
C Having and displacing should not be
expressed in parallel form since the first is a
permanent characteristic of leafy spurge and
the second refers to an effect of the plant’s
invasion.
D The subject of the sentence, 5 million acres is
not clearly paired with a verb. The structure
of the sentence suggests that 5 million acres
may be the intended subject of both displaces
and renders but it is illogical to say that
5 million acres displaces grasses and renders
rangeland worthless.
E The subject of the sentence, 5 million acres is
not clearly paired with a verb. The structure
of the sentence suggests that 5 million acres
may be the intended subject of both displaces
and renders but it is illogical to say that
5 million acres displaces grasses and renders
rangeland worthless.
The correct answer is B.
752. While it costs about the same to run nuclear plants as
other types of power plants, it is the fixed costs that
stem from building nuclear plants that makes it more
expensive for them to generate electricity.

9.8 Sentence Correction Answer Explanations

(A)

While it costs about the same to run nuclear
plants as other types of power plants, it is the
fixed costs that stem from building nuclear
plants that makes it more expensive for them to
generate electricity.

(B)

While the cost of running nuclear plants is about
the same as for other types of power plants,
the fixed costs that stem from building nuclear
plants make the electricity they generate more
expensive.

(C)

Even though it costs about the same to run
nuclear plants as for other types of power
plants, it is the fixed costs that stem from
building nuclear plants that makes the electricity
they generate more expensive.

(D)

It costs about the same to run nuclear plants
as for other types of power plants, whereas
the electricity they generate is more expensive,
stemming from the fixed costs of building
nuclear plants.

(E)

The cost of running nuclear plants is about the
same as other types of power plants, but the
electricity they generate is made more expensive
because of the fixed costs stemming from
building nuclear plants.

Agreement; Logical predication
The emphatic construction it is X that does Y
(as in the phrase it is Jane who knows the answer)
should be used only when there is a compelling
reason to emphasize the doer of the action. In
this sentence, the emphatic construction is used
without good reason.
A

B
C

This sentence uses the emphatic structure it
is … that without justification. The singular
verb makes violates the agreement within the
structure. The verb makes should agree with
the notional subject (the fixed costs), not with
the pronoun it.
Correct. This answer choice clearly and
succinctly compares the two types of costs.
In addition to using the more cumbersome
emphatic structure, this version violates
the agreement within the structure. The
verb should agree with the notional subject
(the fixed costs), not with the pronoun it.

D

E

The preposition for is redundant in
comparing the two objects of run. Since it is
not clear what stemming … refers to, this is a
dangling modifier.
The passive construction electricity … is
made more expensive because of … is wordy
and cumbersome. The preposition for is
necessary in the comparison of the costs.

The correct answer is B.
753. The 32 species that make up the dolphin family are
closely related to whales and in fact include the animal
known as the killer whale, which can grow to be 30
feet long and is famous for its aggressive hunting
pods.
(A)

include the animal known as the killer whale,
which can grow to be 30 feet long and is

(B)

include the animal known as the killer whale,
growing as big as 30 feet long and

(C)

include the animal known as the killer whale,
growing up to 30 feet long and being

(D)

includes the animal known as the killer whale,
which can grow as big as 30 feet long and is

(E)

includes the animal known as the killer whale,
which can grow to be 30 feet long and it is

Rhetorical construction; Agreement
The subject of the sentence is the 32 species that
make up the dolphin family, and the sentence
makes two claims about them: They are closely
related, and they include the killer whale. The
relative pronoun which restates the object of the
second verb, reintroducing the animal known as
the killer whale as the subject of a relative clause
followed by two parallel verbs: can grow and is
famous.
A

B

Correct. In this concise sentence, verbs
agree in number with their subjects and the
relative pronoun which indicates clearly that
the animal known as the killer whale is the
subject of the verbs in the dependent clause.
Changing the verb to the participial growing
introduces ambiguity, because it could
refer back to the subject of the sentence
(32 species).

757

GMAT® Official Guide 2018

C

D

E

The participial growing might refer to
the 32 species; the introduction of being is
unnecessarily wordy and adds nothing in
terms of meaning.
as big as is an idiomatically incorrect
expression of the comparison; the plural
verb form include is needed to match the
plural subject the 32 species.
It simply restates the subject of the previous
phrase, introducing more words but no
additional meaning; the singular verb form
includes should be the plural form include.

Rhetorical construction; Agreement;
Grammatical construction
This sentence, explaining interconnections among
a number of events, needs to be streamlined
as much as possible in order to become
understandable. To this end, unnecessary words
and structures should be eliminated. Prominent
among these are the relative clauses beginning
with that. Additionally, the subject of this
sentence is the plural trenches, which requires a
plural verb.
A

The correct answer is A.
754. The first trenches that were cut into a 500‐acre site
at Tell Hamoukar, Syria, have yielded strong evidence
for centrally administered complex societies in
northern regions of the Middle East that were arising
simultaneously with but independently of the more
celebrated city‐states of southern Mesopotamia, in
what is now southern Iraq.
(A)

(B)

that were cut into a 500‐acre site at Tell
Hamoukar, Syria, yields strong evidence that
centrally administered complex societies in
northern regions of the Middle East were arising
simultaneously with but also

C

D

E

The correct answer is E.
755. Companies are relying more and more on networked
computers for such critical tasks as inventory
management, electronic funds transfer, and electronic
data interchange, in which standard business
transactions are handled via computer rather than on
paper.

(C)

having been cut into a 500‐acre site at Tell
Hamoukar, Syria, have yielded strong evidence
that centrally administered complex societies in
northern regions of the Middle East were arising
simultaneously but

(D)

cut into a 500‐acre site at Tell Hamoukar, Syria,
yields strong evidence of centrally administered
complex societies in northern regions of the
Middle East arising simultaneously but also

(A)

in which standard business transactions are
handled via computer rather than on paper

(B)

where computers handle standard business
transactions rather than on paper

cut into a 500‐acre site at Tell Hamoukar, Syria,
have yielded strong evidence that centrally
administered complex societies in northern
regions of the Middle East arose simultaneously
with but

(C)

in which computers handle standard business
transactions instead of on paper

(D)

where standard business transactions are
handled, not with paper, but instead via
computer

(E)

in which standard business transactions are
being handled via computer, in place of on paper

(E)

758

that were cut into a 500‐acre site at Tell
Hamoukar, Syria, have yielded strong evidence
for centrally administered complex societies in
northern regions of the Middle East that were
arising simultaneously with but

B

That were cut … and that were arising …
are unnecessarily wordy and create an
unnecessarily complicated and confusing
sentence structure.
In addition to the unnecessarily wordy
relative clauses, the singular verb yields does
not agree with the plural subject trenches.
Having been cut … is unnecessarily wordy;
arising simultaneously must be followed by
the preposition with in order to make sense.
The singular verb yields does not agree with
the plural subject trenches; also adds no
meaning to the sentence.
Correct. Unnecessary clauses and phrases
are avoided, and the subject and verb of the
main clause agree in number.

9.8 Sentence Correction Answer Explanations

Idiom; Logical predication; Rhetorical
construction

(D)

appeared as equipped to face any obstacle the
environment could put in their paths,

The concluding comparison in this sentence
uses the idiom rather than, which requires
parallel structures. In this sentence the
prepositional phrase via computer parallels on
paper. Substituting where for in which creates a
nonstandard idiom.

(E)

appeared to have been equipped for facing any
obstacle the environment could put in their path,

A

B

C

D

E

Correct. This sentence uses standard
idiomatic constructions and avoids the
problems that are found in the other
versions.
Where is a nonstandard way to refer to a
noun that does not name a location. If
electronic data interchange were a location,
this version would entail the odd claim that
on paper is an alternative location at which
computers would be expected to process
information.
The comparison of the clause computers
handle . . . with the prepositional phrase on
paper illogically treats a location (on paper)
as an alternative to an activity (computers
handle).
As in (B), where is a nonstandard idiom.
The commas around not with paper appear
to make this phrase parenthetical; thus, it is
somewhat unclear what instead via computer
is contrasted with.
The pile of prepositions in the phrase in
place of on paper is unnecessarily confusing
and wordy.

The correct answer is A.
756. Combining enormous physical strength with higher
intelligence, the Neanderthals appear as equipped for
facing any obstacle the environment could put in their
path, but their relatively sudden disappearance during
the Paleolithic era indicates that an inability to adapt to
some environmental change led to their extinction.

Verb form; Diction
Because Neanderthals “disappeared,” the verb
describing their apparent abilities cannot be
present tense, so as equipped must be changed to to
have been equipped. The expression equipped to face
is clearer and more direct than equipped for facing.
A

B

C

D

E

As equipped indicates that Neanderthals
still appear this way; equipped should be
followed by an infinitive form instead of a
prepositional phrase.
Correct. The verb tense clearly indicates
that the current evidence is about
Neanderthals in the past.
As equipped does not indicate that
Neanderthals appeared this way in the past;
while individual Neanderthals may well
have followed different paths, this sentence
is about the single evolutionary path taken
by Neanderthals as a species.
Present‐tense appear is needed to parallel
present‐tense indicates and to reinforce that
this is current evidence about Neanderthals
in the past; as in (C), paths should be singular.
For facing is an incorrect substitution of a
prepositional phrase for an infinitive.

The correct answer is B.
757. To map Earth’s interior, geologists use a network of
seismometers to chart seismic waves that originate
in the earth’s crust and ricochet around its interior,
most rapidly traveling through cold, dense regions and
slower through hotter rocks.
(A)

interior, most rapidly traveling through cold,
dense regions and slower

(B)

interior, which travel most rapidly through cold,
dense regions, and more slowly

(A)

appear as equipped for facing any obstacle the
environment could put in their path,

(C)

(B)

appear to have been equipped to face any
obstacle the environment could put in their path,

interior, traveling most rapidly through cold,
dense regions and more slowly

(D)

appear as equipped to face any obstacle the
environment could put in their paths,

interior and most rapidly travel through cold,
dense regions, and slower

(E)

interior and that travel most rapidly through cold,
dense regions and slower

(C)

759

GMAT® Official Guide 2018

Grammatical construction; Parallelism
This sentence explains in detail an activity of
geologists (using seismometers to chart waves),
focusing primarily on the object, seismic waves.
A description of these waves is developed
in a relative clause (that originate . . . hotter
rocks) that contains a compound verb phrase
(originate . . . ricochet . . .). The action, ricochet,
is further described in a participial phrase in
which traveling . . . is then further described in
a comparison of travel speeds in cold and hot
regions of Earth’s crust.
A

B
C
D

E

The two expressions of comparison should
be parallel. Because most rapidly is placed
before the verb, it appears to modify the
entire ensuing phrase, including slower.
This and the contrast between the forms of
rapidly and slower make the comparisons
nonparallel. Some usage advisers consider
slower to be only an adjective. Although
slower is sometimes used as an adverb,
that usage would be more appropriate
with the parallel faster. The stark contrast
between this typically adjectival form and
the clearly adverbial ly form is somewhat
jarring.
The referent of the relative pronoun which is
unclear.
Correct. The modifiers are parallel and
correctly positioned in relation to the verb.
This version of the sentence offers travel as
a compound verb parallel with originate and
ricochet rather than as a description of how
the waves ricochet. It has the same problems
with parallelism as (A).
Adding a relative clause and that . . . makes
this sentence wordy and awkward.

The correct answer is C.
758. Prices at the producer level are only 1.3 percent
higher now than a year ago and are going down,
even though floods in the Midwest and drought in the
South are hurting crops and therefore raised corn and
soybean prices.
(A)

760

than a year ago and are going down, even
though floods in the Midwest and drought in the
South are hurting crops and therefore raised

(B)

than those of a year ago and are going down,
even though floods in the Midwest and drought in
the South are hurting crops and therefore raising

(C)

than a year ago and are going down, despite
floods in the Midwest and drought in the South,
and are hurting crops and therefore raising

(D)

as those of a year ago and are going down, even
though floods in the Midwest and drought in the
South hurt crops and therefore raise

(E)

as they were a year ago and are going down,
despite floods in the Midwest and drought in
the South, and are hurting crops and therefore
raising

Logical predication; Verb form
The sentence as written makes an illogical
comparison between prices at the producer level
and a time period (a year ago); surely the intended
comparison is between such prices now and
those of a year ago. The clause at the end of the
sentence states that flooding and a drought are
hurting crops, and as a result of this, they have
raised prices of certain crops. Are hurting is in
the present progressive tense, indicating an
ongoing process; raised is in the simple past tense,
indicating a completed action. It would be more
appropriate to use the present progressive tense
here as well, [are] raising.
A

B

C

D

The first part of this version of the sentence
illogically compares prices to a time; the
second part of the sentence indicates that
a completed action (raised . . . prices) results
from an ongoing present condition (floods . . .
and drought . . . are hurting crops).
Correct. This version of the sentence makes
an appropriate comparison (between prices
now and those of a year ago), and uses
tenses in an appropriate way.
Like (A), this version of the sentence illogically
compares prices to time past. Furthermore, in
this version, Prices is the subject not only for
the verb are but also for the verbs are hurting
and [are] raising, which makes no sense.
The comparative adjective higher requires
the comparative term than instead of as; the
tenses of the verbs in the latter half of the
sentence, hurt (simple past) and raise (simple
present), do not work together logically.

9.8 Sentence Correction Answer Explanations

E

Like (D), this version inappropriately uses
as instead of than with higher. Furthermore,
like (C), in this version [p]rices is the subject
not only for the verb are but also for the
verbs are hurting and [are] raising.

The correct answer is B.
759. Fossils of the arm of a sloth found in Puerto Rico in
1991, and dated at 34 million years old, made it the
earliest known mammal of the Greater Antilles Islands.
(A)

sloth found in Puerto Rico in 1991, and dated at
34 million years old, made it the earliest known
mammal of

(B)

sloth, that they found in Puerto Rico in 1991, has
been dated at 34 million years old, thus making
it the earliest mammal known on

(C)

sloth that was found in Puerto Rico in 1991, was
dated at 34 million years old, making this the
earliest known mammal of

(D)

sloth, found in Puerto Rico in 1991, have been
dated at 34 million years old, making the sloth
the earliest known mammal on

(E)

sloth which, found in Puerto Rico in 1991, was
dated at 34 million years old, made the sloth the
earliest known mammal of

Agreement; Logical predication
The subject of the sentence is the plural fossils, not
sloth, and therefore requires a plural verb. It therefore
does not have a singular antecedent. To clarify the
identification of the oldest known mammal, the
noun the sloth must be explicitly identified.
A

B

C

D

Because sloth is the object of a preposition
and not the subject of the sentence, there is
no reasonable antecedent for the pronoun it;
in this construction, the subject of made is
fossils, but it makes no sense to say that the
fossils made it the earliest known mammal.
The introduction of the mysterious they, a
pronoun without a reference, adds confusion
to this sentence; the singular verb does not
agree with the plural subject.
The relative clause that was … is wordy and
awkward; the singular verb does not agree
with the plural subject.
Correct. The plural verb agrees with its
plural subject, and the sloth is explicitly
identified as the earliest known mammal.

E

The singular verb does not agree with the
plural subject.

The correct answer is D.
760. Recently physicians have determined that stomach
ulcers are not caused by stress, alcohol, or rich foods,
but a bacterium that dwells in the mucous lining of the
stomach.
(A)

not caused by stress, alcohol, or rich foods, but

(B)

not caused by stress, alcohol, or rich foods, but
are by

(C)

caused not by stress, alcohol, or rich foods, but
by

(D)

caused not by stress, alcohol, and rich foods,
but

(E)

caused not by stress, alcohol, and rich foods,
but are by

Parallelism; Diction
The formula used in this sentence not this but that
requires parallel elements following not and but.
This means that not by stress, alcohol, or rich foods
must be balanced by but by a bacterium. … There
is no need to repeat the verb are caused, or even
the auxiliary verb are, because the verb precedes
the not by … but by … formula. The substitution
of the conjunction and for the conjunction or
changes the meaning of the sentence: Stress,
alcohol and rich foods identifies the combination
of these three factors as a suggested cause of
stomach ulcers, whereas stress, alcohol, or rich foods
offers three individual possibilities. There is no
way to tell which one of these is the intended
meaning of the sentence.
A
B
C
D
E

To preserve parallelism, but should be
followed by by.
There is no reason to repeat the auxiliary
verb are.
Correct. This sentence correctly uses the not
by … but by … formula.
To preserve parallelism, but should be
followed by by.
To preserve parallelism, but should be
followed by by.

The correct answer is C.

761

GMAT® Official Guide 2018

761. The eyes of the elephant seal adapt to darkness
more quickly than any other animal yet tested,
thus allowing it to hunt efficiently under the gloomy
conditions at its feeding depth of between 300 and
700 meters.

The subject is correct, but in the following
clause it has no clear referent.

The correct answer is C.
762. A mutual fund having billions of dollars in assets will
typically invest that money in hundreds of companies,
rarely holding more than one percent of the shares of
any particular corporation.

(A)

The eyes of the elephant seal adapt to darkness
more quickly than any other animal yet tested,
thus allowing it

(B)

The eyes of the elephant seal adapt to darkness
more quickly than does any other animal yet
tested, allowing them

(A)

companies, rarely holding more than one percent

(B)

The eyes of the elephant seal adapt to darkness
more quickly than do those of any other animal
yet tested, allowing it

companies, and it is rare to hold at least one
percent or more

(C)

companies and rarely do they hold more than
one percent

(D)

companies, so that they rarely hold more than
one percent

(E)

companies; rarely do they hold one percent or
more

(C)

(D)

Because they adapt to darkness more quickly
than any other animal yet tested, the eyes of the
elephant seal allow it

(E)

Because the eyes of the elephant seal adapt
to darkness more quickly than do those of any
other animal yet tested, it allows them

Logical predication; Agreement
Logically, the eyes of the elephant seal should
be contrasted with the eyes of other animals, not
with the animals themselves. The sentence must
make this comparison directly and precisely,
with each subject interpretable as the subject of
adapt. Given the correct subject (those of any other
animal yet tested), which is plural, any reference to
it must also be plural.
A
B

C

D

762

E

Any other animal yet tested is incorrectly set
up as the subject of adapt.
Any other animal yet tested is incorrectly
set up as the subject of adapt. The plural
pronoun them seems to refer to eyes.
Although there is a sense in which eyes can
hunt, it is more reasonable to suppose that,
in this context, the writer’s intention is to
mention how the eyes’ quick adaptation
allows the seal to hunt efficiently.
Correct. The subject of the comparative
phrase is correctly those of any other animal
yet tested, the plural verb do correctly
agrees with this subject (those), and the
singular pronoun it correctly agrees with its
antecedent (elephant seal ).
Any other animal yet tested is incorrectly set
up as the subject of adapt.

Agreement; Logical predication
The participial phrase starting with rarely holding
is predicated of the main subject a mutual fund.
It elaborates on the effect of the main clause
verb: since a mutual fund invests in hundreds of
companies, it rarely holds more than one percent
in any particular corporation.
A

B

C

D
E

Correct. The participle holding in the
embedded clause correctly refers to a mutual
fund. It also correctly expresses the cause‐and‐
effect relationship between investing in many
companies and holding little in each company.
The antecedent of it is rare to hold is not
clear. The use of it is rare instead of rarely
could be misleading.
The use of and between the clauses makes
them both main clauses. Thus, the cause‐
and‐effect relationship between investing
and holding is lost. The referent of they is
unclear. It makes no sense to suppose that it
refers to the hundreds of companies. Since it
presumably refers to a mutual fund, it should
be singular.
The pronoun they refers to a mutual fund
and thus should be singular.
The pronoun they refers to a mutual fund
and thus should be singular.

The correct answer is A.

9.8 Sentence Correction Answer Explanations

763. Positing an enormous volcanic explosion at the end
of the Permian period would explain the presence of a
buried crater, account for the presence of the element
iridium (originating deep within the earth), and the
presence of quartz having been shattered by highimpact shock waves.

E

The three features are not listed in parallel
form since the first and second are verb
phrases, while the third is a noun phrase; the
sentence illogically states that an explosion
would explain the element iridium, rather
than explain the presence of the element.

The correct answer is B.

(A)

account for the presence of the element iridium
(originating deep within the earth), and the
presence of quartz having been

(B)

of the element iridium (originating deep within the
earth), and of quartz

(C)

the element iridium (originating deep within the
earth), and explain the presence of quartz having
been

(D)

the presence of the element iridium (originating
deep within the earth), and explain the presence
of quartz

(A)

winning prestigious awards in both London as
well as Tokyo for his achievement at so young an
age, and he is hoping

(E)

explain the element iridium (originating deep
within the earth), and the presence of quartz

(B)

winning prestigious awards both in London and
Tokyo for his achievement at such a young age,
and hoping

(C)

having won prestigious awards both in London
and Tokyo for his achievement at so young an
age, hoping

(D)

winning prestigious awards in both London and
Tokyo for his achievement at such a young age,
and he hopes

(E)

having won prestigious awards both in London as
well as Tokyo for his achievement at so young an
age, and he hopes

764. The 19‐year‐old pianist and composer performed his
most recent work all over Europe, Asia, and North
America last year, winning prestigious awards in both
London as well as Tokyo for his achievement at so
young an age, and he is hoping to continue composing
now that he has returned to Chicago.

Parallelism; Rhetorical construction
The sentence indicates that a volcanic explosion
would explain the presence of three features,
but those features are not expressed using
parallel grammatical structures. The first two
items in the list are verb phrases that involve
needless repetition—explain the presence of and
account for the presence of—while the third is an
awkwardly worded noun phrase—the presence of
quartz having been shattered. The three features
can be identified more concisely with a list
of prepositional phrases following explain the
presence: of a buried crater, of the element iridium,
and of quartz.
A
B
C

D

The three features are not listed in parallel
form; the sentence is wordy and awkward.
Correct. The three features are identified
with parallel prepositional phrases.
The three features are not listed in parallel
form since the first and third are verb
phrases, while the second is a noun phrase;
the wording is awkward and needlessly
repetitive.
The three features are not listed in parallel
form since the first and third are verb
phrases, while the second is a noun phrase;
the wording is needlessly repetitive.

Idiom; Grammatical construction
This sentence is about the past accomplishments
and the future ambitions of a musician who
recently won awards on a world tour. In some of
the versions of the sentence, the phrase as well as
is redundant with the word both before London
and Tokyo. Idiomatically, the simple conjunction
and completes the phrase beginning with both.
A

B

The phrase as well as between London and
Tokyo is not idiomatic (the idiomatic formula
is both X and Y). The present progressive verb
is hoping is unnecessarily wordy.
Because hoping is parallel with winning,
it suggests that the hoping and winning
are contemporaneous, whereas in fact the
musician won his awards last year but is
now, in the present, upon his return, hoping
to continue composing.
763

GMAT® Official Guide 2018

C

D
E

The present‐perfect participial phrase
having won suggests that his winning
took place before his performance tour.
Furthermore, the use of hoping in this
version of the sentence suggests that hoping
is something the pianist did while on his
performance tour last year, but the final
phrase, now that he has returned to Chicago,
indicates this is not so.
Correct. This version of the sentence uses the
correct idiomatic formula (both X and Y).
The participial phrase having won . . .
inaccurately states that the musician won
his awards prior to his performance tour.
Furthermore, both in London as well as Tokyo
is unidiomatic, as indicated in the discussion
of (A) above, and unparallel (in should
either precede both, or else in should be
added before Tokyo).

The correct answer is D.
765. Starfish, with anywhere from five to eight arms,
have a strong regenerative ability, and if one arm is
lost it quickly replaces it, sometimes by the animal
overcompensating and growing an extra one or two.
(A)

one arm is lost it quickly replaces it, sometimes
by the animal overcompensating and

(B)

one arm is lost it is quickly replaced, with the
animal sometimes overcompensating and

(C)

they lose one arm they quickly replace it,
sometimes by the animal overcompensating,

(D)

they lose one arm they are quickly replaced, with
the animal sometimes overcompensating,

(E)

they lose one arm it is quickly replaced,
sometimes with the animal overcompensating,

B
C

D

E

The correct answer is B.
766. In 2000, a mere two dozen products accounted for
half the increase in spending on prescription drugs,
a phenomenon that is explained not just because of
more expensive drugs but by the fact that doctors
are writing many more prescriptions for higher‐cost
drugs.
(A)

a phenomenon that is explained not just because
of more expensive drugs but by the fact that
doctors are writing

(B)

a phenomenon that is explained not just by the
fact that drugs are becoming more expensive
but also by the fact that doctors are writing

(C)

a phenomenon occurring not just because of
drugs that are becoming more expensive but
because of doctors having also written

(D)

which occurred not just because drugs are
becoming more expensive but doctors are also
writing

In a conditional sentence if X, (then) Y, rhetorical
flow is enhanced by the two clauses sharing the
same structure. If one clause is passive, the other
should be passive; if one clause is active, the other
should be active, too.

(E)

which occurred not just because of more
expensive drugs but because doctors have also
written

A

This sentence explains that a few high‐cost
products account for increased spending for
two reasons—rising drug prices and more
prescriptions for high‐priced drugs. To present
these two causes, the sentence employs a formula
that requires parallel elements: not just because

Agreement; Idiom

764

Correct. The conditional structure is clear
and correct.
This answer choice allows the unintended
reading that the animal replaces the missing
arm by overcompensating. The logical
connection between overcompensating and
growing is unclear.
The conditional clause has an active verb,
while the result clause has a passive verb.
The second they should refer to arm, so
the agreement is not correct. The logical
connection between overcompensating and
growing is unclear.
The conditional clause has an active
verb, while the result clause has a passive
verb. The logical connection between
overcompensating and growing is unclear.

The conditional clause has a passive verb,
while the result clause has an active verb.
The pronoun it should be plural since it
refers to starfish. We know that starfish is
plural in this sentence because it agrees with
have in the main clause.

Rhetorical construction; Idiom

9.8 Sentence Correction Answer Explanations

of x, but because of y, with x and y assuming the
same grammatical form. One way to create this
parallelism is to phrase both contributing causes
as noun clauses beginning with the fact that.
To streamline the sentence, unnecessary words
and redundancies should be eliminated. One
such redundancy is the repetition of meaning in
explained and because of.
A

B
C

D
E

It is redundant and confusing to say that
the phenomenon in question is explained …
because of; the sentence structure is not parallel.
Correct. This sentence correctly uses
parallel structure.
The phrasing drugs that are becoming and
doctors having also written are awkward and
confusing; the placement of also is incorrect.
The structure of this sentence is not parallel.
The placement of also is incorrect; the
structure of the sentence is not parallel.

music of Johnson. The second half of the sentence
describes two stages of the oral tradition from
which blues developed.
A

B
C

D

The correct answer is B.
767. Similar to other early Mississippi Delta blues singers, the
music of Robert Johnson arose from an oral tradition
beginning with a mixture of chants, fiddle tunes, and
religious music and only gradually evolved into the blues.
(A)

(B)

Similar to other early Mississippi Delta blues
singers, the music of Robert Johnson arose from
an oral tradition beginning with
Similar to that of other early Mississippi Delta
blues singers, Robert Johnson made music that
arose from an oral tradition that began with

(C)

As with other early Mississippi Delta blues
singers, Robert Johnson made music that arose
from an oral tradition beginning as

(D)

Like other early Mississippi Delta blues singers,
Robert Johnson’s music arose from an oral
tradition beginning with

(E)

Like the music of other early Mississippi Delta
blues singers, the music of Robert Johnson
arose from an oral tradition that began as

Logical predication; Verb form
The sentence aims to compare the music of
early Mississippi Delta Blues singers with the
music of Robert Johnson. But what it does is
illogically compare singers themselves with the

E

Similar to is a wordy and imprecise form
of comparison here. The second half of the
sentence attempts to describe the stages of
the oral tradition, but in a nonparallel form,
presenting the first stage as a participial
phrase and the second as a verb phrase.
Similar to is wordy and imprecise. The
pronoun that lacks a grammatical referent.
The comparative phrase as with is an
inappropriate form for comparing nouns
and noun phrases. The second half of the
sentence violates parallelism, attempting to
express the first stage of the oral tradition as
a present‐participle phrase and the second
as a past‐tense verb phrase.
This makes a sentence that illogically
compares singers with music. It violates
parallelism by coupling a participial phrase
beginning with . . . with a verb phrase
evolved into. . . .
Correct. The resulting sentence compares
like with like. It uses a relative clause to
describe the oral tradition from which blues
developed, indicating the two stages of
development with two verbs in parallel.

The correct answer is E.
768. Thelonious Monk, who was a jazz pianist and
composer, produced a body of work both rooted in
the stride‐piano tradition of Willie (The Lion) Smith and
Duke Ellington, yet in many ways he stood apart from
the mainstream jazz repertory.
(A)

Thelonious Monk, who was a jazz pianist and
composer, produced a body of work both rooted

(B)

Thelonious Monk, the jazz pianist and composer,
produced a body of work that was rooted both

(C)

Jazz pianist and composer Thelonious Monk,
who produced a body of work rooted

(D)

Jazz pianist and composer Thelonious Monk
produced a body of work that was rooted

(E)

Jazz pianist and composer Thelonious Monk
produced a body of work rooted both

765

GMAT® Official Guide 2018

Grammatical construction; Rhetorical
construction
The subject of the sentence is Thelonious Monk,
and the sentence tells about two things that
he did: produced and stood apart. The work
he produced was rooted in the mainstream
(stride piano) jazz tradition, yet at the same
time, he deviated from this tradition. The use
of a relative clause (who was a jazz pianist …)
or an appositive (the jazz pianist …) introduces
unnecessary wordiness and grammatical
complexity. Since only one point is being made
about Monk’s body of work, the appearance
of the word both in the clause presenting the
claim about Monk’s work is deceptive as well as
grammatically incorrect.
A
B
C
D

E

The relative clause introduces wordiness and
confusion.
The appositive introduces wordiness and
unnecessary grammatical complexity.
The sentence is a fragment because the main
subject, Thelonious Monk, has no verb.
Correct. The sentence concisely identifies
Thelonious Monk and expresses the single
point about his work without unnecessary
or misleading words.
The appearance of both is misleading, since
only one point is being made about where
Monk’s musical roots are located.

The correct answer is D.
769. Nobody knows exactly how many languages there
are in the world, partly because of the difficulty
of distinguishing between a language and the
sublanguages or dialects within it, but those who have
tried to count typically have found about five thousand.

766

(A)

and the sublanguages or dialects within it, but
those who have tried to count typically have found

(B)

and the sublanguages or dialects within them, with
those who have tried counting typically finding

(C)

and the sublanguages or dialects within it, but
those who have tried counting it typically find

(D)

or the sublanguages or dialects within them, but
those who tried to count them typically found

(E)

or the sublanguages or dialects within them,
with those who have tried to count typically
finding

Agreement; Idiom
This sentence first introduces a condition that makes
it difficult to count languages and then, with the
conjunction but, introduces the topic of those who
defy these difficulties and try to count the world’s
languages anyway. Connecting these two parts of the
sentence with but indicates that the second clause of
the sentence is counter to expectation. The challenges
of the task are explained using the example of
a single language and its many sublanguages or
dialects. When this example is referred to with a
pronoun, the pronoun should be singular; when
the languages being counted are referred to with a
pronoun, this pronoun must be plural.
A

B

C

D

E

Correct. The pronoun it agrees in number
to its singular antecedent, and but indicates
that the idea expressed in the final clause
defies expectations.
The plural pronoun them incorrectly refers to
the singular antecedent language; connecting
the two clauses with the preposition with
loses the sense that counting languages
despite the difficulties defies expectations.
The second appearance of it, referring to
world languages, is incorrect because it does
not agree in number with languages.
The conjunction or is incorrect—the
idiomatic expression is distinguishing between
x and y; the plural pronoun them does not
agree with the singular antecedent language.
The plural pronoun them incorrectly refers
to the singular antecedent, language; with is
an imprecise connector for the two clauses,
losing the counter‐to‐expectation relationship
between them.

The correct answer is A.
770. Although a number of excellent studies narrate
the development of domestic technology and
its impact on housewifery, these works do not
discuss the contributions of the women employed
by manufacturers and utility companies as product
demonstrators and publicists, who initially promoted
new and unfamiliar technology to female consumers.
(A)

by manufacturers and utility companies as
product demonstrators and publicists,

(B)

to be product demonstrators and publicists by
manufacturers and utility companies,

9.8 Sentence Correction Answer Explanations

(C)

to demonstrate and publicize their products by
manufacturers and utility companies

(D)

by manufacturers and utility companies to be
demonstrators and publicists of their products

(E)

by manufacturers and utility companies to
demonstrate and publicize their products

Logical predication; Rhetorical
construction
The point of the sentence is that studies do
not include the contributions of women who
promoted new domestic technology. The sentence
indicates clearly that the women were employed
by manufacturers and utility companies, worked as
product demonstrators and publicists, and promoted
new technology to female consumers.
Correct. The sentence clearly describes the
women’s employment and contributions.
B
This sentence structure appears to make who
initially promoted refer to manufacturers and
utility companies rather than to the women.
This conflicts with the use of the word who,
which would normally be expected to refer
to persons rather than to abstract entities
such as companies.
C Their refers to the women, incorrectly
suggesting that the products belong to them.
This sentence structure appears to make who
initially promoted refer to manufacturers and
utility companies rather than to the women. This
conflicts with the word who, which would
normally be expected to refer to persons rather
than to abstract entities such as companies.
D Who initially promoted follows, and appears
to refer to, products, which cannot logically
be the referent.
E Who initially promoted follows, and appears
to refer to, products, which cannot logically
be the referent.
The correct answer is A.
A

771. The absence from business and financial records of
the nineteenth century of statistics about women leave
us with no record of the jobs that were performed by
women and how they survived economically.
(A)

(B)

from business and financial records of statistics
about women from the nineteenth century leave
us with no record of what jobs women performed
or

(C)

of statistics for women from business and
financial records in the nineteenth century leaves
us with no record of either the jobs that women
were performing and of

(D)

of statistics on women from business and
financial records in the nineteenth century
leave us with no record of the jobs that women
performed or of

(E)

of statistics about women from business and
financial records of the nineteenth century leaves
us with no record of either what jobs women
performed or

Rhetorical construction; Agreement
This sentence is phrased awkwardly in two ways.
The first relates to absence of statistics: it is best
to place a noun modifier right next to the noun
that it modifies, with no intervening material.
Second, jobs that were performed by women is
more complicated than necessary—jobs women
performed is better, for example. Also, the singular
noun absence requires the correct agreeing verb
form leaves.
A

B

C

D
E

Absence and of statistics are widely separated,
and leave does not properly agree with its
subject, absence.
Absence and of statistics are widely separated,
and leave does not properly agree with its
subject, absence.
Jobs that women were performing is
unnecessarily long and complex. The either
construction should be completed with or,
not and.
Leave does not properly agree with its
subject, absence.
Correct. Leaves agrees with absence. The
phrase of statistics is next to the noun it
modifies (absence), and jobs women performed
is a nicely simple phrasing.

The correct answer is E.

from business and financial records of the
nineteenth century of statistics about women
leave us with no record of the jobs that were
performed by women and
767

GMAT® Official Guide 2018

772. Heating‐oil prices are expected to be higher this year
than last because refiners are paying about $5 a barrel
more for crude oil than they were last year.
(A)

Heating‐oil prices are expected to be higher this
year than last because refiners are paying about
$5 a barrel more for crude oil than they were

(A)

Even though Clovis points, spear points with
longitudinal grooves chipped onto their faces,
have been found all over North America, they are
named for the New Mexico site where they were
first discovered in 1932.

(B)

It is the expectation that heating‐oil prices will be
higher for this year over last because refiners
are paying about $5 a barrel more for crude oil
now than what they were

Although named for the New Mexico site where
first discovered in 1932, Clovis points are spear
points of longitudinal grooves chipped onto
their faces and have been found all over North
America.

(C)

It is expected that heating‐oil prices will rise
higher this year than last year’s because refiners
pay about $5 a barrel for crude oil more than
they did

Named for the New Mexico site where they have
been first discovered in 1932, Clovis points, spear
points of longitudinal grooves chipped onto the
faces, have been found all over North America.

(D)

Spear points with longitudinal grooves that are
chipped onto the faces, Clovis points, even
though named for the New Mexico site where
first discovered in 1932, but were found all over
North America.

(E)

While Clovis points are spear points whose faces
have longitudinal grooves chipped into them,
they have been found all over North America,
and named for the New Mexico site where they
have been first discovered in 1932.

(B)

Heating‐oil prices are expected to rise higher this
year over last because refiners pay about $5 a
barrel for crude oil more than they did

(C)

Expectations are for heating‐oil prices to be
higher this year than last year’s because refiners
are paying about $5 a barrel for crude oil more
than they did

(D)

(E)

773. Even though Clovis points, spear points with
longitudinal grooves chipped onto their faces, have
been found all over North America, they are named for
the New Mexico site where they were first discovered
in 1932.

Rhetorical construction; Idiom
The sentence connects a comparison between
this year’s and last year’s heating‐oil prices with
a comparison between this year’s and last year’s
crude‐oil prices. The most efficient, parallel
expression of those comparisons is to use two
comparative expressions, higher than and more
than.
A
B

C

D
E

Correct. This sentence expresses the
comparison in succinct, parallel phrases.
The comparative form, higher, anticipates
the comparative term than, not over; in the
second clause, the comparative terms more
than should immediately follow $5 a barrel.
Expectations are for … is an unnecessarily
wordy and indirect expression; the
possessive year’s is not parallel with the
adverbial phrase this year.
It is the expectation that … is wordy and
awkward; for and what are unnecessary.
It is expected that … is wordy and awkward;
the possessive last year’s does not parallel the
adverbial phrase this year.

The correct answer is A.

768

Verb form; Rhetorical construction; Logical
predication
Even though, although, and while introduce
clauses that appear to be logically incompatible
but in fact are not. In this sentence, the apparent
incompatibility that must be clearly expressed is
that although the spear points are named for a
particular place in New Mexico, they are in fact
found throughout North America. Because their
discovery took place in 1932 and is not ongoing,
the correct verb tense is simple past, not present
perfect.
A

Correct. The even though clause expresses
clearly that the seeming incompatibility is
between where the spear points have been
found (all over North America) and the
naming of the spear points for a single site
in New Mexico.

9.8 Sentence Correction Answer Explanations

B

C

D

E

The sentence structure indicates that the
expected incompatibility is between the
geographically based name of the points and
their physical properties, which makes no
sense; where discovered is missing a subject—
the correct form is where they were first
discovered.
Have been first discovered is the wrong
tense, since the discovery is a discrete event
completed in the past.
The sequence of information in this
sentence is confusing; even though and but
both introduce information that is contrary
to expectation, so to use them both to
describe a single apparent contradiction is
redundant and nonsensical.
While introduces a description of Clovis
points and suggests that this appears
incompatible with their appearance all over
North America, which makes no sense; have
been first discovered is the wrong tense.

The correct answer is A.
774. Heavy commitment by an executive to a course of
action, especially if it has worked well in the past,
makes it likely to miss signs of incipient trouble or
misinterpret them when they do appear.
(A)

Heavy commitment by an executive to a course
of action, especially if it has worked well in the
past, makes it likely to miss signs of incipient
trouble or misinterpret them when they do appear.

Rhetorical construction; Logical predication
This sentence explains that an executive who is
blindly committed to a proven course of action is
likely to overlook or misinterpret indicators that
the plan may no longer be working. The sentence
needs to make clear who may misinterpret these
indicators.
A

B

C

D

E

The passive construction causes the sentence
to be wordy and confusing; the reference
for it is ambiguous, leaving the reader with
questions about who or what is likely to
miss these signs.
The sentence structure indicates that the
executive, not his or her strategy, causes signs
to be overlooked; the modifier when they do
appear is misplaced.
The reference for the pronoun it is unclear
because many nouns have intervened
between the appearance of the logical
referent (course of action) and it.
Misinterpreting should be an infinitive verb
form to parallel miss; the phrasing throughout
the sentence is wordy and awkward.
Correct. The grammatical structure of this
sentence and the appropriate placement of
modifiers expresses the meaning clearly and
concisely.

The correct answer is E.
775. According to recent studies comparing the nutritional
value of meat from wild animals and meat from
domesticated animals, wild animals have less total fat
than do livestock fed on grain and more of a kind of fat
they think is good for cardiac health.

(B)

An executive who is heavily committed to a course
of action, especially one that worked well in the
past, makes missing signs of incipient trouble or
misinterpreting ones likely when they do appear.

(C)

An executive who is heavily committed to a
course of action is likely to miss or misinterpret
signs of incipient trouble when they do appear,
especially if it has worked well in the past.

(A)

wild animals have less total fat than do livestock
fed on grain and more of a kind of fat they think is

(B)

wild animals have less total fat than livestock fed
on grain and more of a kind of fat thought to be

(D)

Executives’ being heavily committed to a course of
action, especially if it has worked well in the past,
makes them likely to miss signs of incipient trouble
or misinterpreting them when they do appear.

(C)

wild animals have less total fat than that of
livestock fed on grain and have more fat of a
kind thought to be

(D)

(E)

Being heavily committed to a course of action,
especially one that has worked well in the past,
is likely to make an executive miss signs of
incipient trouble or misinterpret them when they
do appear.

total fat of wild animals is less than livestock
fed on grain and they have more fat of a kind
thought to be

(E)

total fat is less in wild animals than that of
livestock fed on grain and more of their fat is of
a kind they think is
769

GMAT® Official Guide 2018

Logical predication; Rhetorical construction
The sentence reports research findings on the
comparison between the fat content of wild
animals and that of domestic livestock. The most
significant error in the sentence is in the phrase
they think: the pronoun they either lacks a referent
or is meant to refer back to wild animals, which
would be nonsensical.
A
B

C

D
E

The pronoun they fails to refer correctly.
Correct. The phrase thought to be eliminates
the most significant error in the original
sentence. Note that while the phrase less
total fat than livestock differs from the phrase
less total fat than do livestock in the original,
either would be correct here.
The resulting sentence is unnecessarily
wordy and confusing. The pronoun that is
not only superfluous, but it fails to refer
back to anything.
The resulting sentence makes a nonsensical
comparison between total fat and livestock.
The resulting sentence is wordy and
confusing. It lacks the required parallelism
in wild animals . . . in livestock. The
referent of the possessive pronoun their is
ambiguous, as is the referent of the pronoun
they.

Idiom; Logical predication; Rhetorical
construction
This sentence identifies yellow jackets as one of 900
types of social wasps and provides an explanation
of the term social wasps. In this explanation, the
society or population—not the individual wasps
themselves—consists almost entirely of females.
The three descriptors of social wasps (cooperative,
organized, and consisting almost entirely of females) are
most effectively expressed in parallel structures.
A
B

C
D
E

They, referring to wasps, is an incorrect
subject for consist.
Correct. The three descriptors of the wasp
society are in parallel form, and consisting
properly modifies society.
The sentence structure makes it unclear
what almost all females describes.
And it is … violates the parallelism of the
three descriptors of social wasps.
And it consists … violates the parallelism of
the three descriptors.

The correct answer is B.
777. Before 1988, insurance companies in California were
free to charge whatever rates the market would bear,
needing no approval from regulators before raising
rates.

The correct answer is B.
(A)

needing no approval from regulators before raising

(B)

and it needed no approval by regulators before
raising

(C)

and needing no approval from regulators before
they raised

(D)

wasps living in a highly cooperative and
organized society where they consist almost
entirely of

with approval not needed by regulators before
they raised

(E)

with no approval needed from regulators before
the raising of

(B)

wasps that live in a highly cooperative and
organized society consisting almost entirely of

Logical predication; Rhetorical construction

(C)

which means they live in a highly cooperative and
organized society, almost all

(D)

which means that their society is highly
cooperative, organized, and it is almost entirely

(E)

living in a society that is highly cooperative,
organized, and it consists of almost all

776. Yellow jackets number among the 900 or so species
of the world’s social wasps, wasps living in a highly
cooperative and organized society where they consist
almost entirely of females—the queen and her sterile
female workers.
(A)

770

The sentence explains that, prior to 1988,
insurance companies in California could raise
rates without regulators’ approval. This idea is
expressed concisely using a participial phrase and
two prepositional phrases: needing no approval
from regulators before raising rates. Unlike some of
the answer choices that contain errors involving
antecedents, this construction uses no pronouns
and contains no such errors.

9.8 Sentence Correction Answer Explanations

A

B

C

D

E

Correct. The combination of a participial
phrase and two prepositional phrases
expresses the idea clearly with no errors
involving pronouns or antecedents.
The singular pronoun it has no clear
antecedent. If it is taken to refer to the
market (the only grammatically plausible
antecedent), the sentence is illogical.
Whatever rates the market would bear clearly
indicates that market is being used in the
sense of an abstract set of forces affecting
prices. To say that the market, in that sense,
raised taxes and that it needed no approval
to do so is nonsensical.
The construction were … and needing
is ungrammatical. The pronoun they is
intended to refer to companies but could also
seem, illogically, to refer to regulators.
By regulators illogically indicates that
regulators are the ones who did not need
approval; the pronoun they is intended
to refer to companies but could also seem,
illogically, to refer to regulators.
Before the raising of is wordy and awkward.
Both that phrase and with no approval
needed are strangely uninformative and
rhetorically ineffective in that they appear
to pointedly avoid telling who did not need
the approval or who might have raised the
rates.

(E)

Rhetorical construction; Logical
predication
The main point of this sentence is to explain
that while Marconi felt the radio would
substitute for the phone as an instrument of
private communication, in fact it has become
an instrument of mass communication. It is
less wordy to use Marconi as the subject of the
active verb conceived than to use the subject
conception with the static verb was. The pronoun
it positioned as the subject of the final verb
has become refers back to radio. Versions of the
sentence that use the relative pronoun which
indicate that the telephone has become a mass
medium.
A
B

C

D
E

The correct answer is A.
778. Marconi’s conception of the radio was as a substitute
for the telephone, a tool for private conversation;
instead, it is precisely the opposite, a tool for
communicating with a large, public audience.
(A)

Marconi’s conception of the radio was as a
substitute for the telephone, a tool for private
conversation; instead, it is

(B)

Marconi conceived of the radio as a substitute
for the telephone, a tool for private conversation,
but which is

(C)

(D)

Marconi conceived of the radio as a tool for
private conversation that could substitute for the
telephone; instead, it has become
Marconi conceived of the radio to be a tool
for private conversation, a substitute for the
telephone, which has become

Marconi conceived of the radio to be a substitute
for the telephone, a tool for private conversation,
other than what it is,

The nominalized subject, conception, leads to
a wordy and awkward sentence.
The reference for the relative pronoun which
is ambiguous; the sentence as a whole is
awkward.
Correct. An active verb makes the first
clause more concise; it in the second clause
clearly refers to the radio.
Conceived of … should be followed by as
rather than to be.
Conceived of … should be followed by as
rather than to be; other than what it is is
awkward, wordy, and redundant, overlapping
the meaning of precisely the opposite … .

The correct answer is C.
779. Because there are provisions of the new maritime code
that provide that even tiny islets can be the basis for
claims to the fisheries and oil fields of large sea areas,
they have already stimulated international disputes
over uninhabited islands.
(A)

Because there are provisions of the new maritime
code that provide that even tiny islets can be the
basis for claims to the fisheries and oil fields of
large sea areas, they have already stimulated

(B)

Because the new maritime code provides that
even tiny islets can be the basis for claims to the
fisheries and oil fields of large sea areas, it has
already stimulated
771

GMAT® Official Guide 2018

(C)

(D)

(E)

Even tiny islets can be the basis for claims to the
fisheries and oil fields of large sea areas under
provisions of the new maritime code, already
stimulating
Because even tiny islets can be the basis for
claims to the fisheries and oil fields of large
sea areas under provisions of the new maritime
code, this has already stimulated
Because even tiny islets can be the basis for
claims to the fisheries and oil fields of large
sea areas under provisions of the new maritime
code, which is already stimulating

Logical predication; Grammatical
construction
In this sentence, the there are … that …
construction contributes nothing more than
unnecessary words. The sentence needs to make
clear whether provisions or code is the subject of
the main verb stimulated.
A

B

C

D
E

The there are … that … construction is
unnecessarily wordy; in the predicate
nominative instead of the subject position,
provisions is not an obvious referent for the
pronoun they.
Correct. In this sentence, the new maritime
code is clearly the antecedent of it in the
main clause and thus the subject of has
already stimulated.
Under provisions of the new maritime code is a
misplaced modifier, seeming to describe sea
areas; the sentence does not make clear what
is stimulating … disputes.
The referent of this is unclear.
The sentence is a fragment, opening with
a dependent clause (Because … code) and
concluding with a relative clause, but
lacking a main, independent clause.

The correct answer is B.
780. Unlike the automobile company, whose research was
based on crashes involving sport utility vehicles, the
research conducted by the insurance company took
into account such factors as a driver’s age, sex, and
previous driving record.

772

(A)

company, whose research was based on

(B)

company, which researched

(C)

company, in its research of

(D)

company’s research, having been based on

(E)

company’s research on

Logical predication; Rhetorical
construction
The point of the sentence is to contrast the
research conducted by the automobile company
and that conducted by the insurance company,
but the sentence has been written in a way that
contrasts the automobile company with research. The
correct contrast is between automobile company’s
research and research conducted by the insurance
company.
A
B
C
D

E

Automobile company is incorrectly contrasted
with research.
Automobile company is incorrectly contrasted
with research.
Automobile company is incorrectly contrasted
with research.
Having been based on is wordy. This
construction makes it unclear whether
having been based on crashes involving
sport utility vehicles is intended to modify
the automobile company’s research or the
research conducted by the insurance company.
The sentence structure slightly favors the
latter interpretation, but it is somewhat
implausible to suppose that this is the
intended meaning.
Correct. The sentence concisely contrasts
the automobile company’s research and research
conducted by the insurance company.

The correct answer is E.
781. Gusty westerly winds will continue to usher in a
seasonably cool air mass into the region, as a broad
area of high pressure will build and bring fair and dry
weather for several days.
(A)

to usher in a seasonably cool air mass into the
region, as a broad area of high pressure will
build and

(B)

ushering in a seasonably cool air mass into the
region and a broad area of high pressure will
build that

(C)

to usher in a seasonably cool air mass to the
region, a broad area of high pressure building, and

9.8 Sentence Correction Answer Explanations

(D)

ushering a seasonably cool air mass in the
region, with a broad area of high pressure
building and

(E)

to usher a seasonably cool air mass into the
region while a broad area of high pressure
builds, which will

Verb form; Grammatical construction;
Diction
The sentence offers a prediction that two
concurrent weather events will bring a certain
type of weather, but its use of the phrase as
. . . will build causes confusion. The wording
makes the sequence of events and the causal
relationships among them unclear. Future tense
is used to indicate that winds will continue. The
relation of as to the rest of the sentence makes
it unclear whether as is intended as a logical
indicator (similar to because) or as a temporal
indicator (equivalent to while). If as is intended in
the former way, it would be preferable to resolve
the ambiguity by using a word or phrase such
as because or given that. If it is intended in the
latter way, a present tense verb would be needed
following as or while to show that the second
event is concurrent with, or part of, the future
situation: while high pressure builds. In addition,
usher in . . . into is redundant and unidiomatic.
A clear, concise way to express this idea is will
continue to usher a seasonably cool air mass into the
region while a broad area of high pressure builds.
A

B

C
D

E

Usher in … into is redundant and
unidiomatic. As … will build causes
confusion.
The absence of a comma after region makes
this ungrammatical. Ushering in . . . into
is redundant; will build is the wrong verb
tense; the plural verb bring does not agree
with area, the singular antecedent of that.
The grammatical function of a broad area of
high pressure building is unclear.
Ushering … in the region is incorrect since
into is needed to indicate movement from
outside in; the subject of the verb bring is
unclear.
Correct. The idea is expressed with clear,
correct combinations of verbs and subjects.

The correct answer is E.

782. With the patience of its customers and with its network
strained to the breaking point, the on-line service
company announced a series of new initiatives trying
to relieve the congestion that has led to at least four
class-action lawsuits and thousands of complaints from
frustrated customers.
(A)

the patience of its customers and with its
network strained to the breaking point, the online service company announced a series of new
initiatives trying to relieve

(B)

the patience of its customers and its network
strained to the breaking point, the on-line service
company announced a series of new initiatives
that try to relieve

(C)

its network and the patience of its customers
strained to the breaking point, the on-line service
company announced a series of new initiatives to
try to relieve

(D)

its network and with the patience of its
customers strained to the breaking point, the
on-line service company announced a series of
initiatives to try relieving

(E)

its network and its customers’ patience strained
to the breaking point, the on-line service
company announced a series of new initiatives to
try relieving

Logical predication; Rhetorical
construction
The sentence explains the online service
provider’s efforts to relieve congestion, but it
has been written with confusing ambiguities.
Because the patience of its customers is in a separate
prepositional phrase from its network, it is not
clear whether both or only the latter is strained
to the breaking point. The phrase trying to relieve
is probably meant to explain the purpose of the
initiatives, but does not do so unambiguously
(for example, trying could modify either the
company or initiatives, and it is not clear which
is intended). An unambiguous wording of the
sentence would clarify that both the patience
of its customers and its network are strained to
the breaking point and that the purpose of the
initiatives is to try to relieve the congestion.
A

The patience of its customers is not clearly linked
to strained to the breaking point; trying to relieve
is an ambiguous and unidiomatic way of
expressing the purpose of the initiatives.
773

GMAT® Official Guide 2018

B

C

D

E

That try to relieve fails to express the purpose
of the initiatives in a rhetorically acceptable
way; moreover the present tense try is
illogical here. The phrase the patience of
its customers and its network is rhetorically
flawed in that its structure makes it seem to
be attributing patience to the network.
Correct. The sentence indicates clearly
that both the patience of its customers and
its network are strained to the breaking point
and that the company introduced initiatives
aimed at relieving the congestion.
Its network is not clearly linked to strained
to the breaking point; with to try, the
infinitive form to relieve is more rhetorically
appropriate here than the verbal noun form
relieving, in order to indicate the goal of the
intervention.
With to try, the infinitive form to relieve
is more rhetorically appropriate here than
the verbal noun form relieving, in order to
indicate the goal of the intervention.

comparison between sales in one November with
sales in other Novembers. It does not make sense
to compare sales to months, as in (C) and (D).
A

B

C

The correct answer is C.
D
783. November is traditionally the strongest month for sales
of light trucks, but sales this past November, even
when compared with sales in previous Novembers,
accounted for a remarkably large share of total vehicle
sales.

E
(A)

but sales this past November, even when
compared with sales in previous Novembers,

(B)

but even when it is compared with previous
Novembers, this past November’s sales

(C)

but even when they are compared with previous
Novembers, sales of light trucks this past
November

(D)

so that compared with previous Novembers,
sales of light trucks this past November

(E)

so that this past November’s sales, even
compared with previous Novembers’ sales,

Logical predication; Agreement
This sentence identifies November as traditionally
being the month with the strongest sales of light
trucks, and then goes on to indicate that even
when compared to previous Novembers, this past
November’s sales accounted for a notably large
portion of overall sales. It makes sense to make a
774

Correct. This version makes the correct
comparison between sales in one particular
November and sales in previous Novembers.
The antecedent of the word it is unclear. The
sentence begins with the word November,
which is used to refer not to a specific
November, but to the month generally. If
the antecedent of it is taken to be November,
then the sentence compares November,
taken generally, to previous Novembers. But
previous to what? Since taking November
to be the antecedent renders the sentence
nonsensical, we may be inclined to look for
the antecedent elsewhere; the only other
possible candidate, however, is this past
November’s sales, which is ruled out because
it is singular and sales is plural.
The antecedent of the pronoun they is sales,
making the comparison between they and
previous Novembers illogical.
The sentence illogically compares sales
of light trucks with previous Novembers.
The conjunction so that nonsensically
introduces a causal relationship between
November’s typically strong sales and the
aforementioned comparison.
Like (D), this sentence introduces a
nonsensical causal relationship, in this case
between the fact that November typically
has the strongest sales and the fact that
this past November’s sales accounted for a
remarkably large share of total vehicle sales.

The correct answer is A.
784. Most of the country’s biggest daily newspapers had
lower circulation in the six months from October 1995
through March 1996 than a similar period a year
earlier.
(A)

a similar period

(B)

a similar period’s

(C)

in a similar period

(D)

that in a similar period

(E)

that of a similar period

9.8 Sentence Correction Answer Explanations

Logical Predication; Parallelism
The sentence compares newspaper circulation
during two separate periods, but the comparison
is not parallel because it has been drawn using a
prepositional phrase, in the six months, and a noun
phrase, a similar period. Both phrases compared
by than should be prepositional phrases: lower in
the six months . . . than in a similar period.
In the six months and a similar period are not
grammatically parallel. The sentence appears
illogically to compare a period of time with
an amount of circulation.
B
In the six months and a similar period’s are
not grammatically parallel.
C Correct. In the six months and in a similar
period are both prepositional phrases,
making the comparison clear and properly
idiomatic.
D In the six months and that in a similar period
are not grammatically parallel; it is unclear
what the antecedent of that is supposed to
be.
E In the six months and that of a similar period
are not grammatically parallel; it is unclear
what the antecedent of that is supposed to be.
The correct answer is C.
A

785. Mauritius was a British colony for almost 200 years,
excepting for the domains of administration and
teaching, the English language was never really spoken
on the island.
(A)

excepting for

(B)

except in

(C)

but except in

(D)

but excepting for

(E)

with the exception of

Idiom; Grammatical construction
This two‐clause sentence describes an apparent
incompatibility: as a British colony, Mauritius
might be expected to be English‐speaking, but
in fact it was not. To describe this apparent
contradiction and to avoid a comma splice, the
clauses should be joined by the conjunction but.
Domains describes places in which English is
spoken; for is the incorrect preposition. Excepting
is not idiomatic English in this case.

A
B
C

D
E

The lack of a conjunction causes a comma
splice; excepting for is non‐idiomatic.
The lack of a conjunction causes a comma
splice.
Correct. The two independent clauses
are separated by but, and except in is an
appropriate idiom.
Excepting for is non‐idiomatic.
The lack of a conjunction causes a comma
splice.

The correct answer is C.
786. Although appearing less appetizing than most of
their round and red supermarket cousins, heirloom
tomatoes, grown from seeds saved during the previous
year—they are often green and striped, or have plenty
of bumps and bruises—heirlooms are more flavorful
and thus in increasing demand.
(A)

Although appearing less appetizing than most
of their round and red supermarket cousins,
heirloom tomatoes, grown from seeds saved
during the previous year

(B)

Although heirloom tomatoes, grown from seeds
saved during the previous year, appear less
appetizing than most of their round and red
supermarket cousins

(C)

Although they appear less appetizing than most
of their round and red supermarket cousins,
heirloom tomatoes, grown from seeds saved
during the previous year

(D)

Grown from seeds saved during the previous
year, heirloom tomatoes appear less appetizing
than most of their round and red supermarket
cousins

(E)

Heirloom tomatoes, grown from seeds saved
during the previous year, although they appear
less appetizing than most of their round and red
supermarket cousins

Rhetorical construction; Grammatical
construction
The intended meaning could be communicated
more effectively by mentioning heirloom
tomatoes as early as possible in the sentence,
so that we know that the writer is comparing
heirloom tomatoes with supermarket tomatoes.
The placement of heirloom tomatoes and heirlooms
makes the sentence ungrammatical.
775

GMAT® Official Guide 2018

A

B

C

D

E

This is ungrammatical. If heirloom tomatoes
is the subject of are more flavorful …
then heirlooms has no predicate and is
nonsensically superfluous. If heirlooms is the
subject, heirloom tomatoes has no predicate.
Correct. The noun heirloom tomatoes is
mentioned early in the sentence, followed
by a parenthetical definition, and is the
subject of the verb appear, and heirlooms is
the subject of are.
The noun heirloom tomatoes appears too late
in the sentence. Parsing is made harder by
introducing the pronoun they and revealing
its antecedent later in the sentence. The
sentence is also ungrammatical. If heirloom
tomatoes is the subject of are more flavorful
… then heirlooms has no predicate and is
nonsensically superfluous. If heirlooms is the
subject, heirloom tomatoes has no predicate.
Beginning the sentence with the
explanatory clause grown from seeds …
gives it too much importance. It could
be construed as the reason why heirloom
tomatoes appear less appetizing, which is
contrary to the truth. The sentence is also
ungrammatical.
Rhetorical structure requires that although
appear in the beginning of the clause to
which it pertains. Placing it later necessitates
the pronoun they with antecedent heirloom
tomatoes, which is redundant. The sentence is
also ungrammatical.

The correct answer is B.

which most scientists agree to be a phenomenon
caused by fossil fuels burned by human beings,

Logical predication; Rhetorical
construction
The underlined portion of the sentence is
an appositive defining global warming as a
phenomenon caused by the burning of fossil fuels
by humans. Because this appositive intervenes
between the subject (global warming) and verb
(will create) of a clause, it should be expressed as
clearly and economically as possible so as not to
confuse the meaning of the sentence as a whole.
A
B
C

D
E

To be caused and in burning are wordy,
awkward, and indirect.
That is should immediately follow
phenomenon, not agree.
Correct. The phrase human beings’ burning
is more economical than constructions with
prepositional phrases or relative clauses.
The phrasing is wordy and indirect.
The phrasing is wordy and the meaning is
imprecise; it is not fossil fuels that cause
global warming—it is the burning of fossil
fuels by humans.

The correct answer is C.
788. The largest of all the planets, not only is Jupiter three
times so massive as Saturn, the next larger planet, but
also possesses four of the largest satellites, or moons,
in our solar system.
(A)

not only is Jupiter three times so massive as
Saturn, the next larger

(B)

not only is Jupiter three times as massive as
Saturn, the next largest

(C)

Jupiter, not only three times as massive as
Saturn, the next largest

(D)

a phenomenon most scientists agree to be
caused by human beings in burning fossil fuels,

Jupiter not only is three times as massive as
Saturn, the next largest

(E)

(B)

a phenomenon most scientists agree that is
caused by fossil fuels burned by human beings,

Jupiter is not only three times so massive as
Saturn, the next larger

Diction; Idiom

(C)

a phenomenon that most scientists agree is
caused by human beings’ burning of fossil fuels,

(D)

which most scientists agree on as a
phenomenon caused by human beings who burn
fossil fuels,

787. The World Wildlife Fund has declared that global
warming, a phenomenon most scientists agree to be
caused by human beings in burning fossil fuels, will
create havoc among migratory birds by altering the
environment in ways harmful to their habitats.
(A)

776

(E)

This sentence begins with a phrase, [t]he largest
. . ., describing the main subject Jupiter. The
remainder of the sentence describes Jupiter’s size
and possession of moons, using the idiom not only
x but y to introduce parallel adjective phrases.

9.8 Sentence Correction Answer Explanations

A

B
C

D

E

In this version of the sentence, so massive as
Saturn violates the parallelism established
by the idiom as + adjective + as + noun.
For the sake of clarity, the noun described
by the opening adjectival phrase should
immediately follow that phrase. The phrase
next larger is unidiomatic and unclear. The
superlative (largest) is appropriate in this
consideration of all the planets.
As in (A), Jupiter should immediately follow
the opening phrase.
This version of the sentence violates the
parallelism of the idiom not only x but y,
following the first half of the template with
an adjective phrase and the second half with
a verb phrase.
Correct. The placement of the subject
in relation to the opening modifier, the
properly constructed phrasing, and the
proper use of comparison words make the
meaning of the sentence clear.
The phrase so massive as violates the idiom as
x as y. Consideration of Jupiter’s size among
all the planets, including Saturn, requires
the superlative form, largest.

Agreement; Parallelism
This sentence states that whereas many dinosaur
fossils from China suggest that there is a kinship
between dinosaurs and birds, the wealth of fossils
are more ambiguous about what they suggest about
the ancestry of birds. The word wealth is a mass
noun followed by a prepositional phrase; because
wealth follows the article the, the emphasis is on it
rather than on the noun in the prepositional phrase,
fossils. The singular wealth requires a singular main
verb (seems). The comparative expression more
likely . . . must be followed by an infinitive verb (to
inflame) so it will be parallel to the verb with which
it is compared (to settle).
A

B

C

The correct answer is D.
789. While many of the dinosaur fossils found recently
in northeast China seem to provide evidence of the
kinship between dinosaurs and birds, the wealth of
enigmatic fossils seem more likely at this stage that
they will inflame debates over the origin of birds rather
than settle them.
(A)

seem more likely at this stage that they will
inflame debates over the origin of birds rather
than

(B)

seem more likely that it will inflame debates over
the origin of birds at this stage than

(C)

seems more likely to inflame debates on the
origin of birds at this stage rather than

(D)

seems more likely at this stage to inflame
debates over the origin of birds than to

(E)

seems more likely that it will inflame debates on
the origin of birds at this stage than to

D

E

The singular subject wealth does not agree
with the plural verb seem; both items
being compared should be in parallel form
(inflame and settle); the relative clause that
they will inflame violates the parallelism.
Again, the relative clause violates the desired
parallelism; the singular subject, wealth,
requires a singular verb, seems.
The subject and the verb agree with one
another, but the placement of the modifier
at this stage makes the modifier appear to
describe origin of the birds rather than the
verb seems.
Correct. The verb seems agrees in number
with the noun wealth; the infinitive to
inflame is parallel with the verb to which it
is compared, to settle.
The clause wealth . . . seems more likely that
it will inflame is not idiomatic, and that it
will inflame is not parallel with the infinitive
verb to settle.

The correct answer is D.
790. Found only in the Western Hemisphere and surviving
through extremes of climate, hummingbirds’ range
extends from Alaska to Tierra del Fuego, from sea‐level
rain forests to the edges of Andean snowfields and ice
fields at altitudes of 15,000 feet.
(A)

Found only in the Western Hemisphere
and surviving through extremes of climate,
hummingbirds’ range extends

(B)

Found only in the Western Hemisphere,
hummingbirds survive through extremes of
climate, their range extending
777

GMAT® Official Guide 2018

(C)

Hummingbirds, found only in the Western
Hemisphere and surviving through extremes of
climate, with their range extending

(D)

Hummingbirds, found only in the Western
Hemisphere and surviving through extremes of
climate, their range extends

(E)

Hummingbirds are found only in the Western
Hemisphere, survive through extremes of
climate, and their range extends

Logical predication; Grammatical
construction
This sentence makes three points about
hummingbirds: they live in the Western Hemisphere,
they survive extreme climates, and their range is wide
and varied. Hummingbirds, not hummingbirds’ range,
should be the subject of the sentence.
A

B

C

D
E

The opening modifier, found . . . and
surviving . . . should modify hummingbirds,
not, as it does, hummingbirds’ range.
Correct. Hummingbirds is the subject of the
sentence, and the use of the absolute phrase
their range extending appropriately connects
the final clause to the rest of the sentence.
This version of the sentence has no main
verb for the subject Hummingbirds and as a
result is ungrammatical.
Like (C), this version of the sentence has no
main verb for the subject Hummingbirds.
This version of the sentence begins with the
subject Hummingbirds attached to a pair of
verb phrases (are found . . . and survive . . .)
followed by the conjunction and, which
suggests that what follows should also
be a verb phrase; instead, a new subject
is introduced, their range. Replacing the
comma before survive with and would
render the sentence acceptable.

The correct answer is B.
791. She was less successful after she had emigrated
to New York compared to her native Germany,
photographer Lotte Jacobi nevertheless earned a small
group of discerning admirers, and her photographs
were eventually exhibited in prestigious galleries
across the United States.
(A)

778

She was less successful after she had emigrated
to New York compared to

(B)

Being less successful after she had emigrated to
New York as compared to

(C)

Less successful after she emigrated to New York
than she had been in

(D)

Although she was less successful after
emigrating to New York when compared to

(E)

She had been less successful after emigrating to
New York than in

Idiom; Grammatical construction; Logical
predication
This sentence compares the success Jacobi
experienced after moving to New York to
the success she had previously experienced in
Germany. The phrase less successful anticipates the
conclusion of the comparison with the phrase
than. … The main subject of the sentence is
photographer Lotte Jacobi, and the main verb is
earned. The opening clause She was less successful
… therefore creates a comma splice if the comma
is not followed by a conjunction. The most
efficient way to incorporate the information
about Jacobi’s comparative successes in Germany
and in New York is to turn this clause into an
adjectival phrase describing Jacobi.
A

B

C

D
E

Less successful … anticipates than rather than
compared to …; a comma is insufficient to
join two independent clauses into a single
sentence.
As compared to is an incorrect way to
complete the comparison introduced by
less; Being … is unnecessarily wordy and
awkward.
Correct. The idiomatic construction less
successful … than is incorporated into an
introductory adjectival phrase modifying
Lotte Jacobi.
When compared to is an incorrect phrase to
complete the comparison introduced by less.
A comma is insufficient to join two
independent clauses into a single sentence;
past‐perfect tense is misleading, since it
refers to Jacobi’s experience in New York,
which in fact followed her experience in
Germany.

The correct answer is C.

9.8 Sentence Correction Answer Explanations

792. Scientists have recently found evidence that black
holes—regions of space in which matter is so
concentrated and the pull of gravity so powerful that
nothing, not even light, can emerge from them—
probably exist at the core of nearly all galaxies and
the mass of each black hole is proportional to its host
galaxy.
(A)

exist at the core of nearly all galaxies and the
mass of each black hole is proportional to

(B)

exist at the core of nearly all galaxies and that
the mass of each black hole is proportional to
that of

(C)

exist at the core of nearly all galaxies, and that
the mass of each black hole is proportional to

(D)

exists at the core of nearly all galaxies, and that
the mass of each black hole is proportional to
that of

(E)

exists at the core of nearly all galaxies and the
mass of each black hole is proportional to that of

Logical predication; Agreement
This sentence focuses attention on two
hypotheses about black holes—one about their
location and the other about their mass. These
hypotheses appear as parallel relative clauses that
black holes . . . exist at . . . and that the mass . . .
is proportional. The subject of the first relative
clause (black holes) is plural, so the clause must be
completed with the plural form of exist.
A

B

C

This version of the sentence does not
provide the relative pronoun that to provide
parallel structure for presenting the two
things scientific evidence reveals about
black holes. This leaves it somewhat unclear
whether the final clause is intended to
convey part of what the scientists discovered
or to express a claim that the writer is
making independently of the scientists’
discovery. The sentence illogically compares
mass to galaxy.
Correct. The structure of the sentence
makes the meaning clear, and the plural verb
form agrees with the plural subject.
The comma appears to signal that the
final part of the sentence is intended as an
independent clause expressing a separate
claim rather than describing part of the
scientists’ discovery. But if it were intended

D

E

in that way, it should be set off as a separate
sentence, not conjoined with the preceding
clause. Like (A), this version makes an
illogical comparison between mass and
galaxy.
The singular verb exists does not agree with
the plural subject black holes. As in (C), the
comma is inappropriate because it does not
introduce a new independent clause.
The singular verb exists does not agree
with the plural subject black holes. Like
(A), this version lacks the relative pronoun
that, which would clarify the relationship
between the two clauses.

The correct answer is B.
793. The use of lie detectors is based on the assumption
that lying produces emotional reactions in an individual
that, in turn, create unconscious physiological
responses.
(A)

that, in turn, create unconscious physiological
responses

(B)

that creates unconscious physiological
responses in turn

(C)

creating, in turn, unconscious physiological
responses

(D)

to create, in turn, physiological responses that
are unconscious

(E)

who creates unconscious physiological
responses in turn

Agreement; Rhetorical construction;
Logical predication
This sentence describes a cause‐and‐effect
sequence; in the underlined portion of the
sentence, the relative pronoun that refers to the
plural noun reactions. The verb in the relative
clause must therefore be a plural verb. The causal
sequence is most clearly expressed by a relative
clause that turns the object emotional reactions
(from the clause lying causes emotional reactions
in an individual) into the subject (that) of a new
clause (that in turn create unconscious physiological
responses). In turn is best placed before the verb of
the second relative clause, create, to clarify that a
chain of events is being described.

779

GMAT® Official Guide 2018

A
B

C

D

E

Correct. This construction clearly indicates
the causal sequence.
The singular verb creates does not agree
with the subject referenced by the relative
pronoun that (reactions).
This construction is less successful at
clarifying the chain of events because
creating seems to refer back to lying; if used
as a participial, creating would have to be
preceded by a comma.
This construction does not make clear the
causal chain of events, because it is unclear
which noun to create should attach to; the
infinitive construction implies intent, which
does not really make sense.
Because reactions is not a person, who is the
wrong relative pronoun to use.

The correct answer is A.
794. Australian embryologists have found evidence that
suggests that the elephant is descended from an
aquatic animal, and its trunk originally evolving as a
kind of snorkel.
(A)

that suggests that the elephant is descended
from an aquatic animal, and its trunk originally
evolving

(B)

that has suggested the elephant descended from
an aquatic animal, its trunk originally evolving

(C)

suggesting that the elephant had descended
from an aquatic animal with its trunk originally
evolved

(D)

(E)

to suggest that the elephant had descended
from an aquatic animal and its trunk originally
evolved
to suggest that the elephant is descended from
an aquatic animal and that its trunk originally
evolved

into a participle modifying evidence. The word
descended is a predicate adjective following the
present‐tense verb is and describing the present‐
day elephant. The verb evolved should be past
tense because it describes how the trunk of the
elephant originally evolved, not how it is evolving
today.
A

B

C
D

E

The correct answer is E.
795. Most efforts to combat such mosquito‐borne diseases
like malaria and dengue have focused either on the
vaccination of humans or on exterminating mosquitoes
with pesticides.
(A)

like malaria and dengue have focused either on
the vaccination of humans or on exterminating

(B)

like malaria and dengue have focused either on
vaccinating of humans or on the extermination of

(C)

as malaria and dengue have focused on either
vaccinating humans or on exterminating

(D)

as malaria and dengue have focused on either
vaccinating of humans or on extermination of

(E)

as malaria and dengue have focused on either
vaccinating humans or exterminating

Parallelism; Verb form
The clearest, most economical way of expressing
the two things suggested by Australian
embryologists’ evidence is to format them as
relative clauses serving as parallel direct objects
of the verb suggest. It is awkward and confusing
to string together relative clauses: evidence that
suggests that the elephant. … A clearer way of
making this connection is to turn the verb suggests
780

The string of relative phrases is awkward
and confusing; the phrase following
the conjunction and is not parallel with
the relative clause that the elephant is
descended. …
The evidence still suggests these things about
the evolution of the elephant and its trunk, so
the present‐perfect verb tense is inaccurate.
Had descended is the wrong verb tense; with
cannot be followed by an independent clause.
Had descended is the wrong tense; the phrase
following the conjunction and does not
parallel the relative clause that precedes the
conjunction.
Correct. The two dependent clauses
beginning with that are in parallel form and
contain verbs in the correct tenses.

Diction; Parallelism
The phrase such … diseases like malaria and dengue
is not a correct way in English to indicate that the
two diseases mentioned are examples of a larger
category; the correct expression is such … as. …

9.8 Sentence Correction Answer Explanations

A

B

C
D

E

This use of such … like … is incorrect English;
the correct expression is such … as.… It is
better to keep the preposition on close to the
verb it goes with, focus, so as not to repeat it.
The correct expression is such … as.… It is
better to keep the preposition on close to the
verb it goes with, focus, so as not to repeat it.
This use of the gerund vaccinating (followed
by of ) would normally be preceded by the,
but this would make the phrase awkward.
It would be preferable to use vaccination,
which is parallel to extermination.
This answer choice incorrectly repeats the
preposition on before exterminating.
This answer choice incorrectly repeats the
preposition on before extermination. This use
of the gerund vaccinating (followed by of )
would normally be preceded by the, but this
would make the phrase awkward. It would
be preferable to use vaccination, which is
parallel to extermination.
Correct. This version uses either … or …
correctly and appropriately uses the parallel
forms vaccinating and exterminating.

The correct answer is E.
796. Among the Tsonga, a Bantu-speaking group of tribes
in southeastern Africa, dance teams represent
their own chief at the court of each other, providing
entertainment in return for food, drink, and lodging.

other is unidiomatic and unclear. It could be
intended to indicate, somewhat implausibly, that
each team has a court that the other teams visit,
but represent their own chief strongly suggests that
the court referred to is the court of another chief.
The correct pronoun to refer to a different chief is
another.
The court of each other is unidiomatic and
unclear. Each other seems to refer, somewhat
illogically, to the dancers.
B Correct. The sentence clearly explains the
idea, using the correct pronoun another and
the correct idiom in return for.
C This could be confusing in that the other
indicates that there is only one other chief,
whereas group of tribes suggests that there
may be a number of chiefs. As a return on
is the incorrect idiom; a return on normally
refers to a gain from an investment, not a
direct exchange of one good for another.
D Each other’s court somewhat illogically
indicates that the dancers each have a court.
Entertainment being provided in return for is
awkward and indirect.
E As a return on is the incorrect idiom; a
return on normally refers to a gain from an
investment, not a direct exchange of one
good for another.
The correct answer is B.
A

797. Almost like clones in their similarity to one another, the
cheetah species’ homogeneity makes them especially
vulnerable to disease.

(A)

the court of each other, providing entertainment
in return for

(B)

the court of another and provide entertainment in
return for

(A)

the court of the other, so as to provide
entertainment as a return on

the cheetah species’ homogeneity makes them
especially vulnerable to disease

(B)

(D)

each other’s court, entertainment being provided
in return for

the cheetah species is especially vulnerable to
disease because of its homogeneity

(C)

(E)

another’s court and provide entertainment as a
return on

the homogeneity of the cheetah species makes it
especially vulnerable to disease

(D)

homogeneity makes members of the cheetah
species especially vulnerable to disease

(E)

members of the cheetah species are especially
vulnerable to disease because of their
homogeneity

(C)

Diction; Idiom
The point of the sentence is that dancers
representing one chief perform at the court of
another chief in return for gifts. The court of each

781

GMAT® Official Guide 2018

Agreement; Logical predication

Rhetorical construction; Idiom

Genetic homogeneity is presented as a cause of
cheetahs’ vulnerability to disease. The opening
adjectival phrase refers to the fact that individual
cheetahs are almost like clones of one another
because of how genetically similar they are. This
adjectival phrase should be followed by what it
describes, individual cheetahs. But the structure
of the sentence makes it seem that this adjectival
phrase is meant—illogically—to describe the
cheetah species’ homogeneity. The sentence structure
also fails to make clear that the intended
reference is to just one (the only) cheetah species.

The intended meaning of the sentence seems to
be that the electricity production of windmills
is expected to approximately double by year’s
end. But instead of saying almost double, we have
almost expected, which is an unclear idea. Also
unclear is what the relative pronoun which refers
to.

A
B

C

D

E

The sentence nonsensically presents the
opening phrase as describing homogeneity.
The resulting sentence opens with a
reference to a plurality of individuals (to one
another), but confusingly identifies this with
a single (collective) entity, a species.
The resulting sentence nonsensically
presents the opening phrase as describing
homogeneity.
The resulting sentence nonsensically
presents the opening phrase as describing
homogeneity.
Correct. The sentence is clear and the
opening phrase correctly modifies members
of the cheetah species.

A

B
C

D

E

The placement of almost makes it
nonsensically modify is expected. What
the relative pronoun which refers to is
ambiguous: for example, does it refer to the
expectation, the possible doubling, or the
year?
The resulting sentence misplaces the adverb
almost.
The phrase production is expected that it
will . . . makes no sense—as opposed to, for
example, it is expected that production will. . . .
Correct. This sentence clearly conveys the
expectations of production: almost to double
and thus to provide. There is no ambiguity as
to what will be providing enough electricity.
The referent of the relative pronoun which
is ambiguous, and the conditional verb form
would thus be providing is unnecessarily
wordy.

The correct answer is D.

The correct answer is E.
798. As sources of electrical power, windmills now account
for only about 2,500 megawatts nationwide, but
production is almost expected to double by the end
of the year, which would provide enough electricity for
1.3 million households.

782

(A)

almost expected to double by the end of the
year, which would provide

(B)

almost expected that it will double by the end of
the year, thus providing

(C)

expected that it will almost double by the end of
the year to provide

(D)

expected almost to double by the end of the
year and thus to provide

(E)

expected almost to double by the end of the
year, which would thus be providing

799. While most of the earliest known ball courts in
Mesoamerica date to 900–400 B.C., waterlogged
latex balls found at El Manati and representations of
ballplayers painted on ceramics found at San Lorenzo
attest to the fact that the Mesoamerican ballgame was
well established by the mid-thirteenth century B.C.
(A)

waterlogged latex balls found at El Manati
and representations of ballplayers painted on
ceramics found at San Lorenzo attest

(B)

waterlogged latex balls found at El Manati and
the painting of representations of ballplayers on
ceramics found at San Lorenzo attests

(C)

waterlogged latex balls found at El Manati
and ceramics painted with representations of
ballplayers found at San Lorenzo attests

(D)

the finding of waterlogged latex balls at El Manati
and the painting of representations of ballplayers
on ceramics found at San Lorenzo attests

9.8 Sentence Correction Answer Explanations

(E)

the finding of waterlogged latex balls at El Manati
and of representations of ballplayers painted on
ceramics at San Lorenzo attest

Logical predication; Agreement
The sentence points out two pieces of evidence
that prove the early existence of ballgames
in Mesoamerica: waterlogged latex balls and
representations of ballplayers. The two noun phrases
together serve as subjects for the verb attest,
creating a sentence that logically and correctly
expresses its main idea.
Correct. Waterlogged latex balls and
representations of ballplayers together serve as
subjects for the verb attest.
B
Painting could refer to something that has
been painted, but on that interpretation the
sentence does not make sense, because the
painting is itself the representation and a
single painting would not plausibly be on
multiple ceramics. Alternatively, it could
refer to an ongoing act of painting, which
could not plausibly constitute the kind of
evidence referred to. The singular verb attests
does not agree with the compound subject
balls and painting.
C The singular verb attests does not agree with
the compound subject balls and ceramics.
D Balls, not finding, should be the subject of
the verb since the balls are the evidence.
Painting could refer to something that has
been painted, but on that interpretation the
sentence does not make sense, because the
painting is itself the representation and a
single painting would not plausibly be on
multiple ceramics. Alternatively, it could
refer to an ongoing act of painting, which
could not plausibly constitute the kind of
evidence referred to. The singular verb attests
does not agree with the compound subject
finding and painting.
E Balls and representations should be the
subject of the verb attest since they are the
evidence—not finding; the plural verb attest
does not agree with the singular subject
finding.
The correct answer is A.

800. As criminal activity on the Internet becomes more and
more sophisticated, not only are thieves able to divert
cash from company bank accounts, they can also pilfer
valuable information such as business development
strategies, new product specifications, and contract
bidding plans, and sell the data to competitors.
(A)

they can also pilfer valuable information such as
business development strategies, new product
specifications, and contract bidding plans, and
sell

(B)

they can also pilfer valuable information that
includes business development strategies, new
product specifications, and contract bidding
plans, and selling

(C)

also pilfering valuable information including
business development strategies, new product
specifications, and contract bidding plans, selling

(D)

but also pilfer valuable information such as
business development strategies, new product
specifications, and contract bidding plans to sell

(E)

but also pilfering valuable information such as
business development strategies, new product
specifications, and contract bidding plans and
selling

A

Grammatical construction; Verb form
The two clauses in this not only . . . construction
normally require subjects, which this sentence
has. The second clause (beginning with they can)
further divides into two clauses about pilfering and
selling; here, the two verbs must have the same
form, since each one is the main verb of its clause.
A

B
C

D
E

Correct. They supplies the needed subject,
and pilfer and sell are both in the bare verb
form.
Selling is the wrong form for the main verb
of a finite clause; it should be sell.
There is no subject for the pilfer clause, and
pilfering would be the wrong form even if a
subject were added.
There is no subject for the pilfer clause.
There is no subject for the pilfer clause, and
both selling and pilfering have the wrong
verb for the main verb of a finite clause;
they should be sell and pilfer, respectively.

The correct answer is A.

783

GMAT® Official Guide 2018

801. Last week local shrimpers held a news conference to
take some credit for the resurgence of the rare Kemp’s
ridley turtle, saying that their compliance with laws
requiring that turtle‐excluder devices be on shrimp nets
protect adult sea turtles.
(A)

requiring that turtle‐excluder devices be on
shrimp nets protect

(B)

requiring turtle‐excluder devices on shrimp nets
is protecting

(C)

that require turtle‐excluder devices on shrimp
nets protect

(D)

to require turtle‐excluder devices on shrimp nets
are protecting

(E)

to require turtle‐excluder devices on shrimp nets
is protecting

(A)

A ruined structure found at Aqaba, Jordan, was
probably a church, as indicated in its eastward
orientation and by its overall plan, as well as

(B)

A ruined structure found at Aqaba, Jordan, once
probably being a church, was indicated by its
eastward orientation, overall plan, and

(C)

Indicating that a ruined structure found at Aqaba,
Jordan, was probably a church were its eastward
orientation and overall plan, but also the

(D)

A ruined structure found at Aqaba, Jordan, was
probably a church, as indicates its eastward
orientation and overall plan, as well as the

(E)

That a ruined structure found at Aqaba, Jordan,
was probably a church is indicated by its
eastward orientation and overall plan, as well as
by the

Rhetorical construction; Agreement

Logical predication; Parallelism

The subject of the clause introduced by saying
that is the singular noun compliance. This subject
requires the singular form of the verb protect. The
clearest, most economical way to describe the
laws in question is to follow the word laws with
a present participle requiring. To use an infinitive,
to require, seems to indicate that requiring these
devices is the objective of the laws, when in fact
the objective is to protect the sea turtles.

This sentence explains why a currently ruined
structure probably used to be a church. In the
best‐worded answer choice, the abstract subject
(the probability that a certain hypothesis is true)
is explained abstractly in a relative clause (That
a ruined structure was probably . . .) followed
by a passive verb (is indicated), followed by the
prepositional phrase (by . . .), which is completed
by a parallel listing of forms of evidence, all
presented as noun phrases in the expression, (by)
(its) A + B, as well as (by) C.

A
B

C

D

E

The plural verb protect does not agree with
the singular subject compliance.
Correct. The singular verb is protecting
agrees with the singular subject compliance,
and the participial phrase beginning with
requiring concisely and accurately describes
the laws.
The relative clause that require introduces
unnecessary wordiness; the plural verb
protect does not agree with the singular
subject compliance.
To require obscures the purpose of the laws;
the plural verb phrase are protecting does not
agree with the singular subject compliance.
To require obscures the purpose of the laws.

A

B

The correct answer is B.
C
802. A ruined structure found at Aqaba, Jordan, was
probably a church, as indicated in its eastward
orientation and by its overall plan, as well as artifacts,
such as glass oil‐lamp fragments, found at the site.
784

This version makes the relationship between
as well as . . . and the rest of the sentence
unclear. The most plausible hypothesis is
that the artifacts are another of the types
of evidence, parallel with the structure’s
orientation and its plan. The sentence
violates the parallelism required in the list
by failing to supply the preposition in the
final item following as well as.
The subject of this version of the sentence,
structure, is completed by the verb was
indicated, creating the illogical assertion that
the structure itself, rather than its probable
identity, was indicated by its orientation,
plan, and attendant artifacts.
Opening the sentence with the predicate
adjective indicating leads to a confusing
and awkward withholding of the subjects
orientation . . . plan . . . artifacts.

9.8 Sentence Correction Answer Explanations

D

E

The only plausible subject for the singular
verb indicates is the plural orientation . . . plan,
as well as the artifacts. Very little about this
sentence makes sense.
Correct. The relationships among the parts
of the sentence are clear and logical.

E

Had been is not the appropriate tense for this
situation.

The correct answer is A.
804. By 1999, astronomers had discovered 17 nearby stars
that are orbited by planets about the size of Jupiter.

The correct answer is E.
803. In the major cities of industrialized countries at the
end of the nineteenth century, important public places
such as theaters, restaurants, shops, and banks had
installed electric lighting, but electricity was in less than
1 percent of homes, where lighting was still provided
mainly by candles or gas.

(A)

had discovered 17 nearby stars that are orbited
by planets

(B)

had discovered 17 nearby stars with planets
orbiting them that were

(C)

had discovered that there were 17 nearby stars
that were orbited by planets

(D)

have discovered 17 nearby stars with planets
orbiting them that are
have discovered that 17 nearby stars are orbited
by planets

(A)

electricity was in less than 1 percent of homes,
where lighting was still

(E)

(B)

electricity was in less than 1 percent of homes
and lighting still

Verb form; Rhetorical construction

(C)

there had been less than 1 percent of homes with
electricity, where lighting was still being

(D)

there was less than 1 percent of homes that had
electricity, having lighting that was still

(E)

less than 1 percent of homes had electricity,
where lighting had still been

Rhetorical construction; Verb form
In this type of usage, a participle such as provided
normally must be preceded by some form of the
verb be. The best choice for this is the past tense
was, since the main part of the clause describes the
situation in the past tense (electricity was . . .). To
link the ideas of lighting and electricity in homes,
where is the most efficient and direct expression,
superior to alternatives such as and or having.
A

B

C

D

Correct. The participle is preceded by the
appropriate form was, and the clauses are
linked efficiently by where.
There is no form of be in the second clause,
and and does not clearly indicate the
connection between the two clauses.
Had been and was being represent
inappropriate tenses, and the there had been
construction is longer than necessary.
Both having lighting and the there was
construction are longer and more complicated
than necessary. This sentence structure makes
there, instead of homes, the subject of having.

Opening with a past date (1999) describing the
end point of a period of discovery, this sentence
calls for a past perfect main verb to follow the
subject astronomers. In order to economize on
words and maximize clarity, the object of the main
clause, stars, is modified by a passive relative clause
that are orbited by planets followed by the adjective
phrase about the size of Jupiter. This structure avoids
an awkward and confusing string of relative clauses
and prepositional phrases.
A
B

C
D

E

Correct. This version is clear and uses the
correct verb form had discovered.
The use of a prepositional phrase with planets
necessitates the introduction of a relative clause
that were . . ., in which the referent of the
relative pronoun that is somewhat uncertain
(stars? or planets?). The past tense verb were
suggests, improbably, that the size of the planets
may have changed significantly since 1999.
The string of relative clauses is awkward and
wordy.
If the sentence was written after 1999, the
present perfect tense is illogical, because 1999
is in the past. If it was written in 1999, this
way of referring to the then‐present time is
odd and misleading. The prepositional phrase
is wordy and indirect.
As in (D), the present perfect tense is illogical.

The correct answer is A.
785

GMAT® Official Guide 2018

805. Although she was considered among her
contemporaries to be the better poet than her
husband, later Elizabeth Barrett Browning was
overshadowed by his success.
(A)

(B)

(C)

(D)

(E)

Although she was considered among her
contemporaries to be the better poet than her
husband, later Elizabeth Barrett Browning was
overshadowed by his success.
Although Elizabeth Barrett Browning was
considered among her contemporaries as a
better poet than her husband, she was later
overshadowed by his success.

C

786

The correct answer is D.
806. In no other historical sighting did Halley’s Comet
cause such a worldwide sensation as did its return in
1910–1911.
(A)

did its return in 1910–1911

(B)

had its 1910–1911 return

(C)

in its return of 1910–1911

(D)

its return of 1910–1911 did

(E)

its return in 1910–1911

Although Elizabeth Barrett Browning’s success
was later overshadowed by that of her husband,
among her contemporaries she was considered
the better poet.

Parallelism; Verb form; Logical predication

Elizabeth Barrett Browning’s poetry was
considered among her contemporaries as better
than her husband, but her success was later
overshadowed by his.

The sentence misuses the locution the better poet.
It is acceptable to say a better poet than but not
the better poet than. If you have already mentioned
two poets X and Y, and you want to say that X is
better than Y, you can either say X is a better poet
than Y, or simply, X is the better poet.

B

E

Correct. This version avoids the problems
of the other versions.
This sentence comically compares Elizabeth
Barrett Browning’s poetry to her husband.

Later overshadowed by the success of her
husband, Elizabeth Barrett Browning’s poetry had
been considered among her contemporaries to
be better than that of her husband.

Idiom; Verb form; Logical predication

A

D

This version of the sentence misuses the
idiom the better poet, most likely confusing it
with the idiom a better poet than.
[W]as considered among her contemporaries
as is awkward and unnecessarily creates a
potential misreading (she was considered to
be among her contemporaries).
The subject here is Elizabeth Barrett
Browning’s poetry, not Elizabeth Barrett
Browning, so the antecedent of her in
her husband could comically be taken to
be Elizabeth Barrett Browning’s poetry.
Furthermore, the tense of had been considered
pairs oddly with Later overshadowed. Each
seems to be in reaction to some particular time
T—later than T and earlier than T, but when
T was or its significance is never indicated.

The single subject of this sentence is Halley’s
Comet, and its single verb phrase is did cause.
The comparison presented by the sentence is
between adverbial phrases describing times when
the comet was seen. Grammatically, the items
being compared are parallel prepositional phrases
beginning with the preposition in: in no other
sighting and in its return in 1910–1911. This is the
clearest, most economical way of presenting the
information. The options that introduce a second
verb (did or had) violate the parallelism and
introduce a comparison between the comet itself
(subject of the verb did cause) and the comet’s
return (subject of the verb did or had).
A
B

C

D
E

This sentence implies a comparison between
the comet and its return.
This sentence implies a comparison between
the comet and its return; had is the wrong
auxiliary verb form because it must be
followed by caused instead of cause.
Correct. The parallel prepositional phrases
in this sentence correctly compare times
when the comet was sighted.
This sentence implies a comparison between
the comet and its return.
This sentence violates parallelism, implying
a comparison between a prepositional
phrase and a noun phrase.

The correct answer is C.

9.8 Sentence Correction Answer Explanations

807. Rock samples taken from the remains of an asteroid
about twice the size of the 6‐mile‐wide asteroid that
eradicated the dinosaurs has been dated to be 3.47
billion years old and thus is evidence of the earliest
known asteroid impact on Earth.
(A)

has been dated to be 3.47 billion years old and
thus is

(B)

has been dated at 3.47 billion years old and thus

(C)

have been dated to be 3.47 billion years old and
thus are

(D)

have been dated as being 3.47 billion years old
and thus

(E)

A
B

C
D
E

The subject and verbs do not agree; dated to
be … is not idiomatic.
The subject and verb do not agree; the
conjunction and thus should be followed by
a verb.
Dated to be is not idiomatic.
As being is not idiomatic; the conjunction
and thus should be followed by a verb.
Correct. The plural verbs match the plural
subject, and the wording of the sentence is
idiomatic.

The correct answer is E.

have been dated at 3.47 billion years old and
thus are

Agreement; Idiom
The plural subject of this sentence, Rock samples,
requires plural verb phrases—have been dated
and are rather than has been dated and is. The
idiomatic way of expressing estimation of age is
with the phrase dated at.

787

10.0 Integrated Reasoning

788

10.0 Integrated Reasoning

10.0 Integrated Reasoning
The Integrated Reasoning section measures your ability to understand and evaluate multiple sources
and types of information—graphic, numeric, and verbal—as they relate to one another; use quantitative
and verbal reasoning to solve complex problems; and solve multiple problems in relation to one another.
This section includes text passages, tables, graphs, and other visual information from a variety of content
areas; however, the materials and questions do not assume detailed knowledge of the topics discussed.
The Integrated Reasoning section differs from the Quantitative and Verbal sections in two important
ways: 1) It involves both mathematical and verbal reasoning, either separately or in combination,
and 2) questions are answered using four different response formats rather than only traditional
multiple‐choice.
Four types of questions are used in the Integrated Reasoning section:
• Multi‐Source Reasoning
• Table Analysis
• Graphics Interpretation
• Two‐Part Analysis
Use your unique access code found in the back of this book to access 50 additional Integrated
Reasoning practice questions with full answer explanations.

10.1 What Is Measured
Integrated Reasoning questions assess your ability to apply, evaluate, infer, recognize, and strategize.

Apply concepts presented in the information
Apply questions measure your ability to understand principles, rules, or other concepts in the
information provided and apply them to a new context or predict consequences that would follow if
new information were incorporated into the context provided. You may be asked to
• decide whether new examples would comply with or violate rules established in the
information provided
• determine how a trend present in the information provided would be affected by new scenarios
• use principles established in the information provided to draw conclusions about new data

Evaluate information qualitatively
Evaluate questions measure your ability to make judgments about the quality of information. For
example, you may be asked to
• decide whether a claim made in one source is supported or undermined by information provided
in another source
• determine whether the information provided is sufficient to justify a course of action

789

GMAT® Official Guide 2018

• judge the strength of evidence offered in support of an argument or plan
• identify errors or gaps in the information provided

Draw inferences from the information
Infer questions ask about information or ideas that are not explicitly stated in the materials provided but
can be derived from them. For example, you may be asked to
• calculate the probability of an outcome on the basis of given data
• indicate whether statements follow logically from the information provided
• determine the meaning of a term within the context in which it is used
• identify the rate of change in data gathered over time

Recognize parts or relationships in the information
Recognize questions measure your ability to identify information that is directly presented in the
materials provided, including specific facts or details and relationships between pieces of information.
For example, you may be asked to
• identify areas of agreement and disagreement between sources of information
• determine the strength of correlation between two variables
• indicate which element in a table has a given rank in a combination of categories
• identify facts provided as evidence in an argument

Make strategic decisions or judgments based on the information
Strategize questions ask about the means of achieving a goal within the context of particular needs or
constraints. For example, you may be asked to
• choose a plan of action that minimizes risks and maximizes value
• identify tradeoffs required to reach a goal
• specify the mathematical formula that will yield a desired result
• determine which means of completing a task are consistent within given constraints

10.2 The Question Types
The four Integrated Reasoning question types are described in detail below.

Multi-Source Reasoning
Multi‐Source Reasoning questions begin with two or three sources of information, each labeled with a
tab, which appear on the left side of a split computer screen. One or more of the sources will contain a
written passage. The other sources may be tables, graphs, diagrams, or other types of visual information.
Only one source of information will be displayed at a time. To view a different source, select its tab from
those that appear above the source which is currently displayed.

790

10.2 Integrated Reasoning The Question Types

The sources of information are accompanied by questions that will ask you to synthesize, compare,
interpret, or apply the information presented. As each question associated with the sources appears in
turn on the right side of the screen, the initial source will appear again on the left side. You can click on
the tabs to view any of the sources as many times as needed. However, you will see only one question at
a time and cannot go back to earlier questions.
There are two question formats for Multi‐Source Reasoning:
• Multiple‐choice questions
• Multiple‐dichotomous choice questions
For multiple‐choice questions, select the best of the five answer choices given. Read each question and
series of answer choices carefully. Make sure you understand exactly what the question is asking and
what the answer choices are.
Multiple‐dichotomous choice questions provide three phrases, statements, numerical values, or algebraic
expressions that require an indication as to whether each meets a certain condition. For example, you
may be asked whether
• each statement is true, according to the sources
• each statement or numerical value is consistent with the sources
• each statement or algebraic expression would solve a problem described in the sources
• the value of each algebraic expression can be determined on the basis of the sources
In answering both kinds of Multi‐Source Reasoning questions, be aware of the information from each
source provided and try to determine the process that works best for you. One strategy is to examine
the sources carefully and thoroughly, another is to skim the sources the first time through, or to read the
first question before examining the sources. Read each question carefully and make sure you understand
exactly what the question is asking. If necessary, go back to the sources to review relevant information.
You will have 30 minutes to complete the Integrated Reasoning section, or an average of 2 minutes and
30 seconds to answer each multiple‐choice or multiple‐dichotomous choice question. Keep in mind that
you will need time to examine the source materials that accompany the questions—and that this time
must be factored into the per‐question average.

Table Analysis
Table Analysis questions present a table similar to a spreadsheet. It can be sorted on any of its columns
by selecting the column’s title from a drop‐down menu. There may be a brief text explaining the table
or providing additional information. The question then presents three phrases, statements, numerical
values, or algebraic expressions, and you must indicate for each one whether or not it meets a certain
condition. For example, you may be asked whether
• each statement is true (yes or no), according to the information in the table
• each statement or numerical value is consistent or inconsistent with the information in the table
• each statement or numerical value can or cannot be determined on the basis of the information in
the table

791

GMAT® Official Guide 2018

Read the question thoroughly to make sure you understand what is being asked. Then consider each
phrase, statement, numerical value, or algebraic expression to learn what information in the table you
need to make your decision. For example, in analyzing the table, you may need to,
• determine statistics such as mean, median, mode, or range
• determine ratios, proportions, or probabilities
• identify correlations between two sets of data
• compare an entry’s rank in two or more of the table’s categories
You will have 30 minutes to complete the Integrated Reasoning section, or an average of 2 minutes and
30 seconds to answer each question. Keep in mind that each Table Analysis question has three parts
that all need to be answered in the time allowed.

Graphics Interpretation
Graphics Interpretation questions present a graph, diagram, or other visual representation of
information, followed by one or more statements containing a total of two blanks. The blanks should be
filled in with the option from each drop‐down menu in order to create the most accurate statement or
statements on the basis of the information provided.
Many of the graphs included in Graphics Interpretation questions involve two variables plotted on
vertical and horizontal axes. Graphs of this type include bar graphs, line graphs, scatterplots, and bubble
graphs. To read these graphs, determine what information is represented on each axis. Do this by
carefully examining any information that may be provided, including labels on the axes, scales on the
axes, the title of the graph, and accompanying text. To find the value of a data point on the graph,
determine the corresponding values on the horizontal and vertical axes.
In the simple bar graph below, the first bar indicates that 7 units were sold on Monday of Week 1.

Units Sold in Week 1

Units Sold

10
8
6
4
2

792

ay
id
Fr

ay
sd
Th
ur

ay
sd
W
ed
ne

y
da
Tu
es

M

on

da
y

0

10.2 Integrated Reasoning The Question Types

The same information is presented below as a line graph. Each point indicates the total number of units
sold on a given day. The slope of the line connecting the points shows how the sales changed over time;
a positive slope indicates that sales increased from the previous day, and a negative slope indicates that
sales decreased.

Units Sold in Week 1

Units Sold

10
8
6
4
2

ay

y

y
Th

ur

Fr
id

sd
a

sd
a
ne
W
ed

M

Tu
e

on

sd
a

da

y

y

0

A third variable can be indicated with an additional vertical axis. In the following graph, the bars
indicate the number of units sold on each day, which corresponds to the scale on the left axis. The line
graph shows what percent of the total units were sold on each day. The scale for the percentages is
shown on the right axis.

units sold

18

4

12

2

6

0

0

id
Fr

sd
Th
ur

sd
W
ed
ne

da
Tu
es

on
M

ay

6

ay

24

ay

8

y

30
percent of total

percent of total

10

da
y

Units Sold

Units Sold in Week 1

In a scatterplot, each dot is a single data point. In the scatterplot at the top of the next page, each dot
represents a type of computer product available for purchase. A dot’s position relative to the vertical axis
indicates the product’s price, and its position relative to the horizontal axis indicates its weight. Thus,
the product that weighs 1.0 kg costs approximately 32,000 rupees.

793

GMAT® Official Guide 2018

price in rupees

Products for Sale by Price and Weight
35,000
30,000
25,000
20,000
15,000
10,000
0.5

1.0

1.5
2.0
weight in kg

2.5

3.0

Some scatterplots include a trend line—usually a least squares regression line—that shows the trend of
the data. A trend line with positive slope indicates a positive correlation between the two variables,
and a trend line with negative slope indicates a negative correlation. Thus, in the scatterplot below,
the trend line indicates a negative correlation between price and weight among the products
represented on the graph. The closer the data points are to a trend line, the more strongly the data are
correlated.

price in rupees

Products for Sale by Price and Weight
35,000
30,000
25,000
20,000
15,000
10,000
0.5

1.0

1.5
2.0
weight in kg

2.5

3.0

In a bubble graph, each data point is represented by a circle. The center of the circle indicates the
values on the horizontal and vertical axes, as in a scatterplot. The relative size of a circle introduces a
third variable, number sold. In the bubble graph below, the relative size of the circles indicates how
many of the products were sold. Thus, the number of the lightest product sold was smaller than that
of any other product shown, and the number of the heaviest product sold was greater than that of any
other product shown.

Products for Sale by Price, Weight, and Number Sold
price in rupees

35,000
30,000
25,000
20,000
15,000
0.9

794

1.1

1.3
1.5
weight in kg

1.7

1.9

10.2 Integrated Reasoning The Question Types

Other common kinds of graphics do not use vertical and horizontal axes. These include pie charts, flow
charts, and organization charts.
A pie chart uses a circle divided into sectors to show what percent of the whole is represented by each
component part. The circle represents the whole, and the relative size of each sector indicates its percent
of the whole. Since the whole is 100%, the sum of the percentages of all the sectors is 100 (plus or
minus a bit to account for rounding). Below is a pie chart created from the data used in the bar graph
and line graph shown earlier. In this example, labels indicating the exact percents are not supplied, but it
is still possible to gauge the size of the sectors relative to one another: the smallest percent of sales was
on Thursday, and the largest was on Tuesday. In addition, the two radii that mark the boundaries of the
Tuesday sector form an obtuse angle, which indicates that the sector is greater than one‐fourth of the
circle. Thus, Tuesday’s sales comprised more than 25% of total sales.
Week 1 Sales by Day
Friday
Monday
Thursday

Wednesday

Tuesday

A flow chart is a diagram that shows the steps in a process. Often, the steps are represented by symbols,
which are connected by arrows showing the flow of the process. Flow charts generally progress from
top to bottom or from left to right. In the simple flow chart below, rectangles indicate steps to be
completed. The diamond shape indicates a decision point: if the consultant is new, the process continues
to the next step, Append tax forms. If the consultant is not new, that step is bypassed and the contract is
mailed.
Yes

Prepare
contract for
consultant

New
consultant?

Append
tax forms

Mail
contract

No

An organization chart represents the structure of an organization. Often, hierarchical relationships are
shown with vertical lines and lateral relationships are shown with horizontal lines. In the organization
chart at the top of the next page, each rectangle represents an employee or group of employees at a
small restaurant. All the employees in the second row of rectangles report to the restaurant manager. In
the third row, the food preparation staff and cleaning staff both report to the kitchen manager, and the
serving staff report to the serving staff manager.
795

GMAT® Official Guide 2018

restaurant
manager

kitchen
manager

chef

food preparation
staff

serving staff
manager

cleaning
staff

serving
staff

For all kinds of Graphics Interpretation questions, be sure to read the question carefully to be sure you
understand what is being asked. Then read the statement or statements to determine what information
you need to learn from the graphic. Finally, choose the answer from each drop‐down menu that best
completes the statement or statements.
You will have 30 minutes to complete the Integrated Reasoning section, or an average of 2 minutes and
30 seconds to answer each question. Keep in mind, however, that each Graphics Interpretation question
has two blanks to be filled. Both blanks must be filled using the drop‐down menus in the time allowed.

Two-Part Analysis
Two‐Part Analysis questions present a brief written scenario or problem and ask you to make two
choices related to that information. These choices are connected to each other in some way; for example,
they might be two steps involved in solving a problem or two components required to successfully
complete a task. In Two‐Part Analysis questions you may be asked to, for example,
• calculate the proportions of two different components in a mixture
• determine something that would be lost and something that would be gained in a trade‐off
• find the maximum number of two different products that could be purchased within a certain
budget
• identify a first action and a second action that together would bring a company into compliance
with a new rule
The possible answers and your choices will be given in a table format. The possible answers are listed
in the third column, on the right side of the table. Your choices for the first part and second part of
the question will be recorded in the first and second columns of the table, respectively. Remember that
you need to make a choice for each of the first two vertical columns of the table—not one for each
horizontal row.
In answering Two‐Part Analysis questions, read the scenario or problem carefully. Be sure you
understand what the question is asking. Read all the answer options to be sure that your choices are the
best of those available, and be careful to mark your choices in the proper columns.
You will have 30 minutes to complete the Integrated Reasoning section, or an average of 2 minutes
and 30 seconds to answer each question. Keep in mind that you must make the two choices for each
Two‐Part Analysis question within that average amount of time.

796

10.3 Integrated Reasoning Test-Taking Strategies

10.3 Test-Taking Strategies
Multi-Source Reasoning Questions
1.

Do not expect to be completely familiar with the material presented in Multi‐Source
Reasoning sets.

You may find some graphs, charts, tables, or verbal passages easier to understand than others. All
of the material is designed to be challenging, but if you have familiarity with the subject matter, do
not let this knowledge influence your answer choices. Answer all questions on the basis of what is
given by the various sources of information.
2.

Analyze each source of information carefully, because the questions require a detailed
understanding of the information presented.

Text passages often build ideas sequentially, so note as you read how each statement adds to the
main idea of the passage as a whole. Some of the passages used with Multi‐Source Reasoning
items will be purely descriptive while others may contain strong opinions.
Given that the graphic elements of Multi‐Source Reasoning items come in various forms—such
as tables, graphs, diagrams, or charts—briefly familiarize yourself with the information presented.
If scales are provided, note the marked values and labels. Also note the major graphical elements of
the information presented.
3.

Read the questions carefully, making sure you understand what is being asked.

Some of the questions will require you to recognize discrepancies among different sources of
information, others will ask you to draw inferences using information from different sources, while
others may require you to determine which one of the information sources is relevant. You can
refer back to any of the sources at any time while you are answering the Multi‐Source Reasoning
questions.
4.

Select the answer choices that have the most support based on the information provided.

You may find it helpful to briefly familiarize yourself with the overall information given in the
sources and then to focus more closely on the specific information needed to answer the question.

Table Analysis Questions
1.

Examine the table and accompanying text to determine the type of information provided.

Orienting yourself to the data at the outset will make it easier to locate the information necessary
for completing the question.
2.

Read the question carefully.

The question will contain the condition that each phrase, statement, numerical value, or algebraic
expression does or does not meet (for example, is or is not consistent with the information provided,
or can or cannot be inferred from the information provided). Clearly understanding the condition will
help you to clarify the choice to be made in each case.

797

GMAT® Official Guide 2018

3.

Read each phrase, statement, numerical value, or algebraic expression carefully to determine the
data analysis required.

Often, the phrase, statement, numerical value, or algebraic expression indicates a relationship that
can be clarified by sorting the table on one or more of its columns. Careful reading can help you
work more efficiently by using table sorts strategically to identify data of interest.
4.

Judge each phrase, statement, numerical value, or algebraic expression carefully on the basis of
the condition specified.

For each phrase, statement, numerical value, or algebraic expression, the two answer choices (such
as yes or no, true or false, consistent or inconsistent) are mutually exclusive. Thus, you can focus your
attention on whether or not the given condition has been met.

Graphics Interpretation Questions
1.

Read the graphic carefully.

Quickly familiarize yourself with the information presented in the graphic. If scales are provided
(such as on the axes) make note of the marked values. If there are labels, be sure to note any
discrepancy between the units in the graph and the units discussed in the text.
2.

Read any accompanying text carefully.

If there is accompanying text, it may clarify the meaning of the graphic. Text might also present
information that is not contained in the graphic but that is necessary for answering the question.
3.

Scan the choices in the drop‐down menu before you do any work.

Some statements could be completed equally well with very general responses as with very specific
responses. Checking the menu options gives you additional information about the task involved.
4.

Choose the option that best completes the statement.

More than one option in a drop‐down menu may seem plausible to you; in each menu, choose
the one that makes the statement most accurate or logical. If the drop‐down menu is preceded
by a phrase such as “nearest to” or the “closest to,” choose the option that is closest to the exact
answer you compute. You may find that reading the entire statement again with your answer
choice in place is a helpful way to check your work.

Two-Part Analysis Questions
1.

Read the information given carefully.

All of the material presented is designed to be challenging, but if you have familiarity with the
subject matter, do not let this knowledge influence your answer choices. Answer each question only
on the basis of what is given.
2.

Determine exactly what the question is asking.

Do not assume that the headings in the two response columns are complete descriptions of the
tasks to be performed. Pay close attention to how the question describes the tasks. Often the
headings in the two response columns are shorthand references to the tasks and may lack some
details that could help you to better understand what you are supposed to do.
3.

Remember that only two choices are to be made.

Select one answer in each of the first two columns of the response table. You do not need to
make a choice for each row of the table. The third column contains possible answers for the
two choices to be made.

798

10.4 Integrated Reasoning The Directions

4.

Do not choose an answer before reviewing all of the available answer choices.

Do not assume that you have chosen the best answers in the two columns without reading all of
the available options.
5.

Determine whether tasks are dependent or independent.

Some Two‐Part Analysis questions pose two independent tasks that can be carried out individually,
and others pose one task with two dependent parts, each of which must be carried out correctly to
create a single correct response. With questions of the dependent type, the question asked cannot
be answered coherently without making both choices, so be sure to examine your answers in
relation to one another.
6.

Keep in mind that one answer choice can be the correct response for both columns.

If the tasks associated with the two response columns are not mutually exclusive, it is possible that
one answer choice satisfies the conditions associated with both response columns.

10.4 The Directions
These directions are similar to the directions given for the four question types in the Integrated
Reasoning section of the GMAT® exam. Understanding them clearly before taking the test will save
you time during the test.
• Multi‐Source Reasoning. Click on the tabs and examine all the relevant information from text,
charts, and tables to answer the questions.
• Table Analysis. Analyze the table, sorting on columns as needed, to determine whether each of
the options presented meets the given criterion or not.
• Graphics Interpretation. Interpret the graph or graphical image and select from each drop‐down
menu the option that creates the most accurate statement based on the information provided.
• Two‐Part Analysis. Read the information provided, review the options presented in the table, and
indicate which option meets the criterion presented in the first column and which option meets
the criterion presented in the second column. Make only two selections, one in each column.
For the Integrated Reasoning section, an onscreen calculator is available. To access the calculator, click
“Calculator” on the blue bar at the upper left of the screen. Note that the calculator can be dragged to
any part of the screen.
You can view explanations of the format of the specific Integrated Reasoning questions anytime while
working through this section by clicking on HELP.

799

11.0 Analytical Writing Assessment

800

11.0 Analytical Writing Assessment

11.0 Analytical Writing Assessment
The Analytical Writing Assessment (AWA) consists of one 30‐minute writing task called the Analysis
of an Argument. In this section, you must read a brief argument, analyze the reasoning behind it, and
then write a critique of the argument. You are not asked to state your opinion but rather to analyze
the one given. For example, you may, consider what questionable assumptions underlie the author’s
thinking, what alternative explanations or counterexamples might weaken the conclusion, or what sort
of evidence could help strengthen or refute the argument.
For this task, you will use the computer keyboard to type your response. You will be able to use typical
word‐processing functions—that is, you can cut, copy, paste, undo, and redo. These functions can be
accessed either by using the keyboard or by using the mouse to click on icons on the screen. You will be
able to take notes when planning your response.
It is important that you plan carefully before you begin writing. Read the specific analytical writing
task several times to make sure you understand exactly what is expected. Think about how you might
present your analysis. You may want to sketch an outline to help you plan and organize. Keep in mind
the 30‐minute time limit as you plan your response—keep your analysis brief enough to allow for plenty
of time to write a first draft, read it over carefully, and make any necessary corrections or revisions before
you run out of time. As you write, try to keep your language clear, your sentences concise, and the flow
of your ideas logical. State your premise clearly at the beginning, and make sure you present a strong
conclusion at the end.

11.1 What Is Measured
The Analytical Writing Assessment is designed as a direct measure of your ability to think critically
and communicate your ideas. More specifically, the Analysis of an Argument task tests your ability to
formulate an appropriate and constructive critique of a prescribed conclusion based upon a specific line
of thinking.
The argument that you will analyze may concern a topic of general interest, possibly related to business,
or to a variety of other subjects. It is important to note, however, that no Analysis of an Argument
question presupposes any specific knowledge of business or other specific content areas. Only your
capacity to write analytically is assessed.
Professional essay raters, including college and university faculty members from various subject‐matter
areas, including but not limited to management education, will evaluate your essay. For information on
how readers are qualified, visit mba.com. Readers are trained to be sensitive and fair in evaluating the
responses of nonnative speakers of English. A computer scoring program will also evaluate your essays.
Your responses will be scored on the basis of:

•
•
•
•

the overall quality of your ideas
your ability to organize, develop, and express those ideas
how well you provide relevant supporting reasons and examples
your ability to control the elements of standard written English

801

GMAT® Official Guide 2018

11.2 Test-Taking Strategies
1. Read the question carefully.
Make sure you have taken all parts of a question into account before you begin to respond to it.
2. Do not start to write immediately.
Take a few minutes to think about the question and plan a response before you begin writing. You
may find it helpful to write a brief outline or jot down some ideas on the erasable notepad provided.
Take care to organize your ideas and develop them fully, but leave time to reread your response and
make any revisions that you think would improve it.
3. Focus on the task of analyzing and critiquing a line of thinking or reasoning.
Get used to asking yourself questions such as the following: What questionable assumptions might
underlie the thinking? What alternative explanations might be given? What counterexamples might be
raised? What additional evidence might prove useful in fully and fairly evaluating the reasoning?
4. Develop fully any examples you use.
Do not simply list your examples—explain how they illustrate your point.
5. Discuss alternative explanations or counterexamples.
These techniques allow you to introduce illustrations and examples drawn from your observations,
experiences, and reading.
6. Make sure your response reads like a narrative.
Your response should not read like an outline. It should use full sentences, a coherent organizational
scheme, logical transitions between points, and appropriately introduced and developed examples.

11.3 The Directions
These are the directions that you will see for the Analysis of an Argument essay. If you read them
carefully and understand them clearly before going to sit for the test, you will not need to spend too
much time reviewing them when you take the GMAT® exam. They read as follows:

ANALYSIS OF AN ARGUMENT
In this section, you will be asked to write a critique of the argument presented. You are not asked to
present your own views on the subject.
Writing Your Response: Take a few minutes to evaluate the argument and plan a response before you
begin writing. Be sure to leave enough time to reread your response and make any revisions that you
think are necessary.
Evaluation of Your Response: Scores will reflect how well you:

• organize, develop, and express your ideas about the argument presented
• provide relevant supporting reasons and examples
• control the elements of standard written English

802

11.4 Analytical Writing Assessment GMAT® Scoring Guide: Analysis of an Argument

11.4 GMAT® Scoring Guide: Analysis of an Argument
6 Outstanding
A 6 paper presents a cogent, well‐articulated critique of the argument and demonstrates mastery of the
elements of effective writing.
A typical paper in this category exhibits the following characteristics:

•
•
•
•
•

clearly identifies important features of the argument and analyzes them insightfully
develops ideas cogently, organizes them logically, and connects them with clear transitions
effectively supports the main points of the critique
demonstrates control of language, including diction and syntactic variety
demonstrates facility with the conventions of standard written English but may have minor flaws

5 Strong
A 5 paper presents a well‐developed critique of the argument and demonstrates good control of the
elements of effective writing.
A typical paper in this category exhibits the following characteristics:

• clearly identifies important features of the argument and analyzes them in a generally thoughtful
way

•
•
•
•

develops ideas clearly, organizes them logically, and connects them with appropriate transitions
sensibly supports the main points of the critique
demonstrates control of language, including diction and syntactic variety
demonstrates facility with the conventions of standard written English but may have occasional
flaws

4 Adequate
A 4 paper presents a competent critique of the argument and demonstrates adequate control of the
elements of writing.
A typical paper in this category exhibits the following characteristics:

•
•
•
•
•

identifies and analyzes important features of the argument
develops and organizes ideas satisfactorily but may not connect them with transitions
supports the main points of the critique
demonstrates sufficient control of language to convey ideas with reasonable clarity
generally follows the conventions of standard written English but may have some flaws

803

GMAT® Official Guide 2018

3 Limited
A 3 paper demonstrates some competence in analytical writing skills and in its control of the elements
of writing but is plainly flawed.
A typical paper in this category exhibits one or more of the following characteristics:

• does not identify or analyze most of the important features of the argument, although some analysis of the argument is present

•
•
•
•
•

mainly analyzes tangential or irrelevant matters, or reasons poorly
is limited in the logical development and organization of ideas
offers support of little relevance and value for points of the critique
does not convey meaning clearly
contains occasional major errors or frequent minor errors in grammar, usage, and mechanics

2 Seriously Flawed
A 2 paper demonstrates serious weaknesses in analytical writing skills.
A typical paper in this category exhibits one or more of the following characteristics:

• does not present a critique based on logical analysis, but may instead present the writer’s own
views on the subject

•
•
•
•

does not develop ideas, or is disorganized and illogical
provides little, if any, relevant or reasonable support
has serious and frequent problems in the use of language and in sentence structure
contains numerous errors in grammar, usage, and mechanics that interfere with meaning

1 Fundamentally Deficient
A 1 paper demonstrates fundamental deficiencies in analytical writing skills.
A typical paper in this category exhibits more than one of the following characteristics:

•
•
•
•

provides little evidence of the ability to understand and analyze the argument
provides little evidence of the ability to develop an organized response
has severe and persistent errors in language and sentence structure
contains a pervasive pattern of errors in grammar, usage, and mechanics that results in incoherence

0 No Score
A paper in this category is off topic, not written in English, is merely attempting to copy the topic, or
consists only of keystroke characters.

NR Blank

804

11.5 Analytical Writing Assessment Sample: Analysis of an Argument

11.5 Sample: Analysis of an Argument
Read the statement and the instructions that follow it, and then make any notes that will help you
plan your response.

The following appeared as part of an article in a daily newspaper:
“The computerized on‐board warning system that will be installed in commercial airliners will virtually
solve the problem of midair plane collisions. One plane’s warning system can receive signals from
another’s transponder—a radio set that signals a plane’s course—in order to determine the likelihood of
a collision and recommend evasive action.”
Discuss how well reasoned you find this argument. In your discussion, be sure to analyze the line
of reasoning and the use of evidence in the argument. For example, you may need to consider what
questionable assumptions underlie the thinking and what alternative explanations or counterexamples
might weaken the conclusion. You can also discuss what sort of evidence would strengthen or refute the
argument, what changes in the argument would make it more logically sound, and what, if anything,
would help you better evaluate its conclusion.

Sample Paper 6
The argument that this warning system will virtually solve the problem of midair plane collisions
omits some important concerns that must be addressed to substantiate the argument. The statement
that follows the description of what this warning system will do simply describes the system and how
it operates. This alone does not constitute a logical argument in favor of the warning system, and it
certainly does not provide support or proof of the main argument.
Most conspicuously, the argument does not address the cause of the problem of midair plane collisions,
the use of the system by pilots and flight specialists, or who is involved in the midair plane collisions.
First, the argument assumes that the cause of the problem is that the planes’ courses, the likelihood
of collisions, and actions to avoid collisions are unknown or inaccurate. In a weak attempt to support
its claim, the argument describes a system that makes all of these things accurately known. But if the
cause of the problem of midair plane collisions is that pilots are not paying attention to their computer
systems or flight operations, the warning system will not solve the collision problem. Second, the
argument never addresses the interface between individuals and the system and how this will affect
the warning system’s objective of obliterating the problem of collisions. If the pilot or flight specialist
does not conform to what the warning system suggests, midair collisions will not be avoided. Finally,
if planes other than commercial airliners are involved in the collisions, the problem of these collisions
cannot be solved by a warning system that will not be installed on non‐commercial airliners. The
argument also does not address what would happen in the event that the warning system collapses, fails,
or does not work properly.
Because the argument leaves out several key issues, it is not sound or persuasive. If it included the items
discussed above instead of solely explaining what the system supposedly does, the argument would have
been more thorough and convincing.

805

GMAT® Official Guide 2018

Explanation of Score 6
This response is, as the scoring guide requires of a 6, “cogent”and “well articulated”: all the points
made not only bear directly on the argument to be analyzed, but also contribute to a single, integrated
development of the writer’s critique. The writer begins by making the controlling point that a mere
description of the warning system’s mode of operation cannot serve as a true argument proving
the system’s effectiveness, since the description overlooks several major considerations. The writer
then identifies these considerations—what causes midair collisions, how pilots will actually use the
commercial airline warning system, what kinds of airplanes are typically involved in midair collisions—
and, citing appropriate counterexamples (e.g., what if pilots do not pay attention to their instruments?),
explains fully how each oversight undermines the conclusion that the warning system will virtually
eliminate midair plane collisions.
Throughout, the writer complements the logically organized development of this critique with good,
clear prose that demonstrates the ability not only to control language and vary sentence structure but
also to express ideas forcibly (e.g., “the argument never addresses the interface between individuals and
the system”). Of course, as in any response written under time constraints, occasional minor flaws can
be found. For example, “the argument assumes that the cause of the problem is that the planes’ courses,
the likelihood of collisions, and actions to avoid collisions are unknown or inaccurate” is wordy and
imprecise: how can a course, a likelihood, or actions be inaccurate? But flaws such as these, minor and
infrequent, do not interfere with the overall clarity and forcefulness of this outstanding response.

806

11.5 Analytical Writing Assessment Sample: Analysis of an Argument

Sample Paper 4
The argument is not logically convincing. It does not state whether all planes can receive signals from
each other. It does not state whether planes constantly receive signals. If they only receive signals once
every certain time interval, collisions will not definitely be prevented. Further if they receive a signal
right before they are about to crash, they cannot avoid each other.
The main flaw in the argument is that it assumes that the two planes, upon receiving each other’s
signals, will know which evasive action to take. For example, the two planes could be going towards each
other and then receive the signals. If one turns at an angle to the left and the other turns at an angle to
the right, the two planes will still crash. Even if they receive an updated signal, they will not have time
to avoid each other.
The following argument would be more sound and persuasive. The new warning system will solve the
problem of midair plane collisions. Each plane will receive constant, continual signals from each other.
If the two planes are headed in a direction where they will crash, the system will coordinate the signals,
and tell one plane to go one way, and the other plane to go another way. The new system will ensure that
the two planes will turn in different directions so they don’t crash by trying to prevent the original crash.
In addition, the two planes will be able to see themselves and the other on a computer screen, to aid in
the evasive action.

Explanation of Score 4
This response competently cites a number of deficiencies in the argument presented: the information
given about the nature of the signals sent and received and the evasive action recommended does not
warrant the conclusion that the onboard warning system “will virtually solve the problem of midair
plane collisions.” However, in discussing these insufficiencies in the argument, the response reveals an
unevenness in the quality of its reasoning. For example, while it is perfectly legitimate to point out that
the argument assumes too much and says too little about the evasive action that will be recommended
by the warning system, it is farfetched to suggest that the system might be so poorly designed as to
route two approaching airplanes to the same spot. Likewise, while it is fair to question the effectiveness
of a warning signal about which the argument says so little, it is not reasonable to assume that the
system would be designed to space signals so far apart that they would prove useless. Rather than
invent implausibly bad versions of the warning system to prove that it might be ineffective, a stronger
response would analyze unexplored possibilities inherent in the information that is given—for example,
the possibility that pilots might not be able to respond quickly and effectively to the radio signals the
argument says they will receive when the new system is installed. The “more sound and persuasive
argument” in the last paragraph, while an improvement on the original, continues to overlook this
possibility and also assumes that other types of aircraft without transponders will pose no problems.
The organization of ideas, while generally sound, is sometimes weakened by needless repetition of the
same points, as in sentences 4 and 5 of the last paragraph. The writing contains minor instances of
awkwardness (e.g., “Each plane will receive constant, continual signals from each other” in paragraph 3),
but is free of flaws that make understanding difficult. However, though the writing is generally clean
and clear, the syntax does not show much variety. A few sentences begin with “if ” clauses, but almost all
the rest, even those that begin with a transitional phrase such as “for example” or “in addition,” conform
to a “subject, verb, complement” pattern. The first paragraph, in which the second and third sentences
begin the same way (“It does not state”), is particularly repetitious.

807

GMAT® Official Guide 2018

Sample Paper 2
This argument has no information about air collisions. I think most cases happen in new airports
because the air traffic I heavy. In this case sound airport control could solve the problem.
I think this argument is logically reasonable. Its assumption is that plane collisions are caused by planes
that don’t know each others positions. So pilots can do nothing, if they know each others position
through the system it will solve the problem.
If it can provide evidence the problem is lack of knowledge of each others positions, it will be more
sound and persuasive.
More information about air collisions is helpful, (the reason for air collisions).

Explanation of Score 2
This response is seriously flawed in several ways. First of all, it has very little substance. The writer
appears to make only one point—that while it seems reasonable to assume that midair collisions
would be less likely if pilots were sure of each other’s positions, readers cannot adequately judge this
assumption without more information about where, why, and how such collisions occur. This point,
furthermore, is neither explained by a single reason beyond what is given in the topic nor supported by a
single example. Legitimate though it is, it cannot, alone and undeveloped, serve as an adequate response
to the argument.
Aside from being undeveloped, the response is confusing. At the outset, it seems to be critical of the
argument. The writer begins by pointing to the inadequacy of the information given; then speculates,
without evidence, that “most cases happen in new airports”; and then suggests that the problem should
be addressed by improving “airport control,” not (it is implied) by installing onboard warning systems.
After criticizing the argument in the first paragraph, the writer confusingly seems to endorse it in the
second. Then, in the remainder of the response, the writer returns to a critical stance.
The general lack of coherence is reflected in the serious and frequent writing problems that make
meaning hard to determine—for example, the elliptical and ungrammatical “So pilots can do nothing,
if they know each others position through the system it will solve the problem” (paragraph 2) or “If it
can provide evidence the problem is lack of knowledge of each others positions, it will be more sound
and persuasive” (paragraph 3). The prose suffers from a variety of basic errors in grammar, usage, and
mechanics.

808

11.6 Analytical Writing Assessment Analysis of an Argument Sample Topics

11.6 Analysis of an Argument Sample Topics
The following appeared as part of an annual report sent to stockholders by Olympic Foods, a processor of frozen
foods:
“Over time, the costs of processing go down because as organizations learn how to do things better, they become
more efficient. In color film processing, for example, the cost of a 3‐by‐5‐inch print fell from 50 cents for five‐day
service in 1970 to 20 cents for one‐day service in 1984. The same principle applies to the processing of food.
And since Olympic Foods will soon celebrate its 25th birthday, we can expect that our long experience will enable
us to minimize costs and thus maximize profits.”
Discuss how well reasoned you find this argument. In your discussion be sure to analyze the line of reasoning
and the use of evidence in the argument. For example, you may need to consider what questionable assumptions
underlie the thinking and what alternative explanations or counterexamples might weaken the conclusion. You can
also discuss what sort of evidence would strengthen or refute the argument, what changes in the argument would
make it more logically sound, and what, if anything, would help you better evaluate its conclusion.
____________________________
The following appeared in a memorandum from the business department of the Apogee Company:
“When the Apogee Company had all its operations in one location, it was more profitable than it is today. Therefore,
the Apogee Company should close down its field offices and conduct all its operations from a single location. Such
centralization would improve profitability by cutting costs and helping the company maintain better supervision of
all employees.”
Discuss how well reasoned … etc.
____________________________
The following appeared in a memorandum issued by a large city’s council on the arts:
“In a recent citywide poll, 15 percent more residents said that they watch television programs about the visual arts
than was the case in a poll conducted five years ago. During these past five years, the number of people visiting
our city’s art museums has increased by a similar percentage. Since the corporate funding that supports public
television, where most of the visual arts programs appear, is now being threatened with severe cuts, we can expect
that attendance at our city’s art museums will also start to decrease. Thus some of the city’s funds for supporting
the arts should be reallocated to public television.”
Discuss how well reasoned … etc.
____________________________
The following appeared in a report presented for discussion at a meeting of the directors of a company that
manufactures parts for heavy machinery:
“The falling revenues that the company is experiencing coincide with delays in manufacturing. These delays,
in turn, are due in large part to poor planning in purchasing metals. Consider further that the manager of
the department that handles purchasing of raw materials has an excellent background in general business,
psychology, and sociology, but knows little about the properties of metals. The company should, therefore, move
the purchasing manager to the sales department and bring in a scientist from the research division to be manager
of the purchasing department.”
Discuss how well reasoned … etc.
809

GMAT® Official Guide 2018

The following appeared in an announcement issued by the publisher of The Mercury, a weekly newspaper:
“Since a competing lower‐priced newspaper, The Bugle, was started five years ago, The Mercury’s circulation has
declined by 10,000 readers. The best way to get more people to read The Mercury is to reduce its price below
that of The Bugle, at least until circulation increases to former levels. The increased circulation of The Mercury will
attract more businesses to buy advertising space in the paper.”
Discuss how well reasoned … etc.
____________________________
The following appeared as part of an article in a magazine devoted to regional life:
“Corporations should look to the city of Helios when seeking new business opportunities or a new location. Even
in the recent recession, Helios’s unemployment rate was lower than the regional average. It is the industrial
center of the region, and historically it has provided more than its share of the region’s manufacturing jobs. In
addition, Helios is attempting to expand its economic base by attracting companies that focus on research and
development of innovative technologies.”
Discuss how well reasoned … etc.
____________________________
The following appeared in the health section of a magazine on trends and lifestyles:
“People who use the artificial sweetener aspartame are better off consuming sugar, since aspartame can actually
contribute to weight gain rather than weight loss. For example, high levels of aspartame have been shown to
trigger a craving for food by depleting the brain of a chemical that registers satiety, or the sense of being full.
Furthermore, studies suggest that sugars, if consumed after at least 45 minutes of continuous exercise, actually
enhance the body’s ability to burn fat. Consequently, those who drink aspartame‐sweetened juices after exercise
will also lose this calorie‐burning benefit. Thus it appears that people consuming aspartame rather than sugar are
unlikely to achieve their dietary goals.”
Discuss how well reasoned … etc.
____________________________
The following appeared in the editorial section of a corporate newsletter:
“The common notion that workers are generally apathetic about management issues is false, or at least outdated:
a recently published survey indicates that 79 percent of the nearly 1,200 workers who responded to survey
questionnaires expressed a high level of interest in the topics of corporate restructuring and redesign of benefits
programs.”
Discuss how well reasoned … etc.

810

11.6 Analytical Writing Assessment Analysis of an Argument Sample Topics

The following appeared in the opinion column of a financial magazine:
“On average, middle‐aged consumers devote 39 percent of their retail expenditure to department store products
and services, while for younger consumers the average is only 25 percent. Since the number of middle‐aged
people will increase dramatically within the next decade, department stores can expect retail sales to increase
significantly during that period. Furthermore, to take advantage of the trend, these stores should begin to replace
some of those products intended to attract the younger consumer with products intended to attract the middle‐
aged consumer.”
Discuss how well reasoned … etc.
____________________________
The following appeared in the editorial section of a local newspaper:
“This past winter, 200 students from Waymarsh State College traveled to the state capitol building to protest
against proposed cuts in funding for various state college programs. The other 12,000 Waymarsh students
evidently weren’t so concerned about their education: they either stayed on campus or left for winter break. Since
the group who did not protest is far more numerous, it is more representative of the state’s college students than
are the protesters. Therefore the state legislature need not heed the appeals of the protesting students.”
Discuss how well reasoned … etc.
____________________________
The following appeared in the editorial section of a local newspaper:
“In the first four years that Montoya has served as mayor of the city of San Perdito, the population has decreased
and the unemployment rate has increased. Two businesses have closed for each new business that has opened.
Under Varro, who served as mayor for four years before Montoya, the unemployment rate decreased and the
population increased. Clearly, the residents of San Perdito would be best served if they voted Montoya out of office
and reelected Varro.”
Discuss how well reasoned … etc.
____________________________
The following appeared as part of a promotional campaign to sell advertising space in the Daily Gazette to grocery
stores in the Marston area:
“Advertising the reduced price of selected grocery items in the Daily Gazette will help you increase your sales.
Consider the results of a study conducted last month. Thirty sale items from a store in downtown Marston
were advertised in The Gazette for four days. Each time one or more of the 30 items was purchased, clerks
asked whether the shopper had read the ad. Two‐thirds of the 200 shoppers asked answered in the affirmative.
Furthermore, more than half the customers who answered in the affirmative spent over $100 at the store.”
Discuss how well reasoned … etc.

811

GMAT® Official Guide 2018

The following appeared as part of a campaign to sell advertising time on a local radio station to local businesses:
“The Cumquat Café began advertising on our local radio station this year and was delighted to see its business
increase by 10 percent over last year’s totals. Their success shows you how you can use radio advertising to
make your business more profitable.”
Discuss how well reasoned … etc.
____________________________
The following appeared as part of a newspaper editorial:
“Two years ago Nova High School began to use interactive computer instruction in three academic subjects. The
school dropout rate declined immediately, and last year’s graduates have reported some impressive achievements
in college. In future budgets the school board should use a greater portion of the available funds to buy more
computers, and all schools in the district should adopt interactive computer instruction throughout the curriculum.”
Discuss how well reasoned … etc.
____________________________
The following appeared as a part of an advertisement for Adams, who is seeking re‐election as governor:
“Re‐elect Adams, and you will be voting for proven leadership in improving the state’s economy. Over the past year
alone, 70 percent of the state’s workers have had increases in their wages, 5,000 new jobs have been created,
and six corporations have located their headquarters here. Most of the respondents in a recent poll said they
believed that the economy is likely to continue to improve if Adams is re‐elected. Adams’s opponent, Zebulon,
would lead our state in the wrong direction, because Zebulon disagrees with many of Adams’s economic policies.”
Discuss how well reasoned … etc.
____________________________
The following appeared as part of an article in the education section of a Waymarsh city newspaper:
“Throughout the last two decades, those who earned graduate degrees found it very difficult to get jobs teaching
their academic specialties at the college level. Those with graduate degrees from Waymarsh University had
an especially hard time finding such jobs. But better times are coming in the next decade for all academic job
seekers, including those from Waymarsh. Demographic trends indicate that an increasing number of people will
be reaching college age over the next 10 years; consequently, we can expect that the job market will improve
dramatically for people seeking college‐level teaching positions in their fields.”
Discuss how well reasoned … etc.

812

11.6 Analytical Writing Assessment Analysis of an Argument Sample Topics

The following appeared in an article in a consumer‐products magazine:
“Two of today’s best‐selling brands of full‐strength prescription medication for the relief of excess stomach acid,
Acid‐Ease and Pepticaid, are now available in milder nonprescription forms. Doctors have written 76 million more
prescriptions for full‐strength Acid‐Ease than for full‐strength Pepticaid. So people who need an effective but milder
nonprescription medication for the relief of excess stomach acid should choose Acid‐Ease.”
Discuss how well reasoned … etc.
____________________________
The following is an excerpt from a memo written by the head of a governmental department:
“Neither stronger ethics regulations nor stronger enforcement mechanisms are necessary to ensure ethical
behavior by companies doing business with this department. We already have a code of ethics that companies
doing business with this department are urged to abide by, and virtually all of these companies have agreed to
follow it. We also know that the code is relevant to the current business environment because it was approved
within the last year, and in direct response to specific violations committed by companies with which we were then
working—not in abstract anticipation of potential violations, as so many such codes are.”
Discuss how well reasoned … etc.
____________________________
The following appeared as part of an article in the travel section of a newspaper:
“Over the past decade, the restaurant industry in the country of Spiessa has experienced unprecedented growth.
This surge can be expected to continue in the coming years, fueled by recent social changes: personal incomes
are rising, more leisure time is available, single‐person households are more common, and people have a greater
interest in gourmet food, as evidenced by a proliferation of publications on the subject.”
Discuss how well reasoned … etc.
____________________________
The following appeared in an article in a health and fitness magazine:
“Laboratory studies show that Saluda Natural Spring Water contains several of the minerals necessary for good
health and that it is completely free of bacteria. Residents of Saluda, the small town where the water is bottled,
are hospitalized less frequently than the national average. Even though Saluda Natural Spring Water may seem
expensive, drinking it instead of tap water is a wise investment in good health.”
Discuss how well reasoned … etc.

813

GMAT® Official Guide 2018

The following appeared as part of an editorial in an industry newsletter:
“While trucking companies that deliver goods pay only a portion of highway maintenance costs and no property
tax on the highways they use, railways spend billions per year maintaining and upgrading their facilities. The
government should lower the railroad companies’ property taxes, since sending goods by rail is clearly a more
appropriate mode of ground transportation than highway shipping. For one thing, trains consume only a third of the
fuel a truck would use to carry the same load, making them a more cost‐effective and environmentally sound mode
of transport. Furthermore, since rail lines already exist, increases in rail traffic would not require building new lines
at the expense of taxpaying citizens.”
Discuss how well reasoned … etc.
____________________________
The following appeared in the editorial section of a newspaper:
“As public concern over drug abuse has increased, authorities have become more vigilant in their efforts to
prevent illegal drugs from entering the country. Many drug traffickers have consequently switched from marijuana,
which is bulky, or heroin, which has a market too small to justify the risk of severe punishment, to cocaine. Thus
enforcement efforts have ironically resulted in an observed increase in the illegal use of cocaine.”
Discuss how well reasoned … etc.
____________________________
The following appeared in a speech delivered by a member of the city council:
“Twenty years ago, only half of the students who graduated from Einstein High School went on to attend a college
or university. Today, two‐thirds of the students who graduate from Einstein do so. Clearly, Einstein has improved
its educational effectiveness over the past two decades. This improvement has occurred despite the fact that the
school’s funding, when adjusted for inflation, is about the same as it was 20 years ago. Therefore, we do not need
to make any substantial increase in the school’s funding at this time.”
Discuss how well reasoned … etc.
____________________________
The following appeared in a memo from the customer service division to the manager of Mammon Savings and
Loan:
“We believe that improved customer service is the best way for us to differentiate ourselves from competitors and
attract new customers. We can offer our customers better service by reducing waiting time in teller lines from an
average of six minutes to an average of three. By opening for business at 8:30 instead of 9:00, and by remaining
open for an additional hour beyond our current closing time, we will be better able to accommodate the busy
schedules of our customers. These changes will enhance our bank’s image as the most customer‐friendly bank in
town and give us the edge over our competition.”
Discuss how well reasoned … etc.

814

11.6 Analytical Writing Assessment Analysis of an Argument Sample Topics

The following appeared as part of an article in a magazine on lifestyles:
“Two years ago, City L was listed fourteenth in an annual survey that ranks cities according to the quality of life
that can be enjoyed by those living in them. This information will enable people who are moving to the state in
which City L is located to confidently identify one place, at least, where schools are good, housing is affordable,
people are friendly, the environment is safe, and the arts flourish.”
Discuss how well reasoned … etc.
____________________________
The following appeared in a memorandum from a member of a financial management and consulting firm:
“We have learned from an employee of Windfall, Ltd., that its accounting department, by checking about 10
percent of the last month’s purchasing invoices for errors and inconsistencies, saved the company some $10,000
in overpayments. In order to help our clients increase their net gains, we should advise each of them to institute a
policy of checking all purchasing invoices for errors. Such a recommendation could also help us get the Windfall
account by demonstrating to Windfall the rigorousness of our methods.”
Discuss how well reasoned … etc.
____________________________
The following appeared in a newspaper editorial:
“As violence in movies increases, so do crime rates in our cities. To combat this problem we must establish a
board to censor certain movies, or we must limit admission to persons over 21 years of age. Apparently our
legislators are not concerned about this issue since a bill calling for such actions recently failed to receive a
majority vote.”
Discuss how well reasoned … etc.
____________________________
The following appeared in the editorial section of a local newspaper:
“Commuter use of the new subway train is exceeding the transit company’s projections. However, commuter
use of the shuttle buses that transport people to the subway stations is below the projected volume. If the transit
company expects commuters to ride the shuttle buses to the subway rather than drive there, it must either reduce
the shuttle bus fares or increase the price of parking at the subway stations.”
Discuss how well reasoned … etc.
____________________________
The following was excerpted from the speech of a spokesperson for Synthetic Farm Products, Inc.:
“Many farmers who invested in the equipment needed to make the switch from synthetic to organic fertilizers and
pesticides feel that it would be too expensive to resume synthetic farming at this point. But studies of farmers who
switched to organic farming last year indicate that their current crop yields are lower. Hence their purchase of organic
farming equipment, a relatively minor investment compared to the losses that would result from continued lower crop
yields, cannot justify persisting on an unwise course. And the choice to farm organically is financially unwise, given that it
was motivated by environmental rather than economic concerns.”
Discuss how well reasoned … etc.

815

GMAT® Official Guide 2018

The following appeared in a newspaper story giving advice about investments:
“As overall life expectancy continues to rise, the population of our country is growing increasingly older. For
example, more than 20 percent of the residents of one of our more populated regions are now at least 65 years
old, and occupancy rates at resort hotels in that region declined significantly during the past six months. Because
of these two related trends, a prudent investor would be well advised to sell interest in hotels and invest in
hospitals and nursing homes instead.”
Discuss how well reasoned … etc.
____________________________
The following appeared as part of the business plan of an investment and financial consulting firm:
“Studies suggest that an average coffee drinker’s consumption of coffee increases with age, from age 10 through
age 60. Even after age 60, coffee consumption remains high. The average cola drinker’s consumption of cola,
however, declines with increasing age. Both of these trends have remained stable for the past 40 years. Given that
the number of older adults will significantly increase as the population ages over the next 20 years, it follows that
the demand for coffee will increase and the demand for cola will decrease during this period. We should, therefore,
consider transferring our investments from Cola Loca to Early Bird Coffee.”
Discuss how well reasoned … etc.
____________________________
The following appeared in the editorial section of a West Cambria newspaper:
“A recent review of the West Cambria volunteer ambulance service revealed a longer average response time to
accidents than was reported by a commercial ambulance squad located in East Cambria. In order to provide better
patient care for accident victims and to raise revenue for our town by collecting service fees for ambulance use,
we should disband our volunteer service and hire a commercial ambulance service.”
Discuss how well reasoned … etc.
____________________________
The following is part of a business plan being discussed at a board meeting of the Perks Company:
“It is no longer cost‐effective for the Perks Company to continue offering its employees a generous package of
benefits and incentives year after year. In periods when national unemployment rates are low, Perks may need to
offer such a package in order to attract and keep good employees, but since national unemployment rates are
now high, Perks does not need to offer the same benefits and incentives. The money thus saved could be better
used to replace the existing plant machinery with more technologically sophisticated equipment, or even to build
an additional plant.”
Discuss how well reasoned … etc.

816

11.6 Analytical Writing Assessment Analysis of an Argument Sample Topics

The following appeared as part of a plan proposed by an executive of the Easy Credit Company to the president:
“The Easy Credit Company would gain an advantage over competing credit card services if we were to donate a
portion of the proceeds from the use of our cards to a well‐known environmental organization in exchange for the
use of its symbol or logo on our card. Since a recent poll shows that a large percentage of the public is concerned
about environmental issues, this policy would attract new customers, increase use among existing customers, and
enable us to charge interest rates that are higher than the lowest ones available.”
Discuss how well reasoned … etc.
____________________________
The following appeared as part of a recommendation from the financial planning office to the administration of Fern
Valley University:
“In the past few years, Fern Valley University has suffered from a decline in both enrollments and admissions
applications. The reason can be discovered from our students, who most often cite poor teaching and inadequate
library resources as their chief sources of dissatisfaction with Fern Valley. Therefore, in order to increase the
number of students attending our university, and hence to regain our position as the most prestigious university in
the greater Fern Valley metropolitan area, it is necessary to initiate a fund‐raising campaign among the alumni that
will enable us to expand the range of subjects we teach and to increase the size of our library facilities.”
Discuss how well reasoned … etc.
____________________________
The following appeared in an article in a college departmental newsletter:
“Professor Taylor of Jones University is promoting a model of foreign language instruction in which students
receive 10 weeks of intensive training, then go abroad to live with families for 10 weeks. The superiority of the
model, Professor Taylor contends, is proved by the results of a study in which foreign language tests given to
students at 25 other colleges show that first‐year foreign language students at Jones speak more fluently after
only 10 to 20 weeks in the program than do 9 out of 10 foreign language majors elsewhere at the time of their
graduation.”
Discuss how well reasoned … etc.
____________________________
The following appeared as part of an article in the business section of a local newspaper:
“Motorcycle X has been manufactured in the United States for more than 70 years. Although one foreign company
has copied the motorcycle and is selling it for less, the company has failed to attract motorcycle X customers—
some say because its product lacks the exceptionally loud noise made by motorcycle X. But there must be some
other explanation. After all, foreign cars tend to be quieter than similar American‐made cars, but they sell at least
as well. Also, television advertisements for motorcycle X highlight its durability and sleek lines, not its noisiness,
and the ads typically have voice‐overs or rock music rather than engine‐roar on the sound track.”
Discuss how well reasoned … etc.

817

GMAT® Official Guide 2018

The following appeared in the editorial section of a campus newspaper:
“Because occupancy rates for campus housing fell during the last academic year, so did housing revenues.
To solve the problem, campus housing officials should reduce the number of available housing units, thereby
increasing the occupancy rates. Also, to keep students from choosing to live off‐campus, housing officials should
lower the rents, thereby increasing demand.”
Discuss how well reasoned … etc.
____________________________
The following appeared in an Avia Airlines departmental memorandum:
“On average, 9 out of every 1,000 passengers who traveled on Avia Airlines last year filed a complaint about our
baggage‐handling procedures. This means that although some 1 percent of our passengers were unhappy with
those procedures, the overwhelming majority were quite satisfied with them; thus it would appear that a review of
the procedures is not important to our goal of maintaining or increasing the number of Avia’s passengers.”
Discuss how well reasoned … etc.
____________________________
The following appeared as part of an article in a weekly newsmagazine:
“The country of Sacchar can best solve its current trade deficit problem by lowering the price of sugar, its primary
export. Such an action would make Sacchar better able to compete for markets with other sugar‐exporting
countries. The sale of Sacchar’s sugar abroad would increase, and this increase would substantially reduce
Sacchar’s trade deficit.”
Discuss how well reasoned … etc.
____________________________
The following appeared as part of an article in a trade publication:
“Stronger laws are needed to protect new kinds of home‐security systems from being copied and sold by imitators.
With such protection, manufacturers will naturally invest in the development of new home‐security products and
production technologies. Without stronger laws, therefore, manufacturers will cut back on investment. From this will
follow a corresponding decline not only in product quality and marketability, but also in production efficiency, and thus
ultimately a loss of manufacturing jobs in the industry.”
Discuss how well reasoned … etc.
____________________________
The following appeared in the opinion section of a national newsmagazine:
“To reverse the deterioration of the postal service, the government should raise the price of postage stamps. This
solution will no doubt prove effective, since the price increase will generate larger revenues and will also reduce
the volume of mail, thereby eliminating the strain on the existing system and contributing to improved morale.”
Discuss how well reasoned … etc.

818

11.6 Analytical Writing Assessment Analysis of an Argument Sample Topics

The following appeared in an article in the health section of a newspaper:
“There is a common misconception that university hospitals are better than community or private hospitals. This
notion is unfounded, however: the university hospitals in our region employ 15 percent fewer doctors, have a 20
percent lower success rate in treating patients, make far less overall profit, and pay their medical staff considerably
less than do private hospitals. Furthermore, many doctors at university hospitals typically divide their time among
teaching, conducting research, and treating patients. From this it seems clear that the quality of care at university
hospitals is lower than that at other kinds of hospitals.”
Discuss how well reasoned … etc.
____________________________
The following is part of a business plan created by the management of the Megamart grocery store:
“Our total sales have increased this year by 20 percent since we added a pharmacy section to our grocery
store. Clearly, the customer’s main concern is the convenience afforded by one‐stop shopping. The surest way
to increase our profits over the next couple of years, therefore, is to add a clothing department along with an
automotive supplies and repair shop. We should also plan to continue adding new departments and services, such
as a restaurant and a garden shop, in subsequent years. Being the only store in the area that offers such a range
of services will give us a competitive advantage over other local stores.”
Discuss how well reasoned … etc.
____________________________
The following appeared as part of a column in a popular entertainment magazine:
“The producers of the forthcoming movie 3003 will be most likely to maximize their profits if they are willing to
pay Robin Good several million dollars to star in it—even though that amount is far more than any other person
involved with the movie will make. After all, Robin has in the past been paid a similar amount to work in several
films that were very financially successful.”
Discuss how well reasoned … etc.
____________________________
The following appeared in a memorandum from the directors of a security and safety consulting service:
“Our research indicates that over the past six years no incidents of employee theft have been reported within ten
of the companies that have been our clients. In analyzing the security practices of these ten companies, we have
further learned that each of them requires its employees to wear photo identification badges while at work. In the
future, therefore, we should recommend the use of such identification badges to all of our clients.”
Discuss how well reasoned … etc.
____________________________
The following appeared as part of an article in the business section of a local newspaper:
“The owners of the Cumquat Cafe evidently made a good business decision in moving to a new location, as can be seen
from the fact that the Cafe will soon celebrate its second anniversary there. Moreover, it appears that businesses are
not likely to succeed at the old location: since the Cafe’s move, three different businesses—a tanning salon, an antique
emporium, and a pet‐grooming shop—have occupied its former spot.”
Discuss how well reasoned … etc.

819

GMAT® Official Guide 2018

The following appeared in the editorial section of a local newspaper:
“The profitability of Croesus Company, recently restored to private ownership, is a clear indication that businesses
fare better under private ownership than under public ownership.”
Discuss how well reasoned … etc.
____________________________
The following appeared in the editorial section of a local newspaper:
“If the paper from every morning edition of the nation’s largest newspaper were collected and rendered into paper
pulp that the newspaper could reuse, about 5 million trees would be saved each year. This kind of recycling is
unnecessary, however, since the newspaper maintains its own forests to ensure an uninterrupted supply of paper.”
Discuss how well reasoned … etc.
____________________________
The following appeared as part of a business plan recommended by the new manager of a musical rock group
called Zapped:
“To succeed financially, Zapped needs greater name recognition. It should therefore diversify its commercial
enterprises. The rock group Zonked plays the same type of music that Zapped plays, but it is much better known
than Zapped because, in addition to its concert tours and four albums, Zonked has a series of posters, a line
of clothing and accessories, and a contract with a major advertising agency to endorse a number of different
products.”
Discuss how well reasoned … etc.
____________________________
The following appeared in a magazine article on trends and lifestyles:
“In general, people are not as concerned as they were a decade ago about regulating their intake of red meat and
fatty cheeses. Walk into the Heart’s Delight, a store that started selling organic fruits and vegetables and whole‐
grain flours in the 1960s, and you will also find a wide selection of cheeses made with high butterfat content. Next
door, the owners of the Good Earth Cafe, an old vegetarian restaurant, are still making a modest living, but the
owners of the new House of Beef across the street are millionaires.”
Discuss how well reasoned … etc.
____________________________
The following editorial appeared in the Elm City paper:
“The construction last year of a shopping mall in downtown Oak City was a mistake. Since the mall has opened,
a number of local businesses have closed, and the downtown area suffers from an acute parking shortage, and
arrests for crime and vagrancy have increased in the nearby Oak City Park. Elm City should pay attention to the
example of the Oak City mall and deny the application to build a shopping mall in Elm City.”
Discuss how well reasoned … etc.

820

11.6 Analytical Writing Assessment Analysis of an Argument Sample Topics

The following appeared as part of an editorial in a weekly newsmagazine:
“Historically, most of this country’s engineers have come from our universities; recently, however, our university‐
age population has begun to shrink, and decreasing enrollments in our high schools clearly show that this drop
in numbers will continue throughout the remainder of the decade. Consequently, our nation will soon be facing a
shortage of trained engineers. If we are to remain economically competitive in the world marketplace, then we
must increase funding for education—and quickly.”
Discuss how well reasoned … etc.
____________________________
The following appeared in an Excelsior Company memorandum:
“The Excelsior Company plans to introduce its own brand of coffee. Since coffee is an expensive food item, and
since there are already many established brands of coffee, the best way to gain customers for the Excelsior brand
is to do what Superior, the leading coffee company, did when it introduced the newest brand in its line of coffees:
conduct a temporary sales promotion that offers free samples, price reductions, and discount coupons for the new
brand.”
Discuss how well reasoned … etc.
____________________________
The following appeared as part of an article in a health club trade publication:
“After experiencing a decline in usage by its members, Healthy Heart fitness center built an indoor pool. Since
usage did not increase significantly, it appears that health club managers should adopt another approach—
lowering membership fees rather than installing expensive new features.”
Discuss how well reasoned … etc.
____________________________
The following appeared as part of an article in a popular arts‐and‐leisure magazine:
“The safety codes governing the construction of public buildings are becoming far too strict. The surest way for
architects and builders to prove that they have met the minimum requirements established by these codes is to
construct buildings by using the same materials and methods that are currently allowed. But doing so means
that there will be very little significant technological innovation within the industry, and hence little evolution of
architectural styles and design—merely because of the strictness of these safety codes.”
Discuss how well reasoned … etc.

821

GMAT® Official Guide 2018

The following is from a campaign by Big Boards Inc. to convince companies in River City that their sales will
increase if they use Big Boards billboards for advertising their locally manufactured products:
“The potential of Big Boards to increase sales of your products can be seen from an experiment we conducted
last year. We increased public awareness of the name of the current national women’s marathon champion by
publishing her picture and her name on billboards in River City for a period of three months. Before this time,
although the champion had just won her title and was receiving extensive national publicity, only 5 percent of
15,000 randomly surveyed residents of River City could correctly name the champion when shown her picture;
after the three‐month advertising experiment, 35 percent of respondents from a second survey could supply her
name.”
Discuss how well reasoned … etc.
____________________________
The following appeared as part of an article on government funding of environmental regulatory agencies:
“When scientists finally learn how to create large amounts of copper from other chemical elements, the regulation
of copper mining will become unnecessary. For one thing, since the amount of potentially available copper will no
longer be limited by the quantity of actual copper deposits, the problem of over‐mining will quickly be eliminated
altogether. For another, manufacturers will not need to use synthetic copper substitutes, the production of which
creates pollutants. Thus, since two problems will be settled—over‐mining and pollution—it makes good sense to
reduce funding for mining regulation and either save the money or reallocate it where it is needed more.”
Discuss how well reasoned … etc.
____________________________
The following appeared as part of an article in a popular science magazine:
“Scientists must typically work 60 to 80 hours a week if they hope to further their careers; consequently, good
and affordable all‐day child care must be made available to both male and female scientists if they are to advance
in their fields. Moreover, requirements for career advancement must be made more flexible so that preschool‐age
children can spend a significant portion of each day with a parent.”
Discuss how well reasoned … etc.
____________________________
The following appeared as part of a recommendation by one of the directors of the Beta Company:
“The Alpha Company has just reduced its workforce by laying off 15 percent of its employees in all divisions
and at all levels, and it is encouraging early retirement for other employees. As you know, the Beta Company
manufactures some products similar to Alpha’s, but our profits have fallen over the last few years. To improve
Beta’s competitive position, we should try to hire a significant number of Alpha’s former workers, since these
experienced workers can provide valuable information about Alpha’s successful methods, will require little training,
and will be particularly motivated to compete against Alpha.”
Discuss how well reasoned … etc.

822

11.6 Analytical Writing Assessment Analysis of an Argument Sample Topics

The following appeared in the letters‐to‐the‐editor section of a local newspaper:
“Muscle Monthly, a fitness magazine that regularly features pictures of bodybuilders using state‐of‐the‐art exercise
machines, frequently sells out, according to the owner of Skyview Newsstand. To help maximize fitness levels in our town’s
residents, we should, therefore, equip our new community fitness center with such machines.”
Discuss how well reasoned … etc.
____________________________
The following appeared as part of an article in the business section of a local newspaper:
“The Cumquat Cafe made a mistake in moving to a new location. After one year at the new spot, it is doing about
the same volume of business as before, but the owners of the RoboWrench plumbing supply wholesale outlet that
took over its old location are apparently doing better: RoboWrench is planning to open a store in a neighboring
city.”
Discuss how well reasoned … etc.
____________________________
The following appeared in a memorandum from the director of human resources to the executive officers of
Company X:
“Last year, we surveyed our employees on improvements needed at Company X by having them rank, in order
of importance, the issues presented in a list of possible improvements. Improved communications between
employees and management was consistently ranked as the issue of highest importance by the employees who
responded to the survey. As you know, we have since instituted regular communications sessions conducted by
high‐level management, which the employees can attend on a voluntary basis. Therefore, it is likely that most
employees at Company X now feel that the improvement most needed at the company has been made.”
Discuss how well reasoned … etc.
____________________________
The following appeared in a memorandum from the vice president of Road Food, an international chain of fast‐food
restaurants:
“This past year, we spent almost as much on advertising as did our main competitor, Street Eats, which has fewer
restaurants than we do. Although it appeared at first that our advertising agency had created a campaign along
the lines we suggested, in fact our total profits were lower than those of Street Eats. In order to motivate our
advertising agency to perform better, we should start basing the amount that we pay it on how much total profit
we make each year.”
Discuss how well reasoned … etc.
____________________________
The following appeared in the promotional literature for Cerberus dog food:
“Obesity is a great problem among pet dogs, just as it is among their human owners. Obesity in humans is
typically caused by consuming more calories than the body needs. For humans, a proper diet for losing weight is
a reduced‐calorie diet that is high in fiber and carbohydrates but low in fat. Therefore, the best way for dog owners
to help their dogs lose weight in a healthy way is to restrict the dog’s diet to Cerberus reduced‐calorie dog food,
which is high in fiber and carbohydrates but low in fat.”
Discuss how well reasoned … etc.
823

GMAT® Official Guide 2018

The following appeared in an article in a travel magazine:
“After the airline industry began requiring airlines to report their on‐time rates, Speedee Airlines achieved the
number one on‐time rate, with more than 89 percent of its flights arriving on time each month. And now Speedee
is offering more flights to more destinations than ever before. Clearly, Speedee is the best choice for today’s
business traveler.”
Discuss how well reasoned … etc.
____________________________
The following appeared in a memorandum to the planning department of an investment firm:
“Costs have begun dropping for several types of equipment currently used to convert solar energy into electricity.
Moreover, some exciting new technologies for converting solar energy are now being researched and developed.
Hence we can expect that solar energy will soon become more cost efficient and attractive than coal or oil
as a source of electrical power. We should, therefore, encourage investment in Solario, a new manufacturer
of solar‐powered products. After all, Solario’s chief executive was once on the financial planning team for
Ready‐to‐Ware, a software engineering firm that has shown remarkable growth since its recent incorporation.”
Discuss how well reasoned … etc.
____________________________
The following appeared in a memorandum from a company’s marketing department:
“Since our company started manufacturing and marketing a deluxe air filter six months ago, sales of our economy
filter—and company profits—have decreased significantly. The deluxe air filter sells for 50 percent more than
the economy filter, but the economy filter lasts for only one month while the deluxe filter can be used for two
months before it must be replaced. To increase repeat sales of our economy filter and maximize profits, we should
discontinue the deluxe air filter and concentrate all our advertising efforts on the economy filter.”
Discuss how well reasoned … etc.
____________________________
The following appeared in a memorandum from the president of a company that makes shampoo:
“A widely publicized study claims that HR2, a chemical compound in our shampoo, can contribute to hair loss after
prolonged use. This study, however, involved only 500 subjects. Furthermore, we have received no complaints
from our customers during the past year, and some of our competitors actually use more HR2 per bottle of
shampoo than we do. Therefore, we do not need to consider replacing the HR2 in our shampoo with a more
expensive alternative.”
Discuss how well reasoned … etc.

824

11.6 Analytical Writing Assessment Analysis of an Argument Sample Topics

The following appeared in the editorial section of a local newspaper:
“The tragic crash of a medical helicopter last week points out a situation that needs to be addressed. The medical‐
helicopter industry supposedly has more stringent guidelines for training pilots and maintaining equipment than do
most other airline industries, but these guidelines do not appear to be working: statistics reveal that the rate of
medical‐helicopter accidents is much higher than the rate of accidents for nonmedical helicopters or commercial
airliners.”
Discuss how well reasoned … etc.
____________________________
The following appeared as part of a recommendation from the business manager of a department store:
“Local clothing stores reported that their profits decreased, on average, for the three‐month period between
August 1 and October 31. Stores that sell products for the home reported that, on average, their profits increased
during this same period. Clearly, consumers are choosing to buy products for their homes instead of clothing.
To take advantage of this trend, we should reduce the size of our clothing departments and enlarge our home
furnishings and household products departments.”
Discuss how well reasoned … etc.
____________________________
The following appeared in a letter to the editor of a regional newspaper:
“In response to petitions from the many farmers and rural landowners throughout our region, the legislature has
spent valuable time and effort enacting severe laws to deter motorists from picking fruit off the trees, trampling
through the fields, and stealing samples of foliage. But how can our local lawmakers occupy themselves with such
petty vandalism when crime and violence plague the nation’s cities? The fate of apples and leaves is simply too
trivial to merit their attention.”
Discuss how well reasoned … etc.
____________________________
The following appeared as part of an editorial in a campus newspaper:
“With an increasing demand for highly skilled workers, this nation will soon face a serious labor shortage. New
positions in technical and professional occupations are increasing rapidly, while at the same time the total labor
force is growing slowly. Moreover, the government is proposing to cut funds for aid to education in the near
future.”
Discuss how well reasoned … etc.
____________________________
The following appeared as part of a memorandum from a government agency:
“Given the limited funding available for the building and repair of roads and bridges, the government should not
spend any money this year on fixing the bridge that crosses the Styx River. This bridge is located near a city with
a weakening economy, so it is not as important as other bridges; moreover, the city population is small and thus
unlikely to contribute a significant enough tax revenue to justify the effort of fixing the bridge.”
Discuss how well reasoned … etc.

825

GMAT® Official Guide 2018

The following appeared as part of an article in an entertainment magazine:
“A series of books based on the characters from a popular movie are consistently best sellers in local bookstores.
Seeking to capitalize on the books’ success, Vista Studios is planning to produce a movie sequel based on the
books. Due to the success of the books and the original movie, the sequel will undoubtedly be profitable.”
Discuss how well reasoned … etc.
____________________________
The following appeared in a letter to the editor of a popular science and technology magazine:
“It is a popular myth that consumers are really benefiting from advances in agricultural technology. Granted,
consumers are, on the average, spending a decreasing proportion of their income on food. But consider that the
demand for food does not rise in proportion with real income. As real income rises, therefore, consumers can be
expected to spend a decreasing proportion of their income on food. Yet agricultural technology is credited with
having made our lives better.”
Discuss how well reasoned … etc.
____________________________
The following appeared in the editorial section of a local newspaper:
“This city should be able to improve existing services and provide new ones without periodically raising the taxes
of the residents. Instead, the city should require that the costs of services be paid for by developers who seek
approval for their large new building projects. After all, these projects can be highly profitable to the developers,
but they can also raise a city’s expenses and increase the demand for its services.”
Discuss how well reasoned … etc.
____________________________
The following appeared in the editorial section of a local newspaper:
“In order to avoid the serious health threats associated with many landfills, our municipality should build a plant for
burning trash. An incinerator could offer economic as well as ecological advantages over the typical old‐fashioned
type of landfill: incinerators can be adapted to generate moderate amounts of electricity, and ash residue from
some types of trash can be used to condition garden soil.”
Discuss how well reasoned … etc.
____________________________
The following appeared in the editorial section of a monthly business newsmagazine:
“Most companies would agree that as the risk of physical injury occurring on the job increases, the wages paid to
employees should also increase. Hence it makes financial sense for employers to make the workplace safer: they
could thus reduce their payroll expenses and save money.”
Discuss how well reasoned … etc.

826

11.6 Analytical Writing Assessment Analysis of an Argument Sample Topics

The following appeared as part of a company memorandum:
“Adopting an official code of ethics regarding business practices may in the long run do our company more harm
than good in the public eye. When one of our competitors received unfavorable publicity for violating its own code
of ethics, it got more attention from the media than it would have if it had had no such code. Rather than adopt an
official code of ethics, therefore, we should instead conduct a publicity campaign that stresses the importance of
protecting the environment and assisting charitable organizations.”
Discuss how well reasoned … etc.
____________________________
The following appeared in the editorial section of a daily newspaper:
“Although forecasts of presidential elections based on opinion polls measure current voter preference, many voters
keep changing their minds about whom they prefer until the last few days before the balloting. Some do not even
make a final decision until they enter the voting booth. Forecasts based on opinion polls are therefore little better
at predicting election outcomes than a random guess would be.”
Discuss how well reasoned … etc.
____________________________
The following appeared in the editorial section of a newspaper in the country of West Cambria:
“The practice of officially changing speed limits on the highways—whether by increasing or decreasing them—is
a dangerous one. Consider what happened over the past decade whenever neighboring East Cambria changed its
speed limits: an average of 3 percent more automobile accidents occurred during the week following the change
than had occurred during the week preceding it—even when the speed limit was lowered. This statistic shows that
the change in speed limit adversely affected the alertness of drivers.”
Discuss how well reasoned … etc.
____________________________
The following appeared as part of a memorandum from the vice president of Nostrum, a large pharmaceutical
corporation:
“The proposal to increase the health and retirement benefits that our employees receive should not be
implemented at this time. An increase in these benefits is not only financially unjustified, since our last year’s profits
were lower than those of the preceding year, but also unnecessary, since our chief competitor, Panacea, offers its
employees lower health and retirement benefits than we currently offer. We can assume that our employees are
reasonably satisfied with the health and retirement benefits that they now have since a recent survey indicated that
two‐thirds of the respondents viewed them favorably.”
Discuss how well reasoned … etc.

827

GMAT® Official Guide 2018

The following appeared as part of an article on trends in television:
“A recent study of viewers’ attitudes toward prime‐time television programs shows that many of the programs
that were judged by their viewers to be of high quality appeared on (noncommercial) television networks, and
that, on commercial television, the most popular shows are typically sponsored by the bestselling products. Thus,
it follows that businesses who use commercial television to promote their products will achieve the greatest
advertising success by sponsoring only highly rated programs—and, ideally, programs resembling the highly rated
noncommercial programs on public channels as much as possible.”
Discuss how well reasoned … etc.
____________________________
The following appeared as part of an article in the business section of a daily newspaper:
“Company A has a large share of the international market in video‐game hardware and software. Company B, the
pioneer in these products, was once a $12 billion‐a‐year giant but collapsed when children became bored with its
line of products. Thus Company A can also be expected to fail, especially given the fact that its games are now in
so many American homes that the demand for them is nearly exhausted.”
Discuss how well reasoned … etc.
____________________________
The following appeared as part of an article in a photography magazine:
“When choosing whether to work in color or in black‐and‐white, the photographer who wishes to be successful
should keep in mind that because color photographs are more true to life, magazines use more color photographs
than black‐and‐white ones, and many newspapers are also starting to use color photographs. The realism of color
also accounts for the fact that most portrait studios use more color film than black‐and‐white film. Furthermore,
there are more types of color film than black‐and‐white film available today. Clearly, photographers who work in
color have an advantage over those who work in black‐and‐white.”
Discuss how well reasoned … etc.
____________________________
The following appeared as part of a letter to the editor of a local newspaper:
“It makes no sense that in most places 15‐year‐olds are not eligible for their driver’s license while people who are
far older can retain all of their driving privileges by simply renewing their license. If older drivers can get these
renewals, often without having to pass another driving test, then 15‐year‐olds should be eligible to get a license.
Fifteen‐year‐olds typically have much better eyesight, especially at night; much better hand‐eye coordination; and
much quicker reflexes. They are also less likely to feel confused by unexpected developments or disoriented in
unfamiliar surroundings, and they recover from injuries more quickly.”
Discuss how well reasoned … etc.

828

11.6 Analytical Writing Assessment Analysis of an Argument Sample Topics

The following appeared in an ad for a book titled How to Write a Screenplay for a Movie:
“Writers who want to succeed should try to write film screenplays rather than books, since the average film tends
to make greater profits than does even a best‐selling book. It is true that some books are also made into films.
However, our nation’s film producers are more likely to produce movies based on original screenplays than to
produce films based on books, because in recent years the films that have sold the most tickets have usually been
based on original screenplays.”
Discuss how well reasoned … etc.
____________________________
The following appeared as part of an article in a daily newspaper:
“The computerized onboard warning system that will be installed in commercial airliners will virtually solve the
problem of midair plane collisions. One plane’s warning system can receive signals from another’s transponder—a
radio set that signals a plane’s course—in order to determine the likelihood of a collision and recommend evasive
action.”
Discuss how well reasoned … etc.
____________________________
The following appeared in a memorandum from the ElectroWares company’s marketing department:
“Since our company started manufacturing and marketing a deluxe light bulb six months ago, sales of our
economy light bulb—and company profits—have decreased significantly. Although the deluxe light bulb sells for
50 percent more than the economy bulb, it lasts twice as long. Therefore, to increase repeat sales and maximize
profits, we should discontinue the deluxe light bulb.”
Discuss how well reasoned … etc.
____________________________
The following is taken from an editorial in a local newspaper:
“Over the past decade, the price per pound of citrus fruit has increased substantially. Eleven years ago, Megamart
charged 15 cents a pound for lemons, but today it commonly charges over a dollar a pound. In only one of these
last 11 years was the weather unfavorable for growing citrus crops. Evidently, then, citrus growers have been
responsible for the excessive increase in the price of citrus fruit, and strict pricing regulations are needed to
prevent them from continuing to inflate prices.”
Discuss how well reasoned … etc.
____________________________
The following appeared as part of an article in a local newspaper:
“Over the past three years the tartfish industry has changed markedly: fishing technology has improved
significantly, and the demand for tartfish has grown in both domestic and foreign markets. As this trend continues,
the tartfish industry on Shrimp Island can expect to experience the same overfishing problems that are already
occurring with mainland fishing industries: without restrictions on fishing, fishers see no reason to limit their
individual catches. As the catches get bigger, the tartfish population will be dangerously depleted while the surplus
of tartfish will devalue the catch for fishers. Government regulation is the only answer: tartfish‐fishing should be
allowed only during the three‐month summer season, when tartfish reproduce and thus are most numerous, rather
than throughout the year.”
Discuss how well reasoned … etc.
829

GMAT® Official Guide 2018

The following appeared in a proposal from the development office at Platonic University:
“Because Platonic University has had difficulty in meeting its expenses over the past three years, we need to find new
ways to increase revenues. We should consider following the example of Greene University, which recently renamed
itself after a donor who gave it $100 million. If Platonic University were to advertise to its alumni and other wealthy
people that it will rename either individual buildings or the entire university itself after the donors who give the most
money, the amount of donations would undoubtedly increase.”
Discuss how well reasoned … etc.
____________________________
The following appeared as part of an article in the business section of a local newspaper:
“Hippocrene Plumbing Supply recently opened a wholesale outlet in the location once occupied by the Cumquat
Café. Hippocrene has apparently been quite successful there because it is planning to open a large outlet in a
nearby city. But the Cumquat Café, one year after moving to its new location, has seen its volume of business drop
somewhat from the previous year’s. Clearly, the former site was the better business location, and the Cumquat
Café has made a mistake in moving to its new address.”
Discuss how well reasoned … etc.
____________________________
The following appeared in the editorial section of a local paper:
“Applications for advertising spots on KMTV, our local cable television channel, decreased last year. Meanwhile
a neighboring town’s local channel, KOOP, changed its focus to farming issues and reported an increase in
advertising applications for the year. To increase applications for its advertisement spots, KMTV should focus its
programming on farming issues as well.”
Discuss how well reasoned … etc.
____________________________
The following appeared as part of an article in a computer magazine:
“A year ago Apex Manufacturing bought its managers computers for their homes and paid for telephone
connections so that they could access Apex computers and data files from home after normal business hours.
Since last year, productivity at Apex has increased by 15 percent. Other companies can learn from the success
at Apex: given home computers and access to company resources, employees will work additional hours at home
and thereby increase company profits.”
Discuss how well reasoned … etc.
____________________________
The following was excerpted from an article in a farming trade publication:
“Farmers who switched from synthetic to organic farming last year have seen their crop yields decline. Many of
these farmers feel that it would be too expensive to resume synthetic farming at this point, given the money that
they invested in organic farming supplies and equipment. But their investments will be relatively minor compared
to the losses from continued lower crop yields. Organic farmers should switch to synthetic farming rather than
persist in an unwise course. And the choice to farm organically is financially unwise, given that it was motivated by
environmental rather than economic concerns.”
Discuss how well reasoned … etc.
830

11.6 Analytical Writing Assessment Analysis of an Argument Sample Topics

The following appeared in a letter to prospective students from the admissions office at Plateau College:
“Every person who earned an advanced degree in science or engineering from Olympus University last year
received numerous offers of excellent jobs. Typically, many graduates of Plateau College have gone on to pursue
advanced degrees at Olympus. Therefore, enrolling as an undergraduate at Plateau College is a wise choice for
students who wish to ensure success in their careers.”
Discuss how well reasoned … etc.
____________________________
The following appeared in a memorandum sent by a vice‐president of the Nadir Company to the company’s human
resources department:
“Nadir does not need to adopt the costly ‘family‐friendly’ programs that have been proposed, such as part‐time
work, work at home, and jobsharing. When these programs were made available at the Summit Company, the
leader in its industry, only a small percentage of employees participated in them. Rather than adversely affecting
our profitability by offering these programs, we should concentrate on offering extensive training that will enable
employees to increase their productivity.”
Discuss how well reasoned … etc.
____________________________
The following appeared as part of an article in a trade magazine for breweries:
“Magic Hat Brewery recently released the results of a survey of visitors to its tasting room last year. Magic Hat
reports that the majority of visitors asked to taste its low‐calorie beers. To boost sales, other small breweries
should brew low‐calorie beers as well.”
Discuss how well reasoned … etc.
____________________________
The following appeared in an editorial from a newspaper serving the town of Saluda:
“The Saluda Consolidated High School offers more than 200 different courses from which its students can
choose. A much smaller private school down the street offers a basic curriculum of only 80 different courses,
but it consistently sends a higher proportion of its graduating seniors on to college than Consolidated does. By
eliminating at least half of the courses offered there and focusing on a basic curriculum, we could improve student
performance at Consolidated and also save many tax dollars.”
Discuss how well reasoned … etc.

831

GMAT® Official Guide 2018

The following appeared as part of an article in the book section of a newspaper:
“Currently more and more books are becoming available in electronic form—either free‐of‐charge on the Internet
or for a very low price‐per‐book on compact disc. Thus literary classics are likely to be read more widely than
ever before. People who couldn’t have purchased these works at bookstore prices will now be able to read them
for little or no money; similarly, people who find it inconvenient to visit libraries and wait for books to be returned
by other patrons will now have access to whatever classic they choose from their home or work computers.
This increase in access to literary classics will radically affect the public taste in reading, creating a far more
sophisticated and learned reading audience than has ever existed before.”
Discuss how well reasoned … etc.
____________________________
The following appeared as an editorial in a magazine concerned with educational issues:
“In our country, the real earnings of men who have only a high‐school degree have decreased significantly over
the past 15 years, but those of male college graduates have remained about the same. Therefore, the key to
improving the earnings of the next generation of workers is to send all students to college. Our country’s most
important educational goal, then, should be to establish enough colleges and universities to accommodate all high
school graduates.”
Discuss how well reasoned … etc.
____________________________
The following appeared as part of a business plan created by the management of the Take Heart Fitness Center:
“After opening the new swimming pool early last summer, Take Heart saw a 12 percent increase in the use of
the center by its members. Therefore, in order to increase membership in Take Heart, we should continue to add
new recreational facilities in subsequent years: for example, a multipurpose game room, a tennis court, and a
miniature golf course. Being the only center in the area offering this range of activities would give us a competitive
advantage in the health and recreation market.”
Discuss how well reasoned … etc.
____________________________
The following appeared in a letter from a staff member in the office of admissions at Argent University:
“The most recent nationwide surveys show that undergraduates choose their major field primarily based on their
perception of job prospects in that field. At our university, economics is now the most popular major, so students
must perceive this field as having the best job prospects. Therefore, we can increase our enrollment if we focus
our advertising and recruiting on publicizing the accomplishments of our best‐known economics professors and
the success of our economics graduates in finding employment.”
Discuss how well reasoned … etc.

832

11.6 Analytical Writing Assessment Analysis of an Argument Sample Topics

The following appeared as part of a memorandum from the loan department of the Frostbite National Bank:
“We should not approve the business loan application of the local group that wants to open a franchise outlet for
the Kool Kone chain of ice cream parlors. Frostbite is known for its cold winters, and cold weather can mean slow
ice cream sales. For example, even though Frostbite is a town of 10,000 people, it has only one ice cream spot—
the Frigid Cow. Despite the lack of competition, the Frigid Cow’s net revenues fell by 10 percent last winter.”
Discuss how well reasoned … etc.
____________________________
The following appeared as part of a letter to the editor of a local newspaper:
“Bayview High School is considering whether to require all of its students to wear uniforms while at school.
Students attending Acorn Valley Academy, a private school in town, earn higher grades on average and are more
likely to go on to college. Moreover, Acorn Valley reports few instances of tardiness, absenteeism, or discipline
problems. Since Acorn Valley requires its students to wear uniforms, Bayview High School would do well to follow
suit and require its students to wear uniforms as well.”
Discuss how well reasoned … etc.
____________________________
The following appeared in a memo to the Saluda town council from the town’s business manager:
“Research indicates that those who exercise regularly are hospitalized less than half as often as those who don’t
exercise. By providing a well‐equipped gym for Saluda’s municipal employees, we should be able to reduce the
cost of our group health insurance coverage by approximately 50 percent and thereby achieve a balanced town
budget.”
Discuss how well reasoned … etc.
____________________________
The following appeared in a memorandum written by the assistant manager of a store that sells gourmet food
items from various countries:
“A local wine store made an interesting discovery last month: it sold more French than Italian wine on days when it
played recordings of French accordion music, but it sold more Italian than French wine on days when Italian songs
were played. Therefore, I recommend that we put food specialties from one particular country on sale for a week
at a time and play only music from that country while the sale is going on. By this means we will increase our
profits in the same way that the wine store did, and we will be able to predict more precisely what items we should
stock at any given time.”
Discuss how well reasoned … etc.

833

GMAT® Official Guide 2018

The following appeared in a memorandum from the director of research and development at Ready‐to‐Ware, a
software engineering firm:
“The package of benefits and incentives that Ready‐to‐Ware offers to professional staff is too costly. Our quarterly
profits have declined since the package was introduced two years ago, at the time of our incorporation. Moreover,
the package had little positive effect, as we have had only marginal success in recruiting and training high‐quality
professional staff. To become more profitable again, Ready‐to‐Ware should, therefore, offer the reduced benefits
package that was in place two years ago and use the savings to fund our current research and development
initiatives.”
Discuss how well reasoned … etc.
____________________________
The following appeared as a memorandum from the vice‐president of the Dolci candy company:
“Given the success of our premium and most expensive line of chocolate candies in a recent taste test and the
consequent increase in sales, we should shift our business focus to producing additional lines of premium candy
rather than our lower‐priced, ordinary candies. When the current economic boom ends and consumers can no
longer buy major luxury items, such as cars, they will still want to indulge in small luxuries, such as expensive
candies.”
Discuss how well reasoned … etc.
____________________________
The following appeared in a memorandum from the business office of the Lovin’ Cupful, a national restaurant chain:
“The Lovin’ Cupful franchises in our northeast region have begun serving customers Almost, a brand new
powdered instant tea, in place of brewed tea. Waiters report that only about 2 percent of the customers have
complained, and that customers who want refills typically ask for ‘more tea.’ It appears, then, that 98 percent of
the customers are perfectly happy with the switch, or else they cannot tell powdered instant from brewed tea.
Therefore, in order to take advantage of the lower price per pound of Almost, all of our restaurants should begin
substituting it for brewed tea.”
Discuss how well reasoned … etc.
____________________________
The following appeared in a memorandum from the director of marketing for a pharmaceutical company:
“According to a survey of 5,000 urban residents, the prevalence of stress headaches increases with educational
level, so that stress headaches occur most often among people with graduate‐school degrees. It is well
established that, nationally, higher educational levels usually correspond with higher levels of income. Therefore,
in marketing our new pain remedy, Omnilixir, we should send free samples primarily to graduate students and to
people with graduate degrees, and we should concentrate on advertising in professional journals rather than in
general interest magazines.”
Discuss how well reasoned … etc.

834

11.6 Analytical Writing Assessment Analysis of an Argument Sample Topics

The following appeared as part of an editorial in the Waymarsh city newspaper:
“Last year the parents of first graders in our school district expressed satisfaction with the reading skills their
children developed but complained strongly about their children’s math skills. To remedy this serious problem and
improve our district’s elementary education, everyone in the teacher‐training program at Waymarsh University
should be required to take more courses in mathematics.”
Discuss how well reasoned … etc.
____________________________
The following appeared in a letter to the editor of a River City newspaper:
“The Clio Development Group should not be permitted to build a multilevel parking garage on Dock Street since
most of the buildings on the block would have to be demolished. Because these buildings were erected decades
ago, they have historic significance and must therefore be preserved as economic assets in the effort to revitalize
a restored riverfront area. Recall how Lakesburg has benefited from business increases in its historic downtown
center. Moreover, there is plenty of vacant land for a parking lot elsewhere in River City.”
Discuss how well reasoned … etc.
____________________________
The following appeared in a corporate planning memorandum for a company that develops amusement parks:
“Because travel from our country to foreign countries has increased dramatically in recent years, our next project
should be a ‘World Tour’ theme park with replicas of famous foreign buildings, rides that have international themes,
and refreshment stands serving only foods from the country represented by the nearest ride. The best location
would be near our capital city, which has large percentages of international residents and of children under the age
of 16. Given the advantages of this site and the growing interest in foreign countries, the ‘World Tour’ theme park
should be as successful as our space‐travel theme park, where attendance has increased tenfold over the past
decade.”
Discuss how well reasoned … etc.
____________________________
The following appeared in a memorandum from the publisher to the staff of The Clarion, a large metropolitan
newspaper:
“During the recent campaign for mayor, a clear majority of city readers who responded to our survey indicated
a desire for more news about city government. To increase circulation, and thus our profits, we should therefore
consistently devote a greater proportion of space in all editions of The Clarion to coverage of local news.”
Discuss how well reasoned … etc.

835

GMAT® Official Guide 2018

The following appeared in a memorandum from the assistant manager of Pageturner Books:
“Over the past two years, Pageturner’s profits have decreased by 5 percent, even though we have added a popular
café as well as a music section selling CDs and tapes. At the same time, we have experienced an increase in the
theft of merchandise. We should therefore follow the example of Thoreau Books, which increased its profits after
putting copies of its most frequently stolen books on a high shelf behind the payment counter. By doing likewise
with copies of the titles that our staff reported stolen last year, we too can increase profitability.”
Discuss how well reasoned … etc.
____________________________
The following appeared in a letter to the editor of a River City newspaper:
“The Clio Development Group’s plan for a multilevel parking garage on Dock Street should be approved in order
to strengthen the economy of the surrounding area. Although most of the buildings on the block would have to be
demolished, they are among the oldest in the city and thus of little current economic value. Those who oppose the
project should realize that historic preservation cannot be the only consideration: even Athens or Jerusalem will
knock down old buildings to put up new ones that improve the local economy.”
Discuss how well reasoned … etc.
____________________________
The following appeared in a memorandum from the owner of Carlo’s Clothing to the staff:
“Since Disc Depot, the music store on the next block, began a new radio advertising campaign last year, its
business has grown dramatically, as evidenced by the large increase in foot traffic into the store. While the Disc
Depot’s owners have apparently become wealthy enough to retire, profits at Carlo’s Clothing have remained
stagnant for the past three years. In order to boost our sales and profits, we should therefore switch from
newspaper advertising to frequent radio advertisements like those for Disc Depot.”
Discuss how well reasoned … etc.
____________________________
The following appeared as part of the business plan of the Capital Idea investment firm:
“Across town in the Park Hill district, the Thespian Theater, Pizzazz Pizza, and the Niblick Golf Club have all had
business increases over the past two years. Capital Idea should therefore invest in the Roxy Playhouse, the
Slice‐o’‐Pizza, and the Divot Golf Club, three new businesses in the Irongate district. As a condition, we should
require them to participate in a special program: Any customer who patronizes two of the businesses will receive
a substantial discount at the third. By motivating customers to patronize all three, we will thus contribute to the
profitability of each and maximize our return.”
Discuss how well reasoned … etc.

836

11.6 Analytical Writing Assessment Analysis of an Argument Sample Topics

The following appeared as part of an article in a newsletter for farmers:
“Users of Solacium, a medicinal herb now grown mainly in Asia, report that it relieves tension and promotes
deep sleep. A recent study indicates that a large number of college students who took pills containing one of
the ingredients in Solacium suffered less anxiety. To satisfy the anticipated demands for this very promising
therapeutic herb and to reap the financial benefits, farmers in this country should begin growing it.”
Discuss how well reasoned … etc.
____________________________
The following appeared in a memorandum from the president of Aurora, a company that sells organic milk (milk
produced without the use of chemical additives):
“Sales of organic food products in this country have tripled over the past five years. If Aurora is to profit from this
continuing trend, we must diversify and start selling products such as organic orange juice and organic eggs in
addition to our regular product line. With the recent increase of articles in health magazines questioning the safety
of milk and other food products, customers are even more likely to buy our line of organic products. And to help
ensure our successful expansion, we should hire the founder of a chain of health‐food stores to serve as our vice
president of marketing.”
Discuss how well reasoned … etc.
____________________________
The following appeared in a memorandum from the human resources department of Diversified Manufacturing:
“Managers at our central office report that their employees tend to be most productive in the days immediately
preceding a vacation. To help counteract our declining market share, we could increase the productivity of our
professional staff members, who currently receive four weeks paid vacation a year, by limiting them to a maximum
of one week’s continuous vacation time. They will thus take more vacation breaks during a year and give us more
days of maximum productivity.”
Discuss how well reasoned … etc.
____________________________
The following appeared in a memorandum from a regional supervisor of post office operations:
“During a two‐week study of postal operations, the Presto City post office handled about twice as many items
as the Lento City post office, even though the cities are about the same size. Moreover, customer satisfaction
appears to be higher in Presto City, since the study found fewer complaints regarding the Presto City post office.
Therefore, the postmasters at these two offices should exchange assignments: the Presto City postmaster
will solve the problems of inefficiency and customer dissatisfaction at the Lento City office while the Lento City
postmaster learns firsthand the superior methods of Presto City.”
Discuss how well reasoned … etc.

837

GMAT® Official Guide 2018

The following appeared in a memorandum written by the managing director of the Exeunt Theater Company:
“Now that we have moved to a larger theater, we can expect to increase our revenues from ticket sales. To further
increase profits, we should start producing the plays that have been most successful when they were performed
in our nation’s largest cities. In addition, we should hire the Adlib Theater Company’s director of fund‐raising, since
corporate contributions to Adlib have increased significantly over the three years that she has worked for Adlib.”
Discuss how well reasoned … etc.
____________________________
The following appeared in a memorandum from the human resources department of HomeStyle, a house
remodeling business:
“This year, despite HomeStyle’s move to new office space, we have seen a decline in both company morale and
productivity, and a corresponding increase in administrative costs. To rectify these problems, we should begin
using a newly developed software package for performance appraisal and feedback. Managers will save time by
simply choosing comments from a preexisting list; then the software will automatically generate feedback for the
employee. The human resources department at CounterBalance, the manufacturer of the countertops we install,
reports satisfaction with the package.”
Discuss how well reasoned … etc.
____________________________
The following appeared as part of an article in a weekly newsmagazine:
“The country of Oleum can best solve the problem of its balance‐of‐trade deficit by further increasing the tax on
its major import, crude oil. After Oleum increased the tax on imported crude oil four months ago, consumption of
gasoline declined by 20 percent. Therefore, by imposing a second and significantly higher tax increase next year,
Oleum will dramatically decrease its balance-of-trade deficit.”
Discuss how well reasoned … etc.
____________________________
The following appeared as part of a business plan by the Capital Idea investment firm:
“In recent years the worldwide demand for fish has grown, and improvements in fishing technology have made
larger catches, and thus increased supply, possible: for example, last year’s tuna catch was 9 percent greater than
the previous year’s. To capitalize on these trends, we should therefore invest in the new tartfish processing plant
on Tartfish Island, where increasing revenues from tourism indicate a strong local economy.”
Discuss how well reasoned … etc.

838

11.6 Analytical Writing Assessment Analysis of an Argument Sample Topics

The following appeared in a speech by a stockholder of Consolidated Industries at the company’s annual
stockholders’ meeting:
“In the computer hardware division last year, profits fell significantly below projections, the product line decreased
from 20 to only 5 items, and expenditures for employee benefits increased by 15 percent. Nevertheless,
Consolidated’s board of directors has approved an annual salary of more than $1 million for our company’s chief
executive officer. The present board members should be replaced because they are unconcerned about the
increasing costs of employee benefits and salaries, in spite of the company’s problems generating income.”
Discuss how well reasoned … etc.
____________________________
The following appeared in a memorandum from the business planning department of Avia Airlines:
“Of all the cities in their region, Beaumont and Fletcher are showing the fastest growth in the number of new
businesses. Therefore, Avia should establish a commuter route between them as a means of countering recent
losses on its main passenger routes. And to make the commuter route more profitable from the outset, Avia
should offer a 1/3 discount on tickets purchased within two days of the flight. Unlike tickets bought earlier,
discount tickets will be nonrefundable, and so gain from their sale will be greater.”
Discuss how well reasoned … etc.
___________________________
The following appeared in a memorandum from the vice president of Gigantis, a development company that builds
and leases retail store facilities:
“Nationwide over the past five years, sales have increased significantly at outlet stores that deal exclusively in
reduced‐price merchandise. Therefore, we should publicize the new mall that we are building at Pleasantville as
a central location for outlet shopping and rent store space only to outlet companies. By taking advantage of the
success of outlet stores, this plan should help ensure full occupancy of the mall and enable us to recover quickly
the costs of building the mall.”
Discuss how well reasoned … etc.
____________________________
The following appeared in a memorandum written by the chair of the music department to the president of Omega
University:
“Mental health experts have observed that symptoms of mental illness are less pronounced in many patients
after group music‐therapy sessions, and job openings in the music‐therapy field have increased during the past
year. Consequently, graduates from our degree program for music therapists should have no trouble finding good
positions. To help improve the financial status of Omega University, we should therefore expand our music‐therapy
degree program by increasing its enrollment targets.”
Discuss how well reasoned … etc.

839

GMAT® Official Guide 2018

The following appeared in a memorandum to the work‐group supervisors of the GBS Company:
“The CoffeeCart beverage and food service located in the lobby of our main office building is not earning enough
in sales to cover its costs, and so the cart may discontinue operating at GBS. Given the low staff morale, as
evidenced by the increase in the number of employees leaving the company, the loss of this service could present
a problem, especially since the staff morale questionnaire showed widespread dissatisfaction with the snack
machines. Therefore, supervisors should remind the employees in their group to patronize the cart—after all, it
was leased for their convenience so that they would not have to walk over to the cafeteria on breaks.”
Discuss how well reasoned … etc.
____________________________
The following appeared as part of an article in a trade magazine:
“During a recent trial period in which government inspections at selected meat‐processing plants were more
frequent, the amount of bacteria in samples of processed chicken decreased by 50 percent on average from the
previous year’s level. If the government were to institute more frequent inspections, the incidence of stomach and
intestinal infections throughout the country could thus be cut in half. In the meantime, consumers of Excel Meats
should be safe from infection because Excel’s main processing plant has shown more improvement in eliminating
bacterial contamination than any other plant cited in the government report.”
Discuss how well reasoned … etc.

840

Appendix A Answer Sheets
Diagnostic Answer
Sheet—Quantitative

Diagnostic Answer
Sheet—Verbal

1.

27.

49.

75.

2.

28.

50.

76.

3.

29.

51.

77.

4.

30.

52.

78.

5.

31.

53.

79.

6.

32.

54.

80.

7.

33.

55.

81.

8.

34.

56.

82.

9.

35.

57.

83.

10.

36.

58.

84.

11.

37.

59.

85.

12.

38.

60.

86.

13.

39.

61.

87.

14.

40.

62.

88.

15.

41.

63.

89.

16.

42.

64.

90.

17.

43.

65.

91.

18.

44.

66.

92.

19.

45.

67.

93.

20.

46.

68.

94.

21.

47.

69.

95.

22.

48.

70.

96.

23.

71.

97.

24.

72.

98.

25.

73.

99.

26.

74.

100.
841

GMAT® Official Guide 2018

Problem Solving Answer Sheet

842

1.

32.

63.

94.

125.

156.

187.

218.

2.

33.

64.

95.

126.

157.

188.

219.

3.

34.

65.

96.

127.

158.

189.

220.

4.

35.

66.

97.

128.

159.

190.

221.

5.

36.

67.

98.

129.

160.

191.

222.

6.

37.

68.

99.

130.

161.

192.

223.

7.

38.

69.

100.

131.

162.

193.

224.

8.

39.

70.

101.

132.

163.

194.

225.

9.

40.

71.

102.

133.

164.

195.

226.

10.

41.

72.

103.

134.

165.

196.

227.

11.

42.

73.

104.

135.

166.

197.

228.

12.

43.

74.

105.

136.

167.

198.

229.

13.

44.

75.

106.

137.

168.

199.

230.

14.

45.

76.

107.

138.

169.

200.

15.

46.

77.

108.

139.

170.

201.

16.

47.

78.

109.

140.

171.

202.

17.

48.

79.

110.

141.

172.

203.

18.

49.

80.

111.

142.

173.

204.

19.

50.

81.

112.

143.

174.

205.

20.

51.

82.

113.

144.

175.

206.

21.

52.

83.

114.

145.

176.

207.

22.

53.

84.

115.

146.

177.

208.

23.

54.

85.

116.

147.

178.

209.

24.

55.

86.

117.

148.

179.

210.

25.

56.

87.

118.

149.

180.

211.

26.

57.

88.

119.

150.

181.

212.

27.

58.

89.

120.

151.

182.

213.

28.

59.

90.

121.

152.

183.

214.

29.

60.

91.

122.

153.

184.

215.

30.

61.

92.

123.

154.

185.

216.

31.

62.

93.

124.

155.

186.

217.

Appendix A Answer Sheets

Data Sufficiency Answer Sheet
231.

266.

301.

336.

371.

232.

267.

302.

337.

372.

233.

268.

303.

338.

373.

234.

269.

304.

339.

374.

235.

270.

305.

340.

375.

236.

271.

306.

341.

376.

237.

272.

307.

342.

377.

238.

273.

308.

343.

378.

239.

274.

309.

344.

379.

240.

275.

310.

345.

380.

241.

276.

311.

346.

381.

242.

277.

312.

347.

382.

243.

278.

313.

348.

383.

244.

279.

314.

349.

384.

245.

280.

315.

350.

385.

246.

281.

316.

351.

386.

247.

282.

317.

352.

387.

248.

283.

318.

353.

388.

249.

284.

319.

354.

389.

250.

285.

320.

355.

390.

251.

286.

321.

356.

391.

252.

287.

322.

357.

392.

253.

288.

323.

358.

393.

254.

289.

324.

359.

394.

255.

290.

325.

360.

395.

256.

291.

326.

361.

396.

257.

292.

327.

362.

397.

258.

293.

328.

363.

398.

259.

294.

329.

364.

399.

260.

295.

330.

365.

400.

261.

296.

331.

366.

401.

262.

297.

332.

367.

402.

263.

298.

333.

368.

403.

264.

299.

334.

369.

404.

265.

300.

335.

370.
843

GMAT® Official Guide 2018

Reading Comprehension Answer Sheet

844

405.

433.

461.

489.

517.

406.

434.

462.

490.

518.

407.

435.

463.

491.

519.

408.

436.

464.

492.

520.

409.

437.

465.

493.

521.

410.

438.

466.

494.

522.

411.

439.

467.

495.

523.

412.

440.

468.

496.

524.

413.

441.

469.

497.

525.

414.

442.

470.

498.

526.

415.

443.

471.

499.

527.

416.

444.

472.

500.

528.

417.

445.

473.

501.

529.

418.

446.

474.

502.

530.

419.

447.

475.

503.

531.

420.

448.

476.

504.

532.

421.

449.

477.

505.

533.

422.

450.

478.

506.

534.

423.

451.

479.

507.

535.

424.

452.

480.

508.

536.

425.

453.

481.

509.

537.

426.

454.

482.

510.

538.

427.

455.

483.

511.

539.

428.

456.

484.

512.

540.

429.

457.

485.

513.

541.

430.

458.

486.

514.

542.

431.

459.

487.

515.

543.

432.

460.

488.

516.

Appendix A Answer Sheets

Critical Reasoning Answer Sheet
544.

569.

594.

619.

644.

545.

570.

595.

620.

645.

546.

571.

596.

621.

646.

547.

572.

597.

622.

647.

548.

573.

598.

623.

648.

549.

574.

599.

624.

649.

550.

575.

600.

625.

650.

551.

576.

601.

626.

651.

552.

577.

602.

627.

652.

553.

578.

603.

628.

653.

554.

579.

604.

629.

654.

555.

580.

605.

630.

655.

556.

581.

606.

631.

656.

557.

582.

607.

632.

657.

558.

583.

608.

633.

658.

559.

584.

609.

634.

659.

560.

585.

610.

635.

660.

561.

586.

611.

636.

661.

562.

587.

612.

637.

662.

563.

588.

613.

638.

663.

564.

589.

614.

639.

664.

565.

590.

615.

640.

665.

566.

591.

616.

641.

666.

567.

592.

617.

642.

667.

568.

593.

618.

643.

845

GMAT® Official Guide 2018

Sentence Correction Answer Sheet

846

668.

696.

724.

752.

780.

669.

697.

725.

753.

781.

670.

698.

726.

754.

782.

671.

699.

727.

755.

783.

672.

700.

728.

756.

784.

673.

701.

729.

757.

785.

674.

702.

730.

758.

786.

675.

703.

731.

759.

787.

676.

704.

732.

760.

788.

677.

705.

733.

761.

789.

678.

706.

734.

762.

790.

679.

707.

735.

763.

791.

680.

708.

736.

764.

792.

681.

709.

737.

765.

793.

682.

710.

738.

766.

794.

683.

711.

739.

767.

795.

684.

712.

740.

768.

796.

685.

713.

741.

769.

797.

686.

714.

742.

770.

798.

687.

715.

743.

771.

799.

688.

716.

744.

772.

800.

689.

717.

745.

773.

801.

690.

718.

746.

774.

802.

691.

719.

747.

775.

803.

692.

720.

748.

776.

804.

693.

721.

749.

777.

805.

694.

722.

750.

778.

806.

695.

723.

751.

779.

807.

WILEY END USER LICENSE
AGREEMENT
Go to www.wiley.com/go/eula to access Wiley’s ebook
EULA.



Source Exif Data:
File Type                       : PDF
File Type Extension             : pdf
MIME Type                       : application/pdf
PDF Version                     : 1.6
Linearized                      : No
Create Date                     : 2017:05:06 12:40:56+05:30
Modify Date                     : 2017:07:08 21:31:20+05:30
Has XFA                         : No
XMP Toolkit                     : Adobe XMP Core 5.2-c001 63.143651, 2012/04/05-09:01:49
Producer                        : Adobe Acrobat Pro 9.0.0
Metadata Date                   : 2017:07:08 21:31:20+05:30
Creator Tool                    : Adobe InDesign CS6 (Macintosh)
Document ID                     : uuid:32eb2e11-82f9-4a27-9fe8-941afa3be9e9
Instance ID                     : uuid:dd0cffc4-c831-46ff-be22-f4bfa3253b29
Format                          : application/pdf
Title                           : GMAT® OFFICIAL GUIDE 2018
Creator                         : 
Nxpl Tag                        : D7.1.2-778000-9155540
Schemas Schema                  : NXPowerLite tag schema
Schemas Namespace URI           : http://ns.neuxpower.com/pdf/tagging/
Schemas Prefix                  : nxplPDF
Schemas Property Name           : nxplTag
Schemas Property Value Type     : Text
Schemas Property Category       : internal
Schemas Property Description    : NXPowerLite tag
Page Layout                     : SinglePage
Page Mode                       : UseOutlines
Page Count                      : 855
EXIF Metadata provided by EXIF.tools

Navigation menu